diff --git "a/stack-exchange/math_overflow/shard_3.txt" "b/stack-exchange/math_overflow/shard_3.txt" deleted file mode 100644--- "a/stack-exchange/math_overflow/shard_3.txt" +++ /dev/null @@ -1,35970 +0,0 @@ -TITLE: Guessing a subset of {1,...,N} -QUESTION [11 upvotes]: I pick a random subset $S\subseteq\lbrace1,\ldots,N\rbrace$, and you have to guess what it is. After each guess $G$, I tell you the number of elements in $G \cap S$. How many guesses do you need to determine the subset? (If there is only one possibility left, then you can omit the last guess.) -There is an obvious strategy that requires only $N$ guesses. Guess $\lbrace1\rbrace$, then guess $\lbrace2\rbrace$, then guess $\lbrace3\rbrace$, and so on. But there is a clever strategy that requires only $\lceil 4N/5 \rceil$ guesses. -We know that the minimum number of guesses is at least $\left\lceil \frac{N}{\log_2{(N+1)}}\right\rceil$, because each guess reveals at most $\log_2(N+1)$ bits of information. I seek a proof or disproof of the conjecture that the number of guesses $g(N)$ is sublinear, i.e. $\lim_{N\to\infty} g(N)/N = 0$. -I will donate $100 to the American Red Cross if a proof or disproof is posted to this thread by April 30, 2011. For this purpose, I will accept an argument as correct if I believe it to be correct; or if a user with reputation above 1000 asserts that it is correct, and no user with reputation above 1000 denies that it is correct. Naturally, I would welcome improved upper bounds, even if they are linear. - -REPLY [18 votes]: This is a well-studied problem, sometimes phrased as a coin-weighing problem. It is known that $g(N)$ is $O(N / \log N)$. (We can even specify the guessing sets in advance, without knowing the previous answers.) I believe these three papers are the earliest to show this bound: -B. Lindstrom (1964), "On a combinatory detection problem I", Mathematical Institute of the Hungarian Academy of Science 9, pp. 195-207. -B. Lindstrom (1965), "On a combinatorial problem in number theory", Canadian Math. Bulletin 8, pp. 477-490. -D. Cantor and W. Mills (1966), "Determining a subset from certain combinatorial properties", Canadian J. Math 18, pp. 42-48. -There was a lot of work after these papers too (some with simpler constructions, some to solve more general problems). A book by Aigner covers this topic and more: -M. Aigner (1988), "Combinatorial search", John Wiley and Sons.<|endoftext|> -TITLE: Contour integration of $\zeta(s)\zeta(2s)$ -QUESTION [7 upvotes]: I have been looking at this for days and I am going insane. -I need to show that for a Dirichlet series equal to $\zeta(s)\zeta(2s)$, the sum of the coefficients less than $x$ is $x\zeta(2)+O(x^{(3/4)})$, and then expand that to the $\Pi \zeta(ks)$ for all $k$ in an effort to find the formula for the number of non-isomorphic abelian groups. -I know that using Perron's formula there is a simple pole at $s=1$ that gives a residue of $X\zeta(2)$, but I can't find a contour that converges or the exact error term. - -REPLY [6 votes]: If you havn't done so already, you might find it useful to look at the proof of theorem 12.2 on the divisor problem in Titchmarsh - The theory of the Riemann zeta function. Here, he goes through a detailed application of Perron's formula for the function $\zeta^k(s)$, which I believe to be very similar to your case. -Indeed, for $s=\sigma + it$ and $\sigma>1/2$, $\zeta(2s)$ is absolutley convergent and hence uniformly bounded with respect to $t$. So this will not contribute to the contours that you choose (as long as $\sigma>1/2$!). What you need then is good upper bounds for the order of the zeta function in the critical strip. -To get these, one normally finds the order of the function at two points, and then uses the Phragmén–Lindelöf principle for strips to get estimates for the function between these two points. For example, it is known that $\zeta(1/2 + it) = O(t^{1/4})$ (see The Lindelöf hypothesis), although there are much better bounds available than that. This is all done in Titchmarsh's book. -I hope this helps!<|endoftext|> -TITLE: Fano 3-fold of degree 4 -QUESTION [6 upvotes]: Let $X$ be the intersection of two quadrics in $P^5$. It is well known that the intermediate Jacobian $J(X)$ is isomorphic to $J(C)$ for a genus 2 curve, related to the pencil of quadrics whose base locus is $X$. -It seemed then natural to me to ask the following question: -Is there an explicit construction where $X$ is obtained as a smooth blow-up of $P^3$, or of a smooth quadric, or of a $P^2$ bundle over $P^1$, along a curve isomorphic to $C$? - -REPLY [5 votes]: The projection from a line $L_0$ is a birational isomorphism of $X$ onto $P^3$. It decomposes as the blow-up of the line $L_0$ followed by the contraction of a surface swept by lines intersecting $L_0$ onto a curve of genus 2 in $P^3$.<|endoftext|> -TITLE: Does every irreducible representation of a compact group occur in tensor products of a faithful representation and its dual? -QUESTION [17 upvotes]: (Previously posted on math.SE with no answers.) -Let $G$ be a compact Lie group and $V$ a faithful (complex, continuous, finite-dimensional) representation of it. Is it true that every (complex, continuous, finite-dimensional) irreducible representation of $G$ occurs in $V^{\otimes n} (V^{\ast})^{\otimes m}$ for some $n, m$? -The proof I know for finite groups doesn't seem to easily generalize. I want to apply Stone-Weierstrass, but can't figure out if the characters I get will always separate points (in the space of conjugacy classes). Ben Webster in his answer to this MO question seems to be suggesting that this follows from the first part of Peter-Weyl, but I don't see how this works. -Certainly the corresponding algebra of characters separates points whenever the eigenvalues of an element $g \in G$ acting on $V$ determine its conjugacy class (since one can get the eigenvalues from an examination of the exterior powers). Does this always happen? - -REPLY [21 votes]: You can mimic the standard finite group argument. Recall that, if $X$ and $Y$ are two finite-dimensional representations of $G$, with characters $\chi_X$ and $\chi_Y$, then $\dim \mathrm{Hom}_G(X,Y) = \int_G \overline{\chi_X} \otimes \chi_Y$, where the integral is with respect to Haar measure normalized so that $\int_G 1 =1$. The proof of this is exactly as in the finite case. -Now, let $W=1 \oplus V \oplus \overline{V}$. Your goal is to show that, for any nonzero representation $Y$, $\mathrm{Hom}(W^{\otimes N}, Y)$ is nonzero for $N$ sufficiently large. So you need to analyze $\int (1+\chi_V + \overline{\chi_V})^N \chi_W$ for $N$ large. Just as in the finite group case, we are going to split this into an integral near the identity, and an exponentially decaying term everywhere else. -I'm going to leave a lot of the analytic details to you, but here is the idea. Let $d=\dim V$. The function $f:=1+\chi_V + \overline{\chi_V}$ makes sense on the entire unitary group of $V$, of which $G$ is a subgroup. On a unitary matrix with eigenvalues $(e^{i \theta_1}, \cdots, e^{i \theta_d})$, we have $f=1+2 \sum \cos \theta_i$. In particular, for an element $g$ in the Lie algebra of $G$ near the identity, we have -$$f(e^g) = (2n+1) e^{-K(g) + O(|g|^4)}$$ - where $K$ is $\mathrm{Tr}(g^* g) = \sum \theta_i^2$. -And, for $g$ near the identity, $\chi_X(e^g) = d + O(|g|)$. So the contribution to our integral near the identity can be approximated by -$$\int_{\mathfrak{g}} \left( (2n+1) e^{-K(g) + O(|g|^4)} \right)^N (d+ O(|g|) \approx (2n+1)^N d \int_{\mathfrak{g}} e^{-K(\sqrt{N} g)} = \frac{(2n+1)^N d}{N^{\dim G/2}} C$$ -where $C$ is a certain Gaussian integral, and includes some sort of a factor concerning the determinant of the quadratic form $K$. You also need to work out what the Haar measure of $G$ turns into as a volume form on $\mathfrak{g}$ near $0$, I'll leave that to you as well. -For right now, the important point is that the growth rate is like $(2n+1)^N d$ divided by a polynomial factor. In the finite group case, that polynomial is a constant, which makes life easier, but we can live with a polynomial. -Now, look at the contribution from the rest of $G$. For any point in the unitary group $U(d)$, other than the identity, we have $|f| < 2n+1$. (To get equality, all the eigenvalues must be $1$ and, in the unitary group, that forces us to be at the identity.) So, if we break our integral into the integral over a small ball around the identity, plus an integral around everything else, the contribution of every thing else will be $O(a^N)$ with $a<2n+1$. -So, just as in the finite group case, the exponential with the larger base wins, and the polynomial in the denominator is too small to effect the argument. Joel and I discussed a similar, but harder, argument over at the Secret Blogging Seminar.<|endoftext|> -TITLE: Bounding the number of character degrees of a finite group in terms of the order of the group -QUESTION [8 upvotes]: Let $cd(G)$ be the set of degrees of irreducible complex characters of the finite group $G$ (so $cd(G) = \{\chi(1) | \chi\in Irr(G)\}$). - -What bounds are known of the form $|cd(G)|\leq f(|G|)$ (ie, what functions $f$ are known which satisfies such an inequality)? - -I can show that $|cd(G)|\leq \sqrt[3]{3|G|-7}$ and if $|G|$ is odd that $|cd(G)|\leq \sqrt[3]{\frac{12}{15}|G|}$. -On the other hand, one clearly has $|cd(G)|\leq d(|G|)-1$ (where $d(n)$ is the number of divisors of $n$), and this is better asymptotically. -There are two reasons for this question. -One is that I am looking at certain inequalities which guarantee that a group will be solvable, and having a good bound on $|cd(G)|$ in terms of the order of the group would help. -Also, the question is interesting when compared to the Taketa-inequality ($dl(G)\leq |cd(G)|$ which is conjectured to hold for all solvable groups), since clearly the derived length of a solvable group only grows logarithmically in the order of the group (being bounded by the sum of the exponents in the prime factorization of the order of the group). - -REPLY [4 votes]: The number of divisors at least roughly resembles the best achievable lower bound. For each prime $p$, there is a group $G_p$ with $p^3$ elements which has an irreducible representation of dimension 1 (the trivial representation) and an irreducible representation of dimension $p$ (because it's non-abelian). Now let $p_n$ be the $n$th prime. The number -$$P_n = p_1p_2\cdots p_n$$ -is a type of number with a lot of divisors. If you likewise let -$$H_n = G_{p_1} \times G_{p_2} \times \cdots \times G_{p_n},$$ -then $H_n$ has an irreducible representation for every $d$ that divides $P_n$, and it has $P^3_n$ elements. -Now, it is not quite true that $P_n$ has more divisors than any $N < P_n$. It is a good strategy for making a number with many divisors, but soon enough it is better to add more factors of $p_1$, then eventually more factors of $p_2$, etc., than to keep adding new prime factors. To understand this situation better, we can make many numbers (but not all numbers) that have more divisors than their predecessors with the "threshold method". The idea is to optimize the ratio $\log(d(N))/\log(N)$ globally by optimizing it locally (with respect to prime factorization). Pick a constant $t > 0$, the threshold, and say that $N$ should have at least $k > 0$ factors of a prime $p$ if and only if -$$\frac{\log(k+1) - \log(k)}{\log(p)} \ge t.$$ -Then I think that $d(M) < d(N)$ when $M < N$. -In fact, finding large values of $cd(G)$ (which I will use to mean the cardinality of the character degrees rather than the set) is a very similar problem when $G$ is nilpotent. A finite group is nilpotent if and only if it is the product of its Sylow subgroups. The main idea of the construction above is that in this case $cd(G)$ is multiplicative, i.e., the product of its values for $p$-groups. Following the comment by Frieder Ladisch, $cd(G)$ is maximized for $p$-groups by $C_{p^m} \ltimes C_{p^{m+1}}$. (In the first version of the answer I used other $p$-groups that aren't as good.) I.e., this group has character degrees $1,p,\ldots,p^m$, and no $p$ group with $p^{2m}$ or fewer elements can have an irrep with $p^m$ elements. So you can find many record values of $cd(G)$ for nilpotent groups using instead the threshold formula -$$\frac{\log(k+1) - \log(k)}{\min(4-k,2)\log(p)} \ge t.$$ -Let's incorporate the concept of a "record value" by defining $d'(N)$ to be the maximum of $d(M)$ with $M \le N$. Likewise define $cd'(N)$ to be the maximum of $cd(G)$ with $|G| \le N$. Then I think that the above constructions show that $d'(N)$ and $cd'(N)$ are at least similar functions, and that -$$d'(N) > cd'(N) > \sqrt[3]{d'(N)}$$ -when $N$ is large enough. In fact I think that the exponent of the second inequality climbs from $1/3$ to some higher value, although for nilpotent groups one also has -$$\sqrt{d'(N)} > cd'_{\text{nil}}(N).$$ -Let me also mention that the bound $O(\sqrt[3]{|G|})$ follows immediately from the fact that $|G|$ is the sum of the squares of the dimensions of the irreducible representations --- maybe that's what you have in mind with your bound.<|endoftext|> -TITLE: What is the subfactor planar algebra of type $\tilde{A}_n$, of index 4? -QUESTION [6 upvotes]: As I understand it, there is a subfactor whose principal graph is the affine Dynkin diagram $\tilde{A}_n$. Since every vertex has two neighbors, does that mean the space of 1-boxes is two dimensional? Is that allowed? - -REPLY [6 votes]: One concrete realization of this planar algebra goes as follows. - -Start with the (generalized) PA freely generated by an oriented strand. -Impose "$Z$-homology" relations: (a) oriented saddle moves, and (b) erase small loops (loop value = 1). -Now introduce an unoriented strand which is the formal direct sum of an upward pointing strand and a downward pointing strand. -If we now draw the principal graph of this PA from the point of view of the newly introduced unoriented strand type, we get the $\tilde{A}_\infty$ graph. -I just noticed that you want $\tilde{A}_n$, not $\tilde{A}_\infty$. For this case, start with $Z/n$ homology instead of $Z$ homology. If you want more details let me know.<|endoftext|> -TITLE: finite quotients of fundamental groups in positive characteristic -QUESTION [5 upvotes]: For affine smooth curves over $k=\bar{k}$ of char. $p,$ Abhyankar's conjecture (proved by Raynaud and Harbater) tells us exactly which finite groups can be realized as quotients of their fundamental groups. -What about complete smooth curves, or more generally higher dimensional varieties? Are there results or conjectural criteria (or necessary conditions) for finite quotients of their $\pi_1?$ (Definitely, not too much was known around 1990; see Serre's Bourbaki article on this.) -In particular, let $G$ be the automorphism group of the supersingular elliptic curve in char. $p=2$ or $3$ (see supersingular elliptic curve in char. 2 or 3 for various descriptions of its structure). Is there (and if yes, how to construct) a projective smooth variety in char. $p$ having $G$ as a quotient of its $\pi_1?$ Certainly there are lots of affine smooth curves with this property (e.g. $\mathbb G_m$), and I wonder if for some of them, the covering is unramified at infinity (so that we win!). - -REPLY [2 votes]: For a supersingular elliptic $E$ over an algebraically closed field of characteristic two or three there exists a smooth curve $C$ of higher genus such that $Aut_0(E)$ is a finite quotient of $\pi_1(C)$. -This is explained in section 3 of http://arxiv.org/abs/1005.2142v3 -This is an easy application of a general theory of finite quotients of fundamental groups of smooth curves as explained in the paper -Amilcar Pacheco and Katherine F. Stevenson. Finite quotients of the algebraic fundamental group of projective curves in positive characteristic. Pacific J. Math., 192(1):143–158, 2000 -In this paper, it is explained how to realize groups which have the property that their maximal $p$-Sylow subgroup ($p$ being the characteristic) is normal. The automorphism groups of supersingular elliptic curves satisfy this property.<|endoftext|> -TITLE: Clifford theory: behaviour of a very general irreducible representation under restriction to a finite index subgroup. -QUESTION [19 upvotes]: Let $G$ be a group and let $H$ be a subgroup of finite index. -Let $V$ be an irreducible complex representation of $G$ (no topology or anything: $V$ is just a non-zero complex vector space with a linear action of $G$ and no non-trivial invariant subs). -Now consider $V$ as a representation of $H$. Is $V$ a finite direct sum of irreducible $H$-reps? -I am almost embarrassed to ask this question here. It looked to me initially like the answer should be "yes and this question is trivial". If $G$ is finite it is trivial and Clifford theory tells you basically what can happen. Here is another case I can do: if $H$ has index two in $G$ then $V$ is indeed a finite direct sum of irreducibles. For either $V$ is irreducible as an $H$-rep, in which case we're done, or $V$ is reducible, so there's $0\not=W\not=V$ an $H$-stable sub. Say $g\in G$ with $g\not\in H$. One checks easily that $gW$ is $H$-stable, that $W\cap gW$ is $G$-stable, so must be zero, and that $W+gW$ is $G$-stable, so must be $V$. Hence $V$ is the direct sum of $W$ and $gW$. This implies that $W$ is irreducible as an $H$-rep---for if $X$ were a non-trivial sub then the same argument shows $V=X\oplus gX$ but this is strictly smaller than $W\oplus gW=V$. -I thought that this argument should trivially generalise to, say, the case where $H$ is a normal subgroup of prime index. But I can't even do the case where $H$ is normal and $G/H$ has order $3$, because I can't rule out $V$ being the sum of any two of $W$, $gW$ and $g^2W$, and the intersection of any two being trivial. -Either I am missing something silly (most likely!) or there's some daft counterexample. I almost feel that I would be able to prove something if I knew Schur's lemma [edit: by which I mean that if I knew $End_G(V)=\mathbf{C}$ then I might know how to proceed], but in this generality I don't see any reason why it should be true. Perhaps if I knew a concrete example of an irreducible complex representation of a group for which Schur's lemma failed then I might be able to get back on track. [edit: in a deleted response, Qiaochu pointed out that $G=\mathbf{C}(t)^\times$ acting on $\mathbf{C}(t)$ provided a simple example] [final remark that in the context in which this question arose, $G$ was a locally profinite group and $V$ was smooth and I could use Schur's Lemma, but by then I was interested in the general case...] - -REPLY [9 votes]: Since $V$ is irreducible, it is a finitely generated $\mathbb C[G]$-module, any non-zero element is a generator. Since $H$ is of finite index, $\mathbb C[G]$ is a finitely generated -$\mathbb C[H]$-module. Hence $V$ is a finitely generated $\mathbb C[H]$-module. Zorn's Lemma implies the existence of an irreducible quotient $W$. -Suppose that $H$ is normal in $G$. (It is probably enough to suppose that $H$ contains a subgroup of finite index, which is normal in $G$.) Let $K$ be the kernel of $V\rightarrow W$, for every $g\in G$ the quotient $V/g K$ is an irreducible $H$-rep, isomorphic to $W^g$. The kernel of the natural map -$$V\rightarrow \bigoplus_{g\in G/H} V/g K$$ -is $G$ invariant, and hence $0$. So we may inject $V$ into a finite direct sum of irreducible $H$-reps. Choose a smallest subset $X\subset G/H$, such that $\varphi: V\rightarrow \bigoplus_{g\in X} V/g K$ is injective, then $\varphi$ is also surjective. -Edit. Kevin pointed it out that $H$ always contains a subgroup of finite index, which is normal in $G$, and F.Ladisch finished off the general case, i.e without assuming that $H$ is normal in the comments below.<|endoftext|> -TITLE: Trivial fiber bundle -QUESTION [8 upvotes]: Suppose $(E,p,B;F)$ is a fiber bundle such that $E$ is homeomorphic to $B\times F$, is it true that the fiber bundle is trivial? -A non connected counter example has been provided, so I'll ask for E,B and F to be connected (hopefully low dimensional) manifolds. - -REPLY [13 votes]: Consider the pullback $\xi$ of $TS^2$ via the projection of $S^2\times\mathbb R$ onto the first factor. The bundle $\xi$ is a nontrivial $\mathbb R^2$-bundle over $S^2\times\mathbb R$ because its pullback under the inclusion $S^2\to S^2\times\mathbb R$ is $TS^2$, which is nontrivial. On the other hand, its total space is $\mathbb R\times TS^2$ which is diffeomorphic to $S^2\times\mathbb R^3$, which is the total space of the trivial $\mathbb R^2$-bundle over $S^2\times\mathbb R$.<|endoftext|> -TITLE: Compact open topology on $\mathrm{Homeo}(X)$ -QUESTION [29 upvotes]: Let $X$ and $Y$ be topological spaces. Define the compact open topology on the set $\mathrm{M}(X,Y)$ of continuous maps from $X$ to $Y$ via the subbase $[K,O]$ of all maps $f:X\rightarrow Y$ s.t. $f(K)\subset O$, where $K$ is any compact subset of $X$, and $O$ is any open subset of $Y$. So a basis of open sets is given by the following subsets: $[K_1,\dots,K_n,O_1,\dots,O_n]=[K_1,O_1 ]\cap\dots\cap [K_n,O_n]$, the collection of continuous maps $f:X\rightarrow Y$ that send each $K_i$ into $O_i$ for some specified collection of compact $K_i$'s and open $O_i$'s. -This topology has some nice properties: the exponential law holds under some hypotheses on the spaces $X$ and $Y$, and is certainly true if all spaces involved are locally compact Hausdorff spaces, as will be the case from now on. -My question is as follows: if $X$ is a locally compact Hausdorff space (or even a topological manifold), the compact open topology induces a topology on the set of homeomorphisms of $X$, which is a group. Does this topology turn $\mathrm{Homeo}(X)$ into a topological group? I can show that the product (composition) is continuous, but is the inverse too? $(f\rightarrow f^{-1})$ -I was able to prove continuity for compact spaces, where it is very easy to establish. I also managed to prove it for $X=\mathbb{R}$ because all homeomorphisms of $\mathbb{R}$ are monotone, but that's everything so far. -I tried looking it up in several textbooks on topology and algebraic topology where the C.O. topology is usually discussed, but couldn't find a discussion on this topic anywhere. - -REPLY [8 votes]: R. Arens, Topologies for homeomorphism groups, Amer. J. Math. 68 (1946) 593–610. -If $X$ is locally compact and locally connected (!), then $\mathrm{Homeo}(X)$ is a topological group.<|endoftext|> -TITLE: Why study Lie algebras? -QUESTION [137 upvotes]: I don't mean to be rude asking this question, I know that the theory of Lie groups and Lie algebras is a very deep one, very aesthetic and that has broad applications in various areas of mathematics and physics. I visited a course on Lie groups, and an elementary one on Lie algebras. But I don't fully understand how those theories are being applied. I actually don't even understand the importance of Lie groups in differential geometry. -I know, among others, of the following facts: - -If $G$ and $H$ are two Lie groups, with $G$ simply connected, and $\mathfrak{g,h}$ are their respective Lie algebras, then there is a one to one correspondance between Lie algebra homomorphisms $\mathfrak{g}\rightarrow\mathfrak{h}$ and group homomorphisms $G\rightarrow H$. -The same remains true if we replace $H$ with any manifold $M$: any Lie algebra homomorphism from $\mathfrak{g}$ to the Lie algebra $\Gamma(TM)$ of smooth vector fields on $M$ gives rise to a local action of $G$ on $M$. -Under some conditions like (I think) compactness, the cohomology of $\mathfrak{g}$ is isomorphic to the real cohomology of the group $G$. I know that calculating the cohomology of $\mathfrak{g}$ is tractable in some cases. -There is a whole lot to be said of the representation theory of Lie algebras. -Compact connected centerless Lie groups $\leftrightarrow$ complex semisimple Lie algebras - -How do people use Lie groups and Lie algebras? What questions do they ask for which Lie groups or algebras will be of any help? And if a geometer reads this, how, if at all, do you use Lie theory? How is the representation theory of Lie algebras useful in differential geometry? - -REPLY [3 votes]: I hope you may be interested in your questions restricted to Dynamical Systems (DS): - -How do people use Lie groups and Lie algebras in DS? -What questions do they ask for which Lie groups or algebras will be -of any help in DS? - -In DS people are interested in particular in: dense orbits and invariant measures for actions of groups on a manifold. A tool to study this is Ergodic Theory (ET). -There is an abstract ET of amenable groups actions (Ornstein-Weiss), I have seen that Lie groups provide: concrete examples, counter-examples and extensions of this abstract theory. I have also seen that one important reason why Lie groups allow to prove deep ET results is because they are well adapted to the theory of harmonic analysis, and harmonic analysis is one of the main machinery of ET. -The ET of Lie groups is a very broad theory, just a few examples: -Example 1. -Let $G\subset SL(\mathbb{Z},d)$ (such that $G$ acts irreducibly on $\mathbb{R}^d$ and $G$ does -not contain an abelian subgroup of finite index). Then the $G$-action on the d-torus is strongly ergodic (''the G-invariatn measure is the unique G-invariant mean''). Moreover, there is a characterization of the $G$-action with $G\subset SL(\mathbb{Z},d)$ on the d-torus that are strongly ergodic. And more in abstract, if $G$ is a connected non-compact simple Lie group with finite center, there is a characterization for the action to be strongly ergodic. -These are theorems of A. FURMAN and Y. SHALOM that generalize Rosenblatt: - -A. Furman and Y. Shalom. Sharp ergodic theorems for group actions and -strong ergodicity. -J. Rosenblatt. Uniqueness of invariant means for measure preserving transformations. Trans. Amer. -Math. Soc. 265 (1981), 623–636 - -Example 2. -Orbit-equivalence rigidity (Zimmer): -Let $G_1$ and $G_2$ be noncompact, simple Lie groups, with $R$-$rank(G_1)\geq 2.$ If $(G_1,X_1)$ is orbit equivalent to $(G_2,X_2)$, then $G_1$ is locally isomorphic to $G_2$, and, up to a group automorphism, the actions are isomorphic. -Example 3. -A. Avila, B. Fayad, and A. Kocsard provide in (On manifolds supporting distributionally uniquely ergodic diffeomorphisms) some counterexamples to a conjecture proposed by Forni in “On the Greenfield-Wallach and Katok conjectures in dimension three. -A Good references for ET of actions of Lie group is R. Zimmer. Ergodic Theory and Semisimple Lie Groups. Birkh¨auser, Boston, 1984. -You can find deep results by: A. Avila, V. Bergelson, G. Forni, J. Rosenblatt, A. Furman, H. Furstenberg, D. Y. Kleinbock, G. A. Margulis, Y. Shalom, A. Katok and Zimmer, between many others.<|endoftext|> -TITLE: A geometric series equalling a power of an integer -QUESTION [18 upvotes]: The following problem cropped up whilst considering generalised quadrangles with a product structure, and it boils down to a simple number theoretic problem. Let $s$ be an integer greater than 2 and suppose the geometric series $(s^r-1)/(s-1)$ is a nontrivial power of a positive integer. It seems the following is true: -If $r=3$, then $s= 18$. -If $r=4$, then $s = 7$. -If $r=5$, then $s = 3$. -If $r>5$, there are no solutions. -Does anyone know a proof of this curious property? - -REPLY [32 votes]: This is a well-investigated Diophantine equation known as Nagell--Ljunggren equation (they investigated this equation in the 1920s and 1940s, resp). Indeed, it is conjectured that the three solutions mentioned by the questioner are the only ones; however, it is not even known that the number of solutions is finite, though there are numerous partial result. -Below, I try to give some rough overview of some results that are known, and some references to (recent) articles. - -First, I restate the question to bring the notation in line with some sources I quote. -What are the solutions $(x,y,n,q)$ of the equation -$$ \frac{x^n - 1}{x - 1} = y^q $$ -with integers $x,y>1$, $n>2$, $q \ge 2$ ? -As mentioned, in the question, one finds three 'small' solutions -$(3,11,5,2)$, $(7,20,4,2)$, and $(18,7,3,3)$. -And, the remaining question is: -(A) Are these three solutions all the solutions ? -Or more modestly -(B) Is the number of solutions finite? - -As said, even (B) is open; but (A) is conjectured to be true. -By early works of Nagell and Ljunggren it is known that with any of the following conditions there are no other solutions: $q=2$, $n$ a multiple of $3$, $n$ a multiple of $4$, or ($q=3$ and $n$ not $5$ modulo $6$). -Shorey and Tijdeman proved (1976) that the number of solutions is finite with any of the following conditions: $x$ is fixed, $n$ has a fixed prime divisor, $y$ has a fixed prime divisor. Also, Shorey proved that the ABC-conjecture implies that the number of solutions is finite. -There are numerous additional results, imposing various conditions on $x,y,n$ or $q$ (due to Bennet, Bugeaud, Le, Mignotte, and others) for a survey of the state of the art around a decade ago see, e.g., a 2002 survey (in French) of Bugeaud and Mignotte (which was also the main bases for the above written part) available here. -The early results were obtained via passing to certain rings of algebraic integers; later results often used Baker's method (linear forms in logarithms) and results on Diophantine approximation. -Some years ago, the solution of Catalan's conjecture (which is on a somewhat similar equation), by Mihailescu that (as far as I understand, very surprisingly) avoided all these types of tools and used instead (only) results on cyclotomic fields/integers, provided a new impetus. -Specifically, it is now known, see Bugeaud and Mihailescu (2007), that -a. for any other solution (so not one of three known ones) the smallest prime divisor of $n$ is at least $29$ and $n$ has at most $4$ prime divisors (counted with multiplicity). Moreover, $n$ is prime if $q=3$. And, if $q\mid n$, then $q=n$. -b. to prove that there are no other solutions, it suffices to show that there is no solution with $n\ge 5$ an odd prime and $q$ an odd prime. -Moreover, related to the latter assertion Mihailescu recently proved (see here and here) various results in the case that $n$ and $q$ are odd primes (saying, in one of the abstracts that methods used in the cyclotomic approach to FLT are used, so Yemon Choi's intuition was very right). -This answer does certainly not give a complete picture (in this format, it would be difficult to give one, and no matter the format, it would be impossible for me); I am aware of various omissions I made, and I am afraid there are many of which I am not aware. -The references I mentioned should however allow to retrieve more complete information. -[Note: in case the tex is broken, it is not carelessness; at the moment, for technical reasons, I cannot test it myself.]<|endoftext|> -TITLE: Number of divisors of an integer of form 4n+1 and 4n+3 -QUESTION [6 upvotes]: Suppose $n$ is a large odd integer. Let $D_1(n)$ be the number of divisors of $n$ of the form $4k+1$ and let $D_3(n)$ be the number of divisors of the form $4k+3$. I would like to compute $(D_1(n),D_3(n))$. -As Joe Silverman points out, the number of representations of $n$ as a sum of two squares of integers is $4(D_1(n)-D_3(n))$. For example, $D_1(225)=6$ and $D_3(225)=3$, so there are $4(6-3)=12$ lattice points on the circle of radius $\sqrt {225}$ centered at the origin including $(0,15)$ and $(-9,-12)$. - -Is there a faster way to find $(D_1(n),D_3(n))$ than factoring $n$? - - -Original: -Hi, one way to do so is to list all the divisors of the integer and check each if it is of the form $4n+1$ or $4n+3$. -Is there any faster method to it, especially for large $n$? - -REPLY [12 votes]: Not quite what you're asking, but an interesting theorem of Legendre's says that the number of ways of writing an integer $N$ as a sum of two squares is $4D_1(N)-4D_3(N)$, where $D_1(N)$ is the number of positive divisors of $N$ that are congruent to 1 modulo 4 and $D_3(N)$ is the number of positive divisors of $N$ that are congruent to 3 modulo 4. There are undoubtedly also results proved via analytic methods that describe the distribution of $D_1(N)$ and $D_3(N)$. But I'd have to agree with the other posters that computing $D_1(N)$ and $D_3(N)$ for a specific $N$ sounds about as hard as factoring $N$. Indeed, if $N=pq\equiv 1 \pmod{4}$, computing $D_1(N)$ is equivalent to computing the first bit in the factors of $N$, which seems hard.<|endoftext|> -TITLE: Construction of the determinant line bundle on the degree $g-1$ Picard variety -QUESTION [15 upvotes]: Consider $J^{g-1}$, the variety of degree $g-1$ line bundles on a compact Riemann surface of genus $g$. Recall that $J^{g-1}$ is a torsor for the Jacobian, thus has dimension $g$. We can produce elements in $J^{g-1}$ by choosing $g-1$ points $p_1,\ldots,p_{g-1}$ and constructing the associated line bundle to the divisor $p_1+\cdots+p_{g-1}$. In this way, we obtain a $g-1$-dimensional family of elements of $J^{g-1}$, forming the so-called $\Theta$ divisor. Elements of the $\Theta$ divisor may be distinguished from other points in $J^{g-1}$ by the fact that they represent line bundles admitting sections, i.e. $h^0(L)\neq 0$. -For degree $g-1$ line bundles, the Riemann-Roch theorem gives $h^0(L)=h^1(L)$, so over $J^{g-1}$ we see that $h^0(L),h^1(L)$ are generically zero and jump along the $\Theta$ divisor. One might imagine that there are vector bundles $V,W$ over $J^{g-1}$ of the same rank, with fibres over $L\in J^{g-1}$ given by Dolbeault $0$- and $1$-forms with coefficients in $L$ respectively, together with a bundle map $\overline{\partial}:V\rightarrow W$ which is generically an isomorphism but drops rank along $\Theta$; then $\det \overline{\partial}$ would be a section of a line bundle $\det V^*\otimes\det W$ over $J^{g-1}$ which cuts out $\Theta$. This line bundle turns out to be a well-defined object called the determinant line bundle and it was introduced by Quillen. -Quillen's construction of the line bundle proceeds by replacing the family of (2-term) Dolbeault complexes parametrized by $J^{g-1}$ by a quasi-isomorphic family of finite-dimensional (2-term) complexes and taking the determinant line bundle of this. The finite-dimensional replacement for the Dolbeault complex is given by the Dolbeault operator acting on forms lying in the first few eigenspaces of the Laplacian, roughly speaking. Then as we move along $J^{g-1}$, some of the eigenvalues may hit zero and we have a jump of $h^0(L)$. -For a good list of references for the above, see determinant line bundle (nlab). -My question: What is a simple, modern construction of the determinant line bundle on $J^{g-1}$, perhaps one which uses the tools of algebraic geometry? It may well be tautological from some point of view, in which case I would probably be unsatisfied. Also, if you wish to rhapsodize on determinant line bundles, please feel free. -My motivation: Many of the classic papers by Beauville, Narasimhan, Ramanan etc. on moduli of vector bundles expend a lot of effort working with determinant line bundles and extracting information from them in quite ad-hoc and clever ways. I'd like to understand these better. - -REPLY [16 votes]: Let's say that your (compact!) Riemann surface is $X$ (of genus at least $1$). What does it mean to say that $J^{g-1}$ is the "variety of degree $g-1$ line bundles?" One way to formalize this is to say that there is a line bundle $\mathcal{L}$ on the product $X \times J^{g-1}$ with the property the rule $p \mapsto \mathcal{L}|_{X \times \{ p \}}$ defines a bijection between the ($\mathbb{C}$-valued) points of $J^{g-1}$ and the line bundles of degree $g-1$. (A stronger statement is that $J^{g-1}$ represents the Picard functor, but I'm going to try to sweep this under the run.) -Let $\pi \colon X \times J^{g-1} \to J^{g-1}$ denote the projection maps. Recall that the formation of the direct image $\pi_{*}(\mathcal{L})$ does not always commute with passing to a fiber. However, the theory of cohomology and base change describes how the fiber-wise cohomology of $\mathcal{L}$ varies. The main theorem states that there is a 2-term complex of vector bundles -$d \colon \mathcal{E}_0 \to \mathcal{E}_1$ -that computes the cohomology of $\mathcal{L}$ universally. That is, for all morphisms $T \to J^{g-1}$, we have -$\operatorname{ker}(d_{T}) = (\pi_{T})_{*}(\mathcal{L}_{X \times T})$ -and -$\operatorname{cok}(d_{T}) = (R^{1}\pi_{T})_{*}(\mathcal{L}_{X \times T}).$ -(The most important case is where $T = \operatorname{Spec}(\mathbb{C})$ and -$T \to J^{g-1}$ is the inclusion of a point.) -The complex $\mathcal{E}_{\cdot}$ is not unique, but any other complex of vector bundles with this property must be quasi-isomorphic. -In the literature, many authors construct a complex $\mathcal{E}_{\cdot}$ using an explicit procedure, but the existence is a very general theorem. I learned about this topic from Illusie's article "Grothendieck's existence theorem in formal geometry," which states the base change theorems in great generality. -In your question, you described how to convert the complex into a line bundle (take the difference of top exterior powers). But now we must check that two quasi-isomorphic complexes have isomorphic determinant line bundles. This can be checked by hand, but this statement has been proven in great generality by Mumford and Knudsen (see MR1914072 or MR0437541). The determinant of the complex $\mathcal{E}_{\cdot}$ is exactly the line bundle you are asking about. -One subtle issue is that the universal line bundle $\mathcal{L}$ is NOT uniquely determined. If $\mathcal{M}$ is any line bundle on $J^{g-1}$, then -$\mathcal{L} \otimes \pi^{-1}(\mathcal{M})$ also parameterizes the degree $g-1$ line bundles in the sense described above (and in a stronger sense that I am sweeping under the rug). However, one can show that $\mathcal{L}$ and $\mathcal{L} \otimes p^{*}(\mathcal{M})$ have isomorphic determinants of cohomology by using the fact that a degree $g-1$ line bundle has Euler characteristic equal to zero. -Added: Here is one method of constructing the complex. Fix a large collection of points $\{x_1, \dots, x_n\} \subset X$ (at least $g+1$ points will do), and let $\Sigma \subset X \times J^{g-1}$ denote the subset consisting of pairs $(x_i,p) \in X \times J^{g-1}$. The universal line bundle $\mathcal{L}$ fits into a short exact sequence: -$$ - 0 \to \mathcal{L}(-\Sigma) \to \mathcal{L} \to \mathcal{L}|_{\Sigma} \to 0 -$$ -Here $\mathcal{L}(-\Sigma)$ is the subsheaf of local sections vanishing along $\Sigma$ and $\mathcal{L}|_{\Sigma}$ is the pullback of $\mathcal{L}$ to $\Sigma$. -Associated to the short exact sequence on $X \times J^{g-1}$ is a long exact sequence relating the higher direct images under $\pi$. The first connecting map is a homomorphism -$d : \pi_{*}(\mathcal{L}|_{\Sigma}) \to R^{1}\pi(\mathcal{L}(-\Sigma))$ -This is a complex $K_{\cdot}$ of vector bundles that compute the cohomology of $\mathcal{L}$ universally in the sense described earlier. Thus, its determinant is the desired line bundle. -Why is this true? There are two statements to check: that the direct images are actually vector bundles and that the complex actually computes the cohomology of $\mathcal{L}$. Both facts are consequences of Grauert's theorem. Indeed, the hypothesis to Grauert's theorem is that the dimension of the cohomology of a fiber $\pi^{-1}(p)$ is constant as a function of $p \in J^{g-1}$. Using Riemann-Roch (and the fact that we chose a large number of points), this is easily checked. The theorem allows us to conclude that both $\pi_{*}(\mathcal{L}|_{\Sigma})$ and $R^1 \pi(\mathcal{L}(-\Sigma))$ are vector bundles whose formation commutes with base change by a morphism $T \to J^{g-1}$. -An inspection of the relevant long exact sequence shows that the cohomology of the complex -is $\pi_{*}(\mathcal{L})$ and $R^1\pi(\mathcal{L})$ (i.e. the complex compute the cohomology of $\mathcal{L}$). We would like to assert that this property persists if we base-change by a morphism $T \to J^{g-1}$ and work with the complex -$K_{\cdot} \otimes \mathcal{O}_{T}$. -But we already observed that the formation of the direct images appearing in the complex commutes with base-change, and so the base-changed complex fits into a natural long exact sequence. Again, an inspection of this sequence shows that base-changed complex compute the cohomology of $\mathcal{L}|_{X \times T}$, and so the original complex computes the cohomology universally.<|endoftext|> -TITLE: When does symmetry in an optimization problem imply that all variables are equal at optimality? -QUESTION [33 upvotes]: There are many optimization problems in which the variables are symmetric in the objective and the constraints; i.e., you can swap any two variables, and the problem remains the same. Let's call such problems symmetric optimization problems. The optimal solution for a symmetric optimization problem - like many of the ones that show up in calculus texts - frequently has all variables equal. To take some simple examples, - -The rectangle with fixed area that minimizes perimeter is a square. (Minimize $2x+2y$ subject to $xy = A$ and $x,y \geq 0$.) -The rectangle with fixed perimeter that maximizes area is a square. (Maximize $xy$ subject to $2x + 2y = P$ and $x,y \geq 0$.) -The difference between the arithmetic mean and the geometric mean of a set of numbers is minimized (and equals $0$) when all the numbers are equal. - -There are also more complicated symmetric optimization problems for which the variables are equal at optimality, such as the one in this recent math.SE question. -However, it is not true that every symmetric optimization problem has all variables equal at optimality. For example, the problem of minimizing $x +y$ subject to $x^2 + y^2 \geq 1$ and $x, y \geq 0$ has $(0,1)$ and $(1,0)$ as the optimal solutions. - -Does anyone know of general conditions on a symmetric optimization problem that guarantee the optimal solution has all variables equal? - -The existence of such conditions might be very nice. Unless the conditions themselves are ugly, they ought to vastly simplify solving a large class of symmetric optimization problems. -(Maybe convexity plays a role? My last example has a nonconvex feasible region.) - -REPLY [2 votes]: Suppose you have any optimization problem that is symmetric. I somehow weaker question is: How much of the original symmetry carries over to the solutions? For the symmetric group $S_n$ if the degree $d$ of the polynomials that describe the problem is low (compared to the number of variables) the "degree and half principle" says that one always finds minimizers contained in the set of points invariant by a group $S_{l_1}\times\ldots\times S_{l_d}$ where $l_1+\ldots+l_d=n$<|endoftext|> -TITLE: Torus based cryptography -QUESTION [5 upvotes]: In cryptography one needs finite groups $G$ in which the discrete logarithm problem is infeasible. Often they use the multiplicative group $\mathbb{G}_m(\mathbb{F}_p)$ where $p$ is a prime number of bit length $500$, say. -Rubin and Silverberg suggested (cf. [1]) to use certain tori instead, if the goal is to minimize the key size. In the easiest case, this comes down to using the group -$$T_2(p)=ker(Norm: \mathbb{F}_{p^2}^\times\to \mathbb{F}_p^\times).$$ -If I understood correctly, then the underlying philosopy seems to be: The group $T_2(p)$ should be as secure as $\mathbb{F}_{p^2}^\times$, but its size is only $p+1$. (So, if you use groups of type $T_2(p)$ instead of groups of type $\mathbb{G}_m(\mathbb{F}_p)$, then you can achive the same security with half the key size.) -Question. What are the reasons, be they heurisical or strictly provable, to believe -in this philosopy? -Denote by $\mathbb{G}'_m$ the quadratic twist of the algebraic group $\mathbb{G}_m$. -It is easy to see that $T_2(p)$ is isomorphic to $\mathbb{G}'_m(\mathbb{F}_p)$. -(This isomorphism is easy to compute). The philosophy predicts: The quadratic twist of the multiplicative group should be better than the multiplicative group itself. -(Compare with elliptic curves: If $E/\mathbb{F}_p$ is an elliptic curve, then I would certainly not expect its quadratic twist to be better than $E$ itself.) -Remark: I concentrated on the simplest case above. One also considers certain groups $T_n(p)$ which are expected to be as secure as $\mathbb{F}_{p^n}^\times$, while their size is only $\approx\varphi(n)p$. Lemma 7 in [1] is meant to explain this. However, I would be keen on a more detailed explanation. -[1] Lect. Notes in Comp. Sci. 2729 (2003) 349-365. (available at http://math.stanford.edu/~rubin/) - -REPLY [6 votes]: Can I refer you to my paper `On the Discrete Logarithm Problem on Algebraic Tori', Advances in Cryptology – CRYPTO 2005, Lecture Notes in Computer Science, 2005, Volume 3621/2005, 66-85, in which myself and Frederik Vercauteren studied this very problem. -In particular, we showed that the compression mechanism afforded by the birationality of some algebraic tori may be exploited to obtain a faster discrete logarithm algorithm for some cryptographically practical field sizes. In these instances, attacking the discrete logarithm in $\mathbb{F}_{p^n}^{\times}$ via its decomposition -$\prod_{d \mid n} T_d(\mathbb{F}_p)$ -is faster than using L[1/3] index calculus techniques. -Since then, other work has improved the L[1/3] index calculus techniques. However, our work demonstrates that it is naive to argue that the DLP in algebraic tori must be -hard purely because the DLP in the multiplicative group of the extension field is hard, precisely because an attack on the former provides an attack on the latter.<|endoftext|> -TITLE: Why is Casson's invariant worth studying? -QUESTION [20 upvotes]: Hi everybody! I am reading some papers about Casson's invariant for (integral) homology 3-spheres...as the wiki says "Informally speaking, the Casson invariant counts the number of conjugacy classes of representations of the fundamental group of a homology 3-sphere M into the group $SU(2)$". It seems to be something interesting to study, but this is "my first trip" in the realm of 3-manifold topology, so I don't get the deep meaning of this invariant. I mean why should one study this invariant? what should I be expecting from it? which contributions is it likely to bear to this field? In particular I came across the Casson invariant while studying Heegaard splittings...do you have any reading to suggest? -Thank you, -Lor - -REPLY [31 votes]: Wikipedia's description of the Casson invariant gives the first important reason to study it. As an invariant that comes from the $\text{SU}(2)$ representation variety of $\pi_1(M)$, it reveals in particular that $\pi_1(M)$ is non-zero. At the time, before Perelman's proof of the Poincaré conjecture and geometrization, there was a lot of mystery about potential counterexamples to the Poincaré conjecture. For instance, one speculation was that the so-called $\mu$ invariant could reveal a counterexample. Since the Casson invariant lifts the $\mu$ invariant, and since it proves that $\pi_1(M)$ is non-trivial when it is non-zero, it is one way to see that the $\mu$ invariant can never certify a counterexample to the Poincaré conjecture. (Of course, no we know that there are no counterexamples.) -A second fundamental reason to study the Casson invariant is that it is the only finite-type invariant of homology spheres of degree 1. Many interesting 3-manifold invariants are finite-type, or (conjecturally) carry the same information as a sequence of finite-type invariants. This is known more rigorously at the level of knots; for instance, the derivatives of the Alexander polynomial, the Jones polynomial, and many other polynomials at $1$ are all finite-type invariants. At the level of knots, the second derivative of the Alexander polynomial, $\Delta''_K(1)$, is known to be the only non-trivial finite-type invariant of degree 2, and there is nothing in degree 1. So it means that this invariant appears over and over again as part of the information of many other invariants; there are many different definitions of the same $\Delta''_K(1)$. The same thing should happen to the Casson invariant, and indeed there are already two very different-looking types of definitions: (1) Casson's definition; (2) either the first LMO invariant or the first configuration-space integral invariant. -A third fundamental reason is that Casson invariant has an important categorification, Floer homology, which is the objects in the theory whose morphisms come from Donaldson theory. One wrinkle of this construction is that it is only a categorification of one of the definitions of Casson's invariant, Casson's definition. If Casson's invariant has many definitions, then it might (for all I know) have many different categorifications. -If your question is meant in the narrow sense of what topology you can prove with the Casson invariant, then you can definitely prove some things but only (so far) a limited amount. However, if you are interested in quantum topological invariants in their own right, and not just as a tool for pre-quantum topology problems, then the Casson invariant is important because it is a highly non-trivial invariant that you encounter early and often.<|endoftext|> -TITLE: Is the unitary group of $l^2(A)$ with the strict topology contractible? -QUESTION [5 upvotes]: Let $A$ be a $C^*$-algebra with countable approximate unit. Let $\mathbb{K}$ denote the compact operators on a separable Hilbert space. Mingo and later Cuntz and Higson have shown that the unitary group in the multiplier algebra $M(A \otimes \mathbb{K})$ is contractible in the norm topology. It was then shown by Troitsky in a paper with the title -Geometry and Topology of operators on Hilbert $C^*$-modules -that $U(M(A \otimes \mathbb{K}))$ is also contractible, if it is equipped with the left strict topology, i.e. the topology generated by the semi-norms $\lVert xa \rVert$ for $x \in M(A \otimes \mathbb{K})$ and $a \in A \otimes \mathbb{K}$. Is the theorem still true, if we change from left strict to strict (which is the topology that includes the semi-norms $\lVert ax \rVert$)? - -REPLY [4 votes]: I would post this as a comment but as it just happens I can't do that. I do think that -the exercise that you mention proves strict contractibility. The same formula for the homotopy, -$$ -(u,t)\mapsto w_tuw_t^*+(1-w_tw_t^*), -$$ -is given in Proposition 12.2.2 of Blackadar's book on K-theory, although the statement -only says that the unitary group of $M(A\otimes K)$ is path connected. This formula goes back to Dixmier and Douady's paper on fields of Hilbert spaces, applied to $B(H)$.<|endoftext|> -TITLE: Ideals in smooth subalgebras of C*-algebras -QUESTION [12 upvotes]: Let $B$ be a $C^{*}$-algebra and $\mathcal{B}$ a dense *-subalgebra stable under holomorphic functional calculus and $C^{1}$-functional calculus for selfadjoint elements. Also, $\mathcal{B}$ is a Banach algebra in a norm $\|\cdot\|_{1},$ satisfying -$\|\cdot\|\leq\|\cdot\|_{1}$. -Also, there is a countable bounded approximate unit $u_{n}$ for $\mathcal{B}$ which is a contractive, increasing approximate unit for $B$. Let $\mathcal{I}$ be a closed two sided ideal in $\mathcal{B}$, and denote by $I$ its closure in $B$. -Is it true that $\mathcal{I}=I \cap \mathcal{B}$ ? -The pertinent examples are Lipschitz functions on the circle and on the real line, both with norm $\|f\|_{1}=\|f\|+\|\partial f\|$. - -REPLY [5 votes]: $\newcommand{\norm}[1]{\Vert#1\Vert}$ -In general, I think the answer to your question is no. Take ${\mathcal B}=C^1[-1,1]$ with the norm -$\norm{f}= \norm{f}_\infty+\norm{f'}_\infty$ -and let ${\mathcal I}$ be the closed ideal consisting of those $C^1$-functions which vanish at $x=0$ and whose 1st derivative vanishes at $x=0$. Then $I\cap {\mathcal B}$ contains the function $f(x)=x$ which is evidently not in ${\mathcal I}$. - -[Some general remarks follow, in a rambling style owing to lack of sleep. I may try to edit these later.] -In the commutative unital setting, taking $B=C(X)$, we know what the closed ideals of $B$ are (they are precisely the "kernels" of closed subsets of $X$, in the language of hulls and kernels). -If your subalgebra ${\mathcal B}$ also has maximal ideal space (homeo to) $X$, then your question is related to -- perhaps is equivalent to, I have not thought in detail -- the following one: -Can I find a closed two sided ideal in ${\mathcal B}$ which is not the kernel of its hull? -Without your restrictions on stability-under-func-calc, this kind of question has been much studied for commutative examples, and I think also for certain noncommutative examples related to group algebras. -For little Lipschitz algebras (on the circle) the answer is no -- this ought to be in a paper of Sherbert from the 1970s --- so I expect the answer to your original question is "yes". (For the "big" Lipschitz algebras my suspicion is that the counter-example I gave for $C^1[-1,1]$ would also work.)<|endoftext|> -TITLE: Approximate units from strictly positive elements in $C^{*}$-algebras. -QUESTION [6 upvotes]: The existence of a countable approximate unit in a $C^{*}$-algebra $B$ is equivalent to the existence of a strictly positive element $h\in B$. There are several ways to construct an approximate unit from $h$. My question is, does $h(h+\frac{1}{n})^{-1}$ constitute an approximate unit for $B$? - -REPLY [7 votes]: I think this works: Functional calculus shows that $h h (h+1/n)^{-1} \rightarrow h$. Then $h$ is strictly positive if and only if $Bh$ is dense in $B$ (See, for example, Jensen+Thomsen, "Elements of KK-Theory", Lemma 1.1.21). So for $b\in B$ and $\epsilon>0$, we can find $c\in B$ with $\|b-ch\|<\epsilon$, and for all $n$ sufficiently large, also $\|ch h(h+1/n)^{-1} - ch\| < \epsilon$. Thus -\begin{align*} &\| b - bh(h+1/n)^{-1} \| \\&< \epsilon + \| ch - chh(h+1/n)^{-1}\| -+ \|chh(h+1/n)^{-1} - bh(h+1/n)^{-1}\| \\ -&< 2\epsilon + \epsilon \|h(h+1/n)^{-1}\| < 3\epsilon. \end{align*} -Thus we're done.<|endoftext|> -TITLE: coherent sheaves on affine formal schemes -QUESTION [7 upvotes]: Let $\hat{X} = \text{Spf} \hat{A}$ be obtained as the formal completion of an affine scheme $X = \text{Spec} A$ where $A$ is an adic noetherian ring. Given a coherent sheaf $\mathfrak{F}$ on $\hat{X}$, is it always possible to find a coherent sheaf $\mathcal{F}$ on $X$ such that $\hat{\mathcal{F}} = \mathfrak{F}$? - -REPLY [6 votes]: Take $A=k[x,y]$, $\hat A=k[[x,y]]$, and suppose coherent sheaf corresponds to the $\hat A$-module $N=\hat A/(f)$, where $f\in k[[x,y]]$ is not "algebraic": say, $f=y-exp(x)$, assuming $k$ has characteristic zero. It is clear that $N$ does not come from completion of any f.generated $A$-module $M$. (Proof: Suppose $N=\hat M$. Since $N$ has no $A$-torsion, any torsion of -$M$ maps to zero, so we can replace $M$ with $M/(torsion)$ and assume $M$ is torsion-free. Then $M$ embeds in a locally free module (its second dual), but that would make $N$ embed in the free $\hat A$-module, which is false.)<|endoftext|> -TITLE: Curve integral of exponent of superharmonic function. -QUESTION [15 upvotes]: Let $\phi$ be a real smooth superharmonic function on unit disc $D$ in $\mathbb C$; i.e. $\triangle \phi\le 0$. - Then there is a curve $\gamma$ from the center of $D$ to its boundary such that - $$\int\limits_\gamma e^\phi<\infty.$$ - -The question came from my failed answer to this question. -I know that the answer is YES, but I do not see a direct proof. - -REPLY [5 votes]: I'll assume that $\varphi=-\psi$ where $\psi$ is subharmonic and not too weird (say, with isolated non-degenerate critical points; it seems like you can always add something bounded to achieve it but I haven't checked the details). Take any piece of some level curve of $\psi$ inside the disk parameterized by length $\ell$ and start the gradient accent from each point parameterized by the level $t$ of $\psi$. All but countably many of those escape to the boundary. Let $v(t)$ be the absolute value of the gradient and $S(t)$ be the "cross-section factor". Then $S(t)v(t)$ is non-decreasing (divergence of the gradient field is positive), the length element is $1/v(t)dt$ and the area element is $S(t)/v(t)d\ell dt$. Note that the total area of the disk is finite and the gradient curves cannot meet (we use the non-negativity of the divergence again here). Thus $\int S(t)/v(t)dt<+\infty$ most of the time. But then, since $Sv$ is non-decreasing, we also have $\int 1/v(t)^2dt<+\infty$ while we need just $\int e^{-t}/v(t)<+\infty$ and Cauchy-Schwartz ends the story.<|endoftext|> -TITLE: Cyclic Permutations - but not what you think -QUESTION [12 upvotes]: This question is not about elements of $S_n$ that consist of a single $n$-cycle, though naturally it's related. -Instead, consider permutations modulo the action of $(123\ldots n)$. That is, we want ABCD to be the same as BCDA and CDAB and DABC. (It's optional whether this also is the same as DCBA, but for now let's say it's not.) I am primarily interested in the graph that these generate, sort of like the Cayley graph for $S_n$ with generators $(12),(23),\ldots (n-1 n),(n1)$, but with vertices and edges identified. (I don't think this is a Cayley graph of a quotient of $S_n$; I don't even think this set is identifiable with a group since that subgroup isn't normal, if I recall correctly.) -What are these things called, and are there references to them in the literature? (Say to their symmetry groups, rep. theory, or whatever else.) I can't imagine there aren't, but because 'cyclic permutations' nearly always means something else, it's frustrating to look for this. I found pages of MathSciNet references to those terms, and none were about this. Not surprisingly! But presumably combinatorics experts have studied them - not just counted them, though Polya enumeration immediately comes to mind. -Edit: For a concrete example, imagine people around a dinner table, where you don't care which chair you sit in, you just care what the arrangement is. Maybe it's been thought of that way before? -Edit: Well, I have to say that Tilman and Mark Sapir both have been very helpful, but I guess Tilman answered the actual question. -Very oddly, I can only find ONE paper on MathSciNet that actually deals with the object I am interested in directly - Woodall's "Cyclic-order graphs and Zarankiewicz's crossing-number conjecture" proves some basic facts. Nearly every reference to such things is about using cyclic orders without considering all of them (in graph theory or queueing theory), is using them to create ribbon graphs, or is about extending partial cyclic orders to complete cyclic orders. - -REPLY [2 votes]: Here is one interpretation of this set in terms of Hochschild homology. Let $A$ be the group algebra of $\mathbb{Z}/n\mathbb Z$, and let $B$ be the group algebra of $S_n$ (say over $\mathbb C$ for simplicity). There is a map $A \to B$ taking the generator to the $n$-cycle $(1 2 \cdots n)$, and this makes $B$ an $A$-bimodule. Then the set you define is a basis for the Hochschild homology $HH_0(A, B)$. (Of course, this isn't combinatorial, so I don't know if this is useful for your purposes.) -http://en.wikipedia.org/wiki/Hochschild_homology<|endoftext|> -TITLE: A remark in Swinnerton-Dyer's paper in Cassels-Frohlich -QUESTION [14 upvotes]: In Swinnerton-Dyer's charming paper "An application of computing to classfield theory", in Cassels-Frohlich, he discusses the genesis of the Birch/Swinnerton-Dyer conjecture and numerical tests of it for the curves $y^2=x^3-dx$. At the end of the paper, he speculates on higher-dimensional analogues of the conjecture, linking Chow groups to L-functions, which in hindsight were essentially correct (Beilinson-Bloch made them precise). Regarding these conjectures, he says that Bombieri and himself had found some evidence for them in the special case of cubic threefolds and the intersection of two quadric hypersurfaces. I know that in the case of cubic threefolds $X$, codimension-two cycles can be related to zero-cycles in the Albanese $A_X$ of the Fano surface of lines in $X$, which (I assume) reduces the conjecture in this case to BSD for $A_X$. But what can be done for the intersection of two quadrics? Does anyone know what he was talking about here? Is this in print in more detail somewhere? - -REPLY [4 votes]: Maybe you already know this! But Miles Reid and Ron Donagi have shown that the intermediate Jacobian of the intersection of two quadrics is the Jacobian of a hyperelliptic curve. See -Donagi's old paper where there is a beautiful generalization of the group law on an elliptic curve. Recent results on such questions all use Nori's general results or Bloch-Srinivas's paper on the diagonall see for example Nagel's paper or Voisin's paper.<|endoftext|> -TITLE: Bounds on number of conjugacy classes in terms of number of elements of a group ? -QUESTION [7 upvotes]: What are bounds on number of conjugacy classes in terms of number of elements of a group ? -(I allowed myself to edit the question in spirit of remarkable answers given to it by Gerry Myerson and Geoff Robinson. Below is original text of the question. (Alexander Chervov) ). - -It's about the first step to find an upper bound to the order of a finite group with h conjugacy classes (right or left) that depends only on h. (h a natural non nul integer). -I have some doubts about the rigor of my proof that I am sharing with you so that you can help me find a likely error or an omited step. -I have attached the scan of my proof to this post. -Many thanks - -REPLY [4 votes]: In the meantime, Barbara Baumeister, Attila Maróti and Hung P. Tong-Viet -have obtained a better lower bound on the number of conjugacy classes of -a group of given order. -- Namely, in 2015 they have proved that for every -$\epsilon > 0$ there exists a $\delta > 0$ such that every group of order -$n \geq 3$ has at least -$$ - \frac{\delta \log_2(n)}{\log_2(\log_2(n))^{3+\epsilon}} -$$ -conjugacy classes. -- See here. In this paper, the authors further -cite a conjecture by Edward A. Bertram which asserts that the number of -conjugacy classes of a finite group is bounded below by the logarithm -for base $3$ of its order. This is basically the best one can expect -to be true, as the Mathieu group ${\rm M}_{22}$ has order -$443520 > 3^{11}$ and only $12$ conjugacy classes. The authors also -show that the conjecture holds for groups with trivial solvable radical.<|endoftext|> -TITLE: Ax–Grothendieck and the Garden of Eden -QUESTION [27 upvotes]: It's an obvious consequence of the pigeonhole principle that any injective function over finite sets is bijective. But there are some similar results in different areas of mathematics that apply to less-finite settings. -In algebraic geometry, the Ax–Grothendieck theorem states (if I have it correctly) that any injection from an algebraic variety over an algebraically closed field to itself is bijective; the standard proof involves some sort of local-global principle together with the same fact over finite fields. -In the theory of cellular automata, the Garden of Eden theorem states that any injective cellular automaton (over an integer grid of some fixed finite dimension, say) is bijective; the standard proof involves again the same fact for finite sets of cells together with a limiting argument that shows that for large enough bounded regions of an unbounded grid, the boundary of the region has negligible effect compared to the interior. -Is there some way of viewing these three injective-bijective statements (or others) as instances of a single more general phenomenon? - -REPLY [3 votes]: In the theory of von Neumann algebras, there is a similar phenomenon. -Let $M$ be a type $II_1$-factor. That is, it's an infinite dimensional von Neumann algebra with center $\mathbb C$, and with an (everywhere defined) trace $tr:M\to \mathbb C$. -Then there is a complete invariant of $M$-modules called the von Neumann dimension. -This invariant takes values in $\mathbb R_{\ge 0}\cup \{\infty\}$ and can take any value in that set. It has the property that any isometric map $H_1 \to H_2$ between modules of the same dimension is actually a unitary isomorphism (except if the von Neumann dimension is $\infty$, in which case, that's not true). -In particular, if $H$ is an $M$-module of finite von Neumann dimension, then any isometry (not assumed to be surjective) is actually a unitary isomorphism.<|endoftext|> -TITLE: Polynomial roots and convexity -QUESTION [45 upvotes]: A couple of years ago, I came up with the following question, to which I have no answer to this day. I have asked a few people about this, most of my teachers and some friends, but no one had ever heard of the question before, and no one knew the answer. -I hope this is an original question, but seeing how natural it is, I doubt this is the first time someone has asked it. -First, some motivation. Take $P$ any nonzero complex polynomial. It is an easy and classical exercise to show that the roots of its derivative $P'$ lie in the convex hull of its own roots (I know this as the Gauss-Lucas property). To show this, you simply write $P = a \cdot \prod_{i=1}^{r}(X-\alpha_i)^{m_i}$ where the $\alpha_i~(i=1,\dots,r)$ are the different roots of $P$, and $m_i$ the corresponding multiplicities, and evaluate $\frac{P'}{P}=\sum_i \frac{m_i}{X-\alpha_i}$ on a root $\beta$ of $P'$ which is not also a root of $P$. You'll end up with an expression of $\beta$ as a convex combination of $\alpha_1,\dots,\alpha_r$. It is worth mentioning that all the convex coefficients are $>0$, so the new root cannot lie on the edge of the convex hull of $P$'s roots. -Now fix $P$ a certain nonzero complex polynomial, and consider $\Pi$, its primitive (antiderivative) that vanishes at $0:~\Pi(0)=0$ and $\Pi'=P$. For each complex $\omega$, write $\Pi_{\omega}=\Pi-\omega$, so that you get all the primitives of $P$. Also, define for any polynomial $Q$, $\mathrm{Conv}(Q)$, the convex hull of $Q$'s roots. -MAIN QUESTION: describe -$\mathrm{Hull}(P)=\bigcap_{\omega\in\mathbb{C}}\mathrm{Conv}(\Pi_{\omega})$. -By the property cited above, $\mathrm{Hull}(P)$ is a convex compact subset of the complex plane that contains $\mathrm{Conv}(P)$, but I strongly suspect that it is in general larger. -Here are some easy observations: - -replacing $P$ (resp. $\Pi$) by $\lambda P$ (resp. $\lambda \Pi$) will not change the result, and considering $P(aX+b)$ will change $\mathrm{Hull}(P)$ accordingly. Hence we can suppose both $P$ and $\Pi$ to be monic. The fact that $\Pi$ is no longer a primitive of $P$ is of no consequence. -the intersection defining $\mathrm{Hull}(P)$ can be taken for $\omega$ ranging in a compact subset of $\mathbb{C}$: as $|\omega| \rightarrow \infty$, the roots of $\Pi_{\omega}$ will tend to become close to the $(\deg (P)+1)$-th roots of $\omega$, so for large enough $\omega$, their convex hull will always contain, say, $\mathrm{Conv}(\Pi)$. -$\mathrm{Hull}(P)$ can be explicitly calculated in the following cases: -$P=X^n$, $P$ of degree $1$ or $2$. There are only 2 kinds of degree $2$ polynomials: two simple roots or a double root. Using $z\rightarrow az+b$, one only has to consider $P=X^2$ and $P=X(X-1)$. The first one yields {$0$}, which equals $\mathrm{Conv}(X^2)$, the second one gives $[0,1]=\mathrm{Conv}(X(X-1))$. - -Also, if $\Pi$ is a real polynomial of odd degree $n+1$ that has all its roots real and simple, say $\lambda_1 < \mu_1 < \lambda_2 < \dots < \mu_n < \lambda_{n+1}$, where I have also placed $P$'s roots $\mu_1, \dots, \mu_n$, and if you further assume that $\Pi(\mu_{2j}) \leq \Pi(\mu_n) \leq\Pi(\mu_1) \leq\Pi(\mu_{2j+1})$ for all suitable $j$ (a condition that is best understood with a picture), then $\mathrm{Hull}(P)=\mathrm{Conv}(P)=[\mu_1,\mu_n]$: just vary $\omega$ between $[\Pi(\mu_n), \Pi(\mu_1)]$; the resulting polynomial $\Pi_{\omega}$ is always split over the real numbers and you get -$$[\mu_1,\mu_n]=\mathrm{Conv}(P)\subset\mathrm{Hull}(P)\subset -\mathrm{Conv}(\Pi_{\Pi(\mu_1)})\cap -\mathrm{Conv}(\Pi_{\Pi(\mu_n)}) = \\= [\mu_1,\dots]\cap -[\dots,\mu_n]=[\mu_1,\mu_n]$$ - -The equation $\Pi_{\omega}(z)=\Pi(z)-\omega=0$ defines a Riemann surface, but I -don't see how that could be of any use. - -Computing $\mathrm{Hull}(P)$ for the next most simple -polynomial $P=X^3-1$ has proven a challenge, and I can only conjecture what -it might be. -Computing $\mathrm{Hull}(X^3-1)$ requires factorizing degree 4 -polynomials, so one naturally tries to look for good values of $\omega$, -the $\omega$ that allow for easy factorization of $\Pi_{\omega}=X^4-4X-\omega$---for instance, the $\omega$ that produces a double root. All that remains to be done -afterwards is to factor a quadratic polynomial. The problem is symmetric, -and you can focus on the case where 1 is the double root (i.e., $\omega=-3$). -Plugging in the result in the intersection, and rotating twice, you obtain the following superset of -$\mathrm{Hull}(X^3-1)$: a hexagon that is the intersection of three similar isoceles -triangles with their main vertex located on the three third roots of unity $1,j,j^2$ -QUESTION: is this hexagon equal to $\mathrm{Hull}(X^3-1)$? -Here's why I think this might be. -Consider the question of how the convex hulls of the roots of $\Pi_{\omega}$ vary as $\omega$ varies. -When $\omega_0$ is such that all roots of $\Pi_{\omega_0}$ are simple, then the inverse function theorem shows that the roots of $\Pi_{\omega}$ -with $\omega$ in a small neighborhood of $\omega_0$ vary holomorphically $\sim$ -linearly in $\omega-\omega_0$: $z(\omega)-z(\omega_0)\sim \omega-\omega_0$. If however $\omega_0$ is such that $\Pi_{\omega_0}$ -has a multiple root $z_0$ of multiplicity $m>1$, then a small variation of $\omega$ -about $\omega_0$ will split the multiple root $z_0$ into -$m$ distinct roots of $\Pi_{\omega}$ that will spread out roughly as -$z_0+c(\omega-\omega_0)^{\frac{1}{m}}$, where $c$ is some nonzero coefficient. This -means that for small variations, these roots will move at much higher velocities -than the simple roots, and they will constitute the major contribution to the variation of -$\mathrm{Conv}(\Pi_{\omega})$; also, they spread out evenly, and (at least if the -multiplicity is greater or equal to $3$) they will tend to increase the convex hull -around $z_0$. Thus it seems not too unreasonable to conjecture that the convex hull -$\mathrm{Conv}(\Pi_{\omega})$ has what one can only describe as -critical points at the $\omega_0$ that produce roots with -multiplicities. I'm fairly certain there is a sort of calculus on convex sets that -would allow one to make this statement precise, but I don't know see what it could be. -Back to $X^3-1$: explicit calculations suggest that up to second order, the double root $1$ of $X^4-4X+3-h$ for $|h|<<1$ splits in half nicely (here $\omega=-3+h$), and the convex hull will continue to contain the aforementioned hexagon. -QUESTION (Conjecture): is it true that -$\mathrm{Hull}(P)=\bigcap_{\omega\in\mathrm{MR}}\mathrm{Conv}(\Pi_{\omega})$, where -$\mathrm{MR}$ is the set of all $\omega_0$ such that -$\Pi_{\omega_0}$ has a multiple root, i.e., the set of all $\Pi(\alpha_i)$ where the -$\alpha_i$ are the roots of $P$? -All previous examples of calculations agree with this, and I have tried as best I can to justify this guess heuristically. -Are you aware of a solution? Is this a classical problem? Is anybody brave enough to -make a computer program that would compute some intersections of convex hulls -obtained from the roots to see if my conjecture is valid? - -REPLY [12 votes]: This problem has been considered before: -The notes to chapter 4 of -Rahman/Schmeißer: Analytic Theory of Polynomials, Oxford University Press, 2002 -mention this problem, state that conv(P) is a proper subset of hull(P) in general, and give two references: -1) J. L. Walsh: The location of Critical Points of Analytic and Harmonic Functions, -AMS Colloquium Publications Volume 34, 1950, p. 72 -2) E. Chamberlin and J. Wolfe: Note on a converse of Lucas's theorem, Proceedings of the American Mathematical Society 5, 1954, pp. 203 - 205 -I had a look at both of them, the paper of Chamberlin and Wolfe can be obtained online via the AMS for free. -The relevant paragraph in Walsh's book is 3.5.1 (starts on page 71). I state the 4 theorems given there for convenience: -1) conv(P) = hull(P) if P is of degree 1 or 2 -2) conv(P) = hull(P) if P is of degree 3 and its zeros are collinear -3) There exists a P of degree 3, such that conv(P) is a proper subset of hull(P); example $P(z) = z^3 + 1$. -4) There exists a P of degree 4 with real zeros, such that conv(P) is a proper subset of hull(P); example $P(z) = (z^2 - 1)^2$. -Chamberlin and Wolfe prove, that -1) Any point on the boundary of hull(P) is on the boundary of conv($\Pi_\omega$) for some $\omega$. -2) If a side of any conv($\Pi_\omega$) contains a point of hull(P) and only two zeros of $\Pi_\omega$ , counting multiplicities, then P is of degree 1. (Here a side is equal to conv($\Pi_\omega$), if the zeros of $\Pi_\omega$ are collinear.) -3) Vertices of conv(P) need not lie on the boundary of hull(P), example - $\Pi(z) = z^2 (z + 1)(z^2 - 2az + 1 + a^2)$, where a is positive and sufficiently small. -4) Even if P is cubic, hull(P) need not be determined by its primitives with multiple zeros, example P(z) = $4z^3 + 9/2 z^2 + 2z + 3/2$. -While I have corrected what I considered a minor typo in the example $\Pi(z) = z^2 (z + 1)(z^2 - 2az + 1 + a^2)$, I did no thorough proofreading. -Walsh gives no special references to further work in section 3.5.1 and the only reference provided by Chamberlin and Wolfe is to Walsh's book cited above.<|endoftext|> -TITLE: Arnoldi method to compute the dominant eigenvector -QUESTION [6 upvotes]: Hi, everyone! -I have a problem of computing the dominant eigenvector. When I want to approximate the dominant eigenvector of a large sparse matrix via the famous Arnoldi method, I am wondering how to choose the reduced order $k$ (i.e., the number of Arnoldi iterations). I know that the approximation accuracy is relevant to the choice of $k$. As $k$ approaches to $n$ (the dimension of the full space) , the accuracy is very high. Now the problem is that given an accuracy $e$, can we find an approperate $k$ (depending on $e$) such that the difference between my approximate dominant eigenvector via Krylov subspace and the exact one is less than $e$? That's to say, I need to find an a-prior error bound, rather than an a-posterior error bound (however, I only found some a-posterior error bound results in some papers). Could you give me some suggestions? Thanks! - -REPLY [5 votes]: This is a very preliminary answer; my knowledge of these things is rather dated, but I got curious and found something of potential interest, so I thought I might as well share it. -There is a recent paper - -Bellalij, M.; Saad, Y.; Sadok, H. - Further analysis of the Arnoldi process for eigenvalue problems. - SIAM J. Numer. Anal. 48 (2010), no. 2, 393–407. - -(In case you do not have access to the journal there is a technical report that is similar to be found here http://www-users.cs.umn.edu/~saad/reports.html (year 2007). However, the discussion below referes to the article.) -At the end of Section 4.1 (where they analyse the algorithm under some assumptions) one can find the following: - -There does not seem to exist in the nonnormal case any formal theoretical results which - establish the convergence of an approximate pair obtained by the Arnoldi process - toward an exact eigenpair. Establishing the link with classical iterative methods - helps understand the nature of the convergence in some instances where the error - may be quite large early in the process. - -And at the beginning of the conclusions: - -The convergence analysis of the Arnoldi process for computing eigenvalues and eigenvectors is difficult, and results in the nonnormal case are bound to be limited in scope. - -So, it seems to me, that the a priori bounds that you seek in general might not exists/be known. In case you know something on the matrix (in particular, symmetric/hermitian) then the situaton seems to be better. -Of course, I quoted the 'negative' passages of the paper only, and it mainly contains 'positive' results (so, if you are not yet aware of it, might well be worth looking at), but as far as I could see not of the strength you seem to be hoping for.<|endoftext|> -TITLE: Non-degenerate alternating bilinear form on a finite abelian group -QUESTION [15 upvotes]: I asked this question on math.stackexchange yesterday, but nobody has helped so far, and only 44 people have seen it! So I hope people do not mind me asking it here... -Let $A$ be a finite abelian group, and let -$ \psi : A \times A \to \mathbb{Q}/\mathbb{Z} $ -be an alternating, non-degenerate bilinear form on $A$. Maybe I should say what I mean by these words; bilinear means it is linear in each argument separately; alternating means that $\psi(a,a) = 0$ for all $a$; non-degenerate means that, if $\psi(a,b) = 0$ for all $b$, then $a$ must be $0$. - -Why must $A$ have square cardinality? - -I believe it will follow from the following theorem in Linear algebra: -Theorem. Let $V$ be a finite dimensional vector space over a field $K$ that has an alternating, non-degenerate bilinear form on it (from $V \times V \to K$). Then dim $V$ is even. -My idea was to proceed as follows: If the size of $A$ is not square, then for some prime $p$, $A(p)$ is not square, where $A(p)$ means the $p$-primary part of $A$. The original $\psi$ induces a map on $A(p)$ that is non-degenerate, alternating and bilinear. I then wanted to say that $A(p)$ is an $\mathbb{F}_p$-vector space, and then applying the theorem I am done, but this is not true, e.g, $\mathbb{Z}/25\mathbb{Z}$ is not an $\mathbb{F}_5$-vector space. -Any pointers anyone? - -REPLY [27 votes]: Actually, one can show the following stronger result: - -Proposition. Assume that a finite abelian group $A$ admits a non-degenarate, bilinear alternating form $\psi$. Then $A$ has a lagrangian decomposition, i.e. there exists a subgroup $G$, isotropic for $\psi$, such that $$A \cong G \times \widehat{G},$$ where $\widehat{G}$ denotes as usual the group of characters of $G$. In particular, $|A|=|G|^2$. - -Therefore, the elements of $A$ can be written as $(x, \chi)$, with $x \in G$ and $\chi \in \widehat{G}$. Moreover, in such a presentation the form $\psi$ take the following form: $$\psi((x, \, \chi), \, (y, \, \eta))=\chi(y)\eta(x)^{-1}.$$ -An easy proof, by induction on the order of the group, can be found in Lemma 5.2 of A. Davydov, Twisted automorphisms of group algebras, arXiv:0708.2758 -Remark. It is interesting to notice the analogy with symplectic vector spaces. In fact, any symplectic vector space $(V, \omega)$ can be written as $V = W \oplus W^{*}$, where $W$ is a lagrangian (=isotropic of maximal dimension) subspace for $\omega$. In particular, $\dim V = 2 \dim W$. Moreover, with respect to this decomposition, $\omega$ has the following form: $$\omega(x \oplus \chi, \, y \oplus \eta) = \chi(y) - \eta(x).$$ -In the case of finite abelian groups the "dual role" is played by the group of characters, as usual.<|endoftext|> -TITLE: Degeneration of the Hodge spectral sequence -QUESTION [27 upvotes]: Let $f\colon X \to S$ be a smooth proper morphism of schemes. If $S$ is of characteristic zero (i.e., $S$ is a $\mathbb Q$-scheme), then Deligne has shown: - -$R^af_*\Omega^b_{X/S}$ is locally free for all $a,b \geq 0$. -The Hodge-De Rham spectral sequence -$E^{ab}_1 = R^af_*\Omega^b(X/S) \Rightarrow H_{\rm DR}^{a+b}(X/S)$ -degenerates in $E_1$. - -This is known to fail in positive characteristic. Mumford gave examples of smooth projective surfaces over algebraically closed fields. Nevertheless there are several interesting cases of schemes $X \to S$ in characteristic $p > 0$ where I know this to be true: -a. $X$ is an abelian scheme, a relative curve, a global complete intersection in projective space, or a K3-surface over $S$. -b. $X$ is a smooth projective toric variety over a field. -c. There is also a criterion of Deligne and Illusie which in particular shows 1. und 2. to hold if $\dim(X/S) < p$ and $X$ can be lifted to $W_2(S)$. -Question: What are other examples in positive characteristic, where 1. and 2. hold? -There is also a variant of the result of Deligne for logarithmic schemes. In particular I would be also interested for examples where the logarithmic analogue of 1. and 2. hold. -ADDITION: I am taking the risk to name two examples of smooth projective schemes over a field, where I would not be too surprised if (1. and) 2. hold, but where I know of no results: -d. $X$ is Calabi-Yau (i.e., its canonical bundle is trivial). -Edit: As Torsten Ekedahl pointed out below this definition of "Calabi-Yau" is not the "right" one (not even in char. $> 2$ as I wrote in an earlier edit) and does not imply in general that the Hodge-De Rham spectral sequence degenerates. -e. $X$ is $G$-spherical for a reductive group $G$ (i.e., $X$ carries a $G$-action such that there exists a dense $B$-orbit, where $B$ is a Borel subgroup of $G$). -Edit: Again one might to have exclude some small primes depending on the Dynkin type of $G$. - -REPLY [11 votes]: [I misunderstood Torsten Ekedahl's earlier comment. I'm reverting the lemma -to its original form which was a bit stronger.] -Since the question seemed to resonate with me, I've been thinking about this on and off (but mostly off) for a couple of days now. Here's what I've come up with. -What seems to make the example of complete intersections work is the fact that -the Hodge numbers can be computed by formulas independent of the characteristic -(a standard generating function can be found in SGA7, exp XI). -Here's the underlying principle. - -Lemma. - Suppose that $D$ is the spectrum of a mixed characteristic DVR with closed point $0$ and - generic point $\eta$. Let $\mathcal{X}\to D$ be a smooth projective family such that - $$\dim H^q(\mathcal{X}_0,\Omega_{\mathcal X_0}^p)= \dim H^q(\mathcal{X}_\eta,\Omega_{\mathcal X_\eta}^p)$$ - for all $p,q$. Then Hodge to De Rham degenerates on the closed fibre. - -Proof. It degenerates on $\mathcal{X_\eta}$ by Hodge theory. This plus semicontinuity -implies -$$\dim E_1(\mathcal{X}_0)\ge \dim E_\infty(\mathcal{X}_0)\ge \dim E_\infty(\mathcal{X}_\eta) -=\dim E_1(\mathcal{X}_\eta)=\dim E_1(\mathcal{X}_0)$$ -This can be used to check degeneration for the following cases: -Ex 1. Complete intersections in projective spaces as noted already. -Ex 2. Products of smooth projective curves and complete intersections. Use -Kuenneth and the fact that curves lift into characteristic $0$ (the obstruction -lies in $H^2$ of the tangent sheaf; or see Oort, Compositio 1971). -Ex 3. Certain cyclic branched covers of projective space, and more generally -certain hypersurfaces in weighted projective space. I'm too lazy to say -what "certain" means exactly. But a careful reading of Dolgachev's notes -on weighted projective spaces ought to yield something more precise.<|endoftext|> -TITLE: "Résumé de cours" by Jacques Tits -QUESTION [7 upvotes]: I have been reading a number of papers by Jacques Tits (mostly written in the second half of 1980s) and in them he frequently refers to following publications of his: - -Résumé de cours, Annuaire du collège de France, 81e année (1980-1981), 75-86. -Résumé de cours, Annuaire du collège de France, 82e année (1981-1982), 91-105. - -Unfortunately I have not been able to locate these two papers (internet, library, faculty). Any help with finding these two would be immensely appreciated. - -REPLY [5 votes]: The "résumés de cours" of Tits are now published by the SMF, see -here.<|endoftext|> -TITLE: Does de Branges's theorem extend to several variables? -QUESTION [12 upvotes]: Consider injective homolomorphic functions $f:\mathbb D\to \mathbb C$ on the unit disk $|z|\leq 1$, normalized by the conditions $f(0)=0$ and $f'(0)=1$. -Thus for $|z|\leq 1$ we have $ f(z)=\sum_{k=0}^{\infty} a_k z^k $ with $a_0=0$ and $a_1=1$. -Ludwig Bieberbach conjectured in 1916 and Louis de Branges proved in 1984 that for all $k \in \mathbb N$ the inequality $|a_k|\leq k$ holds. -Question Is there an analogue to de Branges's result for functions defined on a suitable domain (polydisk, ball,...?) in $\mathbb C^n$ for $n\geq 2$ ? - -REPLY [14 votes]: Such a result would have to be quite different in several variables, because the holomorphic automorphism group of $\mathbb{C}^n$ is very big when $n \geq 2$. For injectivity, we need to look at equidimensional mappings $F$ from the domain (whatever it may be), and into $\mathbb{C}^n$ say. For simplicity, choose $n = 2$, which already contains the main features of the general case. Any mapping $\psi$ of the form $(z,w) \mapsto (z,w + h(z))$ (a "shear") with $h$ an arbitrary entire function is holomorphic everywhere on $\mathbb{C}^2$ and injective. Thus -$\psi \circ F$ is also an injective holomorphic mapping from the domain into $\mathbb{C}^2$. Assuming the domain contains the origin and that $F$ is normalized (taking the origin to the origin and having the identity as derivative at the origin), we can ensure that $\psi \circ F$ is also normalized by choosing the entire function $h$ to have a double zero at the origin. The freedom of choosing $h$ implies that the power series coefficients of the mappings in the normalized family of injective holomorphic mappings have no universal bounds over the family; unlike in the one-dimensional case, the normalized family is not compact. -The key difficulty is this: In one variable the automorphism group of the target $\mathbb{C}^1$ consists of mappings of the form $z \mapsto az + b$, and we can mod out this automorphism group simply by fixing $f(0)$ and $f^{\prime}(0)$, and then it turns out that injectivity implies compactness. When $n \geq 2$ the automorphism group is enormous (a dense subgroup of it was determined explicitly by E. Andersén and L. Lempert, so it is known in a sense), thus a linear normalization is very inadequate, and we should normalize by the whole automorphism group. But it is not clear what it means in practice to mod out by the automorphism group, and whether injectivity would imply compactness after moding out when $n \geq 2$. If we obtain a compact family, there will exist sharp bounds on all coefficients, of course, though we might not be able to establish them. -A precise question (though in an unexplicit form) would be: When can we write the family of all injective holomorphic mappings from a domain $\Omega$ into $\mathbb{C}^n$ by composing $\mathrm{Aut}(\mathbb{C}^n)$ with a compact family? Trivially we can do it when -$\Omega = \mathbb{C}^n$ by choosing the compact family to consist of the identity map alone. -If one is willing to impose strong geometric conditions (e. g. starlikeness) on the injective mappings, one can get compactness. See the book Geometric function theory in one and several variables by I. Graham and G. Kohr for example. -ADDED: Having thought about this a little more, I believe it unlikely that for $n \geq 2$ compactness can hold for $F : \mathbb{B}_n \rightarrow \mathbb{C}^n$ even after moding out by the automorphism group of $\mathbb{C}^n$. For generic domains have only the identity automorphism, and so we could replace $\mathbb{C}^n$ by a slightly smaller domain $D$ with only the identity automorphism and look at injective holomorphic mappings $F : \mathbb{B}_n \rightarrow D$. Since $D$ is nearly as "roomy" as $\mathbb{C}^n$ the family should be noncompact, but obviously cannot be made compact by moding out by automorphisms. So moding out by automorphisms seems to be an "accidental" device, so to speak, and unlikely to work when $n \geq 2$. I would support this conclusion as follows: -One way to see that the family of injective holomorphic functions $f : \mathbb{D} \rightarrow \mathbb{C}$ with $f(0) = 0$ and $f'(0) = 1$ is compact is to apply the Schwarz Lemma to the inverse $f^{-1}$ to see that $f$ omits a point on the circle $|w| = 1$. Since the omitted set of $f$ on the unit sphere is a continuum containing $\infty$, $f$ also omits a point on the circle $|w| = 2$. A well known theorem states that if a family of holomorphic functions omits three points on the Riemann sphere, then it is precompact ("normal" in complex analysis parlance). Here we have three points, but two are "movable" (on the circles) and the third is fixed (at infinity) and a routine extension of the theorem mentioned shows that the family is still precompact, because the three omitted points stay uniformly away from each other. Since the normalized family of injective holomorphic functions $f$ is closed by a theorem of Hurwitz, the family is compact. -Now let us see what this yields for injective holomorphic mappings $F : \mathbb{B}_n \rightarrow D$ with $F(0) = 0$ and $F'(0) = I$ (we assume $0 \in D$) for $n \geq 2$. We can still apply the Schwarz Lemma (in several variables) to the inverse $F^{-1}$ to conclude that $F$ omits a point on the sphere $|w| = 1$, so that the omitted set of $F$ is an unbounded continuum with a point on that sphere. But this information is hardly strong enough to conclude that the family is precompact. The higher-dimensional analogue of the theorem on three omitted points is that if we remove $2n+1$ hyperplanes in general position from $\mathbb{C}^n$ then any family of holomorphic mappings into what remains is precompact. So it seems that we have removed much too little when $n \geq 2$.<|endoftext|> -TITLE: Polynomial with Galois Group $D_{2n}$ -QUESTION [8 upvotes]: How does one construct a polynomial with Galois Group $D_{2n}$? A general method would be preferable or if that's impractical then an example of it being done for any n would be appreciated. -Thanks! - -REPLY [8 votes]: One general method proceeds by making use of invariant polynomials. Let $G$ be a candidate Galois group for an irreducible polynomial of degree $n$ over a field $F$ so that in particular $G$ has a transitive permutation action on $n$ objects $r_{i}$ which we identify with the roots of the polynomial. Basic Galois theory then tells us that any polynomial in the $r_{i}$ which is invariant under the action of $G$ must then lie in $F$. Thus $G$ stabilizes the invariant polynomial ring $F[r_{1}, \cdots, r_{n}]^{G}$. -Now, since $G$ is a subgroup of $S_{n}$ the invariant ring includes the elementary symmetric polynomials $\sigma_{i}$ defined as -$ -\sigma_{1}=r_{1}+r_{2}+\cdots +r_{n}, -$ -$ -\sigma_{2}=r_{1}r_{2}+r_{1}r_{3}+\cdots +r_{n-1}r_{n}, -$ -$\cdots = \cdots ,$ -$\sigma_{n} = r_{1}r_{2}\cdots r_{n}. $ -On the other hand in the splitting field, a polynomial with roots $r_{i}$ can be completely factored as -$\prod_{i}(z-r_{i}) = z^{n}-\sigma_{1}z^{n-1}+\sigma_{2}z^{n-2}+\cdots + (-1)^{n}\sigma_{n}.$ -To construct a polynomial with Galois group $G$ we can then simply choose the $\sigma_{i}$ to be consistent with whatever relations occur in the invariant ring and write a polynomial as above. In general this leads to a polynomial whose Galois group is a subgroup of $G$, but provided we choose the invariants sufficiently generically the Galois group will in fact be $G$ itself. In general this method works well for groups of small order where the invariant rings are managable. -Now let us apply this to produce an example of a degree four polynomial over $\mathbb{Q}$ with Galois group $D_{4}$. Up to a shift in the indeterminate we may assume that this polynomial takes the form -$ -p(z)=z^{4}+\sigma_{2}z^{2}-\sigma_{3}z+\sigma_{4}. -$ -In other words without loss of generality we may assume that the sum of the roots of $p(z)$ vanishes. -An elementary problem in the theory of finite group representations shows that the invariant ring of $D_{4}$ acting as permutation on four objects $r_{i}$ subject to the constraint that $\sum_{i}r_{i}=0$ is generated by four objects $\alpha, \beta, \chi, \lambda$ subject to the single relation $\alpha \lambda =\chi^{2}$. Thus the relevant invariant polynomial ring is simply -$ -\mathbb{Q}[r_{1}, r_{2}, r_{3}, -r_{1}-r_{2}-r_{3}]^{D_{4}}\cong \mathbb{Q}[\alpha, \beta, \chi, \lambda]/\langle \alpha \lambda=\chi^{2}\rangle. -$ -Then the symmetric polynomials are expressed in terms of the generators of the invariant ring as -$ -\sigma_{2}=-\frac{1}{8}(\beta+\alpha), -$ -$\sigma_{3}= \frac{\chi}{16},$ -$\sigma_{4}= \frac{1}{256}((\alpha-\beta)^{2}-\lambda).$ -Thus any polynomial with Galois group $D_{4}$ can be written by choosing arbitrary $\alpha, \beta, \chi, \lambda$ subject to the single constraint in the invariant ring and plugging into the above. For example, one solution with integer coefficients is given by -$ -p(z)=z^{4}+z^{2}+2z+1. -$<|endoftext|> -TITLE: Advances and difficulties in effective version of Thue-Roth-Siegel Theorem -QUESTION [16 upvotes]: A fundamental result in Diophantine approximation, which was largely responsible for Klaus Roth being awarded the Fields Medal in 1958, is the following simple-to-state result: -If $\alpha$ is a real algebraic number and $\epsilon > 0$, then there exists only finitely many rational numbers $p/q$ with $q > 0$ and $(p,q) = 1$ such that -$$\displaystyle \left \lvert \alpha - \frac{p}{q} \right \rvert < \frac{1}{q^{2 + \epsilon}}$$ -This result is famous for its vast improvement over previous results by Thue, Siegel, and Dyson and its ingenious proof, but is also notorious for being non-effective. That is, the result nor its (original) proof provides any insight as to how big the solutions (in $q$) can be, if any exists at all, or how many solutions there might be for a given $\alpha$ and $\epsilon$. -I have come to understand that to date no significant improvement over Roth's original proof has been made (according to my supervisor), and that the result is still non-effective. However, I am not so sure why it is so hard to make this result effective. Can anyone point to some serious attempts at making this result effective, or give a pithy explanation as to why it is so difficult? - -REPLY [2 votes]: There have been some effective results on Roth's theorem -See this work by Luckhardt from 1989: -H. Luckhardt: Herbrand-Analysen zweier Beweise des Satzes von Roth: Polynomiale Anzahlschranken, - The Journal of Symbolic Logic, vol. 54 (1989), pp. 234–263. -https://www.cambridge.org/core/journals/journal-of-symbolic-logic/article/herbrandanalysen-zweier-beweise-des-satzes-von-roth-polynomiale-anzahlschranken/2C2B71C11AC25B06B0F1FD0E7F08D936<|endoftext|> -TITLE: Slices of infinity sheaves -QUESTION [5 upvotes]: I know from classical category theory that if $C$ is a small category and $X$ is a presheaf, that there is a canonical equivalence $$Set^{C^{op}}/X \simeq Set^{\left(C/X\right)^{op}},$$ where $C/X$ is the category of elements of $X$ (i.e. the Grothendieck construction of $X$). Moreover, if $C$ carries a Grothendieck topology, this statement is true for sheaves, where $C/X$ inherits a canonical topology from $C$. -Can I make a similar statement if I go to infinity sheaves, and if so, does anyone have a reference? Thanks. If someone knows a way of proving this model theoretically, that would also be nice. - -REPLY [2 votes]: This is Corollary 5.1.6.12 in HTT. Somehow I overlooked this.<|endoftext|> -TITLE: Examples of naturally occurring Quadratic forms or quadrics. -QUESTION [9 upvotes]: I am always fascinated when a quadratic form (or a quadric) arises naturally. I have -some elementary examples, but most of all, I want to learn more examples. I hope this question isn't considered too vague for MO. Most forms I list are really -elementary, and all are finite dimensional. -I got most of the following examples from M.Berger, Geometry I & II, and from the truly beautiful book "Eléments de géométrie : actions de groupes" by french author Rached Meinmné. -$(0)$ the discriminant on the affine space of unitary degre 2 polynomials -$(i)$ the determinant on endomorphisms of a 2 dimensional vector space, and -$\mathrm{Tr}^2-4\mathrm{det}$ -$(ii)$ the radical on the space of quadratic forms on a 2 dimensional vector space, -and the isotrope cone (not sure about the name, degenerate cone?). -$(iii)$ the family of hermitian forms (built from the Wronskian) on the solution -space of the discrete Schroedinger equation that allow one to show the existence of -right and left side $L^2$ solutions, and the Weyl m function. -$(iv)$ If $\Delta$ is any $2$ dimensional complex vector space, then -$\mathrm{Herm}(\Delta)$, the real vector space of hermitian forms on $\Delta$, -carries a natural quadratic form obtained by constructing an essentially unique -morphism $\rho$ from $\mathrm{Herm}(\Delta)$ to -$\mathrm{Hom}(\Delta\oplus\overline{\Delta})$ such that for all -$h\in\mathrm{Herm}(\Delta),~\rho(h)^2$ is proportional to $\mathrm{Id}$, the proportionality defining the quadratic form. Here, $\rho$ only depends on a choice of -a nonzero element $\omega\in\Lambda^2\Delta^*$. -$(v)$ If $V$ is a 4 dimensional vector space, then $\Lambda^2 V$ carries the natural -quadric $Q(v)=v\wedge v$ where $\Lambda^4 V$ is identified with the underlying -field, which vanishes exactly when $v$ comes from the canonical map -$\mathrm{Gr}(2,V)\rightarrow P\Lambda^2V$. -I remember reading about one on the space of circles, but I forgot the details. What other examples of natural quadratic forms are there? - -REPLY [2 votes]: Binary quadratic forms arise in nature as norm forms for a quadratic field. This point of view has various consequences in number theory. - -For a fixed negative discriminant (the definite case), Gauss discovered that the quadratic forms (or their $\mathrm{SL}_2(\mathbb{Z})$ equivalence classes) can be composed. This led him to the phenomenon of the ideal class group before ideals were invented by Kummer and Dedekind. Besides in the ideal class group for more general number fields, Gauss's composition law has found an extension in Bhargava's higher composition laws. These are based on the representation theory of arithmetic groups ($\mathrm{SL}_2(\mathbb{Z})$ and its generalizations), in which regard they are natural structures in themselves. They have striking applications to old problems regarding mean asymptotics of Selmer ranks of elliptic curves, the $3$-parts of class groups of quadratic fields, etc. -The Epstein zeta function takes the shape $\zeta_Q(s) := \sum_{\mathbf{n} \neq \mathbf{0}} Q(\mathbf{n})^{-s}$, for a given signature $(d,0)$ quadratic form $Q$. It has all the right analytical properties (meromorphic with simple pole at $s = 1$ and a functional equation relating $s \leftrightarrow 1-s$), allowing to decompose the zeta function of an imaginary quadratic field over a set of representatives $Q$ for the class group. This has consequences for the arithmetic of these fields, beautifully developed in Siegel's Lectures on Advanced Analytic Number Theory (Tata Institute lecture series, 1961). -For $d = 2$, $\zeta_Q(s)$ is in effect an Eisenstein series ($|mz+n|^2$ being a binary quadratic form in $m,n$), which is a natural structure all over mathematics, being a continuum of modular forms in the spectral resolution of the hyperbolic Laplacian. Siegel apparently had much interest in the conceptual role played in number theory by the higher rank quadratic forms and their Epstein zeta function. Much of his work was put on representation theoretic footing in Weil's 1964 paper Sur certains groupes d'operateurs unitaires. Michael Berg's book, The Fourier-Analytic Proof of Quadratic Reciprocity, is a terrific introduction to these ideas.<|endoftext|> -TITLE: semisimplicity of braid reps? -QUESTION [5 upvotes]: Here's something I really feel I should know, but do not: -Let $q$ be some sufficiently nice complex number (just pretend we're working over $\mathbb Q(q)$, for example), and $V$ some simple representation of $U_q(\mathfrak g)$. Then you have the usual representation of the braid group $B_n$ on $V^{\otimes n}$. The question is: is this representation semisimple? -I haven't been able to find a direct reference to the problem, but it really sounds like it should be known so I thought I'd check here. - -REPLY [3 votes]: A complete analysis of this is given in the paper by Orellana-Ram. Actually, they consider the action of the affine braid group on $M\otimes V^{\otimes n}$, but you can recover your case by taking $M$ to be the trivial module. I believe that this is is a semisimple representation (assuming you mean $V$ to be finite dimensional), but in any case it is all spelled out in the paper above.<|endoftext|> -TITLE: Generic filter over $V$ -QUESTION [5 upvotes]: I re-read Jechs chapter about forcing, and got a question. There he characterizes a (what he calls) modern way to make the forcing argument legitimate which (I think) goes like this: -It is pointed out there that, in order to establish the consistency of a statement $\varphi$ relative to ZFC, it is sufficient to exhibit a complete Boolean algebra $B$ such that the Boolean value of $\varphi$ in the Boolean-valued model $V^B$ is not zero, i.e. $|| \varphi|| = p \ne 0$. -This fact gives us an alternative approach to forcing, which avoids the assumption of a transitive, countable model $M$ of ZFC, in order to construct the generic extension $M[G]$. -Nevertheless one hardly sees a proof where the Boolean value of an interesting $\varphi$ is really evaluated because it is more easy to pretend that for the p.o. $P \quad$there exists a generic $G$ over $V$, then build $V[G]$ and show that $V[G] \models \varphi$, which is equivalent to the existence of a $p \in G$ such that $p \Vdash \varphi$, which implies that $||\varphi|| \ne 0$. -My problem with this approach is that a generic $G$ over $V$ cannot exist whenever $P$ satisfies the following property: For every $p \in P$ there exist $q \le p, r \le p$ such that $q$ and $r$ are incompatible. -(To see this it suffices to realize that for every filter $F$ on $P$ the set {$ p \in P \quad : p \notin F$} is dense, thus a generic $G$ leads to a contradiction). The existence of a generic $G$ does not change the Boolean value of $\varphi$, so I think that the argumentation in the break above remains valid although $G$ cannot exist. -However it seems strange to me that one assumes the existence of thing that must not exist, to faciliate calculation. One can say that this resembles the complex numbers but to me adding a new element to the reals is way weaker than adding an element that actually must not exist to the universe. -So my question is: is the just described strategy of showing the relative consistency of a statement really valid? - -REPLY [12 votes]: The existence of a generic $G$ over $V$ is indeed impossible in $V$ (for nontrivial forcing notions), but it has truth value 1 in appropriate Boolean-valued models. In more detail: If $P$ is a partially ordered set (to be used as a notion of forcing) and $B$ is the complete Boolean algebra of regular open subsets of $P$, then the following paragraph is true (i.e., has truth value 1) in the Boolean-valued model $V^B$: -There is a transitive class $\check V$ that contains all the ordinals and satisfies all the sentences true in the original ground model $V$ (so it can serve as a "copy" of $V$ in $V^B$). There is a subset $G$ of $P$ generic over $\check V$. Every set is the value of some forcing name in $\check V$ with respect to $G$. -Together, these say that, with truth value 1, the universe (of $V^B$) is a $P$-generic extension $\check V[G]$ of the ground model. So when people pretend to move to a (nonexistent) $P$-generic extension of $V$, a correct interpretation of this is that they move to $V^B$ and that whatever they assert about $V[G]$ is really asserted to have truth value 1 in this Boolean-valued model. -(Technicalities: "All the sentences true in the ground model $V$" should really be specified by a theory $T$, which should include a name (or at least a definition) for $P$ so that one can talk, in $V^B$, about the $P$ that lives in $\check V$.)<|endoftext|> -TITLE: Complex Hypersurface in Complex Projective Space -QUESTION [14 upvotes]: Apparently 2 smooth complex hypersurface in complex projective space that have the same degree -are diffeomorphic. Does anyone know where the proof of this can be found ? Is there a counter example for symplectic manifolds? - -REPLY [5 votes]: There is a proof of this result in Gompf and Stipsicz's book "4-Manifolds and Kirby Calculus": See Claim 1.3.11.<|endoftext|> -TITLE: What is the "Physically Consistent" proper subset of arithmetic? -QUESTION [10 upvotes]: Suppose 1st-order arithmetic is inconsistent along with Voevodsky http://video.ias.edu/voevodsky-80th. -It nevertheless remains true that when you have 2 apples and 2 apples, you have 4 apples. Preforming an experiment gives you the result of an experiment, which cannot be inconsistent. So there is a subset of arithmetic that is "necessarily" consistent, given the notion (maps) that arithmetic models reality. The question is, what is this "physically consistent" proper subset of arithmetic? -The second question is, what happens if the physical theory is quantum field theory, where quanta loose their individual identity or "primitive thisness"? - -REPLY [7 votes]: Presburger arithmetic which is the first order theory of natural numbers with addition has been proven to be consistent by Mojżesz Presburger. My reference for this is the wikipedia article on Presburger arithmetic.<|endoftext|> -TITLE: Is every field extension of an ultrafield an ultrafield? -QUESTION [10 upvotes]: Let $K=\lim(K_{i})$ be an ultrafield (over a non-principal ultrafilter), and let $K\hookrightarrow K'$ be a field extension of $K$. -When the field $K'$ is finite over $K$ it is also an ultrafield by Łoś's theorem. What can be said when the trascendence degree of $K'$ over $K$ is infinite? - -REPLY [5 votes]: I have mentioned in a comment that the accepted answer is not correct, although the argument is correct when the index set is countable. -Here's a result with no algebraic closedness assumption, which actually shows that the algebraic structure of the multiplicative group is enough to set up the argument, and also purports to clarify what we should allow on the index set, or even on the ultrafilter. I'm starting with the countable case. - -Proposition. Let $K$ be a field. Then the field $K(t)$ is not isomorphic to any nonprincipal countable ultraproduct of fields. Actually, the multiplicative group $K(t)^*$ is not isomorphic to any nonprincipal countable ultraproduct of groups. - -Since $K(t)^*\simeq K^*\times\mathbf{Z}^{(J)}$, where $\mathbf{Z}^{(J)}$ is the free abelian group on the nonempty set $J$ of monic irreducible polynomials, we have $\mathrm{Hom}_{\mathrm{Group}}(K(t)^*,\mathbf{Z})\neq\{0\}$. Hence the proposition follows from the following lemma. -Given a family $(G_n)$ of groups, I denote by $\prod^\star_nG_n$ the near product, i.e., the quotient of the product $\prod_n G_n$ by the finitely supported (aka restricted, aka finitely supported) product $\bigoplus_n G_n$. - -Lemma. Let $(G_n)$ be a sequence of abelian groups. Then $\mathrm{Hom}(\prod^\star_n G_n,\mathbf{Z})=\{0\}$. In particular, for every nonprincipal ultrafilter $\sigma$ on $\mathbf{N}$, we have $\mathrm{Hom}(\prod^\sigma_n G_n,\mathbf{Z})=\{0\}$, where $\prod^\sigma_n G_n$ is the ultraproduct with respect to $\sigma$. - -Proof: This is the classical Specker proof for $G_n=\mathbf{Z}$. The proof is an immediate adaptation. Consider a homomorphism $\prod^\star_nG_n\to\mathbf{Z}$, vanishing on $\bigoplus_nG_n$. For each $n$, choose a Bézout relation $2^na_n+3^nb_n=1$. -For $x=(x_n)_n\in\prod_nG_n$, write $p^\omega y=(p^ny)_n$. Denote by $x\mapsto \bar{x}$ the quotient map $\prod\to\prod^\star$. Since for every $x$ and for $p=2,3$, the element $\overline{p^\omega x}$ is $p$-divisible (i.e., has $p^n$-roots for all $n$), so is its image by $f$, which forces $f\big(\overline{2^\omega x}\big)=f\big(\overline{3^\omega x}\big)=0$. Hence, for every $y=(y_n)_n\in\prod_nG_n$, we have $y=2^\omega ay+3^\omega by$ and hence $f(\overline{y})=0$. So $f=0$. -[Note: this also follows from the $G_n=\mathbf{Z}$ case, by a simple composition argument.] -The second assertion follows from the first since $\prod^\sigma_n G_n$ is a quotient of $\prod^\star_n G_n$. - -Let me now considerably relax the countability assumption. Let me say that a set $I$ is reasonable if every ultrafilter on $I$, stable under countable intersections, is principal. (If there's a standard terminology I'd be happy to change.) -This only depends on the cardinal of $I$. It's consistent that every set is reasonable. If there's a non-reasonable cardinal, there's a minimal one, which is known as smallest measurable cardinal. All "reasonably small" cardinals are reasonable; obviously $\omega$ is, and notably $\alpha$ reasonable implies $2^\alpha$ reasonable and in particular sets of cardinal $\le \mathfrak{c}$ (continuum), $\le 2^{\mathfrak{c}}$, etc, are reasonable. -The point is that the proposition and lemma above still hold when the index set is supposed to have reasonable cardinal. - -Lemma'. Let $I$ be a reasonable set, and $(G_i)$ a family of abelian groups. Then $\mathrm{Hom}(\prod^\star_{i\in I} G_i,\mathbf{Z})=\{0\}$. -Remark: using a nonprincipal ultrafilter stable under countable intersections, conversely if $I$ is not reasonable then $\mathrm{Hom}(\prod^\star_{i\in I} \mathbf{Z},\mathbf{Z})\neq\{0\}$, since a nontrivial homomorphism is obtaining by taking the limit along such an ultrafilter. - -Proof: again, this follows from the case $G_i=\mathbf{Z}$: let $f:\prod_iG_i\to\mathbf{Z}$ vanish on $\bigoplus G_i$. If $f$ is nonzero, say on some element $(g_i)_i$, consider the homomorphism $\mathbf{Z}^I\to\prod_i G_i$ mapping $(n_i)_i$ to $(n_ig_i)_i$; it maps $\mathbf{Z}^{(I)}$ into $\bigoplus G_i$ and we can conclude from the fact that $\mathrm{Hom}(\prod^\star_{i\in I}\mathbf{Z},\mathbf{Z})=\{0\}$. -I don't know a reference, so let me provide a proof. -Choose, by contradiction, $f\neq 0$ as above. For every subset $J$ of $I$, denote by $R_J(f)$ the restriction of $f$ to $\prod_{j\in J}G_j$. Let $\mathbf{F}$ be the set of $J$ such that $R_J(f)\neq 0$. Clearly $J'\subset J\notin\mathcal{F}$ implies $J'\notin\mathcal{F}$. Let us say that $J\in\mathcal{F}$ is simple if it is not disjoint union of two elements of $\mathcal{F}$. -Claim: There exists a simple $J\in\mathcal{F}$. -Proof of claim: otherwise, one chooses $X'_0\in\mathcal{F}$, partition it as $X'_0=X_0\sqcup X'_1$ with $X_0,X'_1\in\mathcal{F}$, then partition $X'_1=X_1\sqcup X'_2$, etc, to obtain a partition $(X_n)$ of disjoint elements in $\mathcal{F}$. Then choosing $x_n$ with $f(x_n)\neq 0$, supported in $X_n$, for $u=(u_n)_n\in\mathbf{Z}^{\mathbf{N}}$, we define $f'(u)=f(\sum u_nx_n)$, we obtain a contradiction with the previously proved countable case. -Now choose $J$ simple, and define $\mathcal{U}=\{J'\in\mathcal{F}:J'\subseteq J\}$. Then $\mathcal{U}$ is an ultrafilter on $J$. Furthermore, it is stable under countable intersections: it is enough to show that the set of complements of elements of $\mathcal{U}$ (which is also the complement of $\mathcal{U}$) is stable under countable disjoint unions. This is, again, a straightforward consequence of $\mathrm{Hom}(\prod_{n\in\mathbf{N}}^\star\mathbf{Z},\mathbf{Z})=\{0\}$. - -Corollary: let $K$ be a field. Then the field $K(t)$ is not isomorphic to any nonprincipal ultraproduct of fields indexed by any reasonable (in the above sense) set. More precisely, the group $K(t)^*$ is not isomorphic to any nonprincipal ultraproduct of groups indexed by any reasonable set. - -Again, this is false if one allows non-reasonable cardinals. Indeed, if the ultrafilter is stable under countable intersections, and the field $K$ is countable, then the field $K(t)$ is isomorphic to its own ultrapower with respect to such an ultrafilter. - -Edit: - -Fact. Let $\sigma$ be an ultrafilter on a set $I$. Equivalences: - -Then $\sigma$ is not stable under countable intersections. -There exists a function $u:I\to\mathbf{N}$ such that $\lim_{i\to\sigma}u=\infty$ -There exists a strictly decreasing sequence $(I_n)$ of subsets of $I$, with empty intersection and $I_n\in\sigma$ for all $n$. - - -(An ultrafilter stable under countable intersections is usually called $\omega_1$-complete, or $\sigma$-complete.) -Proof: Suppose 1. So we have a sequence $(J_n)$ with $J_n\in\sigma$ and $J=\bigcap J_n\notin\sigma$. Setting $I_n=J^c\cap \bigcap_{i\le n}J_n$, we obtain 3. Supposing 3 (with $I_0=I$), we obtain 2 by defining $u(i)=\sup\{n:i\in J_n\}$. 2 implies 1 is clear, choosing $I_n=\{i:u(i)\ge n\}$. $\Box$ -Therefore, the ultrafilter being non-$\sigma$-complete (rather than non-principal) is the optimal assumption for the proposition to hold. Namely: - -Proposition: let $K$ be a field. Then the field $K(t)$ is not isomorphic to any non-$\sigma$-complete ultraproduct of fields; more precisely, the group $K(t)^*$ is not isomorphic to any non-$\sigma$-complete ultraproduct of groups. Conversely, if $K$ is a countable field, then the field $K(t)$ is isomorphic to any $\sigma$-complete ultraproduct of itself.<|endoftext|> -TITLE: What are the pillars of Langlands? -QUESTION [31 upvotes]: I had previously asked: -Narratives in Modular Curves -Since then, I've read quite a bit more (but not nearly enough) and I have a few follow up questions about the big picture. As you will soon see, I'm confused about how to think about things, and seeing the big picture will help me a lot in learning the specifics (learning in the dark is difficult!). -As I understand it, the story goes like this. First, one defined for every number field $\zeta_K(s)=\sum_{\mathfrak{a}} \frac{1}{(N\mathfrak{a})^s}$. One then defines a Dirichlet character, and for any such one defines $L(\chi,s)$. Further, for any $1$-dimensional Galois representation, $\rho: Gal(K/\mathbb{Q}) \rightarrow \mathbb{C}$, one defines $L(\rho,s)$. Now, in the $1$-dimensional case, the main two theorems that comprise class field theory are: if $K$ is abelian over $\mathbb{Q}$ with group $G$, then $\zeta_K(s)=\prod_{\rho \in \hat{G}} L(\rho,s)$; and for any such $\rho$ there exists a unique primitive Dirichlet character $\chi$ such that $L(\rho,s)=L(\chi,s)$. So far I follow the story perfectly. -There is also the issue of what if the base field is not $\mathbb{Q}$, which, admittedly, I don't fully have down. -Already in dimension $2$ I have a hard time figuring out what generalizes what corresponding thing from dimension $1$. For Galois representations, one continues to define $L(\rho,s)$ in a similar manner: as the product over $p$ of the characteristic polynomials of the action of the corresponding Frobenius (whenever defined for that $p$! It is still a little murky to me what happens at the bad primes). But now we have modular forms coming in to the picture, and the whole theory of modular curves. So how does this fit in as a generalization of the 1-dimensional case? Here's my best guess, you can tell me if I'm right. For a modular form $f$, one defines the $L$-function for it by the $q$-expansion of $f$: If $f(z)=\sum a(n)e(nz)$ then $L(f,s)=\sum \frac{a(n)}{n^s}$. Then various things that I do not fully understand come into play, claiming things like: $L(s,f)=\prod_{q|N}(1-a(q)q^{-s})^{-1} \prod_{p\not |N} (1-a(p)p^{-s}+f(p)p^{k-1-2s})^{-1}$ (probably just for $f$'s with some property, akin to being primitive). It seems (is this true?) that Hecke theory implies that these $L$'s are ``nice'' in the sense that they generalize Dirichlet $L$ functions. Is this the right way to see it? How? What is the $1$-dimensional analogue of modular functions, and modular curves? -Then I imagine that one has the modularity theorem, one of whose versions is(?) that for every $2$-dimensional Galois representation there's a modular function for which $L(\rho,s)=L(f,s)$. -You will notice that at no point did I talk about the adelic aspect. This is because I don't know where to put it. Is the adelic side easily equivalent to the (Dirichlet characters)-(modular functions) side?(are these two even on the same side?) Is it another pillar with which equivalence is far from trivial with both the Galois representations side AND the (Dirichlet characters)-(modular functions) side? In short -- I'm not sure what the pillars of Langlands are! -Further, let us assume that we have some version of Langlands. Is there a conjectural equivalent form of $\zeta_K(s)=\prod_{\rho \in \hat{G}} L(\rho,s)$ for $K/\mathbb{Q}$ not abelian? - -REPLY [36 votes]: In either the one or two dimensional case, there are two sides: Galois and automorphic. -Let me talk for a moment about the $n$-dimensional situation. -The Galois side involves studying continuous $n$-dimensional representations of the Galois group of -a number field $K$. -There are subtleties here about over what field these representations are defined, which I will return to. For now, let's imagine that they are representations into $GL_n(\mathbb C)$, -hence what are usually called Artin representations. -The automorphic side involves so-called automorphic representations of $GL_n(\mathbb A)$, -where $\mathbb A$ is the adele ring of $K$. These are irreduible representations which appear as Jordan--Holder factors in the -space of automorphic forms, which is, roughly speaking, the space of functions -on the quotient $GL_n(K) \backslash GL_n(\mathbb A)$. (To provide some orientation with regard to this notion: Note that, by Frobenius reciprocity, -any irreducible representation of a group $G$ appears in the space of functions on $G$ --- here I am ignoring topological issues --- and so any irreducible rep. of $GL_n(\mathbb A)$ will appear in the space of functions on $GL_n(\mathbb A)$. On the other hand, appearing in the space of functions on the quotient $GL_n(K)\backslash GL_n(\mathbb A)$ turns out to be a serious restriction --- most representations of $GL_n(\mathbb A)$ don't appear there.) -The idea --- roughly --- is that $n$-dimensional Galois representations will match -with automorphic representations. How does this happen? -Well, $n$-dimensional representations have traces of Frobenius elements, -so to each unramified prime we can attach a number. On the other hand, it turns out -that automorphic representations have essentially canonical generators, which can -be interpreted as Hecke eigenforms, and so (for primes not dividing the conductor -of the representation) we get a Hecke eigenvalue. The matching is given by the rule that traces of Frobenius elements should equal Hecke eigenvalues. -Let's consider the case of dimension $n = 1$: -First, any $1$-dim'l rep'n of the Galois group of $K$ factors through $G_K^{ab}$, while -the space of functions on the abelian group $K^{\times}\backslash \mathbb A^{\times}$ -(which is the adelic quotient considered above in the case $n = 1$) is (by Fourier -theory for locally compact abelian groups, and speaking somewhat loosely) the sum of spaces spanned by characters, so that automorphic representations are just characters of -$K^{\times}\backslash \mathbb A^{\times}$. -So our goal is to match characters of $G_K^{ab}$ with characters of $K^{\times}\backslash -\mathbb A^{\times}$. -Now global class field theory actually does something stronger: it directly relates -$G_K^{ab}$ to $K^{\times}\backslash \mathbb A^{\times}$. In particular, finite order -characters of the latter (which are just Dirichlet characters in the case $K = \mathbb Q$) -correspond to finite order characters of the former, the $L$-functions match, and so on. -Now consider the case $n = 2$: -The quotient $GL_2(K)\backslash GL_2(\mathbb A)$ is not a group, -just a space (and this is the same for any $n \geq 2$). Also, there is no quotient -of the Galois group $G_K$ akin to $G_K^{ab}$ with the property that all $2$-dimensional reps. of $G_K$ factor precisely through that quotient. -In short, unlike in the case $n = 1$, we can't hope to find groups that will be related in some direct manner (unlike in the case $n = 1$, where class field theory gives a direct relation between $G_K^{ab}$ and $K^{\times}\backslash \mathbb A^{\times}$); the relation -really has to be made at the level of representations. -Also, when you consider the quotient $GL_2(K)\backslash GL_2(\mathbb A)$, it is much harder -(in fact, impossible) to separate the archimedean and non-archimedean primes. If you play around and follow up some of the references suggested by others, you will see (in the case $K = \mathbb Q$) that this quotient is related to the upper half-plane and congruence subgroups, and the generating vectors for automorphic representations are precisely primitive Hecke eigenforms (either holomorphic modular forms or else Maass forms). The archimedean prime contributes the upper half-plane, and the finite primes contribute the level of the congruence subgroup and the Hecke operators. -Thus the analogue of a Dirichlet character in the $2$-dimensional case is a Hecke eigenform. -(But it is better to regard the latter as corresponding more generally to idele class characters --- also called Hecke characters or Grossencharacters; these are not-necessarily-finite-order generalizations of Dirichlet characters.) -If you really want to study just Galois represenations with finite image (i.e. Artin representations), on the automorphic side (for $K = \mathbb Q$ and $n = 2$) you should restrict to weight one holomorphic modular forms and Maass forms with $\lambda = 1/4$. (The former are proved to correspond precisely to two-dimensional reps. for which complex conjugation has non-scalar image, while the latter are conjectured to correspond to two-dimensional reps. for which complex conjugation has scalar image.) The other Maass forms -are not supposed to correspond to any Galois representations (just as non-algebraic idele class characters --- those that are not type $A_0$ in Weil's terminology --- don't correspond to anything on the Galois side in class field theory), while higher weight holomorphic modular forms are supposed to corresond to compatible systems of two-dimensional $\ell$-adic Galois representations with infinite image (just as infinite order algebraic Hecke characters correspond to -compatible systems of one-dimensional $\ell$-adic Galois representations --- see Serre's bok on this topic). -As the preceding summary shows, the big picture here is pretty big, and the technical details are quite extensive and involved. There is the added complication that the proofs of what is known in the non-abelian case are very involved, and often use methods that don't play a role in the general story, but seem to be crucial for the arguments to go through. (E.g. Mellin transforms don't play any role in the general theory of attaching $L$-functions to automorphic representations, but in the case $n = 2$, the fact that you can pass from a modular form to its $L$-function via a Mellin transform is often useful and important.) -What I would suggest is that you try to learn a little about Hecke characters beyond the finite order case, perhaps from Weil's original article and also from Serre's book. (Silverman's treatment of complex multiplication elliptic curves may also help.) This will help you become familiar with the crucial idea of compatible families of $\ell$-adic Galois representations, and see how it relates to objects on the automorphic side, in the fundamental $n = 1$ case. -Then, to begin to grasp the $n = 2$ case, you will want to understand how modular forms -give compatible systems of two-dimensional $\ell$-adic representations. The general statement here is due to Deligne, and can be learnt (at least at first) as a black-box. The case of weight 2, which includes the case of elliptic curves over $\mathbb Q$, is easier, and it's possible to learn the whole story (other than proof of the modularity theorem itself) in a reasonable amount of time. The case of weight one is special, and is treated in a beautiful paper of Deligne and Serre. The converse here (that every two-dimensional -Artin rep'n of appropriate type comes from a weight one form) is at least as difficult as the modularity of elliptic curves, and so learning the statement should suffice at the beginning. -And now you confront the difficulties in learning non-abelian class field theory: already -in the two-dimensional case, the theory has some aspects that remain almost entirely conjectural, -namely the conjectured relationship between certain Maass forms and certain two-dimensional -Galois representations. So at this point, you have to choose whether you just want to get some sense of the big picture and the expected story in general (say by looking over Clozel's Ann Arbor article and the recent preprint of Buzzard and Gee), or to begin actually working in the area. If you choose the latter, then the big picture is useful, but only in a limited way: to make progress, it seems that one has to then start learning many techniques that don't seem to be essential to the story from the big picture point of view, -but which are currently the only known methods for making progress (depending on what direction you want to pursue, these include the trace formula, Shimura varieties, the deformation theory of Galois representations, the Taylor--Wiles method, ... ).<|endoftext|> -TITLE: Injections to binary sequences that preserve order -QUESTION [5 upvotes]: Suppose we have a countable set S with a total order. Can we give an injection from S to the set of finite binary sequences that end in all zeros that preserves the ordering? The order on binary sequences is the dictionary ordering (e.g. 001001 <= 01). -For a finite set this is easy: arrange the set in order and assign an increasing sequence of binary sequences. -For the natural numbers this is also easy: send a number n to the sequence that starts with n ones (a similar solution works for negative numbers). -For the rationals this is already a bit more difficult. I believe the following works: Take the Stern-Brocot tree. Start at the root and walk down to the rational number. Every time you go left, write a 0. Every time you go right, write a 1. Finally write another 1. -So an equivalent formulation seems to be: can we arrange S into a binary tree such that the elements are arranged in order from left to right as in the Stern-Brocot tree. -My question is: can this be done for any countable set with a total order? -The question came up in a discussion whether radix sort can be used to sort any set (radix sort can sort binary sequences). - -REPLY [6 votes]: It is easy to prove that for any countable linearly ordered set there is an order preserving injection to the rationals. This can be proven by enumerating the base set and then specifying the values of the mapping by induction. -Since you have a solution for $\mathbb Q$, for other sets just compose the order preserving injection from above with this solution.<|endoftext|> -TITLE: Blow-ups at points in non-general position -QUESTION [5 upvotes]: Is it well known what happens if one blows-up $\mathbb{P}^2$ at points in non-general position (ie. 3 points on a line, 6 on a conic etc)? Are these objects isomorphic to something nice? - -REPLY [2 votes]: I'll just add to Francesco's answer by saying that general position of the points on the plane is equivalent to ampleness of the anticanonical sheaf $\omega_X^{\otimes -1}$. -The key observation is that on a del Pezzo surface, an irreducible negative curve ($C^2 < 0$) must be an exceptional curve (i.e. $C^2 = C\cdot K_X = -1$). This follow from the adjunction formula and the Nakai-Moishezon criterion. -If you blow up 3 colinear points, then the strict transform of the line containing these points will have self-intersection $-2$, which is not allowed by the key observation. Similiarly for the strict transform of a conic through 6 blown-up points. There is one more condition you have to impose: if you blow up 8 points, they cannot lie on a singular cubic with one of the points at the singularity. -If you relax the requirement that $\omega_X^{\otimes -1}$ be ample to just big and nef, then you can have some degenerate point configurations: this time $C^2 = -2$ is allowed, so 3 colinear points ar OK. However, 4 colinear points would not be OK.<|endoftext|> -TITLE: Torus minimizer of Willmore energy -QUESTION [10 upvotes]: It is my understanding that the torus that minimizes the Willmore energy -is not yet known -(this from a sentence in a 2005 paper by John Sullivan). -Willmore conjectured that the Willmore energy for any smooth, -immersed torus is $\ge 2 \pi^2$. -From what I can gather, -this energy is achieved by a standard rotationally round torus, -derived from the Clifford torus. -Assuming the problem remains open, my question is: - -Are there any viable candidates for different torus - Willmore minimizers, or does the evidence point to the one - just mentioned? - -I'm wondering if the problem remains open because all -strange, unlikely alternatives have not been ruled out, -or because there are actually some viable alternatives. -Likely the right reference I am not finding would suffice. -Thanks! - -Addendum (2Sep13). -As Renato Bettiol first pointed out below, the Willmore conjecture has -been solved by -Fernando Marques and -André Neves. They posted a 96-page paper to the arXiv: - -Fernando Marques and André Neves. - "Min-Max theory and the Willmore conjecture." - arXiv:1202.6036 (2012). Updated March 2013. - - - - -The Willmore Torus. Image by Tom Banchoff, cited in Morgan article. -The result was hailed in a Huffington Post article by -Frank Morgan: - -Frank Morgan. "Math Finds the Best Doughnut." - Huffington Post, 2 April 2012. - -And there is a very nice description by Dana Mackenzie -in an article that also describes the related resolution of -the Lawson conjecture by Simon Brendle: - -Dana Mackenzie. "What's Happening in the Mathematical Sciences." - Vol. 9. American Mathematical Soc., 2013. (AMS link) - -(7Sep13). One more remark. It remains open what might be the 2-holed or -3-holed Willmore surface of minimal bending energy. Dana says (p.28), - -..., it is difficult to know what even to conjecture about such surfaces. - -REPLY [10 votes]: Just a few days ago, Fernando C Marques and Andre Neves posted a preprint on the arxiv in which they (claim to) provide a complete proof of Willmore's Conjecture. I have no idea how much of it has been verified, especially given it is so recent, but the geometric ideas used (min-max theory for minimal submanifolds) seem very elegant.<|endoftext|> -TITLE: Generalization of plane geometric trees? -QUESTION [5 upvotes]: View a plane tree drawn in $\mathbb{R}^2$ as a joining of geometric (straight) segments at endpoints such that (a) they avoid intersecting one another (except where they share a vertex), and (b) they avoid creating a cycle, which would enclose a positive planar area. -I am interested in the generalization to $\mathbb{R}^3$ as follows. -Join together (flat) polygons, glued edge-to-edge, such that (a) they avoid intersecting one another (except where they share vertices and/or edges), and (b) they avoid enclosing (water-tightly) -a positive volume. -My question is: - -Is there a name for this construct? Has it been studied? - -I am mainly seeking references to any literature on this or related concepts. -Of course there is a generalization to $\mathbb{R}^d$, but I would be happy to learn -of work just generalizing plane trees to ??? in $\mathbb{R}^3$. -I cannot think of what it might be named: open panel structures? -It's come up in my work, and I would be delighted to christen it, but surely it has been -studied...? -Thanks in advance! -Addendum. As Greg Kuperberg kindly explained, the concept I described is -a collapsible complex. It is usually defined for simplicial complexes, but works -as well when the constituents are polytopes rather than simplices, e.g., polygons in $\mathbb{R}^3$. - -REPLY [3 votes]: Also check out the House with One Room notes form Allen Hatcher<|endoftext|> -TITLE: Is a non-compact Riemann surface an open subset of a compact one ? -QUESTION [10 upvotes]: Let $X$ be a non-compact holomorphic manifold of dimension $1$. Is there a compact Riemann surface $\bar{X}$ suc that $X$ is biholomorphic to an open subset of $\bar{X}$ ? -Edit: To rule out the case where $X$ has infinite genus, perhaps one could add the hypothesis that the topological space $X^{\mathrm{end}}$ (is it a topological surface?), obtained by adding the ends of $X$, has finitely generated $\pi_1$ (or $H_1$ ). Would the new question make sense and/or be of any interest? -Edit2: What happens if we require that $X$ has finite genus? (the genus of a non-compact surface, as suggested in a comment below, can be defined as the maximal $g$ for which a compact Riemann surface $\Sigma_g$ minus one point embeds into $X$) - -REPLY [3 votes]: Useful references for your question are Robert Brooks' "Platonic surfaces" and Dan Mangoubi's "Conformal Extension of Metrics of Negative Curvature" (both on arxiv). -I emailed Luca Migliorini requesting his paper. He told me it was basically his undergraduate thesis, published in a defunct italian journal, and that no copy of it remains. In otherwords, utterly useless. -The basic fact on compactifying a riemann surface is this: if $S$ is a finite area riemann surface, then there exists a compact riemann surface $S^c$ and a finite set of points $p_1, \ldots, p_k$ on $S^c$ such that $S^c \setminus {{p_1, \ldots, p_k}}$ is conformally equivalent to $S$. -In Brooks' paper, he states that this riemann surface $S^c$ is unique. However I'll admit to not be convinced of this uniqueness. The expression he uses throughout is "conformally filling punctures" -- a phrase which I think deserves more explanation than is given. -Lemma 1.1 in Brooks is interesting, and justifies the above claim. Of course we know what cusps on riemann surfaces look like. A cuspidal neighborhood $C$ of a Riemann surface can be taken isometric to the quotient of $\{ z\in \mathbb{H}^2: \Im(z)\geq 1/y \}$ by the isometry $z\mapsto z+1$, for some $y>0$. The parameter $y$ gives a measure on the size of the cusp, i.e. gives a geodesic loop homotopic to the puncture with hyperbolic length $y$. So the cusp $C$ is really isometric to the punctured ball of euclidean radius proportional to $1/y$ via the mapping $z\mapsto e^{2\pi i z}$ on the punctured open unit disk $D^\ast$ equipped with the metric $ds^*=\frac{-1}{r \log r} |dz|$. However $ds^*$ blows-up as $r\to 0$ like $1/r$. -Brooks (and afterwords Mangoubi more explicitly) gives, for any $\epsilon>0$, smooth bump functions $\delta$ concentrated at the origin on $D$ such that $e^\delta ds^*$ extends to a smooth metric past the origin and whose curvature remains pinched $-1 \pm \epsilon$. -I am going to include the details of this construction, together with some remarks relating to Donaldson's compactification of algebraic curves (from his book) shortly.<|endoftext|> -TITLE: Toy Models of Quantum Mechanics -QUESTION [33 upvotes]: Do toy models of quantum mechanics help us better understand "regular" quantum mechanics? For example, if we look at quantum mechanics over a finite field $F$ (e.g. $\mathbb{Z}_2$), can this lead to new insights for "regular" quantum mechanics? Or do these toy models just help clarify our understanding of "regular" quantum mechanics without actually providing any new insight? In other words, what is the utility of having toy models of quantum mechanics? - -REPLY [2 votes]: The theory of nonlocal boxes is quite illuminating to the beginner (or expert) trying to get a handle on entanglement. (Google "nonlocal box" or "box world".)<|endoftext|> -TITLE: Question on eigenvalue square root subadditivity -QUESTION [9 upvotes]: ORIGINAL QUESTION -Let $\lambda_{1}\left(\cdot\right)$ be the larger eigenvalue of a -$2\times2$ matrix and $\lambda_{2}\left(\cdot\right)$ the smaller -eigenvalue of a $2\times2$ matrix. Is it true that $$ -\left|\sqrt{\lambda_{1}\left(A+B\right)}-\sqrt{\lambda_{1}\left(B\right)}\right|+\left|\sqrt{\lambda_{2}\left(A+B\right)}-\sqrt{\lambda_{2}\left(B\right)}\right|\leq\sqrt{\lambda_{1}\left(A\right)}+\sqrt{\lambda_{2}\left(A\right)}$$ -for any two $2\times2$ positive-definite symmetric real matrices $A$ and $B$? -EDITED QUESTION (after Mikael de la Salle's original answer) -If $A$ is not positive definite, does one have $$ -\left|\sqrt{\left|\lambda_{1}\left(A+B\right)\right|}-\sqrt{\left|\lambda_{1}\left(B\right)\right|}\right|+\left|\sqrt{\left|\lambda_{2}\left(A+B\right)\right|}-\sqrt{\left|\lambda_{2}\left(B\right)\right|}\right|\leq\sqrt{\left|\lambda_{1}\left(A\right)\right|}+\sqrt{\left|\lambda_{2}\left(A\right)\right|}$$ -for any $2\times2$ symmetric real matrices $A$ and $B$, where $\lambda_{1}\left(\cdot\right)$ is the larger absolute value -eigenvalue and $\lambda_{2}\left(\cdot\right)$ -is the smaller absolute value eigenvalue? -Thanks for any helpful answers. - -REPLY [13 votes]: The answer to both your questions are yes. Let me start with the first question, which more straightforward. -A first remark: since $A$ is positive-definite, $\lambda_i(A+B) \geq \lambda_i(B)$ for $i=1,2$. -(to check this, use the formulas $\lambda_1(X) = \max_{\xi} \langle X\xi,\xi\rangle$ and $\lambda_2(X) = \min_{\xi} \langle X\xi,\xi\rangle$ where the min and max run over all unit vectors $\xi$. This formulas hold whenever $X$ is a symmetric $2 \times 2$ matrix.) -Your question is therefore whether $Tr(\sqrt{A+B})\leq Tr(\sqrt A)+ Tr(\sqrt B)$ for any symmetric positive definite matrices $A$ and $B$, or equivalently $Tr( \sqrt{X X^{T} + Y Y^{T} } ) \leq Tr(\sqrt{X X^{T} })+Tr(\sqrt{Y Y^{T} })$ for any matrices $X$ and $Y$, where $X^{T}$ denotes the transpose of $X$, or hermitian tranpose if you work with complex matrices. This inequality is true in any dimension (not just 2), and it is just the triangle inequality for the Schatten 1-norm given by $\|X\|_1 = Tr(\sqrt{X X^{T} })$. The expression $Tr(\sqrt{X X^{T} + Y Y^{T} })$ is indeed the 1-norm of the matrix $\begin{pmatrix}X&Y \\\\ 0&0\end{pmatrix}$. - -EDIT: It seems from the comments that my answer to your second question was far from clear. Let me try to explain differently the proof I had in mind. -For 6 real numbers $\alpha_1 \geq \alpha_2$, $\beta_1\geq \beta_2$ and $\gamma_1 \geq \gamma_2$, denote by $f(\alpha_1,\alpha_2,\beta_1 , \beta_2,\gamma_1,\gamma_2)$ the quantity $\sqrt{|\alpha_1|} + \sqrt{|\alpha_2|} - |\sqrt{\max(|\gamma_1|,|\gamma_2|)} - \sqrt{\max(|\beta_1|,|\beta_2|)}| - |\sqrt{\min(|\gamma_1|,|\gamma_2|)} - \sqrt{\min(|\beta_1|,|\beta_2|)}|$. -You are asking whether $f \geq 0$ provided that $\alpha,\beta,\gamma$ are the ordered eigenvalues of respectively $A,B,A+B$ for symmetric $2 \times 2$ matrices $A$ and $B$. The answer is yes, and I am sketching a proof. Denote by $D$ the possible values for $(\alpha_1,\alpha_2,\beta_1 , \beta_2,\gamma_1,\gamma_2)$. -$D$ is exactly described by Horn's inequalities. These inequalities are -$$\alpha_1 \geq \alpha_2 \ \ , \ \ \beta_1\geq \beta_2,$$ -$$\gamma_1 + \gamma_2= \alpha_1 + \alpha_2+\beta_1+\beta_2,$$ -$$\alpha_2+\beta_2 \leq\gamma_2 \leq \min(\alpha_1+\beta_2,\alpha_2+\beta_1).$$ -In particular, $D$ is a convex subset of dimension $5$ of $\mathbb R^6$, and one easily checks that its boundary corresponds to the case when $A$ and $B$ commute. Since the inequality is true when $A$ and $B$ commute (this is eay to check, see the other answer), your question reduces to whether $\inf_D f = \inf_{\partial D} f$. This transforms your eigenvalue question to a purely calculus question. -Notice now that $\beta,\gamma$ and $\alpha_1+\alpha_2$ being fixed, $f(\alpha,\beta,\gamma)$ decreases as $\min(|\alpha_1|,|\alpha_2|)$ decreases. Moreover, if you started with $\alpha,\beta,\gamma$ in the interior of $D$, you stay in $D$ if you make $\min(|\alpha_1|,|\alpha_2|)$ decrease, until you reach the boundary of $D$, or $\min(|\alpha_1|,|\alpha_2|)=0$. You are therefore left to prove that $f(\alpha,\beta,\gamma) \geq \inf_{\partial D} f$ if $(\alpha,\beta,\gamma) \in D$ with $\min(|\alpha_1|,|\alpha_2|)=0$. -In the same way, fixing $\alpha,\beta$ and $\gamma_1+\gamma_2$, you reduce the question to proving that $f(\alpha,\beta,\gamma) \geq \inf_{\partial D} f$ if $(\alpha,\beta,\gamma) \in D$ with $\min(|\alpha_1|,|\alpha_2|)=0$ and $\min(|\gamma_1|,|\gamma_2|)=0$. -Last, fixing $\alpha, \gamma$ and $\beta_1+\beta_2$ with $\min(|\alpha_1|,|\alpha_2|)=0$ and $\min(|\gamma_1|,|\gamma_2|)=0$, you see that $f(\alpha,\beta,\gamma)$ decreases as $\min(|\beta_1|,|\beta_2|)$ increases, until you reach the boundary of $D$. This proves that $\inf_D f = \inf_{\partial D} f$.<|endoftext|> -TITLE: More on universal homeomorphisms -QUESTION [6 upvotes]: I would like to understand this notion better; where could I find some examples? In particular, I am interested in the following questions (and references for the answers). - -Is a universal homeomorphism of connected regular (excellent finite dimensional) schemes an isomorphism if these schemes are not positive characterstic ones? -Suppose that a finite morphism $f:X\to Y$ of connected regular (excellent finite dimensional) schemes is generically purely inseparable. Does $f$ restrict to a universal homeomorphism of some open (non-empty) subschemes of $X$ and $Y$? - -REPLY [8 votes]: Yes. Let $f\colon X \to Y$ be a universal homeomorphism of locally noetherian schemes. Assume that $X$ and $Y$ are integral and $Y$ is normal, and the function field $k(Y)$ has characteristic 0. Then $k(X) = k(Y)$, and $f$ is an isomorphism by Zariski's main theorem. -Under these conditions $f$ is a universal homemorphism. In fact $X$ is the normalization of $Y$ in $k(X)$, again by Zariski's main theorem. But $k(X)$ is purely inseparable over $Y$, so it is obtained by a successive extraction of $p^{\rm th}$ roots of 1. If $F \colon Y \to Y$ is the absolute Frobenius, some power of $F$ will factor through $X$, and then it is easy to see that $X$ is universally isomorphic to $Y$, since $F$ is a universal isomorphism.<|endoftext|> -TITLE: Solvability in differential Galois theory -QUESTION [15 upvotes]: It is well known that the function $f(x) = e^{-x^2}$ has no elementary anti-derivative. -The proof I know goes as follows: -Let $F = \mathbb{C}(X)$. Let $F \subseteq E$ be the Picard-Vessiot extension for a suitable homogeneous differential equation for which $f$ is a solution. -Then one may calculate $G(E/F)$ and show it is connected and not abelian. -On the other hand, a calculation shows that if $K$ is a differential field extension of $F$ generated by elementary functions then the connected component of $G(K/F)$ is abelian, so it is impossible for an anti-derivative of $f$ to be contained in such a field $K$. -However, in classical Galois theory we can do much better, there, we know that a polynomial equation is solvable by radicals if and only if the corresponding Galois group is solvable. -So to my question - is an analog of this is available in differential Galois theory? Is there a general method to determine by properties of $G(F/E)$ if $F$ is contained in a field of elementary functions? - -REPLY [11 votes]: The analogue to "solvable by radicals" in differential Galois theory is "solvable by quadratures". The theorem says that a PV-extension is Liouvillian (adjoining primitives and exponentials) iff the connected component of the differential Galois group is solvable. See "A first look at differential algebra" by Hubbard and Lundell, for an expository account. - -I slightly misread the question at first, thinking you were looking for the analogue of solvbility by radicals in differential algebra. When it comes to determining if the primitive of a function is elementary or not, the characterization is given by Liouville's theorem. Now, for the general case of differential equations solvable in terms of elementary functions, there is a generalization of Liouville's theorem, that you can find in the article "Elementary and Liouvillian Solutions of Linear Differential Equations", by M.F. Singer and J. Davenport (link to Singer's papers here).<|endoftext|> -TITLE: Is there a table of (fibred knot) monodromies? -QUESTION [10 upvotes]: Background/motivation -I'm working on contact topology (in dimension three): a fundamental theorem of Giroux gives us a bijection between contact structures (up to isotopy) and open books (up to negative stabilisation). -Moreover, a stabilisation is a "hands on" operation both at the abstract level (i.e. surface with boundary and monodromy) and at the concrete level (i.e. fibred link in a three-manifold). -Whenever we have a fibred knot $K$ (say) in $S^3$, the corresponding fibration is an open book for $S^3$, which in turn supports a contact structure $\xi$ on $S^3$, where $K$ sits as a transverse knot. - -Now that we have a good theoretical framework, we'd like to put our hands on some examples, and we can start stabilising the open books coming from the Hopf bands and obtain many fibred knots/links and their monodromies, and these examples are not too hard to identify as knots/links in $S^3$. -My question is purely topological: what about the inverse approach? I want to recover the monodromy on the fibre, and I have an "algorithm" to find it, once I have compressing discs for the complement of a fibre (that is a Seifert surface of minimal genus), but to me it's rather crafty. I figure that someone must have done it at some point, so: - -Is there a table associating to each fibred knot the monodromy on its fibre? - -I could find only one source giving concrete examples, namely Burde and Zieschang's Knots, where monodromies for the trefoil and the figure-eight knot are computed. Also, I'm pretty sure that there's much material available for algebraic knots, and that's as far as I've got. - -REPLY [13 votes]: I've produced a table of monodromies for about 63% of the hyperbolic, fibred knots listed on knotinfo. This is available at: http://surfacebundles.wordpress.com/knot-complements/ https://bitbucket.org/Mark_Bell/bundle-censuses/overview (this link now contains significantly more data - for the origional data look under /source/Fibred Knots/). -This was done by producing a triangulation of every possible surface bundle over the circle for the surfaces $S_{1,1}, \ldots, S_{5,1}$ made from a composition of at most 15 Dehn twists about generators. Non-hyperbolic and non-knot complement manifolds were discarded and for each pair of isometric triangulations the short-lex later one was also discarded. Finally, for each hyperbolic, fibred knot complement listed on knotinfo, SnapPy was used to find a bundle on this list isometric to it if it existed. -As Sam points out, there is no canonical choice of generating set for $\mathop{Mod}(S_{g,1})$ so I used the Humphries generating set in each case. However, the monodromies obtained are the short lex earliest for each knot with respect to this generating set and the ordering of the generators shown at the bottom of the page. This ordering was chosen to minimise the running time; a different ordering can run several orders of magnitude slower. -I should point out that these results don't show the millions of knot complements that were also found but that don't (yet) appear on the knot tables. This simply comes from the fact that my tables are ordered by monodromy length whereas knotinfo's is ordered by crossing number.<|endoftext|> -TITLE: An example of a complex manifold without a finite open cover -QUESTION [6 upvotes]: Are there non-compact complex manifolds that -a) Don't embed in C^n (holomorphically) -and -b) Cannot be covered by a finite number of coordinate open sets? -If b) can be satisfied, then I think so can a) be by taking a product with a compact complex manifold. If one takes a Riemann surface of infinite genus, one does not have a "good" finite open cover, but I allow non-contractible open covers as well. Apologies in advance for this elementary question. - -REPLY [5 votes]: If $\widetilde X$ is a compact complex manifold of dimension $\geq 2$ and $x \in \widetilde X$ then $X = \widetilde X - \lbrace x \rbrace$ is a non-compact manifold that cannot be holomorphically embedded in $\mathbb C^N$. This is because, by Hartogs' Theorem, we have $\mathcal O(X) = \mathcal O(\widetilde X)$ and therefore global holomorphic functions on $X$ are constant, which is not the case for complex submanifolds of $\mathbb C^N$.<|endoftext|> -TITLE: The Monster Group uses in mathematical physics -QUESTION [19 upvotes]: I am doing a project on the inverse Galois problem, and am seeking to show that the monster group is realisable over the rationals. I have heard that the monster group has found uses in theoretical physics, and was wondering what those uses might be. Also, is there any practical significance to theoretical physics in the result I am aiming to prove? - -REPLY [32 votes]: There are presently no applications of the monster group in physics, though there is a lot of misleading speculation about this. However in the other direction there are some applications of ideas from physics to the monster group. In particular the no-ghost theorem in string theory is used to construct the monster Lie algebra acted on by the monster group. -The ideas from physics seem to have no direct connection with the problem of realizing the monster as a Galois group over the rationals. This was solved by Thompson, who showed by character-table calculations that the monster satisfies a sufficient condition ("rigidity") for it to be a Galois group.<|endoftext|> -TITLE: Occurrences of a simple reflection in the longest element of a Weyl group? -QUESTION [6 upvotes]: While looking at a preprint I've just bumped into a question about the longest element $w_0$ of a Weyl group $W$ (say irreducible of a Lie type $A$ - $G$ and of rank $n>1$, to simplify). Suppose this element is written as a not necessarily reduced product of the form $w s_n w'$, where both $w$ and $w'$ lie in the proper parabolic subgroup of $W$ generated by simple reflections other than $s_n$. My first impression is that this can occur only for type $A_n$, where for example when $n=2$ we get $w_0 = s_1 s_2 s_1 = s_2 s_1 s_2$. There is some relevant discussion of reduced expressions for $w_0$ in an earlier post here. But getting to a reduced expression from an arbitrary expression takes some work, as shown in the standard Tits algorithm. So I may be overlooking something. - -Is my impression stated above correct, and if so how can it best be made rigorous without case-by-case arguments? - -EDIT: As Ben points out, I need to avoid type A here, so my question is really about the remaining simple types. (I've edited the language above.) - -REPLY [4 votes]: First, I'll show that you can assume that $ws_nw'$ is a reduced product. -To see this, first note that if we multiply $s_n$ by a reduced expression for $w'$, the result will still be reduced, and this can then be extended to a reduced expression for the longest element by multiplying some reduced expression $s_{i_1} \cdots s_{i_k}$ on the left, where $k$ is $1 + l(w')$ less than the length of the longest element...although, as of yet, we don't know that none of the $i_p$ are equal to $n$. -But then $s_{i_1} \cdots s_{i_k} s_nw' = ws_nw'$ so that $s_{i_1} \cdots s_{i_k} = w$. By the nature of the braid relations, any simple reflection that appears in $s_{i_1} \cdots s_{i_k}$ must appear in any expression for $w$, so if $s_n$ appears in $s_{i_1} \cdots s_{i_k}$, then it must also appear in $w$, which is a contradiction. -So, if we have your expression $ws_nw'$, we then in fact have a reduced expression for the longest element that has only a single occurrence of $s_n$. -Now consider any positive root $\alpha$ that has a nonzero coefficient on the simple root $\alpha_n$ corresponding to $s_n$ and also satisfies $w_0 \alpha = -\alpha$. -Claim: $\alpha$ must go negative at the location of $s_n$ in our reduced expression. -Proof: As you apply the simple reflections in our reduced expression one by one to $\alpha$, the only simple reflection that can affect the coefficient of $\alpha_n$ is $s_n$, and so when you get to $s_n$, the simple reflection $s_n$ must negate the coefficient of $\alpha_n$ (for otherwise $w_0 \alpha$ could not be equal to $-\alpha$). (That is, if $s_n$ just nullified the coefficient on $\alpha_n$, because $s_n$ doesn't occur again, one could not end up at $-\alpha$ after all the simple reflections in the reduced expression for $w_0$ have been applied.) -In every case except for type A, there are at least two such $\alpha$...for instance, you can take the sum of the simple roots and the highest root. But they cannot both go negative at the occurrence of $s_n$ in our reduced expression for $w_0$! -Therefore, no such expression exists except in type A. -And then in type A, there's the expression 1 21 321 4321 ... n (n-1) (n-2) ... 321.<|endoftext|> -TITLE: equations defining a subvariety -QUESTION [6 upvotes]: The following question feels to me like a standard sort of 'fact' in birational geometry, but I can't seem to write down a correct set of details. Hopefully someone can point me to a reference and not a counter example! -Suppose $X$ is a variety (reduced and irreducible over an algebraically closed field, perhaps of characteristic zero) and suppose that there exist a very ample line bundle $L$ and a linear system $V \subset H^0(X,L)$ such that $Y = Bs(V)$ is the singular set of $X$ scheme theoretically, that $Y$ is smooth of codimension at at least 2, and that $\tilde X$, the blow up of $X$ along $Y$ is smooth. Further assume that $\phi_{|V|} X--> S$ birationally maps $X$ onto a smooth variety $S$. Let $\tilde \phi$ be the map from $\tilde X \to S$ induced by $V$. Further assume that, denoting by $f$ the map $\tilde X \to X$, that $f^{-1}(Y) = T$ surjects onto $S$. Let $v_1, \dots v_s$ be $s = \dim(S)$ general sections of $V$ so that the intersection $Z(v_1) \cap \dots Z(v_s) \cap S$ consists of finitely many smooth points say $p_1, \dots p_m $. -Also assume the $P = f( \tilde \phi^{-1}(\cup_{i=1:m} p_i))$ is a proper subset of $Y$. Then can one say that away from $P$, the sections $ v_1 \dots v_s$ generate the ideal of $Y$ in $X$ ? -The case I have in mind is where $Y$ is a smooth curve embedded in a sufficiently ample manner so that 1) $Y$ is defined by quadrics and 2) $X = Sec(X)$ is singular only along $Y$. Then $V$ would be the quadrics through $Y$. The point would be to use this sort of an argument to establish a minimum depth of $Sec(Y)$ along $Y$. -This is my first question, so please feel free to correct etiquette with this question as well as the mathematics. - -REPLY [2 votes]: Credit: This answer came out of trying to understand why auniket's answer (a.k.a. counterexample) works. -1) auniket is correct that for dimension reasons $T$ cannot surject onto $S$, so in particular my comment about $X$ being normal possibly helping is irrelevant. So is $T$. -2) It seems to me that there is a much more general problem with your desired statement. Namely I believe the following is true: -Claim: Under the conditions of the question, if in addition $\dim Y=0$ and $Y$ is reduced, then the desired statement cannot be true. -Proof: We may assume that $Y$ is a single point. Since by assumption $X$ is singular at $Y$, the local ring of $X$ at $Y$ is not a regular local ring. Therefore the ideal of $Y$ cannot be generated by $\dim X$ number of elements. On the other hand, by assumption $X$ is birational to $S$, so $\dim X=\dim S=s$. Therefore $v_1,\dots,v_s$ cannot generate the ideal of $Y$. $\square$ -Note: I think this actually covers both of auniket's examples and would definitely give an arbitrary number of normal examples. -3) It seems that this still leaves a sliver of hope for you as your $Y$ is a curve (and even in the zero-dimensional case if $Y$ is non-reduced, it could work out). However, if it is reduced then you are at the absolute minimal number of generators that the singularity condition allows.<|endoftext|> -TITLE: Where can I find a modern write-up of Heegner's solution of Gauss' class number 1 problem? -QUESTION [9 upvotes]: In a recent MO question someone mentioned Heegner's solution of the Gauss "class number 1" problem which takes the following form: - -When the class number of an imaginary quadratic form is 1 an elliptic curve is defined over $\mathbb{Q}$ and a modular function takes on integer values at certain quadratic irrationalities which leads to a collection of Diophantine equations: The solution of which finishes the theorem. - -I sadly can't read Heegner's original work (since I cannot read German) but also I don't think it's necessarily the best thing to read for this proof due to an alleged gap. So if anyone recognizes this proof sketch sketch and knows where I could read this in detail that would be wonderful! Thanks. - -REPLY [12 votes]: In his article On the "gap'' in a theorem of Heegner, Stark does a pretty thorough job of explaining where people thought the purported gap came from, to what extent it actually was a gap, and what you would need to fix such a thing if it existed. I'm paraphrasing, but he basically argues that the confusion stemmed from some errors (typos?) in some analytic results of Weber that Heegner had heavily used. So in a literal sense, Heegner had not proved it because he had cited faulty results, but Stark shows that he deserved credit for the theorem since using Heegner's argument with the correct versions of Weber results (which were indeed known to Weber), the job gets done. -Here's the mathscinet review of the article: -http://www.ams.org/mathscinet-getitem?mr=241384<|endoftext|> -TITLE: Question about the representation theory of SL(n,Z) -QUESTION [16 upvotes]: In this question, all representations are finite-dimensional representations over $\mathbb{C}$. -Fix some $n \geq 3$. Assume that $V$ is a representation of $\text{SL}(n,\mathbb{Z})$. Also, assume that $W$ is a subrepresentation of $V$ and set $V' = V/W$, so we have a short exact sequence -$$0 \longrightarrow W \longrightarrow V \longrightarrow V' \longrightarrow 0$$ -of $\text{SL}(n,\mathbb{Z})$ representations. Assume that the actions of $\text{SL}(n,\mathbb{Z})$ on $W$ and $V'$ extend to actions of $\text{SL}(n,\mathbb{C})$. Question : Must the action of $\text{SL}(n,\mathbb{Z})$ on $V$ extend to an action of $\text{SL}(n,\mathbb{C})$? -Margulis superrigidity says that the action virtually extends, but I can't construct representations where it doesn't extend on the nose. Of course, if it extends then $V = V' \oplus W$ as $\text{SL}(n,\mathbb{C})$-representations. - -REPLY [10 votes]: Consider the surjective map of $SL(n,\Bbb Z)$-modules $Hom_{\Bbb C}(V',V)\to Hom_{\Bbb C}(V',V')$. -Tim tells us that the identity map from $V'$ to $V'$ lifts to an $f:V'\to V$ which is invariant under a finite index subgroup $\Gamma $ of $SL(n,\Bbb Z)$. Then by averageing one can make it invariant under $SL(n,\Bbb Z)$.<|endoftext|> -TITLE: Can you flip the end of a large exotic $\mathbb{R}^4$ -QUESTION [28 upvotes]: Can you flip the end of a large exotic $\mathbb{R}^4$ - -Background -Definition (Exotic $\mathbb{R}^4$): -An exotic $\mathbb{R}^4$ is a smooth manifold $R$ homeomorphic but not diffeomorphic to $\mathbb{R}^4$, where $\mathbb{R}^4$ is equipped with its standard smooth structure. -Definition (Large exotic $\mathbb{R}^4$): -A large exotic $\mathbb{R}^4$ is an exotic $\mathbb{R}^4$ containing a four-dimensional compact smooth submanifold $K'$ that can not be smoothly embedded into $\mathbb{R}^4$. -Definition (End of a large exotic $\mathbb{R}^4$): -If $R$ is a large exotic $\mathbb{R}^4$ and $D^4$ is a four-dimensional disk topologically embedded into $R$ such that $K' \subset D^4$, then $R - D^4$ is an end of $R$. -Remark: -The previous definition varies slightly from the standard definition of "end", however it will be used for the remainder of this question. (See Gompf and Stipsicz Exercise 9.4.11 for the standard definition.) -Remark: -If $R - D^4$ is the end of a large exotic $\mathbb{R}^4$, then $R - D^4$ is a smooth manifold that inherits a smooth structure from $R$. -Definition (Flip of the end of a large exotic $\mathbb{R}^4$): -Given $R - D^4$, the end of a large exotic $\mathbb{R}^4$, a flip of $R - D^4$ is a diffeomorphism $f: R - D^4 \rightarrow R - D^4$ that maps the "inner region" of $R - D^4$, that "near" the removed $D^4$, to the "outter region", that "near infinity", and vica-versa. -Remark: -The previous definition is also non-standard. I am not aware of any standard definitions that carry, more-or-less, the same meaning. -So, at this stage the meaning of the question is hopefully clear. -Foreground -In our attempt to flip the end of a large exotic $\mathbb{R}^4$ an inconvenient truth stands in our way: -Theorem 1 (Uncountably many flips fail): -There are uncountably many large exotic $\mathbb{R}^4$'s that one can not flip the end of. -Quickly, let us see why this is true. Lemma 9.4.2 along with Addendum 9.4.4 of Gompf and Stipsicz state: -Lemma 1: -There exist pairs $(X,Y)$ and $(L,K)$ of smooth, oriented four-manifolds with $X$ simply connected, $Y$ and $K$ compact, $X$ and $L$ open (i.e. noncompact and boundaryless), $L$ homeomorphic to $\mathbb{R}^4$, and $X$ with negative definite intersection form not isomorphic to $n\langle-1\rangle$, such that $X - int(Y)$ and $L - int(K)$ are orientation-preserving diffeomorphic. -Theorem 9.4.3 of Gompf and Stipsicz states: -Theorem: -Any $L$ as appears in Lemma 1 is a large exotic $\mathbb{R}^4$. -The two statements above lead to: -Lemma: -One can not flip the end of any $L$ as appears in Lemma 1. -Proof: -Assume one could flip the end of $L$. Thus, one could use this flip to glue $L$ to the "end" of $X$ and obtain a simply connected closed smooth four-manifold with negative definite intersection form not isomorphic to $n\langle-1\rangle$. However, according to Donaldson's Theorem (Gompf and Stipsicz Theorem 1.2.30) there exists no such manifold. Thus, there exists no such flip. QED -Now we have shown that $L$ can not be flipped. Before we show how uncountably many large exotic $\mathbb{R}^4$'s can not be flipped, we need the definition: -Definition (Radial Family): -Let $R$ be an exotic $\mathbb{R}^4$. Thus, there exists a homeomorphism $h:\mathbb{R}^4 \rightarrow R$. Define $R_t$ as the image under $h$ of the open ball of radius $t$ centered at $0$ in $\mathbb{R}^4$. A radial family is a set of the form $\{R_t | 0 < t \le \infty \}$. -Remark: -If $R_t$ is a member of a radial family, then $R_t$ is a smooth manifold as it inherits a smooth structure from $R$ -Theorem 9.4.10 of Gompf and Stipsicz states: -Theorem: -If $\{L_t | 0 < t \le \infty \}$ is a radial family for an $L$ as appears in Lemma 1 and $r$ is such that $K \subset L_r$, then $\{L_t | r \le t \le \infty \}$ is an uncountable family of non-diffeomorphic large exotic $\mathbb{R}^4$'s. -This leads directly to a proof of Theorem 1. -Proof: -Assume one could flip the end of $L_t$ for $r \le t \le \infty$, where all notation is as in the previous theorem. Thus, one could use this flip to glue $L_t$ to the "end" of $X$ less the image of $L - L_t$ and obtain a simply connected closed smooth four-manifold with negative definite intersection form not isomorphic to $n\langle-1\rangle$. Again, according to Donaldson's Theorem (Gompf and Stipsicz Theorem 1.2.30) there exists no such manifold. Thus, there exists no such flip.QED -Things are seeming rather hopeless at this point. In fact, things are worse -than they seem! But, before we can revel in this despair, we must introduce two -definitions: -Definition (Simply Connected at Infinity): -Let $Z$ be a topological manifold. $Z$ is simply connected at infinity if for -any compact subset $C$ of $Z$ there exists a compact subset $C'$ of $Z$ that -contains $C$ and is such that the inclusion $Z - C' \rightarrow Z - C$ induces -the trivial map $\pi_1(Z - C') \rightarrow \pi_1(Z - C)$. -Definition (End Sum): -Let $Z_1$ and $Z_2$ be non-compact oriented smooth four-manifolds that are -simply connected at infinity. Choose two proper smooth embeddings $\gamma_i : [0, \infty) -\rightarrow Z_i$. Remove a tubular neighborhood of $\gamma_i((0, \infty))$ from -each $Z_i$ and glue the resulting $\mathbb{R}^3$ boundaries together respecting -orientations. The result is the end sum $Z_1 \natural Z_2$ of $Z_1$ and $Z_2$. -Remark: -The requirement that $Z_i$ is simply connected at infinity guarantees -that $\gamma_i$ is unique up to ambient isotopy and thus $Z_1 \natural Z_2$ -is unique up to diffeomorphism (Gompf and Stipsicz Definition 9.4.6). -Remark: -If $R_1$ and $R_2$ are exotic $\mathbb{R}^4$, then they are non-compact oriented -smooth four-manifolds that are simply connected at infinity and $R_1 \natural R_2$ -is a smooth manifold homeomorphic to $\mathbb{R}^4$. -Remark: -$X$ of Lemma 1 is simply connected at infinity. -Theorem 2: -If $\{L_t | 0 < t \le \infty \}$ is a radial family for an $L$ as appears in -Lemma 1 and $r$ is such that $K \subset L_r$, where $K$ is as in Lemma 1, then for -$R$ an exotic $\mathbb{R}^4$ and $t$ such that $r \le t \le \infty$ there exists -no flip of $R \natural L_t$. -Proof: -The proof is basically a slight variation on the above theme. Assume one could flip -the end of $R \natural L_t$ for $r \le t \le \infty$. Thus, one could use this flip -to glue $R \natural L_t$ to the "end" of $X$ less the image of $L - L_t$ end summed -with $R$, in other words with the flip glue $R \natural L_t$ to -$R \natural (X - (L - L_t))$, and obtain a simply connected closed smooth -four-manifold with negative definite intersection form not isomorphic to -$n\langle-1\rangle$. Again, according to Donaldson's Theorem -(Gompf and Stipsicz Theorem 1.2.30) there exists no such manifold. Thus, there -exists no such flip.QED -Now we can revel in this despair! -However, other ways of creating large exotic $\mathbb{R}^4$'s exist. For example, given a topologically slice knot that is not smoothly slice one can create a large exotic $\mathbb{R}^4$. (See, for example, Davis.) Such a large exotic $\mathbb{R}^4$, as far as I can see, might admit an end flip. But, I'm not sure. Thus, we end where we began. -Question - -Can you flip the end of a large exotic $\mathbb{R}^4$? - -REPLY [14 votes]: As stated, your question is equivalent to the existence of a large exotic 4-ball (a smooth $D^4$ which cannot be smoothly embedded into $\mathbb{R}^4_{std}$). -The existence of a flip would give rise to an exotic $S^4$, by gluing the $D^4$ at infinity using the flip diffeomorphism. Removing a (small) standard ball from this $S^4$ gives a large exotic $D^4$, since it contains a smooth submanifold $K'$ which cannot embed in $\mathbb{R}^4_{std}$. -Conversely, if you had a large exotic $D^4$, then you could adjoin a collar neighborhood of $S^3\times \mathbb{R}$ to get an exotic $\mathbb{R}^4$ which has a standard end (diffeomorphic to $S^3\times \mathbb{R}$), and therefore admits a flip. -Although I'm not an expert, I'm certain that the existence of a large exotic 4-ball is open (otherwise, the 4D smooth Schoenflies conjecture would imply the 4D smooth Poincare conjecture). -I realize that this does not answer the spirit of your question, which is whether there is a large exotic $\mathbb{R}^4$ which does not have a standard product end and which admits a flip.<|endoftext|> -TITLE: Will a ball fired through a focus of an ellipse eventually tend to a horizontal line? -QUESTION [8 upvotes]: A couple of years ago I came across this phenomenon which appears to be true although I am having difficulty proving it. -F and F' are foci of a billiard table in the shape of an ellipse. A ball is fired through one of the foci, I can prove that it will subsequently pass through the other focus (this is in fact the question that sparked off this idea). Now if we continue to follow the path of the ball then it seeems as though it will eventually tend to travel in a horizontal line. Some ideas I had were to create triangles and develop a recursive sequence of an angle and see if this tended to 0 or do a similar thing with the gradient of the path of the ball and see if this tended to 0 but couldn't quite get a nice formula either way. -This question is actually Exercise 4.3 in the following notes http://www.math.psu.edu/tabachni/Books/billiardsgeometry.pdf - -REPLY [8 votes]: Edit: sorry, there used to be a completely wrong solution here (I thought that a certain singular curve was a projective line). Now it is fixed. -There is another solution using algebraic geometry. Identify the ellipse with the projective line by sending the two points where the line through the foci meets the ellipse to $0$ and $\infty$. The map we get by starting from a point on the ellipse, getting the second intersection of the line through it and $F$ with the ellipse, then getting the second intersection of the line through that point and $F'$ with the ellipse is an invertible algebraic map sending $0$ to $0$ and $\infty$ to $\infty$, so it must have the form $x \mapsto cx$ for some constant $c$ (depending on the eccentricity). Thus, the billiard ball approaches the horizontal line at an exponential rate.<|endoftext|> -TITLE: Why can projective varieties just have abelian group operations? -QUESTION [10 upvotes]: I just started to read Shimura - Automorphic forms and number theory (Lecture notes in mathematics, 54). On page 20 or so, he mentions that every projective variety which is an algebraic group, is necessary abelian. -Why? - -REPLY [35 votes]: There are several different ways to see this. Here is one: -Let $G$ be our irreducible projective algebraic group variety over the field $k$, with identity element $e$. The group $G$ acts on itself by conjugation, and this action fixes $e$. Thus this induces -a $k$-linear action of $G$ on the local ring $\mathcal O_e,$ and hence on the (finite-dimensional) quotients -$\mathcal O_e/\mathfrak m_e^n$ for each $n$. Now any morphism from the irreducible projective variety $G$ to the affine variety $End_k(\mathcal O_e/\mathfrak m_e^n)$ must be constant, and so the $G$-action on each $\mathcal O_e/\mathfrak m_e^n$, and hence on $\mathcal O_e$ itself, must be trivial. -Since $G$ is irreducible, it is now easy to see, using the fact that the conjugation action -induces a trivial action on $\mathcal O_e$, that the conjugation action is in fact trivial on $G$ itself, and hence that $G$ is commutative. -(This argument breaks down if $G$ is not projective, because then $G$ can have non-trivial morphisms to the matrix rings $End_k(\mathcal O_e/\mathfrak m_e^n)$; it is instructive to think about this in the case $G = GL_n(k)$. It also breaks down if $G$ is not irreducible, e.g. if $G$ is a finite non-abelian group, then we can think of it as a zero-dimensional projective algebraic group. The point in this case is that one can't make the "analytic continuation" argument from the action on $\mathcal O_e$ to all of $G$.) - -REPLY [9 votes]: I borrow this proof from [Birkenhake-Lange, Complex Abelian Varieties, Lemma 1.1.1]. -Let $X$ be a projective variety having a group structure. I assume that we are working over $\mathbb{C}$. -Consider the commutator map $\Phi(x,y)=xyx^{-1}y^{-1}$, and let $U$ be a coordinate neighborhood of $1 \in X$. By the continuity of $\Phi$, and since $\Phi(x,1) =1 \in U$, for all $x \in X$ we can find open neighborhoods $U_x$ and $W_x$ such that $\Phi(U_x, W_x) \subset U$. -Since $X$ is compact, finitely many $V_x$ cover $X$. Calling $W$ the intersection of the corresponding subsets $W_x$, we get $\Phi(X, W) \subset U$. -Now $\Phi(1, y)=1$ for all $y \in W$. Since holomorphic functions on a compact variety are constant, it follows $\Phi(X, W)\equiv 1$. Being $W$ open and non-empty, this in turn implies $\Phi(X, X) \equiv 1$, which is our claim. -Notice that "projective" is not really necessary, in fact what we actually use in the proof is "compact complex". Indeed, pushing further this argument (by a straightforward use of the exponential map) one can show that any compact complex connected Lie group is a complex torus. - -REPLY [3 votes]: A very accessible proof (at the beginning of the book for the case over $\mathbb{C}$, and further on in the book for any characteristic) of that statement is present in Mumford's book Abelian varieties.<|endoftext|> -TITLE: Frobenius splitting and derived Cartier isomorphism -QUESTION [20 upvotes]: Let $X$ be a smooth algebraic variety over an algebraically closed field $k$ of characteristic $p>\dim X$. The motivation for my question comes from the following results. - -1. If $X$ is Frobenius split (the $p$-th power map $\mathcal{O}_X \to F_* \mathcal{O}_X$ admits n $\mathcal{O}_X$-linear splitting) then the Kodaira vanishing theorem holds for $X$. - -The proof uses nothing but Serre vanishing and the projection formula. - -2. If the complex $F_* \Omega^\bullet_X$ is quasi-isomorphic to a complex with zero differentials, then the Kodaira-Akizuki-Nakano vanishing theorem holds for $X$. - -The proof uses Cartier isomorphism, hypercohomology spectral sequences, Serre vanishing and the projection formula and is similar to that of 1. - -3 (Deligne-Illusie 1987). If $X$ lifts to $W_2(k)$, then the complex $F_* \Omega^\bullet_X$ is quasi-isomorphic to a complex with zero differentials. -4 (Buch-Thomsen-Lauritzen-Mehta 1995). If $X$ is strongly Frobenius split (that is, $0\to B_1\to Z_1\to \Omega^1_X\to 0$ splits, where $Z_i$ and $B_i$ are cocycles/coboundaries in $F_* \Omega^\bullet_X$), then $X$ and $F$ lift to $W_2(k)$ and the Bott vanishing theorem holds for $X$. - -My (maybe incorrect) feeling is that strong Frobenius splitting and lifting of the Frobenius to $W_2(k)$ are quite uncommon, Frobenius splitting is a common behavior "on the Fano side" and that lifting of $X$ to usually $W_2(k)$ exists. - -Question. Are there examples of Frobenius split varieties for which $F_* \Omega^\bullet_X$ is not quasi-isomorphic to a complex with zero differentials (for example, because the Hodge spectral sequence does not degenerate, see also this question on the Hodge spectral sequence)? If yes (that's my intuition here), does Frobenius splitting imply some weaker property of $F_* \Omega^\bullet_X$ which implies Kodaira vanishing? - -Edit. Note that Frobenius splitting just states that the complex $F_* \Omega^\bullet_X$ is quasi-isomorphic to a complex whose first differential $C^0 \to C^1$ is zero. - -REPLY [4 votes]: I just found a nice reference for this question: K. Joshi "Exotic Torsion, Frobenius Splitting and the Slope Spectral Sequence" (Canad. Math. Bull. Vol. 50 (4), 2007), section 9. -Theorem 9.1 (unpublished work of V. B. Mehta). Let X be a smooth, projective, F-split variety over an algebraically closed field $k$ of characteristic $p>0$. Then for all $i+j < p$, the Hodge to de Rham spectral sequence degenerates at $E^{i,j}_1$. In particular, for $i+j = 1$ we have the following exact sequence -$$ 0\to H^0(X, \Omega^1_X)\to H^1_{DR}(X/k)\to H^1(X, \mathcal{O}_X)\to 0. $$ -Moreover, any F-split variety with $dim(X)
-TITLE: What elementary problems can you solve with schemes? -QUESTION [223 upvotes]: I'm a graduate student who's been learning about schemes this year from the usual sources (e.g. Hartshorne, Eisenbud-Harris, Ravi Vakil's notes). I'm looking for some examples of elementary self-contained problems that scheme theory answers - ideally something that I could explain to a fellow grad student in another field when they ask "What can you do with schemes?" -Let me give an example of what I'm looking for: In finite group theory, a well known theorem of Burnside's is that a group of order $p^a q^b$ is solvable. It turns out an easy way to prove this theorem is by using fairly basic character theory (a later proof using only 'elementary' group theory is now known, but is much more intricate). Then, if another graduate student asks me "What can you do with character theory?", I can give them this example, even if they don't know what a character is. -Moreover, the statement of Burnside's theorem doesn't depend on character theory, and so this is also an example of character theory proving something external (e.g. character theory isn't just proving theorems about character theory). -I'm very interested in learning about similar examples from scheme theory. - -What are some elementary problems (ideally not depending on schemes) that have nice proofs using schemes? - -Please note that I'm not asking for large-scale justification of scheme theoretic algebraic geometry (e.g. studying the Weil conjectures, etc). The goal is to be able to give some concrete notion of what you can do with schemes to, say, a beginning graduate student or someone not studying algebraic geometry. - -REPLY [3 votes]: In line with Felipe Voloch's remark "Spec $\mathbb{Z}$ are where schemes really shine", I thought I'd also add this beautiful (expository and very readable!) paper of Serre: -"How to use finite fields for problems concerning infinite fields" -which makes crucial use of being able to do algebraic geometry over (finitely generated algebras over) $\mathbb{Z}$ in order to prove geometric statements (many of which are easily understable without any background in algebraic geometry!) over fields like $\mathbb{C}$ and $\mathbb{Q}$.<|endoftext|> -TITLE: Bivariate polynomials with special properties -QUESTION [7 upvotes]: I recently came across some polynomials with some remarkable properties. -A polynomial $P(u,v) \in \mathbb{R}[u,v]$ in 2 variables is remarkable if -the set of solutions to the system $P(u,v)=P(v,u)=0$ is a finite number of points in $\mathbb{C}^2$ such that each point is of the form $(x, \overline{x}).$ -Here are some examples of such polynomials: -$v^2-u$ -$-2 u v+v^3+1$ -$u^2-3 u v^2+v^4+2 v$ -$3 u^2 v-4 u v^3-2 u+v^5+3 v^2$ -$-u^3+6 u^2 v^2-5 u v^4-6 u v+v^6+4 v^3+1$ -$-4 u^3 v+10 u^2 v^3+3 u^2-6 u v^5-12 u v^2+v^7+5 v^4+3 v$ -$u^4-10 u^3 v^2+15 u^2 v^4+12 u^2 v-7 u v^6-20 u v^3-3 u+v^8+6 v^5+6 v^2$ -A quite complicated algorithm is behind this sequence, any help identifying -a formula for these polynomials would be helpful. -Also, a proof that these polynomials are remarkable would be nice, -these are only checked by numeric computations in Mathematica. -Question: Can one classify the set of remarkable polynomials? - -REPLY [3 votes]: Your sequence $p_n (u,v)$ can be defined by $p_n (u,v) = vp_{n - 1} (u,v) - up_{n - 2} (u,v) + p_{n - 3} (u,v)$ with initial values $p_0 (u,v) = 1,p_1 (u,v) = v,p_2 (u,v) = v^2 - u.$ -In my above remark I have overlooked a term in the fifth polynomial. This is now the same as the formula given by ARupinski. -Added later: -Extend the sequence $p_n (u,v)$ to negative indices by $p_{ - 1} (u,v) = p_{ - 2} (u,v) = 0$ and $p_{ - n} (u,v) = p_{ n - 3} (v,u)$ for $n >2 .$ -Define a new sequence of polynomials $r_n (u,v)$ by the same recurrence and initial values $r_{ - 1} (u,v) = 1,r_0 (u,v) = 0,r_1 (u,v) = - u.$ Extend it to negative values by $r_{ - n} (u,v) = r_{n - 2} (v,u).$ -Let $A$ be the matrix with rows $(0,1,0),(0,0,1),(1, - u,v).$ -Then $A^n$ is the matrix with the following rows: $\left( {p_{n - 3 + j} (u,v),r_{n - 2 + j} (u,v),p_{n - 2 + j} (u,v} \right)$ for $0 \le j \le 2.$ -It seems that the sequence $r_n (u,v)$ or the sequence $r_{2n} (u,v)/(1 - uv)$ has analogous properties with respect to the zeroes. Is it also related to the group representation?<|endoftext|> -TITLE: Motivation for and history of pseudo-differential operators -QUESTION [54 upvotes]: Suppose you start from partial differential equations and functional analysis (on $\mathbb R^n$ and on real manifolds). Which prominent example problems lead you to work with pseudo-differential operators? -I would appreciate any good examples, as well as some historical outlines on the topic's development. (Shubin's classical book spends a few lines on history and motivation in the preface, but no "natural" examples. I am not aware of any historical outlines in the literature.) - -REPLY [3 votes]: Disclaimer. This answer stems from a draft of an entry on the symbol of a singular integral operator I am writing for the Wikipedia, thus it is by no means complete. -From the sources I have read until now (for example Gaetano Fichera's papers [1], p. 475 and [2], pp. 52-54 and Vladimir Maz'ya's book [5], p. 143), it seems that the motivation for the development of the theory of pseudodifferential operators lies in the development of the theory of singular integral operators, emerged as means for solving variable coefficients PDEs: precisely, Fichera and Maz'ya identify the starting point of the theory with the discover of the symbol of a singular integral operator made by Solomon Mikhlin. -Short answer -While studying the $3$-dimensional potential generated by a plane thin disk (of arbitrary shape), Francesco Tricomi succeeded for the firs time ever in giving a closed form expression for the composition of two $2$-dimensional (constant coefficient) singular integral operators (see [11], [12]). Tricomi's work led Solomon Mikhlin (see [5], pp. 2124-2125, [6], pp. 535-536 and for a brief but fairly complete historical survey see [8], §1.1.4 pp. 8-10), followed by Georges Giraud, to the discovery of the concept of the symbol of a singular integral operator. Be it noted that at that time, due to the form in which it was discovered, it was not clear that the symbol was strictly related to the Fourier transform the kernel respect to the integration variable, therefore it was not clear its relation with the Fourier transform of partial derivatives: however, its discovery made clear that the symbol is the key to uncover the algebraic structure of the composition of such operators and the structure of their spaces. Later on, Solomon Mikhlin ([2], p. 54 and [8], §1.1.7 p. 11) discovered that the symbol is the partial Fourier transform, respect to the integration variable, of the kernel of the singular integral operator. The use of the machinery of multidimensional Fourier integrals, developed by Alberto Calderón and Antoni Zygmund, became thus one of the strong points of the theory, and finally Kohn & Niremberg and Hörmader synthesized operator algebras which include both linear partial differential and singular integral operators: the theory of pseudodifferential operators was born. -A more detailed survey - -Background on PDE theory at the beginning of the twentieth century: reduction to integral equations. -One of the more extensively used techniques for the solution of PDEs which was developed at the beginning of the 20th century was the "Method of Potentials": it stemmed from the methods developed the in the nineteenth century for solving Laplace's equation and consist in representing the solution of a given problem as the sum of properly chosen potential type integrals (volume, single and double layer potentials) (see [8], Chapter III, p. 39 and §18, pp. 44-49 and [9], Chapter III, p. 49 and §18, pp. 54-59). These integral representations are obtained for example by using Hadamard's elementary solutions and Levi functions (now called parametrices): the set of integral equations obtained is hopefully analyzable by applying Fredholm's theory. -However, this is not always so. These integral equations are, as a rule, often singular i.e. do not exist in the ordinary sense but only in the sense of their principal values: in this case, Fredholm's theory cannot be applied directly. This happens, for example, for the Oblique Derivative Problem for Laplace's equation, as Henri Poincaré and Gaston Bertrand noted in their analysis of the problem of tides, de facto the first oblique derivative ever posed (see for example [3], pp. 251-252). Thus the time was ready for a theory of singular integral equations to be developed. - -The symbol of a singular integral: Tricomi, Mikhlin and Giraud. -Tricomi, while studying problem of determining the harmonic potential of a plane thin disk (of arbitrary shape) immersed in $\Bbb R^3$, found a formula for the explicit calculation of the composition of two $n$-dimensional (precisely $2$-dimensional) singular integral operators (see [11] and [12], §4 pp. 107-112 and also [8] §1.2, pp. 2-4). Building on the the work of Tricomi, Mikhlin discovered the concept of symbol in the form of a complex Fourier series (see Fichera [2], p. 53): let -$$ -Su(z)=\iint u(\zeta)K(z,z-\zeta)\mathrm{d}\xi\mathrm{d}\eta\quad z=x+iy,\;\zeta=\xi +i\eta,\;u\in L^p\label{1}\tag{1} -$$ -be a syngular integral operator whose kernel has the form -$$ -K(z,\zeta)=\frac{1}{|\zeta|^2}f\left(z\frac{\zeta}{|\zeta|}\right), -$$ -and let -$$ -f(z,\theta)=\sum_{h\in\Bbb Z\setminus \{0\}} c_h(z)e^{ih\theta}. -$$ -where the zero order term is missing since the condition -$$ -\int\limits_{-\pi}^{+\pi} f(z,e^{i\theta})\mathrm{d}\theta=0 -$$ -must be fulfilled in order for the integral \eqref{1} to exists if, for example, $u\in\operatorname{Lip}$. Then Mikhlin defines the symbol of $S$ as -$$ -\sigma(z,\zeta)=\sum_{h\in\Bbb Z\setminus \{0\}} c_h(z)\frac{-i^{|h|}}{|h|} - e^{ih\theta}. -$$ -In the paper [5] he shows how to extend the results of [4] to singular integrals defined on a closed surface. -Georges Giraud, who was the leading mathematician in the development of the method of potentials (to the point that, according to Carlo Miranda ([7], p. 44 and [8], p. 54) he provided "the most general and definitive contributions"), was consequently quite interested in the analysis of singular integrals. -Closely after the appearance of the works [4] and [5], Giraud published the note [3] in the Comptes rendus, giving (without proof) formulas for the symbol of a singular integral operator, expressed in the form of a series of harmonic polynomials, and for the composition of two singular operators that extends the work of Mikhlin to the $(n\ge 2)$-dimensional case: nor Giraud's nor Mikhlin's definitions of the symbol rely on multidimensional Fourier transform theory. -A proof of Giraud's formulas was given later by Mikhlin ([6], §1.4, p. 9), and only later, in 1956 ([2], p. 54 and [8], §1.1.7 p. 11) he also proved that the symbol is the partial multidimensional Fourier transform respect to the integration variable of the singular integral, connecting his theory to the one Alberto Càlderon and Antoni Zygmund were developing in the same years. - - -References. -[1] Fichera, Gaetano, "Francesco Giacomo Tricomi", Atti della Accademia Nazionale dei Lincei. Serie VIII. Rendiconti. Classe di Scienze Fisiche, Matematiche e Naturali 66, pp. 469-483 (1979), MR0606447, Zbl 0463.01022. -[2] Fichera, Gaetano, "Solomon G. Mikhlin (1908-1990)", Atti della Accademia Nazionale dei Lincei, Rendiconti Lincei, Matematica e Applicazioni, Serie XI, Supplemento 5, pp. 49-61, 1 plate (1994), Zbl 0852.01034. -[3] Giraud, Georges, "Équations à intégrales principales; étude suivie d’une application", Annales Scientifiques de l'École Normale Supérieure. (3) 51, pp. 251-372 (1934), MR1509344, Zbl 0011.21604. -[4] Giraud, Georges, "Sur une classe générale d’équations à intégrales principales", Comptes rendus hebdomadaires des séances de l'Académie des sciences, Paris 202, pp. 2124-2127 (1936), JFM 62.0498.01, Zbl 0014.30903. -[5] Maz’ya, Vladimir, Differential equations of my young years. Translated from the Russian by Arkady Alexeev, Cham: Birkhäuser/Springer (ISBN 978-3-319-01808-9/hbk; 978-3-319-01809-6/ebook), pp. xiii+191 (2014), MR3288312, Zbl 1303.01002. -[6] Mikhlin, Solomon G., "Equations intégrales singulieres à deux variables independantes", Recueil Mathématique (Matematicheskii Sbornik) N.S. (in Russian), 1(43) (4), pp. 535-552 (1936), JFM 62.0495.02, Zbl 0016.02902. -[7] Mikhlin, Solomon G., Complément à l’article “Equations intégrales singulières à deux variables indépendantes”, Recueil Mathématique (Matematicheskii Sbornik) N.S. (in Russian), 1(43) (6), pp. 963-964 (1936), JFM 62.1251.02, Zbl 62.1251.02." -[8] Mikhlin, Solomon G., Multidimensional singular integrals and integral equations. Translated from the Russian by W. J. A. Whyte. Translation edited by I. N. Sneddon. (International Series of Monographs in Pure and Applied Mathematics. Vol. 83), Oxford-London-Edinburgh-New York-Paris-Frankfurt: Pergamon Press, pp. XII+255 (1965). MR0185399, ZBL0129.07701. -[9] Miranda, Carlo, Equazioni alle derivate parziali di tipo ellittico, Ergebnisse der Mathematik und ihrer Grenzgebiete, Neue Folge, 2. Heft., Berlin- Göttingen-Heidelberg: Springer-Verlag. VIII, 222 S. (1955), MR0087853, ZBL0065.08503. -[10] Miranda, Carlo, Partial differential equations of elliptic type, Ergebnisse der Mathematik und ihrer Grenzgebiete, Band 2, Berlin-Heidelberg-New York: Springer-Verlag, pp. XII+370 (1970), MR0284700, Zbl 0198.14101. -[11] Tricomi, Francesco, "Formula d’inversione dell’ordine di due integrazioni doppie “con asterisco”"., Atti della Accademia Reale dei Lincei, Rendiconti della Classe di Scienze Fisiche, Matematiche e Naturali, (6) 3, pp. 535-539 (1926), JFM 52.0235.05. -[12] Tricomi, Francesco, Equazioni integrali contenenti il valore principale di un integrale doppio, Mathematische Zeitschrift 27, pp. 87-133 (1927), JFM 53.0359.02.<|endoftext|> -TITLE: Weierstrass' function and Brownian motion -QUESTION [5 upvotes]: Is there a known connection between Weierstrass' function -$W_\alpha (x) = \sum_{n=0}^\infty b^{- n \alpha} \cos(b^n x)$ -and Brownian motion? Specifically, when $\alpha = 1/2$, the Weierstrass function has same Holder continuity peoperties that Brownain sample paths do. Some crude Matlab experiments seem to suggest this function has linear quadratic variation for low values of $b$, too. -The expression reminds me a little of the Karhunen-loeve expansion of Brownian motion, but I don't see how the two might relate. -Many thanks. - -REPLY [3 votes]: Some quick Googling brought me to this paper. The idea is to take the coefficients in the summation to be suitable independent random variables according to a suggestion of Mandelbrot. I can't actually access the paper right now so I can't say if the authors were able to include the case of standard Brownian motion in their results. -``Convergence of the Weierstrass-Mandelbrot process to Fractional Brownian Motion'' Murad Taqqu and Vladas Pipiras. Fractals. 8 (2000) 369-384. - -REPLY [3 votes]: You might find this account useful. In particular, see the end of Section 1, page 7, and the end of Section 4.<|endoftext|> -TITLE: Looking for deterministic criteria to generate the symmetric group? -QUESTION [7 upvotes]: So let $S_N$ be the symmetric group of degree $N$. We think of it as a permutation group via its -natural action on the set $T=\{1,2,\ldots,N\}$. -Say that $H\leq S_N$ is a subgroup which acts transitively on $T$. However, I DONT'T WANT to assume necessarily that $H$ is primitive (that is the whole point of my question). Assume furthermore that there is an onto group homomorphism -$$ -f:H\rightarrow S_n -$$ -where $n=\lfloor{N/2}\rfloor$. In fact, as was pointed out by Schmidt, the existence of this onto group homomorphism implies that $H$ is imprimitive. -In general, one cannot rule out the existence of such an $H$. For example -one could have $H=S_n\ltimes\mathbf{F}_2^n$ where $N$ is even and $n=\frac{N}{2}$. -We let $H$ act on $T$ in the following way: We divide $T$ in $n$ disjoint blocks of size $2$. We let $S_n$ permute the $n$ blocks without swapping the pair in each block, and we let $\mathbf{F}_2^n$ permute (resp. acts like the identity) the two elements in the i-th block if the i-th coordinate of an element $\sigma\in \mathbf{F}_2^n$ is $\overline{1}$ (resp. $\overline{0}$). It thus follows that $H$ acts transitively (but imprimitively) on $T$. -Furthermore, suppose that I can produce " a lot of elements " in $H$ which contain a cycle of length $r$ in their cycle presentations (their writing as a product of disjoint cycles of $T$) for $r>n$. Then may I conclude that such an $H$ does not exist? -Q1: Is there some kind of results that would allow me to conclude that $H\supseteq A_N$, so that this would contradict the imprimitivity and therefore rule out the existence of such an $H$? -For example here is one key result which is good to know: if $H$ is assumed to be primitive and contains a cycle of length $\ell$ with $2\leq \ell\leq N-7$ ($\ell$ not necessarily prime) then combining classical results on permutation group theory one may show that $H\supseteq A_N$. However, since in my setting $H$ is imprimitive I cannot apply this result. -Q2: Do we have a good understanding of the tree of subgroups of $S_N$, especially -the maximal subgroups? -Q3: Is there some kind of probabilistic result that could be used in my context? - -REPLY [3 votes]: Embarrassingly, the posting below contains another incomplete proof, as was pointed out by Hugo Chapdelaine. I am working on a new (third) version, but after two wrong proofs, I want to hold off for a while on posting it until it is written up carefully and checked. What I am trying to prove is this: -THEOREM: Let $H$ be a transitive subgroup of $S_N$, and suppose -$M \triangleleft H$ and $H/M \cong S_n$, where $N/2 \le n < N$ and -$5 < n$. Then $n = N/2$ and either $M = 1$ or all orbits of $M$ have size $2$. - -Unfortunately, the proof in the original version of this post had -a gap that I do not see how to repair. The following weaker -result seems to be true, however. -THEOREM. Let $H \subseteq S_N$ and assume that $M \triangleleft H$ and $H/M \cong S_n$, where $n \ge N/2$. Then either $M$ is an elementary abelian $2$-group and $n = N/2$ and $H$ is transitive, or else there exists $K \subseteq H$ such that $H = MK$ and -$M \cap K = 1$. In particular, $K \cong S_n$. -Proof. Induct on $N$. Let $S$ be a point stabilizer in $S_N$. Write -$u = |H:(H \cap S)|$, so $u \le N$ with equality only if $H$ is transitive. -Write $v = |M:(M \cap S)|$, so $v = |M(H \cap S):(H \cap S)|$, and this divides $u$. Also, $u/v = |H:M(H \cap S)|$ is the index of a subgroup of -$H/M \cong S_n$, so this index is either $1$ or at least $n$. -Suppose $u/v = 1$. Then $M(H \cap S) = H$, and so -$(H \cap S)/(M \cap S) \cong H/M \cong S_n$. Also, $S \cong S_{N-1}$ and $(N-1)/2 < n$, so the inductive hypothesis applies with $H \cap S$ in place of $H$ and $M \cap S$ in place of $M$. Since $n$ is not $(N-1)/2$, there exists $K \subseteq H \cap S$ such that $K \cap (M \cap S) = 1$ and $H \cap S = (M \cap S)K$. Then $K \cap M = 1$ and -$H = M(H \cap S) = M(M \cap S)K = MK$ as required. -We can assume now that for every choice of point stabilizer $S$ we have -$u/v \ne 1$. Then $N/v \ge u/v \ge n \ge N/2$, and thus $v \le 2$. Thus all orbits of $M$ have size $1$ or $2$, so $M$ is an elementary abelian 2-group. If we always have $v = 1$, then $M = 1$ and we can take $K = H$. We can thus assume that $v = 2$ for some choice of $S$. Then $N/2 \ge u/2 = u/v \ge n \ge N/2$ and we have equality. Thus $n = N/2$ and $u = N$, and the latter equality shows that $H$ is transitive. QED - -In the case where $H = MK$ and $M \cap K = 1$, we have $K \cong S_n$, so we can ask what copies $K$ of $S_n$ are contained in $S_N$ if -$n \ge N/2$. If $n > 5$, It seems that the only possibility is that $K$ is the stabilizer of $N - n$ points. This can be proved using the fact that $S_n$ has no proper subgroup of index less than or equal to $2n$ except for point stabilizers. (At least I think that is a fact.)<|endoftext|> -TITLE: Bimodules over division rings -QUESTION [7 upvotes]: Inspired by other questions i have two questions about modules over division rings: given a division ring $D$ with center $Z(D)=K$. One has the notion of dimension for left modules (vector spaces) $V$ over $D$, which is well behaved like in linear algebra. -But does the following also hold: Given a left vector space $W$ of dimension $n$ and a right vector space $V$ of dimension $m$. Assume that at least one of them is even a $D$-bimodule, so that $V\otimes_D W$ has the structure of a $D$-module. -$Q1$: Do we have $dim_D(V\otimes _D W)=dim_D(V)dim_D(W)=nm$ in this case? -Now $D$ is naturally a $D$-bimodule, so is $D^{\*}=Hom_K(D,K)$. -$Q2$: Is there a $D$-bimodule isomorphism between $D$ and $D^{\*}$? -Both modules are one dimensional, so pick generators $x\in D$ and $y\in D^{\*}$. Now $x\mapsto y$ defines a map $D \rightarrow D^{\*}$, which satisfies $ax\mapsto ay$. Assume that $ax=xb$ for some $b\in D$, then we need to have $xb\mapsto yb$, which leads to $x^{-1}ax=y^{-1}ay$ for all $a\in D$. Can we find $x$ and $y$ such that the last relation is fulfilled? Or is there no such bimodule isomorphism? - -REPLY [12 votes]: The answer to both questions is positive if $D$ is finite-dimensional over its center $K$, and negative, in general, otherwise. -Q1. Suppose $V$ is a $D$-bimodule over $K$ (i.e., a $D\otimes_K D^{op}$-module), while $W$ is a left $D$-module of dimension $n$, as in your question. Then $W$ is isomorphic to a direct sum of $n$ copies of $D$, hence $V\otimes_D W$ as a left $D$-module is isomorphic to a direct sum of $n$ copies of $V$. So the $\dim_D (V\otimes_D W) = n\dim_D V$, where $\dim_D$ denotes the dimension of left $D$-modules. Now, to answer your original question, it remains to notice that the dimensions of $V$ as a left and a right $D$-module coincide, since both are equal to $\dim_KV/\dim_KD$. -It does not matter in this argument that $K$ is the whole center of $D$, only that $K$ is a field contained in the center of $D$ and $D$ is finite-dimensional over $K$. On the other hand, if $D$ is infinite-dimensional over $K$, it may happen that $D$ can be $K$-linearly embedded into itself as a proper subring. This would allow to define a $D$-bimodule structure on $V=D$ where $V$ is one-dimensional as a right module over $D$ and more than one-dimensional (possibly infinite-dimensional) as a left module over $D$. This would break your dimension formula. -Q2. Left $D$-module maps $D\to D^\ast$ are in one-to-one correspondence with elements of $D^\ast$, i.e., $K$-linear functions $D\to K$. A linear function $l\colon D\to K$ defines a $D$-bimodule map $D\to D^\ast$ if and only if one has $l(xy)=l(yx)$ for all elements $x$, $y\in D$. Taking $l$ to be the trace map (defined e.g. by tensoring $D$ with the separable closure of $K$ over $K$, identifying the result with matrices over the separable closure, and taking the traces of matrices), one can obtain a $D$-bimodule isomorphism between $D$ and $D^\ast$ when $D$ is finite-dimensional over its center $K$. -The above argument can be easily extended to the case when $K$ is just a subfield of the center of $D$ and $D$ is finite-dimensional over $K$ (one just composes the trace map with an arbitrary nonzero linear function from the center of $D$ to $K$). On the other hand, when $D$ is infinite-dimensional over $K$, it is no longer true that $D^\ast$ is a one-dimensional left $D$-module.<|endoftext|> -TITLE: A density on the natural numbers invariant with respect to the multiplication -QUESTION [9 upvotes]: The "classical Beurling density" of a subset of the natural numbers is $d(A)=lim_{n\rightarrow\infty}\frac{|A\cap[1,n]|}{n}$, when it exists. It defines a finitely additive probability measure on the natural numbers which is invariant with respect to the sum. Here is my question: does there exist a "nice formula" to describe a finitely additive probability measure on $\mathbb N$ which is invariant with respect to the multiplication? -A couple of remarks: I don't know if it is trivial that such a measure exists, but anyway it follows from the application of a general result of Vern Paulsen (on arxiv "Syndetic sets and amenability"). -Another problem would be that of finding the measure of particular sets. What about the measure of {$1!,2!,3!,4!...$}? Sets with measure differente from 0 and 1? For example the set of numbers whose first digit through 4 to 9 seems to have measure $=\log_{10}4$... any other? -Thanks in advance, Valerio - -REPLY [7 votes]: The natural thing to do here is to replace the intervals $[1,n]$ (and $n = |[1,n]|$) in the definition of $d(A)$ with a sequence $F_n$ of subsets of $\mathbb{N}$ which is multiplicatively asymptotically invariant (or, in other words, a Folner sequence for the semigroup $(\mathbb{N},\cdot)$). For an exploration of this idea, as well as applications, see for instance this article by Vitaly Bergelson: -Multiplicatively large sets and ergodic Ramsey theory, Israel Journal of Mathematics 148 (2005), 23-40. -EDIT: One particular example (mentioned in the article) is to take $F_n$ to be the set of all positive integers which can be written as a product of powers of the first $n$ primes, where the powers are allowed to be any non-negative integer which is less than or equal to $n$. The motivation for choosing $F_n$ this way is that just as $1$ generates the additive semigroup $(\mathbb{N},+)$, the primes generate the multiplicative semigroup. Think of balls in the corresponding Cayley graphs with radius getting larger and larger. The article contains many other examples of such $F_n$, and each of them gives a notion of "multiplicative density" by setting $d(A) = \lim_{n \to \infty} \frac{|A \cap F_n|}{|F_n|}$ (if the limit exists. Otherwise one usually considers the limsup and liminf).<|endoftext|> -TITLE: local class field theory (Norm map) -QUESTION [9 upvotes]: Let $K$ be a local field, for example the $p$-adic numbers. In Neukirch's book "Algebraic number theory", there is the statement: if $K$ contains the $n$-th roots of unity and if the characteristic of $K$ does not divide $n$, and we set $L=K(\sqrt[n]{K^{\times}})$, then one has $N_{L/K}(L^{\times})=K^{\times n}$. -My questions are the following, for a (finite) Galois extension $L$ of $K$:\ -(1) What happens if the characteristic of $K$ divides $n$? Can one obtain an explicit form of the image of the norm of $L^\times$?\ -(2) If we don't add the condition that $K$ contains the $n$-th roots of unity, what is the image of the norm operator? If $L = K(x)$, can one write it in terms of the primitive element $x$ and $K^{\times n}$?\ - -REPLY [7 votes]: Here is another interesting case where the image of the norm map can be written down explicitly. Let $F$ be a finite extension of $\mathbf{Q}_p$ containing a primitive $p$-th root $\zeta$ of $1$, and denote the filtration on the multiplicative group $F^\times$ by -$$ -\ldots\subset U_2\subset U_1\subset\mathfrak{o}^\times\subset F^\times. -$$ -We thus get the extensions $L_n=F(\root p\of{U_n})$. It can be checked that $L_{pe_1+1}=F$, where $e_1$ is the absolute ramification index of $F$ divided by $p-1$, and that $L_{pe_1}$ is the unramified degree-$p$ extension of $F$, so the image of the norm map $L_{pe_1}^\times\to F^\times$ is $\mathfrak{o}^\times F^{\times p}$. -What is the image of the norm map $L_{n}^\times\to F^\times$ for other $n$ ? Local class field theory and a certain orthogonality relation for the Kummer pairing (see for example the last section of arXiv:0711.3878) can be used to answer this question. Basically, for $n\in[1,pe_1]$, $N(L_n^\times)=U_mF^{\times p}$, where $m=pe_1+1-n$. -There are similar results for elementary abelian $p$-extensions of finite extensions of $\mathbf{F}_p((t))$. See for example the last section of arXiv:0909.2541. -These two papers have appeared in J. Ramanujan Math. Soc. 25 (2010), no. 1, 25–80, -and 25 (2010), no. 4, 393–417. -There are other instances where the image of the norm map can be computed explicitly. This happens for the cyclotomic extension $K_m$ of $\mathbf{Q}_p$ obtained by adjoining $\root{p^m}\of1$. It can be shown that $p\in N(K_m^\times)$, and that the image of the units of $K_m$ under the norm map down to $\mathbf{Q}_p$ is $1+p^m\mathbf{Z}_p$. See for example Artin, -Algebraic numbers and algebraic functions, p. 208, or Neukirch, -Class Field Theory, p. 45.<|endoftext|> -TITLE: Assigning positive edge weights to a graph so that the weight incident to each vertex is 1. -QUESTION [7 upvotes]: Let $\Gamma=(G,E)$ be a connected undirected graph, with no loops or multiple edges. $G$ is finite or countably infinite. For each edge $e=\{x,y\}\in E$, we assign a positive, symmetric edge weight $c_e := c_{\{x,y\}} = c_{xy} = c_{yx}$. I would like to know for which graphs $\Gamma$ it is possible to choose $(c_e)_{e\in E}$ so that for each $x\in G$, -\begin{equation*} -\sum_{y\sim x} c_{xy} = 1. -\end{equation*} -For example, this is possible on any $d-$regular graph if one sets $c_e \equiv 1/d$. The graph with vertex set $\{x,y,z\}$ and edges $\{x,y\}$ and $\{y,z\}$ shows that it is not always possible. - -REPLY [8 votes]: Here is a solution along the lines of JBL's answer. -First a couple of definitions: - -A disjoint cycle cover of a graph is a collection of cycles of our graph which are disjoint subgraphs, and contain all the vertices of our graph. A special case of a disjoint cycle cover is a perfect matching, for instance. -We will call the the permanent of a graph, the permanent of its adjacency matrix - -An easy fact is that the permanent of a graph counts its disjoint cycle covers. -Now, to our result: - -Theorem: A graph admits edge weights as in the problem if and only if every edge is contained in a disjoint cycle cover. Equivalently if and only if removing an edge decreases the permanent. - -proof: -Suppose the graph $G$ has such weights. Then the matrix $A$ with $a_{ij}$ being the weight of the edge connecting vertices $v_i$ and $v_j$, is doubly stochastic, and thus by the Birkhoff-von Neumann theorem can be written as a convex combination of permutation matrices. For every edge of $G$, there is a non zero term $a_{ij}$ in $A$ which means that there is a permutation matrix in our sum with a $1$, in the $ij$ entry, call this matrix $M_{ij}$. The first observation is that $M_{ij}$ has all zeros on the diagonal, and secondly that all it's non-zero entries correspond to edges in $G$. This collection of edges is of course a disjoint cycle cover. -Now for the other direction, each cycle cover can be assigned weights as in the problem (just assign $1/2$ to all edges in proper cycles and $1$ to all isolated edges). So taking an appropriate convex combination of all such covers gives us weights for $G$. -A special case is of course when all edges are contained in perfect matchings, but this property doesn't characterize all graphs as in the question, as the example I gave in the comment to JBL's answer shows (also just look at odd cycles). Which is why one must include more general cycle covers. - -Perhaps it is a bit more clear if we phrase it in the following way. When restricting to bipartite graphs, the property of each edge being contained in a disjoint cycle cover is equivalent to every edge being in a perfect matching (there are no odd cycles). Now the result above follows because weights on our graph induce weights on its bipartite double cover which sum to 1 at each vertex. A disjoint cycle cover of a graph is equivalent to a perfect matching of its bipartite double cover.<|endoftext|> -TITLE: Dual space of $\ell^\infty$ -QUESTION [8 upvotes]: Why can the elements of the dual space of $\ell^\infty(\mathbb N)$ be represented as sums of elements of $\ell^1(\mathbb N)$ and Null$(c_0)$? - -EDIT: As confirmed in the comments, the OP intended to ask about this sentence -"$f\in\ell_\infty^*$ is the sum of an element of $\ell_1$ and an element null on $c_0$" from the paper D. H. Fremlin and M. Talagrand: A Gaussian Measure on $l^\infty$ http://jstor.org/stable/2243023 (Which is different claim from what was in the original version of the question.) - -REPLY [6 votes]: Let's recall a simple, elementary, and general fact that hasn't been explicitly mentioned: a dual Banach space is always a splitting subspace in the isometric embedding into its double dual. -Let $i_X:X\to X^{**}$ denote the natural isometric embedding of $X$ in $X^{**}$. If we dualize, we have a transpose operator, $i_X^*:X^{***}\to X^*$ (that we may identify as the restriction map, which takes a linear form on $X^{**}$ to its restriction on $X$ as a subspace of $X^{**}$). On the other hand we also have the isometric embedding $i_{X^*}:X^*\to X^{***}$. It is a straightforward (though a bit formal) computation checking that $i_{X}^*$ is left-inverse to $i_{X^*}$, that is $i_{X}^*i_{X^*}=1_{X^*}.$ As a consequence of this, $P:=i_{X^*}i_{X}^*$ is a linear projector with $\operatorname{ker}P=\operatorname{ker}i_X^*=X^\perp$ corresponding to the splitting $X^{***}=X^*\oplus X^\perp$. -$$*$$ -Checking the identity $i_{X}^*i_{X^*}=1_{X^*}.$ This means $i_{X}^*i_{X^*}f=f$ for all $f\in X^*$, which also means $\langle i_{X}^*i_{X^*}f, x\rangle=\langle f, x\rangle$ for all $x\in X$ and $f\in X^*$. Indeed -$$\langle i_{X}^*i_{X^*}f, x\rangle_{X^*\times X}=\langle i_{X^*}f, i_{X} x\rangle_{X^{***}\times X^{**}}=\langle i_{X} x,f\rangle_{X^{**}\times X^*}=\langle f, x\rangle_{X^*\times X},$$ -by the definition of the transpose operator $i_{X}^*$, respectively by the definition of the embeddings $i_{X^*}$ and $i_{X}$.<|endoftext|> -TITLE: Galois Bicentennial? -QUESTION [20 upvotes]: The 200-th anniversary of the birth of Galois will be on October 25th, 2011. For Abel's bicentennial birth year in 2002, Norway had a big conference and initiated the Abel prize. A cursory web search doesn't reveal any major (or minor) conference to mark the bicentennial for Galois. Is there something being planned that hasn't been broadly announced yet? -[Edit: Before posting this question, I looked at the number theory conference page http://www.numbertheory.org/ntw/N3.html and found nothing there related to my question.] -I realize that is not a math research question, but I don't think it should be closed since it should interest many users of this site, it should have a definite answer, and it is not "argumentative", "too localized" (at what prime?) or "not a real question". -[Edit: Now that Emerton has answered the question well and nobody else has followed up after several days with other replies, I suspect he has given the best possible answer and I have designated it as such.] - -REPLY [3 votes]: There is also some events (exhibitions, large audience talks) in Bourg la Reine, the town where Galois grew up (near Paris). -Parlons d'Évariste Galois<|endoftext|> -TITLE: Reference Request: Independence of the ultrafilter lemma from ZF -QUESTION [10 upvotes]: I'm looking for references for the following facts concerning the ultrafilter lemma (~ "there exist non-principal ultrafilters"): - -The ultrafilter lemma is independent of ZF. -ZF + the ultrafilter lemma does not imply the Axiom of Choice. - -I would prefer an overview article / book that links to the original papers instead of the original papers themselves. That's because I don't rely on these facts; I only want to give some context for my readers. - -REPLY [22 votes]: In any of the formulations mentioned (so far) in the comments, the ultrafilter lemma is independent of ZF but weaker than AC. That the strongest form (all filters can be extended to ultrafilters) doesn't imply AC is a theorem of J.D. Halpern and A. Lévy ["The Boolean prime ideal theorem does not imply the axiom of choice" in Axiomatic Set Theory, Proc. Symp. Pure Math. XIII part 1, pp. 83-134]. That ZF doesn't prove even the weakest form (there exists a nonprincipal ultrafilter on some set) is a theorem of mine ["A model without ultrafilters," Bull. Acad. Polon. Sci. 25 (1977) pp. 329-331], building on S. Feferman's construction of a model with no non-principal ultrafilters on the set of natural numbers ["Some applications of the notions of forcing and generic sets," Fundamenta Mathematicae 55 (1965) pp. 325-345].<|endoftext|> -TITLE: Example of an infinite index subgroup of a non-amenable group whose normalizer is of non-zero finite index, and such that the Schreier graph is of subexponential growth -QUESTION [6 upvotes]: EDIT: In this question I forgot to put one of the assumptions, and the question was easier than it should be. Here is the revised question. Please vote to close this question as it is no longer relevant. - - -Question. Let $G$ be a finitely generated non-amenable discrete group, and $H$ be a subgroup of $G$ of infinite index. Can it happen that the index of the normalizer $N(H)$ of $H$ in $G$ is finite greater than $1$, and the Schreier graph of $G/H$ has subexponential growth? - -If the answer is yes, I would very much like to see an example. It would be especially nice if $G$ could be taken to be a property $(T)$ group. - -REPLY [9 votes]: The answer to the question is yes. Here are two examples, the first one answers the question as it is posted and in the second one I assume that $N(H)$ is the normal closure of $H$ in $G$. The second example seems to be more interesting, in the sense that one have to do more computation on Schreier graph, compering to the first example, where the computation of the growth of the Schreier graph of $G/H$ is straightforward. - -If $N(H)$ is normalizer of $H$, then the following example works. $G:=\mathbb{Z}\times\mathbb{F}_2$ and $H$ is a subgroup $(e,H_0)$, where $H_0$ is of finite index but not normal in $\mathbb{F}_2$. Then index of $N(H)=\mathbb{Z}\times N_{\mathbb{F}_2} (H_0)$ is finite and not equal to $1$, since $N_{\mathbb{F}_2}(H_0)\neq \mathbb{F}_2$, because $H_0$ is not normal in $\mathbb{F}_2$. -If $N(H)$ is normal closure of $H$ in $G$, then the following example works. Let $\mathbb{F}_2$ be the free group on two generators $a$ and $b$. Define a homomorphism $\phi:\mathbb{F}_2\rightarrow Aut(\mathbb{Z})=\mathbb{Z}_2$ by $\phi(a)=0,\phi(b)=1$. Then the question is valid for the semidirect product $G=\mathbb{F}_2\ltimes_{\phi} \mathbb{Z}$. The Schreier graph of $G/H$ has polynomial growth and the normal closure of $H$ is $\mathbb{F}_2\ltimes_{\phi} 2\mathbb{Z}$.<|endoftext|> -TITLE: Wiener Sausages in Riemann Surfaces -QUESTION [11 upvotes]: Let $M$ be a Riemann surface (or a higher dimensional manifold) and let's assume that it's geodesically complete. Let $W(t)$ be a Brownian motion on the surface accordingly to the manifold's Laplacian and let $r>0$. -Define the Wiener sausage as: -$$ -W_{r}(t):=\{ x\in M: d(x,W(s))\leq r\quad\text{for}\quad 0\leq s\leq t \}. -$$ -It is known that in $\mathbb{R}^{2}$ and for t sufficiently large and $r$ fixed -$$ -\mathbb{E}[\mathrm{vol}(W_{r}(t))]=\frac{2\pi t}{\log(t)}(1+o(1)). -$$ -Is there any analogue result for a general Riemann surface or at least the hyperbolic space? -Thanks! ---Gabriel - -REPLY [3 votes]: I just found out that the case $r$ fixed and $t\to\infty$ for simply connected symmetric manifolds of non-positive sectional curvature and dimension $d\geq 3$, and strictly negative curvature for dimension $d=2$, was solved by Chavel and Feldman in "The Wiener Sausages and a Theorem of Spitzer in Riemannian Manifolds", Probability and Harmonic Analysis, New York, pp. 45-60, 1986.<|endoftext|> -TITLE: Up to $10^6$: $\sigma(8n+1) \mod 4 = OEIS A001935(n) \mod 4$ (Number of partitions with no even part repeated ) -QUESTION [11 upvotes]: Up to $10^6$: -$\sigma(8n+1) \mod 4 = OEIS A001935(n) \mod 4$ -A001935 Number of partitions with no even part repeated -Is this true in general? -It would mean relation between restricted partitions of $n$ and divisors of $8n+1$. -Another one up to $10^6$ is: -$\sigma(4n+1) \mod 4 = A001936(n) \mod 4$ -A001936 Expansion of q^(-1/4) (eta(q^4) / eta(q))^2 in powers of q -$\sigma(n)$ is sum of divisors of $n$. - -sigma(8n+1) mod 4 starts: 1, 1, 2, 3, 0, 2, 1, 0, 0, 2, 1, 2, 2, 0, 2, 1, 0, 2, 0, 2, 0, 3, 0, 0, 2, 0, 0, 0, 3, 2 -sigma(4n+1) mod 4 starts: 1, 2, 1, 2, 2, 0, 3, 2, 0, 2, 2, 2, 1, 2, 0, 2, 0, 0, 2, 0, 1, 0, 2, 0, 2, 2 - -Update -Up to 10^7 -A001935 mod 4 is zero for n = 9m+4 or 9m+7 -A001936 mod 4 is zero for n = 9m+5 or 9m+8 -Question about computability - -REPLY [23 votes]: Let's call A001936(n) by $a(n)$. Here is a sketch of why $$a(n)\equiv \sigma(4n+1)\pmod{4}$$ -Firs note that the generating function of $a(n)$ is -$$A(x)=\sum_{n\geq 0}a(k)x^n=\prod_{k\geq 1}\left(\frac{1-x^{4k}}{1-x^k}\right)^2$$ for $\sigma(2n+1)$ the generating function is $$B(x)=\sum_{k\geq 0}\sigma(2k+1)x^k=\prod_{k\geq 0}(1-x^k)^4(1+x^k)^8$$ -So $$B(x)\equiv \prod_{k\geq 1}(1+x^{2k})^2(1+x^{4k})^2\equiv \prod_{k\geq 1}(\frac{1-x^{8k}}{1-x^{2k}})^2\equiv A(x^2)\pmod{4}$$ -Now the proof is complete once we know that $$B(x)\equiv \sum_{k\geq 0} \sigma(4n+1)x^{2n}\pmod{4}$$ this is an other way of saying $\sigma(4n-1)$ is divisible by $4$, which can be shown by pairing up the divisors $d+\frac{4n-1}{d}\equiv 0\pmod{4}$. -The proof for the other congruence is similar, but slightly longer, I might update this post later to include it. - -Let's prove that $\sigma(8n+1)\equiv q(n)\pmod{4}$, where $q(n)$ is the number of partitions with no even part repeated. The generating function is $$Q(x)=\sum_{n\geq 0}q(n)x^n=\prod_{k\geq 1}\frac{1-x^{4k}}{1-x^k}$$ -Since we know from above that $$\sum_{n\geq 0}\sigma(4n+1)x^{2n}\equiv \prod_{k\geq 1}(1+x^{2k})^2(1+x^{4k})^2 \pmod{4}$$ we conclude that $$L(x)=\sum_{n\geq 0}\sigma(4n+1)x^n\equiv Q(x)^2 \pmod{4}$$ -so that $$\sum_{n\geq 0} \sigma(8n+1)x^{2n}\equiv \frac{L(x)+L(-x)}{2}\pmod{4}$$ -So to finish off the proof we need the following -$$\frac{Q(x)^2+Q(-x)^2}{2}\equiv Q(x^2)\pmod{4}$$ which I will leave as an exercise Actually let me write the proof, just to make sure I didn't mess up calculations. This reduces to proving -$$\frac{\prod_{k\geq 1}(1+x^{2k})^4(1+x^{2k-1})^2+\prod_{k\geq 1}(1+x^{2k})^4(1-x^{2k-1})^2}{2}$$ $$\equiv \prod_{k\geq 1}(1+x^{4k-2})(1+x^{4k})^2 \pmod{4}$$ and since $$(1+x^{2k})^4\equiv (1+x^{4k})^2 \pmod{4}$$ this reduces to -$$\frac{\prod_{k\geq 1}(1+x^{2k-1})^2+\prod_{k\geq 1}(1-x^{2k-1})^2}{2}\equiv \prod_{k\geq 1} (1+x^{4k-2})\pmod{4}$$ but we can write $$\prod_{k\geq 1}(1-x^{2k-1})^2\equiv \left(\prod_{k\ geq 1}(1+x^{2k-1})^2\right) \left(1-4\sum_{k\geq 1}\frac{x^{2k-1}}{(1+x^{2k-1})^2}\right)\pmod{8}$$ therefore now we have to show -$$\prod_{k\geq 1}(1+x^{2k-1})^2\left(1-2\sum_{k\geq 1}\frac{x^{2k-1}}{(1+x^{2k-1})^2}\right)\equiv \prod_{k\geq 1}(1+x^{4k-2})\pmod{4}$$ Now everything is clear since $$\prod_{k\geq 1}(1+x^{2k-1})^2\equiv \prod_{k\geq 1}(1+x^{4k-2})\left(1+2\sum_{k\geq 1}\frac{x^{2k-1}}{(1+x^{2k-1})^2}\right)\pmod{4}$$<|endoftext|> -TITLE: Regge calculus: Questions of consistency resolved? -QUESTION [13 upvotes]: Hello, -Regge calculus is an approximation scheme for General Relativity, which has been introduced in early-sixties and has been adopted both in numerical relativity and numerical quantum relativity. In contrast to its widespread use in computational science, there does not seem to exist much theory on whether the Regge calculus is actually consistent - i.e. whether there is some degree of exactness (like mesh width) we can adapt arbitrarly to obtain a solution with arbitrarly small error (say in some Sobolev-norm). -In fact there has been debate about this: - -The Regge Calculus is not an approximation to General Relativity, 1995 -On the convergence of Regge calculus to general relativity, 2000 - -The question of consistency is a purely mathematical one, and therefore I do not expect it to be "debatable". I will have to work with this theory and hence I do wonder in how far there is a theoretical basis to tell apart "It works" and "It does not work". -Thank you very much! - -REPLY [12 votes]: The consistency is proved by Cheeger, M\"uller and Schrader in 1984, "On the Curvature of Piecewise Flat Sapces". Roughly speaking, given a smooth Riemannian manifold with a smooth metric, there exists a sequence of triangulation, on which Regge's definition converges to the smooth curvature as a measure. -At the linearized level, there is also a recent paper on the consistency: Christiansen 2011, "On The Linearization of Regge Calculus". One of the theorems in the paper is that we have consistency between linearized Regge and linearized Einstein equation as well. -That said, when you talk about the convergence of a numerical algorithm, it depends on a lot of other things as well, such as your formulation of the Einstein's equation. You will also need some form of stability to ensure convergence. Those questions remain to be solved (hopefully in my thesis:-).<|endoftext|> -TITLE: Is a flat, locally finite type morphism open? -QUESTION [8 upvotes]: Let $f: X\to Y$ be a morphism between arbitrary schemes. It is proved in EGA IV Prop 2.4.6 that if f is flat and locally of finite presentation, then f is open. If one replaces locally of finite presentation by locally of finite type, I don't think that the statement is still true (unless X and Y are locally noetherian) but I don't know of any counterexample. - -REPLY [4 votes]: Over a field $k$, another related (and somehow more algebraic) example is the "universal" $k$-algebra $A$ generated by countably many orthogonal idempotents. In other words $A$ is the $k$-algebra obtained as the quotient of the polynomial ring $k[X_1,X_2,...]$ in countably many variables, by the ideal generated by all polynomials $X_i^2-X_i$ and $X_iX_j$ for $i\ne j$. It is easy to see that the maximal ideal $m=(X_1,X_2,...)$ gives a nonisolated closed point of $Spec(A)$. Moreover the local ring $A_m$ is just $k$, so that the closed immersion $i:Spec(A/m)\to Spec(A)$ is flat (the extension of local rings is an isomorphism). Thus it is a flat, finite type, morphism which is not open.<|endoftext|> -TITLE: Fixing a mistake in "An introduction to invariants and moduli" -QUESTION [9 upvotes]: On page 13 of the book "An introduction to invariants and moduli" of Mukai -http://catdir.loc.gov/catdir/samples/cam033/2002023422.pdf there is a mistake, in the end of the proof of Proposition 1.9. It seems to me that this proof can not be fixed, without using the notion of Noetherian rings and Hilbert basis theorem. -The question is: Can this proof be fixed, without using commutative algebra -- i.e., by the elementary reasoning that Mukai is using there? -I reproduce here the proof from the book for completeness. $S$ is the ring of polynomials, $G$ a group, $S^G$ is the ring of invariants -Proposition. If $S^G$ is generated by homogeneous polynomials $f_1,...,f_r$ -of degrees $d_1,...,d_r$, then the Hilbert series of $S^G$ is the power -series expansion at $t=0$ of a rational function -$$P(t)=\frac{F(t)}{(1-t^{d_1})...(1-t^{d_r})}$$ -for some $F(t)\in \mathbb Z[t]$. -Proof. We use induction on $r$, observing that when $r=1$, the ring $S^G$ is just $\mathbb C[f_1]$ with the Hilbert series $$P(t)=1+t^{d_1}+t^{2d_1}+...=\frac{1}{1-t^{d_1}}.$$ -For $r>1$ consider the injective complex linear map $S^G\to S^G$ defined by $h\to f_rh$. Denote the image by $R\subset S^G$ and consider the Hilbert series for the graded rings $R$ and $S^G/R$. Since $R$ and $S^G/R$ are generated by homogeneous elements, we have -$$P_{S^G}(t)=P_{R}(t)+P_{S^G/R}(t).$$ -On the other hand, $dim(S^G\cap S_d)=dim(R\cap S_{d+d_r})$, so that $P_R(t)=t^{d_r}P_{S^G}(t)$, and hence -$$P_{S^G}(t)=\frac{P_{S^G/R}(t)}{1-t^{d_r}}.$$ -But $S^G/R$ is isomorphic to the subring of $S$ generated by the polynomials $f_1,...,f_{r-1}$, and hence by the induction hypothesis $P_{S^G/R}(t)=F(t)/(1-t^{d_1})...(1-t^{d_{r-1}})$ for some $F(t)\in \mathbb Z[t]$... -Mistake: It is not true that $S^G/R$ is isomorphic to the subring of $S$ generated by polynomials $f_1,...,f_{r-1}$. For example consider $\mathbb C^2$ with action $(x,y)\to (-x,-y)$. Then let $f_1=x^2$, $f_2=y^2$, $f_3=xy$. -Motiviation of this question. Of course this proposition is a partial case of Hilbert-Serre theorem, proven for example at the end of Atiyah-Macdonald. But the point of the introduction in the above book is that one does not use any result of commutative algebra. - -REPLY [3 votes]: This is not an answer to the question, I just decided to give for completeness a standard proof of the above statement that uses (a version of) Hilbert-Serre theorem. In this proof we need to use Hilbert basis theorem. In the above statement $S^G$ is clearly a finitely generated graded module over the ring of polynomials $\mathbb C[x_1,...,x_r]$, so it is sufficient to prove: -Theorem (Hilbert, Serre). Suppose that $S=\sum ^{\infty}_{j=0}S_j$ is a commutative graded ring with $A_0=\mathbb C$, finitely generated over $\mathbb C$ by homogeneous elements $x_1,...,x_r$ in positive degrees $d_1,...,d_r$. Suppose that -$M=\sum_{j=0}^{\infty} M_j$ is a finitely generated graded $S$-module (i.e., we have $S_iS_j\subset S_{i+j}$ and $S_iM_j\subset M_{i+j}$). -Then the Hilbert series $P(M,t)$ is of the form -$$\sum_{j=0}^{\infty}dim(M_j)t^j=P(M,t)=\frac{F(t)}{\Pi_{j=1}^r(1-t^{d_j})}, -\;\; F(t)\in \mathbb Z[t].$$ -Proof. -We work by induction on $r$. If $r=0$ then $P(M,t)$ is a polynomial with integer coefficients, so suppose $r>0$. Denote by $M'$ and $M''$ the kernel and cokernel of the multiplication by $x_r$, we have an exact sequence for each $j$ -$$0\to M'_j\to M_j \to^{x_r}M_{j+d_r}\to M''_{j+d_r}\to 0.$$ -Now $M'$ and $M''$ are finitely generated graded modules for $K[x_1,...,x_{r-1}]$, and so by induction their Hilbert series have the given form. From the above exact sequence we have -$$t^{d_r}P(M',t)-t^{d_r}P(M,t)+P(M,t)-P(M'',t)=0.$$ -Thus -$$P(M,t)=\frac{P(M'',t)-t^{d_r}P(M',t)}{1-t^{d_r}}$$ -has the given form. -Where did we use Hilbert basis theorem? We use it when we say that $M'$ is finitely generated.<|endoftext|> -TITLE: Hopfian property preserved by extensions with finite kernel? -QUESTION [7 upvotes]: Let $G$ be a group which is Hopfian and given a short exact sequence $1\to F \to H \to G \to 1$ -with $F$ a finite normal subgroup of $H$. Is $H$ Hopfian? - -REPLY [6 votes]: It seems that the answer is no: there exists an exact sequence -$$1\to F\to H\to Q\to 1$$ -with $F$ finite (central), $H$ non-Hopfian, and $Q$ Hopfian (with in addition, $H$ finitely generated solvable). -Mark Sapir's answer refers to a group constructed here (see 5.10), which is Abels' group over the ring $\mathbf{F}_p[t,1/t]$, and which probably be used to provide a negative answer to the question. Define the group $G$ as the group of matrices -$$\left(\begin{array}{rrrr} -1 & u_{12} & u_{13} & u_{14}\newline -0 & d_{22} & u_{23} & u_{24}\newline -0 & 0 & d_{33} & u_{34}\newline -0 & 0 & 0 & 1\newline -\end{array}\right) -$$ -where $u_{ij}\in\mathbf{F}_p[t,1/t]$, -and $d_{ii}\in\mathbf{F}_p[t,1/t]^\times=\langle t\rangle\mathbf{F}_p^\times$. -(Actually one can restrict to $d_{ii}\in\langle t\rangle$ but it's not important.) Let $e_{14}$ denote the map $x\mapsto\begin{pmatrix} 1&0&0&x\\0&1&0&0\\0&0&1&0\\0&0&0&1\end{pmatrix}$. -As suggested by user "BS.", let $N$ be the central subgroup $e_{14}(\mathbf{F}_p[t^2])$ and $M$ the larger central subgroup $e_{14}(\mathbf{F}_p[t^2]\oplus \mathbf{F}_pt^{-1})$. (In a the initial post, $M$ and $N$ were chosen as other central subgroups but unfortunately $G/M$ failed to be Hopfian). So we have the central exact sequence with finite kernel -$$1\to M/N\to G/N\to G/M\to 1.$$ -Conjugation by the diagonal matrix $(t^2,1,1,1)$ is an automorphism of $G$, which maps $N=e_{14}(\mathbf{F}_p[t^2])$ strictly into itself and hence $H=G/N$ is non-Hopfian. -I haven't completely checked but here are some guidelines to show the group $Q=G/M$ is Hopfian. -Write the original group (given by $4\times 4$ triangular matrices) as $G=D\ltimes U$ with $D=\mathbf{Z}^2$ and $U$ its unipotent part. Set $U^2=[U,U]$ and $U^3=[U,U^2]$, which is central and equal to $e_{14}(\mathbf{F}_p[t,1/t])$. -Let $f$ be a surjective endomorphism of $Q$. - -check that the center of $G$ is precisely $U^3$. -It follows that $f$ induces a surjective endomorphism of $G/U^3$. Since this group is linear, it is Hopfian so this is an automorphism of $G/U^3$. - -Describe the group of automorphisms of $G/U^2 = \mathbf{Z}^2\ltimes F_p[t,1/t]^3$. (It should be reasonably easy to describe). - -Deduce a description of the group of automorphisms of $G/U^3$, or at least describe how these automorphisms act on $U^2/U^3$, showing that modulo something, the coefficient $12$ is multiplied by a monomial $w\cdot t^a$ ($w\in \mathbf{F}_p^*$) and the coefficient $24$ is multiplied by $vt^b$. So, taking a commutator (which should kill the "modulo something"), we obtain that in the "action of $f$ on $G$", the coefficient $14$ should be multiplied by a nonzero monomial. This multiplication should stabilize $M$ so this is multiplication by a scalar in $\mathbf{F}_p^*$, which implies that f actually induces an automorphism of $Q$.<|endoftext|> -TITLE: Generating finite simple groups with $2$ elements -QUESTION [43 upvotes]: Here is a very natural question: -Q: Is it always possible to generate a finite simple group with only $2$ elements? -In all the examples that I can think of the answer is yes. -If the answer is positive, how does one prove it? Is it possible to prove it without using the -classification of finite simple groups? - -REPLY [5 votes]: Carlisle King recently posted an arXiv preprint which (claims to) show that every finite simple group is generated by an involution, together with another element of prime order. -http://arxiv.org/abs/1603.04717 -The paper uses the classification, as most of these do. -Elements of prime (or even prime power) order seem to be particularly easy to work with for a number of kinds of arguments. See e.g. this mathoverflow question. -Now, if you want a result that doesn't use the classification, Paul Flavell gave an elementary proof that every non-solvable group has 2 elements that generate a non-solvable group. See here.<|endoftext|> -TITLE: Which primes can divide orders of Tate-Shafarevich groups? -QUESTION [10 upvotes]: Heuristic arguments due to (I believe) Delauney predict that every prime divides the order of the Tate-Shafarevich group of infinitely many elliptic curves over $\mathbf{Q}$. However, is it even known that for every prime $p$, there's at least one elliptic curve over $\mathbf{Q}$ whose Tate-Shafarevich group is finite and has order divisible by $p$? - -REPLY [10 votes]: Just to record an answer: yes, it is believed to be true that for each prime $p$ there is an elliptic curve $E_{/\mathbb{Q}}$ whose Shafarevich-Tate group contains an element of order $p$. But this is wide open. -One can however prove the following: -Theorem (R. Kloosterman): If you fix a prime $p$, then there is a number field $K_p$ -- i.e., depending on $p$ -- and an elliptic curve $E/K_p$ whose Shafarevich-Tate group contains an element of order $p$. -As Kloosterman remarked (in his 2005 paper on the subject, and also just now in the above comments) via a simple "Weil restriction" argument, this implies that there exists an abelian variety $A_{/\mathbb{Q}}$ whose Shafarevich-Tate group contains an element of order $p$. But now the dimension of $A$ goes to infinity with $p$. -Others have since worked out various improvements of Kloosterman's Theorem. Here are three different directions: -1) What is the minimal degree $d_p = [K_p:\mathbb{Q}]$ of a number field $K_p$ in above theorem? Kloosterman's argument gave $d_p = O(p^4)$. In a 2005 paper I showed that one can take -$d_p = 2p^3$. In a 2010 paper with Shahed Sharif, we showed that one can take $d_p = p$. (Note that whether $d_p = p$ is in fact best possible is an open question. At one point I thought I had an argument to show that it was, but this was wrong. I now suspect that this is only best possible by a method proceeding along the lines of our construction. For instance, I believe it is conceivable that $d_p = 2$ for all $p$, and this has something to do with how Sha behaves in quadratic twists...) -2) What kind of elliptic curves $E$ can be used to produce these elements in Sha? Sharif and I showed that in fact one can start with any elliptic curve $E_0$ over $\mathbb{Q}$ and take its base change to $K_p$ a degree $p$ number field. (Similarly, one can start with any elliptic curve $E$ over any number field and get order $p$ elements of Sha in an extension field of degree $p$.) And in fact $p$ does not need to be a prime number here: it holds for every integer $n$. And in fact you can get as many elements of order $n > 1$ as you want. -3) What kind of control can one get over the field extension $K_p/\mathbb{Q}$? Can one for instance choose it to be Galois, abelian, or cyclic? Matsuno proves that for any cyclic degree $p$ extension $K_p/\mathbb{Q}$ one can find elliptic curves $E_{\mathbb{Q}}$ such that the base change to $K_p$ has as many order $p$ elements in its Shafarevich-Tate group as one wants. There are similar results along these lines (with dihedral extensions and elements of the Selmer group) by Alex Bartel. -P.S.: As Tim Dokchitser points out, replacing $\mathbb{Q}$ by any fixed number field does not help to answer the OP's question, although the results of Sharif and myself hold equally well in this relative setting. (I was going to ask "In what way could an arbitrary but fixed number field be any better than $\mathbb{Q}$?" but then I remembered an amazing result about $2$-Selmer parity that only holds over certain imaginary number fields...due to Dokchitser and Dokchitser.)<|endoftext|> -TITLE: Importance of Poincaré recurrence theorem? Any example? -QUESTION [22 upvotes]: Recently I am learning ergodic theory and reading several books about it. -Usually Poincaré recurrence theorem is stated and proved before ergodicity and ergodic theorems. But ergodic theorem does not rely on the result of Poincaré recurrence theorem. So I am wondering why the authors always mention Poincaré recurrence theorem just prior to ergodic theorems. -I want to see some examples which illustrate the importance of Poincaré recurrence theorem. -Any good example can be suggested to me? -Books I am reading: -Silva, Invitation to ergodic theory. -Walters, Introduction to ergodic theory. -Parry, Topics in ergodic theory. - -REPLY [2 votes]: This is an old question but I don't see the obvious answer, so here we go. -A huge field of research in mathematical physics during the XIXe century revolved around giving explicit solutions to the equations of classical mechanics, using brute computation. At that time, finding a new first integral in the equations of motions of some physical system was a sure path to academic fame. If sufficiently many first integrals are found, the system is integrable. If moreover the motion is constrained to a bounded domain of the phase space , it is quasi-periodic and thus both regular and recurrent. The long term behavior of the system is thoroughly described. -There was certainly some hope at first to show the stability of the solar system, or at least the three body problem, by finding these first integrals. Huge efforts went into that line of research. A century later, we know that the three body problem is not integrable in general and integrability is a pretty rare property of dynamical systems. In particular it is not stable by perturbation. -But wait, we know that Earth won't suddenly fly towards Pluto and stay there forever. Actually, we are pretty sure that it will come back to its current position, and this follows from Poincaré recurrence theorem, once you note that the Liouville measure is left invariant by the motion. No need to explicitly solve the equations of motion, and the result is so general that it applies to all hamiltonian systems in restriction to a compact level of energy. And the proof is short and elegant! To be pedantic, this was a paradigm shift and the birth of a new method in the field of mathematical physics, best described by Poincaré himself in his numerous books. -We now consider the Poincaré recurrence theorem as marking the birth of a new mathematical discipline called ergodic theory, with striking applications to arithmetic, Lie groups, foliations, moduli spaces etc. To the point that people seem to have forgotten its celestial origins.<|endoftext|> -TITLE: Index theorem interpretation of the spectral flow for a pseudo holomorphic curve -QUESTION [9 upvotes]: Let $(M , \omega)$ be a symplectic manifold, $J$ a compatible almost complex structure. We call pseudo holomorphic strip a solution $u : \mathbb R \times I \to M$ of the equation $\partial_s u + J \partial_t u = 0.$ Given a pair of Lagrangian submanifolds $L_0, L_1$, such a strip is said to be bounded by the pair if $u(s, i) \in L_i, i = 0, 1.$ Under mild conditions, such a strip has limits as $s \to \infty$ that are intersection points between the Lagrangian submanifolds. -Robbin & Salamon proved that If the Lagrangian intersections are transverse, the Fredholm index between suitable Sobolev spaces of this linearized Cauchy-Gromov-Riemann operator coincides with the Maslov-Viterbo of the strip. Their proof involves general considerations for linear operators of the form $\partial_s + A_s$ defined on the space of paths $\mathbb R$ to some Hilbert space and rely on reducing the problem to a finite dimensional Hilbert space. -However the eventual result admits a purely intrinsic formulation : it states that the Fredholm index of a Dirac operator is given in terms of a characteristic class. This seems like a particular instance of the Atiyah-Singer index theorem, only on a manifold with boundary (the strip) and with totally real boundary conditions. -Can this particular result (index = Maslov class) be obtained through a less coordinate-bound and maybe more striking way ? - -REPLY [6 votes]: I try to elaborate on my own comment to Tom Mrowka's answer. -Assume that $f : \mathbb S^1 \to \mathbb S^1$ is càdlàg and has a unique discontinuity point at $1 \in \mathbb S^1$. This discontinuity is therefore a "jump" by a certain angle $\alpha.\pi$. This matches the behavior of the function $f_\alpha : z \mapsto (z-1)^\alpha,$ so that upon normalizing inside $\mathbb D^2$ by this holomorphic function, the problem reduces to the modelization proposed by Tom Mrowka. -By using multiple such normalizations, one gets a geometric description of the virtual dimension for discs with marked points on their boundary, each segment thus delimited abutting on a different "real condition", as relevant in the study of products in Floer theory.<|endoftext|> -TITLE: What is the cover time of a random walk on a cube? -QUESTION [11 upvotes]: I can't quite figure this problem yet. There is an ant at one vertex of a cube. The ant goes from one vertex to another by choosing one of the neighboring vertices uniformly at random. What is the average minimum time it takes to visit all vertices? - -REPLY [6 votes]: Yet another reference: Some sample path properties of a random walk on the cube, by Peter Matthews (1989). This covers the asymptotic distribution of the time $T$ taken to visit all vertices, the distribution of the number of vertices not visited at times near to $\mathbb{E}[T]$, and the expected time taken for the walk to come within a distance $d$ of all vertices.<|endoftext|> -TITLE: What relations exist among (quasi-) modular forms of different levels? -QUESTION [13 upvotes]: Given a (quasi-) modular form $f(\tau)$ for some congruence subgroup (say) $\Gamma(k)$, we know that $f(N\tau)$ is a (quasi-) modular form for $\Gamma(N k)$. Is there anything known about when we can do a partial reverse, that is, when we can take linear combinations of (quasi-) modular forms for some higher level subgroup to obtain one of stricktly lower level? -An example is the following: -Let $E(q) = \sum_{k=0}^\infty \sigma_1(2k+1)q^{2k+1}$ and $A(q) = \sum_{k=1}^\infty \sigma_1(k)q^k$. Then $E(q)$ is modular with respect to a non-trivial character, and both $A(q^2)$ and $A(q^4)$ are quasi-modular of level 2 and 4, respectively (though not of pure weight). -However: it turns out that -$$ -E(q) + 3A(q^2) - 2A(q^4) = A(q) -$$ -which shows that a linear combination of higher level terms (and one which is modular with respect to a non-trivial character) yields one of lower level. -Is this simply random chance? Are there known relations of this type? - -REPLY [2 votes]: There are (at least) two answers: there is a Galois theory of modular functions (as in G. Shimura's 1971 book, or in Lang's book on elliptic functions), and a theory of newforms" (Atkin-Lehner, and also Casselman in a representation-theoretic context). -Thinking in terms of general Galois theory, it ought not be so surprising that various sums of non-invariant things become invariant, I suppose, but the particulars are often non-trivial.<|endoftext|> -TITLE: Confidence intervals when the number of samples is random -QUESTION [6 upvotes]: I am interested in computing confidence intervals for the mean of a random variable $X$ given $\require{cancel}\xcancel{N \text{ i.i.d. samples}}$ an i.i.d. sample of $N$ copies of $X$, where $N$ is $\operatorname{Binomial}(n, p)$. Any time I read about confidence intervals for the mean it is assumed that the number of samplessize of the sample is fixed, which makes the asymptotic distribution of the sample mean Gaussian, and therefore allows for student-based confidence intervals and the like to be justified. However, if the number of samples size $N$ of the sample is a random variable itself, then the ratio -$$ \frac{\sum_i X_i}{N} $$ -will not be necessarily normal (see, for instance, http://en.wikipedia.org/wiki/Ratio_distribution#Gaussian_ratio_distribution). -What is the best way to deal with this scenario? Will bootstrapping be theoretically justified in this case? - -REPLY [2 votes]: Let $\hat{\mu}=\frac{\sum_i X_i}{N}$ and $\mu = \mathbf{E}(X)$ -$$\mathcal{P}(\mu \in [x,y]\,|\, a) = \sum_i \binom ni p^i(1-p)^{n-i}\mathcal{P}(\mu \in [x,y]\,|\,a,N=i)$$ -If $pN$ is relatively large, and $\hbox{var}(X) = \sigma^2$ you can represent the true mean as taken from a mixture of gaussian with mean $a$ and variance $\sigma^2/N$. This is not gaussian, but you can compute the pdf. -While the mixture isn't gaussian, you can compute its variance as -$$\sigma^2 \frac{np(1-p)^{n-1} F_{3,2}\left(1,1,1-n;2,2;\frac{p}{p-1}\right)}{1-(1-p)^n}$$ -(N.B this only valid when $X$ is normally distributed or approximately valid when $n >> (1-p)/p$) -$F_{3,2}$ is the hypergeometric function -For $p = 0.25$ and $N=100$ your average has about $4.13\%$ of the variance of $X$ -You can then use conservatively Chebyshev's inequality. - -REPLY [2 votes]: You should speak of the size of the sample, rather than of the number of samples. -You haven't said whether you can actually observe the sample size. Nor whether the probability distribution of the sample size in any way depends on the mean that you're trying to estimate. If you can observe the sample size and if you know it doesn't depend on that which you're trying to estimate, then it is an ancillary statistic (see http://en.wikipedia.org/wiki/Ancillary_statistic) and you can in effect treat it as non-random by conditioning on it.<|endoftext|> -TITLE: The ring of SL_2 invariants in sums of conjugation and tautological modules -QUESTION [5 upvotes]: Rings of Invariants -Consider $G=SL_2(\mathbb{C})$, and let $V$ be a finite-dimensional $G$-representation. Let $\mathbb{C}[V]$ denote the ring of polynomial functions on the space $V$; it is a free polynomial ring in $\dim(V)$-many variables. Then the ring of invariants of $V$ is the subring -$$\mathbb{C}[V]^G:=(f\in\mathbb{C}[V] s.t. \forall g\in G, f(gv)=f(v))$$ -The Conjugation Representation -Let $M_2(\mathbb{C})$ denote the space of $2\times 2$ complex matrices. The conjugation representation $V_C$ of $G$ on $M_2(\mathbb{C})$ is defined by $g\cdot m:= gmg^{-1}$. Thus, $V_C$ is a 4-dimensional $G$-representation. -The ring of invariants $\mathbb{C}[V_C]^G$ is well-known. It is freely generated by the functions $tr$ and $det$; this is a special case of the general fact that the only conjugation-invariant functions on $M_n(\mathbb{C})$ are symmetric functions of eigenvalues. -The Tautological Module -Let $V_T$ denote the $2$-dimensional $G$-representation where $G$ acts in the 'obvious' way, by the inclusion $G=SL_2(\mathbb{C})\subset GL_2(\mathbb{C})$. This is called the tautological $G$-representation. Since $SL_2(\mathbb{C})$ acts on $V_T$ with a dense orbit, the ring of invariants $\mathbb{C}[V_C]^G$ is boring; it is just $\mathbb{C}$. -Now, let $V_T^n$ denote the $n$-fold direct sum of $V_T$. This is a natural $2n$-dimensional $G$-representation. We can choose a $G$-invariant skew-symmetric bilinear form $\omega$ on $V_T$; this is big words for the usual scalar cross product $v_1\times v_2$. -This defines a natural $G$-invariant function on the space of pairs $(v_1,v_2)\in V_T^2$. -Then the ring of invariants $\mathbb{C}[V_T^n]^G$ is generated by $\omega(v_i,v_j)$ for $1\leq i j\leq n$. However, these do not (in general) freely-generate the ring of invariants, there are relations between them. The relations are all of the form -$$ \omega(v_i,v_k)\omega(v_j,v_l)=\omega(v_i,v_j)\omega(v_k,v_l)+\omega(v_l,v_i)\omega(v_j,v_k)$$ -for $1\leq ijkl\leq n$. -Both of these facts can be deduced by observing that $\mathbb{C}[V_T^n]^G$ can be identified with the homogeneous coordinate ring of the Grassmanian $Gr(2,n)$. Then the generators above are the Plucker coordinates, and the relations are the 3-term Plucker relations (there are no higher Plucker relations here). -The Question -Both of these examples have clever solutions and pretty answers. I am curious about the combination of both cases. -Let $m$ and $n$ be positive integers, and consider the direct sum $G$-representation $V^m_C\oplus V_T^n$. What is the ring of invariants $\mathbb{C}[V_C^m\oplus V_T^n]^G$? -I am aware of general procedures for producing these rings, for arbitrary finite-dimensional $G$-reps; see e.h. Sturmfel's Algorithms in Invariant Theory. However, I suspect that there is a clever solution to this particular problem. Not only because it is a combination of two problems with a clever solution, but because I have a guess as to what the answer is, and all the relations seem to be similar to the 3-term Plucker relations. I also suspect the answer is ancient (like much invariant theory), which is why I am trying to find an answer rather than try to prove my guess is correct by brute force. - -REPLY [2 votes]: I don't have the literature with me, but yes the answer to your question was well known to classical invariant theorists. My recollection is that it is one of the examples (sections) in Grace and Young (J.H. Grace and A. Young, The algebra of invariants, Cambridge Univ. Press, Cambridge, 1903). They give an answer without using polarization. All the generating invariants are the ones you describe or else invariants of degree 3 which are quadratic in (one or 2) of the tautological reps and linear in a conjugation rep. Describing the relations is harder but was understood classically. I am not sure of a reference though.<|endoftext|> -TITLE: Induction, the infinitude of the primes, and workaday number theory -QUESTION [15 upvotes]: There are various open problems in the subject of logical number theory concerning the possibility of proving this or that well-known standard results over this or that weak theory of arithmetic, usually weakened by restricting the quantifier complexity of the formulas for which one has an induction axiom. It particular, the question of proving the infinitude of the primes in Bounded Arithmetic has received attention. -Does this question make known contact with "workaday number theory" - number theory not informed by concepts from logic and model theory? I understand that proof of the infinitude of the primes in bounded arithmetic could not use any functions that grow exponentially (since the theory doesn't have the power to prove the totality of any such function). So especially I mean to ask: -1) If one had such a proof, would it have consequences about the primes or anything else in the standard model of arithmetic? -2) If one proves that no such proof exists, would that have consequences... -3) Do any purely number theoretic conjectures if settled in the right way, settle this question of its kin? - -As a side question, I'd be interested to know the history of this question. I first heard about it from Angus Macintyre and that must have been 25 years ago. - -REPLY [11 votes]: Two comments: - -Work of Paris, Wilkie and Woods shows that we can prove the existence of infinitely primes, and indeed that there is always a prime between $n$ and $2n$, assuming $I\Delta_0+\forall x -\exists y\ y=x^{|x|}$, where $|x|$ is the length of a binary expansion of $x$. So we know functions of exponential growth aren't necessary, but we are still using a function of super-polynomial growth. -Actually, they proved that this theory implies a weak Pigeon-hole Principle which Woods had shown earlier implied the infinitude of primes. -Another question in this spirit is whether $I\Delta_0$ proves that for any prime $p$ there is a non-square mod $p$. - -It is known that neither of these number theoretic results can be proved if the base theory is weakened to allow induction only for quantifier free formulas.<|endoftext|> -TITLE: Is the $\infty$-category of stable $\infty$-categories stable? -QUESTION [12 upvotes]: More generally, are there any remarkable properties enjoyed by the $\infty$-category of stable $\infty$-categories? - -REPLY [14 votes]: No. It's pointed by the zero category, but then taking loops of a stable category C (in the sense of the pullback of 0 --> C along itself) always gives the zero category, so loops is definitely not an equivalence. -One important structural feature of the category of stable categories along these lines is that it has some nice cofiber sequences (Verdier localization sequences), but I'm not sure the categorical properties these satisfy have been axiomatized (into a possible definition of ``stable (infty,2)-category''?)<|endoftext|> -TITLE: local Langlands and the Jacquet module -QUESTION [14 upvotes]: Let $G = GL_n(F)$ be the general linear group over a finite extension $F$ of $Q_p$. This question could be posed for a larger class of groups, but let us stay with $Gl_n$ for the moment. -Let $\pi$ be a smooth irreducible complex representation of $G$. Let $P \subset G$ be a parabolic subgroup and $P =MN$ a Levi-decomposition. The Jacquet module $\pi_N$ of $\pi$ is by definition the module of $N$-coinvariants in $\pi$. -Via the local Langlands correspondence $\pi$ corresponds to a Weil-Deligne representation $\sigma_\pi$. Furthermore, in the cases where $\pi_N$ is irreducible, the representation $\pi_N$ has a corresponding Weil-Deligne representation via local Langlands for $M$. -My question is, does the operation $\pi \to \pi_N$ from $G$-representations to $M$-representations has a "satisfactory" interpretation on the Galois side, via local Langlands for $G$ and $M$? -A point of caution is that $\pi_N$ need not be $M$-irreducible, so it does not go directly into local Langlands for $M$. - -REPLY [9 votes]: Let us consider the simple case: $G=GL_2(F)$, $n=2$. (cf. ''The local langlands conjecture for $GL_2(F)$'' C.J. Bushnell and G.Henniart) -In order to tell the story, first we need to give some definitions. Clearly we only need to consider the non-cuspidal case. Let $\chi=\chi_1\otimes \chi_2$ be the character of $T$, we denote $ \chi^{\omega}=\chi_2\otimes \chi_1$, we define $\pi_{\chi}=Ind_B^G(\delta_B^{-\frac{1}{2}}\otimes \chi)$ where $\delta_B$ is the modular function of the group $B$ i,e $\delta_B(tn)=||t_2t_1^{-1}||$ for $t=diag(t_1,t_2)$, $n\in N$, we write $\phi\circ det$, $\phi \cdot St_G$ two other kind of principal series for $GL_2(F)$. -Now we arrive to write the Jacquet functor $J: Rep(G) \longrightarrow Rep(T); (\pi, V) \longrightarrow -(\pi_N, V_N)$. -(1) For $\chi_1\chi_2^{-1}\neq ||.||^{\pm}$, $\pi=\pi_{\chi}$ is irreducible, then -$\pi_N=\delta_B^{-\frac{1}{2}}\otimes \chi \oplus \delta_B^{-\frac{1}{2}}\otimes \chi^{\omega}$. -(2) $\pi=\phi\circ det$, then $\pi_N=\phi\otimes \phi$. -(3) $\pi=\phi \cdot St_G$, then $\pi_N=||.||\phi\otimes ||.||^{-1}\phi$. -We recall some result about local langlands correspondance for general linear group. We denote $\mathcal{G}_2(F)$ to be the set of equivalence classes of 2-dimensional Frobenius semisimple, Deligne representation of the Weil group $\mathcal{W}_F$; also $\mathcal{A}_2(F)$ to be the set of equivalence classes of irreducible smooth representations of $GL_2(F)$. The local langlands correspondance tell us that there is a natural bijective map $l_2$ between $\mathcal{A}_2(F)$ and $\mathcal{G}_2(F)$. The naturality often involves some compatibility conditions. ( For detail one should see the article of Borel in Corvallis). -Assume $\pi$ is irreducible, lying in $\mathcal{A}_2(F)$, we denote $l_2(\pi)=(\rho,W,\mathbf{n})$. -(1) if $\pi=\pi_{\chi}$, then $\rho=\chi_1 \oplus \chi_2$ and $\mathbf{n}=0$, here we regard $\chi_i$ as the representation of Weil group $\mathcal{W}_F$. -(2) if $\pi=\phi\circ det$, then $\rho=||.||^{-\frac{1}{2}}\phi \oplus ||.||^{\frac{1}{2}}\phi$ and $\mathbf{n}=0$. -(3) if $\pi=\phi \cdot St_G$, then $\rho=||.||^{-\frac{1}{2}}\phi \oplus ||.||^{\frac{1}{2}}\phi$, but in this case -$\mathbf{n}\neq 0$. -Finally we comme to the question that Arno asks. We translate directly ''the Jacquet functor'' to the Galois side via the local langlands correspondence. -$J: \mathcal {G}_2(F) \longrightarrow \mathcal{G}_1(F)^{\otimes 2}$. More precisely, the result is outlined as follows: -(1) $J\big((\pi_{\chi}, \mathbf{n}=0)\big)=(\delta_B^{-\frac{1}{2}}\otimes \chi) \oplus (\delta_B^{-\frac{1}{2}}\otimes \chi^{\omega})$; -(2) $J\big((\phi\circ det, \mathbf{n}=0)\big)=\phi\otimes \phi$. -(3) $J\big((\phi \cdot St_G, \mathbf{n}=0)\big)= ||.||\phi\otimes ||.||^{-1}\phi$. -Remark: for general case, we take $\pi \in Irr_{\mathbb{C}}(G)$, one knows $\pi_N$ has finite length and is admissible as the representation over its Levi subgroup $M$, although we don't even know it is semi-simple or not.<|endoftext|> -TITLE: Geometry of Whitehead manifolds. -QUESTION [11 upvotes]: I'm currently studying some problems about the Whitehead manifold $W$ (the open 3-manifold which is contractible but not homeomorphic to $\mathbb{R}^3$). Does there exists some survey paper on its properties ? -I'm particularly interested in geometric results in the spirit of "Taming 3-manifolds using scalar curvature" by Chang, Weinberger and Yu (MathSciNet page) which shows that $W$ admits no metrics of uniformly positive scalar curvature. -Thanks. - -REPLY [15 votes]: McMillan proved that any contractible 3-manifold is obtained as a union of handlebodies, each of which is homotopically trivial in the next (this generalizes the method of construction of Whitehead). -Later he proved that there are uncountably many topologically distinct contractible 3-manifolds. -There is a survey of these and other results by McMillan. Even though these are old results, they are essentially state-of-the-art. The outstanding open problem was whether one of these contractible manifolds could cover a closed 3-manifold, but this is now resolved by the geometrization theorem (any closed aspherical 3-manifold is covered by $R^3$). Robert Myers obtained partial results on this problem, extended by David Wright, but this is superseded by geometrization. Myers also studied the space of end-reductions for such manifolds. -Of course, the paper you are interested in needs a very minimal amount of 3-manifold topology (aside from the Poincare conjecture!), which seems to be contained in Lemma 4.1 (although I'm not sure what 3-manifold results are used in papers which they cite, in particular reference 14).<|endoftext|> -TITLE: Quantitative measurement of infinite dimensionality -QUESTION [5 upvotes]: I recently encountered the metric mean dimension, which is a numerical metric invariant of (discrete time, compact space) dynamical systems that refines topological entropy for infinite-entropy systems. I am wondering if anything similar can be found in the literature for any metric notion of dimension (Let say that by ``metric'' means bi-Lipschitz invariant). -Put another way, I have a compact metric space $X$ that has infinite dimension for any sensible notion of dimension, and I would like to make this statement quantitative. I see two ways to do this. -The first one is to mimic the box-dimension, and consider the (extra-polynomial) growth rate of the smallest number of $\varepsilon$-balls needed to cover $X$ when $\varepsilon^{-1}$ goes to infinity. This is the simplest way to go, but I am concerned by the fact that box dimension have not as nice a behavior than Hausdorff dimension (for example countable spaces can have positive box dimension). -The second one, suggested by Greg Kuperberg, is to mimic Hausdorff dimension but replacing the family of "size functions" $(x\mapsto x^s)_s$ by another family with similar properties, like $(x\mapsto\exp(-\lambda/x)_\lambda)$. -My question is the following: do you know any example of such an invariant in the literature? Where is it used, in what purpose? - -REPLY [2 votes]: See my paper LINK -Centered densities and fractal measures, New York Journal of Mathematics 13 (2007) 33-87 -Some references are also at the end of it. In particular, Boardman, Goodey, and McClure.<|endoftext|> -TITLE: "Sums-compact" objects = f.g. objects in categories of modules? -QUESTION [29 upvotes]: Hello, -Let us call an object of an additive category sumpact (contraction of "sums" and "compact") if taking $Hom$ from it (considered as functor from the category to $Ab$) commutes with coproducts. Note that to be sumpact is weaker than to be compact (which means that $Hom$ from you commutes with filtered colimits). -Let us take, for our additive category, the category of left modules over some ring. It is known that compact objects in this category are exactly the finitely presented objects. What about sumpact objects? -It is clear that every finitely generated module is sumpact. When I try to prove the converse, I get into some pathological things. -Say, if a module has an increasing $\mathbb{N}$-sequence of submodules whose union is the whole module, and such that the union of every finite subsequence is not the whole module, then it is clear that this module is not a sumpact object (by considering the morphism from it to the direct sum of the quotients by members of our sequence). But it seems not clear (perhaps not true) that every non finitely generated module has such a sequence. -Also, when I check in the internet, it seems people put some condition: the ring is assumed to be perfect. Then indeed sumpact = f.g. -So my question is: for a general ring it is not true that sumpact implies f.g.? Can you give an example? Can you give an example when the ring is commutative? Can you indicate what perfect means and why then everything is OK? -Thank you - -REPLY [12 votes]: If it's considered bad form to resurrect year-old threads, then please slap my wrist (gently, please; I'm new here!) -A fairly simple explicit example of a "sumpact" module that is not f.g. is as follows. -Let $R$ be the ring of functions from an uncountable set $X$ to, say, a field $k$. Let $M$ be the ideal of functions with countable support. -Then it's very easy to show that $M$ isn't f.g., and fairly easy to show that it is "sumpact", using no set theory beyond the fact that a countable union of countable sets is countable. -Edit to add details requested in comments: -To show that $M$ is "sumpact", suppose that $\alpha:M\to\bigoplus_{i\in I}N_i$ is a homomorphism that doesn't factor through a finite subsum. I.e., for infinitely many $i$ the composition $\pi_i\alpha:M\to\bigoplus_{i\in I}N_i\to N_i$ of $\alpha$ with projection onto the summand $N_i$ is non-zero. Replacing $I$ with a countable collection of such $i$ we can assume that $I$ is countable and that $\pi_i\alpha$ is non-zero for all $i\in I$. -For each $i\in I$ choose $f_i\in M$ so that $\pi_i\alpha(f_i)\neq0$. Then the union of the supports $\text{supp}(f_i)$ is countable, so there is some $f\in M$ with $\text{supp}(f)=\bigcup_{i\in I}\text{supp}(f_i)$. -But then the ideal generated by $f$ contains every $f_i$, and so $\pi_i\alpha(f)\neq0$ for every $i$, contradicting the fact that $\alpha(f)\in\bigoplus_{i\in I}N_i$.<|endoftext|> -TITLE: Why is p=2 special, if we want to classify cplx. representation of GL2(Zp)? -QUESTION [8 upvotes]: I am currently reading Shalika's article "Representation of the two by two unimodular group over local fields" and various other related articles, which deal with the classification of complex representation of reductive groups over local rings. It is cumbersome, that some authors consider only local rings with residue fields of characteric $p \neq 2$. -Why is $p=2$ special here? -Perhaps some words about the strategy, which one should use - at least from my perspective: -Consider a finite extension $K$ of $\mathbb{Q}_p$. Let $\mathfrak{o}$ be the ring of integers in $K$ and $\mathscr{p}$ its maximal ideal. We want to classify all representation of $\mathrm{GL}_2( \mathfrak{o})$. Since we deal with a pro-$p$-group, the representations live on $\mathrm{GL}_2( \mathfrak{o}/\mathfrak{p}^n)$ for some $n>0$. We proceed by induction over $n$: -1) Classify all representation of $\mathrm{GL}_2(\mathbb{F}_q)$, where $\mathbb{F}_q$ is the residue field. -2) Use Mackey's formalism for the group extension (non split) -$$ 0 \rightarrow M_{2\times2}(\mathbb{F}_q) \rightarrow \mathrm{GL}_2( \mathfrak{o}/\mathfrak{p}^n) \rightarrow \mathrm{GL}_2( \mathfrak{o}/\mathfrak{p}^{n-1}) \rightarrow 0.$$ -Apparently the difficulties do already arise in step 1, since Piatesko-Shapiro in his lecture "Complex representations of $\mathrm{GL}_2$ over a finite field" only considers characteristic $\neq 2$. - -REPLY [4 votes]: A few more reasons/repetitions of reasons above in addition to the answers above : - -For $p = 2$, two different quadratic extensions can give rise to the same representation of $GL_2$ (this can happen for p not equal to 2, but only in the case of a Weil representation associated to a non-trivial character that has order 2 when restricted to the norm 1 elements of the extension). I don't know how bad this makes things, but criteria for this happening involves the discriminant (which can be pretty weird when $p = 2$). -For p = 2, the characters are hard to compute, even for the "Weil representations" Mander referred to above. This is because Weil representations are not so useful for character computation as with applying Frobenius formula to induction with a relatively simple representation of a subgroup of $GL_2(F)$ of the form $T B_n$, where T is an elliptic torus and B_n is something like a congruence subgroup. Such a representation is often a just a (quasi)character, and hence easier to handle. You see - the approach is not even to compute the character of a representation of $GL_2$ of the integers or an Iwahori but to start from a smaller subgroup times torus. The characters are computed on a torus-by-torus basis, and the presence of other tori in $T B_n$ can create havoc, when $p \neq 2$. Or so I hear. That the characters for $p \neq 2$ were computed enabled Sally and Shalika to compute Fourier transforms of orbital integrals and deduce Plancherel formula (for $SL_2$) from them.<|endoftext|> -TITLE: Completion of a category -QUESTION [37 upvotes]: For a poset $P$ there exists an embedding $y$ into a complete and cocomplet poset $\hat{P}$ of downward closed subsets of $P$. It is easy to verify that the embedding preserves all existing limits and no non-trivial colimits --- i.e. colimits are freely generated. $\hat{P}$ may be equally described as the poset of all monotonic functions from $P^{op}$ to $2$, where $2$ is the two-valued boolean algebra. Then we see, that $P$ is nothing more than a $2$-enriched category, $2^{P^{op}}$ the $2$-enriched category of presheaves over $P$ and that $y$ is just the Yoneda functor for $2$-enriched categories. -However, for a poset $P$ there is also a completion that preserves both limits and colimits --- namely --- Dedekind-MacNeille completion link text, embedding $P$ into the poset of up-down-subsets of $P$. -Is it possible to carry the later construction to the categorical setting and reach something like a limit and colimit preserving embedding for any category $\mathbb{C}$ into a complete and cocomplete category? - -REPLY [8 votes]: The following is what I wrote about six months ago during a private discussion (posted on request :-) -Let me start with a positive (obvious :-) result. -$\newcommand{\word}[1] { -\mathit{#1} -} -\newcommand{\catl}[1] { - \mathbb{#1} -} -\newcommand{\catw}[1] { - \mathbf{#1} -} -\newcommand{\mor}[3] { - {#1 \colon #2 \rightarrow #3} -}$ -1) Let $\catl{C}$ be a small category. Denote by $\word{Cont}(\catl{C}^{op}, \catw{Set})$ the full subcategory of presheaves $\catw{Set}^{\catl{C}^{op}}$ that consists of such $\mor{H}{\catl{C}^{op}}{\catw{Set}}$ that preserve small limits that exist in $\catl{C}^{op}$ (i.e. map existing colimits from $\catl{C}$ to limits in $\catw{Set}$). It follows from abstract nonsense that $\word{Cont}(\catl{C}^{op}, \catw{Set})$ has small (co)limits and, in fact, is a reflective subcategory of $\catw{Set}^{\catl{C}^{op}}$. -Of course, every presheaf preserves colimits, therefore the restricted Yoneda embedding $A \mapsto \hom(-, A)$ gives functor: $$\mor{y}{\catl{C}}{\word{Cont}(\catl{C}^{op}, \catw{Set})}$$ -This functor almost by definition preserves all limits that exist in $\catl{C}$. Observe, that it also preserves colimits. Let $\mor{F}{\catl{J}}{\catl{C}}$ be a functor from a small category, and assume that the colimit $\mathit{colim}(F)$ exists in $\catl{C}$. Consider any $H \in \word{Cont}(\catl{C}^{op}, \catw{Set})$. Then morphisms: -$$y(\mathit{colim}(F)) = \hom(-, \mathit{colim}(F)) \rightarrow H(-)$$ -by Yoneda, are tantamount to elements: -$$H(\mathit{colim}(F)) \approx \mathit{lim}(H \circ F)$$ -Similarly, morphisms: -$$y(F(J)) = \hom(-, F(J)) \rightarrow H(-)$$ -are tantamount to elements: -$$H(F(J))$$ -and one may easily verify, that the above exhibits $\hom(-, \mathit{colim}(F))$ as the limit $\mathit{lim}(y \circ F)$. -The above construction is described, for example, in "Basic concepts of enriched categories" of Max Kelly (Section 3.12). Moreover, as pointed out there, $\word{Cont}(\catl{C}^{op}, \catw{Set})$ inherits any monoidal (closed) structure from $\catl{C}$ via (restricted) convolution. -On the other hand, if $\catl{C}$ is not (monoidal) closed, then $\word{Cont}(\catl{C}^{op}, \catw{Set})$ generally will not be (monoidal) closed, due to the following fact. -2) There is no universal limit and colimit preserving fully faithful embedding of a small category to a cartesian closed complete and cocomplete category. -In fact, there is no universal embedding into cartesian closed category that preserves terminal object and binary coproducts. -For let us assume that $\catl{C}$ is non-degenerated and has a costrict terminal object (terminal object $1$ is costrict if whenever there exists a morphism $1 \rightarrow X$ then $X \approx 1$) and binary coproducts (you may take $\catl{C} = \catw{FinSet}^{op}$), and there is such an embedding $\mor{E}{\catl{C}}{\overline{\catl{C}}}$. -Because $1$ is costrict in $\catl{C}$ we have that $1 \sqcup 1 \approx 1$ in $\catl{C}$ and since coproduct and the terminal object are preserved by $E$, we have also $1 \sqcup 1 \approx 1$ in $\overline{\catl{C}}$. Let $\mor{f,g}{1}{A}$ be two morphisms in $\overline{\catl{C}}$. By the universal property of coproducts they induce the copairing morphism $\mor{[f,g]}{1 \sqcup 1}{A}$ that commutes with the coproduct's injections. However, since $1\approx 1 \sqcup 1$, the coproducts injections are identities and so $f = [f,g] = g$. Therefore, for every $A \in \overline{\catl{C}}$ there is at most one arrow $1 \rightarrow A$. If we take for $A$ an exponent $E(Y)^{E(X)}$, then by cartesian clossedness of $\overline{\catl{C}}$ and faithfulness of $E$: -$$\hom_\catl{C}(X, Y) \leq \hom_\overline{\catl{C}}(E(X), E(Y)) \approx \hom_\overline{\catl{C}}(1, E(Y)^{E(X)}) \leq 1$$ -which contradicts the fact that $\catl{C}$ is non-degenerated. -3) Generally, the completions obtained in any of the mentioned ways will not be the smallest completion. In fact, for a general $\catl{C}$, the smallest reflective subcategory of $\catw{Set}^{\catl{C}^{op}}$ containing representables may not be a smallest limit and colimit completion of $\catl{C}$. -Let us take for example $\mathcal{Z}_3$ group thought as of a category with a single object. Actually one may easily compute the category $\overline{\mathcal{Z}_3}$ induced by the idempotent monad associated to the monad $T$ on the Isbell cojugation for $\mathcal{Z}_3$ (by the theorem of Fakir this category can be computed by taking the objects that respects $T$-weak equivalences). Explicitly, category $\overline{\mathcal{Z}_3}$ is the full subcategory of $\catw{Set}^{\mathcal{Z}_3^{op}}$ on free permutations, together with the terminal permutation (up to the terminal permutation it is equivalent to the Kleisli resolution of the monad $\mor{(-) \times \mathcal{Z}_3}{\catw{Set}}{\catw{Set}}$, but I'm not sure if this is deep or meaningless...). -One may easily see that $\overline{\mathcal{Z}_3}$ is not self-dual, thus cannot be the smallest limit and colimit completion (since $\mathcal{Z}_3$ is self-dual, if $\overline{\mathcal{Z}_3}$ was the smallest, then $(\overline{\mathcal{Z}_3})^{op}$ would be the smallest). Moreover, $\overline{\mathcal{Z}_3}$ satisfies the following properties: - -its every object is a colimit over $\mathcal{Z}_3$, -its every object is a double-limit over $\mathcal{Z}_3$ - -Therefore (by the second property) $\overline{\mathcal{Z}_3}$ is the smallest reflective subcategory of $\catw{Set}^{\mathcal{Z}_3^{op}}$ containing representables. -4) The Dedekind-MacNeille construction as described in Todd's answer works when the monad induced by the Isbell conjugation is itself idempotent. For example, this is always true in the world of posets, in the world of Lawvere metric spaces, and more generally in the world of categories enriched over affine quantales (because in such a case every enriched monad is idempotent). -I actually think that only Dedekind-MacNeille completitions for categories enriched over (co)complete posets deserve the name. -The reason is that there is a subtlety here (which may look minor at first, but I think is crucial for the whole construction) that makes that the direct categorification of the original Dedekind-MacNeille requirements for completion have no sense --- the completion should be complete and cocomplete, which means that it should be closed under all limits/colimits. On the other hand, the term "a category is complete and cocomplete" means that the category is closed under small limits and small colimits (obviously, there is no (co)completion under all (co)limits in a $\catw{Set}$-enriched world). Therefore, the direct categorification of the requirement is possible only when the base of the enrichment has all limits/colimits, which in turn, implies that the base of the enrichment is a poset (at least in the classical mathematics). -As I said, the distinction between small and all may look minor at first, but in fact the distinction is not about quantity, but about quality. This distinction shows up on many occasions. For example, constructively if a category is complete (with respect to all cones) then it is also cocomplete (with respect to all cocones), and every colimit can be expressed as a limit; however, this is no longer true if the category is only small complete --- the cone from the canonical representation of a small cocone may be large, and there may be no reason for its limit to exists.<|endoftext|> -TITLE: Iwasawa mu-invariant for abelian extensions of quadratic number fields -QUESTION [12 upvotes]: Let K be a number field and $p$ an odd prime. Let $\mu$ be the Iwasawa $\mu$-invariant of the class group of the cyclotomic $\mathbb{Z}_p$-extension of $K$. If $K$ is abelian over $\mathbb{Q}$ then it is known that $\mu=0$ (Ferrero-Washinton, see Washington 7.5). Iwasawa conjectured that $\mu=0$ for all $K$. -Is something known for the case when $K$ is abelian over an imaginary quadratic field $k$ ? - -REPLY [2 votes]: If I am not mistaken, it proves that mu invariant of the Z_p times Z_p extension is 0 and this was Schneps's thesis. It is unfortunately not enough to show the conjecture of Iwasawa in this case even using the vanishing of anticyclotomic mu invariant proven by Hida. -Edit: In fact, what Schneps proves is that the mu invariant of the Z_p extension in which only one of the primes above p is ramified.<|endoftext|> -TITLE: A Question on Random Matrices -QUESTION [14 upvotes]: Consider the following $n\times n$ random matrix $V_{n}$ where the $(p,q)$ entry is given by -$$ -V_{n}(p,q):= \frac{1}{\sqrt{n}}\exp(2\pi i(p-1) x_{q}) -$$ -where $x_{1},x_{2},\ldots,x_{n}$ are iid random variables with uniform distribution on $[0,1]$. -It is not difficult to prove that $V_{n}^{*}V_{n}$ has the same eigenvalues as $X_{n}$ where -$$ -X_{n}(p,q)=\frac{\sin(n(x_p-x_q)/2)}{n\sin((x_p-x_q)/2)}. -$$ -This matrix is positive definite and invertible with probability one. The minimum eigenvalue, $\lambda_{1}(n)$, goes to zero as $n\to\infty$. I'm interested in the rate at which this eigenvalue goes to zero. Simulations suggest that -$$ -\mathbb{E}(\lambda_1(n))\sim \exp(-\alpha n), -$$ -the expected value decays exponentially. Using the Cauchy interlacing theorem I can only get the upper bound $O(\frac{1}{n^2}).$ -Any idea of what can work here? -Thanks! ---Gabriel - -REPLY [12 votes]: It is actually more like $e^{-\sqrt n}$. Let's look at the norm of the inverse matrix. The entries are $\pm\prod_{i:i\ne j}\frac 1{z_j-z_i}\sigma_m(z_1,\dots,z_{j-1},z_{j+1},\dots,z_n)$ where $z_k=e^{2\pi i x_k}$ is a random point on the unit circle and $\sigma_m$ is the $m$-th symmetric sum. Since $\log |Z-z_j|$ has zero mean and finite variance, you expect the first factor to be $e^{O(\sqrt n)}$ most of the time. The size of second factor is essentially the size of the random polynomial $\prod_i(z-z_i)$ on the unit circle. The typical value at one point is $e^{O(\sqrt n)}$ and we need about $n$ points to read the true maximum (plus we have $n$ rows to serve), so my educated guess (which I can try to convert into a proof if this subexponential dependence is of any value for you) would be $e^{-\alpha \sqrt{n\log n}}$ with some $\alpha$ (with high probability; the expectation may be a bit larger because there is a chance that the rare large values will still dominate). -I hope it makes sense but I'm in quite a hurry right now, so accept my apologies if I said some nonsense somewhere.<|endoftext|> -TITLE: square root of diffeomorphism of R: does it always exist? -QUESTION [23 upvotes]: Let $f:\mathbb R\to\mathbb R$ be a smooth, orientation preserving diffeomorphism of the real line. -Is it the case that there always exist another diffeomorphism $g:\mathbb R\to\mathbb R$ -such that $g\circ g = f$? -Note: it is relatively easy to show that a continuous $g$ exists, but I am not managing to find a smooth (i.e. $\mathcal C^\infty$) solution of the above equation. - -REPLY [2 votes]: Here this problem (and some more general) are studied for the case $f$ has only one fixed point. In general, one can get $C^1$ square roots, but for higher smoothness there are obstructions when the derivative at the origin is the identity which are studied there.<|endoftext|> -TITLE: Kähler metrics for projective space that are not the Fubini-Study metric -QUESTION [6 upvotes]: For projective $N$-space $CP^{N}$, there is a canonical Kähler metric called the Fubini-Study metric. Do there exist other Kähler metrics for $CP^N$. If so, is there any classification of such metrics? -More generally, how does this work for the Grassmanians, or even flag manifolds? - -REPLY [4 votes]: This is not a classification, but you can get a grip on the space of Kahler metrics on $CP^N$ using Bergman metrics and the Segre embeddings. -To explain this conside let $\{s_\alpha\}$ be a basis of homogeneous polynomials of degree k in N+1 variables. This gives an embedding $CP^N\to CP(H^0(O(k)))$. Now define a metric on H^0(O(k)) by declaring that the s_i are orthonormal. This defines a Fubini-Study metric on P(H^0(O(k)) and which pulls back to a metric on $\omega'$ on $CP^N$. -Now it can be proved that any Kahler metric $\omega$ on $CP^N$ is the limit of metrics of the form $k^{-1}\omega'$ for suitable bases $\{s_\alpha\}$ as $k$ tends to infinity (You can take the $\{s_\alpha\}$ to be orthonormal with respect to the $L^2$-metric induced by $\omega$). -In fact there is nothing special about $CP^N$ here. The above works for any projective manifold $X$ with ample line bundle $L$.<|endoftext|> -TITLE: Group theory required for further study in von Neumann algebra -QUESTION [12 upvotes]: After over half a year's study on operator algebra (especially on von Neumann algebra) by doing exercises in Fundamentals of the theory of operator algebras 1, 2 --Kadison, I was told that the current research focus is on the Ⅱ1 factor, and certain background on group theory is necessary, such as studying the free product, specific group construction and the ergodic action. Then, I want to know are there any good books on the group theory that might be necessary for further study on von Neumann theory? - -REPLY [15 votes]: As for a book on group theory that may be useful or interesting to read for further study of $II_{1}$ factors, I think that de la Harpe's book Topics in Geometric Group Theory is good for this. The reason I say this is that geometric group theory is concerned with the "large scale" structure of groups, and concerns ways that groups can be equivalent in ways that are weaker than group isomorphism. A lot of contemporary $II_{1}$ factor theory is also concerned with weak equivalence of groups and their measure-preserving actions. I'll say a bit more below, for context. -Before I do, though, let me mention that you should check out Sorin Popa's ICM talk, Deformation and rigidity for group actions and von Neumann algebras, a preprint listed on his website, and read all of it. This gives a really good intuition about a big part of what's going on in the subject right now, and says everything I'll say here and more. -One classical construction of a $II_{1}$ factor using a group $G$ is the (left) group von Neumann algebra, i.e. the commutant of the right regular represention of an i.c.c. (all nontrivial conjugacy classes are infinite) group G in $B(\ell^2 G)$. If two i.c.c. groups are isomorphic, then certainly their group von Neumann algebras are too. On the other hand, it is very difficult in general to tell if the group von Neumann algebra construction "remembers" the group used to construct it. For example, any two i.c.c. amenable groups have isomorphic group von Neumann algebras, so if you begin with an i.c.c. amenable group and whip up the group von Neumann algebra, it won't remember which group you used, but only will remember the amenability. It turns out that this construction is also sensitive to Kazhdan's Property (T), the Haagerup property (a weak amenability that is strongly non-(T)), in the sense that these properties are reflected in the structure of the von Neumann algebra. This construction is also sensitive to freeness, as in the freeness of the generators of a group. (See Gabriel's answer above.) Gromov's hyperbolicity is also reflected in the structure of the group von Neumann algebra, in that this "large scale" property severely governs the structure of the von Neumann algebra: this construction for Gromov hyperbolic groups give rise to solid factors. These things all seem to be "global" group properties, and so this is why I'm suggesting geometric group theory. -We're sort of listening for echos of the group in the von Neumann algebra built from it... -The broad question is: What properties of a group survive the construction of a $II_{1}$ factor using that group? -Another classical construction of a $II_{1}$ factor the group-measure space construction, which in modern terms is the way we build the crossed product von Neumann algebra from a discrete group and an ergodic measure-preserving action of that group on a standard Borel space. Check out Popa's above-mentioned ICM talk for more weak equivalences for groups surrounding this construction. -If you look at other ways of constructing $II_{1}$ factors, you can consider the same question. -Good luck with your study of von Neumann algebras!<|endoftext|> -TITLE: amenable equivalence relation generated by an action of a non-amenable group -QUESTION [8 upvotes]: Question. Give a (possibly elementary) example of a probability measure preserving action $\rho\colon G \curvearrowright X$ of a finitely-generated discrete group $G$ on a standard borel space $X$ with a probability measure, such that - -the equivalence relation generated by $\rho$ is ergodic and amenable, -the action $\rho$ is faithful, -the group $G$ is non-amenable. - - -A friend of mine asked me this question couple of days ago, which led us to another question, but perhaps there is an easier way to provide an example. - -REPLY [6 votes]: The answer is yes, such an action exists. -What is needed for the construction is the following very nice example of an action of a non-amenable group on $\mathbb Z$, which I just learned from Gabor Elek. -Consider a graph with vertices given by $\mathbb Z$ and unoriented edges between $n$ and $n+1$. -Pick a random labelling of the edges by the letters $a,b$ and $c$ with no $a$, $b$ or $c$ adjacent to the same letter. This defines an action of the group $G=\mathbb Z/2 \mathbb Z \ast \mathbb Z/2 \mathbb Z \ast \mathbb Z/2 \mathbb Z$. Indeed, just act according to existing labels or fix the element. -This action has the nice feature that it keeps invariant all counting measures on $\mathbb Z$, i.e. all $\mathbb Z$-Folner sequences sets are also Folner sequences for the $G$-action. -Now, the space of labellings (as above) of the graph is itself a probability measure space (a Bernoulli space), which carries an ergodic p.m.p. $\mathbb Z$-action by shifting. It is easy to see that $G$ acts on this space by measure preserving transformations (just by the method described above, done orbit by orbit) and induces an action as required. Indeed, the orbits are just the $\mathbb Z$-orbits, so its ergodic and amenable. Faithfulness follows the fact that you considered all labellings, so that with positive probability (on the space of labellings), an element will act non-trivially. Note also that $G$ is not amenable. -EDIT: As requested, more details on the action. The elements of the shift space are maps $f: \mathbb Z \to \lbrace a,b,c \rbrace$ with $f(n) \neq f(n+1)$. A letter shifts $f$ to the right if $f(1)$ equals that letter, it shifts to the left, if $f(0)$ is equal to the letter; otherwise you fix $f$. It is obvious that the orbits are just the orbits of the shift-action of $\mathbb Z$. Hence, the induced equivalence relation is just the one induced by the action of $\mathbb Z$.<|endoftext|> -TITLE: Computing squaring operations in the Adams spectral sequence -QUESTION [9 upvotes]: This question is about the classical Adams spectral sequence. Squaring operations are defined on its $E_2$ term. I'd like to know how to compute some of the non-trivial operations, such as $Sq^2 ( c_0 ) = h_0 e_0$. I feel like this ought to be doable in the May spectral sequence, but I don't know the details. -I'm aware of some work of Milgram on the subject, but there are some problems with his approach because of indeterminacies of Massey products. -Thanks! - -REPLY [2 votes]: This is an answer Bob's question after my previous answer. It doesn't fit in a comment. -As a spectrum, $\mathrm{End}(H\mathbb{Z}/2)$ is known to be a product of Eilenberg-MacLane spectra, so it is the classifying spectrum $\mathbb{H} C_{*}$ of a chain complex $C_{*}$, i.e. it is in the image of the Eilenbeg-MacLane functor $\mathbb{H}$ from chain complexes to spectra. The work of Shipley shows that this functor is well behaved with respect to multipliciative structures, since the symmetric monoidal model category of chain complexes is Quillen equivalent to that of $H\mathbb{Z}$-algebra spectra. In particular $\mathbb{H}$ takes chain algebras to ring spectra. However, we know that the ring spectrum $\mathrm{End}(H\mathbb{Z}/2)$ is not an $H\mathbb{Z}$-algebra, so it is the classifying spectrum of a chain complex, but not the classifying ring spectrum of a chain algebra. -Your question would be answered in full generality if we understood what a ring spectrum structure over the classifying spectrum of a chain complex is, i.e. what structure on a chain complex $C_{*}$ is equivalent to an $S$-algebra structure on $\mathbb{H} C_{*}$. -This is an achievable task, since there are nice models for the functor $\mathbb{H}$ taking values on symmetric spectra, and if $X$ is a symmetric spectrum, it is easy to describe a ring spectrum structure in terms of maps of simplicial sets $X_p\wedge X_q\rightarrow X_{p+q}$. The outcome should be some sort of non-additive multiplication on the chain complex $C_*$. -Now, let me remind you that Leif Kristensen constructed a long time ago a chain complex $C_{*}$ whose cohomology is the Steenrod algebra (reference below). This chain complex is endowed with a multiplication, which induces the product on the Steenrod algebra in cohomology. However, Kristensen's multiplication is not a chain algebra structure on $C_{*}$ since it is left distributive but not right distributive. This could be an approximation to a solution to the previous problem. -So far, I've been very lazy to pursue this project. There'd be a lot of work to do, but the outcome can be very valuable and surprising, and it looks like a feasible project. -MR0159333 (28 #2550) Kristensen, Leif On secondary cohomology operations. Math. Scand. 12 1963 57–82. (Reviewer: J. F. Adams)<|endoftext|> -TITLE: M12 simple sporadic group -QUESTION [6 upvotes]: I've spent quite a bit of time studying the Mathieu Groups, and I own the ATLAS. -My question is about M12. It is based on the ternary Golay Code, and is the automorphism -group of a Steiner S(5,6,12) system. Now, all of these Steiner systems are isomorphic -up to labelling. The order of M12 is 95040, which is 132 x 720. Since there are -132 blocks in this Steiner system, one can see that the 720 or S6 piece merely scrambles -the six elements of the hexad. Then, the 132 part is just sending the elements of -one hexad to another, of which there are 132 ways to do this. -Can someone give me an intuitive construction for this, not just generators...would it -make sense to say that the (sharply) quintuple transitive action might be to send -block 1 to 2, and 2 to 3, and perhaps another action to send block 1 to 3, 3 to 5 etc. or -something of this nature? Is there a hard and fast way to look at this action (M12) in -terms of the blocks? Or was I wrong about the 720 X 132 decomposition of the order of -the group...Thanks, Paul. - -REPLY [3 votes]: If you want an intuitive presentation of M12, also take a look at Curtis's construction.<|endoftext|> -TITLE: Karhunen–Loève approximation of Brownian motion and diffusions -QUESTION [12 upvotes]: The Karhunen–Loève theorem says that Brownian motion on the interval [0,1] can be represented as follows: -$W_t = \sum_{n=1}^\infty Z_n \frac{\sin((n-1/2)\pi t)}{(n-1/2)\pi},$ -where $Z_n \sim \mathcal{N}(0,1)$ \re i.i.d random variables. -Suppose we truncate this sum at some finite $N$, and define the process -$V_t = \sum_{n=1}^N Z_n \frac{\sin((n-1/2)\pi t)}{(n-1/2)\pi}.$ -Let $V^{(k)}$ be a vector of $k$ i.i.d copies of $V$. -We can define a new process -$X_t = \int_0^t \mu(X_t) dt + \int_0^t \sigma(X_t)dV^{(k)}_t,$ -where the second integral is defined pathwise as a Lebesgue-Stieltjes integral. As $N \rightarrow \infty$, will this process converge weakly to a diffusion in either the Ito or Stratonovich sense? -Karatzas and Shreve's book discusses the the one-dimensional case - one can show strong convergence, so clearly weak convergence follows too. They give a caveat about strong convergence of multidimensional processes, which is partly what prompted this question. -I'm interested in weak convergence for multidimensional processes, since this might give an interesting alternative to the Euler-Maruyama numerical approximation scheme. This is a fairly straightforward idea, so I'm sure there are results in the literature. I'd appreciate any references you could provide. -Many thanks. - -REPLY [4 votes]: Even in the multidimensional case, from the Wong-Zakai theorem, the sequence of processes which are solutions of the equation -$X^N_t=\int_0^t \mu(X^N_s) ds+\int_0^t \sigma(X_s^N)dV^N_s -$ -will converge uniformly in probability on the interval $[0,T]$ to the solution of the Stratonovitch stochastic differential equation -$X_t=\int_0^t \mu(X_s) ds+\int_0^t \sigma(X_s)\circ dW_s -$ -A survey on Wong-Zakai type theorems is -Wong-Zakai approximations for stochastic differential equations -By using the more recent rough paths theory, we can see that this convergence also holds in the $p$-variation topology for $2
-TITLE: Bounds on the size of sets not containing a given finite pattern
-QUESTION [6 upvotes]: Recall the following version of Szemerédi's Theorem: let $r_k(N)$ be the largest cardinality of a subset of $[N]:=\{1,\ldots, N\}$ which does not contain an arithmetic progression of length $k$. Then, for any $k\ge 3$, $r_k(N)/N\to 0$ as $N\to\infty$.
-It follows that the same is true for any finite pattern. i.e. if $A\subset\mathbb{N}$ is a finite set, then, if $r_A(N)$ is the largest cardinality of a subset of $[N]$ which does not contain a set of the form $t+n.A$, we again have $r_A(N)/N\to 0$ as $N\to\infty$. This is obvious since $A\subset [\max A]$.
-For $k=3$, Tom Sanders has recently substantially improved the best known upper bound for $r_k(N)$, namely $O(N/\log^{1-o(1)}N)$. I believe for $k=4$ the current "world-record" is due to Green and Tao and for $k>4$ to Gowers (corrections welcome).
-My question is about quantitative bounds for $r_A(N)$. Obviously, $r_A(N)\le r_{\max A}(N)$, but if $A$ is sparse this is likely to be far from optimal. Can one do better? Are there any quantitative results for more general sets $A$?
-In particular, is it true that $r_A(N)$ ``behaves like'' $r_{|A|}(N)$?
-To be concrete, what type of bounds can one get for $r_A(N)$ when $A=\{1,2,m\}$? Will they be more like $r_3(N)$, or more like $r_m(N)$ (or something strictly in between)?
-
-REPLY [4 votes]: Your question is quite broad, I won't be able to answer everything.
-Let me start by answering your most concrete question regarding the pattern
-$ \{1,2,m\} $. I prefer to use $\{0,1,k\}$, which is equivalent.
-Recall that the existence of a three term arithmetic progression can be recast as asking for a soution to $x-2y+z=0$ with distinct $x,y,z$ in the subset.
-Similarly, for the set you ask for one can recast the question whether there is such a pattern into the question whether there is a solution to the equation
-$$(k-1)x - k y + z =0$$
-with distinct $ x, y, z$ in the subset. (Note that the sum of the coefficients is $0$.)
-By contrast, one cannot encode the existence of a $k$-term arithmetic progressions, for $k>3$, by a single equation of the above form. Instead, one needs to consider a system of linear equations.
-For all I know this is the crucial difference between $3$-term and $k$-term, with larger $k$, arithmetic progressions; at least, if a Fourier-analytic approached is used (as in the papers you mentioned); other approaches often yield only much weaker bounds or no explcit bounds at all.
-So, the problem for $\{1,2,m\}$ is 'close' to $3$-term arithemtic progressions.
-When comparing problems of this form a way to compare them is to compare the encoding of the 'pattern' as a solution to a (system) of linear equations.
-Roughly, if you can encode it with one equation (and the sum of the coefficients is $0$) then it is close to $3$-term arithemtic progressions if not, then not.
-In particualr, the number of variables in the equation is not the main problem (in some sence, it even helps to have more variables; however to guarantee that the variables all have distinct values can then become more of a problem), it is the number of equations.
-See for example this paper by Liu and Spencer http://www.math.ksu.edu/~cvs/liu_spencer-roth_group.pdf
-dealing with arbitrary linear equations (subject to some technical conditions) in finite abelian groups; but the integer problem is basically the problem for cyclic groups.
-(There is also a second part of this paper.)
-So, it really depends on the precise form of the pattern how hard the problem is; as you suspected its maximum is not a very good measure.
-
-To compare different systems of linear equations with respect to problems of this form one needs to compare the right parameters.
-I am not the right person to explain this in detail; however, for example, the recent paper of Gowers and Wolf 'The true complexity of a system of linear equations' should give an impression of this.
-And, there is a large variety of (recent) work related to this circle of ideas, also studying higher dimensional analogues or 'polynomial progressions' (e.g., Bergelson and Leibman, Polynomial extensions of van der Waerden's and Szemerédi's theorems).
-
-A final remark: as far as I know, the fact that the bounds for $r_3$ and $r_k$
-are so relatively far appart might well not be due to the true behavior of these functions, but due to the fact that in one case the bound was provable and in the other case it is not (yet) provable. For example, it is a conjecture of Erdős that for $S$ a subset of the natural numbers the divergence of $\sum_{s\in S}1/s$ implies the existence of $k$ arithmetic progression,for any $k$, in $S$, which roughly translates into a bound of $N/\log N$ for $r_k$ for any $k$.
-Moreover, while the recent progress for $r_3$ that you mentioned is impressive, it is not (or at least not known to be) close to optimal; however, it is close to establishing the above mentioned conjecture for $k=3$. The best known lower bound (Behrend recently improved by Elkin) is roughly only of the form $N \exp(- c \sqrt{N})$ for a $c > 0$, which is smaller than $N/ (\log N)^D$ for any $D$; and as far as I know some people believe this might be closer to the truth than the upper bound.
-Thus, if you ask about the size of your fucntions it is difficult to say close to which of the classical functions the functions you mentioned are, as even the classical ones are not completely understood (I believe not even conjecturally). However, how difficult it would be to prove bounds for your functions can be judged to some extent by what I said in the middle.
-Much more could be said, but I have to stop.<|endoftext|>
-TITLE: Applications of the group completion theorem
-QUESTION [9 upvotes]: I've been reading Hatcher's exposition of the Madsen-Weiss theorem here. One of the key results needed is the group completion theorem. Hatcher gives several applications of the group completion theorem (the Barratt-Priddy theorem relating the stable cohomology of the symmetric group to the stable stems, the description of the loop space associated to the stable braid group, the Madsen-Weiss theorem, and Galatius's theorem on the stable cohomology of automorphism groups of free groups). Looking around the internet, I have not been able to locate other applications. The above theorems are all proven in kind of the same way (using "scanning"). Can people provide other interesting applications of the group completion theorem?
-This should probably be community wiki, but I can't seem to find the box to check to make it so.
-
-REPLY [3 votes]: I use the group completion theorem quite a lot.
-For example, I used it (along with Yang-Mills theory and work of Tyler Lawson) to study the spaces Hom$(\pi_1 S, U)$ and Hom$(\pi_1 S, U)/U$ when $S$ is an aspherical surface (or a product of aspherical surfaces) and $U =$ colim $U(n)$ is the infinite unitary group. These spaces turn out to be the homotopy group completions of the monoids $\coprod_n$ Hom$(\pi_1 S, U(n))$ and $\coprod_n$ Hom$(\pi_1 S, U(n))/U(n)$, respectively. Here the monoid structure comes from block sum of unitary matrices. This means, in particular, that both of these representation spaces are infinite loop spaces (because block sum is commutative up to coherent isomorphisms). For orientable surfaces, the picture is quite nice:
-
-Hom$(\pi_1 S, U)/U \simeq (S^1)^{2g} \times \mathbb{C} P^\infty$.
-
-The first factor can be seen entirely via the determinants of representations ($g$ is the genus). The second factor gives a (non-canonical) 2-dimensional cohomology class (or line bundle) over the moduli space of representations. This should be a reflection of Goldman's symplectic form, but I don't have any idea how to prove that.
-The group completion story in this case is spelled out in quite a bit of detail in my paper "Excision for deformation K-theory..." (AGT, 2007). It goes back to the basic ideas of McDuff and Segal. The applications to surface groups show up in "The stable moduli space..." (Trans. AMS, 2011). You also can find these papers on my webpage or on the arXiv.<|endoftext|>
-TITLE: Why does so much recent work involve K3 surfaces?
-QUESTION [38 upvotes]: I've been noticing that a whole lot of papers published to the Arxiv recently involve K3 surfaces. Can anyone give me (someone who, at this point, knows little more about K3 surfaces than their definition) an idea why they are coming up so often?
-Some questions that might be relevant: Are there particular reasons that they are so important? Are there special techniques that are available for K3 surfaces, but not more generally, making them easier to study? Are they just "in vogue" at the moment? Are they more like a subject of research (e.g., people are carrying out some sort of program to better understand K3 surfaces) or a testing ground (people with ideas in all sorts of different areas end up working the ideas out over K3 surfaces, because more general versions are much more difficult)?
-
-REPLY [11 votes]: K3 surfaces are also interesting from the point of view of complex dynamics. To quote from Curtis T. McMullen's introduction to his paper ``Dynamics on K3 surfaces: Salem numbers and Siegel disks", Journal fur die Reine und Angewandte Mathematik 2002(545): 201–233,
-"The first dynamically interesting automorphisms of compact
-complex manifolds arise on K3 surfaces.
-Indeed, automorphisms of curves are linear (genus 0 or 1) or of finite
-order (genus 2 or more). Similarly, automorphisms of most surfaces (including
-$\mathbb{P}^2$, surfaces of general type and ruled surfaces) are either linear, finite
-order or skew-products over automorphisms of curves. Only K3 surfaces,
-Enriques surfaces, complex tori and certain non-minimal rational surfaces
-admit automorphisms of positive topological entropy [Ca2]. The automorphisms of tori are linear, and the Enriques examples are double-covered by
-K3 examples."
-In this paper McMullen gives examples of K3 surfaces admitting automorphisms with Siegel disks (i.e., domains on which the automorphism is conjugate to a rotation). There are countably many such surfaces, all of them non-projective. The citation [Ca2] is to the paper by S. Cantat, Dynamique des automorphismes des surfaces
-projectives
-complexes.
-CRAS Paris S ́er. I Math.
-328
-(1999), 901–906, which grew into a larger work: S. Cantat : Dynamique des automorphismes des surfaces K3 ; Acta Math. 187:1 (2001), 1--57
-These papers are only a few examples (perhaps of landmark character); there has been a lot of study of dynamics on K3 surfaces going on indeed.
-
-REPLY [5 votes]: In the mathematical physics community, there was recently a resurgence of interest for K3-surfaces due to the observation of Egushi, Ooguri and Tashikawa that the elliptic genus of K3-surfaces seems to be built out of representations of the Mathieu group M24. This phenomenon has been dubbed the "Mathieu moonshine". There is still no definitive understanding of this relation, as far as I know.<|endoftext|>
-TITLE: Why chain homotopy when there is no topology in the background?
-QUESTION [21 upvotes]: Given two morphisms between chain complexes $f_\bullet,g_\bullet\colon\,C_\bullet\longrightarrow D_\bullet$, a chain homotopy between them is a sequence of maps $\psi_n\colon\,C_n\longrightarrow D_{n+1}$ such that $f_n-g_n= \partial_D \psi_n+\psi_{n-1}\partial_C$. I can motivate this definition only when the chain complexes are associated to some topological space. For example if $C_\bullet$ and $D_\bullet$ are simplicial chain complexes, then for a simplex $\sigma$, a homotopy between $f(\sigma)$ and $g(\sigma)$ is something like $\psi(\sigma)\approx\sigma\times [0,1]$, whose boundary is $f(\sigma)-g(\sigma)-\psi(\partial\sigma)$.
-But chain homotopy also features in contexts where there is no topological space lurking in the background. Examples are chain homotopy of complexes of graphs (e.g. Conant-Schneiderman-Teichner) or chain homotopy of complexes in Khovanov homology. In such contexts, the motivation outlined in the previous paragraph makes no sense, with "the boundary of a cylinder being the top, bottom, and sides", because there's no such thing as a "cylinder". Thus, surely, the "cylinder motivation" isn't the most fundamental reason that chain homotopy is "the right" relation to study on chain complexes. It's an embarassing question, but:
-
-What is the fundamental (algebraic) reason that chain homotopy is relevant when studying chain complexes?
-
-REPLY [5 votes]: This was a comment, but I guess it was not so clear, although it is hiding in some of the above answers:
-There is an object in $Ch^+(R)$ that behaves like an interval. Think of the simplicial interval, and you get a chain complex $I_*$ over $R$ with $I_0=Re_0 \oplus Re_1$, $I_1=Rf_0$ and all other groups 0, with differential $d(f_0)=e_1−e_0$. This plays the role of the unit interval. Now a chain homotopy of two chain maps $f,g:A_∗ \to B_∗$ really is $H:A_∗ \otimes I_* \to B_*$ where the tensor product takes place in R chain complexes. I am not sure how well this object behaves with respect to the Dold-Kan maps, but it certainly is the normalized chains on $\Delta^1$. You could similarly look at maps of $R$ complexes from $I_*$ into $Hom_R(A_*,B_*)$.
-With this in hand you can form cylinder objects and cofiber sequences in the "obvious" way, and things start looking a bit more geometric.<|endoftext|>
-TITLE: Is it true that no one-dimensional group variety acts transitively on $\mathbb{P}^1$?
-QUESTION [6 upvotes]: This question may be trivial to people with the right background, but I do not see the answer.
-
-Let $\Bbbk$ be an algebraically closed field. Can any one-dimensional group variety (over $\Bbbk$) act transitively on $\mathbb{P}^1_{\Bbbk}$?
-
-Here is my reasoning so far:
--Every $g \in G$ fixes a point of $\mathbb{P}^1$ since the associated linear map has an eigenvector.
--Let $G_0$ be the identity component of $G$. If $G_0$ fixes a point $p$, then $G/G_0$ surjects onto the orbit of $p$, so $p$ has finite $G$-orbit. Hence $G$ does not act transitively. So, it suffices to assume $G$ is connected and show that it fixes a point.
--I think (but am not confident here) that the only one-dimensional connected group varieties are $(\Bbbk, +)$, $(\Bbbk^*, \cdot)$, and abelian varieties (i.e., elliptic curves). The last are not an issue since any morphism $E \to PGL_2$ is constant, where $E$ is complete and $PGL_2$ is affine.
--Since $G$ is one-dimensional irreducible and the isotropy subgroup of a point is closed, to show that $G$ fixes $p$, it suffices to show that infinitely many elements of $G$ fix $p$.
-The last step, I think I can do for the multiplicative group, and for the additive group if $\Bbbk$ has characteristic zero, but my basic line of approach does not seem to work for the additive group in characteristic $p$. I also am less than completely confident of the classification I've stated for one-dimensional connected group varieties; if someone could point me to a good reference for this, I would appreciate it.
-
-REPLY [9 votes]: The case of a 1-dimensional connected affine group (which in particular is solvable) is settled most easily by invoking the Borel Fixed Point Theorem: a connected solvable (affine) group acting as an algebraic group on an irreducible projective variety has a fixed point. If the given "group variety" is an abelian variety, of course, you have to argue separately. Similarly for a non-connected affine group, in which the connected component of the identity has a fixed point.
-ADDED: The question here involves only the 1950s work on algebraic groups ("group varieties") by Borel, Chevalley, Rosenlicht, ... By now parts of the theory have been streamlined, but anyway it's a little artificial to extract from the structure theory just the minimal amount of information you need for your purpose. However it's done you probably need to distinguish somewhere between characteristic 0 and characteristic $p$. Here is a more detailed outline of what might be argued:
-1) It's useful to reduce the problem first to the case where $G$ is connected, which is an elementary step.
-2) To handle the case when $G$ is not affine (thus an abelian variety of dimension 1), you implicitly rely on Chevalley's deep structure theorem on algebraic groups which was recently revisited by Brian Conrad: A modern proof of Chevalley’s theorem on algebraic groups, J. Ramanujan Math. Soc. 17 (2002), no. 1, 1–18.
-3) When $G$ is an abelian variety, it has no everywhere-defined regular functions (like other complete varieties) and thus can't have a nontrivial algebraic group morphism to an affine group such as $PGL_2$, which is shown concretely to be the automorphism group of the projective line (6.4 in my book).
-4) If $G$ is connected and affine of dimension 1, it is commutative: see 20.1 in my book, which doesn't yet get into the precise classification of such groups.
-5) To finish the argument as Peter suggests you need something like Lemma 21.1 in my book, which is the key step toward the Borel Fixed Point Theorem in 21.2.
-(That theorem is relatively easy to prove after some preparation, but is the key to further deep structure theory for reductive groups.) In Peter's sketch, you get only a bijective morphism from $G/H$ to the projective line, not necessarily an isomorphism; so 21.1 is needed in characteristic $p$. For me the best conceptual viewpoint is found in the fixed point theorem, which of course applies much more generally to actions of connected solvable groups on projective (or complete) varieties.<|endoftext|>
-TITLE: When is a quasi-isomorphism necessarily a homotopy equivalence?
-QUESTION [22 upvotes]: Under what circumstances is a quasi-isomorphism between two complexes necessarily a homotopy equivalence? For instance, this is true for chain complexes over a field (which are all homotopy equivalent to their homology). It's also true in an $\mathcal{A}_\infty$ setting.
-Is it true for chain complexes of free Abelian groups? The case I'm particularly interested in is chain complexes of free $(\mathbb{Z}/2\mathbb{Z})[U]$ modules or free $\mathbb{Z}[U]$ modules, but I'm also interested in general statements.
-
-REPLY [21 votes]: If your complexes are bounded, this is always true for any ring more generally replacing free modules with projectives. The statement is that $\mathrm{D}^b(A\text{-}mod)$ is equivalent to $\mathrm{Ho}(Proj\text{-}A)$ and you can find it in Weibel Chapter 10.4. If your complexes are unbounded things are more tricky. Then your statement is true in over any ring of finite homological dimension. Basically you have two notions K-projective(which have the property that you want) and complexes of projectives. Bounded complexes of projectives are K-projective, but unbounded ones are not unless you have the finiteness hypothesis(see Matthias' answer). See this post for the injective version of this story Question about unbounded derived categories of quasicoherent sheaves. In the cases you are interested in there is no problem.<|endoftext|>
-TITLE: Best way to find a closest vector in a lattice
-QUESTION [11 upvotes]: Let $v_1,\dotsc,v_n$ be linearly independent vectors in $\mathbb{R}^n$, and let $\Lambda=\bigoplus_{i=1}^n \mathbb{Z}v_i$. The question is, given a vector $w$ in $\mathbb R^n$, find the element $v$ of the lattice $\Lambda$ closest to $w$. It is assumed that an inner product structure has been imposed on $\mathbb{R}^n$. This is called the CVP (for closest vector problem).
-I've found various algorithms that can find the closest vector, but is there a particular algorithm that is especially simple to understand and to program? I want to test a few low dimensional examples, and easy to write computer code would be preferable.
-Any suggestion?
-
-REPLY [7 votes]: A nice rahter-simple option would be Babai's Nearest Plane Algorithm. This is an approximation algorithm which outputs a vector of the lattice which is "close" the given target $w$. The accuracy of the approximation depends on the rank $n$ of the lattice. The good things are: the algorithm runs in polynomial-time and it is fairly easy to implement.
-
-Input: Basis $B\in\mathbb{Z}^{m\times n}, w\in\mathbb{Z}^{m}$
-Output: A vector $x\in \mathcal{L}(B)$ such that $\lVert x - w\rVert \leq 2^{\frac{n}{2}} \:\text{dist}(w,\mathcal{L}(B))$
-
-Run $\delta$-LLL on $B$ with $\delta=3/4$.
-$b \leftarrow w$
-for $j = n$ to $1$ do
-$\qquad b=b-c_j b_j$ where $c_j = \lceil \langle b, \tilde{b}_j \rangle / \langle \tilde{b}_j, \tilde{b}_j \rangle\rfloor$
-Output $x:=w-b$
-
-
-Above, $\delta$-LLL denotes the Lenstra-Lenstra-Lovasz algorithm, used as a subroutine to obtain a $\delta$-LLL Reduced Basis $\lbrace b_1,\ldots,b_n \rbrace$ of your original basis $\lbrace v_j \rbrace$. The vectors $\lbrace \tilde{b}_j \rbrace$ are the Gram-Schmidt orthogonalization of the LLL basis.
-It seems that this algorithm is normally the first to be taught in university courses. I read these algorithms and definitions in Oded Regev's course about Lattices in Computer Science.
-For exacts algorithms, this short review might be helpful. Alternatively, this text is more technical but rather complete.<|endoftext|>
-TITLE: CM for primary ideal
-QUESTION [5 upvotes]: Let $R$ be a regular local ring, $I$ a prime ideal and $J$ an $I$-primary ideal in $R$. Is it true that if $R/I$ is CM then also $R/J$ is CM?
-This question is in some way the inverse of this one.
-
-REPLY [6 votes]: A useful way to think about this issue is to consider $J=I^{(n)}$, the $n$-symbolic power of $I$, which by definition is the $I$-primary component of $I^n$.
-When $R$ is a polynomial rings over $\mathbb C$, this is the ideal consisting of functions vanishing to order at least $n$ on $X = \text{Spec}(R/I)$.
-It is then well-known that the depth of $R/I^{(n)}$ can go down. For example, take $I$ generated by the $2\times2$ minors of the generic $2\times 3$ matrix inside the polynomial rings of the $6$ variables, localized at the maximal ideal of those variables. Then $R/I$ is Cohen-Macaulay of dimension $4$, but $R/I^{(n)}$ would have depth $3$ eventually. For more general statements about ideals of maximal minors, see for example Section 3 of:
-
-Powers of Ideals Generated by Weak d-Sequences, C. Huneke, J, Algebra, 68 (1981), 471-509.
-
-EDIT: the example above looks specific, but such examples should be abound. I expect most Cohen-Macaulay ideals which are not complete intersections to give an example (it is known that $R/I^n$, the ordinary powers, are CM for all $n>0$ iff $I$ is a complete intersection). The $2\times 2$ minors gives is a generic situation of non-complete intersection but CM ideal.
-A philosophical comment: it is unlikely that Cohen-Macaulayness will be preserved by basic operations on ideals. So if $R/I, R/J$ are CM, we do not expect $R/\sqrt{I}, R/I^n, R/I^{(n)}, R/P$ ($P$ an associated primes), or $R/(I+J), R/IJ$ etc. to be CM.
-The reason is that to preserve depth one needs to control the associated primes, and these operations only allow you to control the support. However, finding an explicit example is usually not so obvious.<|endoftext|>
-TITLE: A rank 3 geometry for the sporadic simple group of Suzuki
-QUESTION [7 upvotes]: I am actually studying coset geometries (in the sense of Tits and Buekenhout) for the sporadic simple group of Suzuki. I came aware that Buekenhout found in 1979 a geometry over the following diagram
- c 6
-O----------O----------O
-1 4 4
-
-However, I couldn't find any information about the maximal (or minimal) parabolic subgroups of this geometry.
-Has anyone ever studied this geometry? Is there a paper where I could find the informations I am looking for?
-As usual, thanks in advance!
-
-REPLY [7 votes]: I finally found the maximal parabolic subgroups of this geometry. Let us first denote the types of the elements with 0,1 and 2 when reading the diagram from left to right, and let us denote with $G_0$, $G_1$ and $G_2$ the stabilizer of an element of type 0, 1 and 2 respectively. Then we have:
-$$
-G_0 = G_2(4),\quad G_1 = 2^{2+8}:(A_5 \times S_3),\quad G_2 = 2^{4+6} : 3 A_6
-$$
-which are all maximal subgroups of $Sz$, and the Borel is $$B = 2^{12}.3^2$$
-Historically, I read that this geometry was built using polar spaces (see Francis Buekenhout, Diagrams for geometries and groups, Journal of Combinatorial Theory A, 27, 121-151, 1979 doi:10.1016/0097-3165(79)90041-4). However, I have not studied yet how to build it geometrically.<|endoftext|>
-TITLE: Maximal number of directed edges in suitable simple graphs on $n$ vertices without directed triangles.
-QUESTION [8 upvotes]: We consider the class $C$ of directed simple (no multiple edges) graphs having the property that every vertex is reachable by a directed path from every other vertex.
-Given an integer $k$, what is the maximal possible number of (directed) edges in a graph of $C$ with $n$ vertices
-such that there are no directed cycles of length $\leq k$?
-For $k=2$ this means simply
-that the existence of an edge from $v$ to $w$ forbids the existence of an edge from $w$ to $v$ and one can thus choose arbitrary orientions (giving rise to a graph in $C$) on the edges of the complete unoriented graph.
-For $k=3$, one has also to forbid oriented triangles which is not possible by orienting all edges of a complete graph on $n\geq 3$ vertices such that the result is in $C$.
-On the other hand, there are of course no triangles by choosing arbitrary orientations
-(giving rise to an element in $C$) of a complete bipartite graph.
-There are thus such graphs having roughly $n^2/4$ directed edges.
-There should be better upper and lower bounds.
-Motivation: G. Higman (A finitely generated infinite simple group. J. London Math. Soc. 26,
-(1951). 61--64) constructed finitely generated infinite simple groups
-by considering quotients of the finitely presented group $$\langle g_1,\dots,g_n|g_{i-1}^{-1}g_ig_{i-1}=g_i^2\rangle$$
-where indices are modulo $n$.
-This group is trivial for $n=2,3$ and infinite for $n\geq 4$. Given a directed graph,
-one can consider the corresponding group-presentation with generators corresponding to vertices and directed edges corresponding to relations $a^{-1}ba=b^2$. The triviality
-of the group constructed by Higman associated to $n=2,3$ implies that we want to avoid
-oriented cycles of length $\leq 3$ when searching for interesting examples.
-
-REPLY [8 votes]: Updated 4/17/11:
-(Originally, this answer contained a different proof of the result below for $k=3$. Not only did the proof not generalize, but it was wrong.)
-
-The maximum number of edges in a strongly-connected digraph with $n \geq k+1$ vertices and no cycles of length at most $k$ is $${\binom{n}{2}} - n(k-2) + \frac{(k+1)(k-2)}{2}.$$
- (A digraph where every vertex is reachable from every other vertex by a directed path is called strongly connected.)
-
-Gordon Royle conjectured this bound an gave an example achieving it for $k=3$. For general $k$ and $n$ the bound is attained by the following construction, almost identical to the one provided by Nathann Cohen in the comments:
-Let vertices $x_1,x_2,\ldots,x_{n-k+2}$ form a transitive tournament with $x_i \to x_j$ being an edge for all $1 \leq i < j \leq n-k+2$. Now delete the edge $x_1 \to x_{n-k+2}$ and replace it with a path $x_{n-k+2} \to x_{n-k+3} \to \ldots \to x_n \to x_1$. (The vertices $x_{n-k+3},\ldots, x_n$ will have in-degree one and out-degree one in the resulting graph.)
-It remains to prove that the above number is a valid upper bound. The proof is by induction on $n$.
-Simple counting shows that the bound is valid if $G$ is a directed cycle. It is tight if $G$ is a cycle of length $k+1$. Assume now that $G$ is not a cycle. Then there exist $\emptyset \neq X \subsetneq V(G)$ such that $G|X$ is strongly connected. (For example, one can choose the vertex set of any induced cycle in $G$.) Choose $X$ maximal subject to the above. Let $u \to v_1$ be an edge of $G$ with $u \in X$, $v_1 \not \in X$, and let $P$ be a shortest path in $G$ from $v_1$ to $X$. Let $P=v_1 \to v_2 \to \ldots \to v_l \to w$.
-Note that adding to $G|X$ any path starting and ending in $X$ produces a strongly connected digraph. It follows from the choice of $X$ that any non-trivial such path must include all the vertices in $V(G)-X$. In particular, if $l\geq 3$, $v_2,\ldots,v_{l-1}$ have no neighbors in $X$.
-Let us further assume that $u$ and $w$ are chosen so that the directed path $Q$ from $w$ to $u$ in $G|X$ is as short as possible. (Perhaps, $w=u$.) Then $V(P) \cup V(Q)$ induces a cycle in $G$, and so $v_1$ and $v_l$ have at least $k-2$ non-neighbors on $V(P) \cup V(Q)$. At least $k-l$ of those non-neighbors are in $X$ if $l\geq 2$. Therefore there are at least $k-2$ non-edges (pairs of non-adjacent vertices) between $X$ and $V(G)-X$ if $l=1$, and at least
-$$2(k-l)+(l-2)(k+1) \geq l(k-2)$$
-non-edges if $l \geq 2$.
-By the induction hypothesis there are at least $|X|(k-2)- \frac{(k+1)(k-2)}{2}$ non-edges between vertices of $X$, and therefore at least
-$$(l+|X|)(k-2)- \frac{(k+1)(k-2)}{2}=n(k-2) - \frac{(k+1)(k-2)}{2}$$
-non-edges in total, as desired.<|endoftext|>
-TITLE: Characteristic classes of lifted bundles
-QUESTION [6 upvotes]: Suppose $V$ is a vector bundle with structure group $SO(3)$, and suppose that it can be lifted to a $\text{Spin}(3) = SU(2)$ bundle (i.e. $w_2(V) = 0$). Let us call the lifted bundle $E$. Then it is stated on page 42 in The Geometry of Four-Manifolds by Donaldson and Kronheimer that we have the relation $p_1(V) = -4c_2(E)$. My question is:
-
-How does one show this?
-
-More generally, how does one compute the effect of going over to a lifted bundle on the characteristic classes? Generally one has $p_1(E) = c_1^2(E) - 2c_2(E)$. In our case the first term drops out, so that the claim in the book can also be written as $p_1(V) = 2p_1(E)$. This factor $2$ undoubtedly somehow comes from the covering homomorphism $SU(2) \rightarrow SO(3)$ which is 2 : 1, but how?
-Probably related: on the same page he defines at the bottom for the so-called instanton number $\kappa = \frac1{8\pi^2}\int_M\text{Tr}(F^2)$, and then claims that this is $\kappa = c_2$ for $SU(r)$ bundles $E$ and $\kappa = -\frac14p_1$ for $SO(r)$ bundles $V$. There again is that factor 4; again, from the formula $p_1(E) = c_1^2(E) - 2c_2(E) = \left[\frac{-1}{4\pi^2}\text{Tr}(F^2)\right]$ one would expect this to be 2. I can see that this factor is chosen so that if one lifts the bundle that $\kappa$ does not change, but on the other hand, not every bundle is liftable, and for bundles which have both Chern classes and Pontryagin classes (such as complex $SU(r)$ bundles), I would expect that one would want the two formulae to give the same answer. As it is, they don't.
-(I guess the real problem is I haven't managed to find any readable sources on $SO(r)$-bundles in the context of gauge theories. For special unitary groups there are sources in abundance, but for special orthogonal they are a lot harder to find.)
-
-REPLY [2 votes]: Let me rephrase your question so that I understand it: let $P \to X$ be an $SU(2)$-principal bundle. Then we get a $2$-dimensional complex vector bundle $E \to X$ by $P \times_{SU(2)} C^2$ (with the defining representation). And we get a $3$-dimensional real vector bundle $V:= P \times_{SU(2)} R^3$ (with the adjoint representation).
-You (or Donaldson-Kronheimer) claim that $p_1(V)=-4c_2(E)$.
-It is enough to consider the universal case $X=BSU(2)$. The map $BU(1) \to BSU(2)$ induced by the inclusion of the maximal torus is injective in cohomology, so it suffices to check your identity on $BU(1)$ (splitting principle). The pullback of $E$ to $BU(1)$ is $L \oplus L^{\ast}$, so its $c_2$ is a generator $\pm u$ of $H^4 (BU(1))$. The pullback of $V$ to $BU(1)$ is $L^2 \oplus \mathbb{R}$ (here the fact that $SU(2) \to SO(3)$ has degree comes in), hence the total Chern class of $V \otimes \mathbb{C}$ is $(1+2u)(1-2u)$, from which you can read off the Pontrjagin class. For the correct sign, you need a bit more care, though.
-EDIT: The inclusion of the maximal torus is the group homomorphism $U(1) \to SU(2)$; $z \mapsto diag (z,z^{-1})$. Thus the defining rep. of $SU(2)$ restricts to a sum of the defining rep. of $U(1)$ and its dual, giving the pullback of $E$.
-Consider the standard basis the Lie algebra $\mathfrak{su}(2)$ (http://en.wikipedia.org/wiki/Special_unitary_group#SU.282.29) and compute the action of $diag (z,z^{-1})$ via the adjoint rep in this basis. The result is that the adjoint rep. of $SU(2)$ restricts to a sum of the tensor square of the defining rep of $U(1)$ with the trivial real one-dim. rep. (this computation is basic in representation theory, it is the computation of the roots of $SU(2)$).<|endoftext|>
-TITLE: Is this a characterization of Dedekind domain?
-QUESTION [8 upvotes]: Let $R$ be an integral domain. Suppose that for any two nonzero ideals $I$ and $J$, we have $I \oplus J$ is isomorphic to $R \oplus IJ$ as $R$-modules. Does this implies $R$ is a Dedekind domain?
-
-REPLY [3 votes]: As Sampath has pointed out, we may assume that $R$ is local. Your hypothesis implies for any two non-zero ideals $I,J$, you have a surjection onto $R$. By Nakayama, this implies either $I\to R$ or $J\to R$ is surjective, since if neither is, then the both have images contained in the maximal ideal and so does their sum. But this means one of them is principal.
-Having recognized the confusion I caused by my terseness, let me be more explicit. First, my definition of DD is: R a domain (not a field) and for any non-zero ideal $I$, there exists another ideal $J$ such that $IJ$ is isomorphic to $R$. Easy to see that $\text{Hom} (I,R)$ for any non-zero ideal $I$ can be identified naturally with $J=\{x\in K|xI\subset R\}$ where $K$ is the fraction field of $R$. Then, the definition of DD means that for any non-zero ideal $I$, defining $J$ as above, $IJ=R$.
-Now assume that for any non-zero ideal $I$ of $R$, there exists a surjection $I\oplus I$ to $R$. Then, I claim that $R$ is a DD. The hypothesis implies, there exists $a,b\in I$, $x,y\in K$ with $xI,yI\subset R$ and $xa+yb=1$. Easy to check then that $I$ is generated by $a,b$. Thus all ideals are generated by atmost two elements and in particular $R$ is Noetherian. Now, by the above localization argument, $I$ is locally principal and the rest is clear. I hope this is clearer.
-Even without localizing, letting $J$ as above, we have $x,y\in J$ and thus $1\in IJ\subset R$ and hence $IJ=R$.<|endoftext|>
-TITLE: Is the group generated by two almost disjoint infinite cycles amenable?
-QUESTION [6 upvotes]: Let $x$ and $y$ be two permutations of $\mathbb{Z}^2$ defined as follows. The permutation $x$ sends $(n,0)$ to $(n+1,0)$ and fixes all else while $y$ sends $(0,n)$ to $(0,n+1)$ and fixes all else. Is the group generated by $x$ and $y$ amenable?
-I do know that the group does not contain a copy of the free group on two generators, so it is very likely to be amenable. I also know that if $y$ is defined, instead, by sending $(n,m)$ to $(n,m+1)$ then the group generated by $x$ and $y$ is amenable, in fact, it is a solvable extension of a locally finite group.
-
-REPLY [8 votes]: The derived subgroup of your group consists of permutations with finite support. Indeed, suppose that $w$ is a commutator word in $a$ and $b$ so the total exponent of $a$ (of $b$) is 0. Take a point $(m,n)$ where $m$ or $n$ are very large (comparing to $|w|$). Then $w(a,b)$ fixes that point. Therefore your group is an extension of a locally finite group by the Abelian group ${\mathbb Z}^2$, and is amenable.<|endoftext|>
-TITLE: Why are viscosity solutions useful solutions?
-QUESTION [23 upvotes]: I refer to definition of viscosity solution in user's guide to viscosity solutions of second order partial differential equations by Michael G. Crandall, Hitoshi Ishii and Pierre-Louis Lions.
-Viscosity solutions are generalized solutions which can be implied if the Sobolev theory (or similar) doesn't provide "useful" solutions. A standard example is the problem
-$|u'| = -1, u(-1) = 1, u(1)=1$
-All "zig-zag" functions with appropriate boundary conditions provide a solution, but $u(x)=|x|$ is the unique viscosity solution.
-But except its formal beauty why do we regard a viscosity solution as useful, and what is the 'physical' or 'intuitive' interpretation of being a viscosity solution?
-
-REPLY [23 votes]: Viscosity solutions are the "appropriate" notion of solutions for second-order elliptic equations in nondivergence form, and for some classes of first-order equations. Here is a brief summary of why.
-From the point of view of applications, the viscosity solution is almost always the right solution. For example, in optimal control theory, it has long been known that if the value function is smooth, it satisfies a certain PDE-- but the value is known to not be smooth in most cases. When Crandall and Lions invented viscosity solutions, it was clear immediately that the viscosity solution is precisely the value function. This story has played out over and over again, for many different applications, to the point that people in the field are completely shocked and stunned when there is some reason to consider a notion of solution other than viscosity solution.
-From a mathematical point of view, viscosity solutions are natural. For equations in nondivergence form, energy methods are unavailable. Therefore, all one usually has is the maximum principle. Viscosity solutions are to weak solutions as the maximum principle is to energy methods. The term "viscosity solution" is rather unfortunate in this sense-- they should be called "comparison solutions" or something. The point is, these equations should satisfy a maximum principle. So it makes sense to define your weak solution as a one for which the maximum principle holds when you compare to smooth functions.
-Finally, in the cases where there is overlap, like for the p-Laplacian, viscosity solutions are equivalent to (bounded) weak solutions in the integration-by-parts sense.<|endoftext|>
-TITLE: Boundaries of the eigenvalues of a symmetric matrix (or of its Lapacian)
-QUESTION [5 upvotes]: Given the adjacency matrix $A_{ij}$ of a graph with $N$ vertices and $M$ links (or any binary symmetric matrix of size $N \times N$), is it possible to establish lower and upper boundaries of its eigenvalues? I mean, do $N$ and $M$ determine the lowest and the largest possible eigenvalues of the matrix?
-In other words, let $L$ be the Lapacian of $A_{ij}$. We know that its eigenvalues obbey the following: $\lambda_1 = 0 \leq \lambda_2 \leq \lambda_3 \leq \ldots \leq \lambda_N$. The lowest eigenvalue is always $\lambda_1 = 0$ and hence, the eigenvalues of $L$ have a lower bound. Is there an upper bound for $\lambda_N$ that depends only on the size $N$ of the graph and/or on $M$?
-Stated yet in another manner. Does anyone know a graph $G'$ whose $\lambda'_N$ is largest than the $\lambda_N$ of any other graph of the same size $N$ and/or number of links $M$?
-Thank you!
-
-REPLY [7 votes]: There are several notions of Laplacian for graphs. For instance, there is the normalized Laplacian, the classical Laplacian and Laplacians with boundary conditions as the Dirichlet Laplacian and so on. Assuming that you are taking about the classical Laplacian then its eigenvalues are
-$$
-0=\lambda_1\leq \lambda_2\leq \ldots\leq\lambda_{n}
-$$
-where $n$ is the number of nodes. Typically it is $\lambda_2$ the eigenvalue that gives the most information about the graph (like expanding properties, connectivity, isoperimetric properties, etc). For instance, $\lambda_2>0$ iff the graph is connected.
-Here are a few known results:
-
-[Fiedler] $\lambda_2\leq \frac{n}{n-1}\min\{d(v):v\in V\}$ where $d(v)$ is the degree of the node $v$.
-[Anderson and Moreley] The maximum eigenvalue satisfies
-$$
-\lambda_n\leq\max\{ d(u)+d(v):uv\in E\}.
-$$
-If the graph $G$ is connected then the equality holds iff the graph is bipartite.
-[Kelmans] If the graph is simple (no multiple edges or loops) then $\lambda_n\leq n$ with equality iff the complement of $G$ is not connected.
-$\sum_{i}^{n}{\lambda_i}=2m=\sum_{v}{d(v)}$ where $m$ is the number of edges in $G$.
-[Fiedler] $\lambda_{n}\geq \frac{n}{n-1}\max\{d(v):v\in V\}$.
-
-The second and last bullet give you lower and upper bounds for the maximum eigenvalue in terms of the degree of the nodes. If the graph is regular of course much more can be said.
-I hope it helps!<|endoftext|>
-TITLE: A line bundle not big but with good intersection numbers
-QUESTION [6 upvotes]: Let $X$ be a complex projective manifold of complex dimension $n$ and $A\to X$ an ample line bundle. Let $L\to X$ be a line bundle such that
-$$
-c_1(L)^k\cdot c_1(A)^{n-k}>0,\quad k=1,\dots,n.
-$$
-Is it true that then $L$ is big?
-The answer is yes if $n\le 2$: for $n=1$ there is nothing to prove, and for $n=2$ the positivity of the top self intersection $c_1(L)^2>0$ says that $L$ or its dual is big. But then $c_1(L)\cdot c_1(A)>0$ implies that in fact $L$ is big.
-The answer is again yes in all dimensions if $X$ is an abelian variety: in this case $L$ is moreover ample. This is because one can represent $c_1(L)$ and $c_1(A)$ by "constant" hermitian forms, the second being positive definite, and thus the intersection conditions simply tell that the elementary symmetric polynomials in the eigenvalues of the hermitian form representing $c_1(L)$ are all positive. Thus, $L$ is positively curved and hence ample.
-I strongly suspect anyway that the result is false in general.
-Could you give for instance a counterexample in dimension three?
-Thanks in advance.
-
-REPLY [6 votes]: Here is a very straightforward contre-example. Let $X=\mathbb CP^2\times \mathbb CP^1$ blown up in one point. Denote by $E$ the exceptional divisor, and denote by $\pi$ the projection of $X$ to $\mathbb CP^2$, and take the following bundle:
-$$L_n=\pi^*(O(n))\otimes O(E),$$
-where $n$ satisfies two conditions: $$c_1(O(n))^2\cdot c_1(A)>-c_1(O(E))^2\cdot c_1(A),\;\;\;\;
-c_1(O(n))\cdot c_1(A)^2>-c_1(O(E))\cdot c_1(A)^2,$$
-it is obvious that such $n$ exists.
-To that this bundle is what you want we just need the following two simple facts: $c_1(\pi^*(O(n)))\cdot c_1(O(E))=0$ and $c_1(O(E))^3=1$. $L_{n}$ is not big because the $H^0(kL_n)$ grows quadratically with $k$.
-Idea of this example works in dimensions $2m+1$. We take a semi-ample line bundle $L_{sa}$ with Itaka dimension $2m$ http://en.wikipedia.org/wiki/Iitaka_dimension (in particular it is not big), and tensor it with a line bundle corresponding to an exceptional divisor $E$. We chose them so that $c_1(L_{sa})\cdot c_1(O(E))=0$, i.e., these bundles "don't interact". For large $n$ the class $(c_1(nL_{sa}))^k$ (provided
-$k\le 2m$) is represented by a cycle of a high degree (with respect to $A$), so it "beats" $(c_1(O(E)))^k$. Finally
- $E^{2m+1}=1$.<|endoftext|>
-TITLE: whence commutative diagrams?
-QUESTION [71 upvotes]: It seems that commutative diagrams appeared sometime in the late 1940s -- for example, Eilenberg-McLane (1943) group cohomology paper does not have any, while the 1953 Hochschild-Serre paper does. Does anyone know who started using them (and how they convinced the printers to do this)?
-
-REPLY [32 votes]: Eduard Study in Von den Bewegungen und Umlegungen, Math. Ann. 39 (1891) 441-566, writes on p. 508:
-
-Here $g, g^*, g'$ are rays in space with polar planes $\gamma, \gamma^*, \gamma'$, $\mathfrak P$ is the “polarity” taking rays to planes and conversely, the $\mathfrak t_i$ are commuting collineations acting on both rays and planes, and Study uses $\mathfrak t_i\mathfrak P$ to denote the composition we would write $\mathfrak P\circ\mathfrak t_i$.
-(Essentially the same diagram is repeated in Study’s book Einleitung in die Theorie der Invarianten linearer Transformationen auf Grund der Vektorenrechnung (1923, p. 217), with credit to (1891).)<|endoftext|>
-TITLE: Examples of "Unusual" Classifications
-QUESTION [7 upvotes]: When one says "classification" in math, usually one of a handful of examples springs to mind:
--Classification of Finite Simple Groups with 18 infinite families and 26 sporadic examples (assuming one believes the Classification is indeed complete)
--Classification of finte-dimensional semisimple Lie algebras with 4 infinite families and 5 exceptional examples
--Classification of (Simple, Formally Real) Jordan Algebras with 4 infinite families and 1 exceptional example
-I'm sure there are other examples that non-algebraists would think of before these. All the examples I cited take the basic form of having several infinite families and some number of exceptional examples which do not fall into any of these families. Thus I am wondering:
-
-
-Question: Does anyone know of examples of classifications of some mathematical objects such that the classification consists (A) only of infinite families or (B) only of a finite number of examples/ an infinite number of examples which do not seem to be closely related to one another (i.e. they do not "appear" to form any infinite families).
-
-
-One example of case (B) that comes to mind would be finite dimensional division algebras over $\mathbb{R}$ of which there are 4. On the other hand, for case (B) I would like to rule out way too specific "classifications" such as "finite simple groups with an involution centralizer of such and such a form" since this is really a subclassification within the classification of FSG's. Although I am an algebraist, I would like to hear about examples from any branch of math, for comparison's sake.
-(If anyone thinks of better tags for this, feel free to add or suggest them).
-
-REPLY [2 votes]: In extensions of number fields $E/k$, you can look at primes in the ring of integers of $k$ that either (a) split in the integers of $E$, (b) remain prime in the integers of $E$, or (c) ramify. The case (c) consists of finitely many exceptions.<|endoftext|>
-TITLE: Kontsevich's formality theorem from an explicit homotopy
-QUESTION [11 upvotes]: Suppose that $X$ is a smooth manifold, whose $C^{\infty}$-functions we denote by $A$. Let $D_{poly}^*(A):=\bigoplus_{n\geq -1}Hom(A^{\otimes n+1},A)$ be the Lie algebra of polydifferential operators with the Hochschild differential and the Gerstenhaber bracket. A version of the Hochschild-Kostant-Rosenberg theorem shows the cohomology of $D_{poly}^*(A)$ is isomorphic to $\bigoplus_{n\geq -1}\wedge^{n+1}T_X$, polyvector fields. The graded vector space of polyvector fields is also a Lie algebra with the Schouten-Nijenhius bracket. However, the HKR-isomorphism is not a morphism of Lie algebras. The formality theorem of Kontsevich shows the HKR-isomorphism can be corrected to an $L_{\infty}$-morphism whose first term is the HKR-isomorphism. The upshot is this proves star-products on $C^{\infty}(X)$ correspond to formal Poisson structures.
-In the article: M. de Wilde and P. B. A Lecomte: An homotopy formula for the Hochschild cohomology, Compositio Mathematica, tome 96, no. 1 (1995), the authors construct an explicit homotopy for the HKR-isomorphism. From this explicit homotopy the authors construct a star-product on $\frak g^*$, the dual of the Lie algebra $\frak g$. My question is the following: can one do the same thing for the general case of a Poisson manifold. In particular, is the explicit homotopy which induces Kontsevich's $L_{\infty}$-quasiisomorphism known? I believe that one has to exist since two $L_{\infty}$-algebras are quasiisomorphic if and only if they are homotopy equivalent. But, can you write down the explicit homotopy from the quasiisomorphism.
-
-REPLY [5 votes]: The only way I know to construct a formality quasi-morphism for poy-vector fields out of an homotopy is via Tamarkins approach (i.e. $G_\infty$-formality). What Tamrakin does is
-
-prove that there is a suitable $G_\infty$-structure on Hochschild cochains
-prove that the obstruction to construct a $G_\infty$-formality step by step is unobstructed. At each step you have to make some choice, and the homotopy for the Hochschild complex of De Wilde-Lecomte gives you a way to do such choices.
-
-But Tamarkin construction (part 1) involves the choice of an associator (i.e. choice of appropriate weights in Kontsevich's $L_\infty$-qausi-isomorphism)... so I think that this is hopeless to construct the formality out of an homotopy.
-Moreover, even the other way I don't see how you could associate an homotopy for the Hochschild complex to an $L_\infty$-quasi-isomorphism from $T_{poly}$ to $D_{poly}$.
-I might be wrong but I have the feeling that you are mixing two different notions of homotopy: that of higher homotopies in the $L_\infty$-morphism, and that of homotopy retract for the Hochschild cochain complex. In particular, what do you mean by a "homotopy equivalence" between $L_\infty$-algebras (whatever you mean, a homotopy for the Hochschild complex in the sens of De Wilde-Lecomte won't produce an example).<|endoftext|>
-TITLE: Unbalancing lights in higher dimensions
-QUESTION [16 upvotes]: In ''The Probabilistic Method'' by Alon and Spencer, the following unbalancing lights problem is discussed. Given an $n \times n$ matrix $A = (a_{ij})$, where $a_{ij} = \pm 1$, we want to maximise the quantity
-$x^T A y = \sum_{i=1}^n \sum_{j=1}^n a_{ij} x_i y_j$
-over all $n$-dimensional vectors $x$, $y$ such that $x_i,y_j = \pm 1$. The name of the problem comes from interpreting $A$ as a grid of lights that are on or off, and $x$ and $y$ as sets of light switches, each associated with a row or column (respectively); flipping a switch flips all lights in that row or column, and the goal is to maximise the number of lights switched on.
-Let $m(A)$ be the maximum of $x^T A y$ over $x$ and $y$ such that $x_i,y_j = \pm 1$. As Alon and Spencer discuss, for any $A$ it is possible to show that $m(A) \ge C n^{3/2}$ for some constant $0
-TITLE: Reference for Mathematical Economics
-QUESTION [17 upvotes]: I'm looking for a good introduction to basic economics from a mathematically solid(or, even better, rigorous) perspective. I know just about nothing about economics, but I've picked up bits and pieces in the course of teaching Calculus for business and social sciences, and I'd like to know more, both for my personal culture and to incorporate into my courses. My Platonic ideal of such a book would be along the lines of T W Körner's Naive Decision Making, but I'll take what I can get.
-
-REPLY [9 votes]: Let me first tell you that there are three types of economic theory:
-
-price theory
-general equilibrium
-game theory
-
-Nowadays, the last type of theory is by far the most popular, but maybe you need some familiarity with the other two to appreciate game theory as an economist does. I guess you could say the mathematics is very easy judged from what mathematicians are used to. What is difficult is the economic interpretation. For that you need a bit economic intuition and a bit of culture about classic economic models.
-My recommendations for a mathematically mature neophyte would be:
-
-george stigler's price theory for the first
-debreu's classic for general equilibrium; there is also a book by JWS Cassels
-Fudenberg & Tirole or Vega Redondo for game theory<|endoftext|>
-TITLE: The conformal group of $S^n$.
-QUESTION [12 upvotes]: Is there any explicit computation of Conf($S^n$, $g_{std}$), the group of conformal diffeomorphisms of the standard $n$-sphere?
-
-REPLY [4 votes]: Try the book A Mathematical Introduction to Conformal Field Theory by Martin Schottenloher. Chapters 1 and 2 go over some of the proofs you are looking for and the book is example driven.<|endoftext|>
-TITLE: Missing mass conjecture
-QUESTION [6 upvotes]: Let $n,t$ be positive integers and $p_1,p_2,\ldots,p_n$ positive numbers summing to 1. Conjecture:
-$$
-\sum_{i=1}^n p_i (1-p_i)^t \le \frac{n(1-1/n)^n}{t}
-$$
-always holds.
-The motivation comes from my missing mass question; the quantity $\sum_{i=1}^n p_i (1-p_i)^t$ is precisely the expected unseen mass after $t$ draws from the distribution $(p_i)$ on $n$ objects.
-
-REPLY [7 votes]: Since the single-variable optimization that David mentions still requires some work, I will present another solution. Let $f(p) = p(1-p)^t$. Define the function $g$ that is equal to $f$ on $[0, 1/t]$, and on $[1/t, 1]$ is a linear interpolation between the points $(1/t, f(1/t))$ and $(1, 0)$. Then we can check that $g$ is concave on all of $[0, 1]$ and $g \ge f$ on all of $[0, 1]$. (I learned this concept of "concave majorants" or "convex minorants" from Steele's book called The Cauchy-Schwarz Master Class.) Applying Jensen's inequality, we have $\sum f(p_i) \le \sum g(p_i) \le n g(1/n)$.
-We now split into two cases, $t \le n -1$ or $t \ge n$. First, suppose that $t \le n - 1$. Then $g(1/n) = f(1/n) = (1/n) (1 - 1/n)^t$. So we need to show that $(1 - 1/n)^t$ is at most $n (1 - 1/n)^n / t$. That's equivalent to showing $t(1 - 1/n)^t \le n (1 - 1/n)^n$. We can check that the left side is an increasing function of $t$ for $t \le n - 1$, and when $t = n - 1$ we have an equality. So we have established the inequality in this case.
-Next suppose that $t \ge n$. Then by linear interpolation, we find $g(1/n) = (1 - 1/t)^{t-1} (1 - 1/n) / t$. So we need to show that $(1 - 1/t)^{t - 1} (n - 1) \le n(1 - 1/n)^n$. That's equivalent to $(1 - 1/t)^{t-1} \le (1 - 1/n)^{n - 1}$. By taking reciprocals, that's equivalent to $(1 + 1/(t - 1))^{t - 1} \ge (1 + 1/(n-1))^{n - 1}$. The left side is an increasing function of $t$, and we have an equality when $t = n$. So we have established the inequality in the second case too.<|endoftext|>
-TITLE: Simple connectedness via closed curves or simple closed curves?
-QUESTION [5 upvotes]: I've recently read some papers and books involving simply connected domains in Euclidean space (dimension at least 2), where domain is an open connected set. The usual definition is a (connected) set for which every continuous closed curve is (freely) contractible while some authors only require that every continuous simple closed curve is contractible. The authors who define simple connectedness using simple closed curves do so in order to use Stokes' Theorem or the Jordan curve theorem somewhere in the sequel; however, they never mention (not even with a reference) that their definition is equivalent to the usual one! My question is if there is a proof written down somewhere (with all the details) proving the equivalence (for domains in $\mathbb{R}^n$ with $n \geq 2$)? If not, does someone know of an "easy" proof using a minimal amount of knowledge, say that of a first course in topology?
-
-REPLY [14 votes]: As pointed out by Pierre and Paul in comments, there are several standard ways to deal with this kind of issue. A good answer really depends what you're assuming you start from, and where you're trying to go to. The Jordan curve theorem and Stoke's theorem are both fairly sophisticated and difficult for beginners to grasp, so it's a bit hard to see how only analyzing embedded curves is streamlining anything, except perhaps helping with people's intuitive images---but even so, it may do more harm than good.
-Perhaps it's worth pointing out that this statement is false in greater generality, for instance for closed subsets of $\mathbb R^3$. Here's an example in $\mathbb R^3$:
-consider a sequence of ellipsoids that get increasingly
-getting long and thin; to be specific, they can have axes of length $2^{-k}$, $ 2^{-k}$ and $2^k$. Stack them in $\mathbb R^3$ with short axes contained in the $z$-axis, so each one touches the next in a single point with long axes parallel to the $x$-axis, and let
-$X$ be their union together with the $x$-axis.
-Any simple closed curve in $X$ is contained in a single ellipsoid, since to go from one to the next it has to cross a single point, so every simple closed curve is contractible.
-However, a closed curve in the $yz$-cross-section that goes down one side and back up the other sides is not contractible. The fundamental group is in fact rather large and crazy.
-Anyway, here are some lines of reasoning that can overcome whatever hurdle needs to be ovrcome:
-
-PL approximation, as suggested by Pierre: this is easy, the keyword is "simplicial approximation". I'll phrase it for maps of a circle to Euclidean space as in the question, even though essentially the same construction works in far greater generality. Given an open subset $U \subset \mathbb{R}^n$ and given a map $f: S^1 \to U$,
-then by compactness $S^1$ has a finite cover by neighborhoods that are components of $f^{-1}$ of a ball. If $U_i$ is a minimal cover of this form, there is a point $x_i$ that is in $U_i$ but not in any other of elements of the cover; this gives a circular ordering to the $U_i$. There is a sequence of points $y_i \in U_i \cap U_{i+1}$, indices taken mod the number of elements of the cover. The line segment between $y_i$ and $y_{i+1}$ is contained in $U_i$, since balls are convex. (This generalizes readily to the statement that for any simplicial complex, there is a subdivision where the extension that is affine on each simplex has image contained in $U$. It also generalizes readily to the case that $U$ is an open subset of a PL or differentiable manifold).
-Raising the dimension: if you take the graph of a map of $S^1$ into a space $X$, it is an embedding. If you're (needlessly) worried about integrating differential forms on non-embedded curves, pull the forms back to the graph, where the curve is embedded. If you want to map to a subset of Euclidean space with the same homotopy type, just embed the graph of the map
-(a subset of $S^1 \times U$ into $\mathbb R^2 \times U$. (There's a very general technique to do this, if the domain is a manifold more complicated than $S^1$, even when it's just a topological manifold, using coordinate charts together with a partition of unity to embed the manifold in the product of its coordinate charts).
-The actual issue for integration, using Stoke's theorem etc., is regularity --- to make it simple, restrict to rectifiable curves, and don't worry about embededness. Any continuous map into Euclidean space is easily made homotopic to a smooth curve, by convolving with a smooth bump function---the derivatives are computed by convolving with the variation of the bump, as you move from point to point.
-Similarly, you can approximate any continuous map by a real-analytic function, if you convolve with a time $\epsilon$-solution of the heat equation (a Gaussian with very small variance, wrapped around the circle). This remains in $U$ if $\epsilon$ is small enough. A real analytic map either has finitely many double points, or is a covering space to its image; in either case you reduce simple connectivity to the case of simple curves.
-Sard's theorem and transversality, as mentioned by Paul. Sard's theorem is nice and elegant and has many applications, including the statement that a generic smooth map of a curve into the plane is an immersion with finitely many self-intersection points, as is any generic smooth map of an $n$-manifold into a manifold of dimension $2n$. If the target dimension is greater than $2n$, then a generic smooth map is an embedding.<|endoftext|>
-TITLE: Splitting of the double tangent bundle into vertical and horizontal parts, and defining partial derivatives
-QUESTION [9 upvotes]: Let $M$ be a manifold and $g$ a metric on $M$.
-Let $TM$ denote the tangent bundle of $M$, and denote points in $TM$ by $(x,v)$ where $v \in T_xM$.
-The Levi-Civita connection of $(M,g)$ induces a splitting of the double tangent bundle $TTM = V \oplus H$, where $V$ is the vertical distribution, defined by $V_{(x,v)} = T_{(x,v)}T_xM$ (i.e. the tangent space to the fibre), and $H_{(x,v)}$ is the horizontal distribution, which is determined by the connection.
-Suppose $A:TM \rightarrow TM$ is a map such that $A(x,v)\in T_xM$ (so the map $A(x,\cdot)$ is a map from $T_xM$ to itself for all $x \in M$).
-How does one use the splitting described above to define "partial derivatives" $\nabla_xA$ and $\nabla_vA$, which should be maps:
-$(\nabla_xA)(x,v):T_xM \rightarrow T_xM$,
-$(\nabla_vA)(x,v):T_xM \rightarrow T_xM$.
-These should have the property that if $\gamma(t)$ is a curve on $M$ and $u(t)$ is a vector field along $\gamma$ (so $u(t) \in T_{\gamma(t)}M$ for all $t$), and $\nabla_t$ denotes the covariant derivative along $\gamma$, then
-$\nabla_t(A(\gamma,u)) = (\nabla_xA)(\gamma,u) \cdot \dot{\gamma} + (\nabla_vA)(\gamma,u) \cdot \nabla_t u$
-(here on the LHS, $A(\gamma,u)$ is itself a vector field along $\gamma$, so the notation $\nabla_t(A(\gamma,u))$ is meaningful).
-The expression above "makes sense" intuitively, but I can't get the formalism to work properly.
-
-REPLY [4 votes]: Denote $E=TM$. We have a vertical projection $P_{(x,v)}:T_{(x,v)}E\to E_x$ determined by the splitting. The covariant derivative of a vector field $(\gamma,u)$, regarded as a curve in $E$, is the vertical projection of its derivative. We also have linear maps $d\pi:T_{(x,v)}E\to T_xM$ (where $\pi:E\to M$ is the bundle projection), the horizontal lift $h:T_xM\to H_{(x,v)}$ such that $h\circ d\pi=id_H$, and the natural isomorphism $i:T_xM=E_x\to V_{(x,v)}$ (which is the only thing depending on the fact that $E$ is the tangent bundle and not an arbitrary vector bundle). In this notation, we have $\xi=h\circ d\pi(\xi)+i\circ P_{(x,v)}(\xi)$ for every $\xi\in T_{(x,v)}E$.
-Define $\nabla_x A=P_{A(x,v)}\circ (dA)\circ h$ and $\nabla_v A=P_{A(x,v)}\circ (dA)\circ i$ where $dA:T_{(x,v)}E\to T_{A(x,v)}E$ is the ordinary derivative. Then, for a vector $\xi\in T_{(x,v)}E$ (the derivative of our vector field $(\gamma,u)$) we have
-$$
-\begin{aligned}
- \nabla_t (A(\gamma,u)) &= P_{A(x,u)}\circ dA(\xi) = P_{A(x,u)}\circ dA \circ h\circ d\pi(\xi) +P_{A(x,u)}\circ dA \circ i\circ P_{(x,v)}(\xi) \cr
- &=(\nabla_xA)(d\pi(\xi)) + (\nabla_vA)(P_{(x,v)}(\xi)) =(\nabla_xA)\cdot\dot\gamma + (\nabla_vA)\cdot\nabla_t(\gamma,u) .
-\end{aligned}
-$$<|endoftext|>
-TITLE: Tameness for the Galois closure of a map of curves
-QUESTION [7 upvotes]: Say we are working over some $K=\overline{K}$, of characteristic $p>0$. Let $\phi: Y\rightarrow X$ be a nonconstant map of smooth projective curves. To this map we can associate a map $\psi: Z\rightarrow X$, where on the level of fields this is the Galois closure of $k(X)\subseteq k(Y)$. I would like to know about the tameness of this map.
-Let $e_P$ denote the ramification indices (with the maps understood to be either $\psi$ or $\phi$ depending on where $P$ lives). Now obviously if $p|e_P$ and if $Q$ lies above $P$, $p|e_Q$ as well, so $\psi$ has wild ramification at $Q$. I am wondering when we can ensure this map is (everywhere) tamely ramified. For instance if $d=deg(\phi) < p$, then the degree of the Galois closure of $k(Y)$ over $k(X)$ has degree dividing $d!$, and hence $\psi$ remains tame.
-My question is this: Suppose we can show for each $P\in Y$ such that $e_P \geq p$ that each point above $P$ is tamely ramified. Can we conclude that $\psi$ is (everywhere) tamely ramified? It seems to me that this isn't true but I cannot produce a counterexample. It would be fortuitous if it were true, however. Any help is greatly appreciated.
-
-REPLY [3 votes]: For the first glance
-it should follow form the Abhaynkar's lemma (see "Algebraic Function Fields" by Stichtenoth, Theorem 3.9.1) and the fact the Galois closure is the composite of all the different embeddings of L over K into fixed algebraic closure of K (so each of them has the same properties of tame ramifications).
-Then we just apply the lemma and get the result that $p=char(K)$ does not divide $e_P$ for any place P in K.<|endoftext|>
-TITLE: Profinite completion of a semidirect product
-QUESTION [10 upvotes]: If we have two finitely generated residually finite groups $G$ and $H$, is there are relation between
-the profinite completions $\hat{G},\hat{H}$ and the profinite completion of a semidirect
-product $\hat{G \rtimes H}$
-(and analogous question for pro-p completions)
-
-REPLY [2 votes]: Let $\mathscr{P}$ be any property
-such that whenever a group has $\mathscr{P}$ then all its subgroups also have $\mathscr{P}$.
-In [1] Theorem 3.1, K. W. Gruenberg has proved that if the wreath product
-$W= A \wr B$, is residually $\mathscr{P}$,
-then either $B$ is $\mathscr{P}$ or $A$ is abelian.
-Consider $W= S_3 \wr \mathbb{Z}$, where $S_3$ is the symmetric group of
-degree 3.
-Since $S_3$ is not abelian, $\mathbb{Z}$ is not finite, and the subgroup of any finite group is finite,
-the group $W$ is not RF.
-Clearly, $W= \prod_{i \in \mathbb{Z}} S_3 \rtimes \mathbb{Z}$, where $\mathbb{Z}$ and $\prod_{i \in \mathbb{Z}} S_3$
-are residually finite.
-[1] K. W. Gruenberg, Residual properties of infinite soluble groups},
-Prec. London Math. Soc., Ser. 3, 7 (1957), 29--62.<|endoftext|>
-TITLE: Projecting the unit cube onto subspaces
-QUESTION [8 upvotes]: Given a set $S$ of non-zero vectors in $\mathbb{R}^n,$ and a subspace $L,$ consider $f(S,L)=\max_{s \in S}\frac{\|Ps\|_2}{\|s\|_2}$ where $Ps$ is the orthogonal projection of $s$ onto $L.$
-Specifically, consider the set $X$ consisting of the $2^n-1$ (nonzero) vectors with all coordinates $0$ or $1$.
-A recent question concerns criteria which might show, for a given subspace $L$, that $f(X,L)$ is small. Here I am concerned with choosing the subspace:
-
-For each $n$ and $d x^{1/2+\epsilon}\}.
-$$
-Let $q$ be a prime in $(x^{1/2-\epsilon/2},x^{1/2-\epsilon/3}]$. If $1+kp\equiv 0\pmod q$ for some $k\le x^{\epsilon/2}/2$, then $1+kp=qd$ for some $d\le (1+kx)/x^{1/2-\epsilon/2}\le x^{1/2+\epsilon}$. In particular, $p$ is not in $P$. Therefore, if $p$ is counted by $P$, then for each $q\in (x^{1/2-\epsilon/2},x^{1/2-\epsilon/3}]$, it must avoid the congruence classes $\{-\overline{k}\pmod q: 1\le k\le x^{\epsilon/2}/2\}$. Since $p$ is prime is prime, it must also avoid the classes $0\pmod q$ for all primes $q\le \sqrt{x}$. We then apply the arithmetic form of the Large Sieve (see, e.g., page 159 in Davenport) to conclude that
-$$
-\#P \ll \frac{\pi(x)}{x^{\epsilon/2}} ,
-$$
-which proves the claim.
-The above result is essentially sharp: the number of $p\in(x/2,x]$ for which there are $m,n\le p^{1/2}(\log p)^c$ with $mn\equiv 1\pmod p$ is $o(\pi(x))$ for small enough $c$. Indeed, we would then have that there is some $k\le (\log x)^{2c}$ such that $1+kp$ can be written as $1+kp=mn$ with $m\le n\le x^{1/2}(\log x)^c$. In particular, $1+kp$ would have a divisor $m\in[0.1k\sqrt{x}/(\log x)^c, \sqrt{1+kx}]$. The number of such primes $p\in(x/2,x]$ is
-$$
-\asymp f(k) \frac{x}{\log x} \frac{1+\left(\log\frac{(\log x)^{2c}}{k}\right)^\delta}{(\log x)^\delta(\log\log x)^{3/2}},
-$$
-where $\delta=1-(1+\log\log2)/\log 2$ is the constant appearing in Erdos's multiplication table problem and $f(k)$ is some tame multiplicative function that is usually $\asymp1$ (see http://arxiv.org/abs/math/0401223 and http://arxiv.org/abs/0905.0163. This result is not stated there but it should follow from the methods. The upper bound uses the sieve. For the lower bound, instead of the Bombieri-Friedlander-Iwaniec result on primes in APs to large moduli, we would have to use Zhang's result because we need information about the distribution of primes in progressions $a\pmod q$ with $1+ak\equiv0\pmod q$, so $a$ is not fixed.) Summing over $k\le(\log x)^{2c}$, we find that the number of $p\in(x/2,x]$ for which there are $m,n\le p^{1/2}(\log p)^c$ with $mn\equiv 1\pmod p$ is
-$$
-\ll \frac{x}{\log x} \frac{(\log x)^{2c}}{(\log x)^\delta(\log\log x)^{3/2}} .
-$$
-Taking $c=\delta/2$ yields the claimed result.
-The above line of thought should be able to produce, at least heuristically, the optimal value of $c$ for which the following two propositions hold:
-$$
-\#\{p\le x: \exists m,n\le p^{1/2}(\log p)^{c+\epsilon}\ \text{with}\ mn\equiv 1\pmod p\} \sim \pi(x)
-$$
-and
-$$
-\#\{p\le x: \exists m,n\le p^{1/2}(\log p)^{c-\epsilon}\ \text{with}\ mn\equiv 1\pmod p\} =o(\pi(x)).
-$$
-The point is that if $k\neq k'$, then the multiplicative structures of $1+kp$ and of $1+k'p$ should be independent from each other. Therefore the events that $1+kp=mn$ for some $m,n$ in some range and that $1+k'p=m'n'$ for some $m',n'$ in some range should be independent. So a Borel-Cantelli argument would then yield the optimal value of $c$.<|endoftext|>
-TITLE: Estimates on the Green function of an elliptic second order differential operator.
-QUESTION [8 upvotes]: Let $D$ be a linear differential elliptic operator of second order
-with infinitely smooth coefficients acting on real valued functions
-on a compact manifold $M$. Let us assume that $D$ has no free term, i.e. $D(1)=0$.
-Let us fix a smooth positive measure
-(density) $\mu$ on $M$. Does there exist a (integrable) Green
-function $G\colon M\times M\to \mathbb{R} $ with the following properties:
-(1) $\int_M G(x,y) \cdot D\phi(y) d\mu(y) =\int_M\phi(y) d\mu(y)
--\phi(x)$ for any function $\phi$ and $x\in M$ (this is the definition of Green function);
-(2) $G$ is infinitely smooth outside of the diagonal;
-(3) $G$ is bounded from below.
-The last property can be asked in a stronger form:
-(3') Does $G$ satisfy the asymptotic estimate near the diagonal:
-$$c|x-y|^{2-n}\leq G(x,y)\leq C|x-y|^{2-n}$$
-where $c,C>0$ and $n=\dim M>2$. If $n=2$ there should be a
-logarithmic estimate.
-I am pretty sure that this is true and should be well known. I would
-need a reference. The special case when $D$ is the Laplacian for a
-Riemannian metric on $M$ is contained explicitly in some textbooks I
-am familiar with.
-
-REPLY [2 votes]: Green's functions are constructed in Aubin's book for operators such as you mentioned, but with some sign condition on the lowest order term. I have not had a close look but my suspicion is that (3') is fine but for (3) you need a maximum principle.<|endoftext|>
-TITLE: When does Zariski closure commute with base change?
-QUESTION [5 upvotes]: This should be an elementary question for anyone who knows SGA by heart (alas, not for me). It smells a lot like a descent problem. All schemes are supposed to be noetherian, and all morphisms to be locally of finite presentation.
-Let $X$ be a scheme of finite type over a (base) scheme $S$, and let $R \subseteq X(S)$ be a subset of the set of $S$--rational points of $X$. Denote by $\overline R$ the smallest closed subscheme of $X$ whose $S$--rational points contain $R$.
-Let $f:S'\to S$ be a (base--change) morphism of schemes, write $X' := X\times_SS'$ and denote by $R'$ the image of $R$ in $X(S') = X'(S')$. Again, let $\overline{R'}$ be the smallest closed subscheme of $X'$ whose $S'$--rational points contain $R'$. Then, $\overline{R'}$ is contained in $\overline R\times_SS'$, and the question is:
-
-Suppose $f:S'\to S$ is flat. Does the equality $\overline{R'} = \overline R \times_SS'$ hold?
-
-Clearly some hypothesis on $f$ is needed, and I just guess it's flatness.
-
-REPLY [6 votes]: It seems that the condition you need is that the generic points of $S'$ go to generic points of $S$; this is much weaker than flatness.
-Assuming this condition, we can reduce to the case that $S$ and $S'$ are both $Spec$ of some field and we can also assume that $\overline{R} = X$. If $X$ is a variety over a field $k$ any Zariski dense subset of $X(k)$ is also dense as a subset of $X(K)$ where $K$ is any extension field of $X$, proving the claim.<|endoftext|>
-TITLE: Almost-direct product and 1-formality
-QUESTION [8 upvotes]: Let $G$ be a finitely presented group. To $G$ is associated in a functorial way a Malcev Lie algebra which can be constructed in several equivalent ways. Roughly speaking, it is the quotient of the completed free Lie algebra on the same number of generators as $G$, by the (formal) logarithm of the defining relations of $G$ (see edit below).
-$G$ is called 1-formal if its Malcev Lie algebra is isomorphic as a filtered Lie algebra to the degree completion of a finitely presented Lie algebra with quadratic relations. (If $X$ is a path-connected topological space such that $G=\pi_1(X)$, this is equivalent to say that the Malcev Lie algebra of $G$ is isomorphic to the holonomy Lie algebra of $X$)
-It is quite well known that fundamental groups of complementary of hyperplane arrangements in $\mathbb{C}^n$ (e.g. pure braid groups) are 1-formal.
-On the one hand, an important fact in the study of these groups is that they are (under some conditions on the underlying arrangement) iterated "almost-direct product" of free groups, and free groups are themselves 1-formal. An almost direct product is a semi-direct product $H\rtimes K$ for which the action of $K$ on the abelianization of $H$ is trivial. On the other hand, according to this interesting survey http://www.arxiv.com/abs/0903.2307 the direct product of two 1-formal groups is again 1-formal, even if no proof is given (it's probably easy..)
-So it is temptating to ask:
-
-is an almost-direct product of
- 1-formal groups again 1-formal ?
-
-Edit: Some details about the construction and its relation with almost direct product. If $G$ is an abelian group, then one can take the tensor product $G \otimes_{\mathbb{Z}} \mathbb{Q}$ leading to a uniquely divisible abelian group. The Malcev construction extends this to any nilpotent group, leading to a uniquely divisible nilpotent group. Let $G^{(0)}=G$, $G^{(n+1)}=[G^{(n)},G]$ be the lower central serie, then the quotients $G/G^{(n)}$ are nilpotent by construction.
-The Malcev completion of $G$ is the inverse limit of the $(G/G^{(n)}) \otimes_{\mathbb{Z}} \mathbb{Q}$. It is a pro-unipotent group, hence it has a (pro-nilpotent) Lie algebra $\mathfrak g$. On the other hand, one can define a Lie algebra by
-$$gr\ G=\mathbb{Q}\otimes_{\mathbb{Z}} \bigoplus G^{(n)}/G^{(n+1)}$$
-the bracket being induced by the commutator in $G$. It as a graded Lie algebra, and in fact it is the associated graded of the filtered Lie algebra $\mathfrak g$.
-A group is called 1-formal if there is an isomorphism of filtered Lie algebras $gr\ G \cong \mathfrak g$. Now, the point is that almost-direct product behave well with respect to the lower central series. If $G=G_1\rtimes G_2$ is an almost-direct product, then it seems to me that a result of Falk and Randell implies that we have
-$$G^{(n)}=G_1^{(n)}\rtimes G_2^{(n)}$$
-and
-$$gr\ G = gr\ G_1 \rtimes gr\ G_2$$.
-
-REPLY [16 votes]: Yes, the direct product of two 1-formal groups is again 1-formal, and so is the free product of two 1-formal groups. A proof is given in arxiv:0902.1250, Proposition 9.2.
-And no, the almost direct product of two 1-formal groups need not be 1-formal. A proof is given in the same paper, Example 8.2. The group in question is a semi-direct product of the form $G=F_4\rtimes F_1$, where $F_n$ is the free group of rank $n$, which is of course 1-formal. The action of $F_1$ on $F_4$ is given by a certain pure braid $\beta \in P_4$, acting via the Artin representation on $F_4$; thus, the action is trivial on $H_1(F_4)$. For this extension, the ``tangent cone formula" fails: the tangent cone to the characteristic variety $V_2(G)$ is strictly included in the resonance variety $R_2(G)$. In view of Theorem A from the cited paper, the group $G$ is not 1-formal.
-It is worth noting that $G$ is the fundamental group of the complement of a certain link of 5 great circles in $S^3$. Alternatively, $G$ can be realized as the fundamental group of the complement of an arrangement of 5 planes in $\mathbb{R}^4$, meeting transversely at the origin (of course, this real arrangement cannot be isotoped to an arrangement of 5 complex lines in $\mathbb{C}^2$). For more details on the construction and properties of such arrangements, see arxiv:math.GT/9712251. In particular, the pure braid $\beta$ is described there in Propositions 4.4, 4.6, and 4.9.<|endoftext|>
-TITLE: Uniformly Convex spaces
-QUESTION [8 upvotes]: My first question here would fall into the 'ask Johnson' category if there was one (no pressure Bill). I'm interested in constructing a uniformly convex Banach space with conditional structure without using interpolation. The constructions of Ferenczi and Maurey-Rosenthal both use interpolation.
-Using existing methods for constructing spaces with conditional structure I think it is possible to construct a hereditarily indecomposable space whose natural basis statisfies a lower $\ell_2$ estimate on any $n$ disjointly supported blocks vectors supported after the $n^{th}$ position on the basis and an upper $\ell_2$ estimate on all finite block sequences. The space $X$ is sure to be reflexive and probably doesn't contain $\ell_\infty$ finitely represented.
-I would like to have some way of showing that $X$ is uniformly convex and this is where I'm stuck. Perhaps one could show that $\ell_1$ is not finitely represented in $X$ but as far as I can see this is not good enough (or is it?).
-My question: If a space is reflexive and does not contain $\ell_1$ finitely represented is it necessarily uniformly convex?
-I suspect the answer is no but I don't have a counterexample.
-Another question: Are there any known conditions on a basis, which (1) do not imply the basis is unconditional and (2) do imply the space is uniformly convex?
-
-REPLY [2 votes]: I think James also showed that if $X$ does not contain almost isometric copies of $\ell_1^2$ (he called such a space uniformly non-square) then $X$ is superreflexive. This is no longer true for $n>2$, as James later constructed a non-reflexive, uniformly non-octahedral (no almost isometric copies of $\ell_1^3$) space, thus also having non-trivial type.
-Maybe you can check whether your space is uniformly non-square. Connecting it with your last question I think that you would have to verify that $\exists \delta>0$ such that for any normalized block vectors $x$ and $y$ (but not necessarily disjointly supported) there exist a choice of signs such that $||x\pm y||<2-\delta.$ I don't think this condition implies unconditionality.
-Hopefully this makes sense...<|endoftext|>
-TITLE: Good books on Dirichlet's class number formula
-QUESTION [11 upvotes]: I refrained from asking the technical questions; maybe everyone didn't like my attitude. At least, help me finding good books.
-Can anyone suggest a good book that gives a complete reference to "Dirichlet's class number formula" and Class number theory, and explaining each nook and corner of it? Or any reference material which is free?
-
-REPLY [3 votes]: Well, I don't know any reference that examines ''each nook and corner'', but as Kevin says in a comment above, Washington's ''Cyclotomic fields'' is one good place to start (assuming you have enough grounding in algebraic number theory).
-In addition, Lang's ''Algebraic number theory'' contains some things on class number formulas I think; also we have his two-volume (now as one-volume at Springer) book(s) on Cyclotomic fields. But beware, I would say that these/this require a firmer background, than Washington's book.
-(Without encouraging illegal activities, I'm sure there are some bootleg versions on the net of the above books.)
-Otherwise, there are dozens of nice books on the subject. Also, there must be tonnes of free lecture notes out there in cyberspace.<|endoftext|>
-TITLE: Undecidable theories easier than $Q$
-QUESTION [15 upvotes]: Most proofs of undecidability for various theories (pure logic with binary relation, group theory, etc.) show that the natural numbers and Robinson's $Q$, in one form or another, can be encoded appropriately. Hence the decision problem for these theories is as hard as $K$, the halting set.
-Are there are recursively axiomatized theories which are undecidable, but yet easier than $K$ (i.e. $K$ is not Turing reducible to deciding to the theory)?
-
-REPLY [18 votes]: When I was looking around trying to find some inspiration to answer your question, I found the following result of Feferman from 1957:
-
-For any set $X$ of natural numbers there is a theory $T(X)$ such that:
-
-The set $X$ and the set of Gödel numbers of consequences of $T(X)$ have the same degree of unsolvability.
-If $X$ is r.e. then $T(X)$ is effectively axiomatizable.
-
-
-Because there are nonzero r.e. Turing degrees strictly weaker than $K$, I think this may answer the question.
-The result is in the paper "Degrees of Unsolvability Associated with Classes of Formalized Theories", Solomon Feferman, The Journal of Symbolic Logic, Vol. 22, No. 2 (Jun., 1957), pp. 161-175. http://www.jstor.org/stable/2964178<|endoftext|>
-TITLE: Blow-up removes intersections?
-QUESTION [8 upvotes]: Assume that $\beta:\tilde{X}\to X$ is the blow-up of a nonsinular $\Bbbk$-variety $X$ along a sheaf of ideals $\mathcal{I}$. Let $Y:=Z(\mathcal{I})$. Given nonsingular, closed subvarieties $Z_1,\ldots,Z_r\subseteq X$ such that $\bigcap_i Z_i \subseteq Y$, is it true that $\bigcap_i \tilde{Z}_i=\emptyset$, where $\tilde{Z}_i$ denotes the strict transform of $Z_i$? If not, does this hold if we require $Y$ to be nonsingular and/or the $Z_i$ to intersect transversally?
-
-REPLY [2 votes]: I have tried to generalize the Exercise referenced by Karl, even though he told me that it shouldn't be possible this way. I think, however, it works:
-Edit: I made a mistake concerning $J_i$ - it cannot be equal to $I_i\oplus\bigoplus_{d\ge 1} I_i^dT^d$ because that is not necessarily an ideal - it might not be closed under multiplication by elements from the ring $S$. The version below looks better.
-
-Proposition. Let $Z_0,\ldots,Z_r$ be closed subschemes of a Noetherian scheme $X$ such that $Z_i\not\subset Z_j$ for $i\ne j$. Let $I_i:=I(Z_i)$ and denote by $\tilde{Z}_i$ the respective strict transform of $Z_i$ under the blow-up $\beta:\tilde{X}\to X$ of $X$ along $I:=\sum_{i=0}^rI_i$. Then, $\bigcap_{i=0}^r\tilde{Z}_i=\emptyset$.
-Proof. The statement can be checked locally, so we may assume that $X=\mathrm{Spec}(A)$ is affine. Let $f_i:Z_i\hookrightarrow X$ be the respective closed immersion, so $Z_i=\mathrm{Spec}(A/I_i)$ and $f_i^\sharp:A\twoheadrightarrow A/I_i$. Then, the inverse image ideal sheaf of $I$ under $f_i$ is $I\cdot A/I_i$ and hence,
-$\displaystyle\tilde{Z}_i=\mathrm{Proj}\left(\bigoplus_{d\ge 0} \left(I\cdot A/I_i\right)^d\cdot T^d\right)$
-With $S=\bigoplus_{d\ge 0} I^d\cdot T^d$, the homogeneous ideal defining $\tilde{Z}_i$ inside $\tilde{X}=\mathrm{Proj}(S)$ is equal to
-$\displaystyle J_i = \bigoplus_{d\ge 0} (I^d\cap I_i)$
-In particular, $J_0+\cdots+J_r\supseteq S_+$, so any point $P\in\tilde{Z}_0\cap\cdots\cap\tilde{Z}_r$ would correspond to a homogeneous prime ideal containing each of the $J_i$ and hence, the irrelevant ideal. There is no such point.
-
-Did I miss something? Or is this correct?<|endoftext|>
-TITLE: Various flavours of infinitesimals
-QUESTION [37 upvotes]: I'm not sure if this is a soft question, and whether it may be too broad or, on the contrary, too localized. Well, in Mathematics the concept of "infinitesimal" has been of extreme importance for centuries.
-The present mathematical tecnology allows one, according to the context, to formalize this notion in several ways:
-
-Differential geometry. The classical epsilon-delta formalism of limits in elementary analysis leads to the concept of first-order (or $n$-th order) approximation in Calculus, hence to many standard notions in differential geometry: the differential of a map between smooth manifolds, jets of a map, tangent vectors, differential forms, and Riemannian metrics.
-Algebraic geometry. The lack of a useful notion of convergence of sequences due to the coarseness of Zariski topology prevents us from using epsilon-delta arguments to define "infinitesimals". But then one retains the notion of first-order (or $n$-th order) "approximation" in a more formal way, e.g. by means of universal properties of modules and derivations (Kahler differentials...), and that of "infinitesimal space" e.g. by means of local Artinian $\Bbbk$-algebras, and of "infinitesimal neighbourhood" e.g. by completion of local rings, formal schemes etc. (But after all the algebro geometric perspective is just a high brow way of doing the formal derivative of polynomials, which coincides with the "topological one" once we work over $\mathbb{R}$ or $\mathbb{C}$)
-Synthetic differential geometry. More recently some mathematicians are exploring the realm of synthetic differential geometry, of which I know nothing except that it kind of unifies the perspectives of the previous two approaches and uses relevant amounts of category theory (please correct me if I'm not being correct).
-Non-standard analysis. The concept of infinitesimal element is of course fundamental in non-standard analysis, where the field $\mathbb{R}$ and the use of epsilon-delta arguments is replaced by the introduction of the field ${}^* \mathbb{R}$ of hyperreal numbers, which hosts several hierarchies of infinitesimals (as well as "infinite elements").
-Noncommutative geometry. According to A.Connes (in his book Noncommutative Geometry there's a paragraph on a "Quantized calculus") given an infinite dimensional separable Hilbert space $\mathcal{H}$ and a certain operator $F$ on it, compact operators on $\mathcal{H}$ with characteristic values such that $\mu_n=O(n^{-\alpha})$, $n\to\infty$, can be interpreted as "infinitesimals of order $\alpha$", while the differential of a "complex variable" (read "operator on $\mathcal{H}$") $f$ is just defined to be the commutator $[F,f]$.
-
-At the risk of seeing my question closed as too vague ("not a real question"), it would be my curiosity to know:
-
-Is there a theory that encompasses all the above instances of "infinitesimals" within a unique formal picture? Or, on the contrary, are some of the above notions of infinitesimals inherently specific to their field and embody formalizations of different heuristic notions? [A situation as in the second question occurs with the notion of "infinity": it seems to me there's almost no deep relation between the infinity as in $\lim_{n\to\infty}$ and the infinite cardinals of Cantor]
-
-REPLY [11 votes]: I don't know of the Connes calculus, but the others (including nonstandard analysis à la Robinson) have been brought under a common framework using models of synthetic differential geometry. However: it is important to point out that the infinitesimals used in algebraic geometry (for jet bundles, etc.) are nilpotent infinitesimals, whereas the infinitesimals used in nonstandard analysis are invertible. So in a sense the answer to the question is, "yes and no", but I'm going to concentrate here on the "yes".
-This is explored in Models for Smooth Infinitesimal Analysis by Moerdijk and Reyes, which I recommend. This book can be read as "applied sheaf theory" or "applied Grothendieck topos theory", where the art is to choose a site (small category + covering sieves) judiciously to achieve several aims at once. In many of the models, one takes the underlying category of the site to be something like affine spectra of commutative rings, except one is not dealing with commutative rings exactly, but with richer algebraic structures called $C^\infty$-rings. The formal definition of these is in terms of a Lawvere algebraic theory which allows one to apply not just polynomial operations but more general operations based on $C^\infty$ functions. So the underlying category of the site in these models is the opposite of finitely generated $C^\infty$-rings, which Moerdijk and Reyes call $\mathbb{L}$ (for "locus").
-The representing object which gives the locus of invertible infinitesimals is the spectrum of the $C^\infty$-ring given by $C^\infty$ functions $\mathbb{R} - \{0\} \to \mathbb{R}$, modulo the ideal of functions that vanish on some neighborhood of $0$, aka the $C^\infty$ ring of germs at $0$. This is really not much different from nonstandard infinitesimals: usually infinitesimal elements are thought of in some way as germs of functions at infinity, i.e., of functions $\mathbb{R} \to \mathbb{R}$ modulo those which vanish for sufficiently large $x$ (compare here the infinitesimals of Du Bois-Reymond and Hardy). Here Moerdijk and Reyes use $0$ instead of $\infty$. Either way, there are nonarchimedean elements, i.e., nonzero elements less than any $1/n$ in absolute value.
-[In nonstandard analysis, one typically refines this idea by considering germs of functions $\mathbb{R} \to \mathbb{R}$ at an "ideal point at infinity", i.e., at a non-principal ultrafilter $U$ on $\mathbb{R}$, or alternatively germs of functions $\mathbb{N} \to \mathbb{R}$ at a non-principal ultrafilter $U$ on $\mathbb{N}$. The more familiar buzzword here is "ultrapower", but see this MO answer by François Dorais, where the implicit message is that an ultrapower along $U$ is really the same as taking a stalk at $U$. (I call $U$ an "ideal point at infinity" because we can think of a non-principal ultrafilter $U$ on $\mathbb{N}$ as a point in the fiber over $\infty$ with respect to the canonical continuous map $\beta(\mathbb{N}) \to \mathbb{N} \cup \{\infty\}$, from the Stone-Cech compactification of $\mathbb{N}$ to the one-point compactification of $\mathbb{N}$.)]
-On the other hand, a typical representing object for nilpotent infinitesimals is the spectrum of the $C^\infty$-ring of functions $\mathbb{R} \to \mathbb{R}$ modulo the ideal of squares of elements which vanish at $0$. The "internal hom" represented by this spectrum gives the tangent bundle functor, and other jet bundles can be similarly represented, by using $C^\infty$-rings with different types of nilpotent elements.
-The tricky part of all this is to get the right notion of covering sieves, i.e., of sheaves w.r.t. a Grothendieck topology, to achieve disparate aims. One aim would be to embed the usual category of manifolds fully and faithfully in the category of sheaves, so as to preserve "good colimits", such as a manifold $M$ obtained as a colimit along an open covering of $M$. A different aim would be to arrange the topology so that the locus of invertible infinitesimals, as a presheaf on the site category $\mathbb{L}$, is a sheaf w.r.t. the topology. In summary, both aims can be achieved simultaneously so as to accommodate both nilpotent and invertible infinitesimals.<|endoftext|>
-TITLE: What invariants of a matrix or representation can be used to find its GL(n,Z)-conjugacy class?
-QUESTION [19 upvotes]: First question: For a semisimple invertible $n \times n$ matrix with entries over a field K, its characteristic polynomial completely describes the similarity class of the matrix. For non-semisimple elements, the characteristic polynomial is no longer a complete description of the similarity class, but there exists the rational canonical form, which is a complete invariant.
-I'm interested in whether we can find invariants that help determine the similarity class of elements of $GL(n,\mathbb{Z})$ (where $\mathbb{Z}$ is the ring of integers) under the $GL(n,\mathbb{Z})$-conjugation action. Clearly, the invariants for similarity class over $\mathbb{Q}$, but there are semisimple matrices in the same similarity class over $\mathbb{Q}$ but not conjugate over $\mathbb{Z}$:
-1 0
-0 -1
-and
-0 1
-1 0
-These are clearly conjugate in $GL(2,\mathbb{Q})$ but not in $GL(2,\mathbb{Z})$. To see why they aren't conjugate in the latter, note that on reducing mod 2, the first matrix becomes the identity matrix and the second matrix becomes a non-identity matrix, so they cannot be conjugate mod 2, and hence cannot be conjugate in $GL(2,\mathbb{Z})$.
-Second question: For a finite group G, call representations $\alpha, \beta: G \to GL(n,R)$ "locally conjugate" if $\alpha(g)$ is conjugate to $\beta(g)$ for every $g \in G$ via some element of $GL(n,R)$ depending on g.
-We say that $\alpha$, $\beta$ are equivalent as representations if we can choose a single element of $GL(n,R)$ that conjugates $\alpha(g)$ to $\beta(g)$ for every $g \in G$.
-My question is: does locally conjugate imply equivalent when $R = \mathbb{Z}$?
-NOTE 1: When R is a field of characteristic not dividing the order of G, then locally conjugate implies equivalent, and we can prove this by noting that $\alpha$, $\beta$ have the same character (SORRY, the "same character" is enough to complete the proof only in characteristic zero -- in prime characteristics, even those that don't divide the order of the group, having the same character isn't good enough to conclude the representations are equivalent. However, the "locally equivalent implies equivalent" seems to still hold when the characteristic does not divide the order of G using more indirect arguments). However, for $\mathbb{Z}$, the character no longer determines the representation, so the proof used for fields (taking the character) does not work for $\mathbb{Z}$.
-NOTE 2: When the characteristic of R divides the order of G, there exist examples of locally conjugate representations that are not equivalent -- in fact, we can construct such examples for the Klein four-group with the field of four elements. I haven't had success with using these to generate counter-examples over $\mathbb{Z}$, though there may be a way.
-
-REPLY [11 votes]: Here is the requested example of two representations of the Klein 4 group over $\mathbb{Z}$, locally conjugate but not conjugate.
-Let $K:= \mathbb{Z}/2 \times \mathbb{Z}/2$ act on $\mathbb{Z}^4$ by permuting the coordinates. Inside $\mathbb{Z}^4$, consider the following two lattices:
-$$L_1 := \{ (a,b,c,d) \in \mathbb{Z}^4 : a \equiv b \equiv c \equiv d \mod 2 \}$$
-$$L_2 := \{ (a,b,c,d) \in \mathbb{Z}^4 : a + b + c + d \equiv 0 \mod 2 \}$$
-Verification that $L_1$ and $L_2$ are locally isomorphic:
-Let $\sigma$ be the element of $K$ which switches the first two and the last two coordinates. Consider the following bases for $L_1$ and $L_2$:
-$$(1,1,1,1),\ (1,-1,1,-1),\ (2,0,0,0), (0,2,0,0)$$
-and
-$$(1,1,0,0),\ (1,-1,0,0),\ (1,0,1,0),\ (0,1,0,1)$$
-In both cases, $\sigma$ fixes the first basis element, negates the second and switches the last two. So $L_1$ and $L_2$ are isomorphic modules for $\mathbb{Z}[\sigma]/\langle \sigma^2-1 \rangle$.
-Verification that $L_1$ and $L_2$ are not isomorphic:
-For each character $\chi$ of $K$, let $L_i^{\chi}$ be the sublattice of $L_i$ on which $K$ acts by $\chi$. Let $M_i = \bigoplus_{\chi} L_i^{\chi}$. Then $L_1/M_1$ has order $2$, and $L_2/M_2$ has order $8$.<|endoftext|>
-TITLE: Signs and functoriality of tensor products
-QUESTION [9 upvotes]: Let $C,C',D,D'$ be chain complexes of $R$-modules (let's say with upper indexing, so perhaps I should call them cochain complexes, though they're not duals of anything). Let $f\in Hom^\ast(C,C')$ and $g\in Hom^*(D,D')$. Then the standard convention is that $$(f\otimes g)(x\otimes y)=(-1)^{|g||x|}f(x)\otimes g(y),$$ where $|g|$ is the degree of $g$ and $|x|$ is the degree of $x$. As observed on page 171 of Dold, this is consistent with having a degree 0 chain map $$Hom^\ast(C,C')\otimes Hom^\ast(D,D')\to Hom^\ast(C\otimes C',D\otimes D').$$
-What bothers me, though, is that this formula forces $$(h\otimes k)\circ(f\otimes g)=(-1)^{|k||f|}hf\otimes kg,$$ which seems to violate the definition of a bifunctor as given, for example, on page 17 of Kashiwara and Schapira's "Categories and Sheaves", which would seem to require (adapting the notation) $$(1_{C'}\otimes g)(f\otimes 1_D)=(f\otimes 1_{D'})(1_C\otimes g).$$ (Here I suppose we assume that the relevant categories are the category of chain complexes of $R$ modules with $Mor(X,Y)=Hom(X,Y)$ (certainly such things can be composed functorially and the identity behaves properly) and the products of this category with itself). If I'm reading it correctly, this requirement in Kashiwara-Schapira seems to be the same as what Mac Lane is asking for in Proposition II.3.1 of "Categories for the Working Mathematician".
-So are we to believe $\otimes$ is not a functor or is there a way to reformulate all of this to be consistent (or am I just getting something wrong)?
-Thanks in advance!
-
-REPLY [14 votes]: There are two options. If you just want an ordinary category of cochain complexes, then you have to take the morphisms to be cochain maps of degree zero. In that context we have $(-1)^{|k||f|}=1$ so there is no problem.
-Alternatively, you can have an enriched category of cochain complexes. In more detail, for any symmetric monoidal category $(\mathcal{V},\otimes)$ there is a theory of $\mathcal{V}$-enriched categories. Such a thing has a class of objects, and for each pair of objects $X$ and $Y$, it has an object $\text{Hom}(X,Y)\in\mathcal{V}$. Given a third object $Z$ there is also a composition morphism $c:\text{Hom}(Y,Z)\otimes\text{Hom}(X,Y)\to\text{Hom}(X,Z)$, subject to some obvious axioms. The symmetric monoidal structure on $\mathcal{V}$ includes natural twist isomorphisms $\tau_{PQ}:P\otimes Q\to Q\otimes P$ for all $P,Q\in\mathcal{V}$. When formulating the definition of a bifunctor in an enriched context, you find that you need to use the morphisms $\tau_{PQ}$ in various places.
-In the case of interest, we can regard cochain complexes as a category enriched over graded abelian groups. The sign $(-1)^{|k||f|}$ is provided automatically by the relevant twist maps, so the tensor product becomes a bifunctor in the enriched sense.<|endoftext|>
-TITLE: Carleson's Theorem (on the Adeles and other exotic groups)
-QUESTION [7 upvotes]: I have redone this question:
-On $\mathbb R^n$ the Carleson Operator if defined by
- $$Cf(x) = \sup_{R>0} \left \vert \int_{B_R(0)} e^{2\pi i x\cdot \xi} \widehat{f}(\xi) d \xi \right \vert. $$ (In the previous version I had an incorrect version of this written down which lead to some stupid conclusions).
-
-For $n=1$ Carleson proved that this operator is strong (p,p) which is equivalent $L^p(\mathbb R)$ convergence of $S_Rf(x)=\int_{B_R(0)} e^{2\pi i x\cdot \xi}\widehat{f}(\xi)d\xi\to f(x)$ as $R\to \infty$. The modern proof uses boundedness of the Hilbert-Transform and the ``translation trick''
-$$\chi_{[-r,r]}(\xi)\widehat{f}(\xi) = \widehat{[\frac{i}{2}(m_{-r}H m_r - m_r H m_{-r})f] }(\xi).$$
-For $n>1$ Fefferman proved (using a Becosivitch set) that the transform $T$ defined by $$ \widehat{Tf}(\xi) = \chi_{B_1(0)}(\xi) \widehat{f}(\xi)$$ is unbounded in $L^p$ for $p\neq 2$. (There is no Hilbert Transform in higher dimensions, these become the Reisz Transform and the ``translation tricks'' to express $S_R$ in terms of the Hilbert Transform don't work).
-The Bochner-Reisz conjecture is about boundedness of smoothened characteristic functions and an MO question about it can be found here Recent progress on Bochner-Riesz conjecture
-
-I was originally thinking that $Cf(x)$ could be defined easily for locally compact abelian groups and compact groups and that perhaps a ``translation trick'' existed in some generality. This appears to not be the case for two reasons
-
-The second bullet shows that it is not true in $\mathbb{R}^n$ for $n>1$.
-The third bullet shows you that multiplier operator in $\mathbb{R}^n$ are delicate which makes the definition of a Carleson operator for general abelian groups difficult.
-
-Is the operator $$Cf(x) = \sup_{n\geq 0} \left \vert \int_{\frac{1}{p^n}\mathbb{Z}_p} e^{2\pi i \lbrace x \xi\rbrace } \widehat{f}(\xi) d \mu(\xi) \right \vert. $$
-bounded as an operator $L^r(\mathbb Q_p) \to L^r(\mathbb Q_p)$. (I think one can define a similar operator on the Adeles no?). How does smoothness of multipliers effect the problem in more general groups?
-I'll make it a community wiki so people can fix it if it has errors.
-
-REPLY [4 votes]: There is a p-adic analogue of Carleson's theorem. This was worked out by Hunt and Taibleson and you can find an exposition in Taibleson's book "Fourier analysis on local fields". (NB Taibleson did a lot of work extending Euclidean harmonic analysis results such as Carleson's theorem and boundedness of singular integrals to local fields.) I don't know if an adelic Carleson's theorem has been worked out.
-The notion of smoothness in the p-adics is different than in real spaces since the p-adics are totally disconnected. Instead of say differentiability (which can be defined at least formally), one looks at locally constant functions. For more general groups, you need to have an appropriate notion(s) of smoothness to ask how changing the smoothness affects your problem.<|endoftext|>
-TITLE: Field with one element example?
-QUESTION [18 upvotes]: $$\frac{1}{\mu(B)}\int_B \vert x \vert d\mu(x) = \frac{1}{p+1}$$
-This formula holds for the unit ball in $\mathbb{Q_p}$. This formula also holds for
-$\mathbb{R}$ when $p=1$. Should one expect $$\mathrm{Frac}(W_{1^{\infty}} (\mathbb{F_1}))=\mathbb{R}?$$
-What (mathematical) criteria do people use to rule-out field with one element phenomena? What makes point counting formulas better (or worse)?
-
-REPLY [4 votes]: It is true that if $f(T)\in \mathbb Z[T]$ then,
- $$ \frac{1}{\mu_p(B)}\int_{\mathbb Z_p} f(\vert x \vert_p) d\mu_p(x) \to \frac{1}{\vert B \vert}\int_{B}f(\vert x \vert)dx \mbox{ as } p\to 1.$$
-It is not in general true that
- $$ \frac{1}{\mu_p(B)}\int_{\mathbb Z_p} \vert f( x )\vert_p d\mu_p(x) \to \frac{1}{\vert B \vert}\int_{B}\vert f( x )\vert dx \mbox{ as } p\to 1.$$
-When $f(x) = x^2-1$ we have
- $$\int_{\mathbb Z_p} \vert x^2-1 \vert_p d\mu(x) = \frac{1+p(p-2)}{p}+ \frac{1}{(p+1)p} \to 1/2 \mbox{ as } p \to 1 $$
-and
- $$\frac{1}{2}\int_{-1}^1 \vert x^2-1\vert dx = 2/3.$$
-http://imgur.com/a/m6KYA
-A couple remarks:
-
-You should have also written $\mathbb Q_1 = \mathbb{R}$ instead of that terrible notation $\mathrm{Frac}(W_{1^{\infty}}(\mathbb{F}_1))$.
-The question as posed is a little stupid since if someone had a procedure for "ruling our phenomena" not only would they would probably have category in mind, but they would be able to compute with it. I think the correct answer is $\mathbb F_1$ numerology is justified when it can be categorified''. This is kind of a weak version of the statement "a conjecture is true when you can prove it". I guess a vague question deserves a vague answer.
-
-The first part of the question seems to be about Arakelov geometry and replacing the place at infinity with the place 1. It seems that the answer is no. Another bad question would be to ask for more examples where it does make sense.
-The last part of the original question can be made more precise:
--Are there examples of formulas for $|X(\mathbb F_q)|$ such that $X$ is definable in either monoidal algebraic geometries or Borger's $\Lambda$-ring categorifications of schemes over $\mathbb F_1$ such that the point counting formula's in the category do not agree with $|X(\mathbb F_q)|$ as $q\to 1$? (if you know of another categorification that justifies $GL_n(\mathbb{F}_1)=S_n$ this question applies there too (Maybe Durov's Category?).)<|endoftext|>
-TITLE: Axiom of choice: ultrafilter vs. Vitali set
-QUESTION [26 upvotes]: It is well known that from a free (non-principal) ultrafilter on $\omega$ one can define a non-measurable set of reals. The older example of a non-measurable set is the Vitali set,
-a set of representatives for the equivalence classes of the relation on the reals "the same modulo a rational number". Is it known whether you can have one without the other?
-I.e., is ZF consistent with the existence of a set of representatives for the Vitali equivalence relation without having a free ultrafilter on $\omega$?
-What about the other direction? I thought I had convinced myself that using an ultrafilter, you can choose representatives for the Vitali equivalence relation, but right now that does not seem clear to me anymore.
-
-REPLY [12 votes]: Here is a tentative (negative) answer to the question of whether the existence of an $E_{0}$ (Vitali) selector implies the existence of a nonprincipal ultrafilter on $\omega$. I need to check some of the details (I'll say where below) but it seems ok to me. I'm posting this now in hope that someone has any ideas for an easier solution.
-Let $P$ be the partial order consisting of countable partial $E_{0}$ selectors, ordered by inclusion. Let (a) be the statement (meant to be applied in the context of Choice) that whenever $D$ is a dense subset of $P$ in $L(\mathbb{R})$ and $F \subseteq P$ is a filter of cardinality less than the continuum ($\mathfrak{c}$), $F$ can be extended to a filter containing a member of $D$. If (a) holds, the continuum is regular, and $|\mathcal{P}(\mathbb{R}) \cap L(\mathbb{R})| = \mathfrak{c}$ (which holds if a measurable cardinal exists, for instance), then one can build $L(\mathbb{R})$-generic filters for $P$.
-For all I know, (a) is a consequence of ZFC plus suitable large cardinals. It is clearly a consequence of the Continuum Hypothesis, but that doesn't seem to be of much use. Assuming that the theory of $L(\mathbb{R})$ is fixed by set forcing (which happens for instance if there exist proper class many Woodin cardinals; this hypothesis can be fine-tuned for our purposes), you would get that forcing with $P$ over $L(\mathbb{R})$ does not add a nonprincipal ultrafilter, if you knew that each of the following statements could be forced along with (a) + `$\mathfrak{c}$ is regular" : (1) for every nonprincipal ultrafilter $U$ on $\omega$ there is a Ramsey ultrafilter in $L(\mathbb{R})[U]$; (2) there are no Ramsey ultrafilters. Each of (1) and (2) is forceable along with $\mathfrak{c}= \aleph_{2}$. I'm not sure who showed this; my guess is Blass in the first case, and Kunen in the second.
-I don't know anything about producing models of (a) by forcing over models of ZFC, but (a) holds in the $\mathbb{P}\mathrm{max}$ extension of any model of $\mathrm{AD}^{+}$ (such as $L(\mathbb{R})$, if there exist infinitely many Woodin cardinals below a measurable cardinal). This does not seem to give much information on the original question, either, though it shows (via a $\Delta$-system argument) that if $G \subseteq \mathbb{P}\mathrm{max}$ is an $L(\mathbb{R})$-generic filter, then in $L(\mathbb{R})[G]$ there are $L(\mathbb{R})$-generic filters for $P$ whose extensions do not contain any ultrafilter on $\omega$ generated by a tower of length $\omega_{2}$ from the point of view of $L(\mathbb{R})[G]$.
-It seems to me that there are natural variations of $\mathbb{P}\mathrm{max}$ for producing models in which the cardinal characteristic $\mathfrak{u}$ is equal to $\aleph_{1}$ (I believe that Woodin has done this, in fact). In these models, (a) seems to hold, and the cardinal characteristic $\mathfrak{g}$ should be $\aleph_{2}$, which gives statement (1), by results of Laflamme.
-More importantly, it gives Near Coherence of Filters (NCF), the statement that any two nonprincipal ultrafilters on $\omega$ are finite-to-one reducible to a common ultrafilter. I don't know about getting (2)+(a) with a $\mathbb{P}\mathrm{max}$ variation. However, if there is a $P$-name for an nonprincipal ultrafilter on $\omega$, then one can map this name to two (continuum many, in fact) independent parts of $P$, and thus show that NCF cannot hold in the $P$-extension of $L(\mathbb{R})$ unless there are no nonprincipal ultrafilters there.
-This argument seems to have nothing to do with $E_{0}$. If it is correct, then it should show, for instance, that if $L(\mathbb{R}^{\#}) \models \mathrm{AD}^{+}$, then for any Borel action on the reals by a countable group, the partial order which adds a choice function for the orbits, with countable conditions, does not add a nonprincipal ultrafilter on $\omega$.
-I expect that this argument is more complicated than necessary in more than one way. One would expect the proof to go though a determinacy-style $\Delta$-system lemma, but I don't see how to do that.<|endoftext|>
-TITLE: On what kind of objects do the Galois groups act?
-QUESTION [40 upvotes]: I am neither number theorist nor algebraic geometer. I am wondering
-whether Galois groups of number fields (say the absolute Galois
-group $Gal(\overline{\mathbb{Q}}/\mathbb{Q})$) act on objects which
-are not related a priori to the number theory.
-I am aware of two such situations of rather different nature:
-(1) Grothendieck's dessins d'enfants:
-$Gal(\overline{\mathbb{Q}}/\mathbb{Q})$ acts on certain graphs (with extra properties and extra data) on 2-dimensional surfaces.
-(2) $Gal(\overline{\mathbb{Q}}/\mathbb{Q})$ acts on the profinite
-completion of the topological $K$-theory (of sufficiently nice
-spaces, e.g. finite $CW$-complexes).
-As far as I understand (am I wrong?) the most important and best
-studied examples of actions of Galois groups are actions on $l$-adic
-cohomology of varieties over number fields. But this is not what I
-am looking for: number fields appear in the formulation of the
-problem from the vary beginning.
-
-REPLY [7 votes]: This is not exactly an incarnation of the question you asked, in the sense that is not so much an action of a Galois group but rather an action whose existence is governed by a Galois group of number-theoretic origin, but it seems likely to be of interest.
-Let $K$ be a number field, and let $K^{(1)}$ be the maximal unramified abelian extension of $K$. The Galois group of $K^{(1)}/K$ is a subquotient of Gal$(\overline{\mathbb{Q}}/\mathbb{Q})$) which is isomorphic to the class group of $K$. Note that by Minhyong Kim's answer here, we can characterize this subquotient purely Galois-theoretically. Several authors have discovered surprising links between the arithmetic of number fields and actions of groups on spheres. In particular, when $K$ is the real cyclotomic field $K_m=\mathbb{Q}(\zeta_m+\zeta_m^{-1})$, the class group appears to govern the free actions of binary dihedral groups on spheres $S^n$ with $n\equiv 3\pmod{4}$. Let me loosely quote/paraphrase from Lang's "Units and Class Groups in Number Theory and Algebraic Geometry" (bolding mine):
-
-C. T. C. Wall has already shown to depend in part on the
-2-primary component of the ideal class group in real cyclotomic fields $K_m^+$ for suitable $m$...Using the algebraic background of a paper
-of Wall, applied to the surgery exact sequence, Thomas gives examples
-for the binary dihedral group $D_{4p}$ of order $4p$ operating freely on $S^{4k-1}$ with $k\geq 2$, when the order of $[(K_p^+)^{( 1)}:K_p^+]$ is odd.
-
-...
-
-Furthermore, according to Thomas, there exist free actions by $D_{4p}$ which
-can be topologically distinguished only by an invariant in the 2-primary part
-of the ideal class group of $K_p^+$.
-
-Perhaps needless to say, the study of these degrees $[(K_p^+)^{( 1)}:K_p^+]$, even their 2-part, is of tremendous interest in algebraic number theory (Vandiver's conjecture, etc.), so the link to actions on spheres is surprising.<|endoftext|>
-TITLE: Ping-pong relief map of a given function z=f(x,y)
-QUESTION [15 upvotes]: I have an idea to design a type of
-Galton's Board
-to "draw" a relief map of a given two-dimensional function $z=f(x,y)$.
-A typical Galton's Board drops, say, ping-pong balls through a series
-of evenly spaced pins into vertical bins to demonstrate that the
-balls distribute according to the binomial distribution, approximating
-the normal distribution:
-
-
-
-
-(See this
-this link
-for an animation.)
-First I would like to generalize this design to approximate
-an arbitrary function $y=f(x)$, which leads to my first question:
-Q1.
-Which class of functions can be represented as a convex combination
-of normal distributions?
-I know these functions are called
-mixture distributions,
-but I have not found a description of the total class representable.
-I am hoping that (say) any smooth function can be approximated.
-Q2. Given a function $f(x)$ to approximate, how could one work backward
-to a pin distribution that would realize the approximation?
-The result would be a type of user-designed Pachinko machine.
-Q3. Can the above be generalized to two-dimensional functions $f(x,y)$?
-Presumably the answer is Yes. If so, one could imagine
-a potentially mesmerizing
-Museum of Math
-display in which some famous visage emerges slowly as a ping-pong relief map.
-
-
-
-
-Q4. This final thought raises the question of which mathematician's face would be simultaneously
-most appropriate and most recognizable. :-) Sir Francis Galton is certainly appropriate...
-
-REPLY [2 votes]: This is a partial answer to Q2, and suggests to me that there is a physical arrangement which would give a yes answer to Q3.
-If you use ordinary pins, you can probably get a dyadic approximation with some arrangement. Let me suggest using weighted pins as a partial solution, and then perhaps someone can implement a close enough approximation to a weighted pin with a series of dyadic pins.
-So normalize things so that the function f has integral one over the interval [0,1], and is to be approximated by 2^k bins. Suppose p in [0,1] is the fraction of balls needed to
-represent the function on [0, 1/2], equivalently p is the integral of f from [0,1/2]. Then place a weighted pin very high such that it dumps p of the balls toward the pin
-over the interval [0, 1/2]. (You may want to put a divider right under this pin so that
-the ball doesn't jump to the [1/2,1] side.) Now recurse (k-1) more levels. Working backwards from this to get a horizontal arrangement should be clear, and of course one can use the physics of the situation to change the endpoints from dyadic rationals to something more appropriate to the desired function f.
-It may be possible to emulate the bias by ever so slight horizontal adjustments of
-the pins, but you need to place the later pins just so that their bias accomodates the various trajectories of the incoming ball. But of course we have infinite precision pins and balls, so what's to worry?
-Gerhard "Likes The Unreality of Mathematics" Paseman, 2011.07.06<|endoftext|>
-TITLE: "double" semidirect product
-QUESTION [8 upvotes]: Let $A$, $B$ and $C$ be discrete countable groups. Let $\alpha$ be an action of $A$ on $B$ and let $\beta$ be an action of $B$ on $C$.
-
-Question Does there always exist a group $G$ which has $A$, $B$ and $C$ as subgroups and such that the group generated by $A$ and $B$ is $A\ltimes B$ and the group generated by $B$ and $C$ is $B\ltimes C$?
-
-One obvious situation when such a group $G$ exists is when the action $\beta$ extends to an action of $A\ltimes B$. Then we can take $G=(A\ltimes B)\ltimes C$. But I'm interested in situations when $\beta$ doesn't extend.
-For example take $A$ to be the infinite cyclic group with generator $t$, $B$ to be the free group on infinitely many generators indexed by $\mathbb Z$ (denote the generators by $g_n, n\in \mathbb Z$), and $C$ to be the rational numbers. Let $g_n$ act on $C$ by multiplication by $n$ if $n\neq 0$ and by identity for $n=0$, and let $t$ act on $B$ by sending $g_n$ to $g_{n+1}$.
-
-Question In this specific situation does there exist $G$ as above?
-
-REPLY [12 votes]: The answer to the specific question is "yes". Let $U$ be the semidirect product of $C$ and $B$. Then as $G$, you take the HNN extension of $U$ with the free letter $t$ and associated subgroups equal to $B$, with the automorphism provided by the shift of generators. The result then follows from the usual properties of HNN extensions. The same argument applies in general when $A$ is cyclic.
-In general you can take the semidirect products and take their amalgamated product with amalgamated subgroup $B$.<|endoftext|>
-TITLE: Example of a continous functor between locally presentable categories which has no left adjoint
-QUESTION [12 upvotes]: It is known a version of Adjoint Functor Theorem for locally presentable categories, which says that a functor between such categories has a left adjoint iff it is continous (i.e. preserves all limits) and accessible (preserves $\lambda$-filtered colimits for some cardinal $\lambda$) (see for this Theorem 1.66 of J. Adamek, J. Rosicky, "Locally Presentable and Accessible Categories", London Math. Soc. Lecture Notes, Cambridge, 1994). My question is then to find an example of a continous functor between locally presentable categories which is not accessible.
-
-REPLY [15 votes]: The following example was coincidentally mentioned by André Joyal on the categories mailing list today; he attributed it to Mac Lane. For every infinite cardinal number k, let $G_k$ be a simple group of cardinality k. Define the functor ML: Group → Set to be the product of all the representable functors $\mathrm{Hom}(G_k,-)$. Since no group can admit a nontrivial homomorphism from proper-class-many of the $G_k$, this functor does indeed land (or can be redefined to land) in Set. Since it is a product of representables, it is continuous (and of course Group and Set are locally presentable), but it is not itself representable (hence has no left adjoint).<|endoftext|>
-TITLE: trace(xy)=trace(yx) in full generality
-QUESTION [15 upvotes]: It is well known that, for square matrix $x$ and $y$, we have $\operatorname{tr}(xy)=\operatorname{tr}(yx)$. Here of course the trace of a matrix is just the sum of the elements of the diagonal.
-The notion of trace has a lot of generalization. As I know, the most general definition is the following: let $(\mathcal C, \otimes, 1, ^\vee)$ be a rigid symmetric monoidal category, $X$ an object of $\mathcal C$ and $f$ an endomorphism of $X$. Then $\operatorname{tr}(f) \in \operatorname{End}(1)$ is defined by the following composition
-$$ 1 \longrightarrow X^\vee \otimes X \stackrel{\operatorname{id}_{X^\vee} \otimes f}{\longrightarrow} X^\vee \otimes X \longrightarrow X \otimes X^\vee \longrightarrow 1 $$
-So my questions is: it is true, in this generality, that $\operatorname{tr}(f\circ g)=\operatorname{tr}(g \circ f)$, for $f$ and $g$ in $\operatorname{End}(X)$?
-Ricky
-
-REPLY [17 votes]: Yes. The string diagram chase can be found on page 8 of Ponto and Shulman - Traces in symmetric monoidal categories.<|endoftext|>
-TITLE: Rosenlicht theorem about uniruledeness and zeroes of holomorphic vector field on complex projective manifold
-QUESTION [5 upvotes]: I heard that there is a theorem due to Rosenlicht which says the following:
-
-Theorem. Let $X$ be a complex projective manifold and $V$ a non-trivial holomorphic vector field on $X$. Then $X$ is uniruled,ie,can be covered by rational curves,if $V$ has a zero.
-
-I have thought for a few days and failed to give myself a proof. Can somebody give me the reference or say something about the idea of proof?
-Thanks in advance.
-
-REPLY [3 votes]: You can look at Lieberman's paper Holomorphic Vector Fields on Projective Manifolds.
-His proof is more or less as follows. A result of Grothendieck asserts that $\mathrm{Aut}^0(X)$, the connected component
-of the identity of the automorphism group of $X$, is an algebraic group which acts
-algebraically on $X$.
-Look at the (analytic) subgroup generated by your vector field and let $G$ be its Zariski closure in $\mathrm{Aut}^0(X)$. Notice that $G$ is abelian.
-If $p \in X$ is a zero of your vector field then
-$p$ is fixed by the action of $G$ on $X$. Thus for $k \in \mathbb N$, $G$ acts on $$\frac{\mathcal O_{X,p}}{\mathfrak{m}_p^k}, $$ where $\mathfrak m_p$ is the maximal ideal of $\mathcal O_{X,p}$. Moreover, if $k \gg 0$ then the action is faithfull.
-Thus $G$ is isomorphic to a linear algebraic group and yet another result of Rosenlicht says that a Zariski-closed abelian
-subgroup of a linear algebraic group is of the form $(\mathbb Cˆ*, \cdot)^r \times (\mathbb C,+)^s$. The action of the factors of this decomposition generate the sought rational curves.
-
-Added later: For an alternative proof see Theorem 6.4 of this paper. There it is proved that the existence of a non-zero section of $\bigwedge^q TX$ vanishing at a point suffices to ensure that $X$ is uniruled.<|endoftext|>
-TITLE: Coherence for pull-backs and push-forwards
-QUESTION [5 upvotes]: Let $p:X \to S$ and $q:Y\to S$ be two objects in the category of ringed spaces over the ringed space
-$S$, and let $f:X \to Y$ be a morphism over $S$.
-Given a sheaf $\mathcal{F}$ of $\mathcal{O}_Y$-modules, there are at least
-two different ways to produce a morphism
-$$
-q_*\mathcal{F} \to p_*f^*\mathcal{F}
-$$
-of sheaves of $\mathcal{O}_S$-modules by just using canonical operations. We could for instance
-apply the adjunction unit for $(f^*, f_*)$ to $\mathcal{F}$, push the map forward to $S$
-and then use the natural isomorphism $q_*f_* \simeq p_*$. That is
-$$
-\mathcal{F} \to f_*f^*\mathcal{F}
-$$
-$$
-q_*\mathcal{F} \to q_*f_*f^*\mathcal{F}
-$$
-$$
-q_*\mathcal{F} \to p_*f^*\mathcal{F}.
-$$
-On the other hand, we could start by applying the adjunction co-unit for $(q^*, q_*)$,
-pull it back to $X$, use the natural isomorpism $f^*q^* \simeq p^*$ and finally use the
-adjuncton $(p^*, p_*)$ to produce a morphism of $\mathcal{O}_S$-modules. That is
-$$
-q^*q_*\mathcal{F} \to \mathcal{F}
-$$
-$$
-f^*q^*q_*\mathcal{F} \to f^*\mathcal{F}
-$$
-$$
-p^*q_*\mathcal{F} \to f^*\mathcal{F}
-$$
-$$
-q_*\mathcal{F} \to p_*f^*\mathcal{F}.
-$$
-In this case we get the same map (something which at least I find tedious to check;
-but I might be thinking of it in the wrong way).
-My question is:
-Is there a general coherence result which frees us from checking such equalities case by case,
-just as in the case of for instance symmetric monoidal categories. I'm thinking of something like:
-Start with a commutative diagram of ringed spaces and a map of sheaves of modules over one of
-the spaces. Is any map produced from this map, by just applying push-forwards and pull-backs and using
-adjunctions, uniquely determined by the strings of symbols in the domain and co-domain of the new map?
-(One could probably state a similar question where we throw the tensor product into the mix of canonical operations.)
-
-REPLY [3 votes]: I would like to revive this old question by answering it with a pointer to my recent preprint Obvious natural morphisms of sheaves are unique, which addresses precisely this problem. In particular, I give an example in the first section that discuses the kind of natural morphism you ask about. I don't yet know how to add tensor products to the mix in general, though there are examples there as well that do concern it when it enters the problem in a non-invasive kind of way.
-The summary answer, by the way is: yes, such morphisms are always unique, as long as the start and end functors are both of the form $f_*$ or $\mathrm{id}$ (both $f^*$ or $\mathrm{id}$ also works), or can be reduced to such by means of adjunctions and isomorphisms like $(fg)_* \cong f_* g_*$. If they necessarily have more then you need some conditions ensuring that certain base-change maps are isomorphisms or else there are counterexamples.
-I should say that despite the intricacy of the morphisms I consider in that paper, it seems hard in practice to find a map constructed from more than a handful of units and counits and isomorphisms such as $(fg)_* \cong f_* g_*$, so although it is a labor-saving device, it seems so far not to save unbounded labor. I would like to be proven wrong on that assertion, however; I know that when you add the $!$ functors the labor really ramps up, but alas, I haven't been able to add those to the mix either.<|endoftext|>
-TITLE: Examples of non-simply connected manifolds with trivial H^1
-QUESTION [12 upvotes]: It is known that, if a topological space is simply connected,its first homology group vanishes. The converse is not true, since for every presentation of a (say, finite) perfect group G we can construct a CW-complex, via generators and relations, having G as a fundamental group. Are there such examples in the class of topological or differentiable manifolds? In other words, does there exist a non-simply connected manifolds with trivial first homology group?
-
-REPLY [8 votes]: Fake Projective planes
-These are smooth complex projective surfaces with the same betti numbers as $\mathbb{CP}^2$, but with infinite fundamental group $\pi_1(X)$ (in fact it is isomorphic to a torsion-free cocompact arithmetic subgroup of $PU(2,1)$).<|endoftext|>
-TITLE: What is the relation of the Kuznetsov-Bruggeman trace formula and the Selberg trace formula?
-QUESTION [7 upvotes]: I have read that there is an elementary way to show that the above mentioned trace fromulas are equivalent in the sense, that each of them can be derived directly from the other. There should exist a short elegant method by Zagier. Where?
-In short, I know how to deduce the Selberg trace formula from Arthur's trace formula at least in principle, how should one proceed to deduce the Kuznetsov formula from the arthur trace formula. What is the utility of the Kuznetsov formula? For which applications is this trace formula more suitable than the Selberg trace formula?
-
-REPLY [5 votes]: The important point is that the Selberg trace formula includes the contribution of the one-dimensional representations, i.e. the nongeneric spectrum, whereas the Kuznetsov formula does not. Spectrally, the nongeneric spectrum contributes so much that it has a tendancy to obscure the contribution of cusp forms. That is the essential difference between the two trace formulae.
-This is especially important with regard to the idea of beyond endoscopy (see Sarnak's letter http://publications.ias.edu/sarnak/paper/487). But see also the work of Frenkel, Langlands, and Ngo.
-Rudnick's thesis (http://www.math.tau.ac.il/~rudnick/papers/myphdthesis.pdf) in principle discusses how to pass from the Kuznetsov formula to the trace formula. Essentially the passage is just an application of Poisson summation.<|endoftext|>
-TITLE: Rational or elliptic curves on Calabi-Yau threefolds
-QUESTION [20 upvotes]: Let $X$ be a Calabi-Yau threefold. From a complex analytic point of view, it is widely believed that it should not be Kobayashi hyperbolic, that is it should always admit some non-constant entire map from the complex plane $f\colon\mathbb C\to X$.
-One could be even more ambitious and ask whether a Calabi-Yau threefold always contains a rational or an elliptic curve (or, more generally a non-constant image of a complex torus).
-Mostly string theorists have produced lots of examples of such manifolds, mainly by adjunction or crepant resolution of singularities. So my question is:
-Is it true that in all known examples of Calabi-Yau threefold one can always find a rational or an elliptic curve (or, more generally a non-constant image of a complex torus)?
-Thanks in advance!
-
-REPLY [9 votes]: Let me give a partial answer.
-Most of the known examples of Calabi-Yau threefolds contain rational curves. However, there exist examples of Calabi-Yau threefolds without rational curves.
-You can find some of them in the paper by Oguiso and Sakurai Calabi-Yau threefolds of quotient type, Asian Journal of Mathematics 5 (2001).
-These threefolds, that the authors call "of Type A", are constructed as the quotient af an Abelian threefold $A$ by a suitable fixed-point free finite group of automorphisms.
-Moreover, a Calabi-Yau threefold $X$ is of type A if and only if $c_2(X)=0$, and in this case the Picard number $\rho(X)$ is either $2$ or $3$.
-In fact, the authors ask as an open question whether every Calabi-Yau threefold of Picard number $\rho \neq 2,3$ contains rational curves.
-I do not know whether Calabi-Yau threefolds of type A contain elliptic curves, but one can probably check this directly, since the construction is very explicit.<|endoftext|>
-TITLE: Contest problems with connections to deeper mathematics
-QUESTION [80 upvotes]: I already posted this on math.stackexchange, but I'm also posting it here because I think that it might get more and better answers here! Hope this is okay.
-We all know that problems from, for example, the IMO and the Putnam competition can sometimes have lovely connections to "deeper parts of mathematics". I would want to see such problems here which you like, and, that you would all add the connection it has.
-Some examples:
-
-Stanislav Smirnov mentions the following from the 27th IMO:
-"To each vertex of a regular pentagon an integer is assigned in such a way that the sum of all five numbers is positive. If three consecutive vertices are assigned the numbers $x,y,z$ respectively, and $y <0$, then the following operation is allowed: the numbers $x,y,z$ are replaced by $x+y$, $-y$, $z+y$, respectively. Such an operation is performed repeatedly as long as at least one of the five numbers is negative. Determine whether this procedure necessarily comes to an end after a finite number of steps".
-He mentions that a version of this problem is used to prove the Kottwitz-Rapoport conjecture in algebra(!). Further, a version of this has appeared in at least a dozen research papers.
-(Taken from Gerry Myerson in this thread https://math.stackexchange.com/questions/33109/contest-problems-with-connections-to-deeper-mathematics)
-On the 1971 Putnam, there was a question, show that if $n^c$ is an integer for $n=2,3,4$,… then $c$ is an integer.
-If you try to improve on this by proving that if $2^c$, $3^c$, and $5^c$ are integers then $c$ is an integer, you find that the proof depends on a very deep result called The Six Exponentials Theorem.
-And if you try to improve further by showing that if $2^c$ and $3^c$ are integers then $c$ is an integer, well, that's generally believed to be true, but it hadn't been proved in 1971, and I think it's still unproved.
-
-The most interesting part would be to see solutions to these problems using both elementary methods, and also with the more abstract "deeper methods".
-
-REPLY [2 votes]: IMO 2003-6:
-
-Show that for each prime $p$, there exists a prime $q$ such that $n^p − p$ is not divisible by $q$ for any positive integer $n$.
-
-This obviously looks like something you want to apply the Chebotarev Density Theorem to. The condition that is to be satisfied translates to $f(X) = X^p-p$ not having a linear factor over $\mathbb{F}_q$. Now by Chebotarev, there are infinitely many primes $q$ for which $f$ is actually irreducible modulo $q$, since $\operatorname{Gal}(f)\cong\operatorname{Aff}(\mathbb{F}_p)$ contains permutations without fixed points, hence certainly $f$ does not have a linear factor.
-[In fact, the condition is not only implied by the irreducibility of $f$, it is equivalent to it. This is a little exercise in field theory: suppose we have $g \mid f$ with $\deg g < \deg f$. Then we have a finite field extension $k/\mathbb{F}_q$ of degree $d
-TITLE: Statistics of Number fields
-QUESTION [14 upvotes]: A goal which I have been pursuing is to understand how number fields are distributed with respects to their invariants. To be more precise I was captivated by the following question:
-Let $N(X,n,G)$ be the number of number fields of dimension $n$ where $G$ is the Galois group of its Galois closure, and their discriminants is bounded by $|X|$ up to isomorphism. This question might be natural to ask: For which group $G\leq S_n$ one might get a positive proportion of number fields when $X\to \infty$.
-From Class Field Theory or one can show it more elementary, using Delone-Faddeev correspondence, for $n=3$, $C_3$ has a density 0. And also when $G$ is an abelian group then the answer would be same as $C_3$ by using class field theory. Prof. Manjul Bhargava proved for $n=4$ , $D_4$ has a positive density, and also he showed for $n=5$ no other groups, except $S_5$, could have positive density.
-I think, once Manjul told me, one might expect for $n=p$, $p$ is a prime number, the only group which can contribute with positive density is $S_p$. But I could not even find a heuristic that why this should be true.
-Is there a heuristic or even a theorem which can support the above expectation? Or more generally what do we know about $N(X,n,G)$?
-
-REPLY [5 votes]: First of all, the most general conjecture about $N(X,n,G)$ is indeed due to Malle, and there are indeed counterexamples to the original formulation due to Kluners. Seyfi Turkelli has a very nice paper which explains "why" those counterexamples arise by means of a function field analogy, and offers a revised version of Malle's conjecture which seems to me pretty solid.
-Why does only $S_p$ give positive density? Because under Malle's conjecture, $N(G,n,X)$ will be asymptotic to $X^{a(G)} \times$(some power of $\log$), where $a(G)$ is at most 1, with equality only when $G$ contains a transposition. So the class of groups expected to have positive density is precisely those containing a transposition. When $p$ is prime, the only transitive subgroup of $S_p$ containing a transposition is $S_p$ itself, so you're done. On the other hand, both $D_4$ and $S_4$ are transitive subgroups of $S_4$ containing a transposition -- indeed, they are the only such, so they're the only two Galois groups that are supposed to arise for a positive density of quartic fields ordered by discriminant.<|endoftext|>
-TITLE: The Hölder continuity condition of the Schauder estimates
-QUESTION [6 upvotes]: The classical Schauder estimates (see the link)
-http://en.wikipedia.org/wiki/Schauder_estimates
-Requires $f\in C^\alpha$ in order to get a solution $u\in C^{2+\alpha}$ of the equation
-$$\Delta u=f$$
-In fact, we can construct a continuous function f, which is not Hölder of any order on a positive meausre set.
-Is it possible to find solution $u\in C^2$?
-Or is there any counterexample to show such solution $u$ does not exist?
-
-REPLY [11 votes]: For $u$ to be from $C^2$ it is enough that the modulus of continuity of $f$ satisfies the Dini condition. For example, modulus of continuity $\omega(h)=1/(|\log h|+1)^2$ is not marjorised by the Holder modulus of continuity $\omega(h)=C h^\alpha$.
-The same goes for coeffitients of elliptic equations. In the work “A priori bounds and some properties for solutions of elliptic and parabolic equations”, Uspekhi Mat. Nauk, 18:4(112) (1963), 215–216, Kruzhkov notes that the problem of continuity of higher derivatives of solutions of elliptic equations is closely connected with the question of when for a function of several independent variables condition of the existence of pure continuous derivatives of order net $l\ge2$ implies the existence of mixed derivatives of the same order. He gives nesessary and suffitient condition for it. Namely, denote $\omega(h)$ the modulus of continuity of the order $l$ pure derivatives of $u$ and $\omega^*(h)=\int_0^h\frac{\omega(t)}t dt$ the second modulus of continuity. Then in order that all mixed derivatives were continuous (in some domain $Q\subset\mathbb R^n$) it is necessary and sufficient that the module continuity $\omega^* (h) $ satisfies the Dini condition. Moreover, if $\omega^* $ satisfies the Dini condition, then all
-mixed derivatives of order $l$ in any strictly interior subdomain $Q'\subset Q$ have
-modulus of continuity, which does not exceed $Cw^* $. If $\omega^*$ does not satisfy the Dini condition, then there exists a function $u$ which at some point of $Q$ have no mixed derivatives of order $l$.<|endoftext|>
-TITLE: Applications of geometric evolution equations.
-QUESTION [9 upvotes]: Hi everybody,
-I'm looking for applications of geometric evolution equations such as the Ricci flow and the extrinsic flows by Gauss and mean curvature. Applications other than topological applications such as geometrization are what I'm looking for and, even better if the application is to a non-mathematical area, such as Mullins' derivation of mean curvature flow as a model for the motion of `grain boundaries'.
-Thanks,
-ML
-
-REPLY [5 votes]: The Willmore flow and the more complicated Helfrich flow are used to model cell membranes.<|endoftext|>
-TITLE: Did Joseph Doob prove that random sequences don't exist?
-QUESTION [9 upvotes]: In the book "The Mathematical Experience" it says:
-
-"An infinite [binary] sequence $x_1, x_2, \ldots$ is called random in the sense of von Mises if every infinite sequence $x_{n_1}, x_{n_2}, \ldots$ extracted from it and determined by a policy or rule R is $\infty$-distributed. Now comes the shocker. It has been established by Joseph Doob that there are no sequences that are random in the sense of von Mises."
-
-A sequence on $\{H,T\}$ is $\infty$-distributed if for each positive integer $k$ and sequence $\vec y \in \{H,T\}^k$ the set $\{n\in {\mathbb N} \colon \langle x_{n},\dots,x_{n+k-1}\rangle=\vec y\}$ has density $2^{-k}$.
-But the definition of von Mises seems so natural to me that if a sequence does not satisfy it then the sequence is not random.
-
-REPLY [7 votes]: There is an excellent article by Sérgio B. Volchan in the American Mathematical Monthly, titled What Is a Random Sequence, which discusses how the von Mises-Wald-Church model of randomness is unsatisfactory. He goes on to explain the proposed candidate for a definition of a random sequence due to Martin-Löf, that of typicality, or "randomness with respect to effective statistical tests". Here randomness is defined with respect to a given measure $\mu$ on infinite binary strings; it turns out to coincide with a natural notion of incompressibility of the sequence.
-Anyway, in short: there are other natural candidates for what it should mean for a sequence to be random, that turn out to work pretty well (and are beautiful), and Volchan's paper is a good place to learn about them.<|endoftext|>
-TITLE: Why is there a weight 2 modular form congruent to any modular form
-QUESTION [13 upvotes]: I got my copy of Computational Aspects of Modular Forms and Galois Representations in the mail yesterday. The goal of the book is "How one can compute in polynomial time the value of Ramanujan's tau at a prime", well, or any other modular form of level 1. It's all very thrilling!
-The following fact is essential: for any modular form $f$ of level 1, and any prime $l$, the mod $l$ reduction of the semisimplification of the galois representation attached to $f$ by Deligne is a 2-dimensional subrepresentation of the galois representation of the $l$-torsion of the jacobian of the modular curve of level $l$.
-Why?
-From what I understand, this is somewhat equivalent (after Shimura and Deligne) to there being a modular form of weight 2 and level $l$ that is congruent to $f$ mod $l$, or something similar. Is this the right statement? Why is it true then?
-Searching far and wide for an introduction to this topic yields very little.
-
-REPLY [6 votes]: If $N\geq 1$ is an integer not divisible by $p$, one can see that any system of Hecke eigenvalues $(a_\ell)$ arising from $S_k(\Gamma_1(N))$ is congruent mod $p$ to a system $(b_\ell)$ arising from $S_2(\Gamma_1(Np^n))$, for some $n$, using an interplay between a theorem of Serre (describing a purely mod $p$ Jacquet-Langlands correspondence), and the more classical, characteristic zero J-L between ${\rm GL}_2$ and the multiplicative group $G$ of the $\mathbf{Q}$-quaternion algebra ramified at $p$ and infinity.
-I know that what I describe here is perhaps not the right way of proving the result you are asking, but it seems to me worth to mention.
-In his '87 letter to Tate Serre proves:
-${\rm Theorem:}$ Systems of mod $p$ Hecke eigenvalues arising from $M_k(\Gamma_1(N))$ are the same as those arising from locally constant function $f:G(A)\rightarrow\overline{F}_p$ that are left invariant under $G(\mathbf{Q})$ and right invariant under a certain open subgroup $K_N$.
-Here $G(A)$ is the adelic group associated to $G$. Notice that the functions considered on the quaternion side are independent of the archimedean variable. Moreover, the double coset $G(\mathbf{Q})\backslash G(A)/K_N$ is finite and any mod $p$ system of eigenvalues arising from it can be lifted to characteristic zero.
-Therefore applying the theorem and then lifting, we see that for any (char. zero) eigensystem $A=(a_\ell)$ arising from $M_k(\Gamma_1(N))$ there is a (char. zero) eigensystem $B=(b_\ell)$ arising from the space of locally constant functions $f:G(A)\rightarrow\mathbf{C}$ such that $A\equiv B$ mod $P$, where $P$ is a fixed prime of $\overline{\mathbf{Z}}$ lying over $p$.
-Assuming that the automorphic form $\Pi_B$ on $G$ associated to $B$ is infinite dimensional, by the J-L correspondence we have that there is a cuspidal automorphic form $\Pi'_B$ on ${\rm GL}_2$ associated to the same eigensystem $B$. The type of $\Pi'_B$ at any finite place other than $p$ is the same as that of $\Pi_B$, while at infinity $\Pi'_B$ is the discrete series of lowest weight $2$. This basically says that there is a cusp form in $S_2(Np^n)$ whose associated system of eigenvalues is $B=(b_\ell)$.
-We are only left with deciding when $\Pi_B$ is infinite dimensional, or can be chosen as such. This happens only for systems of eigenvalues of the form $B=(\chi(\ell)(1+\ell))_{\ell\nmid pN}$, where $\chi:\mathbf{Z}/p\rightarrow\mathbf{C}^*$ is any character (in order to show this one has to consider the particular shape of $K_N$, which I did not even define..). The reduction mod $P$ of such eigensystems are all of the form $(\ell^k+\ell^{k+1})_{\ell\nmid pN}$.
-Concluding: Let $A=(a_\ell)$ be a sytstem of char. zero eigenvalues arising from $M_k(\Gamma_1(N))$, with $p\nmid N$. Assume that the mod $P$ reduction of $A$ is not of the form $(\ell^k+\ell^{k+1})_{\ell\nmid pN}$. Then, there exists a cusp form in $S_2(\Gamma_1(Np^n))$ such that its associated system of eigenvalues $B$ is congruent to $A$ mod $P$.<|endoftext|>
-TITLE: Generalising Vitali Sets to uncountable dense subgroup selectors...
-QUESTION [5 upvotes]: Does there exist an uncountable dense subgroup, $\Gamma$, of the additive group $\mathbb{R}$, such that every selector of the partition of $\mathbb{R}$ canonically associated with the equivalence relation $x \in \mathbb{R}$ & $y \in \mathbb{R}$ & $x − y \in \Gamma$ is nonmeasurable?
-This question is related to a previous question of mine which has not been answered yet.
-
-REPLY [8 votes]: I edited my former answer in light of a recent e-mail exchange with Sławomir Solecki.
-There is a substantial literature on Vitali-like selectors. A major paper in the area is the following, available here.
-Cichoń, J.; Kharazishvili, A.; Węglorz, B. On sets of Vitali's type. Proc. Amer. Math. Soc. 118 (1993), no. 4, 1243–1250.
-There are many interesting results in the above paper, including the fact, observed by Joel Hamkins in his answer, that $MA+\lnot CH$ implies the existence of uncountable subgroups $G$ of $\Bbb{R}$ with no measurable $G$-selectors; where a $G$-selector is a choice function that selects one element from each $G$-coset.
-
-Moreover, Theorem 6 of the above paper shows that under $MA+\lnot CH$ there is even a subgroup $G$ of $\Bbb{R}$ of power $2^{\aleph_0}$ that has no measurable $G$-selector.
-
-This is to be contrasted with Theorem 2 of the paper, which asserts every analytic subgroup $G$ of reals has a measurable $G$-selector.
-
-According to Solecki (private communication) it is apparently unknown if $ZFC$ can prove that there is an uncountable subgroup of the reals all of whose selectors are Lebesgue non-measurable.
-
-However, if one allows extensions of measures, then there is a fundamental distinction to be made (note: the theorem below was first stated and proved with $CH$; but $CH$ was subsequently removed in the paper cited below).
-
-Theorem (Solecki). The following are equivalent for every subgroup $G$ of reals.
-(a) $G$ is countable and dense.
-(b) Every $G$-selector is non-measurable with respect to any translation invariant extension of the Lebesgue measure.
-
-Solecki's result above is a special case of Cor. 2.2. of the following paper of his:
-Translation invariant ideals, Israel J. Math. 135 (2003), 93--110<|endoftext|>
-TITLE: Can a metric conformal to a Kahler metric be Kahler?
-QUESTION [11 upvotes]: Let $X$ be a non-compact complex manifold of dimension at least 2 equipped with a Kahler metric $\omega$. Take a smooth positive function $f : X \to \mathbb R$, and define a new hermitian metric on $X$ by $\tilde \omega = f \omega$. If $f$ is non-constant, then can this new metric ever be Kahler?
-If $\dim_{\mathbb C} X = 1$ the new metric is automatically Kahler because of dimension. If $\dim_{\mathbb C} X \geq 2$ and if $X$ is compact the new metric is never Kahler. Indeed, we have that $d \tilde \omega = d f \wedge \omega$ is zero if and only if $df$ is zero by the hard Lefschetz theorem, so $f$ must be constant if $\tilde \omega$ is Kahler.
-If $X$ is not compact, then to the best of my knowledge we do not have the hard Lefschetz theorem, but does the conclusion on metrics conformal to a Kahler metric still hold?
-
-REPLY [10 votes]: You don't have to use hard Lefschetz to conclude $df=0$ from $\omega\wedge df=0$.
-This is a linear algebra fact, valid pointwise : if $\alpha \in T_x^*X$ satisfies $\omega_x \wedge \alpha=0$, then $\alpha=0$ (of course, assuming $\dim_R X \geq 4$.
-The short argument is that, $\omega_x^{n-1}\wedge : T^*_x X\to \bigwedge^{2n-1} T^*_x X$ is an isomorphism ("pointwise not so hard Lefschetz", so to speak).
-This said, as in Francesco's answer, you can have non proportional conformal riemannian metrics that are Kähler with respect to different complex structures, so that the corresponding 2-forms are no longer (pointwise) proportional.
-
-REPLY [5 votes]: The paper by Apostolov, Calderbank, and Gauduchon that Francesco mentions find different Kaehler structures whose associated Riemannian metrics are conformal to each other. But they correspond to different complex structures $J_+$ and $J_-$.
-I believe what Gunnar is asking is whether or not one can have $f \omega$ be closed and thus Kaehler with respect to the same complex structure $J$ associated to $\omega$. The answer is no, and this has nothing at all to do with compactness or the hard Lefschetz theorem. On any almost Hermitian manifold $(M, J, \omega, g)$, it is a fact that the wedge product with the Kaehler form $\omega$ on the space of $1$-forms is injective, regardless of the compactness of $M$, the integrability of $J$, or the closedness of $\omega$. This follows, for example, from the identity
-$$ \ast( \omega \wedge (\ast ( \omega \wedge \alpha) ) = - (m-1) \alpha $$
-where $\alpha$ is any $1$-form on $M$, where $\ast$ is the Hodge star operator, and the real dimension of $M$ is $2m$. (One sees that the only requirement is that $m>1$.)
-
-REPLY [2 votes]: There are examples in real dimension $4$ of manifolds having two conformally equivalent
-Kahler metrics, inducing the same conformal structure but with opposite orientation.
-See the paper Ambikahler geometry, ambitoric surfaces and Einstein 4-orbifolds by Apostolov, Calderbank and Gauduchon.<|endoftext|>
-TITLE: Relation between Gerstenhaber bracket and Connes differential
-QUESTION [10 upvotes]: Let $C$ be an arbitrary algebra (more generally, a linear 1-category). The following structures are well-known:
-A degree-0 product on the Hochschild cohomology $HH^*(C)$
-$$
- HH^*(C) \otimes HH^*(C) \to HH^*(C)
-$$
-$$
- a \otimes b \mapsto ab
-$$
-A degree-0 action of Hochschild cohomology on the Hochschild homology $HH_*(C)$
-$$
- HH^*(C) \otimes HH_*(C) \to HH_*(C)
-$$
-$$
- a \otimes \gamma \mapsto a\cdot \gamma
-$$
-A degree-1 unary operation on Hochschild homology (Connes differential)
-$$
- HH_*(C) \to HH_*(C)
-$$
-$$
- \gamma \mapsto B(\gamma)
-$$
-A degree-1 binary operation on Hochschild cohomology (Gerstenhaber bracket)
-$$
- HH^*(C) \otimes HH^*(C) \to HH^*(C)
-$$
-$$
- a \otimes b \mapsto a * b
-$$
-The above operations satisfy some well-known relations. (Note that I am not attempting to get the signs right.)
-
-graded commutativity $ab = \pm ba$
-more graded commutativity $a * b = \pm b * a$
-Poisson identity $a * (bc) = (a * b)c + b(a * c)$
-Jacobi identity $a * (b * c) + b * (c * a) + c * (a * b) = 0$
-$B$ is a differential $B(B(\gamma)) = 0$
-various associativities $(ab)c = a(bc)$; $(a * b) * c = a * (b * c)$; $(ab)\cdot\gamma = a\cdot(b\cdot\gamma)$
-
-The following relation, expressing the action of a Gerstenhaber bracket on Hochschild homology in terms of the Connes differential, seems to be less well-known. At least I haven't been able to find it in the literature.
-$$
- (a*b)\cdot\gamma = ab\cdot B(\gamma) - a\cdot B(b\cdot \gamma) - b\cdot B(a\cdot\gamma) + B(ba\cdot\gamma)
-$$
-(Again, I haven't tried to get the signs right.)
-Question: Is there a reference for the above relation?
-Note: The above relation follows from the fact that the first homology of a certain operad space is 4-dimensional, so there must be some relation between the five degree-1 maps $HH^*(C)\otimes HH^*(C)\otimes HH_*(C)\otimes \to HH_*(C)$ which figure in the relation.
-Another note: In cases where $HH^*(C) \cong HH_*(C)$ and there is a BV algebra structure, I think the relation follows from the usual definition of the Gerstenhaber bracket in terms of the BV structure. See the "Antibracket" section of this Wikipedia article.
-
-REPLY [5 votes]: Hi,
-Your formula is due (without the signs!) due to Ginzburg Calabi-Yau algebras (9.3.2)
-as explained in Lemma 15 of my paper, Batalin-Vilkovisky algebra structures on Hochschild Cohomology, Bull. Soc. Math. France 137 (2009), no 2, 277-295
-(sorry for quoting myself!)
-Here is Lemma 15
-Lemma 15 [17, formula (9.3.2)] Let A be a differential graded algebra.
-For any η, ξ ∈ HH ∗ (A, A) and c ∈ HH∗ (A, A),
-{ξ, η}.c = (−1)|ξ| B [(ξ ∪ η).c] − ξ.B(η.c)
-+ (−1)(|η|+1)(|ξ|+1) η.B(ξ.c) + (−1)|η| (ξ ∪ η).B(c).
-In a condensed form, this formula is
-(34) $i_{\{a,b\}}=(-1)^{\vert a\vert+1}[[B,i_{a}],i_b]=[[i_{a},B],i_b].$
-See formula (34) of my second paper
-Van Den Bergh isomorphisms in String Topology, J. Noncommut. Geom. 5 (2011), no. 1, 69-105.
-(sorry for quoting myself again!)
-In this paper, I thought I gave a new definition of BV-algebras.
-But this definition appears more or less in the
-section "Compact formulation in terms of nested commutators."
-of the Wikipedia article, you quote!
-However, I was unable to find this definition in the bibliography quoted in the
-Wikipedia article.
-Concerning signs,
-in my first paper, I made a mistake, corrected in my second paper.
-So (34) is correct and Lemma 15 has some signs problems.
-ps: David Ben-zvi is absolutely right. This formula is a consequence of Tamarkin-tsygan
-calculus!<|endoftext|>
-TITLE: Finite-dimensional subgroups of circle diffeomorphism group
-QUESTION [8 upvotes]: Is there a sequence of connected finite-dimensional subgroups Gi of the circle diffeomorphism group G with the following properities:
-(a) Gi is contained in Gj for i < j
-(b) The union of Gi is dense in G
-More rigorously "finite dimensional subgroup of circle diffeomorphism group" means a Lie group H with smooth faithful action on the circle.
-In order to make sense of property (b) I have to specify a topology on G. I suspect that all reasonable topologies will yield the same answer, but for the sake of definiteness let's use the
-"sup-norm" topology. That is, given two diffeomorphism g1 and g2, I define the distance d(g1, g2) as
-supremum over x in S1 of d(g1(x), g2(x))
-Here the latter d is the usual distance on the circle.
-This is a metric and it induces a topology.
-I suspect that the answer to my question is "no". Moreover, I suspect that there is no H as above with dimension > 3. But I might be wrong...
-
-REPLY [15 votes]: The answer is indeed no, as described e.g. in the lecture notes by Ghys
-http://www.math.ethz.ch/~bgabi/ghys%20groups%20acting%20on%20the%20circle.pdf
-Section 4.1 has a list of all connected groups acting faithfully and transitively on the circle or the line. They are
-1) $\mathbb{R}$ acting on itself,
-2) the circle acting on itself,
-3) the affine group of the line acting on the line, and
-4) the k-fold cover of $PSL(2,\mathbb{R})$ acting on the circle.
-Any faithful action of a connected Lie Group on the circle is made out of these: if $F$ is the set of fixed points, then on each connected component of the complement of $F$ the action must be conjugate to one of the above.<|endoftext|>
-TITLE: How to motivate and interpret the geometric solutions of Hamilton-Jacobi equation?
-QUESTION [10 upvotes]: Studying the Hamilton-Jacobi equation, I meet a generalization of the notion of its solutions, which is found already in the work of Sophus Lie.
-
-For an H-J eqn, I mean a first order pde $H\circ dS=0$ in an unknown scalar function $S$ defined on a smooth manifold $M$, where $H\in C^\infty (T^\ast M,\mathbb{R})$.
-
-If $S$ is a solution then the image $\Lambda$ of its differential $dS$ is included in $H^{-1}(0)$ and has the following properties:
-
-$\Lambda$ is a lagrangian submanifold of $(T^\ast M,d\theta_M)$,
-$\Lambda$ is transversal to the fibers of $\tau_M^{\ast}:T^\ast M\to M$,
-the restriction of $\tau_M^{\ast}$ to $\Lambda$ is injective.
-
-Conversely, if a submanifold $\Lambda$ of $T^\ast M$, included in $H^{-1}(0)$, satisfies the properties 1, 2, and 3, then it is equal to the image of the differential of a solution, unique up to a constant.
-But if a submanifold $\Lambda$ of $T^\ast M$, included in $H^{-1}(0)$, satisfies only the conditions 1 and 2, then, around each of its points, it is again equal to the image of the differential of a solution, but this can fail to holds globally.
-The idea of Sophus Lie was to give up both conditions 2 and 3.
-
-Adopting this point of view, we define a generalized (or geometric) solution of $H\cic dS=0$ to be any lagrangian submanifold $\Lambda$ of $(T^\ast M,d\theta_M)$ which is included in $H^{-1}(0)$.
-
-I don't think that this generalization is only due to the sake of abstractness.
-Infact, considering generalized solutions, it is possible, arguing with tecniques from symplectic geometry, to prove the local existence and uniqueness theorem, at the same time, for generalized and usual solutions.
-But I am hoping to find "more" practical applications which illustrate the meaningfulness of geometric solutions. I would like to learn if ther is some physical or geometrical problem involving an H.-J. eqn, whose comprehension is sensibly augmented by the consideration of generalized solutions. So my question is:
-
-What are the possible arguments and applications that motivate and help to interpret the notion of geometric solutions for an Hamilton-Jacobi equation?
-
-REPLY [4 votes]: The famous KAM tori arose out of HJ considerations.
-They are Lagrangian torii. They were found by attempting to solve
-the HJ equation generally, and then finding one can only solve it
-when certain appropriately irrational frequency conditions hold.
- They occur in perturbations of integrable systems, or near `typical' linearly stable periodic orbits in a fixed Hamiltonian systems.
-You can read about them in an Appendix to Arnol'd's Classical Mechanics, and
-also get some idea from Chris Golé's book 'Symplectic Twist Maps',
-or from Siegel and Moser's 'Stable and Random Motion'.<|endoftext|>
-TITLE: Surface equivalent of catenary curve
-QUESTION [19 upvotes]: A catenary curve
-is the shape taken by an idealized hanging chain or rope under the influence
-of gravity. It has the equation $y= a \cosh (x/a)$.
-My question is:
-
-What is the shape taken by an idealized, thin two-dimensional sheet,
- pinned on a plane parallel to the ground, under the influence of gravity?
-
-The answer surely depends on how it is pinned to the plane, the boundary
-conditions.
-Natural options are:
-
-A disk sheet fixed to a circle.
-A square sheet fixed to a square.
-A square sheet pinned at its four corners.
-
-The middle option above would look something like this when inverted:
-
-
-
-
-
-
-(Image by Tim Tyler at hexdome.com.)
-
-
-I don't think any of these shapes is a
-catenoid,
-which is the surface of revolution formed by a catenary curve.
-Is there a simple analytic description of any of these surfaces,
-analogous to the $\cosh$ equation for the catenary curve?
-I have been unsuccessful in finding anything but simulations of solutions
-of the differential equations.
-This question arose in imagining a higher-dimensional version
-of the property that an inverted catenary supports smooth rides of a square-wheeled
-bicycle
-(explored in this MO question).
-Thanks for pointers!
-
-REPLY [16 votes]: A model equation for an inextensible, flexible, heavy surface in a gravitational field was deduced by Poisson Lagrange and later the problem was also studied by Poisson (see the references in the linked papers below). The equilibrium condition for a hanging heavy surface of constant mass density reads
-$$\sqrt{1+|\nabla u|^2}\ \nabla\cdot{}\frac{\nabla u}{\sqrt{1+|\nabla u|^2}}=\frac{1}{u+\lambda},\qquad x\in\Omega\subset\mathbb R^2,\qquad\qquad(1)$$
-where $u=u(x)$ is the vertical displacement and $\lambda\in\mathbb R$ is an arbitrary constant (a Lagrange multiplier). (1) is the Euler equation of the variational integral
-$$I(u)=\int_{\Omega}u\sqrt{1+|\nabla u|^2}dx,$$
-which can be interpreted as the vertical coordinate of the center of gravity of the surface
-$$\mbox{graph}(u)=\{(x,u(x)):\ x\in\Omega\}\subset\mathbb R^2\times\mathbb R.$$
-Equation (1) is to be supplemented with the requirement that the surface has a prescribed area $A$
-$$\qquad\qquad\qquad\qquad\qquad\int_{\Omega}\sqrt{1+|\nabla u|^2}dx=A,\qquad\qquad\qquad\qquad\qquad\qquad\quad\quad(2) $$
-and the Dirichlet boundary condition describing the curve from which the surface is being suspended
-$$\qquad\qquad\qquad\qquad\qquad\qquad\qquad\quad\left.u\right|_{\partial \Omega}=g.\qquad\qquad\qquad\qquad\qquad\qquad\qquad\qquad\quad(3) $$
-One can check formally that a solution to (1)-(3) provides a graph of a heavy surface of prescribed area and boundary with the lowest center of gravity, so this is a precise 2D analogue of the classical catenary problem.
-It is known that problem (1)-(3) has no classical solutions for the values of area $A$ outside of some bounded interval $[A_{\min},A_{\max}]$. Moreover, the corresponding variational problem has no global solutions for all $A\in\mathbb R$. A short survey of some old and relatively new results concerning well-posedness of (1)-(3) and its multidimensional analogues can be found in the paper by Dierkes and Huisken, "The N-dimensional analogue of the catenary: Prescribed area", in J. Jost (ed) Calculus of Variations and Geometric Analysis, Int. Press (1996), pp. 1-13.
-Addendum. Here is a more recent survey by Dierkes: "Singular Minimal Surfaces" (in Geometric Analysis and Nonlinear Partial Differential Equations, Springer (2003), pp. 177-194).<|endoftext|>
-TITLE: Ramification in p-division fields associated to elliptic curves with good ordinary reduction
-QUESTION [11 upvotes]: Dear MO,
-Let $p$ be a prime and let $E/\mathbb{Q}_p$ be an elliptic curve. Suppose that $E/\mathbb{Q}_p$ has good ordinary reduction at $p$. In his 1972 paper ``Propriétés galoisiennes des points d'ordre fini des courbes elliptiques'' (more specifically, see Corollaire, in p. 274), Serre shows along the way that the inertia subgroup of $\operatorname{Gal}(\mathbb{Q}_p(E[p])/\mathbb{Q}_p)$ is, with respect to a suitable basis of $E[p]$, isomorphic to either a matrix group of the form {$[\ast\ 0; 0\ 1]$} or {$[\ast\ \ast; 0\ 1]$} as a subgroup of $\operatorname{GL}(2,\mathbb{F}_p)$. After this result, Serre remarks that he doesn't know of any simple criterion that would determine whether one is in the first case or the second case.
-Question: Nowadays, do we know of a criterion to tell whether one is in the first case or the second case?
-A more concrete question: Here is the particular example that I am working with: Let $E/\mathbb{Q}$ be ``1225h1'' in Cremona's tables, given by
-$$E : y^2 + xy + y = x^3 + x^2 - 8x + 6. $$
-This curve has a rational $37$-isogeny and therefore $\operatorname{Gal}(\mathbb{Q}(E[37])/\mathbb{Q})$ is a Borel subgroup of $\operatorname{GL}(2,\mathbb{F}_{37})$. The curve $E$ has good ordinary reduction at $p=37$ and I am trying to find out whether the ramification index of $37$ in the extension $\mathbb{Q}(E[37])/\mathbb{Q}$ is just $\varphi(37)$ or rather $\varphi(37)\cdot 37$, where $\varphi$ is the Euler phi function.
-The $37$th division polynomial of $E/\mathbb{Q}$ has degree $684$ and it factors (over $\mathbb{Q}[x]$) as a product of $4$ polynomials of degrees $6$, $6$, $6$ and $666$, respectively. The extension of degree $666$ is, well, diabolically large and I can't find the ramification at $37$ computationally... or at least I don't know how to!
-Thanks for your help!
-
-REPLY [12 votes]: Assume $p \ne 2$. The condition for the representation to be tamely ramified (i.e $* = 0$ in the upper right entry of the matrix) is that $j(E) \equiv j_0 \mod p^2$ where $j(E)$ is the $j$-invariant of $E$ and $j_0$ is the $j$-invariant of the canonical lift of the reduction of $E$. This is proved in Gross "A tameness criterion for galois representations..." Duke J. 61 (1990) on page 514. For $p=2$ you need the congruence modulo $8$. Serre gives an algorithm for computing $j_0$ in Lubin-Serre-Tate.<|endoftext|>
-TITLE: Reference sought for Conway's observation on stable matchings
-QUESTION [5 upvotes]: Looking for a reference on the observation that the set of stable matchings form a distributive lattice. This is attributed to Conway by Knuth in "Marriages Stables" but I would like an explicit reference if possible, likely a text book. An undergrad student needs it for his bibliography in a term paper.
-
-REPLY [5 votes]: strong text
-Conway discovered it when visiting Montreal contemporaneously to Knuth's
-lectures on the marriage problem. These lectures are in print, published by Centre de
-Recherche Math of Universite de Montreal.<|endoftext|>
-TITLE: Reference letters for teaching positions
-QUESTION [11 upvotes]: I've been told that when applying for a teaching position, your reference letters can be written by anyone who is familiar with your teaching capabilities in detail. I feel that this primarily just means students, but I wonder if there's some unspoken rule that reference letters should come from people in positions of authority, e.g. professors for whom I've served as TA, administrative staff in the math department, etc. In reality, it's the students who know my teaching capabilities in detail, and perhaps to a lesser degree my friends, whereas professors and administrators have no direct knowledge my teaching abilities whatsoever, and might only have heard things here or there from students, or have read my student evaluations and seen the scores.
-
-So should reference letters predominantly come from authority figures, or is it okay to have them all come from students?
-
-REPLY [2 votes]: This is a slightly different response as most people are assuming you are intending to teach at the university level. I respond only for informational purposes for those teaching not at the university level.
-As an undergraduate, I took an "advanced" math course with a Ph.D graduate student as the teacher. After finishing his degree, he ended up seeking prep school and community college positions out of interest to teach small groups of students. Not only did these schools ask for "authoritative" recommendations, but a number also asked for multiple student recommendations.
-Admittedly, I was asked to write one -- I do not think I really had the perspective then, nor now, to write something. But it seems these less than university positions desire that additional information.<|endoftext|>
-TITLE: What is the complexity of this problem?
-QUESTION [23 upvotes]: Recently on Dick Lipton and Ken Regan's blog there was a post about problems of intermediate complexity, that is, NP problems that are harder than P but easier than NP-complete. The main message of the post was that most examples of natural questions that have been conjectured to be intermediate have subsequently been shown not to be -- the two most notable exceptions being the discrete logarithm problem (and factoring), and the graph isomorphism problem. There have been related questions here on Mathoverflow and also on TCS stackexchange.
-I asked about the status of one particular problem in a comment on the blog post but got no answer, so I'll ask it again here. The problem is the following. You're given a non-singular $n\times n$ matrix over $\mathbb{F}_2$ and asked to determine whether it can be reduced to the identity using at most m Gaussian row operations. It seems very unlikely that this problem is in P, since the problem of finding an explicit matrix that needs a superlinear number of operations is open (and, I think, thought to be hard). On the other hand, if I try to take a problem like 3-SAT and reduce it to this one, I don't have any idea where to start. But that is very weak evidence for the assertion that the problem is not NP-complete, so I'm asking again the question I asked in the blog comment: is anything known, or intelligently conjectured, about the status of this problem?
-The question can be reformulated as asking for the length of the shortest path between two vertices in a certain Cayley graph. (NB, that doesn't make it polynomial-time because the number of vertices in the graph is exponential.) I'd be just as happy to be told that the same problem in a different Cayley graph was probably of intermediate complexity: I chose the Gaussian elimination example because I happen to like that graph.
-
-REPLY [5 votes]: If one relaxes the question to asking the row-reduction distance of arbitrary matrices over $\mathbb{F}_2$ then it can be shown that the problem is $NP$-Complete. That is, consider
-
-RRD (Row Reduction Distance):
-Input: $m\times n$ matrices $M$, $N$
- over $\mathbb{F}_2$, and an integer
- $k$
-Output: Whether $M$ can be row-reduces
- to $N$ in $\le k$ steps
-
-The claim is that this problem RRD is $NP$-complete. It is within $NP$, so the hardness is all that remains. To do this, consider the following related problem.
-
-MHW (Min Hamming Weight):
-Input: $P\in\mathbb{F}_2^{m\times
-> n}$, $b\in\mathbb{F}_2^n$, integer $k$
-Output: Is $b$ expressible as a linear
- combination of $\le k$ rows of $P$.
-
-I'll first show that MHW reduces to RRD, then show that RRD is $NP$-hard. This together will show RRD is NP-hard.
-Let $(P,b,k)$ be an instance of MHW. Create $M$ and $N$ as $(m+1)\times n$ matrices, where $M$ is just $P$ with the zero row appended on, and $N$ is just $P$ with the row $b$ appended on. I'll now show that $b$ is expressible as a linear combination of at most $k$ rows of $P$ iff $M$ is row-reducible to $N$ it at most $k$ steps.
-The forward direction of this claim is straightforward. Now for the backward direction. Row-reduction operations over $\mathbb{F}_2$ are just "add this row to that row", or "swap this and that row". It follows that in $\le k$ row-reductions have at most $k$ "source" rows of where the additions come from. Thus, the last row of $N$ is equal to the last row of of $M$ (which is zero) plus at most $k$ other rows of $M$. This is exactly what was wanted, as the last row of $M$ is just $b$.
-This completes the proof that MHW reduces to RRD.
-Now let's show that MHW is NP-hard. We'll do so with:
-
-SET-COVER
-Input: Sets $S_1,\ldots,S_m\subseteq[n]$, integer
- $k$,
-Ouptut: Decide if $[n]$ is the union of $\le k$ of the sets $S_i$
-
-It is know that Set-Cover is NP-complete. Here we need a slightly stronger version of this fact, where all of the sets are of constant size, and this is still NP-complete. (To see this, one first shows that 3SAT is still NP-complete when each variable appears in at most 3 clauses. Then one runs through the "standard" reduction from 3SAT to Set-Cover, and notices that all of the sets are of constant size).
-Now consider a set-cover instance. Note that if we through in all subsets of each $S_i$, the answer to the cover question doesn't change. Note that we can do this as each subset is of constant size, so there aren't too many subsets to add. Thus we get
-
-Hered-Set-Cover
-Input: Let $\mathcal{S}\subseteq
-> 2^{[n]}$ be a family of sets, each of
- size $O(1)$, that is subset closed.
- Let $k$ be an integer.
-Ouptut: Decide if $[n]$ is the union
- of $\le k$ of the sets from
- $\mathcal{S}$.
-
-and Hered-Set-Cover is NP-complete, as argued above. We'll now reduce Hered-Set-Cover to MHW. Take a Hered-Set-Cover instance, with the family of sets $\mathcal{S}$ and integer $k$. Suppose there are $m$ sets. Then write out $P$ to be the $m\times n$ matrx where the rows are the indicator vectors for the sets in $\mathcal{S}$. The target vector $b$ is the all ones vector, and $k$ is as in the original problem. So if $b$ is a $\le k$ linear combination of the rows, then this immediately gives the set-cover of $\le k$ sets. If there is a set-cover of $\le k$ sets, then we can always pass to subsets so that each element in the ground set is covered exactly once, and thus when we sum up the relevant vectors in $\mathcal{F}_2$ we never run into the issue that 2=0.
-So really we are doing a "set-cover with odd covering at each vertex" in the MHW instance. The point is that allowing the subsets in the family makes the exact number of coverings irrelevant, and so we can assume things are covered exactly once.
-It seems like there might be a more direct reduction from the problem Exact-Cover (where in set-cover we require that each element be covered exactly once). Indeed, I sort of just untangled the reductions needed to use Exact-Cover. But Exact-Cover doesn't seem exactly right, because if $b$ is a $\le k$ linear combination this doesn't immediately translate to an exact cover.
-This approach doesn't seem to address the issue when $M$ and $N$ are full-rank in the RRD problem, as the reduction of MHW to RRD needs non-full-rank, and the MHW problem is solvable in polynomial time when $P$ is full rank.<|endoftext|>
-TITLE: Finite-dimensional subgroups of diffeomorphism groups
-QUESTION [8 upvotes]: This question is a generalization of my previous question about the circle to arbitrary manifolds.
-Is there a smooth manifold M with the following property.
-There exists a sequence of connected finite-dimensional subgroups Gi of M's diffeomorphism group G such that
-(a) Gi is contained in Gj for i < j
-(b) The union of Gi is dense in G
-To remove doubt, "finite dimensional subgroup of M's diffeomorphism group" means a Lie group H with smooth faithful action on M.
-The answer to my previous question established that S1 is not such a manifold. I suspect that the answer to the general question is still "no". However, the proof would have to be more sophisticated since in the case of S1 we had essentially a closed list of possible "Hs".
-There is another closely related question. Fix a smooth manifold M. Consider connected Lie groups H with faithful and transitive smooth action on M. Is there an upper bound of H's dimension? For S1 the answer was "yes, 3".
-
-REPLY [10 votes]: I guess that the answer to your first question is no, based on the following: If the union of the $G_i$ were dense in $G=\mathrm{Diff}(M)$, then, presumably, for $i$ sufficiently large, the action of $G_i$ would be primitive (i.e., it would not preserve any nontrivial foliation) and locally transitive. The list of the effective, primitive, transitive Lie group actions is known, and, by examining this list, one sees that the dimension of such a group acting on an $n$-manifold is at most $n^2{+}2n$.
-The answer to your second question depends on the manifold. First, I suppose you have to restrict to the case in which $M$ actually has a transitive smooth action of a finite dimensional Lie group. (For example, any compact orientable surface $M^2$ of genus $2$ or more is not a homogeneous space.)
-It is not hard to come up with cases for which there is no upper bound. For example, let $M = \mathbb{R}^2$ and consider the group $G_d$ that consists of transformations of the form $\phi(x,y) = \bigl(x{+}a,\ y{+}p(x)\bigr)$ where $a$ is any constant and $p$ is any polynomial of degree $d$ or less. Then $G_d$ acts transitively on $M$ for all $d\ge0$ while $\dim G_d = d+2$. Thus, for $M=\mathbb{R}^2$ the dimension of such $H$ can be arbitrarily high. A similar argument with trig polynomials will provide such an example on the torus, which is compact. (N.B.: The action of $G_d$ is not primitive since it preserves the foliation by the lines $x=c$; thus, this example does not contradict my first paragraph.)
-On the other hand, for $M=S^2$, there is an upper bound for the dimension of a connected Lie group that acts faithfully and transitively on $M$. That upper bound is 8 ($=2^2+2\cdot2$) and is achieved by $SL(3,\mathbb{R})$ acting on $S^2$ regarded as the space of oriented lines in $\mathbb{R}^3$. In fact, you can say more: Any connected, transitive finite dimensional Lie subgroup of the diffeomorphism group of $S^2$ is conjugate to one of $SO(3)$, $PSL(2,C)$, or $SL(3,\mathbb{R})$. The latter two are maximal and contain the first one as maximal compact. (The easiest proof that I know of these statements uses the classification of primitive actions, at least in dimension $2$.)<|endoftext|>
-TITLE: Groups which satisfy Mal'cev's theorem (locally residually finite)
-QUESTION [12 upvotes]: Recall that a group $G$ is residualy finite if for every non-zero element $g\in G$ there exists a homomorphism $\sigma:G\rightarrow H$ such that $H$ is finite and $\sigma(g)\neq 0$. Mal'cev's theorem says that if $k$ is a field then any finitely-generated subgroup of $GL_n(k)$ is residually finite. For a proof of this theorem, see Steve D (Smith?)s answer to this question MO:9628. Note that the theorem does not say that $GL_n(k)$ is residually finite.
-
-Does anyone know any other classes of groups with the property that any finitely generated subgroup is residually finite.
-
-I would prefer examples with the following two properties: first, the group is not itself residually finite, and second, it is not simply a subgroup of some $GL_n(k)$. So, this excludes, for instance, free groups.
-Side question: is there a good name for this property?
-
-REPLY [4 votes]: There are uncountably many Tarski monsters, which are finitely generated simple infinity groups whose proper subgroups are cyclic. No infinite simple group is residually finite, so Tarski monsters aren't, but their proper subgroups obviously are residually finite.
-
-REPLY [3 votes]: Perhaps a locally finite simple group would help? How about the subset of all even permutations of the natural numbers with finite support? Unless I am misremembering something, this should be a simple locally finite group that is not a subgroup of GL_n(k).
-Gerhard "Email Me About System Design" Paseman, 2011.07.09<|endoftext|>
-TITLE: Program transformation as alternative for Hoare logic or temporal logic
-QUESTION [6 upvotes]: When trying to prove something about a program, the known techniques are Hoare logic and temporal logics.
-An alternative is to transform a program in a mathematical (logical) expression. So, rather that mathematics is used to prove some properties of the program, the program itself is a piece of mathematics.
-Loops become transitive reflexive closures. Example, if one has a program that calculates a Fibonacci number. If the program keeps the last two numbers of the Fibonacci sequence in variables, then this be converted by taking the transitive reflexive closure of the relation P, that is true (and only true) for the following situation:
-$$ P((x,\space y, \space z), \space (x+1, \space z, \space y+z)) $$
-In the original program, the right value is chosen within the loop. In the transitive reflexive closure, the right value must be selected outside the closure (loop). The transformed program is more like a non-deterministic program.
-The transformation of a program in a logical expression, can be done automatically.
-Although, this is not rocket science, I can not find any reference for this approach. I am busy with writing an article, where this is a part of (it is not the main subject). But I want to refer to the right articles and look if there is interesting material.
-Does someone has interesting references?
-Many thanks,
-Lucas
-Edit: Given the comment of Andreas, some clarification. The goal is to make formal reasoning about the program possible. So, transforming the program in a declarative language is insufficient, because the declarative language may not have means to make conclusions about a program, although the language itself might precisely defined. I was thinking in transforming the program in a FOL + PA expression. After such transformation, formal (that is why I tagged with lo.logic) reasoning can be done about the program. As far to my knowledge, I haven't seen this approach (the methods are always more in the direction of Hoare and temporal logics), although it is not very complicated. In my question I didn't want to restrict to FOL + PA.
-
-REPLY [4 votes]: Hoare logic and temporal logic might be "the only known techniques for proving programs correct" to you, but there are certainly others!
-For example, and this list is not exhaustive:
-
-equational reasoning about fixpoints, this works in languages like Haskell
-properties of programs can be proved via denotational semantics, which in itself is a vast area including domain theory and game semantics, to name just two.
-for certain kinds of programs, for example for parametrically polymorphic ones, there are techniques that go under the name "relational parametricity"
-you can use various logical interpretations to get correctness of programs:
-
-a program extracted as a realizer via the realizability interpretation of logic automatically satisfies a certain specification
-with tools such as Coq you can use type theory to write programs as proofs, or construct programs and prove them correct all at once
-there are other ways of extracting programs from logical statements, one family of which are variants of Gödel's Dialectica interpretation that extract programs from classical logic.
-
-
-Now, regarding your specific question. I think you should look at realizability, type theory, and extraction of programs from proofs. All of these are "logical" methods for developing correct programs, or proving them correct. Some randomly chosen starting points:
-
-start with something fun and surprising, perhaps Paulo Oliva's tutorial on Programs from classical proofs via Gödel's dialectica interpretation
-an accessible paper on realizability interpretation which uses logical methods in computable analysis might be Ulrich Berger's Realisability for Induction and Coinduction with Applications to Constructive Analysis
-if you want to use computers to actually show correctness of programs, you could learn Coq and then proceed to Ynot (Hoare logic on steorids) or go straight to Adam Chipala's Certified Programming with Dependent Types.
-cool people use Agda instead of Coq.
-if you are first-order logic sort of person, you might find Minlog more palatable than Coq and Agda, as it does not throw type theory in your face.
-
-See you in two years.<|endoftext|>
-TITLE: Changing field of study post-PhD
-QUESTION [51 upvotes]: I am doing my PhD in algebraic graph theory, for not much more reason than that was what was available. However, I love deep structure and theory in mathematics, and I do not particularly want to be a graph theorist for the rest of my life.
-I have heard of mathematicians changing from more theoretical subjects to broad ones like combinatorics, but not the other way round, and am concerned that this is because the time it takes to learn the requisite theory for deeper subjects is hard to come by after grad school.
-Has anyone done this, or known of someone who has? Is it at all likely or possible I will be able to get a post-doc position in a subject only marginally connected to my thesis? I have been using small amounts of algebraic number theory, and would ideally like to go on and specialise in something similar.
-Any advice much appreciated!
-EDIT (June '12) - I came across this old question of mine, and now feel rather silly for having asked it at all. For the record, and for anyone who might be having similar doubts to me: a year or so on I have realised that, as was mentioned below, I should not pigeonhole myself, and that the most interesting problems are often those that lead to other seemingly disparate subfields. Perhaps more to the point, I actually can't now think of a subject I'd rather do than algebraic graph theory! If you find yourself in the position I was in, I advise you to just look for those problems you find most interesting in your "chosen" field - they are bound to lead to other areas. And anyway, surely all that matters is that you are interested and inspired?! In my own research I have used number theory, galois theory, group theory, and even a bit of probability. I might even opine that graph theory is one of the subjects most likely to appear in intra-disciplinary work, which seems to be forming an ever higher proportion of mathematical research.
-
-REPLY [42 votes]: Speaking as someone whose thesis was also in algebraic graph theory but who has later gone on to do research in other areas, I would say that it is definitely possible to switch fields. The main skills you need are management skills: the ability to manage your own time so that you can spend some time learning a new field while also producing something that others will value, and the ability to manage other people's view of your work. In this regard, I believe that Deane Yang's comment is right on the money. You probably can't afford to drop your initial specialty abruptly and produce no results while you retrain yourself. But if you manage things carefully then anything is possible, regardless of whether you have tenure or are switching to a field that requires a lot of background study.
-There are some things you can do to help you solve these management problems. If you can find overlap between your current field and your new field, that will obviously help. Mathematics is so interconnected that this is usually not too hard; in the specific case of algebraic graph theory versus algebraic number theory, the first topic to come to mind is the Ihara zeta function of a graph, but I'm sure there are others.
-In my case, I decided it would help to switch from academics to industry/government. I personally found it easier in industry/government to spend x% of my time producing results that pleased my employers and spending the remaining (100-x)% of my time training myself in a new area. Your mileage may vary, of course; in my case, I found that teaching drained me of too much of my energy, but this is not true of everybody.
-One last comment I have is that if you take this route, then you will need the ability to maintain a clear sense of your own identity and goals and not be unduly swayed by other people's categorizations. For example, when I switched out of academics, many others regarded me as "leaving mathematics." In fact I was only leaving academics and not leaving mathematics, and it was important for me to ignore other people's view of the matter. As another example, people will want to pigeonhole you as a "something-ist" (and it seems you have been influenced by this point of view, since you use the phrase "being a graph theorist for the rest of my life"); you should resist this pigeonholing, and instead think of yourself just as someone with certain abilities and interests. Thinking of yourself as either a graph theorist or a number theorist is unnecessarily limiting. (Of course you may need to bill yourself as one or the other for the purposes of managing other people's view of you while you're making a transition, but you should not necessarily believe your own propaganda.)<|endoftext|>
-TITLE: Lie groups admitting flat (bi)invariant metrics.
-QUESTION [8 upvotes]: I would like to see an example of a non-abelian compact lie group admitting a bi/left/right-invariant flat metric.
-
-Is there any non-abelian compact lie group admitting a flat metric that is bi or one-sided invariant?
-
-The motivation is the following: as a manifold a torus does admit a flat metric, but it is also a abelian group. The first impression would be that the triviality of the lie algebra could imply the existence of a flat metric, or even be a necessary condition. Of course it is reasonable to ask for some relationship between the metric and the algebraic structure; thus some invariance would be expected (even though I would be happy to see a less restrictive assumption and a more powerful restriction dictated by the algebraic structure in the compact case).
-Thus one could ask for an example of a non-abelian lie group admitting a flat metric.
-A easy counter example (flat $\Rightarrow$ abelian) is the group of affine transformations of the line. The connected component of the identity is diffeomorphic to the real plane, so as a manifold (forgetting about the group structure) it does admit a flat metric. But if one considers a one-sided invariant metric one gets the Lobachevsky plane.
-Since I could not think of a counter example of a non-abelian compact lie group admitting a flat metric (not necessary related to the group structure), I will also be happy to see a counter example of it. I will keep the invariance of the metric on the question though.
-P.S.: Since for bi-invariant metrics $R(X,Y)Z=\frac{1}{4}[[X,Y],Z]$ (see e.g.: do Carmo, Riemannian Geometry), it is true in this case that abelian $\Rightarrow$ flat.
-EDITED
-Due to Igor's comment I will also include another question:
-
-Is there any non-abelian lie group admitting a flat metric that is bi or one-sided invariant?
-
-REPLY [2 votes]: I'm a few years late with my answer, but if you're interested in the pseudo-Riemannian case as well, there is a structure theorem due to Baues for Lie algebras with invariant scalar products, see Theorem 5.15 in
-http://arxiv.org/abs/0809.0824<|endoftext|>
-TITLE: Why is this theorem (about $L(P(\omega_1))^V$ and $L(P(\omega_1))^{V[G]}$) nice?
-QUESTION [9 upvotes]: I was recently told that the following (due to M. Viale) is a nice theorem:
-
-Suppose there are arbitrarily large supercompacts, and $\mathrm{MM}$ holds in $V$. Let $G$ be generic for a proper forcing and $V[G] \vDash \mathrm{MM}$. Then $L(P(\omega_1))^V$ is elementarily equivalent to $L(P(\omega_1))^{V[G]}$.
-
-My question (borne of ignorance, not skepticism) is:
-
-Why is this theorem nice, and how does it fit into the bigger picture?
-
-Some slightly-more-specific questions that refine my main question are: Do the hypotheses of this theorem often come up in natural settings? What's the upshot of the conclusion? Is it that proper forcing which preserves $\mathrm{MM}$, leaves the theory of a small but not-that-small chunk of the universe unchanged?
-
-REPLY [11 votes]: Let me give a brief answer as the author of the mentioned theorem. I shall first say that (as you might imagine) I was very pleased to see that my theorem raised your interest. Now, as mentioned by Joel in his latest post, your statement of the theorem is not correct. However
-there is a typo also in Joel's post, namely the conclusion of the theorem holds for the Chang model $L([Ord]^{<\omega_2})$ and not for the Chang model $L([Ord]^{<\omega_1})$ as posted by Joel.
-For this Chang model the result is due to Woodin and was already known, it appears for example in
-Chapter 3 of Larson's book on the stationary tower forcing. To complement the very good answer Joel has already given you, I invite you to read the many survey papers related to the philosophical position subsumed by this theorem, here is a sample list:
-Woodin's two short papers for the "notices of the AMS" where he exposes the basic ideas behind the generic absoluteness results for $L(\mathbb{R})$:
-
-Woodin, W. Hugh (2001a). "The Continuum Hypothesis, Part I" (PDF). Notices of the AMS 48 (6): 567–576. http://www.ams.org/notices/200106/fea-woodin.pdf.
-Woodin, W. Hugh (2001b). "The Continuum Hypothesis, Part II" (PDF). Notices of the AMS 48 (7): 681–690. http://www.ams.org/notices/200107/fea-woodin.pdf.
-
-Several of Peter Koellner's papers available at his webpage:
-http://www.people.fas.harvard.edu/~koellner/
-Most of them contains a long introductory part which motivates and explains very carefully and plainly the ideas at the heart of the $\Omega$-logic approach to absoluteness result.
-It has to be noted that the kind of solution to the continuum problem prospected by my theorem follows Woodin's view as exposed in the papers on AMS notices. Currently Woodin has pursued a different approach towards the solution of the continuum problem that leads him to prospect a view of the universe (Ultimate $L$) which is radically different from the one given by MM.
-Finally in case you are interested there are in my webpage several slides of talks I gave on this and related subjects, as well as a proof of the theorem you have mentioned in your question:
-http://www2.dm.unito.it/paginepersonali/viale/index.html
-(unfortunately my university is currently changing the websites locations, so for some time you may have troubles to consult it....)<|endoftext|>
-TITLE: The rain hull and the rain ridge
-QUESTION [8 upvotes]: Rain falls steadily on an island, a 2-manifold $M$, which you may
-assume, as you prefer,
-is: (a) smooth, or (b) a PL-manifold, or perhaps even
-(c) a
-triangulated irregular network (TIN).
-After a time, $M$ is saturated, in the sense that every raindrop
-drains into the ocean rather than filling yet-unfilled crevices or basins.
-At this point, we have what I will dub the rain hull of $M$, $H_R(M)$,
-a uni-directional version of the the reflex-free hull,
-explored (by Bill Thurston) in
-this MO question.
-Q1.
-How difficult is to compute the rain hull $H_R(M)$?
-My sense is that it might be quite difficult, because it seems
-there can be nonlocal influences, as crudely depicted
-in this side-view schematic:
-Update (13Aug11): I have corrected the figure to more accurately reflect
-physical reality. Thanks to Oswin Aicholzer for setting me straight.
-
-
-
-
-Perhaps the computation is NP-hard if $M$ is presented as a PL-manifold? TINs have special properties that might
-render the computation polynomial.
-Update.
-Joel Hamkins has convincingly argued (see below) that the computation is polynomial-time.
-Let us assume we have $\overline{M} = H_R(M)$ computed or given.
-A raindrop falling on
-$p \in \overline{M}$ might follow a unique trickle path
-(that is the technical term: e.g., see
-"Implicit Flow Routing on Triangulated Terrains"
-by deBerg et al.)
-to the ocean, or the drop may randomly 'fracture' to follow distinct paths
-to the ocean.
-Define the rain ridge (my terminology) $R(\overline{M})$ to be the complement of the points of $\overline{M}$
-that have a unique trickle path.
-So points on the rain ridge are akin to points on a cut locus, in that
-they have two or more distinct paths to $\partial \overline{M}$.
-They are, in a sense, continental-divide points, a topic
-explored in
-this inadequately
-answered MO question (inadequately answered by me).
-Q2.
-What can be said about the structure of the rain ridge $R(\overline{M})$?
-Unlike the cut locus, it is not always a tree. All the points in a filled basin are in the rain ridge, for when a raindrop lands in a filled basin, it is
-natural to assume it "spreads out" and spills in equal portions over every boundary point of the basin.
-But surely there are substantive properties to investigate. Surely the rain ridge $R(\overline{M})$ cannot be
-an arbitrary subset of $\overline{M}$?
-I finally come to my main question, which I fear has a negative answer:
-Q3.
-Can an extended metric be assigned to $\overline{M}$ so that its
-geodesics are its trickle paths?
-An
-extended metric
-is one that permits $d(x,y) = \infty$
-(e.g., for points not on the same trickle path).
-What I am hoping for here is a way to view the rain ridge as a cut locus
-of $\partial \overline{M}$, and then apply
-a century of knowledge on the cut locus to the rain ridge.
-Partly baked ideas, subquestion observations, and random literature pointers all welcomed!
-My sense is that the considerable applied-math literature on
-watersheds has not approached these questions in their full mathematical generality,
-leaving room for delightful theorems.
-
-REPLY [4 votes]: Since you've explicitly welcomed half-baked ideas...
-Regarding question 1, can't we just simulate the rain in polynomial time?
-I am thinking of the same-connected-component problem in graph theory. Given two nodes $p$ and $q$ in a graph, we determine whether they are in the same component as follows: color $p$ blue, and then make passes through the graph, coloring blue any node that is directly connected to an already-blue node. Repeat until no new blue nodes arise, and then the component of $p$ consists precisely of the blue nodes. This takes polynomial time in the number of vertices, since the number of passes is bounded by the number of vertices.
-In your case, if we imagine a discrete version of the picture, with tiny finite elements and the edge resolution size $n$, that is, an $n\times n\times n$ cube, then can't we simply simulate the rain until the rain hull is saturated? We can follow a single drop along its trickle path, which always has a downward (or at worst horizontal) component by gravity. The complication, as your diagram suggests, are the non-local effects of rain filling up a basin which overspills a border far away, but it seems that this overspill can still be computed in polynomial time. That is, when a drop is added to a local pool, then we allow it to flow horizontally as much as it likes, and we allow it also to flow to nodes at the same altitude, provided there is a blue path from its current location to the new location. Since there are only $n^3$ locations altogether, we can determine whether it is collected or shed in polynomial time.
-And if the trickle path of a single rain drop can be computed in polynomial time of $n$, then we simply repeat the process of adding raindrops, from each of the $n^2$ sources overhead, until no new drops are added permanently to the rain hull. Since we can collect at most $n^3$ drops of rain altogether, we get a polynomial bound on the number of simulated raindrops we need to consider. And therefore in the end we can compute the rain hull in polynomial time of $n$.
-(Perhaps you will object that your intended input will be much smaller than $n$.)<|endoftext|>
-TITLE: What sheaf topoi classify: attribution request
-QUESTION [7 upvotes]: Is there an accepted name or attribution by which to refer to the following well-known theorem?
-
-If C is a small site, then the topos of sheaves on C is the classifying topos for flat cover-preserving functors out of C.
-
-In the case when C has a trivial topology, the corresponding assertion for its presheaf topos is usually called Diaconescu's theorem. But I don't think I have ever seen a name given to the more general version.
-
-REPLY [3 votes]: If the site has finite limits, then this can be found in SGA4 Proposition 4.9.4.<|endoftext|>
-TITLE: Algorithm for solving systems of linear Diophantine inequalities
-QUESTION [8 upvotes]: So, I posted on StackOverflow looking for a reasonably fast algorithm to solve systems of linear Diophantine inequalities and was pointed to this article by Cheng-Zhi Gao and Yu-Lin Dong. The problem is, they give the algorithm on pages 350-351, but part of step (3) appears to be missing.
-My question for MathOverflow is, therefore, whether anyone knows either of another such algorithm or has an idea as to what the missing part of Gao and Dong's algorithm is.
-
-REPLY [6 votes]: GAP provides a function NullspaceIntMat which solves systems
-of linear diophantine equations. The documentation says:
-25.1-2 SolutionIntMat
-
-* SolutionIntMat( mat, vec ) ───────────────────────────────────── operation
-
-If mat is a matrix with integral entries and vec a vector with integral
-entries, this function returns a vector x with integer entries that is a
-solution of the equation x * mat = vec. It returns fail if no such vector
-exists.
-
-──────────────────────────────── Example ─────────────────────────────────
- gap> mat:=[[1,2,7],[4,5,6],[7,8,9],[10,11,19],[5,7,12]];;
- gap> SolutionMat(mat,[95,115,182]);
- [ 47/4, -17/2, 67/4, 0, 0 ]
- gap> SolutionIntMat(mat,[95,115,182]);
- [ 2285, -5854, 4888, -1299, 0 ]
-────────────────────────────────────────────────────────────────────────────
-
-The source code can be found in the file lib/matint.gi included in the GAP distribution.
-There is also a function SolutionNullspaceIntMat which additionally
-computes a basis of the integral nullspace of the given matrix.<|endoftext|>
-TITLE: Titles composed entirely of math symbols
-QUESTION [36 upvotes]: I apologize for burdening MO with such a vapid, nonresearch question, but
-I have been curious ever since
-Suvrit's popular October 2010
-Most memorable titles MO question
-if there were any "$E=mc^2$-titles," as I think of them—how Einstein in retrospect might have entitled his 1905 paper
-(instead of
-"Zur Elektrodynamik bewegter Körper"!)—paper/book titles composed entirely of math symbols.
-There are two close misses in the responses to that MO question:
-Connes et al.'s
-"Fun with $\mathbb{F}_{1}$",
-and Taubes's
-"${\rm GR}={\rm SW}$: Counting curves and connections."
-The only title entirely composed of math symbols with which I'm familiar is the delightful book A=B, by Marko Petkovsek, Herbert Wilf, and Doron Zeilberger.
-Can you identify others?
-Please interpret this question in a weekend-recreational spirit! :-)
-
-REPLY [6 votes]: MIP*=RE
-Zhengfeng Ji, Anand Natarajan, Thomas Vidick, John Wright, Henry Yuen.
-arXiv Abstract. 13 Jan 2020.
-
-...a negative answer to Tsirelson's problem... our results provide a refutation of Connes' embedding conjecture...<|endoftext|>
-TITLE: Doubts on Reproducing Kernel Hilbert Spaces and orthogonal decomposition
-QUESTION [5 upvotes]: I'm a CS student and I'm trying to learn RKHS theory to understand the passages made in this paper .
-Among the bibliography I'm using there are "On the mathematical fundamentals of learning" and "Learning with Kernels".
-I think I've got a good grasp of the relevant theory, but there's still something that bugs me.
-As far as I understood a RKHS is a Hilbert space $H\subseteq \mathbb{C}^X$, where $X$ is a generic set of objects, with inner product $\langle f,g\rangle=\int_X\int_X \alpha(x')\beta(x)k(x,x')dxdx'$ with $\alpha,\beta \in \mathbb{C}^X$, $f=\int_X\alpha(x')k(x,x')dx'$ and $g=\int_X\beta(x')k(x,x')dx'$ such that $H=\overline{\mathrm{span}\{k_x|x\in X\}}$ and $\langle f,k_x\rangle=f(x)$ where $k_x=k(.,x)\;\forall x\in X$ and $k$ is the (Mercer) kernel for $H$. Let's call this "continuos-whole" definition.
-However, given $D=\{x_1,x_2,...x_n\}\subset X$ a subspace $H_D\subset H$ could be defined by restricting the definition above to $H_D=\overline{\mathrm{span}\{k_{x_i}|x_i\in D\}}=\{f\in \mathbb{C}^X|f=\overset{i=s}{\underset{i=1}{\sum}} \alpha_ik(x,x_i)\quad \alpha_i\in \mathbb{C} \;, r\in \mathbb{N}\}$ and thus define $\langle f,g\rangle_{H_D}=\overset{i=s}{\sum_{i=1}}\overset{j=s}{\sum_{j=1}}\alpha_i\beta_jk(x_i,x_j)$. Let this be the "discrete-finite" definition
-Assuming this is correct, the references tend to define $H$ as both being spanned by $\{k_x|x \in X\}$ and having inner product $\langle .,.\rangle_{H_D}$, and that would be possible if and only if $\overline{\mathrm{span}\{k_x|x\in X\}}=\overline{\mathrm{span}\{k_{x_i}|x_i\in D\}}$ which seems bogus to me.
-Then, regarding the article, there's a passage of which I'm unsure. Let $H$ be a RKHS, i.e. a Hilbert space s.t. every point-evaluation functional is bounded, which would entail a "whole-continuous" definition. Then $H$ can be orthogonal decomposed in $H=H_D\oplus H_D^\bot$ where $H_D^\bot \bot H_D \rightarrow \langle f,g\rangle=0 \forall f\in H_D \;\wedge\; g\in H_D^\bot$. The paper says that $H_D^\bot=\{g|g(x_i)=0\forall x_i\in D\}$. Starting from the definition of orthogonality, I carried out the following proof:
-$\forall f \in H_D \;\wedge\; g \in H_D^\top$ $\langle f,g \rangle=\int_X\int_X \alpha(x')\beta(x)k(x,x')dxdx'=\int_X \beta(x)f(x)dx$ $=\sum_{i \in 1..s}\alpha_i\int_X \beta(x)k(x,x_i)dx=\sum_{i \in 1..s} \alpha_i g(x_i)=0 \forall \alpha_i \in \mathbb{C}\rightarrow g(x_i)=0 \forall i\in 1..s$, for which I applied the reproducing property and the definiton of $g$ in this order. is this correct?
-EDIT:I know the definition via the Riesz theorem, however I'm referring to the definiton I found in "Learning with kernels" (pg. 36):
-"Definition 2.9 (Reproducing Kernel Hilbert Space) Let $X$ be a nonempty set and by $H$ a Hilbert space of functions $f :X\rightarrow\mathbb{R}$ . Then $H$ is called
-a reproducing kernel Hilbert space endowed with the dot product $\langle .,. \rangle$ (and the norm $||f|| : \sqrt \langle f ,f\rangle $ ) if there exists afunction $k :X\times X \rightarrow \mathbb{R}$ with the following properties:
-
-$k$ has the reproducing property
-$\langle f, k(x,. )\rangle= f(x)$ for all $f\in H$ ;
-$k$ spans $H$ , i.e. $H=\overline{\mathrm{span}\{k(x,.)| x \in X\}}$ where $\overline{X}$ denotes the completion of the set $X$"
-
-, and, to my understanding, it is based on the theorems characterising RKHS on Mercer kernels (see pg. 35 of "On the mathematical foundations of Learning").
-The approach they take, as far as I've understood, is inside-out: they start from a Lp space $H=\overline{\mathrm{span}\{k(x,.)| x \in X\}}$ with $k$ being a (Mercer) kernel, for which define a suitable inner product to get a Hilbert space that is also a RKHS. They defined a Mercer kernel to be a function $X^2\rightarrow \mathbb{R}$ that gives rise to a definite positive Gram matrix $K_{ij}=k(x_i,x_j)$ for every $D=\{x_1,x_2..x_n\}\subset X$.
-
-REPLY [3 votes]: To answer your second question:
-$H_D = \operatorname{span}(k_x : x \in D) $ is a finite dimensional (and hence closed) subspace of $H$. A function $f \in H$ is orthogonal to each $k_x, x \in D$ precisely when $\langle f , k_x \rangle = f(x) = 0$. In other words, $H_D^\perp = ( f \in H : f(x_i) = 0, x_i \in D)$ and of course $H = H_D \oplus H_D^\perp$.<|endoftext|>
-TITLE: Is there a Serre Tor formula for nonproper intersections?
-QUESTION [14 upvotes]: Background: Let $X$ be a smooth complex projective algebraic variety, and let $V$ and $W$ be closed subvarieties. For simplicity, let's assume that $\dim V+\dim W=\dim X$.
-Now Serre's famous Tor formula says that if $V\cap W$ has dimension zero, we have:
-$$V\cdot W=\sum_{Z\subset V\cap W}\sum_{i=0}^\infty(-1)^i\operatorname{length}_{\mathcal O_{X,z}}\operatorname{Tor}_{\mathcal O_{X,z}}^i(\mathcal O_{X,z}/I_V,\mathcal O_{X,z}/I_W)$$
-where the sum is over the irreducible components of $V\cap W$ (in this case, a finite number of points).
-However (according to Wikipedia here), if $Z$ is an irreducible component of $V\cap W$ with positive dimension (this is called a nonproper intersection), then the alternating sum of $\operatorname{Tor}$'s, which I'll call $\mu(Z;V,W)$, is zero. Unfortunately, this means it cannot be used in exactly the same way as before to calculate $V\cdot W$. For example, if $V=W$, then the answer would always be zero, though certainly there exist half-dimensional varieties $V$ with $V\cdot V\ne 0$ (in fact it can even be negative).
-Is it possible to remedy this situation? How does one count the intersection multiplicity if the intersection is not proper?
-Comment: I know how to compute the self-intersection $V\cdot V$ as the top chern class of the normal bundle evaluated on $[V]$. I'm looking here for an answer that's the most general, i.e. applies to all $V$ and $W$ of complementary dimension, regardless of the dimension of their intersection.
-
-REPLY [11 votes]: Let $ch_i(F)$ denote the $i$-th coefficient of the Chern character of a sheaf $F$. Then
-$$
-V\cdot W = \sum_{i=0}^n (-1)^i ch_n(Tor_i(O_V,O_W)),
-$$
-where $n = \dim X$, $O_V = O_X/I_V$, $O_W = O_X/I_W$, and one uses a natural identification of $H^{2n}(X,{\mathbb Z})$ with ${\mathbb Z}$.<|endoftext|>
-TITLE: Does random matrix theory make any prediction for the eigenvalue distributions of compact Riemann surfaces?
-QUESTION [7 upvotes]: Under RH, Montgomery has proven equidistribution results for the zeros of the Riemann Zeta function, which suggest a close connection of the distribution to certain results in Random matrix theory. Analogues have been proven for zeta function associated finite fields unconditionally.
-The Riemann zeros with imaginary part less than $T$ grow like $T \log T$. The zeros of the Selberg zeta function of a compact surface, which are connected to the eigenvalues of the Laplace Beltrami operator, grow roughly like $T^2$, but here the Riemann hypothesis is true except for possible zeros with imaginary part being zero.
-Nevertheless, I dare to ask, if there is something similar available for these eigenvalue?
-
-REPLY [16 votes]: You can look at the following survey by Peter Sarnak:
-http://www.math.princeton.edu/sarnak/Arithmetic%20Quantum%20Chaos.pdf
-Basically the prediction is that the eigenvalue distribution is Poisson for arithmetic surfaces and GOE for non-arithmetic surfaces. There are some partial results supporting Poisson in the arithmetic case, in particular by Luo and Sarnak.
-This survey has a lot of really cool stuff, but is quite dated by now. For one, it does not include the solution to the quantum unique ergodicity conjecture (which among other things got Lindenstrauss the Fields medal).<|endoftext|>
-TITLE: Splitting a polynomial with one root
-QUESTION [8 upvotes]: Suppose we have an irreducible polynomial $f\in K[x]$. Is there some way to sometimes tell whether $f$ splits completely after adjoining just one root of $f$ to $K$?
-I am mostly interested in the case where $K$ is a function field $\mathbb{F}_{q}(t_{1},\ldots,t_{m})$ over some finite field, so it might not be feasible to explicitly compute roots.
-
-REPLY [5 votes]: Here is the best I can come up with. Consider an algebraic closure $\bar K$ of $K$ and a root $\alpha \in \bar K$.
-The number of roots of $f$ in $K(\alpha)$ doesn't depend on $\alpha$: call it $ r(f)$
-Moreover call $s(f)$ the number of the different subfields $K(\alpha)\subset \bar K$ obtained by adjoining roots of $f$ to $K$. Then you have the pleasant equality $$deg(f)=r(f).s(f)$$
-This shows in particular that the number of roots that you get by just adjoining one root divides the degree $deg(f)$ of your polynomial.
-For example if $K=\mathbb Q$ and $f(x)=X^8-2$ you have $r(f)=2$ and $s(f)=4$, since the fields you get by adjoining roots of $f$ to $\mathbb Q$ are [with $\omega =\frac{1}{\sqrt 2}(1+i)$]:
-$\mathbb Q(\sqrt[4]2)$
-$\mathbb Q(\pm \omega \sqrt[4]2)$
-$\mathbb Q(\pm \bar{\omega} \sqrt[4]2)$
-$\mathbb Q(\pm i \sqrt[4]2)$
-These results are due to Perlis , and although not difficult have found their way in exactly zero books, as far as I am aware.<|endoftext|>
-TITLE: Is every algebraic extension of a field of absolute transcendence degree one a separable extension of a purely inseparable extension?
-QUESTION [13 upvotes]: Any decent course on field theory will state that in characteristic $p$ an extension of fields $k\subset K$ canonically decomposes as the tower $k\subset K_{sep}\subset K$ with
-$K$ purely inseparable over its subfield $K_{sep}$ of elements separable over $k$.
-The alert student will then ask if the order can be reversed, that is whether $K$ is separable over its subfield $K_{perfect}$ of elements purely inseparable over $k$, and be told that the the answer is no.
-However in all the counterexamples I am aware of, the base field is of absolute transcendence degree two: $trdeg_{\mathbb F_p}(k)=2 $ ( usually $k=\mathbb F_p(x,y)$, the purely transcendental field in two indeterminates over $\mathbb F_p$).
-Of course no counterexample can be found with $k$ of absolute transcendence degree zero, because $k$ would then be algebraic over $\mathbb F_p$ and thus perfect.
-I have the suspicion that no counterexample can be found with $k$ of absolute transcendence degree one either and that is what I'd like to ask here:
-Question Let $k$ be a field with $trdeg_{\mathbb F_p}(k)=1$ and $k\subset K$ an algebraic extension. Is the field $K$ separable over its subfield $K_{perfect}$ in the tower $k\subset K_{perfect} \subset K$ ?
-Edit ulrich has solved the question ( with even more general hypotheses!) in the affirmative: congratulations and thank you, ulrich!
-For reference purposes and because much information is in the comments to his answer, I'd like to sum up and display the main points of his subtle proof.
-General result Let $F\subset k$ be an extension of fields with $F$ perfect and $trdeg_{F}(k)=1$.Then for any algebraic extension $k\subset K$ the field $K$ is separable over its subfield $K_{perfect}$ of purely inseparable elements over $k$.
-[My question was the case $F=\mathbb F_p$]
-Core result Let $F\subset E$ be an extension of fields with $F$ perfect and $trdeg_{F}(E)=1$. Then an algebraic extension $E\subset K$ is separable as soon as the only elements of $K$ purely inseparable over $E$ are already in $E$.
-[This clearly follows from the general result and conversely ulrich reduces the general case to this core result by taking $E=K_{perfect}$ . This core result is false for transcendence degree $trdeg_{F}E \geq2$: cf. my answer, lifted from Bourbaki, here ]
-Dimensional lemma If a field $L$ has transcendence degree $1$ over a perfect field $F$, then the purely inseparable extension $L\subset L^{1/p}$ has dimension $1$ or $p$ according as $L$ is perfect or not.
-[Ulrich uses it to prove the core result. He considers the tower $E\subset L=E^{sep} \subset K$ and proves that if $L\neq K$, then we would have $L\subsetneq L^{1/p} \subset K$ thanks to the lemma . From there the contradiction to the hypothesis of the core lemma $E \subsetneq E^{1/p} \subset L^{1/p} \subset K$ obtains. (Notice that non-perfection of $L$ implies that of $E$, hence the asserted strict inclusion $E \subsetneq E^{1/p}$.) This dimensional lemma was proved in a particular case in this link furnished by Martin, and generalized by ulrich. This key technical lemma is exactly the reason why the transcendence degree one hypothesis is needed: the lemma is false for higher transcendence degree.]
-
-REPLY [12 votes]: Yes. One may also replace $\mathbb{F}_p$ by any perfect field $F$.
-The reason is that any non-perfect algebraic extension $L$ of $k$ has a unique inseparable extension of degree $p$, i.e. $L^{1/p}$, the field obtained from $L$ by adjoining all $p$'th roots. This follows from the fact that $L^{1/p}$ is of degree at most $p$ over $L$.
-Assuming this one can prove the claim as follows: Suppose it is not true. Then we can find subfields $L,M$ of $K$ so that we have inclusions $K_{perfect} \subset L \subset M \subset K$ so that $M$ is inseparable of degree $p$ over $L$. By the above remark we must have $M = L^{1/p}$ so we must have $K_{perfect}^{1/p} \subset L^{1/p}\subset K$. If $K_{perfect}^{1/p} = K_{perfect}$, then it is perfect so $L,M$ as above cannot exist. If not, then it contradicts the definition of $K_{perfect}$.<|endoftext|>
-TITLE: (Sh,Sh-map) represents the category of sheaves on a stack.
-QUESTION [5 upvotes]: I'm trying to understand the following theorem, but I don't think I'm reading it correctly.
-Let $(\mathcal{C},J)$ be a site (with a subcanonical topology). Write $\mathcal{C}/X$ for the groupoid of objects over $X\in \mathcal{C}$. Let $\mbox{Sh}:\mathcal{C}^{op} \rightarrow \mbox{Gpds}$ be the functor taking $X$ to the category of sheaves on $\mathcal{C}/X$ and isomorphisms of sheaves, and let $\mbox{Sh-map}:\mathcal{C}^{op} \rightarrow \mbox{Gpds}$ be the functor taking $X$ to the category whose objects are sheaf morphisms $\mathscr{F} \rightarrow \mathscr{G}$ and whose morphisms are commuting squares of sheaves determined by isomorphisms $\mathscr{F}_1 \stackrel{\sim}{\rightarrow} \mathscr{F}_2$ and $\mathscr{G}_1 \stackrel{\sim}{\rightarrow} \mathscr{G}_2$. These are in fact both stacks on $\mathcal{C}$, and moreover they determine a category-object $(\mbox{Sh},\mbox{Sh-map})$ in the category of stacks.
-
-Theorem: The category of sheaves on a stack $\mathscr{M}$ is equivalent to the category of morphisms of stacks $\mathscr{M} \rightarrow (\mbox{Sh,Sh-map})$. That is, the objects are the 1-morphisms and the morphisms are the 2-morphisms.
-
-I'd like to interpret this to mean that the objects of $Shv(\mathscr{M})$ are associated to 1-morphisms $\mathscr{M} \rightarrow \mbox{Sh}$, and that the morphisms of $Shv(\mathscr{M})$ are associated to 2-morphisms in $Hom_{Stacks}(\mathscr{M},\mbox{Sh})$, which in turn should be the same as 1-morphisms $\mathscr{M} \rightarrow \mbox{Sh-map}$. But there a number of problems with this.
-First, given a sheaf $\mathcal{F} \in Shv(\mathscr{M})$ I'm having trouble constructing a natural transformation $\mathscr{M} \rightarrow \mbox{Sh}$. Perhaps I shouldn't, but to check this I'm using a test object $X\in \mathcal{C}$. By Yoneda, an object of $\mathscr{M}(X)$ is the same as a 1-morphism of stacks $f:X\rightarrow \mathscr{M}$, and so I obtain an object of $Sh(X)$ (i.e. a sheaf on $\mathcal{C}/X$) via $(\alpha:Y\rightarrow X) \mapsto \mathcal{F}(f\alpha:Y \rightarrow X \rightarrow \mathscr{M})$. That's natural enough. Again by Yoneda, a morphism in $\mathscr{M}(X)$ is a 2-morphism between maps $f,g:X\rightarrow \mathscr{M}$ of stacks, i.e. a section $s:X\rightarrow X\times_\mathscr{M} X$ of the projection from the 2-category fiber product. Out of this, I'm supposed to construct a natural transformation from the sheaf $(\alpha:Y\rightarrow X) \mapsto \mathcal{F}(f\alpha:Y \rightarrow X \rightarrow \mathscr{M})$ to the sheaf $(\alpha:Y\rightarrow X) \mapsto \mathcal{F}(g\alpha:Y \rightarrow X \rightarrow \mathscr{M})$. But the only structure in place to give me such a thing is a morphism in $Stacks/\mathscr{M}$ between $f\alpha$ and $g\alpha$, and I don't see how to construct this.
-Second, a 2-morphism between 1-morphisms $f,g\in Hom_{Stacks}(\mathscr{M},\mbox{Sh})$ is a section $s:\mathscr{M} \rightarrow \mathscr{M} \times_{\mbox{Sh}} \mathscr{M}$. Thus for any $(\alpha:X\rightarrow \mathscr{M})\in \mathscr{M}(X)$, we get an object $(\alpha,\beta:X \rightarrow \mathscr{M},\varphi:f\alpha \stackrel{\sim}{\rightarrow} g\alpha)\in (\mathscr{M}\times_{\mbox{Sh}}\mathscr{M})(X)$. On the other hand, a 1-morphism $\mathscr{M} \rightarrow \mbox{Sh-map}$ is for each $\alpha:X \rightarrow \mathscr{M}$ an arbitrary morphism on sheaves on $\mathcal{C}/X$. These can't be the same.
-By the way, I've tried to do (what I think is) the right thing and work out the sheaf in $Shv(\mbox{Sh})$ associated to the 1-morphism $\mbox{Id}:\mbox{Sh} \rightarrow \mbox{Sh}$, following Yoneda and all. From the above, it's easy to see what this sheaf should do to morphisms $X\rightarrow \mbox{Sh}$ from a representable stack. But it appears that I need to make choices if I want to say what it does to arbitrary morphisms of stacks $\mathscr{N} \rightarrow \mbox{Sh}$. Perhaps instead I should take a limit or colimit over its application to the full subcategory of representable stacks over $\mathscr{N}$?
-
-REPLY [3 votes]: The notes you are reading seem to disagree with more commonly accepted language (cf. SGA1 Exp 13, Vistoli's notes, or the Stacks project). Some of this seems to be an attempt at expository ease, e.g., the parenthetical remark in example 8.2 ("We will mention the following technical difficulties but will ignore them for now:") where "for now" really means forever. Oddly enough, one of the mentioned technical difficulties is more or less what prevents $\text{Sh}$ and $\text{Sh-map}$ from having natural stack structures in the sense of the notes - pullback is not strictly functorial. This un-naturality is why the common definition of stack is different - the notion of stack in the notes corresponds to the usual notion of stack in groupoids equipped with a splitting (or cleavage).
-The use of the category object $(\text{Sh}, \text{Sh-map})$ is a kludge to replace the usual stack $Sh/\mathcal{C}$ (in categories rather than groupoids) whose objects are sheaves over comma categories, and whose morphisms over any $f: U \to V$ in $\mathcal{C}$ are $f$-maps of sheaves - see Examples 3.20 and 4.11 in Vistoli. The author of the notes employs $\text{Sh-map}$ in order to add non-invertible sheaf maps, because the 2-morphisms in $Hom_{Stacks}(\mathcal{M}, \text{Sh})$ are all invertible. In other words, you have to throw away the 2-morphisms that are given to you by $\text{Sh}$, and use the larger collection of possibly non-invertible two-morphisms afforded by $\text{Sh-map}$.
-Once you have done that, I think your main problems are resolved. You've already worked out the object part of getting from a sheaf on $Stacks/\mathcal{M}$ to a natural transformation from $\mathcal{M}$ to $\text{Sh}$. If you have a morphism $\beta: X \to Z$ in $\mathcal{C}$, and $f: Z \to \mathcal{M}$, then $\beta$ induces a morphism of stacks over $\mathcal{M}$. If I'm not mistaken, the sheaf $\mathcal{F}$ takes this to the map in $\text{Sh}$ given by base change:
-$$\left( (\alpha: Y \mapsto Z) \mapsto \mathcal{F}(f \circ \alpha) \right) \mapsto \left( \beta^* \alpha: Y \times_Z X \to X) \mapsto \mathcal{F}(f \circ \beta \circ \beta^*\alpha) \right)$$
-Similarly, you can get from a sheaf map on $Stacks/\mathcal{M}$ to a natural transformation from $\mathcal{M}$ to $\text{Sh-map}$. There seems to be a lot of additional checking necessary for proving the equivalence, which I don't feel like doing for you (sorry).<|endoftext|>
-TITLE: Class number parity in pure cubic number fields
-QUESTION [20 upvotes]: Consider the family of pure cubic number fields
-$K = {\mathbb Q}(\sqrt[3]{m})$ for $m = a^3 \pm 3$.
-Proposition. If $4 \mid a$ and $m$ is cubefree, then the
-class number of $K$ is even.
-Proof. Let $\omega = \sqrt[3]{m}$; the element $\alpha = a - \omega$
- has norm $\pm 3$. Since $3$ is completely ramified, the element
- $\varepsilon = \alpha^3/3$ is a unit.
-If $m = a^3 + 3$, then $\varepsilon = 1 - 3a^2\omega + 3a\omega^2$.
- If $4 \mid m$, then $\varepsilon \equiv 1 \bmod 4$, hence
- $K(\sqrt{\varepsilon}\,)/K$ is an unramified quadratic extension.
-Experiments seem to suggest that if $m = a^3+3$ and $a \equiv 2 \bmod 4$,
-then $h$ is also even, but there is no explanation, class field theoretic
-or otherwise. In fact, the class number is even for all cubefree values
-of $m$ for $a = 2, 4, \ldots, 2 \cdot 88$, but is odd for $a = 2 \cdot 89$.
-This cannot be an accident; the parity of the class number in the
-case $m = a^3 - 3$ for $a \equiv 2 \bmod 4$ shows a more typical
-(i.e. more random) behaviour in that the class number is odd quite
-often.
-Question: How can this behaviour in the case $m = 8a^3+3$ be explained?
-My first guess would be that, for fields in this family, there is
-a family of ideals ${\mathfrak a}$ such that ${\mathfrak a}^2$ is
-principal, but I can't seem to find anything in this direction.
- Edit. Dror's comment made me look at the family of elliptic curves $y^2 = x^3 - m$.
-These have rank $\ge 1$, and by the parity conjecture rank $\ge 2$. An inequality due
-to Billing now shows that $K$ has even class number. For details, see this
-pdf file.
-Actually, Paul Monsky stumbled across something similar for pure quartic fields;
-see here.
-
-REPLY [5 votes]: Billing (Beiträge zur arithmetischen Theorie der ebenen
-kubischen Kurven vom Geschlecht Eins, R. Soc. Scient. Uppsala (4) 11,
-Nr. 1. Diss. 165 S. Uppsala 1938; see Ian Connell's Handbook for elliptic curves for
-a modern presentation of the result) proved the following result:
-Let $f(x) = x^3 + ax^2 + bx + c \in {\mathbb Z}[x]$ be irreducible,
-and consider the elliptic curve $E: y^2 = f(x)$. Let $K$ be the
-cubic number field generated by a root $\alpha$ of $f$, and let
-$E_K$ be its unit group. Write $({\mathcal O}_K: {\mathbb Z}[\alpha]) =: m_f^2$.
-Then
-$$ r \ \le \ r_2(K) + r_E(K) + 2n_+ + n_-, $$
-where $r$ is the Mordell-Weil-rank of $E({\mathbb Q})$,
-$r_2(K)$ is the $2$-rank of the ideal class group of $K$,
-$r_E(K)$ is the ${\mathbb Z}$-rank of the unit group $E_K$ of $K$,
-$n_+$ is the number of primes $p \mid m_f$ that split in $K$,
-and $n_-$ is the number of primes $p \mid m_f$ that decompose
-as $p {\mathcal O}_K = {\mathfrak p}{\mathfrak p}'$ or as
-$p {\mathcal O}_K = {\mathfrak p}^2 {\mathfrak p}'$.
-If ${\mathbb Q}(\sqrt[3]{m})$ is a pure cubic field with
-$m \not\equiv \pm 1 \bmod 9$ cubefree, then the index is trivial,
-and $n_+ = n_- = 0$ provides us with the bound
-$$ r \ \le \ r_2(K) + 1. $$
-On the other hand, the parity conjecture (see the article by
-Liverance pointed out by Dror) implies that the Mordell-Weil
-rank of $E$ is even for squarefree values of $m = 8b^3 + 3$,
-and the family of nontorsion points
-$$ P_b\Big( \frac{2b^3+1}{b^2}, \frac{3b^3+1}{b^3} \Big) $$
-shows that $r \ge 1$. Thus the parity conjecture implies $r \ge 2$,
-and Billing's bound finally gives $r_2(K) \ge 1$.<|endoftext|>
-TITLE: Simple object in derived category or stable model category?
-QUESTION [8 upvotes]: Exist any common definition of simple objects in derived categories, or even better, in stable model categories?
-I was only able to find definition for abelian categories.
-Thanks.
-
-REPLY [10 votes]: Sometimes an object $E$ of a $k$-linear category $C$ is called simple if $Hom_C(E,E) = k$. This notion is frequently used in derived categories.<|endoftext|>
-TITLE: Jonsson Boolean algebras?
-QUESTION [12 upvotes]: Let us say that a mathematical structure of cardinality $\omega_1$ is Jonsson whenever every one of its proper substructures is countable.
-There are examples of Jonsson groups due to Shelah or Obratzsov. I am almost sure that there is no Jonsson Boolean algebra but I cannot (dis)prove it by hand. Am I right?
-PS. feel free to give any further examples of Jonsson structures or structures which are never Jonsson.
-
-REPLY [4 votes]: Let $A$ be a unital nonzero Boolean algebra of cardinal $\ge 4$. Then $A$ has a unital Boolean subalgebra of index 2. (This excludes being Jónsson at all cardinals.)
-Indeed, $A$ has at least two (unital ring) distinct homomorphisms onto $\mathbf{Z}/2\mathbf{Z}$. This gives rise to a surjective (unital ring) homomorphism onto $(\mathbf{Z}/2\mathbf{Z})^2$. The inverse image of the diagonal yields the desired subalgebra.
-Topologically, this corresponds to taking two points in the Stone space and gluing them.
-
-(Edited Oct 19, 2018)
-Actually, no scalar (=associative unital commutative) ring is Jónsson. Here by Jónsson I mean: uncountable and every proper unital subring has smaller cardinal.
-First, a domain $A$ is never Jónsson (among unital rings). Indeed, we can consider a maximal algebraically free subset $X$ of $A$; since $A$ is uncountable, we have $|X|=|A|$, and hence we can construct, proper unital subrings of the same cardinal as $A$.
-For $p$ prime or zero, say that a scalar ring is $p$-reduced if it embeds into a product of domains of characteristic $p$. Then an uncountable $p$-reduced scalar ring $A$ is never Jónsson. Indeed, write $A\subset\prod A_i$, with $A_i$ domain of characteristic $p$. If some projection has cardinal $|A|$, uncountable, we can pull back some proper subdomain of cardinal $|A|$. Otherwise, each projection has cardinal $<|A|$. If $A$ is a domain, we are done; otherwise, there exists $i,j$ such that the projection on $A_i\times A_j$ is not a domain. Since $A_i$ and $A_j$ have the same characteristic, it follows that this projection is not cyclic, and hence, pulling back the cyclic subring, we obtain a proper unital subring index $<|A|$ in $A$.
-Next, a reduced scalar ring $A$ is never Jónsson. Indeed, let $J$ be the set of prime numbers $p$ that are not invertible in $A$. For every $p$, let $P_p$ be the intersection of all prime ideals of $A$ containing $p$ (so $P_p=A$ for $p\notin J$). Then $A/P_p$ is $p$-reduced; if $|A/P_p|=|A|$ then $A/P_p$ is Jónsson and this contradicts the preceding paragraph. So $|A/P_p|<|A|$; hence the inverse image of its cyclic subring has index $<|A|$; since $A$ is Jónsson, this means that $A/P_p\simeq\mathbf{Z}/p\mathbf{Z}$ for every $p\in J$.
-By the preceding paragraph, $A$ has at most one prime ideal $P_0$ such that $A/P_0$ has characteristic zero. By the case of domains, $|A/P_0|<|A|$. Hence, again by the argument of pulling back cyclic subrings, we see that $A/P_0$ is an infinite cyclic ring. Hence, if $P_0$ exists, it is contained in $P_p$ for every $p$, and in particular equals the intersection of all prime ideals, i.e., the nilradical, so $P_0=\{0\}$ since $A$ is reduced. Since $A$ is uncountable, we get a contradiction: $P_0$ does not exist. Hence the nilradical $\{0\}$ is equal to $\bigcap_{p\in J}P_j$. That is, the diagonal map $A\to\prod_{p\in J}\mathbf{Z}/p\mathbf{Z}$ is injective.
- Consider the composite map $A\to\prod_{p\in J}\mathbf{Z}/p\mathbf{Z}\to (\prod_{p\in J}\mathbf{Z}/p\mathbf{Z})/(\bigoplus_p\mathbf{Z}/p\mathbf{Z})=B$; it has a countable kernel and hence an image of cardinal $|A|$. Then $B$ is a reduced scalar $\mathbf{Q}$-algebra, hence is 0-reduced. Since the image of $A$ in $B$ has cardinal $|A|$, we are done. (Alternative argument for these last few lines: the set of prime ideals of every scalar ring is compact, and it easily follows that if there exists prime ideals such that the quotient have unbounded characteristic, then there exists a prime ideal with quotient of zero characteristic.)
-Now let us deal with $A$ arbitrary uncountable scalar ring; let $R$ be its nilradical. If $A/R$ has cardinal $|A|$, then $A$ is not Jónsson, by the reduced case, and if $A/R$ has cardinal $<|A|$ and is non-cyclic, then it has a proper unital subring of index $<|A|$. So we can suppose that $A/R$ is cyclic.
-If $A/pA$ is has cardinal $|A|$ for some prime $p$, we can pass to $A/pA$. Otherwise $A/pA$ is has cardinal $<|A|$ for all $p$, and hence $A/nA$ has cardinal $<|A|$ for every $n\ge 1$; in particular, $A$ has characteristic zero, so no nonzero element of $\mathbf{Z}1_A$ is nilpotent: thus $A=R\oplus \mathbf{Z}1_A$. Since this is also true when $pA=0$, we henceforth suppose that $A=R\oplus \mathbf{Z}1_A$ ($A$ being of characteristic zero or prime).
-Then observe that for every (non-unital) subalgebra $S$ of $R$, $S\oplus\mathbf{Z}1_A$ is a unital subalgebra of $R$.
-If $R\neq 0$, there exists $x\in R$ with $x^2=0\neq x$. The kernel $K$ and image $I$ of the multiplication-by-$x$ map on $R$ are both ideals of $R$, and this multiplication induces a bijection $R/K\to I$. So either $I$ or $K$ has cardinal $|A|$. Hence both $K\oplus\mathbf{Z}1_A$ and $I\oplus\mathbf{Z}1_A$ are proper unital subalgebras of $A$, and at least one of them has cardinal $|A|$. So $R$ is not Jónsson.
-
-Consequence: if $A$ is an associative unital ring, and is residually of cardinal $<|A|$ (that is, embeddable as unital subring of a product of unital rings of cardinal $<|A|$), then $A$ is not Jónsson.
-(Note that Boolean algebras are residually finite, so this is a generalization.)
-Indeed, the non-existence of any proper unital subring of countable index implies that every countable quotient of $A$ is cyclic; residual countability then implies that $A$ is commutative, and the previous case discards this.
-Of course the argument says more, since it says that every associative unital ring $A$, which is residually of cardinal $<|A|$ (resp. residually countable, resp. residually finite), has a proper unital subalgebra of index $<|A|$ (resp. countable index, resp. finite index), except possibly in the case where $A$ embeds as a unital ring into the quotient of $\widehat{\mathbf{Z}}$ by some closed ideal.<|endoftext|>
-TITLE: Searching for an inhomogeneous diophantine approximation algorithm
-QUESTION [5 upvotes]: Given two nonzero real numbers $x$ and $y$ such that $y/x$ is irrational, a real number $z$ to be approximated, and a tolerance $\epsilon$, what is an algorithm that will provide coprime integers $a$ and $b$ such that $|ax + by - z| < \epsilon$?
-Note that if the restriction that $a$ and $b$ be coprime is lifted, the problem becomes very simple. One possible algorithm is:
-
-Find $a_1$ and $b_1$ such that $0 < a_1 x + b_1 y < \epsilon$ using the extended Euclidean algorithm.
-Let $\displaystyle a = a_1 \left[ \frac{z}{a_1 x + b_1 y} \right]$ and $\displaystyle b = b_1 \left[ \frac{z}{a_1 x + b_1 y} \right],\,$ where $[\cdot]$ is the nearest integer function.
-
-However, the integers $a$ and $b$ provided by this algorithm are usually not coprime. I'm looking for an algorithm that produces the same kind of approximation but guarantees that $a$ and $b$ are coprime.
-
-REPLY [4 votes]: The problem isn't really about the existence of algorithms: The required coprime integers $a$ and $b$ can be found by a systematic search if they exist at all, assuming any reasonable interpretation of the word Given in the first sentence of the question.
-It will simplify matters to divide the inequality in the first sentence by $x$.
-With this in mind, I'll give a proof of the following assertion:
-Let $\epsilon>0$.` Suppose $a$ is irrational and $b$ is any real number. Then there are coprime integers $x$ and $y$ such that $|ax-y-b|<\epsilon$.
-Proof: The proof has undergone a major rewrite, thanks to Gerry Myerson's helpful comments.
-The argument extends a similar result (not mentioning coprimality) proved in Khinchin's book on continued fractions, which is a good reference for the basic facts I'll use here.
-Let $p/q$ be a convergent (to be specified later) of the continued fraction expansion of $a$. Then it is well known (see Khinchin) that $p$ and $q$ are coprime, and moreover that $|a-p/q|<1/q^2$.
-The latter inequality implies that for some real number $\delta$ with $|\delta|<1$,
-$$a=\frac{p}{q}+\frac{\delta}{q^2}.$$
-We will now produce a peculiar-looking estimate for $b$, the reason for which will become apparent shortly. Note that without loss of generality we can and will take $b$ to be positive.
-Let $t$ be the largest prime not larger than $bq$. Then by Bertrand's Postulate $t\le bq<2t$. From this we deduce the following chain of inequalities:
- $$t/q\le b<2t/q\le t/q+b.$$
- It follows that for some $\gamma$ with $0\le \gamma
-TITLE: How to show that an ind-scheme is not a scheme?
-QUESTION [6 upvotes]: A standard example of an ind-scheme over a field $\mathrm{k}$ which is not a
-$\mathrm{k}$-scheme is $\mathrm{k}((\varepsilon))$.
-My question is how to prove that rigorously? To put it more precisely,
-let $$\mathrm{k}((\varepsilon)) = \{ a \in \prod_{-\infty}^{\infty}\mathrm{k}:
-a_i =0, i \ll 0 \}$$
-An ind-scheme is an injective limit of schemes. So here,
-$$\mathrm{k}((\varepsilon)) =
-\lim_{i \rightarrow -\infty}\varepsilon^i\mathrm{k}[[\varepsilon]]$$
-But why isn't it an algebraic subset of $\prod_{-\infty}^{\infty}\mathrm{k}$?
-EDIT: I seem to have mixed up some notions, and have asked two different questions at once (or maybe even three) so I'll try to make myself clear.
-My motivation for the question was to be able to justify the following
-"$k((\epsilon))$ is not an algebraic subset of $\prod_{-\infty}^{\infty}k$ so
-we define it as $k$-points of an ind-scheme". So the original question is:
-why $k((\epsilon)) \subseteq \prod_{-\infty}^{\infty}k$ isn't algebraic
-and it is answered by Jason Starr (though I'm not sure if I understand the answer).
-We can also define $k((\epsilon))$ as and ind-scheme by
-$$k((\epsilon)) = colim_n Spec(k[x_{-n},x_{-n+1}, \ldots])$$
-Now, one can ask, why isn't the ind-scheme we've constructed a scheme after all
-(btw. wouldn't it contradict Jason's argument?), and
-this question is answered by Scott Carnahan below. Finally, there is a question: if the co-limit exists in the category of schemes which Scott Carnahan addresses below as well ...
-
-REPLY [3 votes]: Based on the comments, it looks like you have two questions mixed up. There is the question of whether the colimit exists in the category of schemes, and there is the question of whether the ind-scheme described by the colimit in the category of set-valued functors on schemes (or the category of Zariski sheaves of sets, or fpqc sheaves, etc.) is represented by a scheme. The two questions are quite different. Since you mentioned ind-schemes, I'll answer the (easier) question about ind-schemes.
-Following Martin's comment, I'm assuming by $k((\epsilon))$, you are referring to $\operatorname{colim}_n \operatorname{Spec}(k[x_{-n},x_{-n+1},\ldots])$ where the colimit is taken in set-valued functors on schemes. It is an ind-scheme in the sense that it is a colimit over a directed system of closed immersions of schemes. This particular ind-scheme is ind-affine, so it can be written as the formal spectrum of the topological ring $A = \varprojlim_n k[x_{-n},x_{-n+1},\ldots]$ (see Beilinson and Drinfeld's Quantization of Hitchin’s integrable system and Hecke eigensheaves section 7.11.2). Since $A$ has a non-discrete localization at zero, and all local rings of schemes at points are discrete, the locally topologically ringed space $\operatorname{Spf} A$ is not isomorphic to a scheme.
-I think the directed system $\{ \operatorname{Spec}(k[x_{-n},x_{-n+1},\ldots]) \}_{n \geq 0}$ has a colimit in schemes, given by $\operatorname{Spec} A$, where $A$ is given the discrete topology. $\operatorname{Spec} A$ is certainly the colimit in affine schemes, by the anti-equivalence between affine schemes and commutative rings. Surely there must be a theorem somewhere ...?<|endoftext|>
-TITLE: Help motivating log-structures
-QUESTION [31 upvotes]: I'm currently reading a thesis that uses log-structures. I should mention that this is my first encounter with them, and the thesis (as well as my expertise) is scheme-theoretic (in fact stack-theoretic) and so the original geometric motivations are lost on me.
-Here is my meek understanding. For any scheme, we can give a log-structure. This is a sheaf , $M$, fibered in monoids, on the etale site over a scheme $S$; together with a morphism of sheaves fibered in moinoids $\alpha:M\rightarrow O_S$ such that when it is restricted to $\alpha^{-1}(O_S^{\times})$ it is an isomorphism.
-This $\alpha$ is called the exponential map, and for any $t\in O_S(U)$ (for some $U$), a preimage of it via $\alpha$ is called $log(t)$($\in M(U)$).
-I am curious about a few things, and puzzled about others. First, in terms of the notation, surely it's no coincidence that these are called exponential maps and log-structures. What is the geometric motivation for it?
-Second, these come up in the thesis I'm reading in the context of tame covers. I am puzzled about what, precisely, log-structures contribute. It seems to me, in extremely vague terms (commensurate with my understanding), that the point of log-structures in this context is that if you add this extra information to tame covers it somehow helps you construct proper moduli spaces of covers.
-On top of everything I'm also confused about the role of `minimal log-structures' in all of this.
-In conclusion, if you can say anything at all about the motivations of log-structures in the geometric setting, or more importantly in the context of tame covers, I would extremely appreciate it. The plethora of notationally different texts on the subject is making it hard to understand the gist of what's going on.
-Also, if you have examples that I should have in mind when thinking about it, that would be ideal.
-
-REPLY [3 votes]: I believe that people refer to $\alpha$ as the exponential map exactly because of the "big example (I)" in Pottharst's notes. Also, a small point, for any scheme we can give many log structures. For instance, in addition to the main example of a log str associated to a closed immersion, the simplest examples are $M = \mathcal O_X^*$ and $M = \mathcal O_X.$ I found K. Kato's paper "Logarithmic structures of Fontaine-Illusie" to be a good introduction.
-Logarithmic structures of Fontaine-Illusie. Algebraic analysis, geometry, and number theory (Baltimore, MD, 1988), 191–224, Johns Hopkins Univ. Press, Baltimore, MD, 1989.<|endoftext|>
-TITLE: Why separating tangent vectors?
-QUESTION [10 upvotes]: Let $X$ be a projective scheme over an algebraically closed field.
-It is a well-known theorem that a line bundle $L$ defines a closed embedding into projective space if and only if it separates points and tangent vectors, which means:
-(1) For any distinct closed points $P, Q \in X$ there is an $s \in \Gamma(X,L)$ with the property that $s\in m_PL_P$ but $s \not\in m_QL_Q$.
-(2) For every closed point $P \in X$ the set $\{s \in \Gamma(X,L)|s_P \in m_PL_P\}$ spans the vector space $m_PL_P/m_P^2L_P$.
-Now in Hartshorne (Prop. IV 3.4.) this criterion is applied in the following situation: One wants to show that every curve $C$ can be embedded into $\mathbb{P}^3$. First one embedds the curve $C$ into an arbitrary $\mathbb{P}^n$. Then one chooses some $O \in \mathbb{P}^n$ andprojects the curve down from $O$ into $\mathbb{P}^{n-1}$.
-Then one wants to show that this projection map is a closed immersion if and only if
-(a) $O$ does not lie on any secant of $C$
-(b) $O$ does not lie on any tangent of $C$.
-At this point Hartshorne applies the above criterion. But at this point I do not understand why the fact that $O$ does not lie on any tangent implies $(2)$ of the above criterion. I understood why $(a)$ implies $(1)$ but why does $(b)$ yield that the line bundle separates tangent vectors
-
-REPLY [6 votes]: [I assume $C$ is smooth]
-The point is this: $m_PL_P/m_P^2L_P$ is $1$-dimensional and hence generating it is equivalent to finding a single element that is not zero. The statement that all sections that are in $m_PL_P$ are also in $m_P^2L_P$ is equivalent with the statement that every member of the corresponding linear system that contains $P$, also contains the tangent line to $C$ at $P$. I think you should be able to finish from here.
-[A modification of this idea actually works in higher dimensions, so the first sentence is not really necessary, but makes it easier in the curve case.]<|endoftext|>
-TITLE: Conic hulls and cones
-QUESTION [7 upvotes]: Suppose I have a number of vectors in $\mathbb{R}^n.$ The first question is: what is the most efficient algorithm to compute their "conic hull" (the minimal convex cone which contains them)? The next question is: suppose I have a number of vectors $v_1, \dotsc, v_n,$ as before, and a convex cone $C.$ I want to find the conic hull of $\{v_1, \dotsc, v_n\} \cup C.$ In case it matters, in my application $C$ is the semidefinite cone. By "compute the conic hull", I mean: I want to find the subset of the $v_i$ on the boundary of the hull.
-EDIT Thanks for all the comments. It is certainly true that the conic hull is equivalent to the intersection with a plane, and as @Will pointed out, the only problem is finding the plane. In the PSD case, we know that identity is PSD, so this gives us a choice of planes.
-As for the algorithm, I had come up with @Matus' algorithm, but was not sure (and still am not) that this is the most efficient, since it looks like there is a lot of recomputation. The fact that the PSD cone is not a polyhedral cone is very true. Notice that you can still ask for the extremal points from the original set, and in fact, the same algorithm works, except that instead of solving a linear program at each step, we need to solve a semidefinite program, which hurts a bit, but is certainly tractable for small dimension.
-If you ask for the full convex hull, I am not at all sure of how the answer should even look like, since one will need to describe the "exposed" pieces of the cone. Surely mankind has wondered about this is in the context of, eg, the convex hull of a collection of disks in the plane, or some such.
-
-REPLY [6 votes]: [Edited: previous version was flaky, sorry; also edited per Matus]
-There is a variant of Matus's approach that takes $O(nT_A)$ work, where $A\le n$ is the size of the answer, that is, the number of extreme points, and $T_A$ is the work to solve an LP (or here an SDP) as Matus describes, but for $A+1$ points instead of $n$.
-The algorithm is: (after converting from conic to convex hull) maintain an output set $S$, that starts empty, and test each point $v_i$ against $S$ one by one. Solve the LP (or SDP) as Matus describes. If $v_i$ is proven to be in the convex hull of $S$, discard it. Otherwise, the dual certificate gives a direction (at least in the LP case, and something similar should apply in the SDP case) perpendicular to that separating hyperplane, such that the input point that is extreme in that direction is not already in $S$. (While $v_i$ is not in the convex hull of $S$, it may not be extreme itself.) Find that input point and add it to $S$.
-Testing each of the $n$ points costs $T_A$, and the $O(n)$ work for finding an extreme point in a given direction yields a new member of $S$, so such tasks need $O(nA)\le O(nT_A)$ work.
-This trick and related ones appeared here ("More output-sensitive..."); the notes for the paper give pointers to some related work.<|endoftext|>
-TITLE: Volume of fundamental domain and Haar measure
-QUESTION [14 upvotes]: In my research, I do need to know the Haar measure. I have spent some time on this subject, understanding theoretical part of the Haar measure, i.e existence and uniqueness, Haar measure on quotient. But I should confess I never felt confident with the Haar measure, essentially because theoretical part did not give me how to construct the Haar measure. Even when I was trying to understand Haar measure on $SL(n,\mathbb{R})$ via Iwasawa decomposition, I could not realize exactly how it works.
-I think, or I should say I fell, I am not the only one how has problem with this delicate subject. So I thought I would want to share this with you and see how people think about the Haar measure and how you compute, for instance, the volume $${\rm Vol}(SL(n,\mathbb{Z})\backslash SL(n,\mathbb{R}))$$
-
-REPLY [8 votes]: Supplementing the other answer (too long for a comment...) First, for examples of innocent-context computations, there is an on-line computation of volumes of $SL(n,\mathbb Z)\backslash SL(n,\mathbb R)$ and of $Sp(n,\mathbb Z)\backslash Sp(n,\mathbb R)$
- here , written in essentially Siegel's style. The same style of computation can be done adelically, over arbitrary number fields, but still does effectively beg the question of normalization. Nevertheless, such computations show that the normalization can be determined inductively, and, in any case, that the global computation can be done nicely once we have a locally-everywhere normalization of measures.
-Yet-another approach is (after Langlands) to look at suitable residues of Eisenstein series (e.g., as in the Boulder conference, AMS Proc Symp IX). In effect, such a computation back-handedly normalizes the Haar measure...<|endoftext|>
-TITLE: Large cardinals and constructible universe
-QUESTION [8 upvotes]: We know that if $V=L$ holds, then $|\cal{P}(\omega)|=|\cal{P}(\omega)\cap \textrm{L}|=\aleph_1$ whereas, in the presence of a measurable cardinal (in fact, even Ramsey) $|\cal{P}(\omega)\cap \textrm{L}|=\aleph_0$. I remark that the cardinalities are of course computed in (the corresponding) $V$.
-The first is just the fact that the constructible universe satisfies CH, while the second has to do with the fact that in the presence of a measurable, $\omega_1^{L}<\omega_1$ i.e. the existence of large cardinals makes the relative $\omega_1^{L}$ "drop" below its "maximum possible" value (which is attained, if you want, in the "extreme case" when $V=L$).
-My question is, what can we say, in general, about the beaviour of $\omega_1^{L}$ given axioms of increasing strength above (or equal to, in strength) $V\neq L$? In particular, what happens if we just assume $V\neq L$?
-
-REPLY [14 votes]: Each of the following implies that (the true) $\omega_1$ is inaccessible in $L$, and hence that there are only countably many constructible reals:
-
-The proper forcing axiom
-There is a Ramsey cardinal
-$0^\#$ exists
-All projective sets are Lebesgue measurable
-All $\Sigma^1_3$-sets are Lebesgue measurable
-
-(EDIT: These are just some of the well-known examples that came to my mind. This list is neither exhaustive nor canonical.)
-The mere existence of a nonconstructible set, or even a nonconstructible real, does not imply that $\omega_1^L$ is countable. There are many forcing notions in $L$ which do not collapse $\omega_1$: adding one or many Cohen reals, destroying Souslin trees, etc. Each such forcing (over L) results in a model where $\omega_1=\omega_1^L$.
-In fact, "Martin's axiom plus continuum is arbitrarily large" is consistent with $\omega_1^L=\omega_1$. (But also with $\omega_1^L<\omega_1$.)
-ADDED: Preserving $\aleph_1$ of the ground model (which may or may not be the constructible universe $L$) is a key component in many independence proofs concerned with the theory of the reals. The
-"countable chain condition", which is enjoyed by all the forcings I mentioned above, is a property of forcing notions that guarantees preservation of $\aleph_1$; there are several other (weaker) properties which also suffice, most prominently (Baumgartner's) "Axiom A" and (Shelah's) "properness".<|endoftext|>
-TITLE: Motivation behind Kac's notation for affine root systems
-QUESTION [11 upvotes]: I'm reading Kac's Infinite Dimensional Lie Algebras. In Chapter 4, he classifies the affine root systems. Bourbaki classified the affine Coxeter groups, but multiple root systems can give the same Coxeter group. Kac denotes the elements of his classification by $X^{(r)}$, where $X$ is a symbol denoting a finite root system (e.g. $G_2$) and $r$ is $1$, $2$ or $3$.
-When $r=1$, everything makes sense to me. $X^{(r)}$ is a root system which gives rise to the affine Coxeter group which Bourbaki would consider to be of type $X$.
-When $r$ is $2$ or $3$, I believe I understand the object Kac is defining. However, I do not understand how he chooses its name. For example, look at the root system he calls $A_{2 \ell-1}^{(2)}$. The root system is of rank $\ell+1$, not $(2 \ell-1)+1$ as you would expect. The corresponding Coxeter group is the affine Coxeter group of type $B_{\ell}$. If you look at Macdonald's very readable paper Affine root systems and Dedkind's $\eta$-function, Macdonald denotes this root system by $B_{\ell}^{\vee}$, which makes much more sense to me.
-Similar issues apply to every entry in table 'Aff 2'.
-I have two questions:
-
-Why does Kac choose the notation he does?
-
-Also,
-
-Is Kac's notation so established that I have to use it? Is there a mainstream alternative? I find Macdonald's much more reasonable, but I'm pretty sure that hasn't caught on.
-
-REPLY [13 votes]: I think the notation might be explained by the explicit construction of the twisted affine Lie algebras as fixed points of automorphisms of the untwisted ones: the $r$ indicates the order of the chosen automorphism of the extended Dynkin diagram corresponding to $X$ and twised affine Lie algebra is a subalgebra of the affine Lie algebra corresponding to $X$. See Chapter 8 of Kac's book.
-This notation is fairly well established, given that Kac's book is the standard reference, but I'm not an expert.<|endoftext|>
-TITLE: Maximize the intersection of a n-dimensional sphere and an ellipsoid.
-QUESTION [5 upvotes]: I have the conjecture that the volume of the intersection between an $n$-dim sphere (of radius $r$) and an ellipsoid (with one semi-axis larger than $r$) is maximized when the two are concentric, but still did not find a way to prove it. Any suggestion?
-
-REPLY [8 votes]: By a rotation, you may assume that the axes of the ellipsoid are parallel to the coordinate axes. Choose the first coordinate of the center of the ellipsoid, and translate the ellipsoid so that this coordinate becomes zero, keeping the other center coordinates fixed (this corresponds to symmetrizing about this axis). The volume of the intersection increases when you do this, since the intersection of an interval of fixed length with an interval centered around the origin is maximal when the interval is centered about the origin too (this is the 1-dimensional case of your question). Repeat with the other coordinates, until the ellipsoid is centered at the origin, and the volume of intersection is maximal.<|endoftext|>
-TITLE: Gale-Shapley stable marriage theorem: can we entrust matchmaking to monkeys?
-QUESTION [13 upvotes]: Disclaimer: This is a question I have not done any real research about. I asked it myself some 5 years ago, and back then I had no idea where to start. Now I have some texts on stable matchings lying around, but from a quick look they don't seem to answer this.
-We have $n$ ladies $L_1$, $L_2$, ..., $L_n$ and $n$ gentlemen $G_1$, $G_2$, ..., $G_n$. Each lady ranks all gentlemen in order of preferability (no ties are allowed), and each gentleman does the same to the ladies. A stable marriage means a permutation $\sigma \in S_n$ such that there are no $j\in\left\lbrace 1,2,...,n\right\rbrace$ and $k\in\left\lbrace 1,2,...,n\right\rbrace$ for which $L_j$ prefers $G_k$ to $G_{\sigma\left(j\right)}$ whereas $G_k$ prefers $L_j$ to $L_{\sigma^{-1}\left(k\right)}$.
-Okay, I should have said that it is a matching where we cannot find a lady and a gentlemen which prefer each other to their respective matching partners. But is it combinatorics if there are no symmetric groups in it?...
-Anyway, this is known to have a simple (but very hard to find) algorithmic proof. What I am wondering is whether the following stupid algorithm can also be forced to terminate:
-We choose some arbitrary matching between the ladies and the gentlemen. Then, at each step, we randomly pick a pair that prefers each other to their respective partners, and marry them to each other, simultaneously marrying their respective partners to each other (no matter what they think about it). Repeat until no such steps are possible anymore.
-(1) Can this "algorithm" loop endlessly if we choose our pairs in a stupid enough way?
-(2) Can we make this algorithm terminate by giving a reasonable choice tactic for the pairs?
-
-REPLY [12 votes]: For (1), yes, it can loop endlessly. Here's a demonstration with 8 people.
-Suppose you have 3 ladies and 3 gentlemen standing in a circle, along with one more lady and gentleman (the pariahs) inside the circle. Here are the preferences you need to know about: everyone around the circle prefers to person to his or her right over the person to his or her left, and prefers either of them to the oppositely-sexed pariah in the center. (The compatibility of people diagonally opposite each other and the opinions of those in the center won't matter for the purposes of this solution.)
-Initially, choose a pair of diagonally opposite people and pair each of them up with the appropriate pariahs in the center. Pair up the remaining four people around the circle in adjacent pairs. This is unstable: those matched with pariahs would prefer to be matched with the neighbor to the left, and the feelings are reciprocated (since everyone around the circle prefers right over left).
-Performing both of these swaps, however, only serves to rotate the whole picture 120° while preserving the symmetry of the preferences we care about. Do it two more times and you've got the original matching again.
-
-REPLY [11 votes]: Regarding (2), the answer is still "no". The following counter-example is from:
-
-Tamura, Akihisa Transformation from
- arbitrary matchings to stable
- matchings J. Combin. Theory Ser. A
- 62 (1993), no. 2, 310–323
-
-Consider $n$ men and $n$ women. With indices periodic modulo $n$, the first four choices of each person are:
-For $m_i$: First choice is $w_i$, then $w_{i-2}$, then $w_{i+1}$, then $w_{i-1}$.
-For $w_i$: First choice is $m_{i+1}$, then $m_{i-1}$, then $m_i$, then $m_{i+2}$.
-Start with the matching that pairs $m_i$ to $w_i$ for $1 \leq i \leq n-2$, pairs $m_{n-1}$ and $w_n$ and $m_n$ and $w_{n-1}$. I leave it as an exercise that there is only one unstable pair, swapping them leaves a situation where there is only one unstable pair, and swapping those brings you back to the original situation with indices shifted.<|endoftext|>
-TITLE: Limits of reduced schemes question from Eisenbud and Harris
-QUESTION [6 upvotes]: My question pertains to exercise II-16 in Eisenbud and Harris' "The geometry of Schemes". For an algebraically closed field $K$ the question is as follows:
-
-Consider zero-dimensional subschemes $\Gamma \subset \mathbb{A}_K^4$ of degree 21 such that $$V(m^3)\subset\Gamma \subset V(m^4)$$
- where $m$ is the maximal ideal of the origin in $\mathbb{A}_K^4$. Show that there is an 84-dimensional family of such subschemes, and conclude that in general one is not a limit of a reduced scheme.
-
-What does it mean for a family of subschemes to have dimension 84? I can only think it means up to isomorphism there are 84 such subschemes, but this doesn't seem to work.
-
-REPLY [8 votes]: Such a subscheme is given by an ideal $I$ such that $m^4 \subset I \subset m^3$. In fact, any vector subspace in $m^3$ containing $m^4$ is an ideal (this is a simple exercise). Since $\dim O/m^3 = 15$ and $\dim O/m^4 = 35$, and we are interested in subspaces $I \subset O$ such that $\dim O/I = 21$, that is in subspaces $I/m^4$ of dimension $35 - 21 = 14$ of the space $m^3/m^4$ of dimension $35 - 15 = 20$. So, the family in question is just the Grassmannian $Gr(14,20)$. Its dimension is $14(20-14) = 14\cdot 6 = 84$.<|endoftext|>
-TITLE: Topology on extensions of topological groups
-QUESTION [7 upvotes]: Let $G$ and $H$ be two topological groups and let $\mathcal{E}:0 \to G \to E \to H \to 0$ be an extension of abstract groups.
-Is there a way to introduce a topology on $E$ such that $\mathcal{E}$ becomes an extension of topological groups? If there is a way, is it unique?
-Similarly, let $G$ and $H$ be Lie groups and let $\mathcal{E}:0 \to G \to E \to H \to 0$ be an extension of topological groups.
-Is there a way to introduce a smooth structure on $E$ such that $\mathcal{E}$ becomes an extension of Lie groups? If there is a way, is it unique?
-Thank you all in advance.
-
-REPLY [2 votes]: You can describe abstract group extensions of H with G by 2-cocycles of group cohomology.
-If you have an extension E, you get an induced H-operation on G, by conjugating in E (take any set-theoretic section of $E\to G$). The extensions of G with H with this H-operation on G are classified up to isomorphism, by the second group cohomology. You get a 2-cocycle corresponding to E by taking any set-theoretic section $s : G\to E$ of $E\to G$ that maps 1 to 1 and write down the 2-cocycle $c : H \times H \to G$ by $c(h,h'):=s(h)s(h')s(hh')^{-1}$.
-Then equip the set $G\times H$ with the multiplication $(g,h)(g',h') := (g+h.g'+c(h,h'),hh')$. More explicitly, this is $(g,h)(g',h') = (g+s(h)g's(h)^{-1}+s(h)s(h')s(hh')^{-1},hh')$.
-This is again a group extension and it is isomorphic to E.
-One can show that all extensions are of the type I just constructed for a given cocycle, up to isomorphism.
-The extensions in the same isomorphism class differ only by a coboundary.
-A good reference would be Weibel's homological algebra book.
-To have an extension with a topological group structure implies that the corresponding cocycle is continuous and in general, this can not be expected. Observe that continuity doesn't follow from the axioms for 2-cocycles and depends on the topological structures of G and H, which you don't want to change.
-So I think, it's wrong in general as well as for central extensions, which would be the case of a trivial H-action on G, where you still don't get any continuity for free. At the same time, I don't know of any trivial counter-example. You might take any non-continuous map from the real numbers times real numbers to the real numbers and form the corresponding "twisted" semi-direct product as sketched above. Then you can not get a topological group structure on the extension such that the extension is in the category of topological groups.
-As for the smooth case, the same idea applies, where one would need the cocycle to be smooth as well.<|endoftext|>
-TITLE: Applications of full integral weight modular forms in elementary number theory
-QUESTION [10 upvotes]: Except for Eisenstein series having the divisor functions as their Fourier coefficients, is there any other full integral weight modular form (of some level, preferably full) having arithmetic functions as their Fourier coefficients.
-More to the point, my question is, apart from the relations you obtain between $\sigma_3, \sigma_5$ and $\sigma_7$ are there any applications of full integral weight modular forms (preferably cusp forms) to elementary number theory.
-
-REPLY [2 votes]: Nobody seems to have mentioned "the master" of this subject, and his use of (classical) Eisenstein series to prove things like $p(5n+4) \equiv 0 \ ({\rm mod} \ 5)$ (here $p(m)$ is of course the usual partition function).
-Here's his proof (prepared by Hardy), published in Math.Z (1921). B.Berndt published another a bit shorter proof , which employs famous Ramanujan's differential equations. BTW, make sure you are familiar with the Ramanujan "J-series" before you jump to (say) formula (2.2) in Berndt's paper :-)<|endoftext|>
-TITLE: What is the fan of the toric blow-up of $\mathbb{P}^3$ along the union of two intersecting lines?
-QUESTION [5 upvotes]: Is there a good way to find the fan and polytope of the blow-up of $\mathbb{P}^3$ along the union of two invariant intersecting lines?
-Everything I find in the literature is for blow-ups along smooth invariant centers.
-Thanks!
-
-REPLY [3 votes]: Since you already know how to blow up along either ${\mathbb P}^1$ individually, we can concentrate on what's happening nearby the intersection. Which means we can work affinely.
-Then the polyhedron for ${\mathbb A}^3$ is the octant $({\mathbb R}_{\geq 0})^3$, and your two lines correspond to two of the three edges.
-To blow them up, take a carpenter's plane to those two edges, and shave them off exactly the same amount. The result will have an edge connecting two vertices, one of degree 3, one of degree 4 (the isolated singularity on the blowup).
-In coordinates, the polyhedron is {$(x,y,z) : x,y,z \geq 0, x+y \geq 1, x+z \geq 1$}.
-If you want the blowup of ${\mathbb P}^3$ not ${\mathbb A}^3$, also impose $x+y+z \leq N$ for some large $N$ (three will do).<|endoftext|>
-TITLE: Distributions and measures
-QUESTION [6 upvotes]: Hello,
-After reading the previous post, I still have some doubts. Let's consider everything on $R$ to avoid complications.
-
-Can we say that any distribution $\mu\in\mathcal{D}'(R)$ of zero order is a signed radon measure?
-Since $\mu\in\mathcal{D}'(R)$ which is non-negative on non-negative test functions $C_c^\infty(R)$ is a positive radon measure, it is natural to ask what is corresponding part for the Schwartz distribution $\mu\in\mathcal{S}'(R)$ which is non-negative on non-negative test functions $\mathcal{S}(R)$? Intuitively, it is the radon measure whose mass grows slowly at infinity. Is there a name for this measure?
-For a radon measure $\mu$ on $R$, can we apply the Lebesgue's decomposition locally with a compact set fixed (say $K=[-a,a]$)? Then $\mu_K=\mu_{ac}+\mu_{sc}+\mu_{pp}$. The absolutely continuous part $\mu_{ac}$ corresponds to an absolutely continuous function; the pure point part $\mu_{pp}$ corresponds to sum of delta functions. How about the singular continuous part $\mu_{sc}$?
-EDIT: This question can also be put in the following way: Let $f$ be a singular function, what can we say $\int f\psi d x$ with $\psi\in\mathcal{D}(R)$?
-
-Thank you for your help! :-)
-Best
-Anand
-
-REPLY [6 votes]: About the second question:
-The Schwartz space $\mathcal{S}(R)$ is a Frechet space, i.e. it's topology is given by a countable sequence of seminorms. Any Schwartz distribution (continuous linear functional on $\mathcal{S}(R)$) has to bounded by one of these seminorms. If the Schwartz distribution is of "order zero" then the only relevant seminorms are $\|f\|_n = \sup (1+|x|)^n |f(x)|$. If $T\in \mathcal{S}'(R)$ is bounded by $\|\ldots\|_n$, then this means $T$ is of the form $\langle T, f\rangle = \int (1+|x|)^{n} f(x) \mu(dx)$ for some finite measure on $R$.
-So I would think measures which are Schwartz distributions are measures "of polynomial growth."<|endoftext|>
-TITLE: A Question on Koszul duality and $B(\infty)$ structures on $HH^*$
-QUESTION [5 upvotes]: The following theorem is known from a paper "Duality in Gerstenhaber Algebras" by Felix, Menichi, Thomas. Given a simply connected space X of finite type.
-There is an equivalence of Gerstenhaber algebras
-$HH^*(C_*(\Omega X,\mathbb{Q}), C_*(\Omega X,\mathbb{Q}) \cong HH^*(C^*(X,\mathbb{Q}),C^*(X,\mathbb{Q})$
-On the left hand side we have Pontryagin product on the based loop space and on the right hand side rational cochains. $HH^*$ denotes Hochschild cohomology.
-I have never seen anyone speak to the following enhanced statement, which makes me wonder if there is a counterexample or if I am simply missing some literature.
-$HCH^*(C_*(\Omega X), C_*(\Omega X) \cong HCH^*(C^*(X),C^*(X))$
-The question is: Is this statement true, false or unknown?
-Here we are looking at Hochschild cochains in the homotopy category of $B(\infty)$ algebras. For the background police, a $B(\infty)$ algebra is a type of dg-Gerstenhaber structure, that naturally gives rise to a Gerstenhaber structure by passing to homology. For more info, see the paper of Keller mentioned below.
-It is possible to prove this theorem when $C^*(X)$ is equivalent to a graded simply connected Koszul algebra( i.e. X is both formal and coformal). I believe this is due to Keller in a paper called the "Derived Invariance of Higher Structures of the Hochschild complex".
-
-REPLY [2 votes]: Looking closer at Keller's paper, the result seems to be in there. Namely, in his main theorem in section 3.3, he proves that fully faithful dg-functors $per(A) \to D(B)$ induced by an $A\otimes B^{op}$ module X induce $B(\infty)$ morphisms $\phi_X: HCH^*(B,B) \to HCH^*(A,A)$. Additionally, in the same theorem, he proves that if the map $per(B^{op}) \to D(A^{op})$ induced by X is also fully faithful, then $\phi_X$ is invertible.
-These criterion all apply to M a simply connected space, $X= \mathbb{Q}$ the trivial local system, $A=C_*(\Omega(M))$, and $B= C^*(M)$.<|endoftext|>
-TITLE: Matrix expression for elements of $SO(3)$
-QUESTION [10 upvotes]: Hi all. Is there any explicit matrix expression for a general element of the special orthogonal group $SO(3)$? I have been searching texts and net both, but could not find it. Kindly provide any references.
-
-REPLY [14 votes]: There is a good way to derive the sort of thing you're looking for: use the double cover $SU(2) \to SO(3)$. $SU(2)$ is diffeomorphic to the 3-sphere $S^3 \subseteq \mathbb{C}^2$
-$$ SU(2) = \left\{ \begin{pmatrix} a & -\overline{b} \\ b & \overline{a} \end{pmatrix} : |a|^2 + |b|^2 = 1 \right\} $$
-Now $SU(2)$ acts on its Lie algebra $\mathfrak{su}_2$ (which is 3-dimensional) by conjugation. This action preserves the inner product
-$$ \langle X, Y \rangle = - \frac12 \mathrm{tr}(XY) = \frac12 \mathrm{tr}(X^*Y)$$
-(which is a scalar multiple of the Killing form of $\mathfrak{su}_2$, FYI) and hence this gives a homomorphism
-$SU(2) \to SO(\mathfrak{su}_2) \simeq SO(3)$. (A priori this gives a map to $O(3)$, but $SU(2)$ is connected so the image lands in $SO(3)$.
-Now consider the orthonormal basis for $\mathfrak{su}_2$ given by
-$$
-e_1 = \begin{pmatrix} i & 0 \\ 0 & -i \end{pmatrix}, e_2 = \begin{pmatrix} 0 & 1 \\ -1 & 0 \end{pmatrix}, e_3= \begin{pmatrix} 0 & i \\ i & 0 \end{pmatrix},
-$$
-let
-$$
-x = \begin{pmatrix} a & -\overline{b} \\ b & \overline{a}\end{pmatrix},$$
-and write down the adjoint action of $x$ on $e_1,e_2,e_3$. For instance you get
-$$
-\begin{align}
-xe_1x^{-1} & = \begin{pmatrix} i(|a|^2 - |b|^2) & 2ia\overline{b} \\ 2i\overline{a} b & -i(|a|^2 - |b|^2) \end{pmatrix} \\
-& = (|a|^2 - |b|^2)e_1 + i(a\overline{b} - \overline{a}b)e_2 + (a \overline{b} + \overline{a}b)e_3.
-\end{align}$$
-This gives you the first column of the matrix representation conjugation by $x$. I'll leave the others to you. But this way you can see where the formulas come from.
-This gives exactly David Speyer's answer (possibly modulo some re-ordering of the basis). His four real numbers $a,b,c,d$ would correspond to my complex numbers $a,b$ via
-$$a_{mine} = a + i b, \quad b_{mine} = c + id.$$<|endoftext|>
-TITLE: On the inverse Galois problem
-QUESTION [18 upvotes]: Q: What is the "simplest" finite group $G$ for which we don't know how to realise it as a Galois group over $\mathbf{Q}$ ?
-So here the word simplest might be interpreted in a broad sense. If you want something precise
-you may take the group of smallest order but I prefer to leave the question as it is.
-Also since naturally one classifies finite groups into families one may also ask the following
-Q: What is the "simplest" example of a family of finite groups for which the inverse
-Galois problem is unknown?
-
-REPLY [5 votes]: I am not an expert and I might be misrembering some talks I've attended. Anyway, $SL_2(\mathbb{F}_q)$ can be done for prime $q$ by using torsion on non-CM elliptic curves. But I don't think it's been done for general prime powers $q$. Also, what about $SL_3$?<|endoftext|>
-TITLE: a Ramsey-type question
-QUESTION [8 upvotes]: This question is related to this one but feels more Ramsey-type, so perhaps it is easier. Let $S$ be a finite set, $|S|=k$. Suppose we color all subsets of $S$ in $1000$ colors. What is the maximal (in terms of $k$) guaranteed length $l=l(k)$ of a monochromatic sequence of pairwise different subsets $A_1,A_2,..., A_l$ such that $|A_i\setminus A_{i+1}|+|A_{i+1}\setminus A_i|\le 2$ for every $i$? Clearly if $A$ is a subset of $S$ such that all 2-element subsets of $A$ are monochromatic, then $l(n)\ge |A|-1$ (there is a sequence of 2-element subsets of $A$ which satisfies the above property). So $l(k)$ is at least as big as the corresponding number from the Ramsey theory. Is it much bigger? The number 1000 is of course "any fixed number".
- Update 1 Fedor and Tony showed below that $l(k)\ge k/1000$. Thus only the first question remains: What is $l(k)$? Is it exponential in $k$, for example?
- Update 2 Although the question I asked makes sense (see Update 1), I realized that it is not the question I meant to ask. Here is the correct question. Same assumptions: $|S|=k$, 1000 colors. We consider monochromatic sequences of pairwise different subsets ${\mathcal A}=A_1,A_2,...,A_l$, where $|A_i\setminus A_{i+1}|+|A_{i+1}\setminus A_i|\le 2$. For each of these sequences we compute $\chi({\mathcal A})=|A_1\setminus A_l|+|A_l\setminus A_1|$. Now the question: what is the maximal guaranteed $\chi({\mathcal A})$ in terms of $k$, call it $\chi(k)$? By Ramsey, this number grows with $k$. Indeed if we color just $s$-element subsets, we will be able (if $k\gg s$) to find a subset of size $2s$ where all subsets of size $s$ are colored with the same color; then we can find a monochromatic sequence of subsets of size $s$ with the above property and $\chi=2s$ because the first and the last subsets in that sequence are disjoint. The question is what is the growth rate of $\chi(k)$. The question is motivated by Justin Moore's answer
- here.
-
-REPLY [3 votes]: Here is a proof of a very weak upper bound for $l(k)$. Consider the colouring of $2^S$ where each set is coloured by its size (mod 1000). A good monochomatic sequence must consist of sets of the same size. Thus we obtain $l(k) \leq \binom{k}{k/2}$.
-However, we can be a bit smarter. Instead of colouring all $i$-subsets of $S$ with the same colour, we can use 333 colours and still guarantee that a good monochromatic sequence must use sets of the same size. Thus, we are lead to the problem of 333-colouring $\binom{S}{i}$ to minimize the length of a good monochromatic sequence inside $\binom{S}{i}$.<|endoftext|>
-TITLE: Proving that a function's image contains (1/n,...,1/n)
-QUESTION [26 upvotes]: This question is a follow-up to a previous question answered by Neil Strickland:
-Map from simplex to itself that preserves sub-simplices
-Let $B$ denote the closed unit ball in $\mathbb{R}^2$ and let $\Delta_{n-1}$ denote the $(n-1)$-simplex. I have a continuous function $f(x_1,\dots,x_n):B^n \rightarrow \Delta_{n-1}$ defined for all subsets $\lbrace x_1,\dots,x_n\rbrace \subset B$ of size $n$ that satisfy $x_i \neq x_j$ for all pairs $i,j$ (in other words, the function is only defined if all of the $n$ arguments are distinct). This function has the property that, if $\sigma$ denotes a permutation, then $f(\sigma(x_1,\dots,x_n)) = \sigma(f(x_1,\dots,x_n))$. In other words, permuting the arguments of the function merely permutes the output. My question is: are there non-trivial sufficient conditions on $f$ under which the point $(1/n , \dots, 1/n)$ lies in the image of this map? (or, even better, is this always the case?)
-Here's one property of the map $f$ that I can add regarding the requirement that arguments be distinct: if $\lbrace \mathbf{x}_k \rbrace$ is a sequence of $n$-tuples (with distinct entries) in $B$ that converges to an $n$-tuple $\bar{\mathbf{x}}$ with (possibly) non-distinct entries, then the limit of $f(\mathbf{x}_k)$ exists if and only if, for each pair of entries $x_i^k$ and $x_j^k$ in the $n$-tuple, the unit direction vector from $x_i^k$ to $x_j^k$ (i.e. $\frac{x_i^k - x_j^k}{||x_i^k - x_j^k||}$) has a limit.
-
-REPLY [12 votes]: Let me rephrase the question and give a complete answer:
-What are conditions for the existence of a $\Sigma_n$-equivariant map
-$$f: F_n(B)\to \Delta_{n-1}\backslash(\tfrac{1}{n},\dots,\tfrac{1}{n})\; ?$$
-Here $B\subset \mathbb{R}^2$ denotes the closed unit ball, $F_n(B)$ is the ordered configuration space of $n$ points on $B$ with $\Sigma_n$ acting by permutation and $\Sigma_n$ acts on $\Delta_{n-1}:=\{(x_1,\dots,x_n)\in\mathbb{R}^n: \sum_ix_i=1, 0\leq x_i\leq 1\}$ by permuting the coordinates.
-As Neil Strickland pointed out, using an equivariant homeomorphism, we can replace
-$\Delta_{n-1}\backslash(\tfrac{1}{n},\dots,\tfrac{1}{n})$ by $\partial\Delta_{n-1}\cong_{\Sigma_n} S^{n-2}.$ This sphere $S^{n-2}$ can be viewed as $S(W_n)$, where $W_n:=\{(x_1,\dots,x_n)\in\mathbb{R}^n: \sum_ix_i=0\}$ and $\Sigma_n$ is again acting by permuting the coordinates. Using equivariant homotopy equivalences, we can replace $F_n(B)$ by $F(\mathbb{R}^2)$ and further use an equivariant model of dimension $n-1$, which Neil Strickland calls $X_0$ and I will call $\mathcal{F}_n(\mathbb{R}^2)$. So an equivalent question is:
-What are conditions for the existence of a $\Sigma_n$-equivariant map
-$$f: \mathcal{F}_n(\mathbb{R}^2)\to S(W_n)\; ?$$
-Fortunately, there is a complete answer to that questions; it is given by Blagojević and Ziegler: arxiv, springer:
-Such a map exists if and only if $n\geq 2$ is a not a prime power
-The authors use equivariant obstruction theory and an explicit model $\mathcal{F_n(\mathbb{R}^2)}$, which is called $\mathcal{F}(d,n)$ in their notation. Just plug in $d=2$ and the result is the main theorem of section 4 in the paper on the arxiv.
-From the nonexistence of the map we conclude that the barycenter $(\frac{1}{n},\dots,\frac{1}{n})$ is hit: If $n$ is a prime power, then your continous equivariant function $B^n\to\Delta_{n-1}$ will always hit the barycenter and if $n$ is not a prime power, there exist equivariant functions that miss the barycenter.<|endoftext|>
-TITLE: Does the random Lorenz gas have a non-trivial diffusion coefficient?
-QUESTION [6 upvotes]: For the periodic Lorenz gas Sinai showed that rescaling the trajectory of the tracer particle yields Brownian motion in the limit. Does there exist a similar result for the random Lorenz gas? If not, do people believe that there is such a limit?
-By the random Lorenz gas I mean: take circular scatterers distributed uniformly at random in the plane conditioned on the scatterers not overlapping. The scatterers are fixed. The tracer particle is a point that moves with constant speed and has perfectly elastic collisions with the scatterers. An initial condition is chosen at random (say, by picking an initial point away from a scatterer and then picking the initial angle uniformly on $[0,2\pi)$.)
-The numerical experiments in Dettmann and Cohen, 2000 suggest that there is diffusive behaviour for the random Lorenz gas. This article by Bunimovich states that it is believed that velocity autocorrelation decays polynomially, but does not mention whether it decays fast enough for there to be a finite diffusion coefficient.
-
-REPLY [2 votes]: I have been looking at random (and periodic) Lorentz gas models recently in preparing a review which has just appeared at arxiv:1402.7010. As far as I know, there are rigorous results only for the low density (Boltzmann-Grad) limit, and for models where the scatterers are known not to overlap (for example placing a scatterer or empty space randomly at each site of a lattice). In the fixed density case there does not even appear to be a proof that "conditional on the scatterers not overlapping" converges. Choosing a point outside a scatterer at random is problematic since the measure is infinite; better to fix the point at the origin and then choose the scatterers conditional on not overlapping each other or the origin. The low density limit suggests that the velocity autocorrelation decays as $t^{-d/2-1}$, which is fast enough for the diffusion coefficient (its integral) to exist.
-Correction: After the above paper appeared online (Commun. Theor. Phys. 62 521-540 (2014).), and following discussions with D. Szasz, I discovered references that do indeed show that the non-overlapping condition converges. See A variational principle for the equilibrium of hard sphere systems, Gallavotti, G and Miracle-Sole, S, Annales IHP A 8 287-299 (1968); appendix B of Observables at infinity and states with short range correlations in statistical mechanics, Lanford III, OE and Ruelle, David, Commun. Math. Phys. 13 194-215 (1969). But the diffusion question remains open.<|endoftext|>
-TITLE: A bijection between sets of Young tableaux of two kinds
-QUESTION [27 upvotes]: Assuming that the problem of exhibiting a bijection is not considerd a frivolous pursuit, allow me to ask a question troubling me for some time now.
-Let $\lambda \vdash n$ denote the fact that $\lambda$ is a partition of $n$. Denote the number of parts by $l(\lambda)$. If $T$ is a standard Young tableau (SYT), we will denote the underlying partition shape by $sh(T)$.
-Given a positive even integer $2n$, let
-$$ Pe_{2n}=\{ \lambda: \lambda\vdash 2n,\text{ } l(\lambda) \leq3 \text{ and all parts of } \lambda \text{ are even} \}$$
-and
-$$ Qe_{2n}=\{ \lambda: \lambda\vdash 2n, \lambda = (k,k,1^{2n-2k}), \text{ }k\geq 1 \}$$
-Using these sets we will define two more sets whose elements are SYTs.
-$$ TPe_{2n}=\{T: T \text{ an SYT, } sh(T)\in Pe_{2n} \}$$
-and
-$$ TQe_{2n}=\{T: T \text{ an SYT, } sh(T)\in Qe_{2n} \}$$
-$\textbf{Question}$:
-Is there a bijective proof exhibiting the fact that the cardinalities of $TPe_{2n}$ and $TQe_{2n}$ are equal?
-The second question is very similar. Given an odd positive integer $2n+1$, let
-$$ Po_{2n+1}=\{ \lambda: \lambda\vdash 2n+1,\text{ } l(\lambda)=3 \text{ and all parts of } \lambda \text{ are odd} \}$$
-and
-$$ Qo_{2n+1}=\{ \lambda: \lambda\vdash 2n+1, \lambda = (k,k,1^{2n+1-2k}), \text{ }k\geq 1 \}$$
-Using these sets we will define two more sets whose elements are SYTs.
-$$ TPo_{2n+1}=\{T: T \text{ an SYT, } sh(T)\in Po_{2n+1} \}$$
-and
-$$ TQo_{2n+1}=\{T: T \text{ an SYT, } sh(T)\in Qo_{2n+1} \}$$
-$\textbf{Question}$:
-Is there a bijective proof exhibiting the fact that the cardinalities of $TPo_{2n+1}$ and $TQo_{2n+1}$ are equal?
-I tried quite a few approaches ( Motzkin path interpretations, matching diagrams etc) but did not succeed. I hope somebody here can guide me.
-The relevant OEIS entry would be link text
-Thanks,
-Vasu
-
-REPLY [10 votes]: Let me explain a way to prove such results. There are (mostly) two types of people out there, those that think of the $n$th Catalan number as $\frac{1}{n+1}\binom{2n}{n}$ and those that think of it as $\binom{2n}{n}-\binom{2n}{n-1}$. The first type of people will likely think of Dyck paths as eqivalence classes or $\mathbb Z/(n+1)\mathbb Z$ orbits of paths, whereas the second type will probably think of Dyck paths as "paths inside an $n\times n$ rectangle that don't cross the origin" (See proof 2 vs proof 3 here). While the first type would probably be happy to count standard young tableaux using the usual form of the hook-length formula, the second type would probably prefer the (equivalent):
-Jacobi-Trudi Formula
-The number of Standard Young Tableaux on a partition of shape $\lambda=(\lambda_1,\dots,\lambda_k)$ of $n$ is
-$$f^{\lambda}=\sum_{\sigma \in S_k}\varepsilon(\sigma)\binom{n}{\lambda_1+k-\sigma(k)\, ,\dots,\,\lambda_k+1-\sigma(1)}.$$
-Where $\varepsilon$ denotes the sign of a permutation. Similar to the Catalan numbers in this presentation one sacrifices obvious positivity but has obvious integrality. For the sake of illustration, in the case $k=3$, this says
-$$f^{(a,b,c)}=\binom{n}{a,b,c}+\binom{n}{a+1,b+1,c-2}+\binom{n}{a+2,b-1,c-1}-\binom{n}{a+1,b-1,c}-\binom{n}{a,b+1,c-1}-\binom{n}{a+2,b,c-2}.$$
-The computation:
-Let's take the case of three even parts, although this method will work for the odd case as well as the general case of partitions with a fixed (arbitrary) number of rows. If one sums the quantity $f^{(a,b,c)}$ over all triples of integers $a\geq b\geq c\geq 0$ with $a+b+c=n$ then the multinomial coefficients form the formula above telescope and we are left with
-$$|TPe_{2n}|=\sum_{k\geq 0}\binom{2n}{k,k,2n-2k}-\sum_{k\geq 0}\binom{2n}{k,k+1,2n-2k-1}$$
-Now we can turn our attention to $TQe_{2n}$. We can count the number of SYT of shape $(k,k,1^{2n-2k})$ from the hook length formula, but instead of presenting it in "type one" as Brian Hopkins did in the other answer, we will use the "type two" presentation $$f^{(k,k,1^{2n-2k})}=\binom{2n}{k}\binom{2n-k-1}{k-1}-\binom{2n}{k+1}\binom{2n-k}{k}$$
-$$=\binom{2n}{k,k,2n-2k}-\binom{2n}{k}\binom{2n-k-1}{k}-\binom{2n}{k-1}\binom{2n-k}{k}.$$
-So in order to get the desired equality we sum over all $k\geq 1$
-$$|TQe_{2n}|=\sum_{k\geq 1}f^{(k,k,1^{2n-2k})}=\sum_{k\geq 1}\left(\binom{2n}{k,k,2n-2k}-\binom{2n}{k}\binom{2n-k-1}{k}-\binom{2n}{k-1}\binom{2n-k}{k}\right)$$
-$$=\sum_{k\geq 1}\binom{2n}{k,k,2n-2k}-\sum_{k\geq 0}\left(\binom{2n}{k}\binom{2n-k-1}{k}+\binom{2n}{k}\binom{2n-k-1}{k+1}\right)+\binom{2n}{0}\binom{2n-1}{0}$$
-$$=\sum_{k\geq 1}\binom{2n}{k,k,2n-2k}-\binom{2n}{k,k+1,2n-2k-1}+\binom{2n}{0,0,2n}$$
-$$=\sum_{k\geq 0}\binom{2n}{k,k,2n-2k}-\sum_{k\geq 0}\binom{2n}{k,k+1,2n-2k-1}=|TPe_{2n}|.$$
-Can it be made bijective?
-Yes. All the manipulations with binomial coefficients have obvious bijective interpretations in terms of lattice paths, so one would have to do the same for the Jacobi-Trudi formula mentioned above. This is classical for the case of Catalan numbers, but it can be generalized for any partition using a similar reflection trick. I have been lazy and didn't work out the final bijection for you, but at least this is a start. I am currently writing a note about such bijections between SYT's with bounded parts, so if no one has explained it by then I'll make sure to update this answer.<|endoftext|>
-TITLE: To what extent do we know the representations of GL(2,Zp)
-QUESTION [8 upvotes]: Consider a local non archimedean field $k$ and its ring of integer $o$. To what extent, do we know the complex irreducible representations of $GL(2,o)$? Is there a specific list giving them all in terms of induction from certain "simple" subgroups?
-I have had read, that they have been classified according to their characters, but I am not quite happy with the various presentations so far.
-What I'd like to see is pretty simple to explain:
-
-Classify all irreducibles of the group (Borel subrgoup) of upper triangular matrices $B$ (easy).
-Given an irreducible representation $\pi$ of $B$, decompose $Ind_{B}^{GL(2,o)} \pi$ in its irreducibles (hard).
-
-Is there such a treatment available? If not, what is the reason why not to proceed along these lines?
-
-REPLY [6 votes]: Paul Broussous has already addressed the question in the title, and since it seems that you are in fact more interested in the last two questions, I will focus on these.
-I don't think there is a treatment following steps 1) and 2) available in the literature, but it should be possible to work this out since the representations of $\mathrm{GL}_2(\mathcal{O})$ are known explicitly. On the other hand, it is not clear how useful this would be, because already for $\mathrm{GL}_2$ over a finite field $\mathbb{F}_q$, this is not the preferred approach to the representations (apart from the principal series, of course). This is partly because it quickly becomes unmanageable, and does not allow for an inductive approach in the same way as parabolic induction.
-Moreover, over a finite field even step 1) is problematic in general, because the irreps of $B(\mathbb{F}_q)$ are related (via Clifford theory) to the irreps of the upper uni-triangular subgroup $U(\mathbb{F}_q)$, and the classification of the latter is known to be a wild problem (for large enough p, one can describe the irreps of $U(\mathbb{F}_q)$ using a Kirillov orbit method formalism, but it is not clear that this would be helpful for the problem at hand).
-Also, in step 2) there is going to be a lot of repetition, that is, two induced representations may have irred constituents in common, and there may be no easy way to tell when this happens and for which irred constituents.
-Finally, as you mention in one of the comments, the approach via steps 1) and 2) can only work if there is a good description of the $B$-$B$ double cosets, and this is not available in general. Note however that this is not the only place where wildness enters, as the above remarks on the situation for groups over a finite field shows. One advantage of the approach via Clifford theory and orbits (following Hill), is that one can avoid wild classification problems by restricting the construction to a subcategory of representations, such as the regular representations.<|endoftext|>
-TITLE: Mutually generics
-QUESTION [7 upvotes]: Given posets $P,Q\in M$, I would like to know under what circumstances there are mutually generic filters $G\subseteq P$ and $H\subseteq Q$ (generic over $M$). Also, what are the characterizations of mutual genericity? And finally, what can we say about the relation between $M[G]$ and $M[H]$ in that case?
-I have a slight difficulty finding references to the notion and properties of mutual genericity (whatever they are).
-
-REPLY [8 votes]: Another fact to add to Joel's list, one which I found surprising when I first came across it, is that adding a single Cohen real adds a perfect set of reals such that any finitely many are mutually generic over each other. To see this simply consider the trace of a Cohen real on the members of an almost disjoint family and use the theorem Joel quotes about products.<|endoftext|>
-TITLE: Dimension 1 prime ideals in the intersection of two maximal ideals
-QUESTION [17 upvotes]: This question/problem really comes from a fact in algebraic geometry, where it says that given an irreducible variety $V$ ($\dim V \geq 2$) then for any given pair of points $x,y\in V$ there is an irreducible curve $C$ connecting them.
-This can proved by invoking Bertini's theorem [See Lazarsfeld R. - Positivity in Algebraic Geometry 1, example 3.3.5].
-Translating this back to coordinate rings we get something like: Given a finitely generated $k$-algebra $R$ which is a domain, $k=\bar{k}$ (alg.closed) then for any pair of maximal ideals $m_1,m_2\subset R$ there is a prime ideal $p\subset m_1\cap m_2 $ with $\dim R/p =1$.
-So my question is therefore: Is there a purely ring theoretical argument for this fact? In what generality does it hold?
-The reason why i emphasize on purely is because, and i am no expert on this, but apparently the Bertini type argument can be (re-)formulated algebraically, however this, I have been informed, will not be pretty...
-
-REPLY [4 votes]: This reminds me of what someone once said to me:
-
-The geometers can always take a hyperplane section. We can't!
-
-The purpose of this post is to analyze the question to see how close it is to Bertini's theorem (it is not obvious to me). The statement is immediately equivalent to: one can always find a nonzero prime ideal $P \subseteq m_1\cap m_2$ (since if we can, then induction on dimension proves the original).
-Now, how can such $P$ exist? Let $U_1 = R-m_1$ and $U_2=R-m_2$. Let $U=\{xy \| x\in U_1, y\in U_2\}$. $U$ is multiplicative and our prime $P$ obviously just has to avoid $U$. So $P$ exists unless the localization $U^{-1}R$ has dimension $0$. But it is a domain, so we have to make sure $U^{-1}R$ is not a field. That statement is equivalent to the existence of some element $f\in R$ such that $f$ does not become an unit in $U^{-1}R$. In other words:
-
-there are no $a,b \in U$ such that $af=b$.
-
-Since $b$ is itself a product of elements in $U_1,U_2$, our condition is obvious if $fR$ is a prime ideal and $f\in m_1\cap m_2$. But it is technically weaker, although not clear to me by how much. Note that we do not require $f$ to be linear, which is common for Bertini's type statement.
-This analysis would seem to rule out certain clever arguments. But may be one can find some!
-For "algebraic" version of Bertini's theorem, I will look at the reference given here. Also, how to rescue Bertini over finite fields using hypersurface instead of hyperplane (which suits your purpose), looks here.<|endoftext|>
-TITLE: Reference for a formula expressing the characteristic polynomial of a sum of endomorphisms
-QUESTION [9 upvotes]: Let $R$ be a ring, $A$ a (not necessarily commutative) $R$-algebra and $M$ a (left) A-module which is free of finite rank as an $R$-module. If $a\in A$ then multiplication with $a$ on $M$ is an $R$-linear endomorphism of $M$ and as such it has a characteristic polynomial $\chi_a$. I've learned from notes by Bart de Smit which I've accidentally found via Google (http://www.math.leidenuniv.nl/~desmit/notes/charpols.pdf) that the function $\chi_\bullet$ is determined by its values on a generating set of $A$ as an $R$-module. He proves this by deriving suitable formulas for the characteristic polynomial of a sum of two endomorphisms.
-I'm looking for a reference for this fact that can be cited more easily than the informal notes above.
-
-REPLY [7 votes]: One reference is: S. A. Amitsur, On the Characteristic Polynomial of a Sum of Matrices,
-Linear and Mult. Algebra 8 (1980), 177-182. (pp. 469-474 in Selected Papers of S. A. Amitsur,
-Part 2, AMS 2001.)<|endoftext|>
-TITLE: Terminology question: "Transverse" v. "Transversal"
-QUESTION [10 upvotes]: Something that's always bothered me is that the word "transversal" is very commonly used as an adjective, but my understanding is that "transverse" is the correct adjective, and that "transversal" is a noun which means "an object which is transverse [to a given object]." So for example you would say "transverse intersection" and "pick a transversal for the line."
-However, I could be wrong. I'd like people to answer with their opinion on which is the correct word for the adjective. I'm making this a community wiki since it's too soft to gain reputation over.
-
-REPLY [19 votes]: "Transversal" is a good old geometry word, a noun, as you say. It goes way back to long before anybody was thinking of transversality in the modern sense.
-It grates on me to hear it used as an adjective, and this owes something to the fact that in my impressionable youth I saw one of the chapter-heading quotations in Hirsch's graduate text on differential topology: From Whitehead, "'Transversal' is a noun. The adjective is 'transverse'". No doubt this also had an impact on others who (like me) tend to be fussy about language.
-On the other hand, language does drift along, and there's no stopping it, and generally no harm is done. By the time you perceive a serious need to tell the world that some usage is wrong, a case can always be made that it is no longer wrong.
-In the case at hand it's understandable that "transversal" has come to be used an adjective; after all, "-al" looks like an adjective ending. (But there are words in English where people have been fooled by that, changing the language. "Bridal" is an example.)
-By the way, if we were going to be sticklers on this point, mightn't we want to go back and change "transversality" to "transversity"?<|endoftext|>
-TITLE: Is there a "trianguline period ring", or is one expected?
-QUESTION [13 upvotes]: Consider a finite-dimensional $\mathbf{Q}_p$-vector space $V$ and a continuous representation $\rho : G_{\mathbf{Q}_p} \to \mathrm{GL}(V)$. Fontaine introduced various $\mathbf{Q}_p$-algebras with $G_{\mathbf{Q}_p}$-actions, notated $B_{\bullet}$ where $\bullet \in \left\{\mathrm{crys}, \mathrm{st}, \mathrm{dR}\dots \right\}$, which "classify" interesting representations $V$; we say $V$ is $\bullet$ if equality holds in the relation $\mathrm{dim}(B_{\bullet} \otimes V)^{G_{\mathbf{Q}_p}} \leq \dim{V}$.
-Recently, the notion of a "trianguline" Galois representation has become increasingly important. Avoiding the precise definition, I will say rather perversely that the trianguline representations are roughly the closure of the crystalline locus in the set of all $\rho$'s as above (made precise, this is a theorem of Chenevier which is of course predicated on the actual definition of trianguline). So my question: is there a ring of periods $B_{\mathrm{tri}}$ which classifies trianguline representations in the above manner, and/or is such a ring expected to exist? Is/should it be a suitable "completion" of $B_{\mathrm{crys}}$?
-
-REPLY [14 votes]: The category of trianguline representation is stable under all the usual representation-theoretic operations (subs, quotients, $\oplus$, $\otimes$), so by some general tannakian formalism, there does exist a ring $B_{tri}$. The rough idea is to look at $Q_p^{alg} \otimes B_{st} \langle \langle \log(t) \rangle \rangle$ where "$\langle \langle \log(t) \rangle \rangle$" means "power series with some non zero radius of convergence" and $t$ is the usual $t$ in this business. It's interesting to note that I first heard about this ring from Fontaine (around 2003-04 maybe - I was still at Harvard) when trianguline representations had not yet been defined. Fontaine told me at the time that repns admissible for this ring should be interesting! A few comments are in order:
-
-$B_{st}$ does not have the structure of a Banach space, so you need to figure out what "radius of convergence" means
-$\exp(\log(t))=t$, so there are relations in the definition of your ring
-you need to decide if you want a ring for "trianguline" or "split-trianguline" repns
-
-I thought about this again a few weeks ago and, if I remember correctly, came to the conclusion that if you take $B = Q_p(\mu_p) \otimes \hat{Q}_p^{nr} \otimes B_e \otimes Q_p \langle \langle \log(t) \rangle \rangle \otimes Q_p[\log(\tilde{p})]$ (whew!), where $B_e=B_{cris}^{\phi=1}$, then $B$-adm reps of $G_{Q_p}$ are trianguline, and conversely split trianguline reps of $G_{Q_p}$ with integer slopes are $B$-adm. This hopefully gives an idea of the kind of ring which one should be looking for.<|endoftext|>
-TITLE: In what generality is the Verdier biduality map an isomorphism?
-QUESTION [5 upvotes]: Let $X$ be a finite-dimensional, locally compact topological space, and consider the dualizing complex $K_X \in \mathbf{D}^b(X,k)$ (bounded derived category of $k$-sheaves, where $k$ is a noetherian ring). We can define the dualizing functor
-$$C \mapsto D(C) = \mathbf{R}\mathcal{H}om(C, K_X),$$
-(derived internal hom),
-which leads to a biduality map $C \to D^2(C)$. In SGA 4.5 "Th. de finitude," Deligne shows that, when $k = \mathbb{Z}/n$, the analogous biduality morphism is an isomorphism on the constructible bounded derived category (so, an anti-involution of said category) when one is working with a scheme of finite type over a field or DVR (with $n$ prime to the characteristic). I have heard that the same is true for topological spaces under certain conditions, although I'm not sure what the statement (or proof) should be: first, presumably we are going to want with a nice (Gorenstein?) ring like $\mathbb{Z}/n$, and second, probably there needs to be some analog of the constructible derived category. What is this statement?
-I had a look at Kashiwara-Schapira's "Sheaves on Manifolds," but I can't parse the biduality statement given in chapter 3. It's not clear to me how to adapt Deligne's argument to the present case, anyway.
-
-REPLY [3 votes]: For an analytic space, you can find this on page 118 of Verdier's article "Classe d'homologie
-d'un cycle" in Asterisque 36-37. And yes this is on an appropriate constructible derived category with $\mathbb{Z}$-coefficients. I seem to recall that Borel, in his book on intersection cohomology, also discusses this for pseudomanifolds, in case you need something
-more general.<|endoftext|>
-TITLE: Eigendecomposition after multiplying by diagonal matrix
-QUESTION [10 upvotes]: Hello,
-If we possess the eigendecomposition of a positive definite matrix: $X = U \Sigma U^T$, is there an efficient way to compute the eigendecomposition of $D X D$ where $D$ is a diagonal matrix?
-
-REPLY [9 votes]: Write $\Sigma$ as $T^2$, for positive definite $T$. Set $Y = U T$.
-So the eigenvalues of $X$ are the squares of the singular values of $Y$, and what you want to compute are the singular values of $DY$.
-There is no formula which gives the singular values of $DY$ in terms of those of $Y$ and $D$. However, there is a famous set of inequalities relating the three sets of singular values, called the Horn inequalities. See Bhatia's article Linear Algebra to Quantum Cohomology, particularly Section 11, for a gentle introduction.<|endoftext|>
-TITLE: BGG resolution and representations of parabolic subalgebras
-QUESTION [8 upvotes]: Everything here is over $\mathbb{C}$.
-Let $\mathfrak{g}$ be a finite-dimensional simple Lie algebra and let $\mathfrak{p}$ be a parabolic subalgebra (relative to some fixed Borel subalgebra that is unimportant for this question).
-Then $\mathfrak{p}$ has a decomposition
-$$ \mathfrak{p} = \mathfrak{l} \oplus \mathfrak{u_+},$$
-where $\mathfrak{l}$ is a reductive subalgebra (the Levi factor of $\mathfrak{p}$) and $\mathfrak{u}_+$ is a nilpotent ideal (the nilradical of $\mathfrak{p}$).
-Finally, we can decompose $\mathfrak{g}$ as
-$$\mathfrak{g} = \mathfrak{u}_- \oplus \mathfrak{l} \oplus \mathfrak{u}_+$$
-(as $\mathfrak{l}$-modules), where $\mathfrak{u}_-$ and $\mathfrak{u}_+$ are dual to each other via the Killing form of $\mathfrak{g}$.
-Assume further that the following (equivalent) conditions hold:
-
-$\mathfrak{g}/ \mathfrak{p}$ is irreducible as a $\mathfrak{p}$-module;
-$\mathfrak{u}_-$ is irreducible as an $\mathfrak{l}$-module;
-$\mathfrak{u}_-$ is an abelian Lie algebra;
-2 and 3 with $\mathfrak{u}_-$ replaced by $\mathfrak{u}_+$.
-
-Buzzwords here are "Hermitian symmetric space" and "generalized flag variety." There is a classification of these in terms of root systems but I don't want to use that.
-I need to understand the decomposition of ${\bigwedge}^2 \mathfrak{u}_- $ into irreducible modules for $\mathfrak{l}$.
-Using the classification of these parabolics, you can just see explicitly what the highest weight of $\mathfrak{u}_-$ is, and then it's not too hard to compute what ${\bigwedge }^2 \mathfrak{u}_-$ is, but I would like a more elegant way to see what's going on here.
-I have been informed that there is some version of the BGG resolution that will be helpful for this - this evidently gives the highest weights of ${\bigwedge }^2 \mathfrak{u}_-$ in terms of the dotted action of some elements of the Weyl group on the highest weight of $\mathfrak{u}_-$, but at this point I'm stuck.
-I don't know enough (ok, anything really) about the BGG resolution to know where to look for this stuff. Either an explanation or a reference would be much appreciated.
-Edit: I would also be happy with a pointer to a nice reference for the BGG resolution in general.
-
-REPLY [4 votes]: The question involves a fairly long history, going back at least to Bott's Annals paper and two long follow-up papers by Kostant, "Lie algebra cohomology and the generalized Borel-Weil theorem", Ann. of Math. (2) 74 (1961), 329–387, plus the 1963 paper mentioned by rObOt. Those papers predate the work on Verma modules and BGG resolution in the 1970s, which however had Bott's theorem as a byproduct. The BGG arguments made a comparison with Lie algebra cohomology, which in turn involved an easy computation of the 1-dimensional representations of a Cartan subalgebra (Levi subalgebra of a Borel subalgebra) on exterior powers of the nilradical; see 6.4 in my book on the BGG category. This gets less elementary in the parabolic case considered here.
-The special situation with Hermitian symmetric spaces is surveyed at the end of Chapter 9 in my book, along with references to what I think are the main developments in that direction. Again Kostant's papers are a key starting point, but the later approaches using the parabolic BGG resolution were explored by people like Lepowsky (a student of Kostant), Boe, Collingwood, Irving, Shelton. Eventually this work enters the more complicated realm of infinite dimensional representations and Kazhdan-Lusztig-Vogan theory. I'm not sure what the best answer is to the narrower question raised here, but most work on Hermitian symmetric spaces has required case-by-case study using the standard classification.<|endoftext|>
-TITLE: Euler Characteristic of a Variety
-QUESTION [9 upvotes]: Let $Y$ be a "nice" scheme. I am thinking projective varieties over an algebraically closed field, for now, but I am open to more general results.
-In terms of singular homology (coefficients in $\mathbb{Z}$), one can define the Euler characteristic $\chi(Y)$.
-My question is:
-
-Can I express $\chi(Y)$ in terms of the Euler characteristic of certain coherent sheaves on $Y$, in terms of sheaf cohomology?
-
-Most preferably, I would like $$\chi(Y)=\chi(Y,\mathcal{F})$$ for some particular sheaf $\mathcal{F}$.
-I am sorry if this is really trivial or widely known, my searching and asking (in the real world) has led me nowhere so far.
-
-REPLY [7 votes]: First, the answer/reference here might be exactly what you are looking for.
-On the other hand, perhaps you just want to relate natural algebro-geometric structure to the classical Euler characteristic. Here is one way to do that:
-A pure Hodge structure of weight $k$ is a finite dimensional complex vector space $V$ such that $V=\bigoplus_{k=p+q} H^{p,q}$ where $H^{q,p}=\overline{H^{p,q}}$. This gives rise to a descending filtration
-$F^{p}=\bigoplus_{s\ge p}H^{s,k-s}$. Define $\mathrm{Gr}^{p}_{F}(V)=F^{p}/ F^{p+1}=H^{p,k-p}$.
-A mixed Hodge structure is a finite dimensional complex vector space $V$ with a real ascending weight filtration $\cdots \subset W_{k-1}\subset W_k \subset \cdots \subset V$ and a descending Hodge filtration $F$ such that $F$ induces a pure Hodge structure of weight $k$ on each $\mathrm{Gr}^{W}_{k}(V)=W_{k}/W_{k-1}$. Then define $H^{p,q}= \mathrm{Gr}^{p}_{F}\mathrm{Gr}^{W}_{p+q}(V)$ and $h^{p,q}(V) =\dim H^{p,q}$.
-Let $Z$ be any quasi-projective algebraic variety. The cohomology groups with compact support $H^k_c(Z)$ are endowed with mixed Hodge structures by seminal work of Pierre Deligne.
-The Hodge numbers of $Z$ are $h^{k,p,q}_{c}(Z)= h^{p,q}(H_{c}^k(Z))$, and the $E$-polynomial is defined as
- $$
- E(Z; u,v)=\sum _{p,q,k} (-1)^{k}h^{k,p,q}_{c}(Z) u^{p}v^{q}.
- $$
-From this, one gets the classical Euler characteristic $\chi(Z)=E(Z;1,1)$.
-Note: if the counting function of $Z$ over finite fields is a polynomial in the order of the finite field, then $E(Z)$ is exactly the counting polynomial. From this point-of-view, in this case, the Euler characteristic is the number of $\mathbb{F}_1$-points.<|endoftext|>
-TITLE: Positive cone of a subgroup of $\mathbb{Z}^n$
-QUESTION [7 upvotes]: This question sounds like it should be very well known, but for some reason I failed to find a decent answer anywhere. Let $G\subset\mathbb{Z}^n$ be a subgroup, and $G_+=G\cap\mathbb{Z}_{\ge0}^n$ be a cone of elements in $G$ whose coordinates are all nonnegative. The semigroup $G_+$ is finitely generated; it can be proved in a couple of ways. My question is, is there an effective upper bound on the number of generators in terms of $n$, or, even better, the rank of $G$?
-
-REPLY [11 votes]: No, there is no upper bound on the number of generators in terms of $n$.
-Let $k$ be arbitrary positive integer.
-Consider the subgroup
-$$G=\{(x,y)\in\mathbb Z^2\mid x+y\equiv 0\pmod k\}.$$
-Any set of generators of $G_+$
-contains all the elements $(x,y)$ such that $x,y\ge 0$ and $x+y=k$.
-Therefore the number of generators of $G_+$ is $k+1$.<|endoftext|>
-TITLE: Current status of Waring-Goldbach problem
-QUESTION [10 upvotes]: Is the following statement proved?
-For any positive integer $k$ there exists positive integer $n$ such that all sufficiently large integers may be represented as $p_1^k+p_2^k+\dots+p_n^k$ for primes $p_1,\dots,p_n$.
-This Wiki article claims that some progress is made only for $k$ up to 7, on the other hand, it refers to Hua Lo Keng's monograph, in which this statement is proved, if we may take zero terms instead some prime powers. This zero does not seem to be so essential on the first glance... And on the third hand, this paper of Chubarikov contains announcement of the complete proof, though I did not succeed in finding any reaction on it, even no MathSciNet-review.
-
-REPLY [9 votes]: This is a corrected version of my original response, incorporating a nice argument by Fedor Petrov.
-Hua in his book (cf. review of MR0124306) proved that there are integers $s,K,N>0$ such that every $n>N$ with $n\equiv s\pmod{K}$ is a sum of $s$ $k$-th powers of primes. For any $t>0$ let $M(t)$ denote the set of residues modulo $K$ which can be represented by a sum of $t$ $k$-th powers of primes. Clearly $M(t+1)$ contains $M(t)+p^k$ for any prime $p$, hence $|M(t+1)|> |M(t)|$ unless $M(t+1)$ equals $M(t)+p^k$ for any prime $p$. In this case $M(t)$ is invariant under the shift of $p^k-q^k$ for any two distinct primes $p$ and $q$. The shifts are coprime (e.g. $p^k-q^k$ is coprime with $q$), hence $M(t)=M(t)+1$, and $M(t)$ contains all residues modulo $K$. This argument shows that $|M(K)|=K$, i.e. modulo $K$ every residue class is a sum of $K$ $k$-th powers of primes. If $p$ denotes the largest of the $K^2$ primes used in the latter representation, and $M$ equals $N+Kp^k$, then we have the following. For every $m>M$ there is a sum of $K$ $k$-th powers of primes, denote it by $m'$, such that $m-m'\equiv s\pmod{K}$ and $m-m'>N$. By Hua's theorem, $m-m'$ is a sum of $s$ $k$-th powers of primes, hence in fact every $m>M$ is a sum of $s+K$ $k$-th powers of primes.
-To summarize: the statement in Fedor Petrov's original question follows from Hua's theorem.<|endoftext|>
-TITLE: Euler product over primes congruent to 3 mod 4
-QUESTION [9 upvotes]: I'm a string theorist and I have come across the following expression in a computation I'm doing (involving a sum over inequivalent Lens spaces):
-$$\widehat{\zeta}(s)=\prod_{\mathrm{primes}\ p\equiv 3\ (\mathrm{mod}\ 4)}(1-p^{-s})^{-1}.$$
-I expected this to be a standard sort of construction, and it's clearly closely related to certain standard zeta-functions and Dirichlet $L$-functions (for instance it seems that $\widehat{\zeta}(s)^2/\widehat{\zeta}(2s)=L(\chi_0,s)/L(\chi_1,s)$, where $\chi_0$ and $\chi_1$ are the trivial and nontrivial Dirichlet characters modulo $4$), but with my poor knowledge of this very wide field, I wasn't quite able to find this particular combination. I would like to extend this analytically in $s$ and am particularly interested in any poles along the real axis and their residues.
-Any help or references would be much appreciated, and apologies if there was an easily found answer that I missed.
-
-REPLY [7 votes]: Your functional equation shows that $\widehat{\zeta}(s)^2$ has a meromorphic continuation to $\Re(s)>1/2$ with a simple pole at $s=1$. This also shows that $\widehat{\zeta}(s)$ does not have a meromorphic continuation to a neighborhood of $s=1$, instead it lives on a double cover of that neighborhood branched at $s=1$.
-If you want to extend further to the left, you need even higher powers of $\widehat{\zeta}(s)$: precisely the $2^k$-th power for $\Re(s)>2^{-k}$. The poles on the positive real axis will be at the points $s=2^{-k}$, for the relevant powers of $\widehat{\zeta}(s)$. This also tells me that there is no reasonable continuation to the left of the imaginary axis.<|endoftext|>
-TITLE: For $\mathfrak g$ A Lie algebra of type $ E_7 $, $\mathfrak h $ a Cartan subalgebra and $\Delta$ the resulting root system, does $ Aut(\mathfrak g,\mathfrak h)\rightarrow Aut(\Delta) $ split over the Weyl group?
-QUESTION [13 upvotes]: Given a complex simple Lie algebra $ \mathfrak g $ of type $E_7$, Cartan subalgebra $ \mathfrak h $ and simple roots $\alpha_1,…\alpha_n $, suppose $\pi $ is an involution of the extended Dynkin diagram. I would like to know whether $\pi $ must be induced from an involution of $ \mathfrak g $. Writing $\Delta $ for the root system and W for the Weyl group, since $ Aut(\Delta)/W $ is isomorphic to the group of automorphisms of the Dynkin diagram and this is trivial for $ E_7 $, the answer is "yes" if the map $ Aut(\mathfrak g,\mathfrak h)\rightarrow Aut(\Delta) $ splits over the Weyl group.
-That is, it would suffice if there is a subgroup of the automorphism group of $Aut(\mathfrak g, \mathfrak h)$ which is isomorphic to the Weyl group under this mapping.
-Is this true? Or do you otherwise know whether every involution of the extended Dynkin diagram for $ E_7 $ must arise from an involution of $ \mathfrak g $?
-Thanks very much!
-
-REPLY [3 votes]: Thanks very much all for these most helpful answers, much appreciated! Regarding the 2nd (and easier) part of the question, as you suspected this is true. In the preprint http://front.math.ucdavis.edu/1111.4028 Katharine Turner and I were needing this for some work on harmonic maps, and the form of the statement we prove there is below. It seems like something that would be known to folks working in this area, but we couldn't find a reference.
-Every involution of the extended Dynkin diagram for a simple complex Lie algebra $\mathfrak {g} ^\mathbb {C} $ is induced by a Cartan involution of a real form of $\mathfrak {g} ^\mathbb {C} $.
-More precisely, let $\mathfrak {g}^\mathbb {C} $ be a simple complex Lie algebra with Cartan subalgebra $\mathfrak {t} ^\mathbb {C} $ and choose simple roots $\alpha_1,\ldots,\alpha_N $ for the root system $\Delta (\mathfrak {g} ^\mathbb {C},\mathfrak {t} ^\mathbb {C}) $. Given an involution $\pi $ of the extended Dynkin diagram for $\Delta $,
-there exists a real form $\mathfrak {g} $ of $\mathfrak {g} ^\mathbb {C} $ and a Cartan involution $\theta $ of $\mathfrak {g} $ preserving $\mathfrak {t} =\mathfrak {g}\cap\mathfrak {t} ^\mathbb {C} $ such that $\theta $ induces $\pi $ and $\mathfrak {t} $ is a real form of $\mathfrak {t} ^\mathbb {C} $. The Coxeter automorphism $\sigma $ determined by $\alpha_1,\ldots,\alpha_N $ preserves the real form $\mathfrak {g} $.<|endoftext|>
-TITLE: How to recognize a finite dimensional algebra is Koszul or quadratic?
-QUESTION [8 upvotes]: I have a family of finite dimensional algebras that are directed quasihereditary. I think they might be Koszul algebras and I am wondering what approaches there are to check Koszulness or even quadraticity. I know the quivers of these algebras and can compute Ext^n between simple modules for all n, but I do not have a quiver presentation. I know that there are paths of length 2 and of higher lengths between all vertices of the quiver with nonvanishing Ext^2 so I cannot prove or eliminate quadraticity for trivial reasons. Any thoughts?
-I should add that I do not have explicit minimal projective resolutions of the simple modules.
-
-REPLY [4 votes]: One incredibly useful fact is that it suffices to find linear resolutions of standard modules, not simples. These are usually much easier to find "by hand." Strictly speaking this is stronger than Koszul (the term is "standard Koszul") but in practice it seems rare for a quasi-hereditary algebra to be Koszul and not standard Koszul.<|endoftext|>
-TITLE: On meromorphic continuation of zeta function(s) and special values at negative integers
-QUESTION [14 upvotes]: Euler developped (at least) two different approaches in order to calculate the values $\zeta(-m)$ of the zeta function $$\zeta(s) = \sum_{n\geq 1} \frac{1}{n^s}$$ at non-positive integers.
-In one approach Euler obtained the following formula for the zeta function $$\zeta(s) = (1-2^{1-s})^{-1}\sum_{n=0}^\infty \frac1{2^{n+1}}\sum_{i=0}^n \binom{n}{i} (-1)^i \frac1{(i+1)^s} $$
-by applying the so called Euler transformation to the series $\sum_{n=0}^\infty (-1)^n \frac1{(n+1)^s}$ (see for example the Wiki article Euler summation).
-One great advantage of this formula is that you can immediately calculate the values at the negative integers because the infinite series reduces to a finite sum in this case.
-Moreover this formula gives at the same time a meromorphic continuation of the zeta functions to the complex plane!
-In another approach Euler introduced a formal parameter $t$ and showed that $$(1-2^{m+1})\zeta(-m) = (t \frac{d}{dt})^m(\frac{t}{t+1})|_{t=1} = (\frac d {dx})^m(\frac{e^x}{1+e^x})| _{x=0} $$ This is for example described in the very nice book of Hida "Elementary theory of L-functions and Eisenstein series".
-Further, this approach could be generalized to "L-functions of totally real number fields" by Shintani, Cassou-Nouges and others (see again Hida's book, e.g.).
-My questions are now the following:
-
-1) Can the first approach described above generalized to other
- Dedekind zeta functions? I completely lack an understanding of the Euler
- transformation. Is there a way to "understand" Euler's formula
- for the zeta function in a broader sense?
-2) The Riemannian approach to the meromorphic continuation of the zeta function is based on looking at the Mellin transformation of Jacobi's $\theta$-function. It is well known, by the work of Hecke, that this approach generalizes to arbitrary Hecke L-functions. On the other hand, with this approach the information about the special values at non-positive integers of the corresponding Hecke L-function are more difficult to reveal (at least as far as I know). (EDIT: I'm aware of the fact, that calculating special values is extremely hard and one of the big challenges in number theory, I just try to understand certain structures underlying this more than beautiful area of mathematics...)
-
-This makes we wondering whether there is a relation between the approaches of Euler and Riemann to the meromorphic continuation of the zeta function.
-
-Is there a "principle" that relates Euler's analytic continuation to Riemann's ?
-
-(One major difference is of course that Euler's expression doesn't involve the archimedean factor at all! I also lack a real understanding on why the archimedean Euler factor appears in Riemann's approach.)
-
-3) Is there a known relation between the two different approaches of Euler for
-calculating the values $\zeta(-m)$? Should one expect one?
-
-Thank you very much for your time.
-
-REPLY [2 votes]: In partial answer to 2), one can in fact get information about special values of $L$-functions via theta series. See, for example, Villegas and Zagier's paper "Square roots of central values of Hecke L-series" in the book Advances in Number Theory. (Disclaimer: I'm no expert,and it's certainly a lot more complex than Euler's approach to special values of $\zeta(s)$.)<|endoftext|>
-TITLE: Hirzebruch Surfaces
-QUESTION [6 upvotes]: Good Morning,
-I'm trying to prove that two different definitions of the Hirzebruch Surfaces coincide, and am having problems. Let $a \geq 0$. My first definition for the $a^{th}$ surface is
-$X_a= \mathbb{P}(\mathcal{O}(a) \oplus \mathcal{O}) \longrightarrow \mathbb{P}^1_{\mathbb{C}}$
-My second definition is as follows. Let $C_a$ be a degree $a$ rational normal curve, ie the image under $\mathcal{O}(a)$ of $\mathbb{P}^1_{\mathbb{C}}$ into $\mathbb{P}^a_{\mathbb{C}}$, and let $D_a$ be the projective cone over $C_a$. That is, $D_a$ is defined by the same equations which define $C_a$, except now $D_a\subseteq \mathbb{P}^{a+1}$. Then $D_a$ is a surface which is smooth except for the possibly singular point $v=[0,...,0,1]$. Define $Y_a$ to be the blow-up of $D_a$ at $v$.
-Why are $X_a$ and $Y_a$ isomorphic? (I want to stick to the algebraic or complex category, no smoothness allowed!)
-Robert
-
-REPLY [7 votes]: The cone $D_a$ has a singular point of type $\frac{1}{a}(1,1)$ at its vertex. Blowing up the vertex, the exceptional divisor is a curve $C \subset Y_a$ isomorphic to $C_a$ and whose self-intersection is $\deg \mathcal{O}_{C_a}(-1)=-a$.
-Since $Y_a$ is clearly a geometrically ruled surface over a rational curve (the ruling is given by the strict transform af the system of lines of $D_a$) and $C$ is a section of self-intersection $-a$, it follows $Y_a \cong X_a.$
-Conversely, starting from the surface $X_a$ one can consider the unique section $C$ of negative self-intersection, namely $C^2=-a$; then this section can be blown down by Artin contractibility criterion.
-The blow-down of $C$ is precisely the map $\varphi$ associated to the complete linear system $|C+aF|$ in $X_a.$
-Indeed $h^0(X_a, C+aF)=a+2,$
-hence $$\varphi \colon X_a \longrightarrow D_a \subset \mathbb{P}^{a+1}.$$
-It is immediate to check that $\varphi$ is birational onto its image $D_a$, that it contracts $C$, that the ruling of $X_a$ is sent into a family of lines passing through the point $\varphi(C)$ and that a general hyperplane section of $D_a$ is a rational normal curve $C_a$ of degree $a$.
-Therefore $D_a$ is a cone over $C_a$ and $X_a$ is isomorphic to the blow-up of $D_a$ at its vertex $\varphi(C)$.<|endoftext|>
-TITLE: Numerically most robust way to compute sum of products (standard deviation) in floating-point?
-QUESTION [6 upvotes]: I stumbled across a paper by Welford (1962), where he proclaims a method that should compute the standard deviation numerically more robust than the naive algorithms (http://www.jstor.org/stable/1266577).
-Here, "numerically robust" means that round-off errors are reduced.
-He gives a recurrence for the sum of squares $S_n = \sum_{i=1}^n (x_i - \mu_n)^2 = S_{n-1} + \frac{n-1}{n} (x_n - \mu_{n-1})^2 $ , $\mu$ being, of course, the mean.
-That way, the standard deviation can be computed iteratively in a single pass.
-As far as I understand, he claims that his iterative recurrence formula is numerically more robust, because all terms in it are of the same order (provided the input data are all of the same order).
-This is what I don't understand. It seems to me that, as $n$ gets incremented, $S_n$ becomes larger and larger, so more and more significant digits from the second term are lost, aren't they?
-Googling a bit further, I have found a paper by
-Youngs & Cramer, 1971, who looked at a number of methods, including Welford's, of computing the sum of products / standard deviation more robustly (http://www.jstor.org/stable/1267176).
-Conducting a number of experiments, they found that Welford's method does not provide any benefits. So that seems to confirm my doubts about Welford's method.
-Now, Youngs & Cramer propose another method, which computes $S_n = S_{n-1} + \frac{n-1}{n}(nx_n - s_n)^2 $, where $s_n = s_{n-1} + x_n$.
-Empirically, they found their method to be superior.
-Again, I don't understand why this should be the case: isn't there some catastrophic cancellation going on in $(nx_n - s_n)$ ? Don't the terms $S_n$ and the fraction differ in their magnitude more and more, so that more and more digits of the fraction term get rounded off?
-I would by most grateful if somebody could shed some light on these questions.
-In addition, I'd like to know which is the best method (in terms of roundoff errors) to compute these sums. Surprisingly, I haven't found anything about this in Numerical Recipes (or I overlooked it).
-Thank you very much in advance.
-Gabriel.
-
-REPLY [2 votes]: I found a discussion of this exact problem in Higham, Accuracy and stability of numerical algorithms, Section 1.9.
-The author suggests an alternative algorithm and claims that it is numerically stable (in the mixed backward-forward sense); proofs for the precise accuracy bounds of the formulas are left as exercises.<|endoftext|>
-TITLE: Successive nth powers mod p?
-QUESTION [11 upvotes]: While working on a project, I have run into a situation where I have integers x and n so that $x^n \equiv (x+1)^n \equiv (x+2)^n$ mod $p$ for a prime $p$. It seems to me that this an extremely restrictive condition, and I was wondering if there are any results about when (or if?) it can happen, but I couldn't figure out what to search. Any thoughts? What if I add the additional restriction that they are also congruent to $(x+3)^n$, etc.
-Thanks!
-
-REPLY [2 votes]: Here's another approach... one can show for every $n$ there can be solutions for at most finitely many $p$; and for any given $n$ it's not hard to find these $p$ explicitly.
-For fixed $n$ the question is when $x^n - (x-1)^n$ and $(x+1)^n - x^n$ can have a common factor (for some integer $x$). Applying the Euclidean algorithm to the two polynomials will yield an integer $N(n)$. For solutions to exist, $p$ must be a factor of $N(n)$, so only finitely many $p$ will do.
-(To be perfectly rigorous about this I have to show that the two polynomials have no common factor in $\mathbb{Z}[x]$. But if they did then they would have a common root, say in $\mathbb{C}$. Considering absolute values, we see that the roots of $x^n = (x-1)^n$ all have real part $\frac{1}{2}$ while the roots of $x^n = (x+1)^n$ all have real part $-\frac{1}{2}$. So indeed the polynomials have no common factor, so the Euclidean algorithm will give a constant $N(n)$.
-To see that $p$ must divide $N(n)$: the Euclidean algorithm guarantees that $N(n)$ is a linear combination of the two polynomials in $\mathbb{Z}[x]$. So for any value of $x$, $N(n)$ is a linear combination of $x^n - (x-1)^n$ and $(x+1)^n - x^n$. Hence if $x$ is a solution then $N(n)$ is a multiple of $p$.)
-A quick calculation by hand gives
-N(3) = 2
-N(4) = 30
-N(5) = 44.
-Since $p$ cannot be $2$ or $3$, we see that...
-For $n=3$ there are no solutions...
-For $n=4$ there are solutions only when $p=5$...
-For $n=5$ there are solutions only when $p=11$.<|endoftext|>
-TITLE: Greatest common divisor of a^{2^n}-1 and b^{2^n}-1
-QUESTION [11 upvotes]: Let a and b be coprime integers. Do we know, expect, or unexpect that there are infinitely many primes p which divide
-$gcd(a^{2^n} - 1, b^{2^n}-1)$
-for some n? Certainly any Fermat prime will divide both if I let n get large enough, but one doesn't know whether there are infinitely many of those.
-
-REPLY [2 votes]: A comment on one of Joe's questions: Let $B$ be any real number. It is known unconditionally that there are infinitely many $m$ for which $\phi(m)$ is a square and for which the smallest prime factor of $m$ exceeds $B$. One can even take $m$ as a product of two primes here; see, e.g., article 4 from
-http://www.integers-ejcnt.org/vol11a.html
-or an arXiv preprint of Tristan Freiberg.
-If we choose $B$ larger than $|a|$ and $|b|$, then $m \mid \gcd(a^{\phi(m)}-1, b^{\phi(m)}-1)$, and so there is a prime $> B$ in the support of $\gcd(a^{n^2}-1, b^{n^2}-1)$.<|endoftext|>
-TITLE: Can a finitely generated free group be isomorphic to a non-trivial quotient of itself?
-QUESTION [7 upvotes]: I'm never sure about free groups whether a question is easy or not. It feels to me like this is impossible, but I couldn't come up with any argument.
-If $F_n$ is a free group on $n$ generators, could there exist a non-trivial $N\triangleleft F_n$ such that $F_n/N \cong F_n$?
-
-REPLY [14 votes]: No, the free groups are Hopfian because they are residually finite.<|endoftext|>
-TITLE: Is it possible to pull back a natural transformation?
-QUESTION [5 upvotes]: Suppose that a 2-category $\mathcal{C}$ has strict pullbacks and one has maps $f:F\to C$, $g_0,g_1:G\to C$ and a natural transformation $\gamma:g_1\implies g_0$. Is there a good notion of a pullback transformation $f^*\gamma$. If so, I would expect its codomain to be $$f^*g_1\times_G f^*g_0\to f^*g_1\to F$$ and similarly for the domain.
-If there is, is the need for the extra pullback here (over $G$) connected to the second layer of pullbacks in a 2-categorical descent diagram (relative to quotients in a 1-cat, which only require a kernel pair=1 layer of pullbacks)?
-
-REPLY [6 votes]: There is a good notion of a pullback transformation, but its domain and codomain aren't what you guessed; rather one asks for a map from $f^*(g_0) \to f^*(g_1)$ and a 2-cell filling the resulting triangle over G. The existence of such a pullback transformation is also not automatic from the existence of pullbacks, but requires $f$ to be a fibration (and the existence of all such pullbacks is equivalent to $f$ being a fibration). This characterization of fibrations is studied in Peter Johnstone's paper "Fibrations and partial products in a 2-category".<|endoftext|>
-TITLE: Convex bodies with constant maximal section function in odd dimensions
-QUESTION [16 upvotes]: In 1970 or so, Klee asked if a convex body in $\mathbb R^n$ ($n\ge 3$) whose maximal sections by hyperplanes in all directions have the same volume must be a ball. The counterexample in $\mathbb R^4$ is trivial and can be described as follows:
-Let $f:[-1,1]\to\mathbb R$ be continuous, strictly concave and satisfy $f(-1)=f(1)=0$. For every such function, let $Q_f$ be the body of revolution given by $y^2+z^2+t^2\le f(x)^2$. Then $Q_f$ and $Q_g$ have the same maximal sections in every direction if (and, actually, only if) $f$ and $g$ are equimeasurable, i.e., $|\{f>t\}|=|\{g>t\}|$ for all $t>0$. (Of course, there are plenty of concave functions equimeasurable with $\sqrt{1-x^2}$).
-With some extra work, one can construct something like this in $\mathbb R^n$ when $n$ is even though I do not know any similarly nice geometric description of such bodies for $n\ge 6$.
-What I (and my co-authors) are currently stuck with is the case of odd $n$ (say, the usual space $n=3$). In view of such simple example in $\mathbb R^4$, I suspect that we are just having a mental block. Can anybody help us out?
-
-REPLY [4 votes]: To those who are still interested: we've finally made it but it's so ugly that a nice alternative approach will be always welcome :) We are still stuck with Bonnesen's question about the possibility to recover a convex body from the volumes of its maximal sections and projections in odd dimensions, so some help will be appreciated. The even-dimensional case can be found here. I feel a bit like a student asking for help with his homework, of course, but why not? We all get stuck now and then :). This should really be a comment but it's too long to fit the number of characters restriction.<|endoftext|>
-TITLE: Turing machines that read the entire program tape
-QUESTION [5 upvotes]: Consider a two tape universal Turing machine with a one-way-infinite, read-only program tape with a head that can only move right, as well as a work tape. The work tape is initialized to all zeros and the program tape is initialized randomly, with each cell being filled from a uniform distribution over the possible symbols. What are the possibilities for the probability that the head on the program tape will move infinitely far to the right in the limit?
-Obviously, this will depend on the specifics of the Turing machine, but it must always be in the range $[0,1-\Omega)$, where $\Omega$ is Chaitin's constant for the TM. Since this TM is universal, $\Omega$ must be in the range $(0,1)$, so the probability must always be in $[0,1)$. Is this entire range, or at least a set dense in this range and including zero, possible?
-
-REPLY [5 votes]: Andreas considered the interpretation of your question where we fix the program and then vary the input. Let me now consider the dual version of the question, where we fix the infinite random input and vary the program. Surprisingly, there is something interesting to say.
-The concept of asymptotic density provides a natural way to measure the size or density of a collection of Turing machine programs. Given a set $P$ of Turing machine programs, one considers the proportion of all $n$-state programs that are in $P$, as $n$ goes to infinity. This limit, when it exists, is called the asymptotic density or probability of the set $P$, and a set with asymptotic density $1$ will contain more than 99% of all $n$-state programs, when $n$ is large enough, as close to 100% as desired.
-What I claim is that for your computational model, almost every program leads to a finite computation.
-Theorem. For any fixed infinite input (on the read-only tape), the set of Turing machine programs that complete their computation in finitely many steps has asymptotic density $1$.
-In other words, for fixed input, almost every program stops in finite time.
-The proof follows from the main result of my article: J. D. Hamkins and A. Miasnikov, The halting problem is decidable on a set of asymptotic probability one, Notre Dame J. Formal Logic 47, 2006. http://arxiv.org/abs/math/0504351. The argument depends on the convention in the one-way infinite tape context that computation stops should the head attempt to move off the end of the tape. The idea has also come up on a few other MO quesstions:
-What are the limits of non-halting? and Solving NP problems in (usually) polynomial time? in which it is explained that the theme of the result is the black-hole phenomenon in undecidability problems, the phenomenon by which the difficulty of an undecidable or infeasible problem is confined to a very small region, outside of which it is easy.
-The main result of our paper is to show that the classical halting problem admits a black hole. In other words, there is a computable procedure to correctly decide almost every instance of the classical halting problem, with asymptotic probability one. The proof method is to observe that on fixed infinite input, a random Turing machine operates something like a random walk, up to the point where it begins to repeat states. And because of Polya's recurrence theorem, it follows that with probability as close as you like to one, the work tape head will return to the starting position and fall off the tape before repeating a state.
-My point now is that the same observation applies to your problem. For any particular fixed infinite input, the work tape head will fall off for almost all programs. Thus, almost every program sees only finitely much of the input before stopping.<|endoftext|>
-TITLE: Numbers in the "Fundamentalis Tabula Arithmetica"
-QUESTION [15 upvotes]: Does anyone know anything about the "Fundamentalis Tabula Arithmetica" or what could be special about the numbers 106 and 117 that would make Europeans in the 1600s want to know their multiples? Googling reveals nothing about a "Fundamentalis Tabula Arithmetica". Is the word "Fundamentalis" really Latin?
-
-From: Alex Bellos
-Date: Mon, Jul 11, 2011 at 11:01 AM
-Subject: [math-fun] tables
-To: math-fun
-Hi - I've just received an email from a reader who saw in a Cologne museum a document called Fundamentalis Tabula Arithmetica from 1638. It is a table of multiples for all the numbers up to 100 and also the tables for these five numbers:
-106, 117, 256, 318 and 365
-It's pretty obvious why 256 and 365 are included.
-Might anyone have any thoughts as to why 106, 117 and 318 (3x106) are also included?
-Alex
-
-REPLY [6 votes]: Regarding 318, Vincent Forest Hopper writes on p.75 of "Medieval Number Symbolism" (Dover):
-"...various of the Church Fathers began to write figurative interpretations of the biblical texts. They found precedent for giving importance to numbers in the precise directions given for the dimensions of the tabernacle, and in the testimony of the Book of Wisdom that 'God has arranged all things in number and measure.' Early interpretation of scriptural numbers is concerned only with the most prominent of them, such as the 12 springs and 70 palm trees of Elim, and the 318 servants of Abraham..." Something of the attitude of gnosis; that is, of scriptural mysteries hidden from the layman, is to be seen in an interpretation of Barnabas: 'Learn then, my children, concerning all things richly, that Abraham, the first who enjoined circumcision, looking forward in spirit to Jesus, practised that rite having received the mysteries of the three letters. For [the Scripture] saith, 'And Abraham circumcised 10, and 8, and 300 men of his household.' What, then, was the knowledge given to him in this? Learn the 18 first and then the 300. The 10 and 8 are thus denoted. Ten by I and eight by H. You have [the initials of the name of] Jesus. And because the cross was to express the grace [of our redemption] by the letter T, he says also 300. No one else has been admitted by me to a more excellent piece of knowledge than this, but I know that ye are worthy.'"
-Added:
-The moral here is that the medieval mind set is so very different from what we can imagine that it's hard to guess why they thought individual numbers were significant. For more on 318, see en.wikipedia.org/wiki/Dispute_about_Jesus'_execution_method#Interpretation_as_cross<|endoftext|>
-TITLE: Rings with finitely generated nilradical
-QUESTION [5 upvotes]: Let $\mathfrak{a}$ be a monomial ideal in a polynomial algebra over some commutative ring $R$. If $R$ is reduced, then the radical $\sqrt{\mathfrak{a}}$ of $\mathfrak{a}$ is again a monomial ideal, and if $\mathfrak{a}$ is moreover finitely generated then so is $\sqrt{\mathfrak{a}}$. If $R$ is not reduced, then the nilradical of $R$ is contained in $\sqrt{\mathfrak{a}}$ and may make the latter somewhat less easy to handle. However, if $\mathfrak{a}$ and the nilradical of $R$ are finitely generated then so is $\sqrt{\mathfrak{a}}$.
-This leads to the following question:
-
-Is there a nice description of rings $R$ such that the nilradical of $R$ is finitely generated?
-
-Or in more geometric terms:
-
-Is there a nice description of schemes $X$ such that the associated reduced scheme $X_{{\rm red}}$ is locally of finite presentation over $X$?
-
-REPLY [6 votes]: Good In a noetherian ring the nilradical, like any ideal, is finitely generated.
-Bad Given a ring $A$ and an $A$-module $M$ you can construct an $A$-algebra $R=A\ast M $ whose underlying $A$-module is $A\oplus M$ and in which the multiplication is given by $(a,m).(a',m')=(aa',am'+a'm)$.
-The first observation is that $M=0\ast M$ becomes an ideal of the ring $R$ satisfying $M^2=0$ and so definitely nilpotent.
-In particular if $A$ is reduced the nilradical of $R$ is exactly $M$.
-The second observation is that a subset $G\subset M$ generates $M$ as an ideal of the ring $R$ exactly when $G$ generates $M$ as an $A$-module.
-And now you can construct rings with as badly non-finitely generated nilradicals as you like, for example by taking for $M$ a free $A$-module of dimension a huge cardinal.<|endoftext|>
-TITLE: Unbased spectral sequences
-QUESTION [11 upvotes]: Suppose we have a tower of fibrations of spectra $\{X_k\}_{k\in\mathbb{N}}$ with inverse limit $X_\infty$, and let $F_k$ be the fibre of the map $X_k\to X_{k-1}$. There is then a spectral sequence $E^1_{jk}=\pi_j F_k \Longrightarrow \pi_{j+k} X_\infty$. If we instead have a tower of fibrations of based spaces, then we still have something like a spectral sequence except that some of the entries may be nonabelian groups or just pointed sets, and the sense in which $E^{r+1}$ is the homology of $E^r$ must be modified to take account of this. The details are in the book 'Homotopy limits, completions and localizations' by Bousfield and Kan. Now suppose we have a tower of fibrations of unbased spaces. I think I have heard it said that there is still some kind of spectral sequence building up to $\pi_\ast(X_\infty,a_\infty)$, where the basepoint $a_\infty$ is not given in advance but is chosen iteratively by lifting basepoints $a_n\in X_n$ as we work through the spectral sequence. This makes life difficult because $F_{n+1}$ and $\pi_\ast(X_n)$ are not defined until we have chosen $a_n$. Has any theory of this type been worked out in detail?
-
-REPLY [7 votes]: As requested, I am reposting this comment as an answer.
-Bousfield covers this material in "Homotopy spectral sequences and obstructions," Israel J. Math 66. The discussion is specific to cosimplicial objects (e.g. the discussion of obstruction cocycles in Section 5) and the general method of obtaining "partially" defined spectral sequences without basepoints would have to be extracted. However, I believe that all the necessary content is already there.<|endoftext|>
-TITLE: Is canonical class a topological invariant?
-QUESTION [13 upvotes]: For a $n$-dim smooth projective complex algebraic variety $X$, we can form the complex line bundle $\Omega^n$ of holomorphic $n$-form on $X$. Let $K_X$ be the divisor class of $\Omega^n$, then $K_X$ is called the canonical class of $X$.
-Question: Is homology class of $K_X$ in $H_{2n-2}(X)$ a topological invariant? If it's true, please tell me the idea of proof or some references. If not, please give me the counterexamples.
-
-REPLY [13 votes]: This answer is about the case of complex surfaces $X$ and their diffeomorphisms (all my diffeos are assumed to be orientation-preserving!).
-(1) Examples of self-diffeomorphisms that reverse the sign of the canonical class.
-Take $X=\mathbb{C}P^1\times \mathbb{C}P^1$. Let $\tau$ be reflection in the equator of $S^2=\mathbb{C}P^1$. Then $\tau \times \tau$ preserves orientation and acts as $-I$ on $H^2(X)$. It therefore sends $K_X$ to $-K_X$.
-One can also realise the automorphism $-I$ of $H^2(X)$ by a diffeomorphism when $X$ is the blow-up of the projective plane at $k$ points, $k = 2,3,\dots,9$. This follows from a result of C.T.C. Wall from
-Diffeomorphisms of 4-manifolds, J. London Math. Soc. 39 (1964) 131–140, MR0163323
-Wall says that if $N$ is a simply connected, closed oriented 4-manifold with $b_2(N)<9$, and $X$ is the connected sum of $N$ with $S^2 \times S^2$, then all automorphisms of the intersection form of $X$ are realised by diffeos. To apply this, recall that the 1-point blow-up of $\mathbb{C}P^1\times \mathbb{C}P^1$ is the 2-point blow up of the projective plane. (Wall's strategy, by the way, is to factor the automorphism into reflections along hyperplanes, and to realise those.)
-(2) Results from Seiberg-Witten theory.
-These results tie complex geometry amazingly closely to differential topology. They say that the unsigned pair $\pm K_X$ is invariant under diffeomorphisms (Witten http://arxiv.org/abs/hep-th/9411102 and others); so too is the Kodaira dimension; so too are the plurigenera (Friedman-Morgan http://arxiv.org/abs/alg-geom/9502026).
-In Kodaira dimension $<2$, one can take this further and prove that oriented-diffeomorphic surfaces are actually deformation-equivalent (to be safe, let me specify the simply connected case). But that's not the explanation in general: there are pairs of simply connected general-type surfaces that are diffeomorphic (by diffeos preserving the canonical class), which are not deformation-equivalent (Catanese-Wajnryb http://arxiv.org/abs/math/0405299).
-(3) How it happens.
-The Seiberg-Witten invariant (for an oriented 4-manifold with $b^+(X)>1$) is a map
-$$SW: Spin^c(X)\to\mathbb{Z}$$
-defined on the $H^2(X)$-torsor of $Spin^c$-structures. The overall sign is equivalent to a "homology orientation". It's natural under diffeomorphisms. It's also invariant under "conjugation" $\mathfrak{s}\mapsto \bar{\mathfrak{s}}$ of $Spin^c$-structures.
-For algebraic surfaces, there's a canonical spin-c structure $\mathfrak{s}$, so $Spin^c(X)$ is identified with $H^2(X)$. Witten (http://arxiv.org/abs/hep-th/9411102) observed that the elliptic equations that define $SW$ simplify drastically in the algebraic case; in evaluating $SW$ on a cohomology class represented by a complex line bundle $L\to X$, you're led to consider a moduli space of pairs consisting of a holomorphic structure on the line bundle and a holomorphic section of it, with an obstruction bundle on the moduli space. Conjugation-invariance becomes Serre duality.
-For general type surfaces, $\pm SW(\mathfrak{s}) = \pm SW(\bar{\mathfrak{s}}) = \pm 1$; all other spin-c structures have vanishing invariant. Since $c_1(\mathfrak{s})=-c_1(\bar{\mathfrak{s}})=-K$, one deduces diffeomorphism-invariance of $\pm K$. For lower Kodaira dimension, a more complicated analysis is needed.<|endoftext|>
-TITLE: Uniform approximation of $x^n$ by a degree $d$ polynomial: estimating the error
-QUESTION [7 upvotes]: The answer to this question should be well known, but it's a hard question to search for online.
-Suppose we want to approximate the function $x^n$ by a polynomial of degree $d$ in the $L_\infty$ norm on $[-1,1]$. What is a good estimate of the error of the best approximator, in terms of $n$ and $d$?
-I know this question was solved exactly by Chebyshev for $d = n-1$ (the error is $2^{-d}$ I think). The range of interest for me is $\sqrt{n} \leq d \leq n$ and I don't mind log factors in the estimate. Thus I would be happy to have an estimate for the error of the Chebyshev expansion truncated to degree $d$.
-(A bonus would be an answer to the same question for $(1-x^2)^d$.)
-Thanks!
-
-REPLY [8 votes]: For large $n$ and fixed $\epsilon > 0$ there is a polynomial of degree $d = O_\epsilon(\sqrt{n})$ that uniformly approximates $x^n$ to within $\epsilon$ on all of $[-1,+1]$. The polynomial can be taken to be the truncated Čebyšev expansion of $x^n$, as the original proposer (OP) suggested. As $\epsilon \rightarrow 0$, the $O_\epsilon$ constant grows only as $(\log(\epsilon^{-1}))^{1/2}$; for example, $d = 2.576 \sqrt{n}$ suffices to get $\epsilon = .01$ if I computed correctly.
-The OP wrote that truncating the Čebyšev expansion will give the correct $L^\infty$ distance to within a log factor. I don't see a priori why this should be, but fortunately the coefficients of the expansion of $x^n$ in Čebyšev polynomials turn out to be elementary and familiar enough to work with explicitly.
-It will be convenient to define $T_k(x)$ for all $k \in \bf Z$ as the polynomial such that $T_k(\cos u) = \cos ku$. Then $T_{-k} = T_k$ is a polynomial of degree $|k|$ satisfying $|T_k(x)| \leq 1$ for all $x\in [-1,+1]$. Now the Čebyšev expansion of $x^n$ is simply
-$$
-x^n = \frac1{2^n} \sum_{m=0}^n {n \choose m} T_{2m-n}(x),
-$$
-which can be checked by writing $x = \cos u = \frac12(e^{iu}+e^{-iu})$ and $T_k(x) = \frac12(e^{iku}+e^{-iku})$. So the coefficients form a binomial distribution, and truncating at degree $d$ eliminates only the tail of the distribution past $d^2/n$ standard deviations. Since each $|T_{2m-n}(x)| \leq 1$, this tail also bounds the truncation error for all $x \in [-1,+1]$, and we conclude that this error can be brought below any positive $\epsilon$ by making $d$ a large enough multiple of $\sqrt{n}$, as claimed.
-This might not be the optimal $L^\infty$ approximation (except for $d=n-1$, when its optimality is the result of Čebyšev that you quoted), but it's not too far, because it is the best $L^2$ approximation with respect to the Čebyšev measure $\pi^{-1} dx/ \sqrt{1-x^2}$, and the $L^\infty$ distance is at least as large as the $L^2$ distance. The $L^2$ distance can be computed from the sums of the squares of the coefficients in the tail.
-Much the same technique should work for $(1-x^2)^n$; indeed I see that while I was writing this Andrew posted an answer for $(1-x^2)^n$ that looks very similar to what I did for $x^n$.
-
-REPLY [5 votes]: For $P_n(x)=(1-x^2)^n$, large $n$ and $a>0$ it's possible to produce a polinomial of degree $a\sqrt{2n}$ with difference in $L_\infty$ less than $C(1-\mathrm{erf}\;a)$, where $C$ is an absolute constant. Taking any positive sequence $a(n)\to+\infty$ as $n\to\infty$ leads to $L_\infty$ norm converging to zero. For large $a$ we have $1-\mathrm{erf}\;a \sim e^{-a^2}/(a\sqrt{\pi})\;$. So to obtain the uniform $\varepsilon$ estimate on $[0,1]$ the degree $\sim C(\log \varepsilon^{-1})^{1/2}\sqrt n\ $ is enough.
-Namely, consider $P_n$ on the segment $[-1,1]$. Let $x=\sin y$. Now it is enough to approximate the function
-$$
-\sin^{2n}y =\sum_{k=0}^n c_n^k\cos 2ky
-$$
-on $[0,2\pi]$ by suitable trigonometric polynomials. Here $c_n^0=\frac1{4^{n}}{2n\choose n}$, $c_n^k =(-1)^{n-k}\frac1{2^{2n-1}}{2n\choose n+k}$, $k=1,\ldots,n$. For degree $2m<2n$ we'll take the polynomial
-$$
-Q_{2m}(x)=\sum_{k=0}^m c_n^k\cos 2ky.
-$$
-Then the $L_\infty$ norm
-$$
-\|P_{2n}-Q_{2m}\| {} \le \sum_{k=m+1}^{n} |c_n^k|=\frac1{2^{2n-1}}\sum_{k=m+1}^{n}{2n\choose n+k}.
-$$
-The last sum can be easily estimated since it is exactly the sum of the tails in the Bernoulli distribution with probability $p=1/2$ and $2n$ independent trials. For $m=[ a\sqrt{2n}]$ and large $n$ by the central limit theorem it is equal approximately to $2(1-\mathrm{erf}\;a)$. From here the above estimates follow.<|endoftext|>
-TITLE: Average over Random Permutations
-QUESTION [10 upvotes]: Consider $S_{n}$ the symmetric group and for each $\sigma\in S_{n}$ let $U_{\sigma}$ be its $n\times n$ permutation matrix. Let $A$ be an Hermitian $n\times n$ matrix. I'm interested in computing the average
-$$
-\mathbb{E}(A):=\sum_{\sigma \in S_{n}}{w(\sigma) U_{\sigma} A U_{\sigma}^{*}}
-$$
-where the $w(\sigma)$ are some positive weight adding up to one.
-For instance some natural weights are the ones coming from the Ewens's probability distribution of parameter $\theta>0$ on $S_{n}$ defined as
-$$
-w(\sigma)=\frac{\theta^{K(\sigma)}}{\theta(\theta+1)\ldots(\theta+n-1)}
-$$
-where $K(\sigma)$ is the number of disjoint cycles of $\sigma$. The case of $\theta=1$ is simply the uniform distribution on $S_{n}$. For the case $\theta=1$, it is known that
-$$
-\mathbb{E}(A)=\alpha \frac{ee^{T}}{n} + \Bigg(\frac{\mathrm{Tr}(A)-\alpha}{n-1}\Bigg)\Bigg(I_{n}-\frac{ee^{T}}{n}\Bigg)
-$$
-where $e$ is the vector $e^{T}=(1,1,\ldots,1)$ and $\alpha=\frac{e^{T}Ae}{n}$.
-My question are:
-
-Is there anything known about the averages for the more general case of $\theta>0$.
-Are there known asymptotics results as $n\to\infty$?
-
-REPLY [4 votes]: You can calculate this directly. All you need to know is the probability that $(\sigma(i),\sigma(j))=(k,l)$ for each pair $(k,l)$. Write $e_i$ for the $i$th basis vector, $v=\sum_i e_i$. Let $E = \mathbb{E}(A)$. Then:
-$$ e_i^T E e_i = \frac{\theta-1}{\theta+n-1} e_i^TAe_i + \frac{1}{\theta+n-1}\mathrm{Tr} A$$
-and
-\begin{equation*}
-\begin{split}
- &(\theta+n-1)(\theta+n-2)e_i^T E e_j\\\\
- &= v^TAv - \mathrm{Tr} A + (\theta-1) \left[e_i^TAv+v^TAe_j+e_j^TAe_i-e_i^TAe_i+e_j^TAe_j\right] + (\theta-1)^2 e_i^TAe_j
-\end{split}
-\end{equation*}
-More details: $E_{ij}$ is the average of $A_{\sigma(i)\sigma(j)}$ over $\sigma$. When $i=j$ you get $E_{ii} = p A_{ii} + \frac{1-p}{n-1}\sum_{k\neq i} A_{kk}$ where $p$ is the probability that $\sigma(i)=i$. To calculate $p$ note that when $\sigma(n)=n$, the restriction of $\sigma$ to $[1,n-1]$ has precisely one less cycle. It follows that $\sum_{\sigma(n)=n}w_n(\sigma) = \frac{\theta}{\theta+n-1} \sum_{\tau\in S_{n-1}} w_{n-1}(\tau)$ where $w_n$ is the weighing above on $S_n$.
-Similarly, for $i\neq j$ we have a sum over $A_{kl}$ with different weights. Easy cases include $k=i, l=j$ (this is $\frac{\theta^2}{(\theta+n-1)(\theta+n-2)}$ for the same reason as the diagonal), $k=j, l=i$ (this is $\frac{\theta}{(\theta+n-1)(\theta+n-2)}$ since now there is only one more cycle than the restriction to $[1,n-2]$), $k=i, l\neq i,j$ or $k\neq i,j, l=j$ (this is $\frac{\theta}{\theta+n-1}\left(1-\frac{\theta}{\theta+n-2}\right)$ since we fix one co-ordinate but not the other).
-A bit more difficult is the case $k=j, l\neq i,j$ and $i\neq i,j, l=i$. Again for $i=n, j=n-1$ this means that $n, n-1$ are consecutive on a cycle of length at least $3$. The probability that $n$ is on a cycle of length $3$ is the complement of the probability that it is on a cycle of length $1$ or $2$ (that is $\frac{\theta}{\theta+n-1} + (n-1)\frac{\theta}{(\theta+n-1)(\theta+n-2)}$. Given that, the successor to $n$ on the cycle is uniformly distributed, so the probability that $k=j, l\neq i,j$ is $\frac{1}{n-1} \left(1 - \frac{\theta}{\theta+n-1} + (n-1)\frac{\theta}{(\theta+n-1)(\theta+n-2)}\right)$.
-Finally, the probability that $k,l$ are distinct from $i,j$ is the complement of the above cases, and if so then the pair $k,l$ is uniformly distributed on the (n-2)(n-3) possibilities. It turns out each of these possibilities has probability $\frac{1}{(\theta+n-1)(\theta+n-2)}$.<|endoftext|>
-TITLE: Time-dependent Markov process: infinitesimal generator
-QUESTION [6 upvotes]: If one talks about homogeneous Markov diffusion
-$$
-\mathrm dX_t = \mu(X_t)\mathrm dt+\sigma(X_t)\mathrm dw_t
-$$
-with $\mu,\sigma$ sufficiently differentiable and of appropriate dimensions, there is nice equation for a function $m_f(x,t) = \mathsf E_x f(X_t)$ for $f\in C^2(\mathbb R)$:
-$$
-\begin{cases}
-\frac{\partial m_f}{\partial t} &= \mu\frac{\partial m_f}{\partial x}+\frac12\sigma^2\frac{\partial^2 m_f}{\partial x^2},
-\\
-m(x,0)&=f(x).
-\end{cases}
-$$
-On the other hand, while answering on the question Stochastic model I advised to use Fokker-Plank equation for the density of the process rather then the very same equation for $m_f$.
-The problem I had is the following. Since $\mu$ and $\sigma$ are time-dependent there, one can construct a process $Z_t = (X_t,Y_t)$ with $\mathrm dY_t = \mathrm dt$ and obtain everything for this process just using theory of homogeneous/time-independent Markov processes (how it is usually written in the books). Unfortunately, if you derive an infinitesimal generator for this process then you obtain
-$$
-\mathcal A_Zg(x,y) = \frac{\partial g}{\partial y}+\mu\frac{\partial g}{\partial x}+\frac12\sigma^2\frac{\partial^2 g}{\partial x^2}.
-$$
-For sure, now one shoud define $m_f(\tau|x,t) = \mathsf E_{x,t}f(X_{t+\tau})$ where $\tau\in \mathbb R_{\geq 0}$. If I am not wrong then it follows
-$$
-\frac{\partial m_f}{\partial \tau} = \frac{\partial m_f}{\partial t}+\mu\frac{\partial m_f}{\partial x}+\frac12\sigma^2\frac{\partial^2 m_f}{\partial x^2}\quad(*)
-$$
-which is kind of strange equation.
-My question has three parts:
-
-is an equation $(*)$ correct? if yes, are there developed methods for its solutions?
-if this equation is not correct, what is the right equation?
-I usually have problems when deal with non-homogeneous Markov processes since trick $Z_t = (t,X_t)$ does not help me. Could you refer me to literature where authors consider non-homogeneous Markov processes in details (rather then saying that this trick will help to use provided theory of homogeneous processes)?
-
-Link to MSE question: https://math.stackexchange.com/questions/51591/infinitesimal-generator-of-time-dependent-markov-diffusion
-
-REPLY [5 votes]: You can directly deal with inhomogeneous Markov processes through the Kolmogorov backward and forward equations. I suppose you are asking for the forward equation (i.e. derivative with respect to the time in the future). Let me discuss things on a formal level, which means here I ignore regularity conditions to ensure the existence of generators, derivatives, etc etc.
-Let me first give the backward equation, which is probably easier (a specific case of Feynman-Kac formula). Let $u(s,x,t)=\mathbb{E}^{s,x} f(X_t)$ for a nice function $f$ (e.g. infinitely differentiable with compact support), where the expectation is under the measure $\mathbb{P}^{s,x}$ such that $\mathbb{P}^{s,x}\{X_s=x\}=1$. Let $A_s$ be the generator (I am ignoring the detail of how $A_s$ is defined, you can probably figure it out yourself or see references I give below). Then the backward equation states that
-$\frac{\partial u}{\partial s}+A_s u =0$.
-The forward equation is usually formulated for the density $p(s,x,t,y)$ of the transition kernel $P_{s,t}(x,B)=\int_B p(s,x,t,y)dy$:
-$\frac{\partial p}{\partial t}=A_t^* p(s,x,t,y)$
-where $A_t^*$ is the adjoint of the operator $A_t$ (again, there's subtlety in how you definite $A_t$ for forward and backward equations separately, I ignore that here).
-For a given diffusion given as an SDE $dX_t=\mu(t,X_t)dt+\sigma(t,X_t)dW_t$ for a Wiener process $W$, it is pretty straightforward that $A_t f(x)=\mu(t,x)f'(x)+\frac{1}{2}\sigma^2(t,x)f''(x)$.
-All these are discussed to certain extent in "The Theory of Stochastic Processes, Vol II" by Gikhman and Skorokhod, and "Multidimensional Diffusion Processes" by Stroock and Varadhan (expressed in an integral form instead of derivatives).<|endoftext|>
-TITLE: What are the primitives of Lusztig's twisted bialgebra $\mathbf{f}$?
-QUESTION [8 upvotes]: I'm trying to understand the coproduct on Lusztig's $\mathbf{f}$ and, apart from the Chevalley generators, I don't know of any more primitive elements. In the $q=1$ case, the Weyl group acts as a Hopf algebra automorphism and maps the Chevalley generators to elements corresponding to the other simple roots, so those are also primitive; but when $q\neq1$, the braid group action is not a coalgebra automorphism, so image of the Chevalley generators under the braid group don't seem to be primitive. Are there more primitive elements, or do the Chevalley generators give a complete basis?
-
-REPLY [3 votes]: Suppose that $x\in \mathbf{f}$ and $r(x)=x\otimes 1+1\otimes x$. Recall $\mathbf{f}$ is graded by $\mathbb{N}I$, WLOG $x$ is homogenous, $x\in \mathbf{f}_\nu$. We know we get primitive elements if $\nu=i$ for some $i\in I$ so let us assume that $\nu$ is not of this form.
-Then $\mathbf{f}_\nu$ is generated by products of the form $yz$ with $y\in \mathbf{f}_\lambda$, $z\in \mathbf{f}_\mu$ with both $\lambda$ and $\mu$ nonzero. Now we consider the symmetric bilinear form on $\mathbf{f}$ and compute
-$$(x,yz)=(r(x),y\otimes z)=0$$
-since $x$ is assumed primitive.
-This shows that $x$ is orthogonal to all of $\mathbf{f}_\nu$. But the bilinear form is nondegenerate, so $x=0$. Thus the Chevalley generators give a basis of the primitive elements of $\mathbf{f}$.<|endoftext|>
-TITLE: Stalks of structure sheaf of fibre product?
-QUESTION [8 upvotes]: What can I say about it?
-Can I say the stalks equal the tensor products of the corresponding factors stalks?
-Thanks!
-
-REPLY [10 votes]: Let $X,Y$ be $S$-schemes. Then a point of $X \times_S Y$ corresponds to a pair of points $x \in X, y \in Y$ lying over the same $s \in S$ together with a prime ideal $\mathfrak{p} \subseteq \mathcal{O}_{X,x} \otimes_{\mathcal{O}_{S,s}} \mathcal{O}_{Y,y}$ which restricts to the maximal ideals in $\mathcal{O}_{X,x}$ resp. $\mathcal{O}_{Y,y}$. The stalk of the structure sheaf in this point is the localization of the tensor product:
-$\mathcal{O}_{X \times_S Y,(x,y,\mathfrak{p})} = (\mathcal{O}_{X,x} \otimes_{\mathcal{O}_{S,s}} \mathcal{O}_{Y,y})_{\mathfrak{p}}$.
-There are at least two ways to prove these statements: a) Use the universal property of $\text{Spec}(K)$ for a field $K$ to get the points and then use the universal property of $\text{Spec}(R)$ for a local ring $R$ to get their stalks. So this assumes, of course, that you already know that the fiber product exists, but you can recover the description of the elements and the stalks just by using the universal property! But actually, b) you can construct the fiber product as above, also more general in the category of locally ringed spaces. I've written this up here.
-Now your actual question seems to be:
-
-As Hartshorne chapter III.9.2 claim,an Ox-module (need not be quasi coherent) F's
- flatness is stable under base change. But the stalks is not the tensor products, how
- can I prove the claim?
-
-The statement is the following: If $f : X \to Y, Y' \to Y$ are morphisms, and $\mathcal{F}$ is a module over $X$ which is flat over $f$, then the pullback of $\mathcal{F}$ to $X \times_Y Y'$ is flat over $X \times_Y Y' \to Y'$. I am pretty sure that Hartshorne understands $\mathcal{F}$ to be quasi-coherent here. Otherwise the sketch of proof also does not make sense. But it is also true in general:
-Pick a point in $X \times_Y Y'$, thus a triple $(x,y',\mathfrak{p})$ as described above. Let $y$ be the underlying point in $Y$. Now $\mathcal{F}_{x}$ is flat over $\mathcal{O}_{Y,y}$. By commutative algebra (base change of flat modules), it follows that $\mathcal{F}_x \otimes_{\mathcal{O}_{Y,y}} \mathcal{O}_{Y',y'}$ is flat over $\mathcal{O}_{Y',y'}$. Again by commutative algebra (localizations are flat) $(\mathcal{F}_x \otimes_{\mathcal{O}_{Y,y}} \mathcal{O}_{Y',y'})_{\mathfrak{p}}$ is flat over $\mathcal{O}_{Y',y'}$. But this is exactly the stalk of the pullback of $\mathcal{F}$ in the given point $(x,y',\mathfrak{p})$.<|endoftext|>
-TITLE: If X is the coarse moduli space of the algebraic stack M, is there a nice description of Hom(_,X)?
-QUESTION [5 upvotes]: Let $\mathcal{M}$ be an algebraic stack, and let $X$ be its coarse moduli space (assume it exists as a scheme).
-We know that $h_X(Spec(k))=\mathcal{M}(Spec(k))$ if $k$ is algebraically closed. Is there anything intelligent we can say about $h_X(U)$ for a general scheme $U$?
-For example, can you come up with an algorithm for knowing what $h_X(U)$ is that would be considerably easier than constructing $X$?
-
-REPLY [5 votes]: What André says is absolutely correct. The functor represented by the moduli space does not have any reasonable description, except in very particular cases. Given a map $U \to X$, it can be hard work to decide whether it comes from an object of $\mathcal M(U)$.<|endoftext|>
-TITLE: What is the relationship amongst all the different kinds of spectra?
-QUESTION [11 upvotes]: The word "spectrum" gets tossed around a lot in mathematics, and there seem to be a number of different concepts to which it applies. There is of course a physical connotation to the word which is commonly associated with scattering processes, rainbows, etc. : http://en.wikipedia.org/wiki/Spectrum
-However, in mathematics the term seems to be quite loaded. To name a just a few concepts which could be called spectral, here is a big list:
-
-Eigenvalues/eigenvectors for normal matrices
-Jordan normal form for matrices
-Spectrum of a bounded linear operator
-Spectrum of a C* algebra
-Spectrum of a commutative ring / Scheme
-Characters of an abelian group
-Irreducible unitary representations of a group
-Spectrum of a graph
-Spectrum of a Riemannian manifold
-Spectral sequences from cohomology theory
-
-Of course some of these concepts are more general than others. For example, normal matrix < bounded linear operator < c* algebra < commutative ring. However, it doesn't seem like any one of these definitions is sufficiently encompassing to give the whole story. As a counter example, the irreducible representations of a non-commutative group are an instance of the Jordan normal form (for finite groups anyway), but are not really captured by the corresponding notion of the spectra of a commutative ring. Similarly, the spectrum of a graph and a Riemannian manifold don't seem to have much to do with the spectrum of schemes, but yet they are related to the spectra of linear operators. And then there are spectral sequences which are just a bit weird...
-I won't profess to completely understand the general idea here, but there do seem to be some patterns. A common theme seems to be `decomposability', for example when finding the eigenvalues/vectors of a matrix one attempts to split apart the domain of the matrix into independent components. This is similar to splitting a space into points; and suggests that there is perhaps a correspondence between eigenvectors/eigenvalues and prime ideals/local rings. The non-commutative picture is of course more complicated, but perhaps the concept might be equally described in terms of irreducible modules and some type of generalized localization at an irreducible module (which is a hazy concept I must admit). In a perfect world, it would be nice if this same concept could even extend to things like the Gauss map from differential geometry, or the Legendre transform from statistical mechanics/convex optimization (of course that might not be feasible).
-Also, I marked the question community wiki in case anyone else has some other good examples of `spectral' concepts in mathematics.
-
-REPLY [5 votes]: I'll extend my comment to an answer. Let $L$ be a complete lattice. Then a prime element of $L$ is an element p such that $a\wedge b\leq p$ implies $a\leq p$ or $b\leq p$. These elements are in bijection with maps from $L$ to the 2-element lattice preserving all sups and finite infs. For example, the prime elements of the lattice of ideals in a commutative ring are the prime ideals. If $A$ is a separable C*-algebra, then the prime elements of the lattice of closed 2-sided ideals are the primitive ideals (kernels of irreducible representations). If $A$ is commutative, these are the maximal ideals.
-The prime elements of a lattice $L$ form a space $spec(L)$ called the spectrum of $L$. The topology has as open sets the sets D(a) with $a\in L$ where $D(a)$ consists of all prime elements $p$ with $a\nleq p$. For example, if $L$ is the lattice of ideals, this is the Zariski spectrum. If $A$ is a separable C*-algebra, then the spectrum of the closed 2-sided ideal lattice is the primitive ideal spectrum. So many of your examples are spectra of lattices.
-Recall a space $X$ is sober if each irreducible closed subset has a unique generic point. Sober spaces are precisely the spectra of complete lattices. The proof $spec(L)$ is sober amounts to showing that the irreducible closed subsets are the complements of the sets D(p) with p prime and p is generic. Conversely, if X is sober, take the lattice of closed subspaces ordered by reverse inclusion. The prime elements are the irreducible closed subsets, which can be identified with points of X by taking generic points.<|endoftext|>
-TITLE: Grothendieck's Galois theory without finiteness hypotheses
-QUESTION [24 upvotes]: This is motivated by the discussion here. The usual definition of the etale fundamental group (as in SGA 1) gives automatic profiniteness: Grothendieck's formulation of Galois theory states that any category with a "fiber functor" to the category of finite sets satisfying appropriate hypotheses is isomorphic to the category of finite, continuous $G$-sets for a well-defined profinite group $G$ (taken as the limit of the automorphism groups of Galois objects, or as the automorphism group of said fiber functor). In SGA1, the strategy is to take finite etale covers of a fixed (connected) scheme with the fiber functor the set of liftings of a geometric point.
-One problem with this approach is that $H^1(X_{et}, G)$ for a finite group $G$ (which classifies $G$-torsors in the etale topology) is not isomorphic to $\hom(\pi_1(X, \overline{x}), G)$ unless $G$ is finite. Indeed, this would imply that the cohomology of the constant sheaf $\mathbb{Z}$ would always be trivial, but this is not true (e.g. for a nodal cubic).
-However, Scott Carnahan asserts on the aforementioned thread that the "right" etale fundamental group of a nodal cubic should be $\mathbb{Z}$, not something profinite.
-How exactly does this work? People have suggested that one can define it as a similar inverse limit of automorphism groups, but is there a similar equivalence of categories and an analogous formalism for weaker "Galois categories"? (Perhaps one wants not just all etale morphisms but, say, torsors: the disjoint union of two open immersions might not be the right candidate.) I'm pretty sure that the finiteness is necessary in the usual proofs of Galois theory, but maybe there's something more.
-
-REPLY [14 votes]: In "The pro-étale topology for schemes" (http://arxiv.org/abs/1309.1198), Bhatt and Scholze introduce the pro-étale fundamental group which seems to give a good answer to your question (see, e.g., Theorem 1.10). The pro-étale fundamental group also compares well against the usual étale fundamental group and the "SGA3 étale fundamental group".<|endoftext|>
-TITLE: symmetric integer matrices
-QUESTION [16 upvotes]: Suppose I have a symmetric positive definite matrix $M$ with integer entries. I want to decide whether $M = A A^t,$ with $A$ likewise integral. I assume that decision problem is NP-complete, as is the question of finding the $A$ even if an oracle tells you such an $A$ exists. Can someone provide a reference (I would very much like to be wrong about the hardness of the problem...)
-EDIT A remark: this question is equivalent to finding a collection of integral vectors (the columns of $A$) with prescribed distances (by the parallelogram law, the inner products give us the distances). If we require $A$ to be a $0-1$ matrix, I am pretty sure that this can encode knapsack, so is NP-complete. It seems that as per Will Jagy and Gerhard Paseman, this question (via Hasse-Minkowski) might only be as hard as factoring (which is generally conjectured to be less than NP-complete), but I haven't yet completely understood what is entailed in the Hasse-Minkowski approach...
-Further EDIT
-In fact, the Hasse local-to-global principle works fine for small dimensions, since the class number of identity equals one in that case, and one can enumerate solutions by the
-Smith-Minkowski-Siegel mass formula. This apparently works only in dimension at most eight. This gives the oracle (the wiki article cited seems to imply that the right hand side can be computed in polynomial time, though I am none-too-certain of this), so this gives the required oracle in small dimensions, though not obviously an algorithm for finding solutions. In dimensions greater than eight we seem to be sunk.
-
-REPLY [12 votes]: A minor observation: $\left( \begin{smallmatrix} N & 0 \\ 0 & N \end{smallmatrix} \right)$ is of the form $A A^T$ if and only $N$ is of the form $a^2+b^2$. Specifically, if $N=a^2+b^2$ then $\left( \begin{smallmatrix} N & 0 \\ 0 & N \end{smallmatrix} \right) = \left( \begin{smallmatrix} a & b \\ -b & a \end{smallmatrix} \right) \left( \begin{smallmatrix} a & -b \\ b & a \end{smallmatrix} \right)$. The converse is left as an exercise.
-So this problem is at least as hard as determining whether or not an integer is a sum of two squares.
-We discussed the complexity of determining whether an integer is a sum of two squares here. Nobody really knew the answer, but it seemed that it might be as hard as factoring.<|endoftext|>
-TITLE: Edge Covering Shortest path
-QUESTION [5 upvotes]: I have a graph of Points G(V,E) and I want to find the shortest path covering all the edge, I want the minimum number of edge repetitions .
-Which is the best way to reduce this problem to well know problems like TSP , Hamiltonian circuit , Hamltonian completion ?
-Thank you.
-
-REPLY [5 votes]: That is Chinese Postman Path. Search for Chinese Postman Problem...
-E.g., this section from some book looks comprehensive: http://ie454.cankaya.edu.tr/uploads/files/Chp-03%20044-064.pdf<|endoftext|>
-TITLE: Haar measures in Solovay's model
-QUESTION [18 upvotes]: Haar measure is a measure on locally compact abelian groups which is invariants to translations. For example, the Lebesgue measure on the reals is such measure.
-It can be shown without the use of the axiom of choice that the Haar measure exists and it is unique up to a scalar, that is if we want the measure of the unit interval (for example) to be $1$ then it is really unique.
-While the measure is defined on Borel subsets, but we can complete the measure in a unique way by adding all the subsets of measure zero sets (and in the case of the real numbers we once again have the Lebesgue algebra)
-As with the Lebesgue measure, when the axiom of choice is present there are cases in which non-measurable sets can be constructed. In the Solovay model, however, we have that all subsets of reals are measurables.
-Are there any similar results about Haar measures of general LCA groups? Is there a model in which all Haar measures (perhaps under some limitations on the groups) are "full measures" (in the sense that every subset is measurable)?
-
-REPLY [5 votes]: Solovay's model will have all subsets measurable as long as the group is (locally compact) metrizable. For "big" groups (non-metrizable) the Haar measure is naturally defined on the Baire sets (the least sigma-algebra so that all continuous real-valued functions are measurable), and extension even to the Borel sets may not be unique.<|endoftext|>
-TITLE: Rational map having a birational restriction.
-QUESTION [7 upvotes]: Let $\sigma\subset |\mathcal{O} _{\mathbb{P} _{\mathbb{C}}^N}(2)|$ be
-an $N-$dimensional linear systems of quadrics on $\mathbb{P} _{\mathbb{C}}^N$ ($N\geq 4$),
-$F:\mathbb{P} _{\mathbb{C}} ^N\dashrightarrow \mathbb{P} _ {\mathbb{C}}^ N$ the rational map associated to $\sigma$,
-and $Q\in \sigma$ a fixed smooth quadric
-such that $F|_{Q}:Q\dashrightarrow \mathbb{P} _{\mathbb{C}}^{N-1}$ is birational.
-Is $F$ birational?
-
-REPLY [2 votes]: Nice question. I think that the answer is yes and more general. I have never seen this but seems natural.
-${\bf Lemma}$ Let $f\colon \mathbb{P}^n_{\mathbb{C}}\to \mathbb{P}^n_{\mathbb{C}}$ be a rational map of degree $d$, given by $(x_0:\dots:x_n)\to (f_0:\dots:f_n)$, where the $f_i$ are homogeneous of degree $d$.
-Suppose that the hypersurface $H\subset \mathbb{P}^n_{\mathbb{C}}$ given $f_0=0$ is irreducible and that the map from $H$ to $\mathbb{P}^{n-1}_{\mathbb{C}}$ given by the restriction (i.e. $(x_0:\dots:x_n)\to (f_1:\dots:f_n)$ on $H$) is birational.
-Then, the map $f$ is birational.
-${\bf Proof}$ Let $\sigma$ be the linear system associated, which corresponds to hypersurfaces of $\mathbb{P}^n$ of equation $\sum_{i=0}^n a_i f_i=0$. The restriction being birational, the intersection of $n-1$ general elements of $\sigma$ with $H$ give one mobile point, and a non-mobile part that we call $R$. Since every member of $\sigma$ contains $R$ and because elements have all the same degree, the intersection of $n$ general elements of $\sigma$ gives $R$, plus exactly one mobile point (the last part follows from Bézout). This shows that $f$ is birational.
-${\bf Remark:}$ We could also view $R$ as a set of points, curves,... with some multiplicities and use intersection form on the blow-up.<|endoftext|>
-TITLE: Do etale maps satisfy the following?
-QUESTION [6 upvotes]: Let $X\rightarrow Y$ be a finite etale map. Let $R$ be a strict henselian ring with residue field $k$. Say that we have a map $Spec(k)\rightarrow X$ and so also $Spec(k)\rightarrow Y$. Assume that the map $Spec(k)\rightarrow Y$ factors thusly: $Spec(k)\rightarrow Spec(R)\rightarrow Y$. Then the question is: would there be (a unique?) map $Spec(R)\rightarrow X$ that would make this commute?
-This would be a little cleaner if I knew how to do commutative diagrams in mathoverflow, but hopefully you get the picture. Intuitively, this means that if you have a point on $Y$ and a point above it on $X$, and if you have a "path" (I use this word very loosely here) near that point on $Y$ then it extends to a "path" near the respective point on $X$. When trying to prove this, the first thing I thought about is that every etale map is formally etale, but in the definition of formally etale they only talk about lifting $1^{st}$ order deformations.
-Do you know how to prove (or god forbid disprove) this?
-
-REPLY [10 votes]: I think this is true. Namely, we can lift each $\mathrm{Spec}(R/\mathfrak{m}^n)\to Y$ to $X$ (because formal etaleness allows the lift to exist for any nilpotent thickening) successively so as to be compatible with all the previous ones (in fact, it has to be, by uniqueness in the lifting property). This successive lifting gives a map $\mathrm{Spec} R \to X$ lifting $\mathrm{Spec} R \to Y$ (to see this, note that without loss of generality, $X, Y$ are affine, and then taking direct limits in the category of affine schemes is the same as taking inverse limits in the category of rings). Thus we get a map from the completion $\\mathrm{Spec} \hat{R}$, hence from $\mathrm{Spec} R$.
-I believe this is true even if we just assume $X \to Y$ to be smooth, since then we can still make successive liftings. Note that finiteness of the map $X \to Y$ is not necessary here. This argument seems to require modification because it is not obvious that the map will factor through the henselianization.
-For another argument, note that we can reduce to the case where $Y$ is $\mathrm{Spec} R$ (by making a base-change via $\mathrm{Spec} R \to Y$). Then we have to show that there is a section $\mathrm{Spec} R \to X$: this is clear because $X$ is a product of strictly henselian rings finite and etale over $R$, and if one of them also has residue field $k$, it must be isomorphic to $\mathrm{Spec} R$. Here one uses the fact that a finite etale morphism of local rings with the same residue field must be an isomorphism, which follows from Nakayama's lemma and since flatness implies injectivity (for local rings).
-(P.S. Just in case this was the question, recall the nilpotent lifting property for an etale morphism $X \to Y$: given any scheme $S$ and subscheme $S_0$ cut out by a nilpotent ideal (one can reduce to the square-zero case by induction), any diagram with $S_0 \to X$ and $S \to Y$ leads to a unique lift $S \to X$.)
-(P. P. S. One doesn't need finiteness even in the second argument: if $X \to Y$ is an etale morphism with $Y$ the spectrum of a henselian ring $R$, then the local ring of a point of $x$ lying above the closed point of $Y$ is finite over $R$, by Zariski's Main Theorem: one characterization of henselian rings is that a quasi-finite local algebra over a henselian local ring is finite.)<|endoftext|>
-TITLE: Spinor space to Euc. vector space: does there exist a universal bilinear map?
-QUESTION [7 upvotes]: Let $S$ be a spin representation of the Euclidean
-spin group $Spin(d)$ and let ${\mathbb R}^d$
-be Euclidean $d$-space with $Spin(d)$ action on it
-in the canonical way, via the 2:1 cover to $SO(d)$.
-(I am being careful here: A spin rep.'', notthe spin rep.'')
-Is there, for all $d$, an onto quadratic $Spin(d)$-equivariant map $S \to {\mathbb R}^d$?
-If so, is there a `universal' ($d$-independent) construction of this map?
-MOTIVATION: For $d=2, 3$ I know these maps.
-They are famous in celestial mechanics and yield the
-standard regularizations of the Kepler problem,
-or, what is the same, of binary collisions in the classical N-body problem.
-They turn Kepler for negative energies into a harmonic oscillator.
-Case $d=2$. I take $Spin(2)$ to also be $S^1$, but wrapped `twice' around
-$S^1 = SO(2)$. $S = {\mathbb C}$. The quadratic map is $w \to w^2$.
-This is the Levi-Civita regularization.
-Case $d= 3$. This is the standard Hopf map ${\mathbb C}^2 \to {\mathbb R}^3$,
-or if you prefer, from the quaternions ${\mathbb H }$ to ${\mathbb R}^3$, sending $q$ to $q k \bar q$.
-The astronomers call this Kuustanheimo-Steifel regularization.
-WHERE I'VE LOOKED SO FAR: I tried to make sense out
-of Deligne's discussion on spinors in the AMS two-volume set
-from some Princeton year on string theory from a decade or so ago.
-I understand that over the complexes, there is either exactly one or exactly two
-spin representations, depending on the parity of $d$.
-So even there , we don't get a `universal' d-dependent map.
-Over the reals things decompose in a rather complicated
-dimension dependent way (mod 8 probably) and there is no clear choice.
-I also looked in Reese Harvey's book which I find too baroque and
-signature depend to penetrate.
-Case $d=4$. Here I am not sure. But I know $Spin(4) = SU(2) \times SU(2)$
-which I can think of as two copies of the unit quaternions, each acting on
-``its own'' ${\mathbb H}$.
-I guess in this case I better take $S = {\mathbb H} \times {\mathbb H}$
-Then I get the desired quadratic map ${\mathbb H} \times {\mathbb H} \to {\mathbb H} = {\mathbb R}^4$
-as $(q_1, q_2) \mapsto q_1 \bar q_2$.
-
-REPLY [9 votes]: The answer is no.
-For $Spin(5)\simeq USp(4)$, the spin representation of $Spin(5)$ is the defining representation $\mathbf{4}$ of $USp(4)$. The tensor product $\mathbf{4}\otimes \mathbf{4}$ decomposes as $$\mathbf{1}\oplus\mathbf{5}\oplus\mathbf{10}.$$
-The vector representation of $SO(5)$ is this $\mathbf{5}$, but it's in the antisymmetric part of this tensor product decomposition:
-$$\wedge^2\mathbf{4} = \mathbf{1}\oplus\mathbf{5}$$
-The $\mathbf{1}$ is there, because of the definition of $USp(4)$.
-So, there's no quadratic $Spin(5)$-equivariant map $\mathbf{4}\to\mathbf{5}$.<|endoftext|>
-TITLE: System of weights for nilpotent Lie algebras
-QUESTION [5 upvotes]: I am studying nilpotent Lie algebra theory. The subject is really new to me and I am studying by myself. I'd love your help with this.
-Let $\mathfrak{n}$ be a finite-dimensional nilpotent Lie algebra (over an algebraically closed field of characteristic zero) and let $\operatorname{Der}(\mathfrak{n})$ be the algebra of derivations of $\mathfrak{n}$. The system of weights of $\mathfrak{n}$ is defined as being that of the natural representation of a "maximal torus" $T$ in $\operatorname{Der}(\mathfrak{n})$ and the $\operatorname{rank}$ is the dimension of $T$. By remarkable result due to Gabriel Favre (see [F]), it is known that for a fixed integer $n$ there are finitely systems of weights. Let $T$ be a system of weights, we denote by $\mathrm{N}(T)$ the class of those Lie algebras having the system of weights $T$.
-My questions are:
-
-For a fixed integer $n$, are these system of weights classified?
-For a fixed integer $n$, can rank-one system of weights explicitly written?
-Are classified rank-one system of weights $T$ such that $\sharp\mathrm{N}(T)=1$
-Is there a good book or resource for learning about this topic and in general, about nilpotent Lie algebras (over $\mathbb{C}$ or $\mathbb{R}$)?
-
-Any help is much appreciated!
-[F] Favre, G.: Système de poids sur une algèbre de Lie nilpotente. Manuscripta Math. 9 (1973), 53-90.
-
-REPLY [4 votes]: 1.) No, the weight systems are not classified in general. However, the weight systems for complex nilpotent Lie algebras of dimension $n\le 7$ have been computed by Roger Carles, in "Weight systems for complex nilpotent Lie algebras and application to the varieties of Lie algebras", in $1996$. Another reference, in addition to Pasha's references, is this paper of Magnin of $1998$.
-2.), 3.) I think no.
-4.) There are several books (e.g., by Goze, Onischik, Vinberg, etc.). Also, several articles discuss nilpotent Lie algebras in general, e.g. the article by E. M. Luks, What is a typical nilpotent Lie algebra ? For faithful representations of nilpotent Lie algebras, see, for example, here.<|endoftext|>
-TITLE: Conjugacy problem for small braid groups
-QUESTION [5 upvotes]: The conjugacy problem for braid groups $B_3$ and $B_4$ can be solved in polynomial time, it is noted in the paper by Birman, Ko and Lee(2001).
-That was a result in 2001. Are there any new results on other small braid groups? Is the conjugacy problem in $B_5$ solvable in polynomial time?
-Also, I'm still curious on exactly how fast the conjugacy problem in $B_3$ can be solved(polynomial time is too broad). The only paper that described such algorithm is behind a paywall.
-
-REPLY [3 votes]: This 2008 paper (freely available) explains the fastest known solution of the conjugacy problem in $B_3$. I think the polynomial time complexity of that solution can be easily extracted from that paper.<|endoftext|>
-TITLE: Why is there such a close resemblance between the unitary representation theory of the Virasoro algebra and that of the Temperley-Lieb algebra?
-QUESTION [41 upvotes]: For those who aren't familiar with the Virasoro or Temperley-Lieb algebras, I include some definitions:
-• The (universal envelopping algebra of the) Virasoro algebra is the $\star$-algebra $Vir_c$ generated by elements $L_n$, ($n \in \mathbb{Z}$), subject to the relations
-$$
-[L_m,L_n]=(m-n)L_{m+n}+\frac{c}{12}(m^3-m)\delta_{m+n,0},
-$$
-and with $\star$-structure $L_n^* = L_{-n}$.
-• The Temperley-Lieb algebra is the $\star$-algebra $TL_{\delta}$ with generators $U_i$ ($i \in \mathbb{Z}$) and relations :
-
-$U_i^2 = \delta U_i$ and $*$-structure $U_i^* = U_i$.
-$U_iU_{i+1}U_i=U_i$ and $U_iU_{i-1}U_i=U_i$
-$U_i U_j=U_j U_i$ for $|i-j|\ge 2$
-
-Both $Vir_c$ and $TL_{\delta}$ depend on a parameter.
-These are the numbers $c$ and $\delta \in \mathbb{R}$.
-
-Let's call a representation $\rho$ of a $\star$-algebra on a Hilbert space unitary if $\rho(x^*)=\rho(x)^*$.
-We are interested in the unitary representations of $Vir_c$ and $TL_{\delta}$. In the case of the Virasoro algebra, we further restrict to positive energy representations, i.e., unitary representations in which the spectrum of $L_0$ is positive.
-Depending on the value of the parameters $c$ or $\delta$, three things can happen:
-1. Discrete series (only) of quotient of Verma modules are unitary and positive energy.
-2. Continuum of Verma modules are unitary, positive energy representations.
-3. The Verma modules are not unitary.
-Now here's the striking thing:
-$\begin{array}{c|c|c|c|c|c|c}
- & \text{Discrete series} & \text{Continuum} & \text{Others} \newline
- \hline
-Vir_c & c \in \{ 1-\frac{6}{m(m+1)} \vert m = 2,3,4 \ldots \} &c \in [1,\infty) & \text{non-unitary} \newline
- \hline
-TL_\delta & \delta\in \{ 2\cos\big(\frac\pi m\big)\quad \vert \quad m = 2,3,4 \ldots \} &\delta \in [2,\infty) & \text{non-unitary}
-\end{array}$
-The parameters $c$ and $\delta$ belong to a countable set (discrete series) exhibiting an accumulation point, followed by a continuum.
-
-
-Is it pure coincidence that those two algebras exhibit such similar behaviour?
-Is there some natural map from $Vir_c$ to $TL_{\delta}$, or vice-versa?
-Is there any way of linking the values $c\in 1-\frac{6}{m(m+1)}$ and $\delta\in 2\cos(\frac\pi m)$?
-Are there other algebras exhibiting a similar phenomenon?
-
-REPLY [16 votes]: Overview of an explanation :
-Jones-Wassermann subfactors for the loop algebra :
-Let $\mathfrak{g} = \mathfrak{sl}_{2}$ be the Lie algebra, $L\mathfrak{g}$ its loop algebra and $\mathcal{L}\mathfrak{g} = L\mathfrak{g} \oplus \mathbb{C}\mathcal{L}$ the central extension :
- $$[X^{a}_{n},X^{b}_{m}] = [X^{a},X^{b}]_{m+n} + m\delta_{ab}\delta_{m+n}\mathcal{L}$$ with $(X^{a})$ the basis of $\mathfrak{g}$.
- The unitary highest weight representations of $\mathcal{L}\mathfrak{g}$ are $(H_{i}^{\ell},\pi_{i}^{\ell})$ with :
-
-$\mathcal{L} \Omega = \ell \Omega$ with $\ell \in \mathbb{N}$ the level, and $\Omega$ the vacuum vector.
-$i \in \frac{1}{2}\mathbb{N}$ and $i \le \frac{\ell}{2}$, the spin (related to the irreducible representation $V_{i}$ of $\mathfrak{g}$)
-
-Let $I \subset \mathbb{S}^{1}$ an interval, and $\mathcal{L}_{I}\mathfrak{g}$ the local Lie algebra generated by $(X^{a}_{f})$ with :
-
-$f(\theta) = \sum \alpha_{n}e^{in\theta}$ and $f \in C^{\infty}_{I}(\mathbb{S}^{1})$
-$X^{a}_{f} = \sum \alpha_{n}X^{a}_{n}$
-
-Let $\mathcal{M}_{i}^{\ell}(I)$ be the von Neumann algebra generated by $\pi_{i}^{\ell}(\mathcal{L}_{I}\mathfrak{g})$.
-We obtain the Jones-Wassermann subfactor :
-$$\mathcal{M}_{i}^{\ell}(I) \subset \mathcal{M}_{i}^{\ell}(I^{c})'$$ of index $\frac{sin^{2}(p\pi/m)}{sin^{2}(\pi/m)}$ with $m=\ell + 2$ and $p=2i+1$.
-Its principal graph is given by the fusion rules :
-$$H_{i}^{\ell} \boxtimes H_{j}^{\ell} = \bigoplus_{k \in \langle i,j \rangle_{\ell}}H_{k}^{\ell}$$ with $\langle a,b \rangle_{n} = \{c=\vert a-b \vert, \vert a-b \vert+1,... \vert c \le a+b , a+b+c \le n \}$
-Let $\mathcal{R}_{\ell}$ be the fusion ring generated.
-Temperley-Lieb case (with $\ell \ge 1$) :
-If $i=1/2$ then index=$\frac{sin^{2}(2\pi/(\ell+2))}{sin^{2}(\pi/(\ell+2))} = \delta^{2}$ with $\delta = 2cos(\frac{\pi}{\ell+2})$ and the principal graph is $A_{\ell+1}$.
-In this case, the subfactors are known to be completely classified by their principal graph.
-The subfactor planar algebra it generates is the Temperley-Lieb planar algebra $TL_{\delta}$.
-Jones-Wassermann subfactors for the Virasoro algebra :
-Let $\mathfrak{W}$ be the Lie algebra generated by $d_{n} = ie^{in\theta}\frac{d}{d\theta}$ and $\mathfrak{Vir} = \mathfrak{W} \oplus C \mathbb{C}$ its central extension:
- $$
-[L_m,L_n]=(m-n)L_{m+n}+\frac{C}{12}(m^3-m)\delta_{m+n,0},
-$$
-Its discrete series representations are $(H_{pq}^{m})$ with :
-
-$C\Omega = c_{m} \Omega$ with $c_{m}= 1-\frac{6}{m(m+1)}$ for $m=2,3,...$
-$L_{0} \Omega = h^{pq}_{m} \Omega$ with $h^{pq}_{m} = \frac{[(m+1)p-mq]^{2}-1}{4m(m+1)}$ with $1 \le p \le m-1$ and $1 \le q \le p $
-
-As for the loop algebra, there are $\mathfrak{Vir}_{I}$ and $\mathcal{N}_{pq}^{m}(I)$ generated by $\pi_{pq}^{m}(\mathfrak{Vir}_{I})$.
-We obtain the Jones-Wassermann subfactor :
-$$\mathcal{N}_{pq}^{m}(I) \subset \mathcal{N}_{pq}^{m}(I^{c})'$$ of index $\frac{sin^{2}(p\pi/m)}{sin^{2}(\pi/m)}.\frac{sin^{2}(q\pi/(m+1))}{sin^{2}(\pi/(m+1))}$.
-Its principal graph is given by the fusion rules :
-$$H_{pq}^{m} \boxtimes H_{p'q'}^{m} = \bigoplus_{(i'',j'') \in \langle i,i' \rangle_{\ell} \times \langle j,j' \rangle_{\ell + 1} }H_{p''q''}^{m}$$ with $p=2i+1, q=2j+1, p'=2i'+1, ..., m=\ell+2$
-Let $\mathcal{T}_{m}$ be the fusion ring they generate, it's an easy quotient of $\mathcal{R}_{\ell} \otimes_{\mathbb{Z}} \mathcal{R}_{\ell+1}$, with $\mathcal{R}_{\ell}$ the fusion ring obtained above for the loop algebra.
-Temperley-Lieb case (with $m \ge 3$) :
-If $(p,q) = (2,1)$, index$=\frac{sin^{2}(2\pi/m)}{sin^{2}(\pi/m)} = \delta^{2}$ with $\delta = 2cos(\frac{\pi}{m})$ and the principal graph is $A_{m-1}$.
-As above, the subfactor planar algebra is Temperley-Lieb $TL_{\delta}$.
-
-$\rightarrow$ We obtain the natural maps $c \leftrightarrow \delta$
- and $\mathfrak{Vir}_{c} \leftrightarrow TL_{\delta}$ that you
- expected.
-
-Generalizations for similar phenomenon :
-Here is a list of possibilities :
-
-take $i$ other than $1/2$ or $(p,q)$ other than $(2,1)$
-take $\mathfrak{g}$ other than $\mathfrak{sl}_{2}$
-take the continuous series
-take a $N$-super-symmetric extension of $\mathfrak{Vir}$ : $N=1$ for the Neveu-Schwarz and Ramond algebras.
-
-References :
-- V.F.R. Jones, Fusion en algèbres de von Neumann et groupes de lacets (d'après A. Wassermann), Séminaire Bourbaki, Vol. 1994/95. Astérisque No. 237 (1996), Exp. No. 800, 5, 251--273.
-- T. Loke, Operator algebras and conformal field theory for the discrete series representations of $\textrm{Diff}(\mathbb{S}^{1})$, thesis, Cambridge 1994.
-- S. Palcoux, Neveu-Schwarz and operators algebras I : Vertex operators superalgebras, arXiv:1010.0078 (2010)
-- S. Palcoux, Neveu-Schwarz and operators algebras II : Unitary series and characters, arXiv:1010.0077 (2010)
-- S. Palcoux, Neveu-Schwarz and operators algebras III : Subfactors and Connes fusion, arXiv:1010.0076 (2010)
-- V. Toledano Laredo, Fusion of Positive Energy Representations of LSpin(2n), thesis, Cambridge 1997, arXiv:math/0409044 (2004)
-- R. W. Verrill, Positive energy representations of $L^{\sigma}SU(2r)$ and orbifold fusion. thesis, Cambridge 2001.
-- A. J. Wassermann, Operator algebras and conformal field theory. Proceedings of the International Congress of Mathematicians, Vol. 1, 2 (Zurich, 1994), 966--979, Birkhuser, Basel, 1995.
-- A. J. Wassermann, Operator algebras and conformal field theory. III. Fusion of positive energy representations of ${\rm LSU}(N)$ using bounded operators. Invent. Math. 133 (1998), no. 3, 467--538.
-- A. J. Wassermann, Kac-Moody and Virasoro algebras, 1998, arXiv:1004.1287 (2010)
-- A. J. Wassermann, Subfactors and Connes fusion for twisted loop groups, arXiv:1003.2292 (2010)<|endoftext|>
-TITLE: Fractional Brownian motion and Laplacian
-QUESTION [6 upvotes]: Having read this link on math stackexchange, I would like to submit to your wisdom the following questions.
-Is it possible, mutatis mutandis, to repeat the same reasoning for a fractional Brownian motion?
-More specifically: a real valued Gaussian process $B^H:=\{B^H(t)\}_{t\geq 0}$
- in a probability space
- $(\Omega,\mathscr{F},\mathbb{P})$ is a fractional Brownian
- motion (fBm) with Hurst parameter $H \in(0,1)$ if for all $s,t\in \mathbb{R}_+$
-
-$B^H(0)=0$,
-$\mathbb{E}B^H(t)=0$,
-$\operatorname{Cov}[B^H(t),B^H(s)]=\frac{1}{2}
- \left(t^{2 H}+s^{2 H}-|t-s|^{2 H}\right)$.
-
-In addition, the It\^o formula for fBm is written as:
-$$
-f(B^H(t))= \displaystyle\int_0^t f'\left(B^H(s)\right)\, d B^H(s) + H \displaystyle\int_0^t f''\left(B^H(s)\right) s^{2H-1}\, ds.
-$$
-Taking the expectation in both sides of the above equality, we obtain:
-$$
-\mathbb{E}\left[f(B^H(t))\right]= H \displaystyle\int_0^t \mathbb{E}\left[f''\left(B^H(s)\right)\right] s^{2H-1}\, ds.
-$$
- I might continue as;
- changing the expectation by the conditional expectation $\mathbb{E}_x$ with respect to the event $\{X_0=x\}$ where $X(t)=B^H(t)+ x $, it follows:
-$$
-\mathbb{E}_x\left[f(X^H(t))\right]= H \displaystyle\int_0^t \mathbb{E}_x\left[f''\left(X^H(s)\right)\right] s^{2H-1}\, ds.
-$$
-And if we put
-$$
-m(x,t; H)= \mathbb{E}_x\left[f\left(X^H(t)\right)\right].
-$$
-We get:
-$$
-\displaystyle\frac{\partial}{\partial t} m(x,t; H)= H \, t^{2H-1}\displaystyle\frac{\partial^2}{\partial x^2 }m(x,t; H)
-$$
-Knowledge that fBm is not a semimartingale nor a Markov process except for cases $H=\frac{1}{2}$.
- I have some doubts about the last deduction.1.
-
-REPLY [4 votes]: Your construction has been carried out for much more general situations. See
-Baudoin, F., Coutin, L. Operators associated with a stochastic differential equation driven
-by fractional Brownian motions, Stoch. Proc. Appl. 117, 5, 550–574, 2007. ArXiv: math/0509511.<|endoftext|>
-TITLE: Can we make Buchberger's algorithm faster for a given ideal if we are allowed to vary the monomial order?
-QUESTION [7 upvotes]: Suppose we have a finite set of generators for an ideal $I \subset R := \Bbbk[x_1,\dotsc, x_n]$, where $\Bbbk$ is a field. If we choose a monomial ordering, then Buchberger's algorithm allows us to produce a Groebner basis for $I$. However, the size of the resulting Groebner basis can be enormous, and moreover can vary greatly depending on the monomial order chosen.
-Sometimes, we have a reason for desiring a monomial order independent of $I$. (E.g., for elimination, we need an order with certain characteristics of lex; if we want to see of two sets of generators give the same ideal, we obviously want to use the same monomial order for both of them.) However, there are times when we may want to find a Groebner basis for $I$ with respect to some monomial, and we don't really care which. This could be useful, for instance, if we want to find a monomial $\Bbbk$-basis for $R/I$, and thereby (assuming $I$ is homogeneous) calculate the Hilbert polynomial of $I$.
-
-Are there studies of algorithms and/or heuristics that design a monomial order based on the given generators of $I$ in an effort to produce a smaller Groebner basis for this particular ideal?
-
-Ideally, it might be possible to choose a monomial order that has a good chance of outperforming grevlex on this particular generating set. At the very least, there should be some sort of heuristics for which grevlex order to choose (i.e., how the variables should be ordered).
-
-REPLY [5 votes]: You might investigate Singular, a software package for algebraic polynomial computations.
-I know little about it, but it does implement a so-called
-Hilbert-driven Buchberger algorithm, which (somehow!) finds "an appropriately chosen fast"
-ordering of the monomials, specifically to circumvent the problem that "the performance of Buchberger's algorithm is sensitive to the choice of monomial order."
-Their documentation provides one example
-with a $100 {\times}$ speedup.
-
-This article by Manuel Kauers in Scholarpedia may help. Here are some quotes:
-
-Change of Ordering
-Some applications require Gröbner bases with respect to a particular ordering of the power products for which Buchberger's algorithm is not as efficient as for other orderings. In such situations it may be advantageous to first compute a Gröbner basis with respect to some ordering where Buchberger's algorithm runs faster and in a second step transform this Gröbner basis to a Gröbner basis for the desired ordering.
-Gröbner Walk
-Two different techniques for performing such a change of ordering are known. One is known as Gröbner walk. It is based on an interpretation of orderings as regions in a space. If two orderings correspond to regions which overlap, then a Gröbner basis for one of the orderings can be turned into a Gröbner basis for the other by calling Buchberger's algorithm on a small auxiliary problem for which it usually terminates quickly. When the regions for two orderings do not overlap, it is always possible to connect them by a path consisting of orderings where the regions of any two consecutive ones have an overlap. The transformation can then be done step by step, switching in each step to the next ordering on the path. [...]
-Linear Algebra
-The second technique uses linear algebra. If $G$ is a Gröbner basis for some ordering,
- then we have [...]
- Using this technique, [...], one can determine the elements of a Gröbner basis with respect to an ordering different from the ordering of $G$.
-
-See the article for more details and references.<|endoftext|>
-TITLE: How are mathematical objects defined from an ultrafinitist perspective?
-QUESTION [6 upvotes]: I remember attending a lecture given by an ultrafinitist who denied that curves are a set of points, he would only say that any particular point may or not be on the curve. Similarly for algebraic or analytic objects, the only way I know how to define them is as a set with operations on the elements of that set. Since ultrafinitists cannot use definitions with infinite sets, what sort of definitions do they use?
-
-REPLY [7 votes]: Point-set definitions are the most common modern way of defining a geometrical object like a line, but they're not the only way.
-In Euclid, lines and circles are primitives.
-The axiomatization of Euclidean geometry in Tarski 1999 has only points, betweenness, and congruence as primitives, and sets are not even referred to in the axioms except for the axiom of continuity, which basically says lines are Dedekind-complete. I don't imagine that ultrafinitists would even want this kind of continuity, so they'd probably leave this axiom out. (Even if you interpret the axiom as a sheaf of axioms over first-order formulas defining the relevant sets, that would be an infinite sheaf, which I think would be unacceptable.) Tarski's axioms, interpreted using classical logic, imply that there are infinitely many points, but classical logic isn't the appropriate logic for ultrafinitism. So I don't really see any reason to believe that there's any problem with doing an ultrafinitist geometry this way.
-As an example, take the unit circle in the Cartesian plane, defined by a first-order formula (not as a set). I can definitely prove to an ultrafinitist's satisfaction that it contains at least four points (i.e., there are at least four points satisfying that formula). Given a line through the origin O (defined by picking some point P outside the circle that the line goes through), the lack of the axiom of continuity means that it's probably not possible to give an ultrafinitist proof that it intersects the circle (i.e., that there is a point between O and P that satisfies the formula defining the circle). However, one can certainly prove that there exist a point on the line and a point on the circle such that the distance between them is no more than 0.0001. There will be points such that it's neither true nor false that the point is on both the line and the circle.
-Even if you're working in a system that has infinities, it's not necessary to describe geometry in terms of point sets. The original presentation of the surreal line didn't use sets, and the surreals are too big to be a ZFC set. In smooth infinitesimal analysis, a curve is not representable as a set of points.
-Tarski and Givant, 1999, http://citeseerx.ist.psu.edu/viewdoc/summary?doi=10.1.1.27.9012<|endoftext|>
-TITLE: n-th integral cohomology of a non-compact manifold of dimension n
-QUESTION [11 upvotes]: Let $M$ be a smooth connected oriented without boundary non-compact manifold
-of dimension n.
-Let $k$ be a principal ideal, e. g. the integers $Z$
-Let $H_n(M)$ and $H^n(M)$ be the homology and cohomology in degree n of $M$
-with coefficients in $k$.
-It is well-known that $H_n(M)=0$.
-Is $H^n(M)$ also trivial ?
-By the universal coefficient theorem for cohomology,
-$H^n(M)=Ext(H_{n-1}(M),k)$.
-Therefore if $k$ is a field, $H^n(M)=0$. I would like to know this answer over
-a principal ideal: $Z$.
-It is also known (Bredon's book) that $H_{n-1}(M)$ is without torsion.
-But I believe that $H_{n-1}(M)$ in the non-compact case, is a not a finitely
-generated $k$-module.
-Therefore I don't know if $H_{n-1}(M)$ is free, i. e. projective.
-Since there exists abelians groups without torsion, non-free, e. g. the rationals $Q$
-I suppose that this must be well-known.
-But I could not find a reference.
-
-REPLY [2 votes]: Maybe too late, and close to the Fernando's argument. Let $c$ be an infinite path going through each center of the $n$-simplex of a triangulation. Let $U$ be a neighbourhood of $c$, we can assume $U$ has the homotopy type of $[0,+\infty[$ and $\partial U$ is homeomorphic to $\mathbb R^{n-1}$. Then $V= \overline{X\setminus U}$ retracts on the $n-1$-skeleton, so $H^{n-1}(V)=0$ and we conclude by Mayer Vietoris
-$$H^{n-1}(U\cap V) \to H^n(X) \to H^n(U)\oplus H^n(V)$$<|endoftext|>
-TITLE: Vladimir Voevodskys 2002 ICM Lecture.
-QUESTION [6 upvotes]: Is Vladimir Voevodskys ICM lecture available in videotaped format somewhere?
-Strangely it is not at the IMU homepage (but Lafforgues is) http://www.mathunion.org/Videos/ICM2002/
-Was it not taped (Why not?)?
-If not is at least a transcript available somewhere?
-Is there a least video of the opening ceremony, and the Laudatio(s)?
-Are there any taped lectures of Voevodsky lecturing on Motivic Cohomology?
-
-REPLY [11 votes]: Some of Voevodsky videos are here:
-http://video.ias.edu/taxonomy/term/42
-http://www.mathnet.ru/PresentFiles/425/425.flv
-http://www.mathunion.org/Videos/ICM98/ICMs/vladimir_voevodsky.html
-http://claymath.msri.org/voevodsky2002.mov
-The last two talks, Algebraic Cycles and Motives and An Intuitive Introduction to Motivic Homotopy Theory, are perhaps the closest to what you look for. As for the last talk, the notes from it are available here:
-http://www.cwru.edu/artsci/phil/Voevodsky.pdf<|endoftext|>
-TITLE: From Zeta Functions to Curves
-QUESTION [41 upvotes]: Let $C$ be a nonsingular projective curve of genus $g \geq 0$ over a finite field $\mathbb{F}_q$ with $q$ elements. From this curve, we define the zeta function
-$$Z_{C/{\mathbb{F}}_q}(u) = \exp\left(\sum^{\infty}_{n = 1}{\frac{\# C(\mathbb{F}_{q^n})}{n} u^n}\right),$$
-valid for all $|u| < q^{-1}$. This zeta function extends meromophicially to $\mathbb{C}$ via the equation
-$$Z_{C / \mathbb{F}_q}(u) = \frac{P_{C / \mathbb{F}_q}(u)}{(1 - u) (1 - qu)}$$
-for some polynomial with coefficients in $\mathbb{Z}$ that factorises as
-$$P_{C/\mathbb{F}_q}(u) = \prod^{2g}_{j = 1}{(1 - \gamma_j u)}$$
-with $|\gamma_j| = \sqrt{q}$ and $\gamma_{j + g} = \overline{\gamma_j}$ for all $1 \leq j \leq g$. This last point tells us that $Z_{C / \mathbb{F}_q}(u)$ has a functional equation and satisfies a version of the Riemann hypothesis.
-What happens if we run this construction in reverse? What if we start with a set of numbers $\gamma_j$, $1 \leq j \leq 2g$, such that $|\gamma_j| = \sqrt{q}$, $\gamma_{j + g} = \overline{\gamma_j}$ for all $1 \leq j \leq g$, and such that the polynomial
-$$P(u) = \prod^{2g}_{j = 1}{(1 - \gamma_j u)}$$
-has coefficients in $\mathbb{Z}$? Is there a way of telling whether the function
-$$\frac{P(u)}{(1 - u) (1 - qu)}$$
-is the zeta function of some curve $C$? Furthermore, what is this curve exactly?
-A simple case of this is if we look at the function
-$$\frac{1 - au + qu^2}{(1 - u) (1 - qu)}$$
-for some $a \in \mathbb{Z}$ with $|a| \leq 2 \sqrt{q}$. How do we determine whether this function is the zeta function $Z_{C / \mathbb{F}_q}(u)$ of an elliptic curve $C$ over $\mathbb{F}_q$? If it is indeed equal to $Z_{C / \mathbb{F}_q}(u)$, what is the Weierstrass equation for $C$ (assuming $\mathrm{char}(q) \geq 5$)?
-
-REPLY [26 votes]: In your last paragraph, you suggest that you are particularly interested in the case of elliptic curves. This is much easier than the general case, which is addressed well by Torsten's answer.
-If $q$ is prime, and $|a| \leq 2 \sqrt{q}$, then there is always an elliptic curve with $\zeta$-function $(1-au+qu^2)/(1-u)(1-qu)$. This is the one dimensional case of Honda's theorem. I'll sketch the proof. As you will see, it uses some very sophisticated methods, and it will be very hard to make it effective.
-Sidenote: What happens when $q = p^k$ for $k>1$? Then there are $2 p^{k-1}-1$ ways to choose $a$ to be $0 \mod p$. When $a=0$, the elliptic curve must be supersingular. But there are only $\approx p/12$ supersingular elliptic curves over $\mathbb{F}_{p^k}$. So, once $2 p^{k-1}$ is much greater than $p/12$, there will be $a$'s which don't occur. To be honest, I am not clear what happens if you feed one of these $a$'s into Honda's theorem.
-Proof Sketch: Let $R$ be the ring $\mathbb{Z}[\phi]/(\phi^2 - a \phi + q)$. $R$ is an order in an imaginary quadratic field (the inequality $a^2-4q<0$ is used to show that this is an imaginary extension.) Let $K$ be the the fraction field of $R$ and let $H$ be its class field. Let $\mathfrak{p}$ be the ideal $(\phi)$ in $\mathcal{O}_K$. Since $\mathfrak{p}$ is principal, it splits in $H$; let $\mathfrak{q}$ lie over $\mathfrak{p}$. So $\mathcal{O}_H/\mathfrak{q} \cong \mathbb{F}_p$.
-Let $E$ be an elliptic curve with complex multiplication by $R$; then $E$ can be defined over $H$. ($E$ is only well defined up to a quadratic twist, at this point.) Take a model of $E$ over $\mathcal{O}_H$. Let $E_0$ be the fiber over $\mathfrak{q}$. Then $E_0$ is an elliptic curve over $\mathbb{F}_p$. One can show that the Frobenius acts on $E_0$ by $\pm \phi$ or $\pm \overline{\phi}$, where the bar is the automorphism of $R$ over $\mathbb{Z}$ coming from complex conjugation. By changing the quadratic twist, one can make sure the sign is $+$. Then the trace of the Frobenius is $\mathrm{Tr}(\phi)$, which is $a$, as desired.
-Sorry for making this so advanced, I don't know an easier way. I think you can find most of the tools I am using in Silverman's Advanced topics in the arithmetic of elliptic curves.<|endoftext|>
-TITLE: Exterior powers in tensor categories
-QUESTION [9 upvotes]: Let $\mathcal{C}$ be a cocomplete $R$-linear tensor category. Many notions of commutative algebra can be internalized to $\mathcal{C}$. For example a commutative algebra is an object $A$ in $\mathcal{C}$ together with morphisms $e : 1 \to A$ (unit) and $m: A \otimes A \to A$ (multiplication) satisfying the usual laws. The $n$-th symmetric power $\text{Sym}^n(X)$ of an object $X$ is the quotient of $X^{\otimes n}$ by identifying $x_1 \otimes ... \otimes x_n = x_{\sigma(1)} \otimes ... \otimes x_{\sigma(n)}$, so formally it is defined as a coequalizer of the $n!$ symmetries $X^{\otimes n} \to X^{\otimes n}$. Then $\text{Sym}(X) = \bigoplus_{n\geq 0} \text{Sym}^n(X)$ is a commutative algebra and in fact $\text{Sym}$ is left adjoint to the forgetful functor $\mathsf{CAlg}(\mathcal{C}) \to \mathcal{C}$.
-But now what about the exterior power $\Lambda^n(X)$? It is clear how to define $X^{\otimes n}$ modulo $x_1 \otimes ... \otimes x_n = \text{sgn}(\sigma) \cdot x_{\sigma(1)} \otimes ... \otimes x_{\sigma(n)}$ in this context, which one might call the anti-symmetric power $\mathrm{ASym}^n(X)$. The correct definition of the exterior power also has to mod out $$... \otimes a \otimes ... \otimes a \otimes ... = 0,$$
-for example because we want to have that $\Lambda^p(1^{\oplus n}) \cong 1^{\oplus \binom{n}{p}}$. But I have no idea how to internalize this to $\mathcal{C}$, even for $n=2$. The reason is that there is no morphism $X \to X \otimes X$ which acts like $a \mapsto a \otimes a$. Another idea would be to define $\Lambda(X)$ as a graded-commutative algebra object with the usual universal property, classifying morphisms $f$ on $X$ which satisfy something like $f(x)^2=0$, but again it is unclear how to formulate this in $\mathcal{C}$.
-If this is not possible at all, which additional structure on $\mathcal{C}$ do we need in order to define exterior powers within them? Is this some categorified $\lambda$-ring structure? This structure should be there in the case of usual module categories (over rings or even ringed spaces). Of course there is no problem when $2 \in R^*$, because then the exterior power equals the anti-symmetric power. The question was also discussed in a blog post.
-Here is a more specific (and a bit stronger) formulation: Is there some $R[\Sigma_n]$-module $T$, such that for every $R$-module $M$, we have that $T \otimes_{R[\Sigma_n]} M^{\otimes n} \cong \Lambda^n M := M^{\otimes n}/(... \otimes x ... \otimes x ...)$? Because then we could define $\Lambda^n X := T \otimes_{R[\Sigma_n]} X^{\otimes n}$ for $X \in \mathcal{C}$.
-Concerning the "hidden extra structure" in the case of modules: Let the base ring be $\mathbb{Z}$, or more generally a commutative ring $R$ in which $r^2 - r \in 2R$ for all $r \in R$; this includes boolean rings such as $\mathbb{F}_2$ and also $\mathbb{Z}/n$. If $M$ is an $R$-module, then there is a well-defined(!) homomorphism $M^{\otimes~ n-1} \to \text{ASym}^n(M), x_1 \otimes ... \otimes x_n \mapsto x_1 \wedge x_1 \wedge ... \wedge x_n$, and its cokernel is $\Lambda^n(M)$.
-Concerning non-linear tensor categories, I've asked here a similar question.
-
-REPLY [6 votes]: Deligne (Categories Tannakiennes, 1990, p165) defines it to be the image of the antisymmetrisation $a=\sum(-1)^{\epsilon(\sigma)}\sigma\colon X^{\otimes n}\rightarrow X^{\otimes n}$.<|endoftext|>
-TITLE: What would the best treatment of Gehring's lemma look like?
-QUESTION [10 upvotes]: In a course about elliptic regularity probably one sooner or later stubles into the reverse Holder inequalities, and has to introduce the Gehring lemma, which in one of its many versions improves a bit the regularity of a function, given the knowledge that such function already satisfies local bounds via another more integrable function (for example one bounds the integral of $f^2$ on any ball via a power of $f^{2-\epsilon}$ on the ball of double radius).
-Even though I saw a proof of this (by Giaquinta-Modica) and I "believe" the result intuitively, I have the feeling that I didn't catch yet the "juice" of it.. for example I don't have the intuition of the following
-1) how far it can be extended,
-or
-2) if the more extended/general versions are meaningful to teach (e.g. because one can find a clearer proof, or a more natural one, or because they can show connections with other subjects..).
-So I would like to ask you if you know how to put this result in a larger context, or about its connections with topics different than elliptic regularity. Also references to nice expositions of it are quite welcome!
-
-REPLY [11 votes]: I'm not sure about "the larger context", but let me try to dissect the proof a bit so that there will be no mystery left there.
-It runs upon 3 main ideas:
-1) The "lack of concentration implies better summability" principle. In the nutshell, it is the following. Assume that we have some positive integrable function $f$ with $\int_X f=1$ on some probability measure space and know that for every subset $E$ of measure $\delta$, we have $\int_E f\le c(\delta)$ where $c(\delta)\to 0$ as $\delta\to 0$. Then we can design an increasing function $\Psi$ that grows to infinity and depends on $c$ only such that $\int_X f\Psi(f)<+\infty$. Indeed, assume that $\int_X f=1$. Put $E_k=\{f>2^k\}$. Then $\mu(E_k)\le 2^{-k}$, so, assuming that $\Psi$ is doubling, $\int_X f\Psi(f)$ is comparable to is enough to ensure that $\sum_{k\ge 0} \Psi(2^k)2^k\mu(E_k)\approx \sum_{k\ge 0}\psi(2^k) \int_{E_k}f\le \sum_{k\ge 0}\Psi(2^k)c(2^{-k})$, so we can choose any $\Psi$ for which this series converges. If $c(\delta)=\delta^q$, we can take $\Psi(x)=x^{q-\varepsilon}$ bringing $f$ to some $L^p$ with $p>1$..
-This principle (or something like it) is useful in many settings. In general it often happens that we can substantially improve the integral bounds if we know a priori that the functions aren't concentrated on small sets or there are no strong correlations between their values. I'm not ready to give impressive particular examples, but once you digest the idea, you can easily both recognize and use it in other contests.
-2) The "multiscale trick". You want to show that $\int_E f$ is small compared to $\int_X f$ when $\mu(E)$ is small. Instead of going from $E$ to $X$ directly, you show first that there is a set $E_1$ of measure just slightly bigger than $E$ such that $\int_{E_1}f$ is noticeably bigger than $\int_E f$. Then you apply this to $E_1$ and so on until you reach $X$. If the path is long, the small improvement at each stage will result in a huge one in the end (another general principle that I wish our students could digest). So, if, say, we can always find $E_1$ such that $\mu(E_1)\le C\mu(E)$ and $\int_{E_1}f\ge a\int_E f$ with $a>1$ (provided that $\mu(E)$ is small enough, of course), we can get the power bound for the integrals over sets of small measure.
-This multiscale reasoning is the core of many results in modern analysis. It can be almost trivial like in our case or extremely sophisticated but the idea is always the same.
-3) The "geometric structure theorems". They are often presented as covering lemmas and density and maximal function theorems but what they really say is that arbitrary measurable sets in nice spaces can be viewed as finite unions of simple almost disjoint pieces (balls in $\mathbb R^n$, say). For the Gehring lemma, we just take a set $E$ of small measure and create the union of "low but noticeable density balls" surrounding it (for almost each point $x\in E$ we can find a ball $B$ such that the $\mu(B\cap E)\approx \gamma \mu(B)$ where $\gamma$ is some number between $0$ and $1$ we can choose to be whatever we want (provided that $\mu(E)<\gamma$).
-If $B$ is one such ball, we can estimate the square of the average of $\sqrt f$ over $B$ by $\gamma\int_{B\cap E} f+\int_{B\setminus E}f$. Since the first part is small compared to $\int_B f$ when $\gamma$ is tiny, the condition that these averages are comparable implies that second part should be comparable to the entire integral, so if we extend $E\cap B$ to the entire $B$ (multiplying the measure by $\gamma^{-1}$), we increase the integral some fixed number of times.
-That would be the end if we could make our balls truly disjoint and covering the entire $E$ (we would just take $E_1$ to be the union of our balls). Since it is not exactly the case, we have to work a bit more. We can use some standard covering lemma to pass to a sequence of "morally disjoint" balls. If, say, we have a doubling measure in a metric space, we can use Vitaly and say that we can find a set of disjoint balls $B$ in our family such that three times larger balls cover $E$.
-This still doesn't seem good enough because if we use $B$'s, we are in danger that they capture only a small part of the integral of $f$ over $E$ and if we use $3B$, they will overlap a lot, so we can count the same piece in the extended integral many times but by now we are pretty convinced that the things should work, so we can go over the steps and do the needed tune-up for this particular theorem.
-This tune-up can be done in infinitely many ways. I will do it in the way that allows to illustrate one more idea. Let us use the triple balls $B'=3B$. Let $I=\int_E f$, $\mu=\mu(E)$. Let $E_k$ be the set of points outside $E$ covered by exactly $k$ balls $B'$ and let $\mu_k=\mu(E_k)$, $I_k=\int_{E_k} f$. The above consideration show that $\sum_{k\ge 1}I_k\ge cI$ and the doubling amounts to $\sum_{k\ge 1}\mu_k\le C\mu$, whence we can find non-zero $\mu_k$ with $\frac{I_k}{\mu_k}\ge \varkappa \frac I\mu$ with $\varkappa=\frac cC$. But then adding $E_k$ to $E$, we shall increase the ratio $I/\mu^{\kappa}$. Now just notice that if $E$ is a fixed set of positive measure, then this ratio attains its maximum over all supersets of $E$ and we just showed that this maximum cannot be attained on a set of small measure but for sets of large measure, the ratio is trivially bounded.
-This "choose the extremal object" idea can also be found almost everywhere.
-Of course, the proof of the Gehring's lemma obtained this way is neither the shortest, nor the slickest. My only excuse for bringing it up is that you seem to be dissatisfied with the proof you know, so I took the liberty to build the argument from the a few standard blocks in modern analysis in the most straightforward way. I also avoided a few technicalities related to localization, but those are totally routine and only obscure the main ideas. I also took the liberty to "misinterpret" your request and talk about the "larger context" for the ideas in the proof rather than for the result itself. In a sense, I tried to show that the Gehring lemma is just a combination of 3-4 well-known and widely used principles and it is given a name just because this particular combination is used as a single block often enough to justify creating a formal reference instead of reproducing the proof every single time. Of course, once you realize what the building blocks are, you can freely play with them and get many other statements in the same style, which, IMHO, is what "catching the juice of a theorem" means. I leave it to others to tell where outside PDE this building block comes handy as a whole but I hope I convinced you that the basic steps of which it consists are ubiquitous.<|endoftext|>
-TITLE: A dual theory to the theory of currents?
-QUESTION [6 upvotes]: The k-currents are defined as dual space to the spaces of all smooth k-forms.
-(These monsters are used to work with the minimal k-surfaces.)
-Assume I want to look at the generalized k-forms;
-they can be defined as certain functionals
-on the space of all smooth k-surfaces with boundary.
-Did anybody consider such "generalized k-forms"?
-In fact I am looking for such generalized 2-forms on $\mathbb R^n$ with values $so(n)$,
-this should be some kind of generalized curvature.
-(Instead of integral I should consider parallel translation around the boundary.
-If $n=2$, it gives me a sign-measure.)
-But I am also interested in the linear case.
-
-REPLY [2 votes]: There is another notion of generalized form called charges. You can find its definition here :
-http://www.math.jussieu.fr/~depauw/preprints/dep-moo-pfe.pdf (they are functionals on the space normal currents with a distribution-like topology).
-Cochains can be represented as $L^1$ forms with $L^1$ exterior derivative, whereas charges are represented as $\omega + d \eta$, where $\omega$ and $\eta$ are continuous forms. Maybe this kind of generalized form is best suited for your purposes.<|endoftext|>
-TITLE: Mayer-Vietoris for sheaf cohomology
-QUESTION [6 upvotes]: Assume the following is given:
-
-$X$ quasi-projective (smooth) variety,
-$\overline X$ a projective variety, such that $X$ is the complement of a divisor $D$ in $X$, and
-$\mathcal F$ a coherent sheaf on $\hat X$.
-
-Suppose, we know the cohomology $H^\ast(X,{\mathcal F}|_X)$ of $\mathcal F$ on $X$. Now, for sure, this is not enough to be able to compute $H^{\ast}({\overline X},{\mathcal F})$.
-My question is what kind of additional data (localized near D) do we need to know in order to be able to compute $H^\ast({\overline X},{\mathcal F})$?
-
-REPLY [3 votes]: You need the cohomology of the extension of $\mathcal{F}$ to the completion of $X$ along $D$ (a formal scheme). There is an equivalence of categories that you can look up in
-Moret-Bailly, Laurent
-Un problème de descente. Bulletin de la Société Mathématique de France, 124 no. 4 (1996), p. 559-585 (numdam)
-see also references therein.
-From this description, a way of computing cohomology should follow.<|endoftext|>
-TITLE: Manifold with all geodesics of Morse index zero but no negatively curved metric?
-QUESTION [19 upvotes]: A closed oriented Riemannian manifold with negative sectional curvatures has the property that all its geodesics have Morse index zero.
-Is there a known counterexample to the "converse": if (M,g) is a closed oriented Riemannian manifold (Edit: assumed to be nondegenerate) all of whose geodesics have Morse index zero then M admits a (possibly different) metric g' with negative sectional curvatures?
-Edit: Motivation for asking this (admittedly naive) question is that Viterbo/Eliashberg have proved that a manifold with a negatively curved metric cannot be embedded as a Lagrangian submanifold of a uniruled symplectic manifold. Actually their proof only seems to use the existence of a nondegenerate metric all of whose geodesics have Morse index zero. I wondered if that was known to be strictly weaker.
-
-REPLY [12 votes]: As mentioned by Rbega the question should be amended to ask whether it's true that a closed manifold $M$ without conjugate points admits a metric of non-positive (rather than negative) curvature (otherwise a torus is an obvious counterexample).
-In that form this is a well-known open problem. The exponential map at any point is a universal covering of $M$ and the geodesics in $\tilde M$ are unique. This does show that $M$ is aspherical but that is a long way from admitting a metric of nonpositive curvature.
-There are some partial results suggesting that fundamental groups of manifolds without conjugate points share some properties of fundamental groups of nonpositively curved manifolds.
-In particular, there is a result of Croke and Shroeder that if the metric is analytic then any abelian subgroup of $\pi_1(M)$ is embedded quasi-isometrically. By the following observation of Bruce Kleiner the analyticity condition can be removed: Croke and Schroeder show that even without assuming analyticity for any $\gamma\in\pi_1(M)$ its minimal displacement $d_\gamma$ satisfies $d_{\gamma^n}=nd_\gamma$ for any $n\ge1$. This then implies that $d_\gamma=\lim_{n\to\infty} d(\gamma^nx,x)/n$ for any $x\in\tilde M$. This in turn implies that the restriction of $d$ to an abelian subgroup $H \simeq \mathbb Z^n$ extends to a norm on $\mathbb R^n$. This implies that $H$ is quasi-isometrically embedded.
-This result implies for example that nonflat nilmanifolds cannot admit metrics without conjugate points and more generally that every solvable subgroup of the fundamental group of a manifold without conjugate points is virtually abelian.
-But it's unlikely that any such manifold admits a metric of non-positive curvature. It is more probable that its fundamental group must satisfy some weaker condition such as semi-hyperbolicity but even that is completely unclear. The natural bicombing on $\tilde M$ given by geodesics need not satisfy the fellow traveler property (at least there is no clear reason where it should come from).
-So it might be worth trying to look for counterexamples and the first place I would look is among groups that are semi-hyperbolic but not $CAT(0)$. Specifically, any $CAT(0)$ group has the property that centralizers of non-torsion elements virtually split. This need not hold in a semi-hyperbolic group with the simplest example given by any nontrivial circle bundle over closed surfaces of genus $>1$. To be even more specific one can take the unit tangent bundle $T^1(S_g)$ to a hyperbolic surface. Note however that it's known that a closed homogenous manifold without conjugate points is flat so if there is a metric without conjugate points on $T^1(S_g)$ it can not be homogeneous.
-Edit: Actually, this last remark is irrelevant as $T^1(S_g)$ can not admit any homogeneous metrics at all.<|endoftext|>
-TITLE: Spectrum of an algebra object and Reconstruction of Schemes
-QUESTION [8 upvotes]: In "Au-dessous de $\text{Spec}(\mathbb{Z})$", Toen and Vaquié define schemes relative to a complete, cocomplete symmetric monoidal category $C$ using a functorial approach.
-In the introduction the authors mention that there should be a description of the underlying topological space $|\text{Spec}(A)|$ of an affine scheme ($A$ algebra object in $C$; always commutative) in terms of ideals, at least if $C$ satisfies some reasonable properties (which one?). But they do not elaborate this. The definition of $|X|$ in the paper can be found in the end of section 2.4 and is rather sketchy and indirect: The category of (functorial) Zariski opens in $X$ is equivalent to the category of open subsets of a topological space $|X|$ by some abstract result.
-Question 1. Is there any more concrete description of $|X|$, or at least of $|\text{Spec}(A)|$?
-If $C$ was also assumed to be abelian, I would define $|\text{Spec}(A)|$ just as follows: First, an ideal of $A$ is the kernel of a morphism of algebras $A \to B$. Equivalently, it is a subobject $I \subseteq A$ such that $I \otimes A \to A \otimes A \to A$ factors through $I$. It is clear how to define ideal sum and (finite) ideal intersection. If $I,J$ are ideals of $A$, then define the ideal product $I*J$ to be the kernel of $I \to I \otimes A/J$. A prime ideal is a proper ideal $\mathfrak{p}$ of $A$ such that for all ideals $I,J$ of $A$, we have $I * J \subseteq \mathfrak{p} \Rightarrow I \subseteq \mathfrak{p} \vee J \subseteq \mathfrak{p}$. For an ideal $I$, let $V(I)$ be the set of prime ideals $\mathfrak{p}$ satisfying $I \subseteq \mathfrak{p}$. Then as usual we get a topological space $|\text{Spec}(A)|$, the spectrum of $A$.
-Question 2. Has this definition already been studied somewhere?
-One way to "test" the above definition of the spectrum is to test if $\text{Spec}(\mathcal{O}_X)$ turns out to be $X$; here $X$ is a (nice) scheme and $\mathcal{O}_X$ is our algebra object in $C=\text{Qcoh}(X)$. Now it is not hard to check that we have an injective map $X \to \text{Spec}(\mathcal{O}_X)$ sending $x$ to the vanishing ideal of $\overline{\{x\}}$ and that for noetherian schemes $X$ (actually I only need that $\text{rad}(\mathcal{O}_X)^n=0$ for some $n$), this is an isomorphism. Again I don't know if this is well-known at all. I also wonder what happens if $X$ is more general, say quasi-compact and quasi-separated.
-But back to the general setting relative to $C$:
-Question 3. If we use the above definition of the spectrum of $A$, how can we define the structure sheaf?
-
-REPLY [9 votes]: Florian Marty studied this question in his thesis. The relevant chapter is available as arXiv:0712.3676 (otherwise, the thesis is available here). He describes the space $|\mathrm{Spec}(A)|$ as the set of prime ideals endowed with the Zariski topology. He also proves that a basis of this topology is given by the subspaces $|\mathrm{Spec}(A[f^{-1}]|$, from which one deduces easily the description of the structural sheaf of $\mathrm{Spec}(A)$.<|endoftext|>
-TITLE: Approximate primitive roots mod p
-QUESTION [10 upvotes]: Artin conjectured that if $a$ is an integer which is not a square and not $-1$ then $a$ is a primitive root for infinitely many primes. This conjecture has not been resolved, but partial results are known: Heath-Brown showed that there are at most two prime numbers $a$ for which the conjecture fails.
-I'd like to know if a different kind of partial result is known. Let $I(p)$ denote the index of the subgroup of $(\mathbf{Z}/p\mathbf{Z})^{\times}$ generated by 2. Thus $I(p)=1$ if and only if 2 is a primitive root mod $p$. Can one show that there is an infinite sequence of primes in which $I$ remains bounded?
-
-REPLY [8 votes]: A result of Erdos and Murty asserts that if $\epsilon(p)$ is any decreasing function tending to zero, then $I(p) \leq p^{1/2-\epsilon(p)}$ for almost all primes $p$ (i.e., all but $o(\pi(x))$ primes $p \leq x$).
-Kurlberg and Pomerance (see Lemma 20 in the paper mentioned below) show that for a positive proportion of primes $p$, one has the stronger bound $I(p) \leq p^{0.323}$. This follows from a result of Baker and Harman on shifted primes with large prime factors.
-The Erdos--Murty paper is #77 at
-http://www.mast.queensu.ca/~murty/index2.html
-and the Kurlberg--Pomerance paper is
-http://www.math.dartmouth.edu/~carlp/PDF/par13.pdf
-See also Theorem 23 of this paper (which is conditional on GRH).<|endoftext|>
-TITLE: Discrete-compact duality for nonabelian groups
-QUESTION [17 upvotes]: A standard property of Pontrjagin duality is that a locally compact Hausdorff abelian group is discrete iff its dual is compact (and vice versa). In what senses, if any, is this still true for nonabelian groups?
-I can guess what this means for a compact (Hausdorff) group $G$: the category of unitary representations of $G$ should be discrete in the sense that every one-parameter family of unitary representations consists of isomorphic representations, or something like that. Is this true? Is the converse true?
-I am less sure what this means for a discrete group $G$. What does it mean for the category of unitary representations to be compact? I suppose that $\text{Hom}(G, \text{U}(n))$ is a closed subspace of $\text{U}(n)^{G}$, hence compact, hence so is the appropriate quotient space of it...
-
-REPLY [3 votes]: Just an example, from which I learned a lot. Consider the free group $F$ on two generators. It makes a lot of sense to think of $n \mapsto hom(F,U(n))$ as some sort of dual. It comes with a natural conjugation action of $U(n)$ and natural operations of $\oplus$ and $\otimes$.
-However, if one considers the bi-dual, which in this context would be the set of natural transformation $F^{xx}$ from the functor $n \mapsto hom(F,U(n))$ to $n \mapsto U(n)$, compatible with conjugation, $\oplus$ and $\otimes$, then this turns out to be too big and not discrete. First of all, $F^{xx}$ is a polish group and there is a natural homomorphism $F \to F^{xx}$. The whole construction goes under the name Chu duality and works just as in the case of Pontrjagin duality or Tannaka-Krein duality. But $F \to F^{xx}$ is not a homeomorphism.
-This follows from the fact that for fixed $n \in \mathbb N$ and a fixed neighborhood $V$ of $1_n \in U(n)$, there exists $w \in F \setminus \lbrace e\rbrace$, such that $\phi(w) \in V$, for all homomorphisms $\phi \colon F \to U(n)$. The same problem appears for every finitely generated group, which is not virtually abelian.
-The problem can be cured completely if one takes the infinite-dimensional representations into account. Then, the appropriate bi-dual is equal to $F$ (and the same holds for any locally compact group). An important step in the proof of this assertion is the Gelfand-Raikov theorem.<|endoftext|>
-TITLE: Hyperbolic Coxeter polytopes and Del-Pezzo surfaces
-QUESTION [23 upvotes]: Added. In the following link there is a proof of the observation made in this question: http://dl.dropbox.com/u/5546138/DelpezzoCoxeter.pdf
-
-I would like to find a reference for a beautiful construction that associates to Del-Pezzo surfaces hyperbolic Coxeter polytopes of finite volume and ask some related questions.
-Recall that a hyperbolic Coxeter polytope is a domain in $\mathbb H^n$ bounded by a collection of geodesic hyperplanes, such that each intersecting couple of hyperplanes intersect under angle $\frac{\pi}{n}$ ($n=2,3,...,+\infty$). Del Pezzo surface is a projective surface obtained from $\mathbb CP^2$ by blowing up (generically) at most $8$ points.
-Now, the construction Del Pezzo $\to$ Coxeter polytope goes as follows.
-Consider $H_2(X,\mathbb R)$, this is a space endowed with quadratic form of index $(1,n)$ (the intersection form), and there is a finite collection of vectors $v_i$ corresponding complex lines on $X$ with self-intersection $-1$. It is well known, for example that on a cubic surface in $\mathbb CP^3$ there are $27$ lines and this collection of lines has $E_6$ symmetry (if you consider it as a subset in $H_2(X,\mathbb Z)$). Now we just take the nef cone of $X$, or in simple terms the cone in $H_2(X,\mathbb R)$ of vectors that pair non-negatively with all vectors $v_i$. This cone cuts a polytope from the hyperbolic space corresponding to $H_2(X,\mathbb R)$, and it is easy to check that this polytope is Coxeter, with angles $\frac{\pi}{2}$ and $0$ (some points of this polytope are at infinity, but its volume is finite). Indeed, angles are $\frac{\pi}{2}$ and $0$ since $v_i^2=-1$, $v_i\cdot v_j=0 \;\mathrm{or}\; 1$.
-Example. If we blow up $\mathbb CP^2$ in two points this construction produces a hyperbolic triangle with one angle $\frac{\pi}{2}$ and two angles $0$.
-The connection between algebraic surfaces an hyperbolic geometry is very well-known, and exploited all the time but for some reason I was not able to find the reference to this undoubtedly classical fact (after some amount of googling). So,
-Question 1. Is there a (nice) reference for this classical fact?
-This question is motivated in particular by the following article http://maths.york.ac.uk/www/sites/default/files/Preprint_No2_10_0.pdf where the polytope corresponding to the cubic surface is used. The authors mention the relation of the polytope to 27 lines on the cubic, but don't say that the relation is in fact almost canonical.
-Question 2. The group of symplectomorhpisms (diffeos) of each Del-Pezzo surface $X$ is acting on $H_2(X,\mathbb R)$, let us denote by $\Gamma$ its image in the isometries of corresponding hyperbolic space. What is the relation between $\Gamma$ and the group generated by reflections in the faces of the corresponding Coxeter polytope?
-PS It one considers rational surfaces with semi-ample anti-canonical bundles, i.e. surfaces that can have only rational curves with self-intersection $-1$ and $-2$ one gets more examples of Coxeter polytopes; the faces of such polytopes intersect under angles $(\frac{\pi}{2}, \frac{\pi}{3}, \frac{\pi}{4}, 0)$.
-Here is a reference on "Algebraic surfaces and hyperbolic geometry" by Burt Totaro (but I don't think that the answer to question one is contained there).
-
-REPLY [4 votes]: For $Q2$, in my paper joint with T.-J. Li http://arxiv.org/abs/1012.4146 there's a description by reflections of $\Gamma$, and Shevchishin proved the same conclusion in http://arxiv.org/abs/0904.0283 but his language is more Coxeter. We actually dealt with all rational surfaces.<|endoftext|>
-TITLE: A name for "not quite saturated" graded modules
-QUESTION [8 upvotes]: Let $M$ be a finitely generated graded module over a graded ring $R$. Let $\mathcal{F}$ be the corresponding coherent sheaf on $\operatorname{Proj} R$. There is a natural map of graded $R$-modules
-$$\phi \colon M \to \Gamma^*(\mathcal{F}) := \bigoplus_{n} \Gamma(\operatorname{Proj} R, \mathcal{F}(n)).$$
-If I recall Ravi Vakil's notes correctly, $M$ is called saturated if $\phi$ is an isomorphism.
-
-Is there a term (perhaps semi-saturated, or some such) for modules $M$ such that $\phi$ is injective?
-
-This concept is appealing for several reasons. For one thing, it is easier to test "semi-saturatedness" than saturatedness; e.g., unless I am mistaken, $\phi$ is automatically injective if $M$ admits any positive-degree homogeneous nonzerodivisor. For another, at least if $R$ is a polynomial ring, $M = R/I$ is "semi-saturated" iff $I$ is a saturated ideal of $R$. (Note that the definition of "saturated ideal" is different from the definition given above for "saturated module", and I do not think the two are equivalent for ideals.)
-
-REPLY [3 votes]: As Hailong wrote, the injectivity means the vanishing of $H^0_m(M)$. But $\operatorname{depth}(m,M)$ is the least non-vanishing local cohomology, thus the map $M \to H^0_*(\tilde{M})$ is injective iff $M$ has depth $\ge$ 1.
-If $M$ has finite projective dimension, e.g. if R is regular, then this happens iff $M$ has projective dimension at most depth $R-1$ by the Auslander-Buchsbaum formula.<|endoftext|>
-TITLE: Probability distributions: The maximum of a pair of iid draws, where the minimum is an order statistic of other minimums?
-QUESTION [6 upvotes]: General question: What is the distribution for the maximum of 2 independent draws from cdf F(x), when we know that the minimum of those same two draws is the kth order statistic of the minimum of n pairs of independent draws from F(x)? Less technically, what is the distribution of the maximum associated with the kth greatest (of n) minima?
-A specific example:
-Assume 8 independent draws from cdf F(x), which is defined over 0 to 1. Then, arbitrarily group the draws into 4 pairs. Compare the minimums of each pair. Label the maximum of these minimums as “a”. Label a’s pair (which is by definition > a) as “b”. Now, choose among the other three pairs arbitrarily, and label the two values in that pair as “c” and “d” (where c is the min of the pair and d is the max of the pair).
-What are the distributions of b and d?
-I know the distribution of a: F(a) = (1-(1-F(x))^2)^4 =Max of 4 draws of the Min of 2 draws of F(x).
-I also know the distribution of c: F(c) = mixture of 1st , 2nd, and 3rd order statistics of 4 draws of Min of 2 draws of F(x). I get this by averaging the integrals (wrt x) for the pdfs that result from substituting (k=1, n=4), (k=2,n=4) and (k=3, n=4) into the following equation:
-(n!/((k - 1)!(n - k)!))(F(x)^(k - 1))*((1 - F(x))^(n - k))*F'(x)
-I don’t know how to define F(b) or F(d)
-And help would be greatly appreciated.
-
-REPLY [3 votes]: This question has been answered by Bogdan Lataianu at this link:
-https://stats.stackexchange.com/questions/13259/what-is-the-distribution-of-maximum-of-a-pair-of-iid-draws-where-the-minimum-is<|endoftext|>
-TITLE: Lorentzian characterization of genus
-QUESTION [6 upvotes]: Suppose we take the "even" indefinite lattice from page 50 in Serre A Course in Arithmetic (1973)
-$$ U \; = \;
- \left( \begin{array}{cc}
- 0 & 1 \\\
- 1 & 0
-\end{array}
- \right),$$
-called $H$ in pages 189-191 of Larry J. Gerstein Basic Quadratic Forms.
-What I cannot find in any detail is a proof of this arithmetic statement in
-SPLAG by Conway and Sloane, page 378 in the first edition(1988), anyway
-chapter 15 section 7, that quadratic forms $f,g$ are in the same genus
-if and only if $f \oplus H$ and $g \oplus H$ are integrally equivalent. Then
-they say this follows from properties of the spinor genus, presumably
-including Eichler's theorem that indefinite rank at least 3 means
-spinor genus and class coincide.
-Also, if f and g do not correspond to "even lattices," I'm not
-entirely sure what is being claimed. Oh, I absolutely cannot assume $f,g$ are in any way "unimodular." Very popular, that unimodular. Matter of taste, though. I'm not sure it matters, but my $f,g$ are going to be positive, which is surely the difficult case here.
-Everybody with whom I have discussed this regards this as either
-obvious or, essentially, an axiom. I would very much like a reference
-for this, plus an explanation of what is meant if $f,g$ correspond to
-"odd" lattices. For example, it would be wonderful if somewhere this claim and the words Theorem or Proposition or Lemma happened in the same sentence. I think I am making progress on the other bits I
-need, essentially ch. 26,27 in SPLAG, but this claim has me snowed,
-or perhaps buffaloed, thrown, stumped. As far as books that I own, I do not see the claim being discussed in Jones, Watson, O'Meara, Serre, Cassels, Kitaoka, Ebeling, Gerstein. I stopped by the office of R. Borcherds and discussed related matters for a while, the relevant articles are 1985 The Leech Lattice and 1990 Lattices Like the Leech Lattice, but I don't see the SPLAG claim in an explicit manner.
-EDIT... Sexy application: the Leech lattice and all the Niemeier lattices are in the same genus. Pointed out in an MO comment by Noam Elkies, who knows things.
-
-REPLY [7 votes]: A good reference for this assertion is Cassels's "Rational Quadratic Forms", though you have to dig a bit. Let me see if I can outline the proof. First, I think Conway and Sloane assume $f$ and $g$ are classical integral (i.e. correspond to even lattices). In my copy of SPLAG, at the end of subsection 2.1 of that chapter, they say "so in this book we call $f$ an integral form if and only if its matrix coefficients are integers (i.e. if and only if it is classically integral ...)".
-Now suppose $f$ and $g$ are in the same genus. Then so are $f\oplus H$ and $g \oplus H$. Next, we want to show they're in the same spinor genus. This follows from the Corollary of Lemma 3.6 of Chapter 11 of Cassels: "If we show $U_p \subset \theta(\Lambda_p)$ for all $p$, then the genus of $\Lambda$ consists of a single spinor genus". Here $\Lambda = f \oplus U$, where I'm identifying the form and the lattice by a bit of abuse of notation. Since $\theta(\Lambda_p) \supset \theta(H_p)$ (see a few sentences below the corollary), and $\theta(H_p) \supset U_p$ by Lemmas 3.7 and 3.8, we've proved that the genus consists of a single spinor genus.
-Finally, since the forms are indefinite of dimension at least $3$, the spinor genus consists of a single class.
-To go back is the easier direction (I think): if $f \oplus U$ is equivalent to $g \oplus U$, then they are equivalent over $\mathbb{Z}_p$ for every $p$. Then an analogue of Witt cancellation will do the job (see Chapter 8 of Cassels).<|endoftext|>
-TITLE: Affine manifolds
-QUESTION [10 upvotes]: An affine manifold is a topological manifold which admits a system of charts such that the coordinate changes are (restrictions of) affine transformations. Let $M$ be a compact affine manifold. Let $G$ be the fundamental group of $M$ and $\tilde M$ be its universal cover. One can show that each $n$-dimensional affine manifold comes with a developing map $D\colon \tilde M \to \mathbb R^n$, and a homomorphism $\varphi \colon G \to {\rm Aff}(\mathbb R^n)$, such that $D$ is an immersion and equivariant with respect to $\varphi$.
-An affine manifold is called complete if $D$ is a homeomorphism, in this case: $\varphi$ is injective, $G$ is a Bieberbach group, and $M$ is aspherical, i.e. $\tilde M$ is contractible. The non-complete case seems to be far more complicated.
-
-Question 1: Is there an easy example, where $D$ is not surjective?
-Question 2: Is there an easy example, where $\varphi$ is not injective?
-Question 3: Is there an easy example, where $M$ is not aspherical?
-
-EDIT: As André suggested, let's ask for examples for which $\varphi$ takes values in $SL(n,\mathbb R) \ltimes \mathbb R^n$ or even $SL(n,\mathbb Z) \ltimes \mathbb R^n$, seen as subgroups of ${\rm Aff}(\mathbb R^n)$.
-
-REPLY [9 votes]: There is a conjecture due to Markus which states that any compact affine manifold has parallel volume (i.e. the linear part of $\varphi$ lies in $\mathrm{SL}(n;\mathbb{R})$) if and only if it is complete. To the best of my knowledge, this conjecture is still open, which goes towards saying that there should be no easy examples to questions 1 and 3 for affine manifolds with parallel volume.
-If the fundamental group $G$ of a compact affine manifold with parallel volume is nilpotent, the beautiful Affine manifolds with nilpotent holonomy by Fried, Goldman and Hirsch, Comm. Math. Helv. 56 (1981) proves that Markus' conjecture holds in this case and, thus, there are no examples to questions 1 and 3 with nilpotent fundamental group. The proof is a cunning mixture of representation theory and geometry, so I strongly recommend taking a look at it. The results in this paper also imply that the above examples (to questions 1 and 3) constructed by Andre Henriques cannot be adapted so that the resulting manifolds admit parallel volume (these are, nonetheless, very nice examples to the original question!).<|endoftext|>
-TITLE: Singular curves in a 3-fold?
-QUESTION [5 upvotes]: Assume that $X$ is a smooth 3-fold and let $C\subseteq X$ a curve with a unique singular point of multiplicity $2$. Does there exist a smooth surface $S$ inside $X$ which contain $C$ ?
-Clearly if the multiplicity of $C$ was at least 3 then it would be very easy to find counter-examples. On the other hand, if the multiplicity is 2, it seems that at least infinitesimally it is possible to find such a surface. In case the answer is negative, is it true at least locally (e.g. in an analytic neighbourhood of p)?
-Finally any smooth curve is locally complete intersection. What about the curve $C$ above?
-
-REPLY [4 votes]: Here is a quick proof that any complete local Cohan-Macaulay ring of dimension $1$ and multiplicity $2$ is a hypersurface, so the answer to your last question is always yes.
-Call such ring $R$ with maximal ideal $m$. Since $e(R)=2$ and $R$ is CM, there is a regular element $x\in m$ such that $$length(R/xR) = e(R)=2$$ (see Bruns-Herzog, chapter 4).
-EDIT: as Graham pointed out below, technically for such $x$ to exist one needs $R/m$ to be infinite. But one can enlarge the field without affecting the conclusion that $R$ is a hypersurface.
-Now the left hand side is $length(m/xR) +1$, so $length(m/xR)=1$, so $m/xR$ is one-generated. It follows that $m$ is $2$-generated. Thus $R=A/I$, $A$ is a regular local ring of dimension $2$. But since $R$ is CM, $I$ must be of pure height one, and since $A$ is regular, $I$ is principal.
-Note: in fact, the dimension one condition is not necessary, you can take a full length regular sequence and argue the same way (with just a little more work). So any CM singularity with multiplicity $2$ is a hypersurface.
-EDIT: Once we know that the local ring at the singular point is a hypersurface, Jason Starr and Qing Liu's nice comments show that there must exist a smooth surface containing $C$, thus completely answer the question.<|endoftext|>
-TITLE: Non-oscillatory behaviour in the subadditive ergodic theorem
-QUESTION [10 upvotes]: I am currently reading an article in which the author goes to certain lengths which could be avoided if the following result were true:
-
-
-Lemma (proposed): Let $T$ be an ergodic measure-preserving transformation of a probability space $(X,\mathcal{F},\mu)$, and let $(f_n)$ be a sequence of integrable functions from $X$ to $\mathbb{R}$ which satisfy the subadditivity relation $f_{n+m} \leq f_n \circ T^m + f_m$ a.e. for all integers $n,m \geq 1$. Suppose that $f_n(x) \to -\infty$ in the limit as $n \to \infty$ for $\mu$-a.e. $x \in X$. Then $\lim_{n \to \infty} \frac{1}{n}\int f_n d\mu <0$.
-
-
-Via the subadditive ergodic theorem, this effectively states that if $f_n(x) \to -\infty$ almost everywhere then it must do so at an asymptotically linear rate. The supposed lemma would also be equivalent to the statement that if $\frac{1}{n} f_n(x) \to 0$ almost everywhere, then for almost every $x$ the sequence $(f_n(x))$ must return infinitely often to some neighbourhood of $0$ which is not a neighbourhood of $-\infty$. If the sequence $(f_n)$ is additive rather than just subadditive then this last formulation of the result follows from a well-known theorem of G. Atkinson, but the more general subadditive case is less clear.
-If the lemma were true then several parts of the paper I am reading would be redundant, which makes me wonder whether it is in fact false. Yet it seems rather plausible. Does anyone know whether this result is true or not?
-
-REPLY [3 votes]: I believe the Lemma you propose is true, via a relatively straightforward adaptation of the proof given for the additive case in Giles Atkinson, Recurrence of co-cycles and random walks, J. Lond. Math. Soc. (2) 13 (1976), 486-488. (I assume this is the result you were referencing.) This may already be known and written somewhere -- I can't speak to that. In case it's not, here's a proof.
-Proof of Lemma. By the subadditive ergodic theorem, there exists a measurable function $f\colon X\to \mathbb{R}\cup\{-\infty\}$ such that $\lim \frac 1n f_n(x) = f(x)$ for $\mu$-a.e. $x\in X$ and $\lim \frac 1n \int f_n\,d\mu = \int f\,d\mu$. So our task is to show that $\int f\,d\mu < 0$.
-To this end, given $x\in X$, let $M_x = \{n \mid f_n(x) \geq -1 \}$, and observe that $M_x$ is finite $\mu$-a.e. Thus writing $A_n = \{x\in X \mid \# M_x < n \}$, we see that there exists $N$ such that $\mu(A_N) > \frac 12$.
-Furthermore, given $x\in X$, write $L_x = \{ k \mid T^k(x) \in A_N \}$. Since $\mu(A_N) > \frac 12$, we see that $\mu$-a.e. $x$ has
-$$
-(*)\qquad\qquad\qquad\# L_x \cap [1,n] \geq \frac n2\qquad\qquad\qquad\qquad\qquad\quad
-$$
-for all sufficiently large $n$. We fix such an $x$ and show that $f(x) < 0$.
-Let $k_0$ be the smallest element of $L_x$. We define $k_i\in L_x$ recursively with the property that
-$$
-f_{k_i}(x) < f_{k_0}(x)-i,
-$$
-as follows. Let $J_i$ be the $N$ smallest elements of $L_x \cap (k_i,\infty)$. Because $T^{k_i}(x)\in A_N$, there exists $k_{i+1}\in J_i$ such that $k_{i+1} - k_i \notin M_{T^{k_i}(x)}$. In particular, we have
-$$
-f_{k_{i+1} - k_i}(T^{k_i}(x)) < -1.
-$$
-Now subadditivity gives
-$$
-f_{k_{i+1}}(x) \leq f_{k_i}(x) + f_{k_{i+1} - k_i}(T^{k_i}(x)) < f_{k_0(x)} - i-1.
-$$
-The next observation to make is that by $(*)$, we have $k_i \leq 2Ni$ for all sufficiently large $i$. Thus we have
-$$
-\frac 1{k_i} f_{k_i}(x) \leq \frac 1{2Ni}(f_{k_0}(x) - i),
-$$
-and sending $i\to\infty$, we obtain $f(x) \leq -\frac 1{2N}$. Since this holds for $\mu$-a.e. $x$, we have $\int f\,d\mu \leq -\frac 1{2N} < 0$, which completes the proof.
-
-Note. It's quite important in this proof that we're dealing with negative values of $f_n$; the proof would fail if we tried to show that $f_n(x)\to+\infty$ for $\mu$-a.e. $x$ implies that $\int f\,d\mu > 0$. I'm not sure if the result is true in this case.<|endoftext|>
-TITLE: Ask some matrix eigenvalue inequalities.
-QUESTION [5 upvotes]: Let $ \begin{bmatrix}
-A& B \\\\ B^* &C
-\end{bmatrix}$ be positive semidefinite, $A,C$ are of size $n\times n$.
-Are the following plausible inequalities true? I have seen a lot of similar results, but for the following inequalities, I cannot locate them in the literature or find that they have been pointed out to be false.
-1 $$\quad \sum\limits_{i=1}^k\lambda_i\begin{bmatrix}
-A& B \\\\ B^* &C
-\end{bmatrix}\le \sum\limits_{i=1}^k\left(\lambda_i(A)+\lambda_i(C)\right)\quad, $$
-where $1\le k\le n$.
-2 Modified
-$$\quad \prod\limits_{i=1}^{2k}\lambda_i\begin{bmatrix}
-A& B \\\\ B^* &C
-\end{bmatrix}\le \prod\limits_{i=1}^k \lambda_i(A)\lambda_i(C) \quad, $$
-where $1\le k\le n$.
-3 $$ 2\lambda_i^{1/2}(B^*B)\le \lambda_i(A+C),$$ where $1\le k\le n$.
-Here, $\lambda_i(\cdot)$ means the $i$th largest eigenvalue of $\cdot\quad$. Any references or counterexamples are appreciated.
-
-REPLY [17 votes]: Item 1 is true. This is part of Problem 22 (b) in Section 3.5 of Horn and Johnson [HJ94], which states that for Ky Fan norm ||⋅|| (and in fact for any unitarily invariant norm) and a positive semidefinite block matrix $\begin{pmatrix}A & B \\ B^* & C\end{pmatrix}$, it holds that $\left\|\begin{pmatrix}A & B \\ B^* & C\end{pmatrix}\right\| \le \left\|\begin{pmatrix}A & 0 \\ 0 & 0\end{pmatrix}\right\| + \left\|\begin{pmatrix}0 & 0 \\ 0 & C\end{pmatrix}\right\|$.
-([Aud06] contains a proof of a slight generalization of this inequality among other results.)
-Item 2 in the original question is false by considering the case where A=C=I/2, B=0, and k=1, where I is the identity matrix. (Did you mean to square the left-hand side?)
-Modified item 2 is false; see Willie Wong’s comment on this answer.
-Item 3 is false. A simple counterexample is n=2, i=1,
-$A=\begin{pmatrix}1 & 0 \\ 0 & 0\end{pmatrix}$,
-$B=\begin{pmatrix}0 & 1 \\ 0 & 0\end{pmatrix}$,
-$C=\begin{pmatrix}0 & 0 \\ 0 & 1\end{pmatrix}$.
-Then $2\sqrt{\lambda_1(B^*B)}=2$ but λ1(A+C)=1.
-References
-[Aud06] Koenraad M. R. Audenaert. A norm compression inequality for block partitioned positive semidefinite matrices. Linear Algebra and its Applications, 413(1):155–176, Feb. 2006. http://dx.doi.org/10.1016/j.laa.2005.08.017
-[HJ94] Roger A. Horn, Charles R. Johnson. Topics in Matrix Analysis. Cambridge University Press, 1994.<|endoftext|>
-TITLE: Endpoint Strichartz Estimates for the Schrödinger Equation
-QUESTION [6 upvotes]: The non-endpoint Strichartz estimates for the (linear) Schrödinger equation:
-$$
-\|e^{i t \Delta/2} u_0 \|_{L^q_t L^r_x(\mathbb{R}\times \mathbb{R}^d)} \lesssim \|u_0\|_{L^2_x(\mathbb{R}^d)}
-$$
-$$
-2 \leq q,r \leq \infty,\;\frac{2}{q}+\frac{d}{r} = \frac{d}{2},\; (q,r,d) \neq (2,\infty,2),\; q\neq 2
-$$
- are easily obtained using (mainly) the Hardy-Littlewood-Sobolev inequality, the endpoint case $q = 2$ is however much harder (see Keel-Tao for example.)
-Playing around with the Fourier transform one sees that estimates for the restriction operator sometimes give estimates similar to Strichartz's. For example, the Tomas-Stein restriction theorem for the paraboloid gives:
-$$
-\|e^{i t \Delta/2} u_0\|_{L^{2(d+2)/d}_t L^{2(d+2)/d}_x} \lesssim \|u_0\|_{L^2_x},
-$$
-which, interpolating with the easy bound
-$$
-\|e^{i t \Delta/2} u_0\|_{L^{\infty}_t L^{2}_x} \lesssim \|u_0\|_{L^2_x},
-$$
-gives precisely Strichartz's inequality but restricted to the range
-$$
-2 \leq r \leq 2\frac{d+2}{d} \leq q \leq \infty.
-$$
-As far as I know, the Tomas-Stein theorem (for the whole paraboloid) gives the restriction estimate $R_S^*(q'\to p')$ for $q' = \bigl(\frac{dp'}{d+2}\bigr)'$ (this $q$ is different from the one above), so I'm guessing that this cannot be strengthened (?).
-So my question is: what's the intuition of what goes wrong when trying to prove Strichartz's estimates all the way down to the endpoints using only Fourier restriction theory?
-
-REPLY [3 votes]: From my less-than-expert (where's Terry when you need him?) point of view, a possible reason seems to be the following (I wouldn't call it something going wrong or even a difficulty):
-The statement of restriction estimates only give you estimates where the left hand side is an isotropic Lebesgue space, in the sense that you get an estimate $L^q_tL^r_x$ with $q = r$. This naturally excludes the end-point, which requires $r > q$.
-Why is this? The reason is that the restriction theorems only care about the local geometry of the hypersurface, and not its global geometry. (For example, the versions given in Stein's Harmonic Analysis requires either the hypersurface to have non-vanishing Gaussian curvature for a weaker version, or that the hypersurface to be finite type for a slightly stronger version. Both of these conditions are assumptions on the geometry of the hypersurface locally as a graph over a tangent plane.) Now, on each local piece, you do have something more similar to the classical dispersive estimates with $r > q$, which is derived using the method of oscillatory integrals (see, for example, Chapter IX of Stein's book; the dispersive estimate (15) [which has, morally speaking $q = r = \infty$ but with a weight "in $t$", so actually implies something with $q < \infty$] is used to prove Theorem 1, which is then used to derive the restriction theorem). But once you try to piece together the various "local" estimates to get an estimate on the whole function, you have no guarantee of what the "normal direction" is over the entire surface. (The normal direction, in the case of the application to PDEs, is the direction of the Fourier conjugate of the "time" variable.) So in the context of the restriction theorem, it is most natural to write the theorem using the $q = r$ version, since in the more general context of restriction theorems, there is no guarantee that you would have a globally preferred direction $t$.
-(Note that Keel-Tao's contribution is not in picking out that time direction: that Strichartz estimates can be obtained from interpolation of a dispersive inequality and energy conservation is well known, and quite a bit of the non-endpoint cases are already available as intermediate consequences of the proof of restriction theorems. The main contribution is a refined interpolation method to pick out the end-point exponents.)<|endoftext|>
-TITLE: Why is the Chebyshev function relevant to the Prime Number Theorem
-QUESTION [20 upvotes]: Why is the Chebyshev function
-$\theta(x) = \sum_{p\le x}\log p$
-useful in the proof of the prime number theorem. Does anyone have a conceptual argument to motivate why looking at $\sum_{p\le x} \log p$ is relevant and say something random like $\sum_{p\le x}\log\log p$ is not useful or for that matter any other random function $f$ and $\sum_{p\le x} f(p)$ is not relevant.
-
-REPLY [24 votes]: There are several ideas here, some mentioned in the other answers:
-One: When Gauss was a boy (by the dates found on his notes he was approximately 16) he noticed that the primes appear with density $ \frac{1}{\log x}$ around $x$. Then, instead of counting primes and looking at the function $\pi (x)$, lets weight by the natural density and look at $\sum_{p\leq x} \log p$. Since we are weighting by what we think is the density, we expect it to be asymptotic to be $x$.
-Two: Differentiation of Dirichlet series. If $$ A(s)=\sum_{n=1}^{\infty} a_{n} n^{-s} $$ then $$ (A(s))'=-\sum_{n=1}^{\infty} a_{n} log(n) n^{-s}$$
-The $\log$ term appears naturally in the differentiation of Dirichlet series. Taking the convolution of $\log n$ with the Mobius function (that is multiplying by $\frac{1}{\zeta(s)}$) then gives the $\Lambda(n)$ mentioned above. The $\mu$ function is really the special thing here, not the logarithm.
-Expanding on this, there are other weightings besides $\log p$ which arise naturally from taking derivatives. Instead we can look at $\zeta^{''}(s)=\sum_{n=1}^\infty (\log n)^2 n^{-s}$, and then multiply by $\frac{1}{\zeta(s)}$ as before. This leads us to examine the sum $$\sum_{n\leq x} (\mu*\log^2 )(n)$$ (The $*$ is Dirichlet convolution) By looking at the above sum, Selberg was able to prove his famous identity which was at the center of the first elementary proof of the prime number theorem:
-$$\sum_{p \leq x} log^2 p +\sum_{pq\leq x}(\log p)(\log q) =2x\log x +O(x).$$
-Three: The primes are intimately connected to the zeros of $\zeta(s)$, and contour integrals of $\frac{1}{\zeta(s)}$. (Notice it was featured everywhere here so far) We can actually prove that
-$$\sum_{p^k \leq x} \log p= x - \sum_{\rho :\ \zeta(\rho)=0} \frac{x^{\rho}}{\rho} +\frac{\zeta'(0)}{\zeta(0)} $$
-Notice that the above is an equality, which is remarkable since the left hand side is a step function. (Somehow, at prime powers all of the zeros of zeta conspire and make the function jump.)<|endoftext|>
-TITLE: Error term of the Prime Number Theorem and the Riemann Hypothesis
-QUESTION [6 upvotes]: I have read that the Riemann Hypothesis is equivalent to
-$\pi(x)=\text{Li}(x)+O(\sqrt{x}\log x)$
-Is there an analogous statement saying the Riemann Hypothesis is equivalent to
-$\pi(x)=\frac{x}{\log x}+ O(f(x))\quad$ for some $f$
-or
-$\pi(x)=\frac{x}{\log x}+ g(x) + O(h(x))\quad$ for some elementary function $g$ and $h$
-I'm guessing that $f$ could not possibly be $\sqrt{x}\log x$ because I plotted
-$\frac{\text{Li}(x)-x/\log(x)}{\sqrt x\log x}$ and it looked like it grew without bound as $x$ goes to infinity.
-
-REPLY [20 votes]: It is not hard to show that
-$$\mathrm{Li}(x) = \frac{x}{\log x} \sum_{k=0}^{m - 1}{\frac{k!}{(\log x)^k}} + O\left(\frac{x}{(\log x)^{m + 1}}\right)$$
-for any $m \geq 0$ (just use the definition of $\mathrm{Li}(x)$ and repeated integration by parts). Thus
-$$\pi(x) = \frac{x}{\log x} \sum_{k=0}^{m - 1}{\frac{k!}{(\log x)^k}} + O\left(\frac{x}{(\log x)^{m + 1}}\right).$$
-It is not possible to improve on this (this is true unconditionally; you don't even need the Riemann hypothesis). So $\mathrm{Li}(x)$ really is the "better" approximation to $\pi(x)$ compared to $x/\log x$.<|endoftext|>
-TITLE: Random manifolds
-QUESTION [55 upvotes]: In the world of real algebraic geometry there are natural probabilistic questions one can ask: you can make sense of a random hypersurface of degree d in some projective space and ask about its expected topology where "expected" makes sense because there are sensible measures on the space of hypersurfaces. See Welschinger-Gayet http://arxiv.org/abs/1107.2288 and http://arxiv.org/abs/1005.3228 for recent progress on such questions (e.g. what is the expected Betti number of a random real hypersurface of degree d?).
-In geometry more generally you might want to make statements like "a general manifold is aspherical" or "a general manifold has positive simplicial volume". It seems difficult to construct sensible measures for which these questions have answers: to talk about probability you need some way of producing manifolds (and then distinguishing them) in a random way.
-However, Cheeger proved that for fixed L there is only a finite set $D_L$ of diffeomorphism classes of manifold admitting a Riemannian metric with sectional curvatures bounded above in norm by L, volume bounded below by 1/L and the diameter bounded above by L (see the first theorem in Peters "Cheeger's finiteness theorem for diffeomorphism classes of Riemannian manifolds" http://www.reference-global.com/doi/abs/10.1515/crll.1984.349.77). This means that you can ask questions like "what is the average total Betti number of a manifold in $D_L$" and "how does this increase with L?" (are there exponential upper bounds?), or one can try to make sense of the limit as $L\rightarrow\infty$ of the proportion of manifolds in $D_L$ with zero simplicial volume.
-Are there any known concrete answers to these questions, or other formulations of the questions which lead to answers?
-
-REPLY [3 votes]: Here are three references that discuss the construction and related properties of "random Riemannian manifolds." The construction is related to random graphs and is "an approach to random Riemann surfaces based on associating a dense set of them - Belyi surfaces - with random cubic graphs."
-Random Construction of Riemann Surfaces
-Robert Brooks, Eran Makover
-http://arxiv.org/abs/math/0106251
-Eran Makover has coauthored two other related papers:
-
-The length of closed geodesics on random Riemann Surfaces
-Eran Makover, Jeffrey McGowan
-On the Genus of a Random Riemann Surface
-Alexander Gamburd, Eran Makover<|endoftext|>
-TITLE: Characterization of locally free modules via exterior powers
-QUESTION [7 upvotes]: Let $X$ be a scheme and $\mathcal{F}$ be quasi-coherent module on $X$. It is clear that if $\mathcal{F}$ is locally free of rank $n$, then $\det(\mathcal{F}) := \wedge^n \mathcal{F}$ is invertible, i.e. locally free of rank $1$. But what about the converse?
-Question. Assume $\wedge^n \mathcal{F}$ is invertible. Does it follow that $\mathcal{F}$ is locally free (necessarily of finite rank $n$)?
-Of course we may assume that $X$ is affine. Then it is enough to prove that $\mathcal{F}$ is flat and of finite presentation, but I don't know how to prove either one. Also it seems to be hard to find counterexamples.
-
-REPLY [8 votes]: I think that $\mathcal F$ is indeed locally free of rank $n$:
-Pick a point $x\in X$. It will be enough to show that there is a neighbourhood
-of $x$ on which $\mathcal F$ is free of rank $n$. Now, the exterior power
-commutes with pullbacks (aka scalar extensions) so that in particular the fibre
-(in the sense of pullback to $\mathrm{Spec}k(x)$) of $\Lambda^m\mathcal F$ at
-$x$ equals $\Lambda^m\mathcal{F}_x$. This shows that $\mathcal{F}_x$ is an
-$n$-dimensional vector space. After possibly shrinking $X$ we may assume that
-there is a an $\mathcal{O}_X$-map $f\colon \mathcal{O}_X^n\to \mathcal F$ which induces
-an isomorphism on fibres at $x$. Thus $\Lambda^nf$ is a map between locally free
-modules (of rank $1$) that gives an isomorphism on fibres at $x$ and hence is an
-isomorphism in a neighbourhood of $x$ so that we may assume that it is a global
-isomorphism. The wedge product induces pairings
-$\mathcal{F}\times\Lambda^{n-1}\mathcal{F}\to \Lambda^{n}\mathcal{F}$ and
-$\mathcal{O}_X^n\times\Lambda^{n-1}\mathcal{O}_X^n\to
-\Lambda^{n}\mathcal{O}_X^n$, the latter being a perfect pairing. Composing the
-second with $\Lambda^nf$ gives a pairing
-$\mathcal{O}_X^n\times\Lambda^{n-1}\mathcal{O}_X^n\to \Lambda^{n}\mathcal{F}$.
-As $\Lambda^\ast f$ is multiplicative we get that the composite
-$$\mathcal{O}_X^n\xrightarrow{f}\mathcal{F}\to
-\mathrm{Hom}(\Lambda^{n-1}F,\Lambda^{n}\mathcal{F})\xrightarrow{\Lambda^{n-1}f^*}\mathrm{Hom}(\Lambda^{n-1}\mathcal{O}_X^n,\Lambda^{n}\mathcal{F})$$
-equals the map induced by the pairing for $\mathcal{O}_X^n$. This is an
-isomorphism (as $\mathcal{O}_X^n$ is free of rank $n$ and $\Lambda^nf$ is an
-isomorphism) so we get that $f$ is split injective and we may write
-$\mathcal{F}$ as `\mathcal{O}_X^n\bigoplus \mathcal G$ for some quasi-coherent sheaf
-$\mathcal{G}$. Now, $\Lambda^n(\mathcal{O}_X^n\bigoplus \mathcal{G})$ splits up
-as
-$$
-\bigoplus_{i+j=n}\Lambda^i\mathcal{O}_X^n\bigotimes \Lambda^j\mathcal{G}
-$$
-and $\Lambda^nf$ is the inclusion into the $j=0$ factor. As that inclusion is an
-isomorphism, the other factors are zero but $\Lambda^{n-1}\mathcal{O}_X^n\bigotimes
-\Lambda^1\mathcal{G}$ has $\mathcal{G}$ as a direct factor and hence $\mathcal{G}=0$.<|endoftext|>
-TITLE: Can the Law of the Iterated Logarithm be strengthened?
-QUESTION [12 upvotes]: http://en.wikipedia.org/wiki/Law_of_the_iterated_logarithm
-
-.$\quad$1. Can the independence assumption be weakened, similar to this?
-
-.$\quad$2. Can the identically distributed assumption be dropped/weakened, in the latter case similar to this?
-
-.$\quad$3. Can the result be fine-tuned, presumably to something of the form
-
-$\displaystyle\limsup_{n\to \infty} \frac{\frac{S_n}{\sqrt{2\cdot n\cdot \operatorname{log}(\operatorname{log}(n))}}-1}{f(n)} \; \; $ some_relation_symbol $\;$ some_constant $\qquad$ almost surely $\qquad \; \; $ ?
-
-REPLY [6 votes]: For your third question, see a paper of Erdös, where he proves an even more precise result (at least, in the special case $S_n=\sum_{i=1}^nY_i$ where $Y_i$ (independent) are $\pm 1$ with probability $\frac 12$), namely that for $\delta>0$, the following holds with probability one:
-$$S_n>\left(\frac{2n}{\log\log n}\right)^{1/2}(\log\log n+\frac 34\log\log\log n+\frac 12\log\log\log\log n+\cdots+(\frac 12-\delta)\log^{(k)}n)\qquad\text{for infinitely many $n$}$$
-and
-$$S_n>\left(\frac{2n}{\log\log n}\right)^{1/2}(\log\log n+\frac 34\log\log\log n+\frac 12\log\log\log\log n+\cdots+(\frac 12+\delta)\log^{(k)}n)\qquad\text{for only finitely many $n$}$$
-In particular, this implies that:
-$$\limsup_{n\to \infty} \frac{\frac{S_n}{\sqrt{2n\log\log n}}-1}{\frac{\log\log\log n}{\log\log n}}=\frac 34$$<|endoftext|>
-TITLE: What do we actually know about logarithmic energy ?
-QUESTION [13 upvotes]: In potential theory, the $\textit{logarithmic energy}$ of a Radon measure $\mu$ acting on $\mathbb{C}$ is defined by
-$$I(\mu)=\iint\log\frac{1}{|x-y|}\mu(dx)\mu(dy).$$ Of course it is not well defined for all measures and may takes values in $[-\infty,+\infty]$. To avoid this annoying fact, one typically restrict to measures which integrate the logarithm around infinity, that is which satisfy the condition (C)
-$$ \int \log(1+|x|)\mu(dx)<+\infty,$$
-so that $I(\mu)>-\infty$ thanks to $|x-y|\leq (1+|x|)(1+|y|) $. A fondamental fact is that if $\mu,\nu$ both satisfy (C), have finite logarithmic energy and $\mu(\mathbb{C})=\nu(\mathbb{C})$, then $I(\mu-\nu)\geq0$ and equality holds iff $\mu=\nu$ (we extend naturally the definition of $I$ to signed measures). I understand the condition (C) to be convenient, but maybe not sharp (one can imagine $\mu$ which does not satisfies (C) but with finite logarithmic energy). This leads to my first question :
-
-What can we says about $I(\mu-\nu)$ when (at least one of) the measures do not satisfy (C) ?
-
-This question is moreover motivated by the appearance of the logarithmic energies in random matrix theory (in large deviations rate functions) and in free probability (reinterpreted up to a sign as a non-commutative entropy by Voiculescu). In this setting, $I(\mu-\nu)$ is a natural candidate for a relative free entropy, and questions of geometric nature are bothering me : Let $A_{c}$ be the set of signed measures $\mu$ with finite logarithmic energy acting on $\mathbb{C}$ with total mass $\mu(\mathbb{C})=c$ such that if $\mu$ has Jordan decomposition $\mu^+-\mu^-$, then both $\mu^+$ and $\mu^-$ satisfy (C). Then the previous fact yields that $A_0$ is a pre-Hilbert space with scalar product
-$$ I(\mu,\nu)=\iint\log\frac{1}{|x-y|}\mu(dx)\nu(dy).$$ Note that $A_0$ is not complete since it is clearly not closed. Moreover, it is not hard to check that $A_0$ acts by translations on $A_1$ which inherits of a structure of affine space, leading to a metric on probability measures with finite logarithmic energy satisfying (C). It is now time for my second question :
-
-What relations may exist between this metric and metrics compatible with the weak topology ? (e.g Prohorov's ? Levy / Bounded Lipshitz ? Wasserstein's ? ...) Or total variation norm ?
-
-REPLY [2 votes]: Concerning the first question: We have $I(\mu-\nu)>0$ whenever $I(\mu-\nu)$ is defined, finite or not, and $\mu$,$\nu$ are different signed Radon measures with equal total masses (or rather charges). This, with any finite dimensional Hilbert space in place of the plane, is Example 3.3 in
-http://www.ams.org/journals/tran/1997-349-08/S0002-9947-97-01966-1/home.html ,
-where the assumption $\sigma\neq0$ is missing.<|endoftext|>
-TITLE: Image of the trace operator
-QUESTION [21 upvotes]: It is well-known that we have the trace theorem for Sobolev spaces. Let $\Omega$ be an open domain with smooth boundary, we know that the map
-$$ T: C^1(\bar\Omega) \to C^1(\partial\Omega) \subset L^p(\partial\Omega) $$
-by $Tu(y) = u(y)$ for $y\in\partial\Omega)$ can be extended continuously to a linear map on Sobolev spaces for $p > 1$
-$$ T: W^{1,p}(\Omega) \to L^p(\partial\Omega)$$
-We also know that this map is not surjective, since the Trace Theorem (Sobolev embedding) tells us that when dropping 1 dimension, we have that the image of $T$ actually lives ([Edited May 10 2012] caveat: see my comment on the answer below) in a fractional Sobolev space,
-$$ T: W^{1,p}(\Omega) \to W^{1-1/p, p}(\partial\Omega) \Subset L^p(\partial\Omega) $$
-On the other hand, we know that this map $T$ has dense image in $L^p$, just using the density of $C^1$.
-Question: Is there a known characterisation of precisely what the image set of $T$ is? A slightly weaker question is: consider‡ $w \in W^{s,q}(\partial\Omega)$ for $1 - 1/p \leq s \leq 1$ and $q \geq p$, does there necessarily exist some function $u\in W^{1,p}(\Omega)$ such that $Tu = w$?
-For example, if we assume that $w$ is Lipschitz on $\partial\Omega$, then we can extend (almost trivially) $w$ to a Lipschitz function $C^{0,1}(\bar\Omega)\subset W^{1,p}$ for every $p$. So the case $s = 1, q = \infty$ has a positive answer. Whereas the Sobolev embedding theorem mentioned above tells us that it is impossible to go below $s < 1-1/p$ and $q < p$.
-‡ The lower cut-off here is clearly not sharp. The trace theorem combined with Sobolev embedding can be used to trade differentiability with integrability. Out of sheer laziness I will not include the numerology here. One should interpret the conditions on $s,q$ to be that $s \leq 1$, $q \geq p$ plus the requirement that $(s,q)$ is at least as good as what can be guaranteed by Sobolev embedding and the trace theorem.
-
-REPLY [14 votes]: The image you are looking for equals the Besov space $B_{p,p}^{1-\frac1p} (\partial \Omega )$. See
-H. Triebel. Interpolation theory, function spaces, differential operators. Leipzig, 1995
-(in fact, I used the earlier Russian edition, Moscow, 1980).<|endoftext|>
-TITLE: Structure of $E(Q_p)$ for elliptic curves with anomalous reduction modulo $p$
-QUESTION [15 upvotes]: For simplicity, take $p\ge7$ a prime and $E/\mathbb{Q}$ an elliptic curve with good anomalous reduction at $p$, i.e., $|E(\mathbb{F}_p)|=p$. There is a standard exact sequence for the group of points over $\mathbb{Q}_p$,
-$$
- 0 \to \hat{E}(p\mathbb{Z}_p) \to E(\mathbb{Q}_p) \to E(\mathbb{F}_p) \to 0.
-$$
-The assumption that $p$ is anomalous implies that $E(\mathbb{F}_p)=\mathbb{Z}/p\mathbb{Z}$, while the formal group $\hat{E}(p\mathbb{Z}_p)$ is isomorphic to the formal group of the additive group, so we obtain an exact sequence
-$$
- 0 \to p\mathbb{Z}_p^+ \to E(\mathbb{Q}_p) \to \mathbb{Z}/p\mathbb{Z} \to 0.
- \qquad(*)
-$$
-In work I'm doing on elliptic pseudoprimes, the question of whether the sequence $(*)$ splits has become relevant. Questions:
-
-Are there places in the literature
-where the split-versus-nonsplit
-dichotomy for anomalous primes comes
-up, e.g., in the theory of $p$-adic
-modular forms?
-Is there a name for
-this dichotomy in the literature?
-Aside from the obvious observation
-that the sequence $(*)$ splits if and only
-if $E(\mathbb{Q}_p)$ has a
-$p$-torsion point, are there other
-natural necessary or sufficient
-conditions for splitting?
-
-REPLY [4 votes]: If you work modulo $p^2$, the sequence $0 \to p\mathbb{Z}/p^2 \to E(\mathbb{Z}/p^2) \to E(\mathbb{Z}/p) \to 0$, splits iff $E$ is a canonical lift (as in Álvaro's answer). This comes up in cryptography (!) in the Smart-Satoh-Araki attack on the ECDLP for anomalous curves. This point is discussed in the Satoh-Araki paper, I believe.<|endoftext|>
-TITLE: Haar measure for large locally compact groups
-QUESTION [8 upvotes]: In this answer, Gerald Edgar mentions that Haar measure is naturally defined on the $\sigma$-algebra of Baire sets (the smallest $\sigma$-algebra that contains all the compact $G_\delta$ sets) of a locally compact group and that the uniqueness of Haar measure can fail for larger $\sigma$-algebras. I wonder if there is a nice example of this.
-Curious, I skimmed through Halmos's classic Measure Theory, and I found that he proves the existence and uniqueness of Haar measure for the slightly larger $\sigma$-algebra generated by all compact sets. (Confusingly, Halmos defines Borel sets to be those in this $\sigma$-algebra; I will stick with the usual definition of Borel sets.)
-
-Is there a nice example of a locally compact group where the uniqueness of Haar measure fails for the $\sigma$-algebra of Borel sets — the $\sigma$-algebra generated by open sets?
-
-To dispell some potential confusion (see comments by Keenan Kidwell and Gerald Edgar) Haar measures are not required to be regular (for the purpose of this question).
-
-REPLY [2 votes]: I may be missing something here but what François was looking for may be the following (it is of course already present in the previous answers, I only put it slightly differently):
-Let $B$ be a Borel subset of the group $\mathbb{R} \times \mathbb{R}_d$. Define $B_y = \{x : (x,y) \in B\}$. Denote the Lebesge measure on $\mathbb{R}$ by $\lambda$.
-First, let $$\mu(B) = \sum_{y \in \mathbb{R}} \lambda(B_y).$$ Second, let
-$$
-\nu(B) =
-\begin{cases}
-\sum_{y \in \mathbb{R}} \lambda(B_y) \textrm{ if } B \textrm{ only intersects countably many sets of the form } \mathbb{R} \times \{y\},\\
-\infty, \textrm{ otherwise. }
-\end{cases}
-$$
-It is easy to check that both $\mu$ and $\nu$ are translation invariant $\sigma$-additive measures on the Borel sets, taking finite values on the compact sets and positive values on the nonempty open sets. (And the open sets are inner regular w.r.t. compact sets.) Do you require anything else?
-Moreover, $\mu(\{0\} \times \mathbb{R}) = 0$, but $\nu(\{0\} \times \mathbb{R}) = \infty$.<|endoftext|>
-TITLE: For models of ZF, if for some $A$ we have $L[A] = L$, what can we deduce about $A$?
-QUESTION [7 upvotes]: Suppose $V$ is a model of ZF. Within $V$ we have $L$ which is a model of ZFC, furthermore $L[A]$ is a model of choice for every $A\in V$.
-Suppose $A=\emptyset$ then clearly $L[A]=L$, furthermore if $A\in L$ then $A\cap L\in L$, therefore $L[A]=L$. Recall also that if $A' = L[A]\cap A$ then $L[A] = L[A']$.
-On the other extreme, suppose $A$ is an amorphous set (an infinite set that every subset of it is either finite or co-finite). Consider $A'=A\cap L[A]$, we have that $A'\in L[A]$ which is a model of choice, so $A'$ cannot be infinite - since infinite subsets of amorphous sets are themselves amorphous. Therefore $A'$ is finite, despite not being able to prove that (at the moment anyway) I have a strong intuition that $A'=A\cap L$ and therefore $L[A]=L$.
-(this conjecture stems from noticing that amorphous sets are, as the name suggests, amorphous. There is no actual reason that any element would be "preferred" into $L[A]$ over another, unless it was already in $L$. Since there can only be finitely many constructible elements in an amorphous set this somewhat supports my intuition)
-Is this conjecture about amorphous sets true? Can it be extended to weaker infinite Dedekind-finite sets? Suppose $L[A]=L$, as Joel points out in his answer this implies $A\cap L\in L$.
-
-Suppose $L[A]=L$ for every $A\in V$, is there something to say about $V$ and $L$? (in the sense that $V$ is somewhat minimal over $L$ (that is if it has non-well orderable sets, then this is the only difference of $V$ from $L$))
-Suppose $V$ is somewhat larger than the above description (for example, $V$ is the Feferman-Levy model in which $\omega_1$ is singular and the reals have cardinality $\aleph_1$), is there anything to say about sets for which $L[A]=L$? Can we in some sense generate a model $L\subseteq M\subseteq V$ which behaves as described above (some minimality property)?
-
-REPLY [7 votes]: Regarding your last question, it is easy to see that if
-$A\cap L=\varnothing$, then $L[A]=L$, because at every stage, if we
-have agreement $L_\alpha[A]=L_\alpha$ so far, then having
-$A$ as a predicate doesn't help us to define any new sets
-beyond the empty predicate (since the answer for whether a
-set in $L_\alpha$ is in $A$ is always no), and so
-$L_{\alpha+1}[A]=L_{\alpha+1}$.
-More generally, for your title question, we have the following:
-Theorem. $L[A]=L$ if and only if $A\cap L\in L$.
-Proof. If $L[A]=L$, then clearly $A\cap L\in L$.
-Conversely, if $A\cap L\in L$, then one can show
-inductively that $L_\alpha[A]=L_\alpha[A\cap L]$, which is contained in $L$. That is, having $A$ as a predicate over $L_\alpha[A]$ is just as good as having $A\cap L$ as a predicate, if you know $L_\alpha[A]\subset L$. Since $A\cap L\in L$, this means that constructing relative to the predicate $A$ never leaves $L$. QED
-Note however that the hypothesis $A\cap L\in L$ does not
-imply $L_\alpha[A]=L_\alpha$, because the predicate $A$ may
-allow you to define sets more quickly, even when they are
-in $L$.
-This seems now to answer your conjecture:
-Corollary. If $A$ is Dedekind finite (in particular, if $A$ is amorphous), then $L[A]=L$.
-Proof. If $A$ is Dedekind finite, then $L\cap A$ must be finite, since otherwise we could construct a countably infinite subset using the $L$-order. Thus, $L\cap A\in L$, and so $L[A]=L$. QED
-And your updated question 1 seems to be answered by:
-Theorem. The following are equivalent:
-
-$L[A]=L$ for every set $A$.
-$V=L$.
-
-Proof. If $V=L$, then clearly $L[A]=L$ for every $A$. Conversely, if $V\neq L$, then let $A$ be any $\in$-minimal set not in $L$. Thus, $A\cap L=A\notin L$, and so $L[A]\neq L$. QED<|endoftext|>
-TITLE: The NP version of Matiyasevich's theorem
-QUESTION [32 upvotes]: By Matiyasevich, for every recursively enumerable set $A$ of natural numbers there exists a polynomial $f(x_1,...,x_n)$ with integer coefficients such that for every $p\ge 0$, $f(x_1,...,x_n)=p$ has integer solutions if and only if $p\in A$.
-Now suppose that $A$ is a set of natural numbers with membership problem in $NP$. Is there a polynomial $f$ with integer coefficients such that $f(x_1,...,x_n)=p$ has integer solutions if and only if $p\in A$ and there exists a solution with $||x_i||\le Cp^s$ for some fixed $s, C$, where $||x_i||$ is the length of $x_i$ in binary (i.e. $\sim \log |x_i|$)? Clearly the converse is true: if such a polynomial exists, then the membership problem for $A$ is in NP.
-
-REPLY [17 votes]: I think this is still an open problem. The idea of a Non-Deterministic Diophantine Machine (NDDM) was introduced by Adleman and Manders. In their paper Diophantine Complexity, they conjecture that the class of problems recognizable in polynomial time by a NDDM are precisely the problems in NP. However, they only prove the following:
-
-If A is accepted on a NDDM within time $T$, then A is accepted on a NDTM within time $T^2$.
-If A is accepted on a NDTM within time $T$, then A is accepted on a NDDM within time $2^{10T^2}$.
-
-They also show that if R0 is the problem of determining whether all even bits of a natural number are zero, then R0 is recognized in polynomial time by a NDDM if and only if all NP problems are recognized in polynomial time by a NDDM.
-PS: Technically speaking, a NDDM is not exactly of the type you ask for in your question. However, one recovers the form you desire using Putnam's trick: the equations $P(x,x_1,\ldots,x_n) = 0$ and $x = x(1 - P(x,x_1,\ldots,x_n)^2)$ have exactly the same solutions.<|endoftext|>
-TITLE: How does the right regular of GL(n, R) and GL(n,Qp) decompose?
-QUESTION [7 upvotes]: The question is contained in the title. I would guess that this question is already answered in the literature.
-Given the reductive group $GL(n)$ over a complete local field, how does the right regular representation on $L^2(G(F))$ or perhaps better on $L^2(G(F)/Z(F))$ or $L^2(G(F)^1)$ decompose, where $Z$ is the center of $G$. Even less intuitive, how does the right regular decomposition on $C_{c}^{\infty}$-version or the Harish-Chandra Schwartz space look?
-
-REPLY [10 votes]: For the Lie (a.k.a. "archimedean") case, interpreting the $L^2$ question as asking for Plancherel measure, Harish-Chandra in-principle did this for a large class of reductive groups. The early non-compact example was $GL_n(\mathbb C)$ treated by Gelfand and Naimark, where the orbital-integral idea already appeared, in a much simpler guise since there's only one conjugacy class of maximal torus. Evidently they thought that all unitary irreducibles should appear in $L^2(G/Z)$, so that it remained to the 1960s for Knapp-Stein to find the unitary degenerate unitary principal series. Effectively because of the "patching" issue between conjugacy classes, the "real" case, even just $GL(n,\mathbb R)$, is in later papers of Harish-Chandra. Knapp's Princeton book is an easy reference, or Harish-Chandra's collected works (which are out of print?)
-In-principle, Harish-Chandra and Arthur further decomposed Schwartz functions, for reductive Lie $G$. Arthur did address compactly-supported and Paley-Wiener space, but in a different style than the question, I think.
-For p-adic $G$, as commented in Silberger's Princeton notes, Harish-Chandra had a Plancherel theorem for $L^2$ and corresponding for Schwartz spaces in 1971-73, although Silberger's notes stop short of developing what was known at the time, perhaps because there was no reasonable parametrization of supercuspidals at the time. By now, with Bushnell-Kutzko and Gross-Reeder et alia, in-principle things could be assembled.
-The two articles by Moeglin in the Proc AMS 61 Edinburgh conference from 1996 address repns of $GL(n,\mathbb R)$ and $GL(n,\mathbb Q_p)$, with references, but not Plancherel.
-But I can't point to a tangible source for unadorned Plancherel for $GL(n)$'s. The Lie case can be obtained by specializing the general cases (the only discrete series on Levis are the holomorphic discrete series on the GL(2)'s). The p-adic case? Someone else?
-Edit: Looking at the Edinburgh 1996 volume, I see that Helgason has a piece in which he recaps G-N's and HC's only-one-conjugacy-class (of maximal tori) argument for "discovering" the Plancherel formula, in the Lie case. (To me, this issue of "discovery" is very important.) He also gives the first case where there's the patching issue, $SL_2(\mathbb R)$.
-Edit 2: in response to comment/further-question: the parametrization of the "spectrum" is qualitatively very similar to the automorphic $L^2$ spectral decomposition, but also qualitatively simpler. Comparing the Lie and automorphic cases roughly: in the Lie case, the "bulk" of $L^2$ is spherical unitary principal series, with parabolic-induced repns from discrete series on Levis also entering, as well as actual discrete series, if any, on the whole group. In the automorphic case, the "bulk" of $L^2$ is spherical cuspidal repns, by Weyl's Law [sic]; in many situations, provably (and conjecturally) the bulk are unitary principal series. (In general, a naive form of Ramanujan-Petersson conjectures cannot be true, for a variety of reasons... e.g., lifts, as in Howe-PiatetskiShapiro, Corvallis, AMS Proc Sympt 33, 1977/79, ... but Weyl's Law stuff still says that probably most are unitary principal series.) The parabolically induced (Eisenstein series) from afms on Levis is a smaller part of the spectrum.
-That is, in broad terms, parabolic induction from special objects on the Levis plays a completely analogous role. (This played a role in some otherwise mysterious choices of terminology in the local repn theory, e.g., "Eisenstein integrals" and "Maass-Selberg relations" in HarishChandra.)
-In both cases, there are "atoms" which are to some degree mysterious: locally, discrete series, globally, cuspforms. There are certainly global issues that have much-more-trivial local counterpart, such as Weyl's Law stuff or Ramanujan-Petersson.
-Yet again, much more can be said, and there is much literature on details. Not sooo many sources on higher-rank.<|endoftext|>
-TITLE: Shortest "painting" of the sphere
-QUESTION [5 upvotes]: Let $S$ be the sphere in $\mathbb{R}^3$ and $C:[0,1]\to S$ a continuously differentiable curve on $S$. Let $T:[0,1]\to\mathbb{R}^3$ denote the tangent vector of $C$. Let $P(t)$ be the plane containing $C(t)$ and having normal vector $T(t)$.
-Given a size $d$ of the "paint brush" we define the "brush" $b:[0,1]\to \mathcal{P}(S)$ by letting $b(t)$ be the points of $S$ that are at most a distance $d$ (metric on the sphere) from $C(t)$ that are contained in $P(t)$.
-We can think of this as saying the "brush" $b(t)$ is an arc on the sphere that is "orthogonal" to the motion $C(t)$ of the "paint brush".
-Given $d$ what is the arclength of the shortest curves such that $\cup_{t\in[0,1]} b(t) = S$. This says that the "paint brush" covered the sphere.
-
-REPLY [17 votes]: The model is that used by Henryk Gerlach and Heiko von der Mosel in their 2010 paper "On sphere-filling ropes" arXiv:1005.4609v1 (math.GT) may be relevant.
-Their question is different: What is the longest rope of a given thickness on a sphere?
-But their explicit solutions are packings, and it seems they could be converted to
-painting paths.
-Here is a piece of their Fig. 6:
-
-REPLY [5 votes]: This question is somewhat related to this recent one. More precisely, the comment by Gjergji Zaimi in the earlier question gives a painting of length $2\sqrt{2}\pi$ for $d=\pi/4$, which, as explained in another comment there, is optimal for a path at constant distance from the sphere. So for $d=\pi/4$ the optimal length should be $2\sqrt{2}\pi$.<|endoftext|>
-TITLE: Is the category of toposes cocomplete ?
-QUESTION [6 upvotes]: Hello.
-[Edits between brackets.]
-Does the [1-]category of [elementary] toposes [with logical morphisms] admit any [1-]colimits ?
-[By colimit I mean initial object in the category of outgoing cocones. I'm afraid I don't know what pseudo-colimits, bicolimits or strict 2-limits are, but I can say that I'm not interested by any 2-categorical aspect of my question (at least, for now), just by the 1-categorical one.]
-Any reference would be greatly appreciated.
-Thanks for any answer and please forgive me for my pityful english.
-
-REPLY [5 votes]: $\mathbf{Log}$, the category of elementary topoi and logical morphisms which preserve everything on the nose is cocomplete; in
-
-Eduardo J Dubuc, GM Kelly - A presentation of topoi as algebraic relative to categories or graphs - Journal of Algebra (website)
-
-this category $\mathbf{Log}$ is shown to be the category of algebras of a finitary monad on $\mathbf{Cat}$; it looks like you can even get this over $\mathbf{Grph}$, like the presentation of cartesian closed categories monadic over graphs in Lambek-Scott intro to higher-order categorical logic. This implies then that it is cocomplete, see the nLab page on colimits in categories of algebras, for example.
-For the sake of completeness, a nice account (including explicit constructions) of 2-limits in $\mathbf{Log}$ as a locally groupoidal 2-category (1-cells the standard notion of logical morphism, 2-cells between them restricted to be isomorphisms) is given in chapter III of Steve Awodey's PhD thesis,
-
-S Awodey - Logic in topoi: functorial semantics for higher-order logic - available from his website<|endoftext|>
-TITLE: Generalised linking numbers (where they shouldn't be)
-QUESTION [5 upvotes]: One can define the linking number of disjointly embedded curves $K,L\subset S^{3}$ in a variety of ways, as is discussed in Chapter 5.D of Rolfsen's "Knots and Links". One way is the Gauss Integral
-$$\mathrm{lk}(K,L) = \frac{1}{4\pi}\int_{K\times L}\dfrac{\mathbf{x}-\mathbf{y}}{|\mathbf{x}-\mathbf{y}|^3}\cdot\mathrm{d}\mathbf{x}\times \mathrm{d}\mathbf{y}$$
-(or symbols to that effect). This will be an integer when the curves are closed, and a real number in general.
-The integral formula has been generalised to deal with the case of disjointly embedded closed manifolds $K^{k} ,L^{\ell}\subset S^{k+\ell+1}$ (see here and here for example). Presumably these formulas output real numbers when the manifolds $K$ and $L$ have boundaries.
-My question concerns the situation of disjointly embedded submanifolds $K^k,L^\ell\subset S^n$, where $k+\ell >n-1$. Is there a useful notion of linking number in this case? For instance, take a surface and a curve in $3$ dimensions (so $k=2$, $\ell=1$ and $n=3$ in the above). Then we could try to define the linking number by somehow "integrating" $\mathrm{lk}(\gamma,L)$ over closed curves in $\gamma\subset K$.
-This generalised linking number should be able to measure, say, how many times a curve passes through a length of tube.
-Have such things been considered useful before? Or am I just talking nonsense?
-Added: As Ryan points out in his comment, I'm not really looking for an isotopy invariant. Also (thanks to Kevin and Tom's answers) I'm slowly coming round to the idea that a single number won't really tell you much about the relative positions of the manifolds, but maybe a matrix valued function (with rows and columns indexed by homology bases for $K$ and $L$ in the appropriate dimensions) might be useful.
-
-REPLY [5 votes]: Just to mention here the ``linking numbers'' introduced by A.Haefliger [1]: for an embedding $f\colon S^k\sqcup S^l\to S^n$ he defines the linking number as certain element $$\lambda_1(f)\in\pi_k(S^{n-k-1}\vee S^{n-l-1}).$$
-This invariant is stronger than all the homological invariants discussed above and than the $\alpha$-invariant suggested by Paul. However, it is still incomplete isotopy invariant.
-[1] A. Haefliger, Enlacements de spheres en codimension superiure a 2, Comm. Math. Helv. 41 (1966-67), 51-72 (in French).<|endoftext|>
-TITLE: Free product of Boolean algebras
-QUESTION [5 upvotes]: Given a family of Boolean algebras $\mathcal{B}=\{B_i\colon i\in I\}$ with respective Stone spaces $S_i$, the algebra of clopen (both closed and open) subsets of the product space $\textstyle\prod_{i\in I}S_i$ is called the free product of $\mathcal B$. This algebra is typically denoted by $\textstyle\bigotimes_{i\in I}B_i$ (and I will use the standard "tensor" notation for finite free products in the obvious manner).
-I am interested in the (possible) Boolean algebras which admit only very particular decompositions in terms of the free product.
-
-Is there an uncountable Boolean algebra $B$ such that if $B$ is isomorphic to $A\otimes C$ then either $A$ or $C$ is countable?
-
-REPLY [6 votes]: By Theorem 15.14 of the Boolean algebra handbook, the interval algebra of the real numbers is such an algebra B.
-Reference: S. Koppelberg. Handbook of Boolean algebras. Vol. 1. Edited by J. D. Monk and R. Bonnet. North-Holland Publishing Co., Amsterdam, 1989.<|endoftext|>
-TITLE: Which graphs are zero-divisor graphs for some ring?
-QUESTION [6 upvotes]: Given a (non commutative) ring $R$, we construct a (directed) graph $G_0(R)$ with vertex set $Z(R)\backslash \{0\}$, the zero divisors of $R$ except for $0$. And an edge from $x$ to $y$ whenever $xy=0$. This is called the zero divisor graph of $R$.
-My question is, what are the known obstructions to a graph being a zero divisor graph for some ring $R$?
-
-(Edit) Following the reference by M. Sapir below, I found several articles which give partial answers. By this result of Dolžan and Oblak we learn that there are restrictions on the diameter and girth (they work over semirings). In the case of commutative rings, one knows by a result of Belshoff and Chapman, which zero-divisor graphs are planar, and there are similar results for projective zero-divisor graphs and other genera. Another restriction is that the number of edges must be even.
-It seems that a characterisation of the set of zero-divisor graphs is open and complicated enough that is only worth studying over special subclasses of graphs. Because of this I would like to focus just on the second question below, which I have the feeling should have an easy counter-example.
-
-I'd also like to ask the same question for a weaker notion of graphs associated to a ring. Let $G_{k}(R)$ be defined for every ring $R$ and $k\in R$ as above but with edges going from $x$ to $y$ whenever $xy=k$ (we can start with the vertex set being all of $R$ and then throw away the isolated vertices).
-What graphs can be written as $G_k(R)$ for some pair $(R,k)$?
-
-Are there graphs which are isomorphic to $G_k(R)$ for some $k\neq 0$ but are not isomorphic to any zero divisor graph?
-
-REPLY [3 votes]: The answer to the second question is "no". Consider first the case of semigroups. Take the bicyclic semigroup $B=\langle a,b \mid ab=1\rangle$. It consists of elements of the form $b^ma^n$, $m,n\ge 0$ (that representation is unique because $ab\to 1$ is a confluent and terminating rewriting system, see also A. H. Clifford and G. B. Preston, "The algebraic theory of semigroups" or http://en.wikipedia.org/wiki/Bicyclic_semigroup). Take $k=1$. The corresponding graph consists of countably many disjoint edges $a^m\to b^m$, $m\ge 0$. Now suppose that this graph is the zero divisor graph of some semigroup $S$ with 0. Then it should have $p,q$ with $pq=0$. Since for every $x\in S$ we have $(xp)q=0$, we should have $xp\to q$ in the zero-divisor graph (the option $xp=0$ does not occur because in our graph there are no edges $x\to p$). Hence $xp=p$ for every $x\ne 0\in S$. Similarly for every $x\ne 0\in S$ we have $qx=q$. Therefore $q=qp=p$, a contradiction. This argument works also for rings. One needs to consider the semigroup ring $FB$ for any field $F$, and take $k=1$. The corresponding graph is not the zero divisor graph of any (associative) ring.
-In general if $R$ is a ring with 1 containing two elements $a,b$ such that $ab=1$ but $ba\ne 1$, and $a,b$ are not zero divisors, then the graph corresponding to 1 in $R$ is not a zero divisor graph of any ring $R'$. Indeed, in such a ring $R'$ we would have $ab=0, ba\ne 0$, and $xa=a, bx=b$ for every $x\ne 0$. Hence $a=ba=b$. The fact that $xa\ne 0$ (hence $xa=a$ in $R'$ is true because in $R$ we have $xa\ne 1$ for every $x$ (if $xa=1$, then $x=xab=b$, so $ba=1$, a contradiction). The fact that $xa=a$ is true because in $R$, $zb=1$ implies $z=a$ since $b$ is not a zero divisor. Similar for $bx=b$.<|endoftext|>
-TITLE: Is the complete functorial structure for Khovanov--Lee homology known?
-QUESTION [6 upvotes]: I'm interested in Lee's modification of Khovanov homology, which I'll denote $\operatorname{Kh}_{\operatorname{Lee}}^\ast$. Below $L$ is a link in $\mathbb R^3$.
-The groups $\operatorname{Kh}_{\operatorname{Lee}}^\ast(L)$ for a link $L$ are very simple: there is an isomorphism:
-$$\bigoplus_{\text{orientations of }L}\mathbb Q\to\operatorname{Kh}_{\operatorname{Lee}}^\ast(L)$$
-I write the left hand side as I do and not simply as $\mathbb Q^{\oplus 2^{\left|L\right|}}$ because given an orientation of $L$ (and a diagram of $L$), there is in some sense a natural choice of element of $\operatorname{Kh}_{\operatorname{Lee}}^\ast(L)$ (and these elements form a basis of $\operatorname{Kh}_{\operatorname{Lee}}^\ast(L)$). However, this map not natural in the sense of being functorial.
-But it's really really close to being functorial! Rasmussen proved (see Khovanov homology and the slice genus Proposition 4.1) that if we identify $\operatorname{Kh}_{\operatorname{Lee}}^\ast(L)$ with $\bigoplus_{\text{orientations of }L}\mathbb Q$, then under a cobordism from $L_1$ to $L_2$, a orientation on $L_1$ is sent to a linear combination of the orientations on $L_2$ which extend to an orientation of the cobordism agreeing with the input orientation on $L_1$. This should be viewed as a sort of approximate functoriality, and the result has been slightly refined by Rasmussen here.
-My question is: has $\operatorname{Kh}_{\operatorname{Lee}}^\ast$ been calculated as a functor? In other words, do we know a canonical way of mapping $\bigoplus_{\text{orientations of }L}\mathbb Q$ to $\operatorname{Kh}_{\operatorname{Lee}}^\ast$ so that the maps associated to arbitrary cobordisms are described elementarily in terms of orientations?
-This doesn't seem to be a deep problem; it comes down to writing down the generators explicitly in the chain complex defining $\operatorname{Kh}_{\operatorname{Lee}}^\ast(L)$, and checking what happens when we do a Reidemeister move or Morse move. However the calculations are kind of messy, and this is presumably why Rasmussen didn't pursue the point further (at least, not that I am aware of). But the last paper of his referred to above was in 2005, so I'm guessing that someone has probably straightened this out since then. I've been unsuccessful in finding a reference, though.
-
-REPLY [8 votes]: As the title suggests, the paper Fixing the functoriality of Khovanov homology by Clark, Morrison and Walker fixes the functoriality in Khovanov homology. The same techniques (disoriented surfaces) fix the functoriality in Lee homology. In fact, the paper is written in such a way that it proves functoriality simultaneously for both Khovanov and Lee homology.
-(Recall Bar-Natan's "free $\alpha$" version of Khovanov homology. If we set $\alpha = 0$, we get the original Khovanov homology. If we set $\alpha$ to any non-zero complex number, we get something identical to Lee homology, up to Euler characteristic normalizations. Lee's original paper has $\alpha = 1$ (or is it $\alpha = 8$?), but it's actually more convenient to let $\alpha = 2$.)
-The upshot is that the Lee theory, after tweaking the definition by using disoriented surfaces, is naturally isomorphic to the simple theory you describe in your question. To a link we assign the vector space generated by orientations of the link, and to an unoriented cobordism we assign the sum, over all orientations of the cobordism, of the induced map between the orientations of the incoming and outgoing boundaries of the cobordism. (If we have set $\alpha$ to something other than 2, then we need to throw in factors of $\sqrt{\alpha/2}$ raised to the Euler characteristic of surfaces.)
-If one were only interested in Lee homology, one could simply the proof in the "Fixing functoriality" somewhat, but there would still be many cases of oriented Reidemeister moves and second order oriented Reidemeister moves (movie moves) to check.<|endoftext|>
-TITLE: Category with a "metric" for arrow composition
-QUESTION [23 upvotes]: Consider a category $\mathcal C$ with a "distance" function $d:\mathcal C^2 \to \mathbb{R}_{\geq 0}$ satisfying the "triangle inequality"
-$$d(x \to z)\leq d(x \to y) + d(y \to z)$$
-for every pair of composable arrows $(x\to z)=(x \to y \to z)$.
-Let's call $(\mathcal C,d)$ a "metric" category.
-The first example is to take any category $\mathcal{C}$, and define
-$$d(f)=\begin{cases} 0 & \text{ if $f$ is an isomorphism} \\\ 1 & \text{ otherwise.}\end{cases}$$
-Then the triangle inequality simply translates the statement : "If $f=gh$, then $f$ is an isomorphism if $g$ and $h$ are isomorphisms."
-Also, it's clear that every metric space can be made into a metric category in a canonical way.
-We can define "open balls" in $\mathcal{C}$: for $c \in \mathcal{C}$, $r\geq 0$, let
-$$B(c, r) = \{d \in \mathcal{C} | \text{ there exists }f: c \to d\text{ such that }d(f) < r \}.$$
-In the category of number fields and monomorphisms, we can let $d(K \hookrightarrow L)=\log ([L:K])$. Then the triangle inequality is actually an equality. It's clear that $d$ is a good measure of "how far" $L$ is from consisting of just $K$. The open ball of radius $r$ around $K$ is the set of extensions of $K$ of degree $< e^r$.
-Is it possible to endow a big category like $\text{Top}$ or $\text{Grp}$ with a meaningful distance?
-
-REPLY [12 votes]: 1) In the category of finite sets (or finite groups or finite topological spaces....) let $d(f)$ be the cardinality of the image of $f$. This satisfies the strong triangle condition
-$$d(x\rightarrow z)\le\text{min }(d(x\rightarrow y),d(y\rightarrow z))$$
-2) In any category, for each object $x$, let $\xi(x)$ be an (arbitrarily assigned) positive real number and define
-$$d(f)=\text{min }\lbrace{\xi(c)|f \hbox{ factors through } c}\rbrace$$
-This also satisfies
-$$d(x\rightarrow z)\le \text{min }(d(x\rightarrow y),d(y\rightarrow z))$$
-3) Fix a formal language for describing arrows in your category, let $l(f)$ be the length of the shortest description of $f$, and let $d(f)=l(f)+5$. The triangle inequality follows because $g\circ h$ always has a formal description just slightly longer than the sum of the shortest formal descriptions of $g$ and $h$ (say by putting each of these descriptions between parentheses and inserting a $\circ$ between them, which adds five characters).
-In case 3), you have to allow $d$ to take the value infinity, or restrict to categories in which everything has a finite description.
-Edited to add: 4) For the category of topological spaces, you can fix a non-negative integer $r$ and let $d(f)= \hbox{rank} (H^r(f,{\mathbb Q}))$ . This requires either allowing $d$ to take the value infinity or restricting to some subcategory where the homology groups are finite dimensional.<|endoftext|>
-TITLE: Duality between proper homotopy theory and strong shape theory
-QUESTION [5 upvotes]: In the n-lab entry about shape theory one can read that
-
-Strong Shape Theory [...] has, especially
- in the approach pioneered by Edwards
- and Hastings, strong links to proper
- homotopy theory. The links are a form
- of duality related to some of the more
- geometric duality theorems of
- classical cohomology.
-
-I would be interested in any reference where I can find a precise formulation of this duality.
-EDIT: according to Gjergji Zaimi's answer the duality might be an improvement of Chapman's complement theorem. One can find it as Theorem 6.5.3 on page 230 of the book by Edwards and Hastings ("Cech and Steenrod Homotopy Theories with Applications to Geometric Topology"). Nevertheless, it seems to me that what was meant on the n-lab entry was more a cohomology type duality (like an instance of Verdier duality in the $(\infty,1)$-ctageorical context). Am I completely wrong?
-
-REPLY [3 votes]: I wrote that part of the nLab entry so can confirm that it is the Edwards and Hastings extension of Chapman's result that was referred to, but my feeling in this is that that result is the geometric form of a lot of the classical cohomological duality results and that there should be more to be said about this ... but I don't know what! Perhaps looking at the Chapman result in the light of modern homotopy theory (say using Lurie's notion of shape) may give an $(\infty,1)$-categorical result. (Note that Batanin did work on strong shape theory and produced an $A_\infty$-structure, which must relate to this. Now I like that set of ideas. Good luck if you try it!)
-(You may spur me on to write more on that entry as it has got stalled... Alternatively anyone else is welcome to write more on strong shape, of course.... and to correct any miswording, typos that they find. :-))<|endoftext|>
-TITLE: Ramification divisor associated to a cover of a regular scheme
-QUESTION [8 upvotes]: Let $S$ be the spectrum of $\mathbf{Z}$ or the spectrum of an algebraically closed field. (Actually, one can take $S$ to be any noetherian integral regular scheme.)
-Let $f:X\longrightarrow Y$ be a finite morphism of integral normal projective flat $S$-schemes which is etale above the complement of $B$, where $B\subset Y$ is a closed subscheme of codimension $1$. Suppose that $Y$ is regular.
-Example. You could take $f$ to be a finite surjective morphism of normal surfaces such that $Y$ is nonsingular.
-Since $Y$ is regular, we have a canonical sheaf $\omega_{Y/S}$. Let $s$ be a nonzero rational section of $\omega_{Y/S}$.
-Define the cycle $K_{X/S} := \mathrm{div}(s)$. Note that $K_{X/S}$ is a canonical divisor.
-Let $f^\ast s$ be the induced nonzero rational section of the line bundle $f^\ast \omega_{Y/S}$ on $X$ and consider the Weil divisor $\mathrm{div}(f^\ast s)$ on $Y$.
-Outside $f^{-1}(B)$, we have that $\mathrm{div}(f^\ast s)$ is the pull-back of $K_{Y/S}$. Therefore, there is a Weil divisor $R_f$, supported on $f^{-1}(B)$, such that $\mathrm{div}(f^\ast s) = f^\ast K_{Y/S} + R_f$.
-Question 1. How are the coefficients of $R_f$ defined?
-Question 2. Is the Weil divisor $\mathrm{div}(f^\ast s)$ a canonical divisor outside the singular locus of $X$?
-Question 3. Is $R_f$ independent of $s$?
-Note that I work with cycles and not with classes up to linear equivalence.
-
-REPLY [3 votes]: Your hypothesis imply that $\omega_{Y/S}$ is an invertible sheaf (because $Y\to S$ is locally complete intersection).
-(EDIT) As $f$ is flat at points of codimension $1$ ($Y$ is normal) and we are only interested on codimension 1 cycles, we can restrict $Y$ and suppose that $f$ is flat.
-Then the dualizing sheaf $\omega_{X/Y}$ is invertible and you have the adjunction formula
-$$\omega_{X/S}=f^*\omega_{Y/S} \otimes\omega_{X/Y}.$$
-The sheaf $\omega_{X/Y}$ is trivial outside of $B$ because $f$ is étale outside of $B$. It can be identified with the sheaf $\mathcal{Hom}_{O_Y}(f_{*}O_{X}, O_{Y})$.
-Write $\omega_{X/Y}=O_X(D)$ for some Cartier divisor $D$ on $X$. Its support is contained in $f^{-1}(B)$. For any point $\eta$ of $X$ over a generic point $\xi$ of $B$, the stalk of $\omega_{X/Y}$ at $\eta$ is given by the different ideal of the extension of discrete valuation rings $O_{X,\eta}/O_{Y, \xi}$. The valuation of the different is known to be the ramification index $e_{\eta/\xi}$ minus $1$ when the ramification is tame and bigger or equal to $e_{\eta/\xi}$ otherwise (see Serre: Local fields). So the support of $D$ is equal to $f^{-1}(B)$ and is the ramification locus by definition.
-In short, the coefficient of $R_f=D$ at the Zariski closure of $\eta$ is the valuation of the different ideal of $O_{X,\eta}/O_{Y, \xi}$. As for the computation, you can pass to the completions.
-A finite extension of complete DVR $R'/R$ is monogenous if the residue extension ($k(\eta)/k(\xi)$ in your case) is separable. If $R'=R[\theta]$, and $P(T)\in R[T]$ is the minimal polynomial of $\theta$, then the different ideal is generated by $P'(\theta)$. See Serre's book for more details.<|endoftext|>
-TITLE: Shortest paths on linked tori
-QUESTION [7 upvotes]: I will make this question specific at first, and general later.
-Suppose we have two linked tori, $T_1$ and $T_2$,
-each of radii $(2,1)$, meaning that each torus is the result of sweeping
-a circle of radius 1 orthogonally around a circle of radius 2.
-$T_2$ is rotated
-$90^\circ$ with respect to $T_1$, so that they link snugly together:
-
-
-
-
-They touch along two orthogonal circles of radius 1,
-$C_1$ and $C_2$.
-Let $T_i$ represent the surface of each torus.
-
-Q1.
-What is the structure of a shortest path $\sigma(p_1,p_2)$ on $T_1 \cup T_2$
-between two points,
-$p_1 \in T_1$ and $p_2 \in T_2$?
-
-In general I believe $\sigma(p_1,p_2)$ follows a shortest path
-from $p_1$ on $T_1$ to a point $x_1$ on one of the circles,
-travels along an arc of that
-circle, perhaps switches at their junction to the second circle,
-follows an arc to $x_2$,
-and finally follows a shortest path on $T_2$ from $x_2$ to $p_2$.
-I am wondering whether or not
-some version of Snell's law applies
-here? Suppose the path from $p_1$ makes an angle $\alpha_1$
-with one circle at $x_1$,
-and an angle $\alpha_2$ where it leaves from $x_2$ on
-that or the other circle on its way to $p_2$.
-Is it the case, with suitable conventions on the definitions
-of these angles, that $\alpha_1 = \alpha_2$?
-
-Answer to Q1: Gjergji Zaimi showed my intuition (concerning following
-arcs of the circles $C_1$ and/or $C_2$) is wrong: "The shortest path [is] the one-point union of two geodesics....
-[I]n particular you will have equal angles along the tangency circle....
-This problem is about shortest paths on a single surface in disguise."
-
-
-
-Q2.
-What is the general principle here, that would apply to several smooth
-surfaces contacting along one-dimensional curves?
-
-Speculations, ideas, references—all welcomed. Thanks!
-
-Answer to Q2: See Anton Petrunin's remarks on the graded distance function
-$|x-y|_n$.
-
-REPLY [4 votes]: This is mostly related to Q2.
-I will make comments on number of switches of geodesic from one space to an other.
-Bad case. When you glue two spaces it is useful to consider "graded distance function"
-$|x-y|_n$ --- the minimal length of curve form $x$ to $y$, which switch from one space to an other at most $n$ times.
-I used it once to prove gluing theorem for Alexandrov spaces.
-
-It is easy to work with "geodesics" for $|x-y|_n$.
-As $n\to\infty$, $|x-y|_n$ converges to the distance in the glued space.
-The function $|x-y|_n$ does not satisfy triangle inequality, but it has graded triangle inequality:
-
-$$|x-z|_{n+m} \le |x-y|_n+|y-z|_m.$$
-Good case. Some times you know that a geodesic will not switch from space to space too many times.
-Say you glue $X$ and $Y$ along $A$ and the metric induces on $A$ from $X$ and $Y$ coincide;
-in particular doubling of $X$ in $A$.
-In this case it is easy to see that at least one geodesic between any two points will not switch more than once.
-With little bit more information you may guarantee the same for any geodesic.
-Your case. You can see that whole glued space admits a strictly short retracting to each of the circles. That means that any of the geodesic switch at most once on each of two circles. So you have only two switching and that should make you happy.<|endoftext|>
-TITLE: Automorphism group of the cartesian product of two graphs.
-QUESTION [7 upvotes]: Given two (simple, undirected, finite) graphs $G_1 = (V_1, E_1)$ and $G_2 = (V_2, E_2)$, let their automorphism groups be $Aut(G_1)$ and $Aut(G_2)$.
-I'll recall that the cartesian product $G_1 \times G_2$ has vertex set $V_1 \times V_2$ , and two vertices $(a,b) , (x,y) \in V_1 \times V_2$ are adjacent iff $(a,x) \in E_1$ and b=y, or $(b,y) \in E_2$ and a=x.
-My question is: does the problem of determining $Aut(G_1 \times G_2)$ in terms of $Aut(G_1)$ and $Aut(G_2)$ has a simple answer? May you suggest some bibliographic reference about this and related problems? Basic texts about graph theory usually barely define the automorphism group, and more algebraically-oriented texts i found did not wuite answered the question.
-I tried to find a way to answer the question by myself, but i did not succeed. I'd like to know if the problem is really not-so-trivial, or if i'm simply not smart enough :)
-Thanks in advance for any comment.
-
-REPLY [12 votes]: All you need is in W. Imrich, S. Klavzar: "Product graphs: structure and recognition".
-John Wiley & Sons, New York, USA, 2000. See also: Imrich, Wilfried; Klavžar, Sandi; Rall, Douglas F. "Graphs and their Cartesian Products". A. K. Peters (2008).
-One issue is that the automorphism group of the Cartesian product of $G$ with itself
-is not isomorphic to $\mathrm{Aut}(G) \times \mathrm{Aut}(G)$, but rather to the wreath product of $\mathrm{Aut}(G)$ by $\mathrm{Sym(2)}$. But essentially this is all that can go wrong. The basic issue is to show that if a graph is connected then it has a unique factorization as a Cartesian product of prime graphs. This fact goes back to
-Sabidussi, G. (1960). "Graph multiplication". Math. Zeitschrift.(1960). 72: 446–457.
-Prime factorization can fail if $G$ is not connected.<|endoftext|>
-TITLE: Multiplicity of eigenvalues of the Laplacian on quaternionic projective space
-QUESTION [7 upvotes]: Using the classic spherical harmonics theory, one obtains the $k$-th eigenvalue of the $n$-dimensional round sphere $S^n$ to be $k(k+n-1)$, and its multiplicity is $\binom{n+k}{k}-\binom{n+k-1}{k-1}$, see e.g. [Berger, Gauduchon,Mazet, "Le spectre d'une variété riemannienne", Lecture Notes in Mathematics, Vol. 194 Springer-Verlag]. By looking at eigenfunctions of the Laplacian on $S^n$,$S^{2n+1}$ and $S^{4n+3}$ (note they are the unit spheres of $\mathbb R^{n+1}$, $\mathbb C^{n+1}$ and $\mathbb H^{n+1}$) that are respectively invariant under the natural actions of $\mathbb Z_2$, $S^1$ and $S^3$, one can obtain the eigenfunctions hence the $k$-th eigenvalue of the projective spaces $\mathbb R P^n$, $\mathbb C P^n$ and $\mathbb H P^n$ respectively. These are, respectively, $2k(n+2k-1)$, $4k(n+k)$ and $4k(k+2n+1)$.
-QUESTION: Compute the multiplicity of the $k$-th eigenvalue $4k(k+2n+1)$ of the Laplacian on $\mathbb H P^n$.
-The multiplicity of the $k$-th eigenvalue on the sphere arises as the dimension of the space of homogeneous harmonic polynomials of degree $k$ in $\mathbb R^{n+1}$, which is the difference between the dimensions of the spaces of homogeneous polynomials of degree $k$ and $k-1$ in $\mathbb R^{n+1}$. This follows from a polar decomposition of the first space as direct sum of spaces of homogeneous polynomials. The multiplicity of the $k$-th eigenvalue $4k(n+k)$ of the Laplacian on the complex projective space $\mathbb C P^n$ is given by the difference of the squares of these dimensions, it is $\binom{n+k}{k}^2-\binom{n+k-1}{k-1}^2$. This again follows from a polar decomposition of the space of harmonic homogeneous polynomials on $\mathbb C^{n+1}$. However, I do not know how to extend this idea to decompose the space of harmonic homogeneous polynomials on $\mathbb H^{n+1}$ and hence obtain the multiplicity of the $k$-th eigenvalue of $\mathbb H P^n$. This actually seems to be an exercise of [Berger, Gauduchon, Mazet]. Any natural guesses with similar differences of binomial powers seem to be wrong, since the multiplicity of the first eigenvalue (i.e., $k=1$) of $\mathbb H P^n$ should be $2n^2+3n$.
-HINT: Apparently the desired multiplicity should coincide with the dimension of the vector space formed by homogeneous polynomials $f$ in $\mathbb H^{n+1}=\mathbb R^{4n+4}$ with coordinates $(x_\alpha,y_\alpha,z_\alpha,w_\alpha), 1\leq\alpha\leq n+1$, of degree $2k$ such that $Lf=0$, where $L=\sum_j L_jL_j$ and
-$$L_1=\sum_\alpha y_\alpha\frac{\partial}{\partial x_\alpha}-x_\alpha\frac{\partial}{\partial y_\alpha}+w_\alpha\frac{\partial}{\partial z_\alpha}-z_\alpha\frac{\partial}{\partial w_\alpha}
-$$
-$$L_2=\sum_\alpha z_\alpha\frac{\partial}{\partial x_\alpha}-w_\alpha\frac{\partial}{\partial y_\alpha}-x_\alpha\frac{\partial}{\partial z_\alpha}+y_\alpha\frac{\partial}{\partial w_\alpha}
-$$
-$$L_3=\sum_\alpha w_\alpha\frac{\partial}{\partial x_\alpha}+z_\alpha\frac{\partial}{\partial y_\alpha}-y_\alpha\frac{\partial}{\partial z_\alpha}-x_\alpha\frac{\partial}{\partial w_\alpha}
-$$
-The space of homogeneous polynomials of degree $2k$ in $4n+4$ variables has dimension $\binom{4n+2k+3}{2k}$, so the desired multiplicity should be this number minus the dimension of the subspace formed by polynomials such that $Lf=0$. Note that the above operators $L_j$, when restricted to the unit sphere $S^{4n+3}$ of $\mathbb H^{n+1}$ give the three vector fields that span the distribution tangent to the Hopf fibers $S^3$.
-
-REPLY [6 votes]: These dimensions have been calculated explicitly for all the compact rank 1 symmetric spaces. See Cahn and Wolf, "Zeta functions and their asymptotic expansions for compact symmetric spaces of rank one", Commentarii Mathematici Helvetici, vol. 51 (1976), pp. 1-21.<|endoftext|>
-TITLE: On the smooth structure of the spaces of $k$-jets
-QUESTION [5 upvotes]: I was asking myself, if the following list of conditions is sufficient to determine the usual smooth structure on the spaces of $k$-jets.
-
-the map $j^k f:M\ni x\to j_x^k f\in J^k(M,N)$ is smooth, for any $f\in C^{\infty}(M,N)$;
-the map $(\alpha,\beta):J^k(M,N)\to M\times N$, defined by $(j^k_x f)\mapsto (x,f(x))$, is a smooth summersion, for any smooth manifolds $M$ and $N$;
-the composition map $\gamma:J^k(N,O)\times_{N} J^k(M,N)\to J^k(M,O)$, defined by $(j^k_{f(x)} g,j^k_x f)\mapsto j^k_x(g\circ f)$, is smooth, for any smooth manifolds $M$,$N$ and $O$, (here $J^k(M,N)\times_{N} J^k(M,N)$ is the fiber product of $\beta:J^k(M,N)\to N$ and $\alpha:J^k(N,O)\to N$);
-the map $(\alpha,\beta)^{-1}(U,V)\to J^k(U,V)$ defined by $j^k_x f\mapsto j^k_x(f|_{U\cap f^{-1}(V)})$ is a smooth isomorphism, for any open subsets $U\subset M$, $V\subset N$;
-for any open subsets $U\subset \mathbb{R}^m$ and $V\subset \mathbb{R}^n$, the map $J^k (U,V)\to U\times V\times \bigoplus_{i=1}^k{L^i_{sym}(m,n)}$, given by $j^k_x f\mapsto (x,f(x),Df(x),\ldots,(D^kf)(x))$, is a smooth isomorphism, (here $L^i_{sym}(m,n)$ is the vector space of the $\mathbb{R}^n$-valued symmetric $k$-multinear maps on $\mathbb{R}^m$).
-
-Probably it is not sufficient, or redundant, but, in such a case, I would know if there is in the literature such a kind of characterization.
-
-My question is: Once prescribed the usual smooth structure on the $J^k(U,V)$, for arbitrary open sets in euclidean spaces $U$ and $V$ (as in point 5), what kind of conditions are sufficient to uniquely determine the usual smooth structure on $J^k(M,N)$ for all other smooth manifolds $M$ and $N$?
-
-REPLY [4 votes]: Let $(U,u)$ is a chart for $M$, and $(V,v)$ be a chart for $N$. $u: U\to u(U)\mathbb R^n$ is diffeomorphism. $u(U)$ and $v(V)$ are open subset of $\mathbb R^n$ and $\mathbb R^m$. Then we can identify $$J^k(u(U),v(V))= u(U)\times v(V)\times \Pi_{j=1}^k L^j_{sym}(\mathbb R^n, \mathbb R^m)$$ People give manifold structure on $J^K(M,N)$ by chart $(J^k(U,V), J^k(u^{-1}, v))$. Main aim is to define map $J^k(u^{-1}, v)$.
-$$J^k(u^{-1}, v): J^k(U,V)\to J^k(u(U), v(V))\text{ is defined as following:}$$
-Firstly for $u:U\to u(U)$ define $J^k(u,V):J^k(U,V)\to J^k(u(U),V)$ by $ J^k(u,V)j^kf_x= j^k(fog)_{g^{-1}(x)}$. This is a well defined map and $J^k(u,V)^{-1}= J^k(u^{-1},V)$
-Now same way for $v$ define map $J^k(U,v): J^k(U,V)\to J^k(U, v(V)$. Take
-$$J^k(u^{-1}, v):= J^k(u^{-1},v)oJ^k(u(U),v)$$ This will be bijective and satisfy coordinate transformation condition:
-For details please see: First Chapter 1.1 to 1.8 of Manifolds of differential mapping: P.W. Michor.<|endoftext|>
-TITLE: Computing an example of monodromy
-QUESTION [6 upvotes]: Consider the double cover $\pi:S^1 \rightarrow S^1, z \mapsto z^2$ and the pushforward of the constant sheaf $\pi_{*}\mathbb{Z}$. This is a locally constant sheaf of rank 2, but not constant (since the space of global sections is rank 1).
-Question: if I choose a basis $u,v$ for the stalk at some point $p$, how to compute the monodromy matrix with respect to this matrix?
-
-REPLY [7 votes]: It looks like this was asked and answered a while ago. But since it floated to the top
-again, let me give another answer.
-First in classical language, the monodromy exchanges
-sheets $ \sqrt{z}\leftrightarrow -\sqrt{z}$. If you imagine taking formal linear combinations
-of these, then you should be able to recover the monodromy matrix given
-in Francesco's answer.
-Here is a more abstract general point of view.
-If $Y$ is a connected (sufficiently nice)
-space, the category of locally constant sheaves is equivalent to representations
-of $\pi_1(Y)$ via monodromy. The pullback of a locally constant sheaf
-along a covering space $\pi:X\to Y$ corresponds to
-restriction from $\pi_1(Y)$ to $\pi_1(X)$. Pushforward would be the right adjoint which
-corresponds to induction in the opposite direction. So in particular $\pi_*\mathbb{Z}$ is the regular representation of
-$\pi_1(Y)\to Aut(\mathbb{Z}[G])$ when $\pi$ is Galois with group $G$.<|endoftext|>
-TITLE: Entropy of a measure
-QUESTION [14 upvotes]: Let $\mu$ be a probability measure on a set of $n$ elements and let $p_i$ be the measure of the $i$-th element. Its Shannon entropy is defined by
-$$
-E(\mu)=-\sum_{i=1}^np_i\log(p_i)
-$$
-with the usual convention that $0\cdot(-\infty)=0$.
-The following are two fundamental properties:
-
-Property 1: $E(\mu)$ takes its minimum on the Dirac measures.
-Property 2: $E(\mu)$ takes its maximum on the uniform probability measure.
-
-Now, for some application, I am really interested in a possible generalization when $\mu$ is a finitely additive probability measure on the natural numbers.
-Question: Is it possible to define a notion of entropy of a finitely additive probability measure on the natural numbers in such a way that it verifies the following properties:
-
-it takes its minimum on the Dirac measures
-it takes its maximum on the finitely additive translation invariant probability measures
-
-Any reference? Idea?
-Thanks in advance,
-Valerio
-
-REPLY [2 votes]: I just wanted to say that topologically, your hand is forced:
-Tapio's supremum definition is the way to go.
-Give the space $X$ of all maps from $\mathcal{P}(\mathbb{N})$ to $[0,1]$
-the topology of pointwise convergence.
-The space $FAM$ of finitely additive probability measures on
-$\mathbb{N}$ is then a closed subspace of $X$.
-Let $FSM$ be the set of finitely supported measures in $FAM$.
-Let $FSA$ be the set of finite subalgebras of of $\mathcal{P}(\mathbb{N})$.
-Given $G\in FSA$ and $\mu\in FAM$, it is easy to (definably) choose $\mu_G\in FSM$ that agrees with $\mu$ on $G$: $\mu_G(\{\min(A)\})=\mu(A)$ for every atom $A\in G$.
-In $FAM$, each point $\mu$ has a neighborhood base consisting of
-sets of the form $U(\mu,G,\varepsilon)$, which I use to denote
-the set of $\nu\in FAM$ that agree with $\mu$ on $G$ up to error $\varepsilon$.
-Therefore, $FSM$ is dense in $FAM$.
-The Shannon entropy Ent is continuous on $FSM$, and
-Ent extends uniquely to a continuous map from $FAM$ to $[0,\infty]$
-given by Tapio's $Ent(\mu)=\sup\{Ent(\mu_G): G\in FSA\}$ (using my notation).
-To see this, the important step is to check that $Ent(\mu_G)\leq Ent(\mu_H)$ if $G\subseteq H$.<|endoftext|>
-TITLE: Simplest example of jumping of cohomology of structure sheaf in smooth families?
-QUESTION [20 upvotes]: Using Hodge theory (and the ill-defined Lefschetz principle), one can show that in characteristic 0, given a proper smooth family $X \rightarrow B$, the cohomology groups of the structure sheaf of the fibers are locally constant (as a function on $B$). I'm aware of the existence of a number of counterexamples to the corresponding statement in positive characteristic. Given the success of this question, I want to ask:
-
-What are the simplest examples of a proper smooth family exhibiting jumping of some cohomology group of the structure sheaf?
-
-By the above discussion, such an example will necessarily be in positive characteristic.
-By "simplest", I mean by one of the following measures.
-(best) An example whose proof is as elementary as possible, and ideally short.
-An example with a simple conceptual underpinning. (Well, the best answer would do well by both of the first two measures.)
-A known example that is simple to state, but may have a complicated proof. (Ideally there should be a reference.)
-An expected, folklore, or conjectured example.
-
-REPLY [5 votes]: By the way, for Enriques this more than just an example - it reflects their classification: the moduli space of Enriques surfaces is connected in any characteristic. All such surfaces arise as desingularizations of quotients $Y/G$, where Y is a (possibly singular) complete intersection of $3$ quadrics in $\mathbb{P}^5$, and $G$ is a finite flat group scheme of length $2$.
-In characteristic $p\neq2$, all Enriques surfaces satisfy $h^{01}=h^{10}=0$, and their moduli space is irreducible. Also, over an algebraically closed field of characteristic $p\neq2$, the only possibility for $G$ is $\mathbb{Z}/2\mathbb{Z}$, which is isomorphic to $\mu_2$.
-In characteristic $p=2$, we have $h^{01}=0$, $h^{10}=1$ if the surface arises as a quotient by $G=\mu_2$ ("classical"), we have $h^{10}=h^{01}=1$ if $G=\alpha_2$ ("supersingular"), and finally, we have $h^{01}=1, h^{10}=0$ if $G=\mathbb{Z}/2\mathbb{Z}$ ("singular"). The moduli space is connected, but has two irreducible components: one corresponds to $\mu_2$-quotients, the other to $\mathbb{Z}/2\mathbb{Z}$-quotients, and their intersection to $\alpha_2$-quotients.
-Thus, the Hodge numbers reflect the position in the moduli space, and a jumping corresponds to a change in type.<|endoftext|>
-TITLE: On the average of continuous functions $f:\mathbb{R}^2\rightarrow[0,1]$
-QUESTION [9 upvotes]: Is it true that if the average of a continuous function $f:\mathbb{R}^2\rightarrow[0,1]$ over a unit circle centered around $(x,y)$ is $f(x,y)$ for all $(x,y)\in\mathbb{R}^2$, then $f$ is necessarily constant?
-
-REPLY [16 votes]: Yes, any such $f$ is constant. In fact, if we relax the condition so that $f$ is only required to be bounded below, but not above, then it is still true that $f$ is constant. This can be proven by martingale theory, as can the statement that harmonic functions bounded below are constant (Liouville's theorem).
-Let $X_1,X_2,\ldots$ be a sequence of independent random random variables uniformly distributed on the unit circle, set $S_n=\sum_{m=1}^nX_m$ and let $\mathcal{F}_n$ be the sigma-algebra generated by $X_1,X_2,\ldots,X_n$. Then, $S_n$ is a random walk in the plane, and is recurrent. Your condition is equivalent to $\mathbb{E}[f(S_{n+1})\vert\mathcal{F}_n]=f(S_n)$. That is, $f(S_n)$ is a martingale. It is a standard result that a martingale which is bounded below converges to a limit, with probability one. However, as $S_n$ is recurrent, this only happens if $f$ is constant almost everywhere. By continuity of $f$, it must be constant everywhere.
-For the same argument applied to functions $f\colon\mathbb{Z}^2\to\mathbb{R}$, see Byron Schmuland's answer to this math.SE question.
-
-In general, for a continuous function $f\colon\mathbb{R}^2\to\mathbb{C}$, if $f(x,y)$ is the average of $f$ on the unit circle centered at $(x,y)$ then it does not follow that $f$ is harmonic. So, we cannot prove the result directly by applying Liouville's theorem. As an example (based on the comments by Gerald Edgar and by me), consider $f(x,y)=\exp(ax)$. The average of $f$ on the unit circle centered at $(x,y)$ is
-$$
-\frac{1}{2\pi}\int_0^{2\pi}f(x+\cos t,y+\sin t)\,dt=\frac{1}{2\pi}f(x,y)\int_0^{2\pi}e^{a\cos t}\,dt=f(x,y)I_0(a).
-$$
-Here, $I_0(a)$ is the modified Bessel function of the first kind. Whenever $I_0(a)=1$ then $f$ satisfies the required property. This is true for $a=0$, in which case $f$ is constant, but there are also nonzero solutions such as $a\approx1.88044+6.94751i$. In that case $f$ satisfies the required property but is not harmonic.<|endoftext|>
-TITLE: Surreal Numbers and Set Theory
-QUESTION [6 upvotes]: Hello,
-I looked through MathOverflow's existing entries but couldn't find a satisfactory answer to the following question:
-What is the relationship between No, Conway's class of surreal numbers, and V, the Von Neumann set-theoretical universe?
-In particular, does V contain all the surreal numbers? If so, then is there a characterization of the surreal numbers as sets in V? And does No contain large cardinals?
-I came across surreal numbers recently, but was surprised by the seeming lack of discussion of their relationship to traditional set theory. If they are a subclass of V, then I suppose that could explain why so few people are studying them.
-Thank you,
-Alex
-
-REPLY [5 votes]: I'm probably very late in answering this, but, in addition to what Stefan and Sridar have already said, I wanted to point out that there is another equivalent definition for a surreal number. You can think of a surreal number as a function from some ordinal $\alpha$ into your favourite two-element set (whose elements are usually denoted $-,+$). Then the order among surreal numbers defined this way is the "lexicographical" one, namely, given two surreal numbers, the bigger will be the one with bigger domain, and if the domains are equal, then look at the first place where they differ: the bigger will be the one that has a "$+$" in that place. With this definition, you don't need to worry about equivalence classes, and I personally find it much easier to work with. You can find all of this in Harry Gonshor's book, "An Introduction to the Theory of Surreal Numbers", volume 110 of the London Mathematical Society Lecture Note Series.
-Edit: I made a mistake in describing the order, so let me describe the actual order with some more detail: given a surreal number $s:\alpha\rightarrow${$+,-$}`, we abuse notation by saying that $s(\beta)=0$ for all $\beta\geq\alpha$. Now given two surreal numbers $s,t$, look at the first ordinal $\alpha$ such that $s(\alpha)\neq t(\alpha)$ (we consider the possibility that $s(\alpha)$ or $t(\alpha)$ equal $0$). Then we will say that $s < t$ iff $s(\alpha) < t(\alpha)$, with the convention that $- < 0 < +$.<|endoftext|>
-TITLE: Consistency strengths related to the perfect set property
-QUESTION [10 upvotes]: I want a model of $\mathrm{MA}_{\sigma\mathrm{-centered}}+\neg\mathrm{CH}$ in which every set of reals in $L(\mathbb{R})$ has the perfect set property. In terms of consistency strength, it is known that I need at least an inaccessible: if $\mathrm{PSP}(L(\mathbb{R}))$, then $\omega_1$ is inaccessible in $L$. I haven't been able to find any other lower bounds on the consistency strength of $\mathrm{MA}+\neg\mathrm{CH}+\mathrm{PSP}(L(\mathbb{R}))$.
-The best upper bound I can find is the fact, due to Woodin, that a measurable cardinal above infinitely many Woodin cardinals outright implies $\mathrm{Det}(L(\mathbb{R}))$. So, starting with these large cardinals in the ground, I can get what I want by forcing MA (using a "small" forcing).
-My question is, do I really need such strong hypotheses?
-The ideal answer would be "this is known; the answer can be found in...".
-One the other hand, if you can tell me with confidence that it's an open
-problem, then at least I'll know that trying to solve it isn't a waste of time.
-If it's easier to answer my question for projective sets, please do!
-Back in 1964, Solovay proved that Levy-collapsing an inaccessible $\kappa$ to $\omega_1$ forces every set of reals definable by an omega-sequence of ordinals---this includes every set of reals in $L(\mathbb{R})$---to have the perfect set property. The catch is that the Solovay model also satisfies CH.
-There's a 1989 JSL paper by Judah and Shelah (http://www.jstor.org/stable/2275017)
-that looks at the consistency strength of $\mathrm{MA}_{\sigma\mathrm{-centered}}+\neg\mathrm{CH}$ (and similar forcing axioms) in conjunction with various regularity properties for projective sets:
-Lebesgue measurability, the Baire property, and the Ramsey property.
-The perfect set property is (from my point of view) conspicuously absent.
-
-REPLY [5 votes]: $\mathrm{MA}_{\sigma-\mathrm{centered}}+\neg\mathrm{CH}+\mathrm{PSP}(L(\mathbb{R}))$ is equiconsistent with a Mahlo cardinal.
-Before Goldstern's comment, I had assumed the perfect set property was important enough to authors to mention if their theorems covered it. With that assumption falsified, I read more carefully, and determined that the Judah-Shelah paper I mentioned in the question had the answer all along. The proof of Lemma 1.1 shows that any forcing extension satisfying the hypotheses of the lemma actually has the same $L(\mathbb{R})$ as some Solovay model. Moreover, the proof of Theorem 3.1 actually builds from a Mahlo cardinal a forcing extension that satisfies $\mathrm{MA}_{\sigma-\mathrm{centered}}+\neg\mathrm{CH}$ and the hypotheses of Lemma 1.1. Therefore, a Mahlo cardinal suffices. A Mahlo cardinal is necessary because $\mathrm{PSP}(L(\mathbb{R}))$ is well-known to imply that $\omega_1^{L[x]}<\omega_1$ for all reals $x$, which in turn implies, by (ii)$\Rightarrow$(i) of 3.1, that if $\mathrm{MA}_{\sigma-\mathrm{centered}}+\neg\mathrm{CH}$ also holds, then a Mahlo cardinal is consistent with ZFC (specifically, $\omega_1$ is Mahlo in $L$ by the proof of (ii)$\Rightarrow$(i)).
-(The hypotheses of Lemma 1.1 are that $\kappa$ is inaccessible, $\mathbb{P}$ satisfies the $\kappa$-chain condition, $\mathbb{P}$ forces $\kappa=\omega_1$, and, for every subset $Q$ of $\mathbb{P}$ of size less than $\kappa$, $Q$ extends to $P'\subseteq \mathbb{P}$ such that $|P'|<\kappa$ and the inclusion map completely embeds $P'$ into $\mathbb{P}$.)<|endoftext|>
-TITLE: minimal model of a "surface" over $Spec(\mathbb{Z})$
-QUESTION [9 upvotes]: If I have a smooth compact algebraic scheme of dimention $2$ over $Spec(\mathbb{Z})$ whose generic fiber is a surface in minimal model (say of general type). Then:
-(a) Is it true that the special fibers are in minimal model as well? (I would guess the answer is no in general but at least in an open subscheme of the base this should be true).
-(b) If the asnwer to (a) is negative, does there exist some scheme whose fibers are minimal? (it is clear that in each special fiber I can do this, but I want the resulting scheme to be a smooth scheme over $Spec(\mathbb{Z})$).
-I had no luck while searching for references for this sort of questions (since this is not a general threefold) so references on the subject are more than welcome!
-
-REPLY [9 votes]: The question is local on the base, so you can replace $\mathbb Z$ with a discrete valuation ring $R$. Moreover, the Kodaira dimension and the minimality of surfaces are stable by field extension, so one can suppose the residue field of $R$ is algebraically closed.
-Then the positive answer is given in Katsura and Ueno: "On elliptic surfaces in characteristic $p$". Math. Ann. 272 (1985), Lemma 9.4 (see also Lemma 9.6). This holds under the hypothesis that the generic fiber has non-negative Kodaira dimension.<|endoftext|>
-TITLE: complete embeddings of boolean algebras and preservation of stationarity
-QUESTION [7 upvotes]: Define a complete embedding of Boolean algebra as an homomorphism of Boolean algebras which preserves also the sup and inf operations. Notice that if $\mathbb{B}$ and $\mathbb{D}$ are complete boolean algebras, $i:\mathbb{B}\to\mathbb{D}$ is a complete embedding and
-$G$ is $V$-generic for $\mathbb{D}$, then $H=i^{-1}[G]$ is $V$-generic for $\mathbb{B}$.
-I'm curious to know if the following can be the case:
-Assume $\mathbb{B}$ and $\mathbb{D}$ are complete stationary set preserving boolean algebras. Can there be two distinct complete embeddings $i_0:\mathbb{B}\to\mathbb{D}$, $i_1:\mathbb{B}\to\mathbb{D}$ such that
-if $G$ is $V$-generic for $\mathbb{D}$ and $H_j=i_j^{-1}[G]$ are the corresponding $V$-generic filters for $\mathbb{B}$ induced by the respective $i_j$, we can have that there is a name $\tau$ in the forcing language for $\mathbb{B}$ such that:
-$\|\tau$ is a stationary subset of $\omega_1\|=1_{\mathbb{B}}$
-$V[G]\models\sigma_{H_0}(\tau)$ is a stationary subset of $\omega_1$
-$V[G]\models\sigma_{H_1}(\tau)$ is non-stationary
-
-REPLY [6 votes]: Your situation can happen.
-Let $\mathbb{B}=\text{Add}(\omega_1,1)$ be the forcing to
-add a Cohen subset $S\subset \omega_1$, and let
-$\mathbb{D}$ be the forcing that first adds such a set $S$,
-and then shoots a club through it $C\subset S$. Note that
-$\mathbb{B}$ is countably closed in $V$ and therefore
-stationary-set preserving, and the generic Cohen set $S$
-that is added is both stationary and co-stationary.
-Further, the forcing $\mathbb{D}$ is stationary-set
-preserving over $V$, because by a bootstrap argument we may
-find a dense set of conditions $(s,c)$, where $s\subset
-\omega_1$ is bounded and $c\subset s\cup\text{sup}(s)$ is
-closed, and the set of such conditions in $\mathbb{D}$ is
-countably closed. Thus, $V[S][C]$ is
-stationary-set-preserving over $V$, even though it is not
-stationary-set-preserving over $V[S]$.
-Notice that we may completely embed $\mathbb{B}$ into
-$\mathbb{D}$ in the natural way, since $\mathbb{D}$ was
-described as first adding $S$, and then shooting a club
-through it.
-But we may also embed $\mathbb{B}$ into $\mathbb{D}$ in a
-different way: by first applying the automorphism of
-$\mathbb{B}$ that flips all bits. This automorphism in
-effect replaces $S$ with its complement, so that under this
-embedding, the club gets added to the complement of $S$.
-Thus, if $\tau$ is the name of the generic set $S$ added by
-$\mathbb{B}$, then $1_{\mathbb{B}}$ forces that $\tau$ is
-stationary, and with the first embedding we have that
-$\text{val}(\tau,H_0)=S$, which remains stationary and in
-fact containing a club in $V[S][C]$, but with the second
-embedding we have $\text{val}(\tau,H_1)=\omega_1\setminus S$, which is non-stationary in $V[S][C]$.
-The two embeddings correspond as you said to the two fundamentally different ways we can think about the Cohen set being treated by the club-shooting forcing, since either we shoot the club through the set, or through its complement, and this difference radically affects the stationarity of this set. But meanwhile, all the ground model stationary sets are preserved, since the composition forcing has a countably closed dense set.<|endoftext|>
-TITLE: When is an integral transform trace class?
-QUESTION [29 upvotes]: Given a measure space $(X, \mu)$ and a measurable integral kernel $k : X \times X \rightarrow \mathbb{C}$, the operator
-$$ K f(\xi) =\int_{X} f(x) k(x,\xi) d \mu(x),$$
-the operator $K$ is Hilbert Schmidt iff $k \in L^2(X \times X, \mu \otimes\mu)$!
-Q1:The main point of this questions, what are necessary and sufficient conditions for it to be trace class?
-I know various instances, where
-$$ \mathrm{tr} K = \int_X k(x,x) d \mu(x).$$
-Q2:What are counterexamples, where $x \mapsto k(x,x)$ is integrable, but the operator is not trace class?
-Q3:What are counterexamples for a $\sigma$ finite measure space, where $k$ is compactly supported and continuous, but the kernel transformation is not trace class and the above formula fails?
-Q4: Is there a good survey/reference for these questions.
-
-REPLY [6 votes]: A remark on (Q3):
-There is this famous example of T.Carleman (1916 Acta Math link) where he constructs a (normal ) operator with a continuous kernel such that it belongs to all Schatten p-classes if and only if $p\geq 2.$
-More precisely it's possible to construct $k(x)=\sum_n c_ne^{2\pi i n x}$ continuous and periodic with $\sum_n|c_n|^p=\infty$ for $p<2$. Then $Tf=f\ast k$ acting on $L^2(\mathbb T)$ yields the desired result.
-Provided some extra regularity on the kernel, the trace formula works fine (there are a lot of results in the literature)
-Regarding (Q4) I personally find C. Brislawn's result very interesting but rather difficult to implement in practice.<|endoftext|>
-TITLE: Martingale representation theorem for Levy processes
-QUESTION [9 upvotes]: Is there an equivalent of martingale representation theorem for Levy processes in some form? I believe there is no such theorem in generality, but maybe there are some specific cases?
-
-REPLY [5 votes]: Hi,
-Here is a theorem that might answer your question (it is coming from Chesnay, Jeanblanc-Piqué and Yor's book "Mathematical Methods for Financial Markets").
-It is theorem (11.2.8.1 page 621) here it is :
-(edit note : be carefull as mentioned by G. Lowther there's a typo in the book regarding the domain of integration in the conditions over $\psi$ (defined hereafter) )
-Let $X$ be an $R^d$ valued Lévy Process and $F^X$ its natural filtration. Let $M$ be an $F^X$-local Martingale. Then there exist an $R^d$-valued predictable process $\phi$ and an predictable function $\psi : R^+ \times \Omega \times R^d\to R$ such that :
--$\int_0^t \phi^i(s)^2ds <\infty$ almost surely
--$\int_0^t \int_{|x|> 1} |\psi(s,x)|ds\nu(dx) <\infty$ almost surely
--$\int_0^t \int_{|x|\le 1} \psi(s,x)^2ds\nu(dx) <\infty$ almost surely
-and
-$M_t=M_0+ \sum_{i=0}^d \int_0^t \phi^i(s)dW^i_s + \int_0^t \int_{R^d} \psi(s,x)\tilde{N}(ds,dx)$
-Where $\tilde{N}(ds,dx)$ is the compensated measure of the Lévy process $X$ and $\nu$ the associated Lévy measure.
-Moreover if $(M_t)$ is square integrable martingale then we have :
-$E[(\int_0^t \phi^i(s)dW^i_s)^2]=E[\int_0^t \phi^i(s)^2ds]<\infty$
-and
-$E[(\int_0^t \int_{R^d} \psi(s,x)\tilde{N}(ds,dx))^2]=E[ \int_0^t ds \int_{R^d} \psi(s,x)^2\nu(dx)]<\infty$
-and $\phi$ and $\psi$ are essentially unique.
-The theorem is not proved in the book but there is a reference to the following parpers :
-1/H. Kunita and S. Watanabe. On square integrable martingales. Nagoya J.
-Math., 30:209–245, 1967
-2/H. Kunita. Representation of martingales with jumps and applications to
-mathematical finance. In H. Kunita, S. Watanabe, and Y. Takahashi, editors,
-Stochastic Analysis and Related Topics in Kyoto. In honour of Kiyosi Itô,
-Advanced studies in Pure mathematics, pages 209–233. Oxford University
-Press, 2004.
-Regards<|endoftext|>
-TITLE: Torsion in triangle groups
-QUESTION [8 upvotes]: A triangle group has a presentation of the form,
-$G=\langle a, b; a^{\alpha}, b^{\beta}, c^{\gamma}, abc\rangle, \alpha, \beta, \gamma \geq 2$
-(I believe that these are also called von Dyke groups, or "ordinary" triangle groups, with triangle groups being something slightly different, but names are beside the point). I have been reading the Fine and Rosenberg paper which proves that these groups are conjugacy separable ("Conjugacy separability of Fuchsian groups and related questions"; the proof, and the statement below, can also be found in their book, "Algebraic generalizations of discrete groups"), and in it the authors state,
-"The conjugacy classes of elements of finite order...are given by the conjugacy classes {$\langle a\rangle$}, {$\langle b\rangle$}, {$\langle c\rangle$}".
-This statement is given without proof or reference. I was therefore wondering if someone could provide either a proof or a reference for this?
-I understand where the comment comes from - it is the obvious generalisation of the one-relator groups case (here we are dealing with one-relator products, which generalise one-relator groups). However, I cannot seem to find a proof of the statement in the literature, although I am sure it must be there. Unless, of course, I am simply missing something and the result is obvious...
-
-REPLY [4 votes]: In fact it's easy, and hopefully enlightening, to give a direct proof. Here's one, using the theory of orbifolds (which happens to be how I think about it). (NB I suspect it's not how Fine and Rosenberger think about it.)
-Your triangle group is the fundamental group of an orbifold $O$ with underlying space a 2-sphere and cone points of order $\alpha,\beta,\gamma$. This orbifold has Euler characteristic
-$\chi(O)= 2-(1-1/\alpha)-(1-1/\beta)-(1-1/\gamma)=1/\alpha+1\beta+1/\gamma-1$.
-For convenience, we will give the proof in the hyperbolic, ie Fuchsian, case where $\chi(O)<0$; a similar proof can be given in the Euclidean ($\chi(O)=0$) case. It seems clear that the statement is false in the spherical (ie $\chi(O)>0$) case, since then $G$ is finite. For more information about orbifolds, see Peter Scott's survey article `The geometries of 3-manifolds'.
-The orbifold $O$ is the quotient of the hyperbolic plane $\mathbb{H}^2$ by your triangle group $G$. That is to say, $G$ acts properly discontinuously and cocompactly, but not freely, on $\mathbb{H}^2$. The cone points on $O$ are precisely the images of the points in $\mathbb{H}^2$ with non-trivial stabilizers. Each cone point on $O$ has a preimage in $\mathbb{H}^2$ whose stabilizer is generated by one of the generators $a,b$ or $c$. Call these preimages $x_a,x_b,x_c$ respectively.
-Now suppose that $g\in G$ has finite order. By the classification of isometries of $\mathbb{H}^2$, it follows that $g$ fixes a point $y$ in $\mathbb{H}^2$. Thus $y$ has non-trivial stabilizer, and so for some $h\in G$, $y=hx_a$ or $hx_b$ or $hx_c$; wlog, let's say $y=hx_a$. Therefore $h^{-1}gh$ stabilizers $x_a$, and so $g$ is conjugate into $\mathrm{Stab}_G(x_a)=\langle a\rangle$, as required.<|endoftext|>
-TITLE: Number of simplicial polytopes with a given f-vector
-QUESTION [7 upvotes]: Plenty of very nice literature is available on the characterization of f-vectors of simplicial complexes of diverse sorts (results by Billera, Bjoerner, Kalai, Stanley, among others). I mention, as an example, the Dehn-Sommerville equations, the Upper- and Lower Bound Theorems, for Simplicial Polytopes.
-Are there any results on the enumeration of simplicial, convex d-polytopes with a given f-vector?
-A slightly simpler question is: How does information about an f-vector (say, specifying the number of vertices, edges and triangles) determine the amount of (simplicial, convex d-) polytopes having this numbers fixed. Are there some cases where the f-vector specifies completely the polytope?
-This is related to these posts:
-Number of graphs with a given number of nodes, edges and triangles
-What is known about the number of permissible simplicial complexes given the number of k-cells?
-And the reason I am concerned about this is that in the first of the posts, it has been commented that the problem may be way too difficult, so I was wondering whether imposing the condition that the simplicial complex be a convex polytope may simplify the situation a bit.
-
-REPLY [4 votes]: These are just some random remarks, with one hopefully useful reference.
-
-"Are there some cases where the $f$-vector specifies completely the polytope?"
-
-This is hardly what you are seeking, but for 3-polytopes, $f_2=2f_0-4$ is achieved exactly for the
-$f$-vectors of simplicial polytopes.
-And of course the stacked and cyclic polytopes achieve the lower and upper bounds respectively.
-I don't know if you have seen
-Günter M. Ziegler's
-"Convex Polytopes: Extremal Constructions and $f$-Vector Shapes"
-(IAS/Park City Mathematics Series Volume 14, 2004),
-which seems to directly address your questions, albeit as of several years ago.
-Here is the PDF.
-Here is one tidbit.
-He mentions, as a measure of our ignorance, that not even this "suspiciously innocuous
-conjecture" of
-Imre Bárány is settled:
-
-For any $d$-polytope, $f_k \ge \min \{f_0, f_{d−1}\}$.
-
-It is (or was in 2004) only proven for $d \le 6$.
-Günter has a particularly careful description of what's known about the $f$-vectors of
-4-polytopes, a specialty of his. In particular, the set of these $f$-vectors
-"is not the set of all integral points in a polyhedral cone, or even in a
-convex set." It has concavities and holes.<|endoftext|>
-TITLE: Cartan Matrices of type B and C.
-QUESTION [5 upvotes]: I was using the built-in functions for Root Systems in SAGE, and I noticed that the Cartan Matrices for Type $B_n$ and type $C_n$ are interchanged from what I thought they would be, i.e. following the Plates in the back of Bourbaki's Lie Groups and Lie Algebras, vol. 4-6.
-Are there different conventions for choosing simple roots from these two root systems? I figured that everyone followed Bourbaki in this regard, but would like to hear about any competing conventions.
-
-REPLY [10 votes]: This question (which I overlooked for a long time) reflects a natural notational confusion but is easy to answer. The Cartan integers themselves are unambiguous for each root system, but the meaning of the two indices used in writing $c_{i,j}$ is conventional and is reversed in some sources.
-For types $B,C$ that reversal leads to transposed matrices, which is what you are seeing. Bourbaki and most other sources now write $c_{i,j} := 2(\alpha_i, \alpha_j)/(\alpha_j, \alpha_j)$ relative to a symmetric bilinear form which historically derives from the Killing form; but sometimes this is instead wrutten $c_{j,i}$. For better or worse, there is no secret supercommittee dictating a single choice.
-The labels $A, B, \dots$ for the simple Lie algebras or associated root systems are of course conventional too (going back to Killing and Cartan) but have become standard by now in the literature. The subscript (often $\ell$) indicating the Lie algebra rank is conventionally restricted in an arbitrary way to avoid assigning multiple labels to isomorphic Lie algebras such as $B_2, C_2$. (You could decide to write $C_1$ and discard $A_1$, but hardly anyone does this.)
-Above rank 2 the series $C_\ell$ is assigned to the Lie algebras of symplectic groups, where there is a unique long simple root. The series $B_\ell$ belongs to Lie algebras of special orthogonal groups in odd dimensions, where there is a unique short simple root. Numbering the simple roots in a Dynkin diagram is again purely conventional (and not always the same in textbooks), but usually the last simple root $\alpha_\ell$ in these cases is the unique one of its length.
-Lie theory has developed in multiple directions, leading to many differences in notation and terminology; but fortunately everyone tends to agree on the underlying ideas.<|endoftext|>
-TITLE: Constructing rings with a desired prime spectrum
-QUESTION [9 upvotes]: Given a partially ordered set $P$, I'm interested in what is known about when $P$ is the prime spectrum of some (not necessarily commutative, not necessarily unital) ring: i.e., when does there exist a ring $R$ having $\mathrm{Spec}(R) \cong P$ (as an order isomorphism).
-Obviously some conditions will be needed on $P$, for example, that every descending chain has a greatest lower bound (because the intersection of a descending chain of prime ideals is prime). From Bergman (personal correspondence, and http://math.berkeley.edu/~gbergman/papers/pm_arrays.pdf), every finite partially ordered set can occur as a subset of the prime ideals of some commutative ring. From other as yet unpublished work, a partially ordered set can occur as precisely the prime spectrum of a (non-commutative, not necessarily unital) ring in case: (a) it has the D.C.C., (b) it is chain-finite, and (c) the set of elements covered by any given element is countable. However, none of those conditions are necessary.
-Is anyone aware of any more results on this subject?
-
-REPLY [4 votes]: In H.A. Priestley, ''Spectral Sets'' (1994), a partially ordered set P is called spectral if it occurs as the specialization order of a spectral topology. The cited paper is a survey of the known results: in particular note Theorem 1.1: a poset is spectral iff it is profinite, iff it is the spectrum of a distributive lattice.<|endoftext|>
-TITLE: Is there a name for this type of matrix? (Reference Request)
-QUESTION [5 upvotes]: I am working on a problem were I encounter matrices of the form
-$X = \begin{bmatrix}\frac{1}{1 - a_ib_j}\end{bmatrix}_{ij}$
-I am aware of Cauchy matrices, which have the form
-$X = \begin{bmatrix}\frac{1}{a_i - b_j}\end{bmatrix}_{ij}$
-(sometimes written with a plus rather than a minus). Many of the results I need I can actually obtain by factoring the above matrix as a product of a diagonal matrix with a Cauchy matrix (assuming the $a_i \neq 0$), as in:
-$X = \mathbb{diag}(a_i^{-1})\begin{bmatrix}\frac{1}{a_i^{-1} - b_j}\end{bmatrix}.$
-These matrices arise when computing solutions to matrix equations of the form
-$X - AXB^T = C$
-which are discrete-time analogs of Sylvester equations:
-$AX + XB = C.$
-(Also, related are Lyapunov equations and algebraic Riccati equations). It seems that these must appear in the literature somewhere, but I haven't been able to find them. My question is:
-
-
-Do matrices of the form $X = \begin{bmatrix}\frac{1}{1 - a_ib_j}\end{bmatrix}_{ij}$ have a name in the literature?
-
-Is anyone aware of good references for general results on these matrices? For example, there are general results on the determinant and inverses of Cauchy matrices.
-
-
-
-As I mentioned, I have already found a determinant formula and a formula for the inverse of the matrix using the factorization I mentioned above. But it would be helpful to know of further results if they exist and I would like to properly cite the literature as well.
-
-REPLY [3 votes]: Given two diagonal matrices $D_1,D_2$, matrices such that $\nabla(X):=D_1X-XD_2$ is low-rank are known in literature as Cauchy-like matrices. This includes your case, as $\operatorname{diag}(a_i^{-1})X-X\operatorname{diag}(b_j)$ is rank 1, assuming $a\neq 0$ as you did.
-Cauchy-like matrices with displacement rank (i.e., $\operatorname{rk} \nabla(X)$) equals to 1 are basically Cauchy matrices diagonally scaled from both sides, as you realized in your case, and the formulas for determinant and inverse can be adapted with little effort.
-Linear systems with all displacement rank-structured matrices, including Toeplitz, Vandermonde, and Hankel, can be solved using the GKO algorithm (Gohberg-Kailath-Olshevsky, '95).
-On related subjects, there are a book by Kailath and Sayed, "fast reliable algorithms for matrices with structure" and an old book by Heinig and Rost, but in many cases you may be better off reading directly the papers.
-Shameless self-plug: if you are interested in solving linear systems with this kind of matrices, you may also like two recent papers on the subject, one by Aricò and Rodriguez and one by myself, appeared on Numer. Algo. less than one year ago.<|endoftext|>
-TITLE: Explicit embeddings of Cappell-Shaneson knots
-QUESTION [13 upvotes]: In 1976 Cappell and Shaneson gave some examples of knots in homotopy 4-spheres and for some time these examples were considered as possible counter-examples to the smooth 4-dimensional Poincare conjecture.
-In a series of papers, Akbulut and Gompf have shown most of these Cappell-Shaneson knots actually are knots in the standard $S^4$, the most recent reference being this.
-In principle, one should be able to work through their arguments to derive a picture of these 2-knots in the 4-sphere. Has anyone done this, for any of the Cappell-Shaneson knots?
-I know various people have created censi of 2-knots, does anyone know if any Cappell-Shaneson knots appear in those censi? (I have a hard time accepting censuses as plural of census, sorry, it sounds so wrong!)
-I'd be happy with any fairly explicit geometric picture of a Cappell-Shaneson knot sitting in $S^4$. The two I'm most familiar with is the Whitneyesque motion-diagram, and the "resolution of a knotted 4-valent graph in $S^3$" picture. What I want to avoid is the "attach a handle and fuss about and argue that the manifold you've constructed is diffeomorphic to $S^4$" situation.
-
-REPLY [5 votes]: I think the explicit embedding of Cappell-Shaneson knot is given in the following paper:
-S. Akbulut and R. Kirby, A potential smooth counterexample to in dimension 4 to the Poincare conjecture, the Schoenflies conjecture, and the Andrews-Curtis conjecture, Topology 24 (1985) 375--390. (See Figure 16 of that paper)
-The paper of Aitchison and Rubinstein mentioned by Scott Carter figures out that there is an error (on the $\mathbb{Z}/2$-framing of $\gamma$-curve which turns out to be 1) in S. Akbulut and R. Kirby's former paper "An exotic involution on $S^4$, Topology 18 (1979) 1--15. Hence, what S. Akbulut and Kirby really showed (in 1979) is that the specific (or the simplest) Cappell-Shaneson sphere is obtained from the Gluck construction of a smooth 2-knot in standard $S^4$. Figure 16 of 1985 topology paper of S. Akbulut and R. Kirby describes that a smooth 2-knot is obtained from gluing two ribbon disks of a knot $8_9$.
-Finally, I would like to say that there is a same stuff given in Figure 6.2, page 17 of Kirby's famous book "The topology of 4-manifolds" Springer Lecture notes in Mathematics 1374.<|endoftext|>
-TITLE: Two-cardinal models of the random graph
-QUESTION [10 upvotes]: For a first-order theory $T$ and cardinals $\kappa < \lambda$, we say that $M$ is a $(\kappa,\lambda)$-model if it is of size $\lambda$ and has a definable (with parameters) subset of size $\kappa$.
-1) Let $T$ be the theory of the countable random graph. Which $(\kappa,\lambda)$-models does it admit?
-2) For an arbitrary $T$, what are the sufficient conditions for the existence of $(\kappa,\lambda)$ models for some $\kappa < \lambda$? This is not a question about transfer from some $(\kappa,\lambda)$ to a different $(\kappa',\lambda')$, there are quite a few theorems there. What I am asking for is some kind of a non-structure theorem, (apart from having a Vaughtian pair).
-
-REPLY [8 votes]: MR1889546 (2003e:03064)
-Cherlin, Gregory(1-RTG); Thomas, Simon(1-RTG)
-Two cardinal properties of homogeneous graphs. (English summary)
-J. Symbolic Logic 67 (2002), no. 1, 217–220.
-03C30 (03C65 05C99)
-The main result of the paper is the following theorem: If G is the Rado graph or the generic $K_{n}$-free graph, and $\kappa \leq \lambda$ are infinite cardinals, then the following are equivalent: (1) $\lambda \leq 2^{\kappa}$; (2) there is a graph $G^{\prime}$ elementarily equivalent to G of cardinality λ and a vertex $v\in V(G^{\prime})$ for which |Δ(v)|=κ; (3) there is a graph $G^{\prime}$ elementarily equivalent to G of cardinality λ and a vertex $v\in V(G^{\prime})$ for which |Δ′(v)|=κ. (Here Δ(v) is the set of neighbors of v in G∗, and Δ′(v) is its complement.)<|endoftext|>
-TITLE: p-adic Gross - Zagier
-QUESTION [6 upvotes]: Let E be an elliptic curve defined over Q. Suppose it has complex multiplication by an order of an imaginary quadratic extension K/Q and p is a prime of good ordinary reduction. Also, suppose that the sign of functional equation of L(E,s) is -1 and p splits in K. Accordingly, the anticyclotomic Katz p-adic L-function vanishes. Is there a version of p-adic Gross - Zagier formula for the two variable Katz p-adic L-function in this case? The results of Perrin - Riou seem to exclude this choice of imaginary quadratic extension.
-
-REPLY [2 votes]: Is there complex Gross - Zagier in this case? Their work also excludes this case. If there is, one can expect the p-adic height of that Heegner point to appear in p-adic version, as in the case of Perrin - Riou. There is a paper of Conrad on complex Gross - Zagier which includes this case. However, I do not know whether complex version follows from that paper. Probably some more work in needed.<|endoftext|>
-TITLE: Can the twin prime problem be solved with a single use of a halting oracle?
-QUESTION [9 upvotes]: It occurred to me that if it were possible to determine whether a given program halts, that could be used to answer the twin primes conjecture
-A) Write a program which takes input n and then counts upward until it's found n pairs of twin primes
-B) Write a program which for any input n returns true if A halts and false otherwise
-C) Write a program which counts upward running B on every n until B returns false
-D) If C halts, there are finitely many twin primes, otherwise infinite.
-I was wondering if there was a way to do this without nesting halting problems... ie if you only get one chance to ask whether a program halts, is that sufficient to answer the twin primes conjecture
-
-REPLY [2 votes]: Let $g(3)=4$, and let $g(n+1)=g(n)!$ for every integer $n \geq 3$.
-For an integer $n \geq 3$, let $\Psi_n$ denote the statement:
-if a system $$S \subseteq \{x_i!=x_{i+1}: 1 \leq i \leq n-1\} \cup
-\{x_i \cdot x_j=x_{j+1}: 1 \leq i \leq j \leq n-1\}$$ has at most
-finitely many solutions in positive integers $x_1,\dots,x_n$,
-then each such solution $(x_1,\dots,x_n)$ satisfies $x_1,\dots,x_n \leq g(n)$.
-We conjecture that the statements $\Psi_3,\dots,\Psi_{16}$ are true.
-The statement $\Psi_{16}$ proves the implication:
-if there exists a twin prime greater than $g(14)$,
-then there are infinitely many twin primes, please see:
-A. Tyszka, A common approach to Brocard's problem, Landau's problem,
-and the twin prime problem,
-http://arxiv.org/abs/1506.08655v21
-That is, assuming the statement $\Psi_{16}$,
-a simple single query to $0′$ decides the twin
-prime problem.<|endoftext|>
-TITLE: The Number of Short Vectors in a Lattice
-QUESTION [11 upvotes]: Given a lattice $L = \bigoplus_{i=1}^{m} \mathbb{Z}v_i$ (the $v_i$ are linearly independent vectors in $\mathbb{R}^n$) and a number $c > 0$, can one quickly compute or find a good estimate on the number of lattice vectors $v$ with $|v| \leq c$ without actually enumerating these vectors? The basis $v_1,\ldots, v_m$ of the lattice can be assumed to be LLL reduced.
-Also asked at: https://cstheory.stackexchange.com/questions/7488/the-number-of-short-vectors-in-a-lattice
-
-REPLY [9 votes]: For this problem one typically employs the so-called Gaussian heuristic:
-
-if $K$ is a measurable subset of the
- span of the $n$-dimensional lattice
- $L$, then $| K \cap L | \approx
-> \mbox{vol}(K)/\det(L)$.
-
-In particular, the case for $K$ a ball is used in some (enumerative) SVP/CVP solvers. See $\S 5$ of
-"Algorithms for the shortest and closest lattice vector problems"
-by Hanrot, Pujol and Stehlé.<|endoftext|>
-TITLE: non-deterministic turing machines
-QUESTION [6 upvotes]: I have one simple question:
-There is a set, which can be decided in polynomial time by a (one-band) non-deterministic Turing Machine.
-Why should there exist one (one-band) non-deterministic Turing Machine, which decides the same set, but with the additional property: There exists one natural number k, such that all the possible calculations last exactly n^k steps, if the input has the length n?
-This is one "without-loss-of-generality-assumption" in the proof of a theorem. (namely the proof of the Theorem of Fagin which says: "ESO captures NP")
-I cannot understand it. How can we manipulate the first machine to get the second machine?
-
-REPLY [6 votes]: This is possible, but it is somewhat tricky to do. Here is an outline of one way to do it...
-Start with your original one-tape Turing machine $M_0$ which runs in time $\leq k + n^k$ (say) on input of length $n$.
-First create a two-tape Turing machine $M_1$ which simulates $M_0$ on one tape and keeps track of a step-counter on the other tape. The counter is initially set to value $k + n^k$ and is decremented at each simulation step. When the simulation of $M_0$ terminates, $M_1$ keeps doing dummy moves until the counter is exhausted. Thus $M_1$ runs in exactly the same time on every input of length $n$.
-Finally, we simulate $M_1$ on a one-tape Turing machine $M_2$ as follows. Think of even cells as belonging to the first tape of $M_1$ and odd cells as belonging to the second tape of $M_1$. To keep track of where the two $M_1$ heads, each symbol will now have a plain and a red variant; there will be only two red variants at any given time and they will mark the two head positions.
-It is straightforward to simulate $M_1$ on such a tape, but the simpler ways do not simulate each step of $M_1$ in a constant number of steps since switching from one head to the other requires a variable number of moves. To remedy this, first note that $M_1$ uses less than $\ell + n^\ell$ cells of the tape for some $\ell$ that can be effectively estimated from $k$ and the above transformations. When it starts, $M_2$ reads the input length $n$ and places a freshly minted marker on the $(\ell + n^\ell)$-th cell, beyond any cell required to simulate $M_1$. Whenever $M_2$ simulates a step of $M_1$, it proceeds as follows:
-
-Starting at the base of the tape, $M_2$ finds the appropriate tape head (red symbol in even/odd position).
-$M_2$ then performs the appropriate action to simulate $M_1$, these each take a fixed finite amount of steps which may vary from operation to operation. Once this is completed, $M_2$ dances around a little so that it returns to the original tape position exactly 1001 steps after it arrived there.
-Then $M_2$ moves right until it finds the marker at position $\ell + n^\ell$ at which point it turns around and returns to the base of the tape.
-
-Although this is very inefficient, $M_2$ does correctly simulate $M_1$ and it takes exactly the same amount of time to simulate each step of $M_1$. Furthermore, $M_2$ still runs in polynomial time, though the polynomial is much worse than the original $k + n^k$.
-Edit: This answer was simplified from its original version.<|endoftext|>
-TITLE: Presentation of the dual of a locally free sheaf
-QUESTION [7 upvotes]: Let $\mathcal{F}$ be a locally free sheaf of rank $d$ on a scheme $X$ together with an epimorphism $\mathcal{O}_X^n \to \mathcal{F}$. Now due to abstract reasons (Plücker embedding, Serre's results on coherent sheaves etc.) there is a canonical exact sequence of the form $(\wedge^d \mathcal{F})^{\otimes k_2})^{r_2} \to (\wedge^d \mathcal{F})^{\otimes k_1})^{r_1} \to \mathcal{F}^* \to 0$. Here $\mathcal{F}^*$ is the dual of $\mathcal{F}$. Canonical means that the left morphism is a matrix which is built up out of exterior powers and tensor products of the given global sections of $\mathcal{F}$, in a way that does not depend on any other choices. But how can we get this sequence explicitly? I already know that there is a canonical exact sequence ${(\wedge^d \mathcal{F})^{\*}}^{\binom{n}{d+1}} \to \mathcal{O}_X^n \to \mathcal{F} \to 0$. The naive idea just to dualize it does not work since the arrows reverse their direction.
-
-REPLY [8 votes]: We have that $\mathcal F^\ast$ is, by the pairing induced by the exterior algebra, canonically isomorphic to $\Lambda^{d-1}\mathcal F\bigotimes(\Lambda^d\mathcal F)^{-1}$. Now, in general if $\mathcal H\to\mathcal G\to \mathcal F\to 0$ is exact then the kernel of the surjective map $\Lambda^\ast \mathcal G\to\Lambda^\ast\mathcal F$ is the ideal generated by the image of $\mathcal H\to\mathcal G$. Hence we get an exact sequence $\Lambda^{i-1}\mathcal G\bigotimes \mathcal H \to \Lambda^i \mathcal H\to\Lambda^i\mathcal F\to0$. Applpy, this your second exact sequence and $i=d-1$ gives a presentation of the desired type for $\Lambda^{d-1}\mathcal F$ and then twist it by $(\Lambda^d\mathcal F)^{-1}$.<|endoftext|>
-TITLE: How many integer partitions of a googol (10^100) into at most 60 parts
-QUESTION [19 upvotes]: [Ed. Prof. Zeilberger has explained why he was asking this question. In joint work with Sills he had developed one approach to this problem, and he asked this question to see how this method compared to the current state of the art. Thus in order to be most useful, answers should explain a technique for computing the number of partitions of a given number and explain how quickly that technique works on large numbers.]
-I am offering $100 (one hundred US dollars) for the EXACT number of integer-partitions of
-10^100 (googol) into at most 60 parts. The answer has to come by 23:59:59 Sat. July 30, 2011,
-by Email to zeilberg at math dot rutgers dot edu . The first correct answer would get the prize. Please have
-Subject: MathIsFun; Computational Challenge for p_60(10^100) ;
-Of course, the answer should also be posted on mathoverflow, this way people would know that
-it has been answered.
-P.S. A quick reminder, the number in question is the coefficient of q^(10^100) in
-the Maclaurin expansion of
-1/((1-q)(1-q^2)(1-q^3) .....(1-q^60))
-
-REPLY [24 votes]: Assuming the generating function is $\frac{1}{\prod\limits_{k=1}^{60}{(1-x^k)}}$ less than two hours of gp/pari computations gave the 5738 digit answer:
- p_60(10^100) =86656581294960581213175060679076908106704497466613021789269100257198717639235565191312316339492985931775559063255086528837813739104710297888706131590925777147446992989596305426558684312353832935177854304281051434707642789976633357008073006172513802039605620390971530655957695816047373679324636257282106902902642334621092094495020475520840128975825078563529533721223665030014973823745969613836273106408827327620882478583495948350012091274039702064403585282561588485459886814177864772537183125213711100412687405422437352195598054377411587222269453652608772595749758879318654363237967684143231492819869859849144643303192846597868095662984235221075244178993104547452731511881759746102529420779361469502083053597121420547771297370551721671548302500091286893415042409909386976998366647667730271589358498087377526580681061646805629741775292578165923941705511503682125111965696369869537104454897622661946756673714986529163822467578554047245492495510507170090948586721472176087890067187205068847873944550040070916438551321351396516213312259752977287529254685370643911843047105714953040605284080636692506692852386290861343553224074351341717615684759689286925891931418713944578600404523388228164949290655986905260347937245272959342219230306425929769819674715342009178833416712244795668148759773978550600441248548844394745844052334403899748841438534047816714279171794504022130682781281338860748886310942696518241739354639469191565831755281487651975490310571755217751161614063490987562368769323044232577035098658230233013237255973046519611557796486164097780600572765717192049207954645676522794492200060665810654697408765940922890681071570384648258924284066002369593013417037460844987760581310856784498330666017867379926932694902997867177534401534253516077108815999378705993544948398373531228390862058340486152600008916251487948597447302087222445574918132585218917475956102854139673493135870736987796597089097002154332556076681792911960393717282441400003891789494260210518618598398188973314601719313940864500046963578450894149843678435403526369044654570577736547494560409713817349951247573596666312987608479125353084495156167369686044416320507047788675965257815987123582326728091964213648779393844782376881912696986212395145284269498793326663270251644721155827704955889017550948057150607407340515621974551535912800395378923117298039858992279692113890732218616186715963928401879310471182014398714662911530318387903719384123094503523206696497344490492638446160520515345185475243301577733068288864511852204443105974689867573265717663992266843175679920468623776646192750361660066693548858340497070820836336832199589479748493873317664342726797884461623034017101485138334977039474244928106432671579231185307999275082811493423960476932458859071426598186180193873112974561741389243099450104083630784275091797080843197458051593402628198584022892884659157159673262136007475012211642068258586790111983004487115216316587714983184061600620955510042347951208525065607903584615122754411093720333997372104303686158116390169328647564729654602496822021852772627790051147655497081758653708323731069376329211820329894720964292204308512951279394392815381531228647996748926021980587998964261559656789935515903400714034275245390786720943646069317133415476977019995552930699089095507184260562010127210662513680400195596743618969063736866638772821443713466075147525295691845979077397586672472600601543352722962465248875034380223555307587552833073676295297462269930775994370058825449794499092333098572763536845613567842007659729575950101377792470604573148496958694358500851681021020417594633650488157803555292646903284496164708723320577897705653022386786879215494849887211690867003601123525598841552234310761205697727878524537734677467073785042424099919327513045482888697810675989587429823477178875557088606857135477182172185015519236919521892931144178066734476513787554644960056748117699500499619773294247239228662598512935502836101729578744392303964471746249068504824095375264500084786240673237030325688054879312901256140411126019876717562604656695370754259610380124733596290494150939078833029847277734279848103655035634005537012357440970178134041530010383962758911236744087548957247418495385838344050488193120198883656103937944047482394289200718538660178091001587195284171526798215466184392607070866663303013384407124487552431242699246058135314350852485524272063343281779127361940425132806724989541986633960769535303122467306918016412450564031174033910533590190086814981425286256620935490998866145273905397216624258819860307507105774061029397025943293906050905675435357400232306328073568576501790379120117667898974043944187447981387549878960713202932947775104921101559396576930010856892403392376903132049196798179726054506703688678257002906484375930174337243347713509605577865904845933780545352176065540746989843910448263598464743883743439272998551440273145595918471962680847257889187644711316597456792335590005247303224359521888113127081850435754309177603997607819802516595862380433682384768629515466542262017881243664146776012340346962436435902670624415630984638855161435213820664151078798248565386059057915029104772346437888653030556324179188240674032973292350468414546941023474634728292600948182617121556982271419859775728848796694902653386079448715972566638129584559954562361057167423402068307675741886341592218981106331969215861131581250781124740005504878113203950194519164264383901294959862497186017815081914576467308984426624348931287895061495731260738434212734724426778408532295618503758135224149897249857036284783900183976954684955435242114417114172532305756633722611320470393047211480479720666959127636504139868217145958399715530772463153560598077341901760982207337893156549165428179577945947050066009093850246495381004264287660200535241771035068278713633310388540
-
-I am not sure this is correct at all, yet Doron seems happy.
-EDIT: Here is how the number was computed. The generating function $\frac{1}{\prod\limits_{k=1}^{60}{(1-x^k)}}$ means the sequence satisfies linear recurrence with constant coefficients. These are known to be efficiently computable assuming arithmetic operations in the range of the result are tractable. A good computational resource for recurrences is the free book "Matters Computational" was: "Algorithms for Programmers" by Jörg Arndt. Basically the method is fast binary exponentiation of a matrix or in $\mathbb{Z}[x]/poly(x)$. The book has code parts of which I used. Got a linear recurrence of order 1830. My gp/pari code is here. A curiosity of the challenge is the result is so small - I wouldn't even try $fibonacci(10^{100})$.
-To my knowledge the monetary bounty was donated to Wikipedia by Doron Zeilberger per agreement with the recipient.
-EDIT2 A closed form possibly leading to faster approach (avoiding computing the recurrence) appears in the paper
-A GENERAL METHOD FOR DETERMINING A CLOSED FORMULA FOR THE NUMBER OF PARTITIONS OF THE INTEGER $n$ INTO $m$ POSITIVE INTEGERS FOR SMALL VALUES OF $m$, W. J. A. COLMAN
-
-REPLY [4 votes]: Can you do p_60(10^1000)? p_60(10^10000)? – Doron Zeilberger
-8.6656581294960581213175060679076908106704497466613.. * 10^5737 Dollar 100
-8.6656581294960581213175060679076908106704497466613.. * 10^58837 Dollar 1000
-8.6656581294960581213175060679076908106704497466613.. * 10^589837 Dollar 10000<|endoftext|>
-TITLE: Finite dimensional vector spaces over a complete but not-necessarily-valued field
-QUESTION [7 upvotes]: I'm essentially reopening this old question of Ricky Demer which was never fully answered.
-Essentially the original question: Suppose we have a topological field $F$ which is complete, Hausdorff, and non-discrete, and we put a Hausdorff topology on $F^n$ so as to make it a topological vector space over $F$; is is this topology necessarily the product topology (and hence complete, and hence closed in anything it embeds in)?
-As the link shows, the answer is yes if $\tau$ comes from an absolute value on $F$, and it's easy to see the same argument works if it comes from a field ordering on $F$.
-Note that the argument there shows that this question reduces to the following lemma:
-Suppose we have a topological field $(F,\tau)$ which is Hausdorff and non-discrete, and we give $F$ a second topology $\tau'$, which is also Hausdorff, such that $(F,\tau')$ is a topological vector space over $(F,\tau)$. Does this force $\tau=\tau'$? What if we assume that $(F,\tau)$ is complete?
-(I'm isolating completeness as a separate, possibly-unnecessary condition because the argument there only uses completeness in the reduction to the lemma, not in proving the lemma for valued fields.)
-Related to this question in that one way to come up with a counterexample for both simultaneously would be to find a field with two (nondiscrete, Hausdorff) topologies with one strictly finer than the other.
-
-REPLY [2 votes]: Well, now I feel silly -- on looking through Wieslaw again, I see he does give examples of non-discrete, non-straight fields, just not in that section. For instance, take two absolute values on the rationals; the topology they generate together still make the rationals a topological field, is not discrete, and is obviously finer than either of the ones you started with. Since it isn't even minimal, it can't be straight.
-I'm not going to accept my own answer on this since this example presumably isn't complete, and I'd like a complete example. Which might well be in here too, if I keep looking... (Is the completion of this again a field? If so that should work, but I'd need to check if that's true.)
-Edit: Nope, the completion of this isn't a field, so I'm still lacking for a complete example.
-Edit again: OK, I'm now pretty sure Wieslaw gives one, so the answer is no. Wieslaw gives an example of a complete normed field which is not given by any absolute value (here "normed" means instead of |ab|=|a||b|, we only require |ab|≤|a||b| and |-a|=|a|). Furthermore, he shows given a power-multiplicative norm, you can find an absolute value that generates a coarser topology (so if a power-multiplicative norm wasn't equivalent to an absolute value, it isn't minimal). (Here power-multiplicative only means we require |an|=|an| for positive n, not for negative n.) And after a bit of staring at his example, I'm pretty sure it's power-multiplicative. So unless anyone can show that I've missed something, I'm going to consider this one closed.<|endoftext|>
-TITLE: Ph.d thesis assessment
-QUESTION [5 upvotes]: Hi!
-I would like to put my thesis (in Poisson geometry) online for an assessment, so I can receive feedback to improve the manuscript. Can you help me to find the right place to my request?
-Thank you.
-Another related question, may one attach some pdf files to questions here?
-
-REPLY [2 votes]: Hello,
-With the help of a nice person, I posted my thesis on Arxiv:
-http://arxiv.org/abs/1108.0452
-I will be grateful to any person who could send me comments to improve the manuscript.<|endoftext|>
-TITLE: non-rigidity of interior points in polyhedral triangulations?
-QUESTION [7 upvotes]: It's well-known that any compact polyhedron $P$ in $\mathbb{R}^n$ (we talk about piecewise-linear setting there, i.e. $P$ is a finite union of compact convex polytopes) can be triangulated into (geometric) simplices, although sometimes it is necessary to add "extra points" in $P$ to serve as vertices of simplices in the triangulation $\mathcal{T}$. E.g. Schönhardt polyhedron requires such extra points. (Here By $\mathcal{T}$ we mean a partition of $P$ into finitely many simplices $T\in\mathcal{T}$ --- more precisely, the interiors $int(T)$ of $T$'s do not intersect, and the closure of $\cup_{T\in\mathcal{T}}int(T)$ equals $P$.
-The vertices of $T$'s that are not vertices of $P$ are these "extra points" we talk about.)
-It looks correct that one can always construct such a $\mathcal{T}$ so that each extra point in it is "non-rigid", i.e. it can be continuously moved inside an open subset of the face of minimal dimension it is inserted into, so that after such a deformation $\mathcal{T}$ remains a triangulation of $P$.
-Is this indeed correct, and can anyone point out a reference?
-Added: A weaker form of the question: show that each extra poing in $\mathcal{T} $ is not prescribed, i.e. for any vertex $y$ of $\mathcal{T}$ which is not a vertex of $P$ there exists another triangulation of $P$ which does not have $y$ as a vertex. [This still suffices for our purpose, of showing that a part of certain kind of moment generating functions, for moments of a uniform measure supported on $P$, does not depend upon $\mathcal{T}$ ].
-This is easy to see that $y$ lying in the interior of $P$ is not prescribed---one can directly construct a new triangulation not using $y$, by choosing the points of intersection of the edges on $y$ with a sufficiently small sphere around $y$ and re-triangulating new convex pieces without using $y$. But it is not obvious for $y$ lying in a proper face of $P$.
-
-REPLY [2 votes]: The answer for arbitrary polyhedra is no. If a 4-dimensional polyhedron has a 3-dimensional Schönhardt polyhedron as one of its faces, there will need to be a new vertex added somewhere within that face, which will not be free to move in an open set.
-I believe that the answer is yes in 3d and yes to higher-dimensional polyhedra all of whose faces are already simplices, but I don't have a proof handy.<|endoftext|>
-TITLE: Entropy of the Ising model
-QUESTION [8 upvotes]: Consider the standard Ising model on $[0,N]^2$ for $N$ large. By that I mean the square-lattice Ising model without external field, inside an $N$-by-$N$ square. What is its entropy for $N$ large? It must behave asymptotically as $c(\beta)N^2$ for some constant $c(\beta)$ depending on the inverse temperature $\beta$. What is $c(\beta)$? Has it been computed?
-
-REPLY [6 votes]: To expand on Steve Huntsman's comment, the entropy follows from Onsager's result for the free energy per site, $F=$
-$$
--\beta^{-1}\left[\ln 2+ \frac{1}{2}\frac{1}{(2\pi)^2}\int_0^{2\pi}d\theta_1\int_0^{2\pi}d\theta_2 \ln(\cosh2\beta E_1\cosh2\beta E_2
--\sinh2\beta E_1\cos\theta_1-\sinh2\beta E_2\cos\theta_2)\right],
-$$
-and the thermodynamic relation,
-$$
-S=-\frac{\partial F}{\partial T},
-$$
-for the entropy per site. Here $\beta=1/(k_BT)$ and $E_1$ and $E_2$ are the horizontal and vertical interaction strengths. If you set both interaction strengths equal to 1 and use units where Boltzmann's constant equals 1, then the critical temperature is $2/\ln(\sqrt2 + 1)\approx2.269$. If you plot $S$, you should find that it interpolates between 0 at low temperature and $\ln2$ at high temperature, as expected. At the critical temperature, the graph has infinite slope.<|endoftext|>
-TITLE: Probability that a Turing machine is universal?
-QUESTION [11 upvotes]: I choose a Turing machine T with n states and an input tape at random.
-What can be proven about the probability P_A(n) that it is not decidable whether T will halt for a particular input? What can be proven about P_B(n) that T is universal, that means that there exists an algorithm that takes an obvious encoding of an arbitrary Turing machine A (without input tape) and transforms it into an input for my random Turing machine, so that T halts with this input if and only if A halts? In particular: Is it known that P_A(n) is strictly smaller than P_B(n) for some n?
-
-REPLY [19 votes]: The answer will depend on which model of Turing machine you
-have adopted.
-For example, here is one easy thing to say. Suppose that
-your Turing machines have alphabet $\{0,1\}$, a set $Q$
-of $n$ states, a single halt state (not counted inside
-$Q$), and the ability to move the head left and right, so
-that a program is a function $p:Q\times\{0,1\}\to
-(Q\cup\{\text{halt}\})\times\{0,1\}\times\{\text{left},\text{right}\}$,
-where $p(s,i)=(t,j,\text{left})$ means that when in state
-$s$ reading symbol $i$, the program changes to state $t$,
-writes symbol $j$ and moves left. In particular, for this
-model the total number of programs is $(4(n+1))^{2n}$.
-Consider now the collection of programs that have no
-transition to the halt state. The total number of such
-programs is $(4n)^{2n}$, and the interesting thing here is
-that $$\lim_{n\to\infty}{(4n)^{2n}\over (4(n+1))^{2n}}=\lim_{n\to\infty}[{n\over
-n+1}]^{2n}=\frac{1}{e^2}$$ which is about 13.5%.
-Thus, for this model of computation, at least 13.5% of the
-programs never halt on any input and are not universal.
-The topic is fun, because we are in effect considering the
-behavior of a random program, where each new program line
-is chosen randomly from among all the legal program lines.
-And such kind of argument is the main theme of my article
-(J. D. Hamkins and A. Miasnikov, The halting problem is
-decidable on a set of asymptotic probability one, Notre
-Dame J. Formal Logic 47, 2006.
-http://arxiv.org/abs/math/0504351), which came up also in a
-few other mathoverflow questions: Solving NP problems in
-(usually) polynomial
-time?,
-Turing machines the read the entire
-tape?.
-The main theorem of that article is the following.
-Theorem. There is a set $A$ of Turing machine
-programs (for machines with one-way infinite tape, single
-halt state, any finite alphabet) such that:
-
-One can easily decide whether a program is in $A$; it is polynomial time decidable.
-Almost every program is in $A$; the proportion of all
-$n$-state programs that are in $A$ converges to $1$ as $n$
-becomes large.
-The halting problem is decidable for members of $A$.
-
-Thus, there is a decision procedure to decide almost every
-instance of the halting problem. The way the proof goes is
-to calculate, for any fixed input, the probability that a
-Turing machine will exhibit a fatally trivializing behavior
-(falling off the left end of the tape before repeating a
-state), and observing that in fact this occurs with
-probability $1$. Basically, the behavior of a random Turing
-machine is sufficiently close to a random walk that one can
-achieve the Polya recurrence phenomenon.
-A corollary to this proof, answering your question, is that
-for this model of computation, the probability that an
-$n$-state program is a universal program goes to zero as
-$n$ becomes large, since almost every program exhibits the
-trivializing behavior, which is incompatible with being
-universal. Furthermore, the set of programs with that
-behavior is decidable.
-The theorem can be extended to other models of computation,
-such as the model with two-way infinite tapes and halting
-determined by specifying a subset of the states to be
-halting states. In this model, as you can guess, machines
-are likely to halt very quickly (since each new state has
-50% chance of being halting), and so there is a large set of programs for which the halting problem
-is decidable for this reason (they halt before they repeat a state).
-Let me also mention, since you asked not merely about the
-probability of halting, but also about the probability of
-decidability of halting, that every computably enumerable
-set $B\subset\mathbb{N}$ that is not computable admits
-infinitely many $n$ for which $n\notin B$ but this is not
-provable in whatever fixed background theory you prefer,
-such as PA or ZFC or ZFC + large cardinals. The reason is
-simply that if non-membership in $B$ were provable for all
-sufficiently large $n$, then we would have a decision
-procedure for $B$ by searching either for $n$ to be
-enumerated into $B$ or else searching for a proof that $n$
-is not in $B$, and this contradicts our assumption that $B$
-is not decidable.
-Thus, every c.e. non-computable set sits in a halo of
-undecidability: there are infinitely many numbers $n$ that
-are not in $B$, but for which it is also consistent with
-your favorite theory that they are in $B$.<|endoftext|>
-TITLE: Regular Conditional Probability given a natural filtration of a stochastic process
-QUESTION [5 upvotes]: OK, this is kind of re-posting, but I think I can clarify the question more, so it's worth a shot.
-Consider a real valued process $(X_t)_{t \leq T}$, cadlag on a probability space $(\Omega, (\mathcal{F}^\circ_t)_{t \leq T}, \mathbb{P}). \mathcal{F}^\circ_t=\sigma(X_s;s\leq t)$ is the uncompleted, natural filtration generated by $X_t$. Unfortunately $X_t$ neither has independent increments, nor is it markov. Since $\Omega$ is a Polish space, $\mathcal{F}^\circ_T$ and also $\mathcal{F}^\circ_t$ are countably generated, so we know, there exists a regular version of the conditional probability of $\mathbb{P}$ for any fixed $t$ for $\mathbb{P}$-a.a. $\omega$, i.e. for fixed $t$, $\mathbb{P}(\cdot|\mathcal{F}_t)(\omega)$ is a prob. measure f.a.a. $\omega$.
-Hence we know, that for all $t\in [0,T]\cup \mathbb{Q}$, we find a regular conditional probability f.a.a. $\omega$, depending on $t$. In words: Given almost any path of the process up to time $t$, we can deduce the probablity of events, taking that information into account.
-On the remaining $\omega$'s, define some meaningless measure, so we have a measure $\forall \omega$. How can I extend this to all $t$ in a reasonable way? Reasonable means: There is one Null set $N$, so that $\forall t$ $\mathbb{P}(\cdot|\mathcal{F}_t)(\omega)$, $\omega\in N^c$, is a measure Anybody seen anything like this?
-I read something like this only for Markov and Feller processes using infinitesimal generators, but this cannot be carried over one to one, because we do not have a transition semigroup.
-Maybe I have a deep misunderstanding here. Grateful for any objections, hints and comments.
-
-REPLY [6 votes]: Let's assume that we are working with the canonical probability space $\Omega = D(\mathbb R)$ of càdlàg functions, and $\mathbb P$ is the law of the process. I would doubt that there is a satisfactory answer at the level of maximal generality you've stated. At the very least, the measure $\mathbb P$ should be Radon. There are extremely general results on the existences of RCPs for Radon measures (cf. Leão, Fragoso and Ruffino, Regular conditional probability, disintegration of probability and Radon spaces).
-The RCP is a measure-valued function $P : [0,T] \times \Omega \to \mathcal M(\Omega)$ such that for $\mathbb P$-almost every $\omega$, the measure $P(t,\omega, \cdot)$ is a version of $\mathbb P(\cdot|\mathcal F_t)$. Do you want the function $(t, \omega) \mapsto P(t,\omega,\cdot)$ to simply exist and be measurable? If so, this can be done in the wide generality stated above; see Leão et al.
-Recently, I have needed more regularity properties for RCPs, namely, continuity. Consider the space $\mathcal M(\Omega)$ of Radon measures on $\Omega$ equipped with the topology of weak convergence of measures. We say that the RCP is a continuous disintegration (or continuous RCP) when it satisfies the following property: $$\mbox{if $\omega_n \to \omega$, then the measures $P(t,\omega_n,\cdot)$ converge weakly to $P(t,\omega,\cdot)$.}$$
-If the law is Gaussian, then my preprint Continuous Disintegrations of Gaussian Processes gives a necessary and sufficient condition for the law $\mathbb P$ to have a continuous disintegration. I haven't thought about this in the case of càdlàg functions, but I'm pretty sure that this will extend easily. Note that this is just for fixed $t$.
-To show that the map $(t, \omega) \mapsto P(t,\omega,\cdot)$ jointly continuous, a little more work is needed. As part of a larger project, Janek Wehr and I have general results in this direction for stationary, Gaussian processes. If this is what you need, I'm happy to discuss this with you further.
-Open Question: If the law $\mathbb P$ is not Gaussian but at least is log-Sobolev, then all the same results should hold. This is because log-Sobolev measures satisfy very strong concentration-of-measure properties. I have some ideas how to do this, but I haven't worked out the details because I've been busy with other projects. If anybody is interested in collaborating on extending this work to the log-Sobolev case, please contact me.<|endoftext|>
-TITLE: Algebraic axiomatization for AB+BA^T operation on matrices
-QUESTION [9 upvotes]: Let us consider a matrix algebra $Mat_{n\times n}(K)$, where $K$ is a field, $char K \neq 2.$
-It is well-known that the axiomatization of commutator operation $[A,B]=AB-BA$ on matrix algebra leads us to the theory of Lie algebras.
-Axiomatization of $A\circ B= \frac{1}{2}(AB+BA)$ leads us to Jordan algebras.
-Let us consider an operation $A \Box B= \frac{1}{2}(AB+BA^T),$ arising, for example, in control.
-How we can describe a class of algebras arising from axiomatization of such an operation?
-UPDATE As it was shown by Pasha Zusmanovich below considering only $\Box$ leads us to a trivial variety(of course we can try to proceed to a quasivariety..).
-But, if we add a transposition to the signature situation becomes much more interesting.
-First of all we have $(A\Box B)^T=A\Box B^T$ and the left unit $I\Box A= A$
-Really, if we consider $T$-invariant subalgebras of some matrix algebra with $\circ$, than we can note that such algebras could be decomposed (as vector spaces) to the direct sum of Jordan algebra and Lie algebra -- symmetric and antisymmetric part,respectively.
-Axiomatizing this decomposition we get...
-Commutativity for symmetric part:
-$$
-(A+A^T)\Box (B+B^T)=(B+B^T)\Box (A+A^T),
-$$
-Power-associativity for symmetric part:
-$$
-(A+A^T)\Box ((A+A^T)\Box (A+A^T))= ((A+A^T)\Box (A+A^T))\Box (A+A^T)
-$$
-Jordan identity for symmetric part:
-$$
-((A+A^T)\Box (B+B^T))\Box ((A+A^T)\Box (A+A^T))= (A+A^T)\Box ((B+B^T)\Box ((A+A^T)\Box (A+A^T)))
-$$
-Anticommutativity for antisymmetric part:
-$$
-A\Box A+A^T\Box A^T=A\Box A^T+A^T\Box A
-$$
-Lie identity for antisymmetric part:
-$$
-(A-A^T)\Box ((B-B^T)\Box (C-C^T))+(C-C^T)\Box ((A-A^T)\Box (B-B^T))+(B-B^T)\Box ((C-C^T)\Box (A-A^T))=0
-$$
-Commutativity and power-associavity for symmetric part could be seen as averaged commutativity and averaged associativity and(!) commutativity, respectively.
-$$
-A\Box B + A\Box B^T + A^T\Box B + A^T\Box B^T= B\Box A + B^T\Box A + B\Box A^T +B^T\Box A^T
-$$
-$$
-\sum_{\sigma\in S_3}\sum_{(i,j,k)\in (\varnothing,T)^3}A_{\sigma(1)}^{i}\Box(A_{\sigma(2)}^j\Box A_{\sigma(3)}^k)
-=\sum_{\sigma\in S_3}\sum_{(i,j,k)\in (\varnothing,T)^3}(A_{\sigma(1)}^{i}\Box A_{\sigma(2)}^j)\Box A_{\sigma(3)}^k
-$$
-Did anyone consider something close to varieties of algebras with identities of that type?
-
-REPLY [3 votes]: Including the operation $A\mapsto A^T$ can be viewed, in the language of operads, in many different closely related ways: via adjoining a new unary operation $J$ that satisfies $J^2=id$ and $J(ab)=J(b)J(a)$; via considering operads over the semisimple algebra $\mathbb{C}[t]/(t^2-1)$, via splitting everything into symmetric/antisymmetric, and considering the corresponding coloured operads (this looks like what you are doing in the examples of identities you give) etc. For either approach, you will find some papers dealing with similar things, though not necessarily literally the structure you are asking about. One way to try and list the possible identities would be to use operadic Groebner bases, - this way, for example, it is possible to show that for pre-Lie algebras the symmetrised operations does not satisfy any identities (http://arxiv.org/abs/0907.4958), but if there are identities, then one can detect them too, using an appropriate ordering. (It's like Groebner bases in the case of commutative algebras: if you are solving a system of polynomial equations and suspect that on all solutions one of coordinates $z_i$ has finitely many values, Groebner bases can detect that, and produce an equation in one variable that $z_i$ satisfies.)<|endoftext|>
-TITLE: Is the singular locus ideal preserved by all derivations?
-QUESTION [17 upvotes]: Let $R$ be a commutative ring, with whatever hypotheses let you answer the question -- e.g. Noetherian, local, finitely generated over $\mathbb C$.
-Let $I$ be the ideal defining the singular locus in Spec $R$. (With the reduced subscheme structure, or defined using minors of a Jacobian matrix, again whatever helps.)
-Is it obvious and/or true that any derivation $d:R \to R$, i.e. additive map satisfying the Leibniz rule $d(ab) = a\ db + b\ da$, has $dI \leq I$?
-Morally, $d$ is defining an infinitesimal automorphism of Spec $R$, and the singular locus should be preserved by automorphisms. So I would have hoped that there was a mindless proof using the Jacobian, but I haven't found one. As usual, a reference would be even better than a proof.
-
-REPLY [12 votes]: The fact that the set-theoretic singular locus is preserved is true only in characteristic zero: for example if you take the curve ($x^p = y^2$) in $\mathbb{A}^2$, then the ideal $(x, y)$ is not preserved by the derivation $d/dx$ in characteristic $p$. On the other hand, the Jacobian ideal in this case, $(y)$, is preserved.
-In characteristic zero one can prove that the set-theoretic singular locus is preserved by exponentiating derivations: given a derivation $D$ of $\mathcal{O}_X$ one can consider $e^{t D}$, a derivation of $\mathcal{O}_X[[t]]$, and the composition of this with a character $\mathcal{O}_X \to k$ corresponding to a singular point gives a character $\mathcal{O}_X((t)) \to k$ also with the same dimension of tangent space, hence corresponding to a singular point. Therefore the singular locus is preserved, since the set-theoretic singular locus is the intersection of all such kernels.
-This theorem is due to, I believe, Seidenberg: it is the corollary to Theorem 12 in "Differential ideals in rings of finitely generated type" in AJM 1967.<|endoftext|>
-TITLE: Expressing Galois actions on fundamental groups explicitly
-QUESTION [6 upvotes]: Let $X$ be some variety over $\mathbb{Q}$, and let $\pi_1(X\times_{\mathbb{Q}}\mathbb{C},x)$ denote its (topological) fundamental group. As is well known $Gal(\mathbb{Q})$ acts on this fundamental group. I was browsing old MSRI videos, and in the middle of one of them I saw an intriguing explicit description of this action:
-http://www.msri.org/realvideo/ln/msri/1999/vonneumann/schneps/1/main/08.html
-(you don't have to know anything from earlier in the talk to understand that page)
-As it says there, there was also a talk by Ihara about this. I'm looking, however, for an explanation of this in a more systematic way, in a paper or a book. Do you know of a good reference for this?
-
-REPLY [4 votes]: Aha, now I think I have a better picture of what you're looking for. I would look at Matsumoto's notes from the Arizona Winter School program on Galois groups and fundamental groups:
-http://math.arizona.edu/~swc/notes/files/05MatsumotoNotes.pdf
-especially sections 2.2 and 4.1.<|endoftext|>
-TITLE: Probabilistic (and other mathematical) methods of physics without the physics?
-QUESTION [13 upvotes]: Many of the methods of physics are vastly more general than their use in that discipline. For example, information theory overlaps with a lot of statistical mechanics, and the latter actually developed first. ET Jaynes wrote a famous paper illustrating the connections. However, each is comprehensible without the language and intuition of the other (though I do not deny that a richer understanding comes from knowing both).
-What other methods of physics (particularly those with a statistical or computational bent) have interpretations (Please mention useful introductory texts!) that are completely physics free? I understand that various field theories meet this criterion; any good non-physics introductions?
-
-REPLY [2 votes]: I found Complexity and Criticality by Christensen and Moloney to by quite excellent. It gives a much more computational approach to the percolation phase transition, the ising phase change and issues of self organized criticality (via the sand pile and rice pile model).
-As a computer science student, I found this book to be invaluable for my work on phase transitions of NP-Complete problems.<|endoftext|>
-TITLE: Are there examples of statements that have been proven whose consistency proofs came before their proofs?
-QUESTION [35 upvotes]: I'm wondering if there are examples of statements that have been proven whose consistency proofs came before the proofs of the statements themselves.
-More informally, I'm wondering how promising in general is the approach of attempting a consistency proof for a statement when faced with a statement that seems true but difficult to prove.
-Background:
-If a statement is provable from a set of axioms, then that statement is obviously consistent (assuming the set of axioms is consistent). So provability is stronger than consistency. This might lead one to think that constructing a consistency proof for a statement should be strictly easier than constructing a proof.
-Yet consistency proofs (at least oft-cited ones, for example those by Godel and Cohen about the Continuum Hypothesis) seem to require a high level of sophistication (though this might be a byproduct of the fact that consistency proofs like these are for the special class of statements that cannot be proven).
-For statements that can be proven then, are there cases where their consistency proofs are easier or came before the proofs themselves?
-Update:
-Thanks a lot, everyone, for the great answers so far. The number and existence of these examples is interesting to me, as well as the fact that they all rely on the same technique of first proving something using an additional axiom (an approach first suggested by Michael Greinecker). That hadn't occurred to me. I wonder if there are other approaches.
-
-REPLY [6 votes]: Let $c(X)$ denote the cellularity of the topological space $X$, that is the supremum of the cardinalities of its families of pairwise disjoint non-empty open subsets.
-In the early 60's Kurepa asked:
-
-
-Is there a compact Hausdorff space $X$ such that $c(X)
-TITLE: Positive definite function zoo
-QUESTION [14 upvotes]: I've asked the following question on math.stackexchange but there has been no response so I'll ask it again here:
-A positive definite function $\varphi: G \rightarrow \mathbb{C}$ on a group $G$ is a function that arises as a "diagonal" coefficient of a unitary representation of $G$.
-For a definition and discussion of positive definite function see here.
-I've often wished I had a collection of diverse examples of positive definite functions on groups, for the purpose of testing various conjectures. I hope the diverse experience of the participants of this forum can help me collect a list of such examples.
-To clarify what I'd like to see:
-
-What is an example of a positive
- definite function on a group $G$ that
- is not easily seen to be a coefficient
- of a unitary representation of $G$?
- What are some positive definite
- functions that arise in contexts
- sufficiently removed from studying the
- coefficients of unitary
- representations?
-
-Also, the weirder the group $G$ the better. I'd like a collection of quirky beasts...
-
-REPLY [6 votes]: Perhaps you are already aware of this, but I thought I'd mention it for other interested google-enabled readers.
-
-Infinitely divisible distributions are one place where positive-definite functions come up (Lévy processes, Lévy-Khintchine formula, etc., are also relevant keywords)
-Infinite divisibility in Free Probability is another related place.<|endoftext|>
-TITLE: On the definition of regularity
-QUESTION [13 upvotes]: In the literature on D-modules, there are many definitions of regularity of holonomic D-modules.
-(1) Bernstein first defines regularity on a curve then says a holonomic D-module is regular if its restriction to any curve is regular
-(2) Mebkhout defines the irregularity complexes of a complex of D-modules along an hypersurface. The complex is then regular if its irregularity complexes are 0 along any hypersurface.
-(3) Kashiwara defines a D-module as regular if it admits a good filtration $F_*M$ such that $\operatorname{Ann}(Gr^F M)$ is a radical ideal of $Gr^F D_X = \pi_*O_{T^*X}$.
-I think there are other definitions (in Deligne for example)
-Where can I find proofs that all these definitions are equivalent? Thanks.
-
-REPLY [5 votes]: There are some comparison results in Chapter 5 of Bjork's `Analytic D-modules and Applications'. Also see Chapter 8. In particular, I think Thm. 8.7.3 combined with Thm. 5.6.5 (almost) gives (1) iff (3). Further, I think Prop. 5.6.22 gives the equivalence with (2). There are also results in there comparing Deligne's description.
-I must admit though that I find Bjork quite notationally dense and am not particularly familiar with it, so I may be quite off with the references above. I am interested in this question also, so please comment/post if you find better references.<|endoftext|>
-TITLE: When is a given matrix of two forms a curvature form?
-QUESTION [14 upvotes]: Let's assume we are working over $\mathbb{R}^n$ (but feel free to change to domain to answer the question). I wish to know if the equation $F = dA + A \wedge A$ can be solved for a matrix of 1-forms $A$, given a (smooth) matrix of 2-forms $F$ which satisfies the condition $dF =B \wedge F - F \wedge B$ for some smooth matrix of 1-forms $B$ (i.e. the Bianchi identity is satisfied). Notice that this is true for line-bundles (in fact over any convex open set).
-
-REPLY [25 votes]: The answer is generally 'no'; for most $F$ that satisfy your condition, there will not exist an $A$ that satisfies $F = dA + A\wedge A$.
-The easiest counterexample I know of is when $n=4$ and the matrix $F$ is $2$-by-$2$. To begin, note that you can reduce to the case when both $F$ and the $A$ you seek have trace zero, i.e., they take values in ${\frak{sl}}(2,\mathbb{R})$. (The reason is that the problem breaks into the part of $F$ that is a multiple of the identity matrix and the trace-free part. I'll leave the details to you.)
-One can easily check that, for the generic ${\frak{sl}}(2,\mathbb{R})$-valued matrix $F$ of $2$-forms on $\mathbb{R}^4$, the kernel of the mapping $C\mapsto F\wedge C - C\wedge F$ from ${\frak{sl}}(2,\mathbb{R})$-valued matrices $C$ of $1$-forms on $\mathbb{R}^4$ to ${\frak{sl}}(2,\mathbb{R})$-valued matrices of $3$-forms on $\mathbb{R}^4$ is zero. By dimension count, it follows that this mapping is surjective as well.
-Thus, for a candidate ${\frak{sl}}(2,\mathbb{R})$-valued $2$-form $F$ that satisfies this open genericity condition, the equation $dF = F\wedge B - B\wedge F$ is always solvable for $B$, and, moreover, the solution is unique. Thus, this $B$ is the only possible candidate for $A$. Heuristically, this makes it almost immediate that, for the generic such $F$, the $B$ that you find will not satisfy $F = dB + B\wedge B$. The reason is that ${\frak{sl}}(2,\mathbb{R})$-valued $1$-forms on $\mathbb{R}^4$ depend on only $3\times 4 = 12$ arbitrary functions of $4$ variables while the generic ${\frak{sl}}(2,\mathbb{R})$-valued $2$-form $F$ depends on $3\times 6 = 18$ arbitrary functions of $4$ variables. There is no chance that you could hit each such $F$ with an $A$.
-To construct an explicit example, choose an ${\frak{sl}}(2,\mathbb{R})$-valued $1$-form $A$ such that $F = dA + A\wedge A$ satisfies the genericity condition. Then, of course, $2F$ will satisfy this genericity condition as well, and, since it satisfies $d(2F) = (2F)\wedge A - A \wedge (2F)$, it follows that the only possible $1$-form whose curvature could be $2F$ is $A$. However, the curvature of $A$ is $F\not=2F$. Thus, $2F$ satisfies your condition, but it is not a curvature form.<|endoftext|>
-TITLE: Central extensions of group schemes
-QUESTION [9 upvotes]: In the category of groups, it is elementary that all central extensions of a cyclic group are abelian. Is the same true, in the category of (finite?) group schemes over a field $k$, for central extensions of the group $\mu_n$ of $n$th roots of unity?
-
-REPLY [14 votes]: If we have a central extension of group schemes $1\rightarrow B \rightarrow C\rightarrow
-A\rightarrow1$ with $A$ abelian, then we get a commutator mapping
-$\Lambda^2A\rightarrow B$ (of sheaves as $\Lambda^2A$ in general is not a group
-scheme) and the extension is abelian precisely when this map is zero. Hence for
-an non-abelian extension to exist there must be a non-zero map
-$\Lambda^2A\rightarrow B$. Let us now assume that $A=\mu_n$ and consider first
-the case when $n=p$, the characteristic of the field $k$ (which we may assume is
-algebraically closed). A non-zero map $\Lambda^2A\rightarrow B$ would give a
-non-zero map $A\rightarrow\mathrm{Hom}(A,B)$, where the right hand side is the
-sheaf of group homomorphisms. As the Frobenius map is zero on $\mu_p$ we may
-replace $B$ by its Frobenius kernel so we may assume that $B$ is either $\mu_p$
-or the Cartier dual of $\alpha_{p^m}$. Now, as sheaves $\mathrm{Hom}(A,B)$ is
-isomorphic to $\mathrm{Hom}(D(B),D(A))$, where $D(-)$ denotes the Cartier
-dual. However $D(\mu_p)=\mathbb Z/p$ so when $A=\mu_p$ we get that
-$\mathrm{Hom}(D(B),D(A))=\mathbb Z/p$ and there is only the zero map from
-$A=\mu_p$ into it. In the other case $D(A)=\alpha_{p^m}$ and
-$\mathrm{Hom}(\alpha_{p^m},\mathbb Z/p)$ is zero. If instead $n=p^k$, the
-argument is the same. The case when $n=\ell^k$ is even simpler so in all cases
-all possible commutator maps are zero and the extension is commutative.
-(When $A=B=\mathbb G_a$ then there are candidates for commutator maps and in
-fact $(a,b)(a',b')=(a+a',b+b'+a^pa')$ gives a non-commutative central extension
-which I imagine is the fake Heisenberg group.)
-Addendum: A general comment is that it is more convenient to work with sheaves (in the fppf topology say) as that means that we essentially can pretend that we work with set-theoretic groups. It is however also necessary if we want to see the commutator map as a map $\Lambda^2A\to B$ as the sheaf $\Lambda^2A$ (of $A$ considered as an abelian sheaf) is in general not reprsentable. The $\langle-,-\rangle\colon\Lambda^2A\to B$ view point is convenient as it allows us to do what one usually does when having a pairing: We get for instance a map $A\to\mathrm{Hom}(A,B)$ given by $a\mapsto (a'\mapsto \langle a,a'\rangle)$ just from the fact that $\langle-,-\rangle$ is biadditive.
-I have implicitly assumed that $B$ is of finite type (as I claim that its Frobenius kernel is finite) even though it may not be necessary (a limit argument anyone?).<|endoftext|>
-TITLE: Top Chern Class = Euler Characteristic
-QUESTION [13 upvotes]: Let $X$ be a (quasi-)projective, nonsingular, complex variety. Denote by $\mathcal{T}_X$ its tangent sheaf and by $X^{\mathrm{an}}$ its analytification. I am looking for a proof for the equality
- $\displaystyle \int_X c_n(\mathcal{T}_X) = \chi(X^{\mathrm{an}})$,
-i.e. the degree of the top chern class is equal to the topological Euler characteristic of $X$. There's Example 3.2.13 in Fulton's book on intersection theory which briefly mentions this, but it does not give a reference. Can someone help me out with one? Thanks in advance.
-
-REPLY [21 votes]: As an alternative to R. Budney's answer, one might also notice that the Gauss-Bonnet formula (the one you mention - mind that you must assume that $X$ is projective, otherwise the integral might not even make sense) is a consequence of the Hirzebruch-Riemann-Roch theorem. Indeed, the HRR theorem says
-$$
-\chi(V)=\int_{X}{\rm Td}({\rm T}X){\rm ch}(V)
-$$
-where $$\chi(V):=\sum_{l}{(-1)}^l{\rm rk}(H^l(X,V))$$ is the Euler characteristic of coherent sheaves. Now there is an universal identity of Chern classes
-$$
-{\rm ch}(\sum_{r}(-1)^r\Omega_X^r){\rm Td}(\Omega^\vee_X)=c^{\rm top}(\Omega^\vee_X)
-$$
-(called the Borel-Serre identity). Here $\Omega_X$ is the sheaf of differential of $X$ and thus $\Omega^\vee_X={\rm T}X$. Plugging the element $\sum_{r}(-1)^r\Omega_{X}^r$ into the HRR theorem, one gets
-$$
-\sum_{k,l}(-1)^{l+k}{\rm rk}(H^k(X,\Omega^l))=\int_{X}c^{\rm top}(TX)
-$$
-and by the Hodge decomposition theorem
-$$
-\sum_{k,l}(-1)^{l+k}{\rm rk}(H^k(X,\Omega^l))=\sum_{r}{(-1)}^r{\rm rk}(H^r(X({\bf C}),{\bf C}))
-$$
-where $H^r(X({\bf C}),{\bf C})$ is the $r$-th singular cohomology group.
-The quantity $\sum_{r}{(-1)}^r{\rm rk}(H^r(X({\bf C}),{\bf C}))$ is the topological Euler characteristic, so this proves what you want.
-The HRR theorem is proved in chap. 15 of Fulton's book (or in Hirzebruch's book "Topological methods...") and the Borel-Serre identity is Ex. 3.2.5, p. 57 of the same book.<|endoftext|>
-TITLE: Uniform lattices in semisimple Lie groups
-QUESTION [6 upvotes]: Let $\Gamma$ be a uniform lattice in a semisimple Lie group $G$.
-
-Must $\Gamma$ be virtually torsion-free?
-If (1) is true, then does this work more generally if $G$ is reductive?
-
-I am motivated by a prima facie knowledge of Theorem B of Armand Borel's paper, "Compact Clifford--Klein forms of symmetric spaces" (1963).
-
-REPLY [9 votes]: If $G$ is linear (which would be the case, for instance, if it is centerless) then it is special case of the more general fact that any finitely generated subgroup of $GL_n(F)$ for a field $F$ of characteristic zero is virtually torsion-free.
-So all you need to know is that the lattices are finitely generated. This, for cocompact lattices, is easy, and for instance follows from what is usually called Milnor-Schwarz lemma, which is a very general lemma about cocompact isometric actions of groups on spaces. You can find a version of it in Pierre de la Harpe's book.
-For general lattices, in higher rank, this follows from property T and in rank 1 by some more geometric methods.
-If not, this may fail. See this for instance:
-http://people.uleth.ca/~dave.morris/talks/deligne-torsion.pdf<|endoftext|>
-TITLE: Disconnecting sets
-QUESTION [7 upvotes]: If E is a metric space, I call a subset C of E a cut if E-C is not connected and if C is minimal for this property (which is obviously equivalent to "for every p in C, E-C union p is connected". The empty set is a cut of any disconnected space, so the obvious conjecture is : every space of more than one point possesses a cut (this is almost obviously true for manifolds, for instance)... but this conjecture is in fact probably wrong, as it is not obvious at all to find cuts in convoluted spaces like, for instance, the common boundary of three open sets in the plane ; on the other hand, i was not able to prove it in this case either. Any hint on such a proof ? (by the way, Zorn's lemma dont apply here ; I wonder is there is any interesting example of a situation where the hypothesis of the lemma fail (ie, like here, the intersection of a decreasing family of sets having the property P does not necessarily have it), but the conclusion (the existence of a minimal such set) is still valid ?
-
-REPLY [6 votes]: Assume $E$ is what remains after the infinite number of the iterations above.
-This is an example of indecomposable continuum, so it can not be presented as a union of any two proper subcontinua.
-Assume there is a cut $C\subset E$.
-Then each connected component of $E\backslash C$ has $C$ as the boundary.
-In particular, $C$ is closed.
-Let us present $E\backslash C$ as a union of two disjoint open sets $A$ and $B$.
-Clearly both $\bar A =C\cup A$ and $\bar B=C\cup B$ are proper subcontinua of $E$,
-a contradiction.<|endoftext|>
-TITLE: When is an algebraic variety $\mathbb{Q}$-factorial?
-QUESTION [12 upvotes]: A variety is $\mathbb{Q}$-factorial if every global Weil divisor is $\mathbb{Q}$-Cartier. How bad singularities are allowed so that the algebraic variety is still $\mathbb{Q}$-factorial? Is a singular curve $\mathbb{Q}$-factorial? For example is a nodal-cuspidal plane curve $\mathbb{Q}$-factorial?
-
-REPLY [9 votes]: A complete intersection $X$ in $\mathbb{P}^n$ is $\mathbb{Q}$ factorial if $\dim X_{sing}<\dim X-3$.
-In general being $\mathbb{Q}$-factorial depends on the type of singularity you have, but also on the position of the singularities.
-E.g., a degree $d$ threefold $X$ in $\mathbb{P}^4$ with only ordinary double points at $p_1,\dots,p_k$ is $\mathbb{Q}$-factorial if and only if the linear system of homogeneous polynomials of degree $2d-4$ polynomials through the singular points of $X$ has no defect (i.e., the codimension of this linear system is precisely $k$).
-An example of a non-$\mathbb{Q}$-factorial threefold with only nodes is $f_1f_2+f_3f_4=0$, where $\deg(f_1)+\deg(f_2)=\deg(f_3)+\deg(f_4)$, and the $f_i$ are chosen sufficiently general.<|endoftext|>
-TITLE: Exact sequences of bundles on Grassmannians
-QUESTION [9 upvotes]: We're looking for a large set of exact sequences of vector bundles on Grassmannians. Here's the set up:
-$V$ and $Q$ are complex vector spaces of dimensions $d$ and $r$ respectively $(d\geq r)$, and we're working on the Grassmannian $Gr(V,Q)$. For simplicity let's fix a trivialization of $det(V)$.
-Now let $\alpha$ be a partition/Young diagram with at most $(r-1)$ rows and at most $(d-r)$ columns. Let $\beta$ be the Young diagram obtained from $\alpha$ by adding an extra row of length $(d-r)$ at the beginning. What we want is an exact sequence of vector bundles that goes
-$$\mathbb{S}_\alpha(Q)\otimes det(Q)^{-1} \rightarrow\;\; ... \;\;\rightarrow \mathbb{S}_\beta(Q) $$
-($\mathbb{S}$ denotes a Schur functor). For $r=1$ there's only one choice for $\alpha$, and the Koszul complex is the required sequence. For $d-r=1$ we have the short exact sequences
-$$\wedge^k Q \otimes det(Q)^{-1}\rightarrow \wedge^{k+1}V\ \rightarrow \wedge^{k+1} Q$$
-We can also solve $r=2$ using Eagon-Northcott complexes. These known cases suggest that the exact sequence should have $(d-r+2)$ terms.
-Does anyone know a general construction?
-Update: we have a precise conjecture for the terms in the sequence. Let $\beta_0$ be the partition obtained from $\alpha$ by deleting the first column. Now define $\beta_i$ recursively as the partition obtained from $\beta_{i-1}$ by adding boxes to the $i$th column until it agrees with the $i$th column of $\beta$. In particular $\beta_{d-r}=\beta$. Then the terms in middle of the exact sequence should be
-$$...\rightarrow \wedge^{(|\beta| - |\beta_i|)} V \otimes \mathbb{S}_{\beta_i} Q \rightarrow... $$
-If we fix a single point on the Grassmannian and split the tautological short exact sequence there then we can show that this works, which is pretty good evidence. Surely this isn't a new discovery?
-
-REPLY [3 votes]: Look at Fonarev's On minimal Lefschetz decompositions for Grassmannians, specifically Proposition 5.3 (link to proposition in the PDF). I guess this exact sequence is what you need.<|endoftext|>
-TITLE: How misleading is it to regard $\frac{dy}{dx}$ as a fraction?
-QUESTION [96 upvotes]: I am teaching Calc I, for the first time, and I haven't seriously revisited the subject in quite some time. An interesting pedagogy question came up: How misleading is it to regard $\frac{dy}{dx}$ as a fraction?
-There is one strong argument against this: We tell students that $dy$ and $dx$ mean "a really small change in $y$" and "a really small change in $x$", respectively, but these notions aren't at all rigorous, and until you start talking about nonstandard analysis or cotangent bundles, the symbols $dy$ and $dx$ don't actually mean anything.
-But it gives the right intuition! For example, the Chain Rule says $\frac{dy}{du} \cdot \frac{du}{dx}$ (under appropriate conditions), and it looks like you just "cancel the $du$". You can't literally do this, but it is this intuition that one turns into a proof, and indeed if one assumes that $\frac{du}{dx} \neq 0$ this intuition gets you pretty close.
-The debate about how rigorous to be when teaching calculus is old, and I want to steer clear of it. But this leaves an honest mathematical question: Is treating $\frac{dy}{dx}$ as a fraction the road to perdition, for reasons beyond the above, and which have not occurred to me?For example, what (if any) false statements and wrong formulas will it lead to?
-(Note: Please don't worry, I have no intention of telling students that $\frac{dy}{dx}$ is a fraction; only, perhaps, that it can usually be treated as one.)
-
-REPLY [6 votes]: A note from a publication (my own) that occurred several years after this question was asked. $\frac{dy}{dx}$ can be considered a fraction of differentials.
-You can think of differentials as infinitesimal values that are related to each other. Non-standard analysis showed that although 19th century mathematics viewed infinitesimals as problematic, they can be easily treated as ordinary mathematical objects, capable of division, multiplication, etc.
-There is no problem treating $\frac{dy}{dx}$ as a fraction, but there is a problem in higher-order derivatives and differentials, but that is because we are using a notation that doesn't support it. If you take the idea of $\frac{dy}{dx}$ being a fraction seriously, then, to find the second derivative, you are taking the derivative of a fraction. Therefore, you have to apply the quotient rule. If you apply the quotient rule to $\frac{dy}{dx}$ you do not get the typical result of $\frac{d^2y}{dx^2}$. Instead, you get:
-$$\frac{d^2y}{dx^2} - \frac{dy}{dx}\frac{d^2x}{dx^2}$$
-Or, written less ambiguously:
-$$\frac{d(d(y))}{(d(x))^2} - \frac{d(y)}{d(x)}\frac{d(d(x))}{(d(x))^2}$$
-When written this way, the second derivative can be considered actual fractions just like the first derivative. Third and higher derivatives are even uglier, because you are taking the derivative of that.
-You can see more details of this in "Extending the Algebraic Manipulability of Differentials", Dynamics of Continuous, Discrete and Impulsive Systems, Series A: Mathematical Analysis 26(3):217-230, 2019. And, if anyone is concerned for its validity, it had a further review in Mathematics Magazine 92(5), pp. 396–397 in their "Reviews" section.<|endoftext|>
-TITLE: construction of the Jacobian of a curve
-QUESTION [7 upvotes]: I am trying to understand the construction of the Jacobian of a curve following the notes of J. S. Milne
-The question is going to be about a particular step in the proof of Proposition 4.2b in Chapter III, but I will first briefly recall the setup.
-Let $X$ be a scheme flat over $T$, a divisor $D$ on $X$ is called relative effective divisor on $X/T$ if it is effective and flat over $T$ as a subscheme of $X$ (definition 3.4). There is a one-to-one correspondence benween relative effective divisors and sheaves $\mathcal L$ over $X$ with a global section $s$ such that $\mathcal{L}/s\mathcal{O}_X$ is flat over $T$.
-We are working over a field. Let $C$ be a non-singular curve of genus $\geq 2$.
-We are trying to construct a section of the natural map of functors $Div^r_C \to P^r_C$ where the first functor is the functor of relative effective divisors on $C\times T/T$ of degree $r$, and is represented by the $r$-fold symmetric product of $C$, and the second functor is the functor of families of degree $r$ invertible sheaves on $C$ parametrised by $T$, modulo trivial families:
-$$
-P^r_C(T) = \{ \mathcal{L} \in Pic(C \times T) \mid deg\ \mathcal{L}_t=r \textrm{ for all }\ t \in T\} / q^* Pic(T)
-$$
-(the natural projections are denoted $p: C \times T \to C$, $q: C\times T \to T$.)
-Proposition 4.2 deals with subfunctors of $Div^r_C$ and $P^r_C$. We pick an effective degree $(r-g)$ divisor $D_\gamma$ and define
-$$
-C^\gamma(T) = \{ D \in Div^r_C(T) \mid h^0(D_t-D_\gamma)=1\ \textrm{ for all }\ t \in T\}
-$$
-$$
-P^\gamma(T) = \{ \mathcal{L} \in Pic^r_C(T) \mid h^0(\mathcal{L}_t \otimes \mathcal{L} _\gamma^{-1})=1\ \textrm{ for all }\ t \in T\}
-$$
-part b) constructs a section $P^\gamma \to C^\gamma$. Take $\mathcal{L} \in P^\gamma(T)$. By definition of $P^\gamma$ and by Riemann-Roch, $h^1 (\mathcal{L}_t \otimes \mathcal{L}^{-1}_\gamma)=0$. This allows us to apply a base change theorem and coclude that $q_*(\mathcal{L} \otimes p^* \mathcal{L}^{-1} _\gamma)$ is locally free and thus an invertible sheaf on $T$ (call it $\mathcal{M}$). The proof then proceeds to construct a section of $\mathcal{L} \otimes (q^* q_*(\mathcal{L} \otimes p^* \mathcal{L} _\gamma^{-1}))^{-1}$.
-In particular, as there is a natural map $q^* q_*(\mathcal{L} \otimes p^* \mathcal{L} _\gamma^{-1}) \to \mathcal{L} \otimes p^* \mathcal{L} _\gamma^{-1}$, one has a canonical global section of $\mathcal{L} \otimes p^* \mathcal{L} _\gamma^{-1} \otimes (q^*\mathcal{M})^{-1}$, and by composing it with the natural map $p^* \mathcal{L} _\gamma^{-1} \to \mathcal{O}_{C\times T}$ one gets the desired.
-We did obtain a section $s_\gamma$ of a sheaf equivalent to $\mathcal{L}$ (in the sense of the definition of $Pic^r_C$)
-My question is: why does this section give rise to a relative effective divisor, that is, why would $\mathcal{L} \otimes (q^* \mathcal{M})^{-1}/s_\gamma \mathcal{O}_{C\times T}$ be flat over $T$?
-
-REPLY [2 votes]: Let $N = L \otimes (q^\ast q_\ast (L\otimes L_\gamma^{-1}))^{-1}$. It suffices to show that the zero locus $D \subset C \times T$ of $s \in \Gamma(N)$ is flat over $T$. If $T$ is nice (Noetherian, blah, blah), it then suffices to show that the fiberwise degree of $D$ is constant. Note that the restriction of $N$ to $C \times \{ t \}$ is isomorphic to the restriction of $L$ to $C \times \{ t \}$. Since $L$ is fiberwise degree $r$ by assumption, it follows that $D$ is fiberwise degree $r$.
-Oh and you need to check that the section $s$ is fiberwise nonzero.
-Well, here's how you do that --- first look at the map $\phi : q^\ast q_\ast (L \otimes p^\ast L_\gamma^{-1}) \to L \otimes p^\ast L_\gamma^{-1}$. What does this map look like on fibers? Well, note that $H^0 (C \times \{ t \} , (L \otimes p^\ast L_\gamma^{-1})|\_{C \times \{ t \}})$ is by assumption 1 dimensional. Hence the restriction of $q^\ast q_\ast (L \otimes p^\ast L_\gamma^{-1})$ to $C \times \{ t \}$ is a trivial line bundle. So, the map $\phi$ on the fiber $C \times \{ t \}$ looks like the map $\mathcal{O}\_{C \times \{ t \}} \to (L \otimes p^\ast L_\gamma^{-1})|\_{C \times \{ t \}}$ corresponding to the one nonzero global section in $H^0 (C \times \{ t \} , (L \otimes p^\ast L_\gamma^{-1})|\_{C \times \{ t \}})$. Clearly this map is not zero.
-I'll let you do the rest...<|endoftext|>
-TITLE: how to use arxiv?
-QUESTION [50 upvotes]: This is a soft question. How do people usually use arxiv to put their papers? At which stage does one usually put his/her paper/report there? Someone suggests me to submit a paper while putting it on arxiv. Is that the convention that people follow?
-Thank you!
-Anand
-
-REPLY [10 votes]: Yet another way in which people could use arXiv is as a repository for material which otherwise cannot find a home in a journal. Sometimes in the course of working on a project I wind up with some material which did not make it into the published article -- or perhaps some notes which record my growing understanding of articles already published by others -- which look like they could be useful to the community as expository or supplemental information, but which in my opinion are not otherwise significant enough to warrant submitting to a journal. I have sometimes wondered whether it would be appropriate to post such material on the arXiv. (I have not done so yet.) For that matter, I wonder whether others have done this very thing.
-One drawback of this use of the arXiv is that everyone knows that most arXiv articles have not been peer-reviewed at the time of first posting, so they must be taken with a grain of salt. If an article is never published in a journal, you must read the arXiv article with a more critical eye. But as I said, I do believe that there exist some notes that are perhaps worth sharing but not worth wasting the effort to peer review.<|endoftext|>
-TITLE: How can one compute the canonical class of the projective completion of the tautological bundle over $P^1\times P^1$?
-QUESTION [7 upvotes]: I am interested in computing the (anti)-canonical class of the (total space of the) projective completion of the tautological bundle over $P^1\times P^1$. That is, the canonical class of $\mathbb P_{P^1\times P^1}(J \oplus \mathscr O)$, where $J$ is the tautological line bundle on $P^1\times P^1$.
-I believe this can be done by computing the fan of the toric variety and summing the classes of the orbit closures of the one-skeleton? I was hoping for insight into perhaps a slicker/less cumbersome way of approaching this computation.
-Thanks in advance.
-
-REPLY [4 votes]: I like the way you asked to avoid. Forgive me if I describe it in
-polytope rather than fan language.
-Step 1: ${\mathbb P}^1 \times {\mathbb P}^1$'s polytope is a square (or
-any rectangle). The four edges, taken clockwise, correspond to the
-${\mathbb P}^1$s giving the classes $h_1,h_2,h_1,h_2$ Michael mentions.
-(EDIT: I had signs there before, by overthinking the Danilov relations.)
-I can only guess that by "tautological line bundle on
-${\mathbb P}^1 \times {\mathbb P}^1$
-you mean ${\mathcal O}(-1) \boxtimes {\mathcal O}(-1)$.
-If we blow down that ${\mathbb P}^1 \times {\mathbb P}^1$, we get the
-affine cone over the Segre embedding of ${\mathbb P}^1 \times {\mathbb P}^1$.
-The polyhedron of that is also a cone, on a square.
-Step 2: Blow the singular point back up, which corresponds to cutting the
-corner off that cone, leaving a square. So far we have an unbounded
-polytope that retracts to the square, just as the line bundle retracts
-to ${\mathbb P}^1 \times {\mathbb P}^1$.
-Step 3: Projectively complete. This corresponds to bounding the cone.
-Combinatorially, we now have a square-based pyramid with the top corner
-cut off, so there's a big square on the bottom (whose class is Michael's
-$h$) and a little square on the top.
-Step 4: Take the anticanonical class. On any toric variety, the boundary
-of the polytope defines an anticanonical divisor.
-So far our anticanonical class is the bottom square $h$ plus the top
-square plus the other four faces. To calculate the linear relations
-between them, one needs to be precise about the locations of the vertices.
-I have the bottom square at $(0,0), (2,0), (0,2), (2,2)$ with $z=0$
-and the top one at $(0,0), (1,0), (0,1), (1,1)$ with $z=1$.
-The Danilov relations from the $z$-axis vector says
-$$ (-1) \text{bottom} + (+1) \text{top}
-+ 0 \text{west} + 0 \text{south} + (+1) \text{north} + (+1)\text{east} = 0 $$
-so the total of the faces is $2\text{bottom} + \text{south} + \text{west}$,
-matching Michael's $2h+h_1+h_2$.
-(As it ought, since I learned at least some of this from him.)<|endoftext|>
-TITLE: extensions of lebesgue measure
-QUESTION [10 upvotes]: The Hahn-Banach theorem implies that Lebesgue measure can be extended give a "measure" on all subsets of [0,1], but this measure is only guaranteed to be finitely additive. It might magically turn out that this measure is countably additive, but this can only happen if the continuum is a real-valued measurable cardinal, a strong set-theoretic assumption. My question is: if it turns out that measure is countably additive on the measure zero sets, does this imply that the measure is countably additive everywhere?
-
-REPLY [3 votes]: The answer is no. A proof can be found here.<|endoftext|>
-TITLE: Blowing up a subvariety - what can happen to the singular locus?
-QUESTION [5 upvotes]: Let $X$ be a variety defined over a number field $k$. If I blow-up along some arbitrary subvariety of $X$, what are the possible outcomes for the dimension of the singular locus of the variety? If the subvariety lies outside the singular locus of $X$, then it stays the same, if it is carefully chosen, it might go down. Can it go up?
-To be more specific, my variety is a high dimensional hypersurface, and the subvariety I am blowing up is a linear space of much smaller dimension than the singular locus. I don't know if this changes the situation.
-I have a feeling this question might be more suited to stackexchange, but it didn't spark much interest over there https://math.stackexchange.com/questions/53676/blowing-up-a-subvariety-what-can-happen-to-the-singular-locus. Apologies for wasting time if so.
-
-REPLY [4 votes]: Any birational map $\pi:X'\to X$ is the blow-up of some ideal sheaf on $X$, so in general one must expect singularities on $X'$, even if the ideal is reduced (as you assume).
-As a concrete example, let $X=\mathbb{A}^n$ and blow-up the complete intersection subvariety given by the ideal $I=(f,g)\subset k[x_1,\ldots,x_n]$. Then the blow-up of $X$ is the Proj of the Rees algebra $R[It]$ which is given by $k[x_1,\ldots,x_n,S,T]/(fS-gT)$. By choosing $f$ and $g$ appropriately one can produce varieties with singular locus of high dimension.
-For your specific example, when $Y$ is a linear space of small dimension, I don't know if the above can happen, but there are certainly cases where the dimension of the singular locus will be unchanged after the blow-up, (e.g when $Y$ a point on a singular surface).<|endoftext|>
-TITLE: Analytic functions with algebraic Taylor coefficients at some point.
-QUESTION [6 upvotes]: This question just came to my mind when reading the question
-When may Function (meromorphic) be expanded as power series with coefficients of integers
-Suppose $f$ is an analytic function on some open subset $U \subseteq \mathbb{C}$.
-Are there sufficient or necessary conditions to be put on $f$ that $f$ has the following property.
-There exist a point $\zeta \in U$ such that the Taylor expansion of $f$ around $\zeta$ has only rational (algebraic) coefficients.
-More generally, let us call the set of functions with this property $\mathcal{R}(U)$. It is easy to see that
-$\mathcal{R}(U)$ is dense in the set of all analytic functions with respect to the topology
-given by locally uniform convergence, since all polynomials with rational coefficients are
-contained in $\mathcal{R}(U)$ and are dense. But of course $\mathcal{R}(U)$ does not contain
-all analytic functions since for example $z^2+\pi$ or constant functions are not all contained. Also $\mathcal{R}(U)$ is uncountable, since $z-\zeta$ for any $\zeta \in \mathbb{C}$ satisfies the property.
-So what can be said about $\mathcal{R}(U)$? Is it in some sense interesting?
-
-REPLY [3 votes]: Suppose you have a polynomial $p(z)=a_nz^n+a_{n-1}z^{n-1}+...+a_0$. Suppose that $p$ has the property you desire, which is equivalent to all of the derivatives of $p$ being simultaneously rational at some point $c$. In particular, this means $a_n$ must be rational, and so we may assume wlog that $a_n=1$.
-If $a_{n-1}\in\mathbb{Q}$, then clearly $c$ must be rational, in which case it is immediate that all the $a_i$ are rational and we are done.
-So, suppose that $a_{n-1}$ is not rational. Then by looking at the $(n-1)$th derivative at $c$, it must be that $a_{n-1}+nc$ is a rational number. Thus $a_{n-1}$ and $c$ must either both be algebraic or both be in the algebraic numbers adjoin $a_{n-1}$. Looking at the higher derivatives, one sees that all the terms are polynomially related, and so all the $a_i$ must all lie in the algebraic closure of the algebraic numbers adjoin $a_{n-1}$.
-This necessary condition guarantees that almost none of the polynomials are in $\mathcal{R}(\mathbb{C})$ and, as a consequence, almost no analytic function is in there either. This is despite them being dense with respect to the topology you mentioned.
-For a necessary condition, one can again see that it comes down to the value of $a_{n-1}$. One can "easily" iteratively generate a list of exact polynomial relations between $a_{n-1}$ and each of $a_i$ with $i\le n-2$. This isn't exactly a pleasant relation, however, since the relations are different for different degrees and also one ends up with complicated polynomials. For fourth- and lower-degree polynomials, this allows you to get exact relations in terms of radicals. However, for higher degree polynomials, this problem is decidedly more complicated because finding roots even for 5th degree polynomials is not universally possible. It might be that the polynomials in question are all of a particularly nice flavor which happen to be solvable nicely by radicals, but I don't know enough about that kind of thing. My guess is it is likely a hopeless venture.<|endoftext|>
-TITLE: when can we lift an action of Lie algebra?
-QUESTION [5 upvotes]: Suppose $G$ is a Lie group, $\mathfrak{g}$ its Lie algebra, if we have a smooth representation $(\pi,V)$, then it induces an action of $\mathfrak{g}$ on $V$. Now conversely, if we have a nice (with properties you may assume) action of $\mathfrak{g}$ on $V$, can we say such action arises from some unique smooth action of $G$?
-Here we may assume $G$ to be simply connected if needed. Thank you.
-
-REPLY [7 votes]: Let $\pi$ represent a finite dimensional real Lie algebra $\mathfrak g$ on a Hilbert space
-$\mathcal H$ by skew-adjoint operators. Then $\pi$ integrates to the connected simply connected Lie group $G$ with Lie algebra $\mathfrak g$ if, and only if, the elements of
-$\pi(\mathfrak g)$ have a common invariant dense domain. This is an old result of Moshe Flato, Daniel Sternheimer and others. My apologies to the mathematical physicists whose names I have omitted.<|endoftext|>
-TITLE: Literature on behaviour of eigenfunctions under multiplication?
-QUESTION [6 upvotes]: Dear community,
-I would be happy about any literature or comments on the behaviour of the pointwise product of eigenfunctions of a self-adjoint operator with discrete spectrum, acting on a separable Hilbert space which is closed under pointwise multiplication. The operator I'm actually looking at is a symmetric Markov operator acting on $L^2(\mathcal{A},\mu)$, where $\mathcal{A}$ is some function algebra and $\mu$ the invariant measure.
-Some questions I'm especially interested in are:
-
-If you multiply two eigenfunctions, can it happen that the product has an infinite eigenfunction expansion? By "infinite eigenfunction expansion" I mean that it can not be expressed as a finite sum of eigenfunctions.
-Somewhat related: If the squares of two eigenfunctions have a finite expansion, respectively, can it happen that the square of the sum of these two eigenfunctions has an infinite expansion?
-In the above Markov setting: Is the following "projected Cauchy-Schwarz inequality" always true?
-$$
-\int \operatorname{proj}(fg \mid E)^2 \ \text{d} \mu \leq
-\sqrt{\int \operatorname{proj}(f^2 \mid E)^2 \ \text{d} \mu}
-\sqrt{\int \operatorname{proj}(g^2 \mid E)^2 \ \text{d} \mu}
-$$
-Here, $f$ and $g$ are eigenfunctions lying in some common eigenspace, $E$ is another eigenspace and $\operatorname{proj}(f \mid E)$ denotes the projection of $f$ on $E$.
-
-Note that the answer to questions 1 and 2 is no, if the eigenfunctions are orthogonal polynomials.
-Thanks for your help,
-Simon
-
-REPLY [3 votes]: For question 1, one example of interest comes from the energy eigenfunctions of the one dimensional quantum harmonic oscillator. The Hilbert space is separable, and the Hamiltonian satisfies your conditions. Under suitable normalization, the eigenfunctions have the form $p(x)e^{-\pi x^2}$ for $p$ a Hermite polynomial, and the product of any two then has the form $q(x)e^{-2\pi x^2}$ for $q$ a nonzero polynomial. These products are not given by finite linear combinations of eigenfunctions.<|endoftext|>
-TITLE: Embedding in f.p. simple groups
-QUESTION [10 upvotes]: Dear All!
-At the time when Lyndon and Schupp wrote their book there was an open question:
-Question: Does every finitely presented group with soluble word problem embed in a finitely presented simple group?
-Is it still open? Could you hint at some useful references about this? Thanks!
-
-REPLY [2 votes]: There is a strengthening of the Boone-Higman result, due to Thompson. He showed that we can take the simple group to be finitely generated. In full, this reads:
-"A finitely presented group has solvable word problem if and only if it can be embedded in a finitely generated simple group that can be embedded in a finitely presented group."
-You can find the full details in:
-R. J. Thompson, "Embeddings into finitely generated simple groups which preserve the word problem", Word Problems II: The Oxford Book, Studies in Logic and the Foundations of Mathematics, Volume 95, (1980).
-As far as I am aware, your original question "Does every finitely presented group with soluble word problem embed in a finitely presented simple group?" is still an open problem.
--Maurice<|endoftext|>
-TITLE: Understanding the analytic index map of the Atiyah-Singer index theorem
-QUESTION [8 upvotes]: Hi,
-I'm currently trying to understand the Atiyah-Singer index theorem and its proof as presented in the book "Spin Geometry" by Lawson and Michelsohn.
-I do not understand why the analytic index map $\operatorname{ind}\colon K_{cpt}(T^\ast X) \to Z$, as defined in chapter III.$13 in equation (13.8), agrees with the Fredholm index of an elliptic pseudo-differential operator.
-Recall how the analytic index map is constructed (this is chapter III.§13 in the book):
-Given an element $u \in K_{cpt}(T^\ast X) \cong K(DX, \partial DX)$ we can represent it by Lemma III.13.3 via a triple $(\pi^\ast E, \pi^\ast F; \sigma)$, where $E$ and $F$ are vector bundles over $X$, $\pi\colon T^\ast X \to X$ is the bundle projection and $\sigma\colon \pi^\ast E \to \pi^\ast F$ is homogeneous of degree 0 on the fibres of $T^\ast X$ (i.e. $\sigma$ is constant on the fibres). Then for any $m$ there exists an elliptic, classical pseudo-differential operator $P \in \Psi CO_m(E,F)$ whose asymptotic principal symbol is $\sigma$ (in particular, the symbol class $[\sigma(P)] \in K_{cpt}(T^\ast X)$ equals u). Then we set $\operatorname{ind}(u) := \operatorname{Fredholm-ind}(P)$. Then it is proven in the book, that this is well-defined, i.e. independent of all choices.
-Now given an elliptic pseudo-differential operator $D \in \Psi DO_m(E,F)$, we can construct its symbol class $[\sigma(D)] \in K_{cpt}(T^\ast X)$ as in chapter III.§1 in equation (1.7). Now I expect that the Fredholm index of $D$ coincides with the analytic index of $[\sigma(D)]$, but I do not see that this is proven in the book. I also can't prove it on my own. Going through the construction above we get an elliptic, classical pseudo-differential operator $P \in \Psi CO_m(E,F)$ with $[\sigma(D)] = [\sigma(P)] \in K_{cpt}(T^\ast X)$. But why do $D$ and $P$ have the same Fredholm index?
-
-Why is $\operatorname{Fredholm-ind}(D) = \operatorname{ind}([\sigma(D)])$ for an elliptic, pseudo-differential operator?
-
-REPLY [6 votes]: The Fredholm index of an elliptic operator only depends on the symbol class. Here is the proof (which I memorize from Lawson-Michelsohn and Atiyah-Singer).
-If $D: \Gamma(E_0) \to \Gamma(E_1)$ has order $k \neq 0$, pick a connection $\nabla$ on $E_0$. Then $A=(1+\nabla^{\ast}\nabla)$ is a self-adjoint invertible operator of order $2$ and $D \circ A^{-k/2}$ has the same Fredholm index as $D$ and the same symbol class; but it has order $0$. So for any operator, there is an order $0$ operator with the same symbol class and the same index; and this reduces the problem to the order $0$ case.
-It has been mentioned that the index of an operator only depends on the homotopy class of its symbol (by the way, the proof of this in Lawson-Michelssohn is incomplete. In the proof of Theorem 7.10, they say ''to construct a family of operators $P_t$ with $\sigma(P_t)=\sigma_t$, [...] is evidently possible locally (in coordinates)''. One needs to know that in $R^n$, the operator norm of a pseudo-DO can be estimated by the symbol. You can see this by going through the proof of Prop. III.3.2 of L.-M.).
-Recall the (modified) definition of $K^0 (TX,TX-0)$: it is the group of all equivalence classes of $(E_0,E_1,f)$; $E_i \to X$ vector bundles and $f: \pi^{\ast} E_0 \to \pi^{\ast} E_1$ a bundle map that is an isomorphism away from the zero section and homogeneous of order $0$ outside the zero section. The equivalence relation is generated by (1) homotopy, (2) isomorphism and (3) addition of things of the form $(E,E,id)$. (1) and (2) preserve the index. (3) also preserves the index; since a pseudo-DO with symbol $(E,E,id)$ is e.g. the identity operator, which has index $0$.<|endoftext|>
-TITLE: Can Thompson's group F be realized as a semigroup of continuous transformations of a tree?
-QUESTION [5 upvotes]: I am not very familiar with F, but I know that it can be realized as a group of homeomorphisms of the boundary of the binary tree. I also know that F cannot be realized as a group of graph automorphisms of any regular rooted tree because F is not residually finite. However, if we topologize our trees with the path metric, can F be realized as a group of continuous prefix-preserving transformations of a regular rooted tree (where the transformations need not be injective or surjective)? If you know the answer, could you provide a reference? Thanks!
-
-REPLY [3 votes]: There is a way to get $F$ to act on the infinite binary tree bijectively, but I doubt it satisfies most of your other requirements. It basically does something sensible with the "missing finite subtree." I have only partly checked this out (meaning it seems to check for one of the two relations needed).
-We let $T$ be the set of finite words (including the empty word) on the alphabet $\{0,1\}$. This is a binary tree by letting the left child of $u\in T$ be $u0$ and the right child of $u$ be $u1$. We define two permutations of $T$.
-The permutation $x_0$ is determined by the following rules:
-$\emptyset\rightarrow 1$;
-$0\rightarrow \emptyset$;
-$00u\rightarrow 0u$;
-$01u\rightarrow 10u$;
-$1u\rightarrow 11u$.
-The permutation $x_1$ is determined by the following rules:
-$\emptyset\rightarrow \emptyset$;
-$0u\rightarrow 0u$;
-$1\rightarrow 11$;
-$10\rightarrow 1$;
-$100u\rightarrow 10u$;
-$101u\rightarrow 110u$;
-$11u\rightarrow 111u$.
-These are the usual rules for the action of $x_0$ and $x_1$ in $F$ on infinite words in $\{0,1\}$ restricted to finite words and extended to the few cases that the rules usually omit.
-As I said, it checks for the relation:
-$(x_1)^{x_0x_0} = (x_1)^{x_0x_1}$.
-Here $a^b$ means $b^{-1}ab$ and the actions are to be composed from left to right (they are right actions).
-The other relation that defines $F$ with the one above is
-$(x_1)^{x_0x_0x_0} = (x_1)^{x_0x_0x_1}$.
-If the second fails while the first succeeds, I will be stunned.
-Assuming that the second relation checks out (not too hard, I am just too lazy), then these two permutations of $T$ generate a copy of $F$. On "most" of $T$, the action agrees with the usual action. How well this cooperates with what you want is for you to decide.
-The definitions can be tinkered with a bit. I doubt that the relations can survive a lot of tinkering though.<|endoftext|>
-TITLE: Maximal number of edges and triangular cells for n points in a triangular lattice
-QUESTION [7 upvotes]: Consider a subset of $n$ points in an equilateral triangular lattice. Draw all the edges between nearest-neighbor points.
-What is the maximum, over all such subsets, of the number of edges? This sequence appears to start 0, 1, 3, 5, 7, 9, 12, 14, 16...
-What is the maximum number of triangular lattice cells? (Not the number of all triangles, just the number of smallest possible equilateral triangles in the lattice.) This sequence appears to start 0, 0, 1, 2, 3, 4, 6, 7, 8, 10, 11, 13...
-http://oeis.org/A047932 is related to the first sequence but I have no proof it's the same. (There might be some other way of arranging the pennies that yields a higher number of contacts. A047932 is a lower bound on my sequence.) I can't find any OEIS sequences relevant to the second one.
-
-REPLY [10 votes]: You might have noticed that the difference between your two sequences is $0,1,2,3,4,5,6,7,8,\ldots$ and that the optimal configurations seem to be the same for both problems.
-This is true in general, and is a nice application of Euler's formula $V-E+F=2$. (NB this requires showing that in an optimal configuration the edges form a connected component.) Here $V=n$, the number of edges is $E$, and the number of triangles, call it $t$, is $F-1$ because we must count the exterior of the graph as a face. So the difference between the edge and triangle maxima is $(n-1)$ — at least assuming we can prove it's never to our advantage in either problem to have holes bigger than a unit triangle in the picture, which seems clear but may be annoying to prove rigorously. [EDIT see below on this point.]
-Another standard graph-theory formula that applies here: the sum of all the faces' edge-counts is $2E$. [Proof: count in two ways the pairs $(e,f)$ where $f$ is a face and $e$ is one of its edges.] In our setting all but one of the faces has $3$ edges, so $2E=3t+p$ where $p$ (for perimeter) is the number of outside edges (counting an edge twice if both sides abut the infinite face, as happens for $n=2$, and again assuming no internal holes). So $t+2 = 2n-p$, and the problem comes down to minimizing $t$. It certainly looks plausible that the "penny spiral" does this, but it's getting late so I'll leave it as an exercise :-) This would identify your first sequence with OEIS A047932, and thus determine the second sequence as well.
-UPDATE Denote the sequences in question by $s_1(n)$ and $s_2(n)$ respectively. As pointed out in G.Zaimi's accepted answer, the formula $s_1(n) = \lfloor 3n-\sqrt{12n-3}\rfloor$, consistent with the original proposer's guess/question, follows from a much more general result of Harbroth that this is the maximal number of times that the minimum distance can occur in any configuration of $n$ points in the plane. I looked up the solution (which is freely available online), and it seems to use ultimately the same technique: see the reference to the "Eulerschen Polyedersatz" before equation (3). Using the Euler "Polyedersatz" we also deduce $$s_2(n) = s_1(n) - (n-1) = \lfloor 2n-\sqrt{12n-3}+1\rfloor,$$ answering the second question. Indeed Euler says that $s_2(n) \leq s_1(n) - (n-1)$ with equality iff there's an optimal configuration without holes bigger than a unit triangle; and Harbroth's solution gives such a configuration.
-
-REPLY [10 votes]: The following was conjectured by D. Reutter in problem 664A, Elemente der mathematik 27 and proved by H. Harborth in Solution to problem 664A, Elemente der mathematik 29, 14-15
-
-The maximum number of times the minimum distance can occur among $n$ points in the plane is $\lfloor 3n-\sqrt{12n-3}\rfloor$.
-
-This is achieved by a hexagonal piece in the triangular lattice, i.e. from points forming a hexagonal spiral. In particular, this gives a formula for your first sequence.<|endoftext|>
-TITLE: How to prove a random d-regular graph is an expander with prob >= 0.5?
-QUESTION [7 upvotes]: Context: Many resources, like
-http://math.mit.edu/~fox/MAT307-lecture22.pdf
-state the theorem in the general case, but then prove it only for the bipartite case.
-The full case is supposedly proved in Pinsker's 1973 paper. However, I can't dig up a copy.
-Anyone know of a proof for the general case (i.e. d-regular, undirected, not-necessarily-bipartitite graph)?
-Thanks!
-
-REPLY [3 votes]: Pinsker's original paper is now available online in the archive of the International Teletraffic Congress:
-http://ww.i-teletraffic.org/fileadmin/ITCBibDatabase/1973/pinsker731.pdf<|endoftext|>
-TITLE: Discrete version of Nullstellensatz?
-QUESTION [12 upvotes]: Hi. I was reading the paper "On the foundations of combinatorial theory (VI): The idea of a generating function" by Doubilet, Rota and Stanley, and there is a relation treated which is very reminiscent of the relation between ideals in a polynomial ring and affine algebraic varieties (i won't go more specific in the definitions).
-It goes as follows (everything quoted from the above paper): Let $P$ be a finite poset (can be generalized to locally finite), and consider its incidence algebra $I(P,K)$, consisting of all the functions from the intervals in $P$ to some field $K$ (of characteristic zero). Sum and product by scalars are inherited from $K$, and product of two functions $f,g \in P$ is defined as the convolution:
-\begin{equation}
-(f*g)(x,y)=\sum_{x\leq z \leq y}f(x,z)g(z,y)
-\end{equation}
-Some special elements in $I(P,K)$ needed to state the connection are the units:
-\begin{equation}
-\delta_{x,y}(u,v)=\begin{cases}1&\text{if $u=x$ and $v=y$,}\\\\0&\text{otherwise.}\end{cases}
-\end{equation}
-Now, the (two sided) ideals in this algebra and the varieties have a very nice relation just very similar to the one from commutative algebra and algebraic geometry. But in this case the relation is tighter, because varieties have an algebraic structure coming from a natural partial ordering.
-Define the support of and ideal $J$, $\Delta (J)$, as the set of all the units $\delta_{x,y}$ belonging to $J$. It turns out that every ideal $J$ in $I(P,K)$ consists of all the functions $f$ for which $f(x,y)=0$ whenever $\delta_{x,y}\notin \Delta(J) $.
-On the other hand, define $Z(J)$ as the set of all intervals $\[ x,y \]$ such that $f(x,y)=0$ for all $f\in J$ (this would be the variety). $Z(J)$ is an order ideal of the poset of all intervals of $P$ (ordered by inclusion).
-Theorem: Let $P$ be a finite poset and $S(P)$ the poset of its intervals, ordered by inclusion. Then there is a natural anti-isomorphism between the lattice of ideals of $I(P,K)$ and the lattice of order ideals of $S(P)$.
-(For more details and background, check the paper, or "Enumerative Combinatorics Vol.1" by Stanley)
-
-My question is: Does anyone know if this ideal-variety duality has been exploited or studied further in the context of posets from an algebraic geometry point of view? (apart from the material in the mentioned paper).
-
-REPLY [7 votes]: Joyal and Tierney have done Algebraic Geometry with lattices (in topoi) somewhat along the lines you describe in their landmark monograph An extension of the Galois Theory of Grothendieck<|endoftext|>
-TITLE: Do the converses of [weak law of large numbers / central limit theorem] hold?
-QUESTION [8 upvotes]: Let $\; X_0,X_1,X_2,X_3,...\;$ be independent and identically distributed (real-valued) random variables.
-1.
-Suppose $\frac1n \cdot\sum\limits_{m=0}^n X_m$ converges in probability. Does it follow that $\operatorname{E}(X_0)$ exists?
-2.
-Suppose $\operatorname{E}(X_0) = 0$ and that $\frac1{\sqrt n} \cdot\sum\limits_{m=0}^n X_m$ converges in distribution to a normal random variable.
-Does it follow that $\operatorname{E}((X_0)^2)$ is finite?
-(I already found that the converse of the strong law of large numbers holds.)
-
-REPLY [8 votes]: (As suggested, I promote my comment to an answer, with pgassiat's complement.)
-Necessary and sufficient conditions (in terms close to those you want) for the WLLN and the CLT can be found, e.g., in "Foundations of modern probability" by Kallenberg (Theorems 4.16 and 4.17 in the first edition, Theorems 5.16 and 5.17 in the second edition).<|endoftext|>
-TITLE: True by accident (and therefore not amenable to proof)
-QUESTION [27 upvotes]: The graph reconstruction conjecture claims that (barring trivial examples) a graph on n vertices is determined (up to isomorphism) by its collection of (n-1)-vertex induced subgraphs (again up to isomorphism).
-The way it is phrased ("reconstruction") suggests that a proof of the conjecture would be a procedure, indeed an algorithm, that takes the collection of subgraphs and then ingeniously "builds" the original graph from these.
-But based on some experience with a related conjecture (the vertex-switching reconstruction conjecture), I am led to wonder whether this is something that is simply true "by accident". By this I mean that it is something that is just overwhelmingly unlikely to be false ... there would need to be a massive coincidence for two non-isomorphic graphs to have the same "deck" (as the collection of (n-1)-vertex induced subgraphs is usually called). In other words, the only reason for the statement to be true is that it "just happens" to not be false.
-Of course, this means that it could never actually be proved.. and therefore it would be a very poor choice of problem to work on!
-My question (at last) is whether anyone has either formalized this concept - results that can't be proved or disproved, not because they are formally undecidable, but just because they are "true by accident" - or at least discussed it with more sophistication than I can muster.
-EDIT: Apologies for the delay in responding and thanks to everyone who contributed thoughtfully to the rather vague question. I have accepted Gil Kalai's answer because he most accurately guessed my intention in asking the question.
-I should probably not have used the words "formally unprovable" mostly because I don't really have a deep understanding of formal logic and while some of the "logical foundations" answers contained interesting ideas, that was not really what I was trying to get at.
-What I was really trying to get at is that some assertions / conjectures seem to me to be making a highly non-obvious statement about combinatorial objects, the truth of which depends on some fundamental structural understanding that we currently lack. Other assertions / conjectures seem, again, to me, to just be saying something that we would simply expect to be true "by chance" and that we would really be astonished if it were false.
-Here are a few unproved statements all of which I believe to be true: some of them I think should reflect structure and others just seem to be "by chance" (which is which I will answer later, if anyone is still interested in this topic).
-(1) Every projective plane has prime power order
-(2) Every non-desarguesian projective plane contains a Fano subplane
-(3) The graph reconstruction conjecture
-(4) Every vertex-transitive cubic graph has a hamilton cycle (except Petersen, Coxeter and two related graphs)
-(5) Every 4-regular graph with a hamilton cycle has a second one
-Certainly there is a significant chance that I am wrong, and that something that appears accidental will eventually be revealed to be a deep structural theorem when viewed in exactly the right way. However I have to choose what to work on (as do we all) and one of the things I use to decide what NOT to work on is whether I believe the statement says something real or accidental.
-Another aspect of Gil's answer that I liked was the idea of considering a "finite version" of each statement: let S(n) be the statement that "all non-desarguesian projective planes of order at most n have a Fano subplane". Then suppose that all the S(n) are true, and that for any particular n, we can find a proof - in the worst case, "simply" enumerate all the projective planes of order n and check each for a Fano subplane. But suppose that the length of the shortest possible proof of S(n) tends to infinity as n tends to infinity - essentially there is NO OTHER proof than checking all the examples. Then we could never make a finite length proof covering all n. This is roughly what I would mean by "true by accident".
-More comments welcome and thanks for letting me ramble!
-
-REPLY [17 votes]: This is a very interesting (yet rather vague) question. Most answers were in the direction of mathematical logic but I am not sure this is the only (or even the most appropriate) way to think about it. The notion of coincidence is by itself very complicated. (See http://en.wikipedia.org/wiki/Coincidence ). One way to put it on rigurous grounds is using probabilistic/statistical framework. Indeed, as Timothy mentioned it is sometimes possible to give a probabilistic heuristic in support of some mathematical statement. But its is notorious statistical problem to try to determine aposteriori if some events represent a coincidence.
-I am not sure that (as the OP assumes) if a statement is "true by accident" it implies that it can never be proved. Also I am not sure (as implied by most answers) that "can never be proved" should be interpreted as "does not follow from the axioms". It can also refers to situations where the statement admits a proof, but the proof is also "accidental" as the original statement is, so it is unlikely to be found in the systematic way mathematics is developed.
-In a sense (as mentioned in quid's answer), the notion of "true by accident" is related to mathematics psychology. It is more related to the way we precieve mathematical truths than to some objective facts about them.
-Regarding the reconstruction conjecture. Note that we can ask if the conjecture is true for graphs with at most million vertices. Here, if true it is certainly provable. So the logic issues disappear but the main issue of the question remains. (We can replace the logic distinctions by computational complexity distinctions. But still I am not sure this will cpature the essence of the question.) There is a weaker form of the conjecture called the edge reconstruction conjecture (same problem but you delete edges rather than vertices) where much is known. There is a very conceptual proof that every graph with n vertices and more than nlogn edges is edge-reconstructible. So this gives some support to the feeling that maybe vertex reconstruction can also be dealt with.
-Finally I am not aware of a heuristic argument that "there would need to be a massive coincidence for two non-isomorphic graphs to have the same 'deck'" as the OP suggested. (Coming up with a convincing such heuristic would be intereting.) It is known that various graph invariants must have the same value on such two graphs.<|endoftext|>
-TITLE: Properties from Tropical Geometry that do not imply their algebraic counterpart.
-QUESTION [9 upvotes]: One of the motivations to study tropical geometry is that there are some hard Algebraic Questions that can be answered by proving them in the Tropical World. For example one can show that tropical Bezout's Theorem implies the Algebraic Bezout.
-What properties are there known that are true (or might be) in tropical geometry that don't imply that their algebraic version is true?
-
-REPLY [7 votes]: There is a simple nice fact which holds in the tropical plane that has no counterpart in algebraic geometry (nor in any kind of standard geometry I might think of): two tropical lines always "intersect" in a single point... even if they coincide!
-Of course this property relies on the fact that "intersection" is not defined in the usual way. We define the "intersection" of two tropical curves $C_1$ and $C_2$ as follows: the union $C_1\cup C_2$ has a natural cellularization into vertices and edges, and "$C_1\cap C_2$" is the union of the vertices contained in the set-theoretic intersection $C_1\cap C_2$. One may also define a multiplicity on each intersection point. With this definition, the intersection of a tropical curve with itself is the union of its vertices.
-Therefore two (possibly coinciding) tropical lines always intersect in a point. By defining anagously an appropriate (dual) notion of "span", the dual sentence is also true: two (possibly coinciding) points always span a single line.<|endoftext|>
-TITLE: What is the theory of polynomials?
-QUESTION [16 upvotes]: We all know what polynomials are, along with their elementary properties and many effective algorithms for different representations of polynomials.
-The question here is more of a universal algebra question: what is the signature of the theory which best corresponds to polynomials? To illustrate what this question means, it is probably easiest to do this by example:
-
-In the category of Unital Rings, the integers are the initial algebra.
-In the category of semirings, the natural numbers are the initial algebra.
-
-So what is a small presentation of a category (in the sense of giving signatures and axioms) for which the polynomials are initial? Naturally, for $R[x]$, one expects that this presentation will either include the presentation of the ring $R$, or be parametric in that presentation. But what else is needed to characterize univariate polynomial rings from general rings?
-The motivation is that I am looking for a semantic type for univariate polynomials. In most cases, the type for polynomials one encounters in the litterate is the type of its representations. This is like saying that a matrix has semantic type 'square array', rather than to say that a matrix (in linear algebra) is a representation of a linear operator (with linear operator being the correct semantic type).
-
-EDIT: one note of clarification. After I figure out the 'theory of polynomials', I then wish to be able to write it down as well, so I want a 'presentation', universal-algebra-style, of the 'theory of polynomials' (whether that is plethories or free V-algebras on one generator or ...). With the integers, it is easy to write down a set of axioms that define unital rings.
-
-REPLY [3 votes]: IMHO the answer to “where polynomials are initial” (not to the title question, which is too broad for me) is already given in “Awodey. Category theory. 9. Adjoints. 9.3. Examples of adjoints. Example 9.10.”
-In that example, the adjunction of functors is constructed, where its free (left adjoint) functor $F$ goes from the category of rings (=RingCat) to the category of rings with distinguished element (= pointed rings). If we define this adjunction via unit ($\eta$), then the definition says that
-
-for every object $R$ in RingCat (= for
- every ring $R$) there is an
- initial object in the category $select(R)\downarrow U$ (comma
- category),
-
-where $U$ is the forgetful functor.
-Furthermore, a chosen initial object for $R$ consists of $F(R)$ (= $R[x]$) and $\eta(R):R\to U(F(R))$ (= a ring homomorphism constructing constant polynomials). The distinguished element in $F(R)$ is “$x$” (the projection polynomial).<|endoftext|>
-TITLE: Cohomology of Theta divisor on Jacobian?
-QUESTION [6 upvotes]: Let $C$ be a curve of genus $g \geq 1$ and let $J^d$ be its degree $d$ Jacobian.
-Inside of $J^{g-1}$ there is the Theta divisor $\Theta$, which can be defined in various ways; the quickest definition is probably: it's the image of the Abel-Jacobi map $C^{(g-1)} \to J^{g-1}$ sending an effective degree $g-1$ divisor to the corresponding line bundle. Picking an isomorphism $J^{g-1} \cong J^d$, we also write $\Theta$ for the corresponding divisor in $J^d$.
-
-How to compute $H^\ast(J;\Theta)$, or $h^\ast(J;\Theta)$? Or alternatively, what is known about these groups?
-
-I suspect this is something embarrassingly standard and/or obvious and/or well-known and/or classical, but I haven't been able to figure anything out. The only thing along these lines that I was able to figure out was how to compute the Euler characteristic $\chi(J;\Theta^k)$ where $k$ is an integer: By Hirzebruch-Riemann-Roch and the Poincare formula it's $$\int_J \operatorname{ch}(\Theta^k) = \int_J e^{k\theta} = \int_J k^g \theta^g / g! = k^g.$$
-
-REPLY [5 votes]: this seems to be the kodaira vanishing theorem. i.e. any line bundle of form K+A where is ample, has no higher cohomology. for an abelian variety K is trivial, and Theta is ample. qed.<|endoftext|>
-TITLE: "Rounding the corners" to get contact boundary
-QUESTION [5 upvotes]: Suppose we have symplectic manifolds $(M_1, \omega_1)$ and $(M_2, \omega_2)$ with non-empty boundary of contact . Often we need to deal with the product $M_1 \times M_2$ with the product symplectic structure. Can we round the corners to get a contact manifold as boundary?
-
-REPLY [6 votes]: In that generality, the answer is no: a symplectic form $\omega$ on $X$ which has contact-type boundary is exact on $\partial X$. Yet $\omega_1 \oplus \omega_2$ need not be exact on $M_1\times \partial M_2$, nor on $\partial M_1 \times M_2$.
-It is possible, however, if $M_1$ and $M_2$ are Liouville domains, i.e., if the symplectic form $\omega_i$ is given as $d\theta_i$ for 1-forms $\theta_i$ whose dual vector field $\lambda_i$ points strictly outwards along the boundary. In fact, if you round corners sensibly, $\theta_1 \oplus \theta_2$ will have those same properties on the product.
-Here's a relevant article by Alex Oancea:
-http://arxiv.org/abs/math/0403376<|endoftext|>
-TITLE: An exercise in Jech's Set Theory
-QUESTION [6 upvotes]: I had a hard time trying to solve exercise 7.24 in Jech's book (3rd edition, 2003) and finally came to the conclusion that the result there, which should be proved might be wrong. The claim goes like this:
-Let $A$ be a subalgebra of a Boolean algebra $B$ and suppose that $u \in B-A$. Then there exist ultrafilters $F,G$ on $B$ such that $u \in F$, $-u \in G$ and $F \cap A= G \cap A$.
-A (perhaps flawed, as I believe) proof of this can be found here.
-http://onlinelibrary.wiley.com/doi/10.1002/malq.19690150705/abstract
-A counterexample to the claim above is the following:
-Let $A$ be the algebra of finite unions of (open, closed, half-open) intervals on $[0,1]$ with rational endpoints, and let $B$ be defined as $A$ but with real endpoints. Each ultrafilter $U$ on $A$ converges to a rational or irrational number $r$ and the elements of $U$ are exactly those sets in $A$ that include $r$. Now if $F$ and $G$ are two ultrafilters on the bigger algebra $B$, both extending $U$ then they converge again towards $r$ and for any $u\in B$ we have that $u\in F$ iff $r \in u$ iff $u\in G$, which makes it impossible to have $u \in B$, yet $-u \in G$.
-My questions are now:
-
-Is my counterexample correct?
-The claim is used to show that each Boolean algebra of size $\kappa$ has at least $\kappa$ ultrafilters (this is theorem of the paper mentioned above). Does this remain valid ( I suppose not, see the comments)
-
-REPLY [9 votes]: The exercise is correct. Let $u\in B\setminus A$. We say that $u$ splits an ultrafilter $H$ of $A$ if $\{u\}\cup H$ and $\{-u\}\cup H$ both have the finite intersection property. (If $u$ splits $H$, then there are ultrafilters $F$ and $G$ of $B$ such that
-$F\cap A=H=G\cap A$, $u\in F$, and $-u\in G$.)
-Suppose no ultrafilter $H$ is split by $u$.
-We say that an ultrafilter $H$ of $A$ is compatible with $b\in B$ iff each $a\in H$ has
-nonempty intersection (in $B$) with $b$. If no ultrafilter of $A$ is split by $u$,
-then each ultrafilter is either compatible with $u$ or compatible with $-u$.
-Let $C$ be the set of ultrafilters of $A$ compatible with $u$, and let $D$ be the set of ultrafilters of $A$ compatible with $-u$.
-Now the set Ult$(A)$ of all ultrafilters of $A$ is the disjoint union of $C$ and $D$.
-Hence an ultrafilter of $A$ is compatible with $u$ iff it is not compatible with $-u$ and vice versa. So, if $H\in C$, then there is $a\in H$ such that $a$ is disjoint from $-u$.
-All ultrafilters of $A$ that contain $a$ are incompatible with $-u$ and hence compatible with
-$u$. This shows that $C$ is open in the Stone space Ult$(A)$ of $A$.
-The same is true for $D$. It follows that the two sets are clopen.
-By the Stone representation theorem, there is $a\in A$ such that $C$ is the set of all ultrafilters $H$ of $A$ that contain $a$. $D$ is the set of all ultrafilters of $A$ that contain $-a$.
-In other words, an ultrafilter $H$ of $A$ is compatible with $u$ iff $a\in H$.
-But this implies that an element $b$ of $A$ has a nonempty intersection with $u$
-iff it has a nonempty intersection with $a$. Hence $-a$ is disjoint from $u$.
-In other words, $u\leq a$.
-The symmetric argument shows that $-u\leq-a$.
-It follows that $a=u$ and hence $u\in A$, a contradiction.
-And yes, this exercise implies that every infinite Boolean algebra of size $\kappa$ has at least $\kappa$ ultrafilters.<|endoftext|>
-TITLE: Reference request: Spec A_* is the automorphism group of the additive formal group law
-QUESTION [7 upvotes]: Dear all,
-I'm seeking a reference for a claim made in lecture 8 of Jacob Lurie's chromatic homotopy theory notes (http://www.math.harvard.edu/~lurie/252xnotes/Lecture8.pdf). More particularly, Theorem 6 of this lecture states that (say over $\mathbb{F}_2$, so that things are commutative) the spectrum $\mathbb{G} = \operatorname{Spec} \mathcal{A}_*$ of the dual Steenrod algebra $\mathcal{A}_*$ is the automorphism group of the additive formal group law, in the obvious sense.
-Lurie argues convincingly that $\mathbb{G}$ does act on the additive formal group law, but I don't think he attempts to prove that this action gives an isomorphism with the automorphism group. I'd be grateful if someone could give me a reference for this fact.
-Cheers,
-Saul
-
-REPLY [7 votes]: If you're looking for a reference in print, it's in Ravenel's book Complex Cobordism and Stable Homotopy Groups of Spheres. See the comments after the proof of Theorem A2.2.18. (This book is available online, and you want Appendix 2.)<|endoftext|>
-TITLE: Dehn's solution to Hilbert's 3rd: 1901 or 1902?
-QUESTION [5 upvotes]: This is a simple bibliographic request that I have been unable to pin down. Max Dehn's
-solution to Hilbert's 3rd problem is:
-
-Max Dehn, "Über den Rauminhalt." Mathematische Annalen 55 (190x), no. 3, pages 465–478.
-
-It is variously cited as either 1901 or 1902 (but always volume 55; Hilbert's own footnote
-cites volume 55 "soon to appear"). E.g.,
-
-Mathworld cites it as 1902.
-The Encyclopedic Dictionary of Mathematics cites it as 1902.
-Wikipedia says 1901.
-Various papers, e.g., this one, and Tao's book, cite it as 1901.
-
-I have been unsuccessful in finding the definitive year via the web, because of all
-the conflicting citations. The next step is to retrieve
-Mathematische Annalen volume 55, but perhaps someone can spare me that trouble...?
-Thanks!
-
-REPLY [6 votes]: Another point to consider is whether "Über den Rauminhalt"
-is in fact Dehn's first solution to Hilbert's 3rd Problem. I
-believe his first solution was in the paper "Über raumgleiche
-Polyeder" in the Nachrichten der Königliche Gesellschaft der
-Wissenschaften zu Göttingen of 1900, pp. 345 -- 354.<|endoftext|>
-TITLE: when mapping cone is contractible
-QUESTION [9 upvotes]: It is quite obvious that if a map is a homotopy equivalence, then its mapping cone is contractible, but is the converse true: mapping cone contractible => the map is a homotopy equivalence? I am thinking about both the topological category and the category of chain complexes.
-
-REPLY [3 votes]: The accepted answer addresses the version of the question for topological spaces. But there is also a positive result for triangulated categories: a morphism $f \colon X \to Y$ is an isomorphism if and only if any distinguished triangle $X \to Y \to Z \to X[1]$ has $Z \cong 0$. See Tag 05QR.
-This in particular applies to the homotopy category $K(\mathcal A)$ of chain complexes with values in an additive category $\mathcal A$, which is a triangulated category by Tag 014S. Applying this to the mapping cone in $K(\mathcal A)$ shows that $C(f)$ is contractible if and only if $f$ is a homotopy equivalence.
-This also applies to the stable homotopy category of spectra, where it implies that a map of topological spaces $f \colon X \to Y$ with contractible mapping cone is a stable homotopy equivalence.
-In examples like David White's answer, you only need to go up to the first suspension $\Sigma X$, by construction (but see also this answer). I'm not sure if something like this is supposed to be true in general, but I'm guessing it's more complicated.<|endoftext|>
-TITLE: What does the "category" of $(\infty,1)$ category look like.
-QUESTION [6 upvotes]: One knows that in higher category theory, the category of $(\infty,n-1)$ categories is naturally an $(\infty,n)$ category ,(I use the word category to mean category in the correct weakened sense). When the category of $(\infty,1)$ categories is regarded as a weakened kan complex, we may regard these objects as a full subcategory of simplicial sets. This is a category in the strict sense. One ought to expect that associativity of the maps between the weakened kan complex be some sort of weakened associativity. The question is: is this weakened associativity there, and if so how is it understood?
-
-REPLY [11 votes]: You can see the collection of $(\infty,1)$-categories as forming themselves an $(\infty,1)$-category, which is sufficient to see where weak associativity shows up: There are many models for the intuiti9ve concept of $(\infty,1)$-category, the simplest is that of a usual 1-category endowed with a class of weak equivalences (see Barwick/Kan's "Relative Categories: Another model for the homotopy theory of homotopy theories" here).
-With the weak Kan complexes - together with the notion of equivalence between them - you happen to have found a strictly associative model for the $(\infty,1)$-category of $(\infty,1)$-categories. You can transform it into different other models, e.g. into simplicially enriched categories or quasicategories or Segal categories, as exposed e.g. in Bergner's "Three models for the homotopy theory of homotopy theories" (available here).
-The Segal category and the quasicategory of $(\infty,1)$-cats do no longer have strict associativity and the fact that they are equivalent descriptions of the $(\infty,1)$-cat of $(\infty,1)$-cats reflects that the strict associativity in your model was an accident and not an essential feature...
-Edit (in response to the comment) About the significance of having a strict model: Well, different models have different advantages. Your strict one is certainly good for computing compositions of functors between $(\infty,1)$-cats. The quasicategory of $(\infty,1)$-cats on the other hand is e.g. a better model to relate the $(\infty,1)$-cat of $(\infty,1)$-cats to other $(\infty,1)$-cats - examples are the relations between the quasicategories of (small) $(\infty,1)$-cats, presentable $(\infty,1)$-cats and stable $(\infty,1)$-cats given in Lurie's "Higher Topoi" and in DAG 1 (now "Higher Algebra"): There are $(\infty,1)$-adjunctions between these - e.g. between $(\infty,1)$-cats and stable $(\infty,1)$-cats given by taking spectra in an $(\infty,1)$-cat, forgetting the stability of a stable $(\infty,1)$-cat, respectively - and these facts would be hard to express using your model.<|endoftext|>
-TITLE: countable union of closed subschemes over uncountable field
-QUESTION [9 upvotes]: I am looking for a reference for the following well-known fact:
-Let $k$ be an uncountable field, and let $X$ be a $k$-variety. Let $Z_1, Z_2, \dots \subseteq X$ be proper closed subschemes. Then $\bigcup Z_i(k) \neq X(k)$.
-Thanks!
-
-REPLY [11 votes]: Suppose $\dim X>0$ and $k$ is algebraically closed and uncountable. Moreover, if a "variety" is not necessarily irreducible, the $Z_i$ are supposed to have positive codimension in $X$ (otherwise one could take the irreducible components of $X$).
-As in MP's answer, one can suppose $X$ is affine. By Noether's Normalization Lemma, there exists a finite surjective morphism $p: X\to \mathbb A^m_k$ with $m=\dim X$. Let $Y_i=p(Z_i)$. This is a closed subset
-of $\mathbb A^m_k$ of positive codimension. Moreover $\mathbb A^m_k(k)=\cup Y_i(k)$ because $k$ is algebraically closed (which implies that $Y_i(k)=p(Z_i(k))$). As $k$ is uncountable, there exists a hyperplane $H$ in $\mathbb A^m$ not contained in any $Y_i$ (note that $H\subseteq Y_i$ is equivalent to $H=Y_i$). So by induction on $m$ we are reduced to the case $m=1$, and the assertion is obvious.
-Without the hypothesis $k$ algebraically closed, one can show similarly that $X\ne \cup_i Z_i$. This is Exercise 2.5.10 in my book. EDIT In fact this statement is trivial because the generic points of $X$ don't belong to any of the $Z_i$'s. But the proof shows that the set of closed points of $X$ is not contained in $\cup_i Z_i$.<|endoftext|>
-TITLE: torsion group of the multiplicative group of a field
-QUESTION [11 upvotes]: Let $F$ be any field of zero characteristic, $F^{\ast}$ its multiplicative group and $T$ is the torsion group.
-Is it true that $T$ is a direct summand for $F^{\ast}$?
-
-REPLY [14 votes]: This was a problem that was asked by Fuchs in his book "Abelian groups" (1958). It was first solved in negative by P. M. Cohn in "Eine Bemerkung uber die multiplikative Gruppe eines Korpers", Arch. Math. (Basel) 13 (1962) 344-48. (MR0146252). Later W. May gave a counterexample as an algebraic extension of $\mathbb Q$, in "Multiplicative groups of fields", Proc. London Math. Soc. (3) 24 (1972), 295–306. (MR0294490)<|endoftext|>
-TITLE: Concrete example of $\infty$-categories
-QUESTION [26 upvotes]: I've seen many different notions of $\infty$-categories: actually I've seen the operadic-globular ones of Batanin and Leinster, and the opetopic, and eventually I'll see the simplicial ones too. Although there are so many notions of $\infty$-category, so far I've only seen the following examples:
-
-$\infty$-groupoids as fundamental groupoids topological spaces;
-$(\infty,1)$-categories, mostly via topological example and application in algebraic geometry (in particular in derived algebraic geometry);
-strict $(\infty,\infty)$-categories, and their $n$-dimensional versions, for instance the various categories of strict-$n$-categories (here I intend $n \in \omega+\{\infty\}$).
-
-
-There are other examples of $\infty$-categories, especially from algebraic topology or algebraic geometry, but also mathematical physics and computer science and logic?
- In particular I am wondering if there's a concrete example, well known, weak $(\infty,\infty)$-category.
-
-(Edit:) after the a discussion with Mr.Porter I think adding some specifications may help:
-
-I'm looking for models/presentations of $\infty$-weak-categories for which is possible to give a combinatorial description, in which is possible to make manipulations and explicit calculations, but also $\infty$-categories arising in practice in various mathematical contexts.
-
-REPLY [6 votes]: A concrete example of a weak $\infty$-category, but not well studied abstractly, is the cubical singular complex of a space, preferably with the connections introduced in our 1981 JPAA paper with Philip Higgins. The clear advantage of the cubical setup is the command of multiple compositions, using an easy matrix notation. Thus one can express for the diagram
- (source)
-that the big square is the composition of the little squares, by simply writing something like $\alpha=[\alpha_{ij}]$. (How does one do that simplicially, or globularly?) This and higher dimensional versions are useful in expressing algebraic inverses to subdivision, a useful tool in local-to-global problems. Because higher groupoids are nonabelian, unlike higher groups, one can also obtain nonabelian results in algebraic topology.
-The book Nonabelian algebraic topology: filtered spaces, crossed complexes, cubical homotopy groupoids (EMS, (2011), pdf available from here) has a large number of uses of algebraic and geometric (e.g. homotopical) conclusions derived from rewriting such multiple arrays. The main results were conjectured and eventually proved by cubical methods.<|endoftext|>
-TITLE: Drawing natural numbers without replacement.
-QUESTION [24 upvotes]: Suppose we start with an initial probability distribution on $\mathbb{N}$ that gives positive probability to each $n$. Let's call this random variable $X_1$ so we have $P(X_1=n)=p_{1,n}>0$ for all $n\in\mathbb{N}$. $X_1$ wil be the first draw from $\mathbb{N}$. For the next draw $X_2$ we define a new distribution on $\mathbb{N}\setminus\{ X_1 \}$ by rescaling the remaining probabilities so they add up to 1. So $p_{2,X_1}=0$ and $p_{2,n}=\frac{p_{1,n}}{1-p_{1,X_1}}$ for $n\neq X_1$. Continuing in this manner we get a stochastic process (certainly not Markov) that corresponds to drawing from $\mathbb{N}$ without replacement. My question is whether this process has ever been studied in the literature. In particular, I'm wondering if a clever choice of the initial distribution could result in tractable expressions for the distributions of $X_n$ for large $n$.
-
-REPLY [4 votes]: Here are some preliminary computations. Assume the reference distribution is $(p(n))$. For every finite subset $I$ of $\mathbb N$, introduce the finite number $r(I)\ge1$ such that
-$$
-\frac1{r(I)}=1-\sum_{k\in I}p(k).
-$$
-Obviously, $P(X_1=n)=p(n)$ for every $n$. Likewise,
-$P(X_2=n)=E(p(n)r(X_1);X_1\ne n)$ hence
-$$
-P(X_2=n)=p(n)(\alpha-p(n)r(n)),\qquad
-\alpha=\sum\limits_kp(k)r(k).
-$$
-This shows that $X_1$ and $X_2$ are not equidistributed (if they were, $\alpha-p(n)r(n)$ would not depend on $n$, hence $p(n)$ would not either, but this is impossible since $(p(n))$ is a measure with finite mass on an infinite set).
-One can also compute the joint distribution of $(X_1,X_2)$ as
-$$
-P(X_1=n,X_2=k)=p(n)r(n)p(k)[k\ne n],
-$$
-and this allows to expand
-$$
-P(X_3=n)=E(p(n)r(X_1,X_2);X_1\ne n,X_2\ne n),
-$$
-as the double sum
-$$
-P(X_3=n)=p(n)\sum_{k\ne n}\sum_{i\ne n}[k\ne i]r(k,i)p(k)r(k)p(i),
-$$
-but no simpler or really illuminating expression seems to emerge.<|endoftext|>
-TITLE: Continuous extensions reals and to p-adic numbers
-QUESTION [7 upvotes]: Assume $f\colon \mathbb Q\to \mathbb Q$ is a function which admits continuous extensions
-
-$f_0\colon\mathbb R\to \mathbb R$ and
-$f_p\colon \mathbb Q_p\to \mathbb Q_p$ for each prime $p$.
-
-
-Is it true that $f$ is a polynomial?
-
-I guess the answer is no, but I do not see a counterexample.
-
-REPLY [14 votes]: The answer is no, and one can essentially use the same construction as in the answer:
-Is a real power series that maps rationals to rationals defined by a rational function?
-Specifically, enumerate the non-zero rationals $\{r_1,r_2, \ldots\}$ in some way. Now consider the function:
-$$f(x) = \sum_{n=1}^{\infty} c_n x^{n^2} \prod_{i=1}^{n} (x - r_i).$$
-If $c_n \in \mathbf{Q}$, then this is a well defined function from rationals to rationals.
-On the other hand, $f(x)$ converges to an analytic function in $\mathbf{Q}_v$ if and only if the coefficients of this power series converge to zero fast enough.
-Since the coefficients of the power series in the range $k = n^2$ to $k < (n+1)^2$
-are simply the cofficients of $c_n x^{n^2} \prod_{i=1}^{n} (x - r_i)$, this can be ensured
-by forcing these coefficients to be very highly divisible by the first $n$ primes, and small in the archimedean sense (by including a very very large prime in the denominator).<|endoftext|>
-TITLE: Relationship between the focal locus and the cut locus
-QUESTION [5 upvotes]: I am seeking
-clarification of
-the relationship between the
-focal locus
-and the
-cut locus
-of a curve $C$ in $\mathbb{R}^2$, and
-of a surface $S$ in $\mathbb{R}^3$.
-Essentially my question is,
-
-Under what conditions is the focal locus and the cut locus identical,
- when do they differ, and when they differ, how do they differ.
-
-For example, it believe the two coincide for a sphere
-(in any dimension): both are simply the center point of the sphere.
-It may be that these issues are primarily definitional
-rather than substantive. Let me offer the definitions
-with which I am working.
-Cut Locus.
-Generally the
-cut locus
-is defined on a Riemannian manifold with respect to a point.
-But instead I want to define the cut locus of a set in $\mathbb{R}^n$.
-Let me follow the definition of
-Franz-Erich Wolter, who wrote his Ph.D. thesis on the topic:
-
-"The cut locus $C_A$ of a closed set $A$ in the Euclidean space $E$
- is defined as the closure of the set containing
- all points $p$ which have at least two shortest paths to $A$."
-
-(This is quoted from reference (1) below.)
-This definition is in accord with that of the
-medial axis,
-extensively explored in computer science.
-Focal locus.
-I am having more difficulty locating a widely accepted definition
-of the focal locus.
-Let me follow Thorpe:
-
-"The focal locus of a plane curve $C$ is the locus of the centers of curvature and is often called the evolute of $C$." ...
- "The set of all focal points along all normal lines to an $n$-surface $S$
- in $\mathbb{R}^{n+1}$ is called the focal locus of $S$."
- ...
- Let $\phi$ be a parametrized $n$-surface,
- and let $\beta$ be "a unit-speed parametrization of the line normal to Image $\phi$ at $\phi(p)$.
- A point $f$ is said to be a focal point of $\phi$ along $\beta$
- if $f = \beta(s_0)$ where $s_0$ is such that the map ...
- $\phi(q) + s_0 N^\phi(q)$ is singular (not regular) at $p$" [where $N^\phi(q)$ is
- the normal at $q$].
-
-(These quotes are from reference (2) below.)
-One aspect of the focal locus that confuses me is that there
-is a notion of focal surfaces, which derive from
-"the reciprocal of the principal curvatures,"
-as described in (3). Here there are two surfaces,
-as opposed to one focal locus, as depicted in this intriguing figure:
-
-
-
-
-It may be that there are references that
-would resolve my definitional confusions, in which case
-pointers would be much appreciated. Thanks!
-
-References.
-
-
-Franz-Erich Wolter.
-"Cut Locus and Medial Axis in Global Shape Interrogation and Representation."
-MIT Ocean Engineering Design Laboratory Memorandum 92-2.
-December 1993.
-PDF.
-
-John A. Thorpe.
-Elementary Topics in Differential Geometry.
-Springer, 1979.
-Google books. Quote from p.137.
-
-
-Jingyi Yu, Xiaotian Yin, Xianfeng Gu, Leonard McMillan, and Steven Gortler.
-"Focal Surfaces of Discrete Geometry." 2007.
-Proceedings of the 5th Euro
-Graphics Symposium on Geometry Processing.
-
-REPLY [3 votes]: I believe that the focal locus is the same thing as the conjugate locus in Riemannian geometry. Given a codimension 1 submanifold $S \subset \mathbb{R}^n$, the exponential map $S \times \mathbb{R} \rightarrow \mathbb{R}^n$ is given by $(x, t) \mapsto x + t\gamma(x)$, where $\gamma$ is the Gauss map. The cut locus is the closure of all points in $\mathbb{R}^n$ that have more than one pre-image. The focal points is the closure of all points where the map is not a diffeomorphism.
-Given any point where the curvature $\kappa$ is nonzero on a smooth curve in the plane, there is a corresponding point on the focal locus at distance $1/|\kappa|$ on the side of the curve that is inwardly curved.
-I believe that the focal locus is always contained in the cut locus.
-ADDED: (Corrected definition of map above)...If you differentiate the exponential map (the one I define above), then since the differential of the Gauss map is the second fundamental form, call it $A$, then you see that if a point $y$ lies in the focal locus, there exists $x \in S$, $t \in \mathbb{R}$, and a nonzero $v \in \mathbb{R}^n$ such that $v + tAv = 0$. I neglected to say above that the focal locus corresponds to the closest point on either side of $S$ along the geodesic normal to $x$ where this equation holds. Therefore, there is a focal point on the half line where $t > 0$ only if there is a negative principal curvature and the focal point is at distance $-1/\kappa$, where $\kappa$ is the negative principal curvature with largest magnitude. There is a focal point on the other half line only if there is at a positive principal curvature, and it is at distance $1/\kappa$, where $\kappa$ is the largest positive principal curvature.<|endoftext|>
-TITLE: Does ZF prove that topological groups are completely regular?
-QUESTION [17 upvotes]: Let $\mathbf{G} = \langle G,\cdot,\mathcal{T}\;\rangle$ be a topological group. Let $\mathbf{e}$ be the identity element of $\langle G,\cdot \rangle$.
-Assume $\{\mathbf{e}\}$ is closed in $\langle G,\cal{T}\;\rangle$. Then, I have managed to convince myself that:
-
-ZF proves that $\langle G,\cal{T}\;\rangle$ is regular Haudorff.
-ZF + (Dependent Choice) proves that $\langle G,\cal{T}\;\rangle$ is completely regular.
-
-My questions are:
-
-Are those right?
-Does ZF prove that $\langle G,\cal{T}\;\rangle$ is completely regular?
-If no to question 2, does assuming one or more of following suffice for ZF to conclude that $\langle G,\cal{T}\,\rangle$ is completely regular?
-
-$\mathbf{G}$ is two-sided complete
-$\langle G,\cdot \rangle$ is abelian
-Countable Choice
-
-REPLY [3 votes]: I'm not sure about the proof I gave, But as I checked, it didn't use full AC, but as Asaf mentioned in a comment it uses DC. The following theorem is due to Pontrjagin. See Book by Montgomery and Zippin(Page 29).
-I will give a sketch of proof.
-Note: Maybe it's needed to add some separation axiom to the following theorem. please edit it, if needed.
-Theorem: Every $T_{0}$ topological group is completely regular.
-Proof.
-It's enough to prove that a given topological group $(G,*)$ is completely regular at $e$. Let $F$ be a closed set not containing $e$. Put $O=F^{c}$. Choose symmetric open neighborhoods of $e$, $U_{n}$ By continuity of $*$, such that $U_{0}=O$(w.l.o.g assume $O$ is symmetric) $U_{n+1}^2 \subseteq U_{n} \cap O ~~~~n=0,1,2...$.
-Now for rational numbers of the form $r=\frac{k}{2^n}~~k \in \{1,2,3,...2^n \},~~ n \in \{0,1,2,...\}$ inductively define open neighborhood $V_{r}$ of $e$ such that:
-1) $V_{\frac{1}{2^n}}=U_{n}~~~~~\forall n$
-2) $V_{\frac{2k}{2^{n+1}}}=V_{\frac{k}{2^n}}$
-3) $V_{\frac{2k+1}{2^{n+1}}}=V_{\frac{1}{2^{n+1}}}V_{\frac{k}{2^n}}$
-The definition does not depend to the representation of $r$ and the family $V_{r}$ has the following properties:
-4) $V_{\frac{1}{2^n}}V_{\frac{m}{2^{n}}} \subseteq V_{\frac{m+1}{2^n}}~~~~~~~~~m+1 \leq 2^n$
-5) $V_{r} \subseteq V_{s}~~~~~$if $~r
-TITLE: For quasi-coherent D-Modules
-QUESTION [7 upvotes]: It is well-know that the category of coherent D-modules over a smooth algebraic $k$-scheme is a Tannakian category. So it is equivalent to the category of finite representations of some affine group scheme G/k. My question is do we have the similar statement for quasi-coherent D-modules? I hope that the category of quasi-coherent D-modules is equivalent to the representation (not necessarily finite) category of some affine group schemes. Is that true, is there any reference for that?
-I hope that any quasi-coherent D-module is the union of its coherent sub D-modules. If the answer to the above question is true then this is true. If the above is wrong. I still believe this is true. At least I think it is true for char$k$=0, where a quasi-coherent D-module is a quasi-coherent sheaf with a flat connection. If this is true, could you give me any reference?
-
-REPLY [7 votes]: The proper Tannakian theory that has a chance to encompass categories of coherent or quasicoherent D-modules deals with tensor categories WITHOUT a faithful fiber functor - for example the categories of (quasi)coherent sheaves on quasicompact varieties, or more generally geoemtric stacks (see Lurie's article or this MO question). (Once you have D-modules that are not local systems, measuring them at a single point can't be faithful any more..)
-Of course to talk about quasicoherent sheaves we need to drop rigidity since
-they are not dualizable objects, but that's not a serious problem, since quasicoherent sheaves (eg D-modules) are unions of coherent ones (eg coherent D-modules, which does NOT mean coherent as O-modules).
-In any case in this context one can hope to recover not an affine group scheme but rather the underlying variety or stack again (the usual Tannakian story is the case of the classifying stack pt/G of an affine group scheme G). Here one builds a stack of all fiber functors (not necessarily faithful), and you can hope to reconstruct the original variety/stack by this procedure (the content of Lurie's theorem).
-So how might this apply to D-modules? D-modules on X (coherent or quasicoherent) are the same as O-modules (coherent or quasicoherent) on the de Rham space (see e.g. this MO question). So it's reasonable to ask whether
-Tannakian reconstruction rebuilds the de Rham space of X from the tensor category of D-modules on X. I believe the answer is yes - certainly there's a natural map
-from the de Rham space of X to the Tannakian functor of D-modules (for every point
-in X you get a fiber functor and infinitesimally nearby points give isomorphic fiber functors). I would imagine this is an equivalence but haven't thought it through.<|endoftext|>
-TITLE: Questions about $\aleph_1-$closed forcing notions
-QUESTION [15 upvotes]: "Foreman`s maximality principle" states that every non-trivial forcing notion either adds a real or collapses some cardinals. This principle has many consequences including:
-1) $GCH$ fails everywhere,
-2) there are no inaccessible cardinals,
-3) there are no $\kappa-$Souslin trees,
-4) Any non-trivial $c.c.c.$ forcing adds a real,
-5) Any non-trivial $\kappa^+-$closed forcing notion collapses some cardinals.
-Consistency of (1) is proved by Foreman-Woodin, (2) clearly can be consistent and the consistency of (4) is shown in "Forcing with c.c.c forcing notions, Cohen reals and Random reals".
-My interest is in the consistency of (5). Let's consider the case $\kappa=\omega.$
-Question 1. Is it consistent that any non-trivial $\aleph_1-$closed forcing notion collapses some cardinals?
-The above question seems very difficult, and it is not difficult to show that for its consistency we need some very large cardinals. But maybe the following is simpler:
-Question 2. Is it consistent that any non-trivial $\aleph_1-$closed forcing notion of size continuum collapses some cardinals? Does its consistency imply the existence of large cardinals?
-
-REPLY [3 votes]: The following results are obtained in a joint work with Yair Hayut and are now presented in our joint paper On Foreman's maximality principle:
-
-Theorem 1. (Assuming the existence of a strong cardinal) There is a model of $ZFC$ in which for all uncountable (regular) cardinals $\kappa,$ any $\kappa$-closed forcing notion of size $\leq 2^{<\kappa}$ collapses some cardinals.
-
-Note that in such a model $GCH$ should fail everywhere and hence some very large cardinals are needed. The following theorem is in the opposite direction:
-
-Theorem 2 (Assuming the existence of a strong cardinal and infinitely many inaccessibles above it) There is a model of $ZFC$ in which $GCH$ fails everywhere and for each uncountable (regular) cardinal $\kappa,$ there exists a $\kappa$-closed forcing notion of size $2^{<\kappa}$ which preserves all cardinals.<|endoftext|>
-TITLE: Dual Borel conjecture in Laver's model
-QUESTION [18 upvotes]: A set $X\subseteq 2^\omega$
-of reals is of strong measure zero (smz) if $X+M\not=2^\omega$
-for every meager set $M$. (This is a theorem of Galvin-Mycielski-Solovay,
-but for the question I am going to ask we may as well take it as a definition.)
-A set $Y$ is strongly meager (sm) if $Y+N\not=2^\omega$ for every Lebesgue null
-set $N$.
-The Borel conjecture (BC) says that every smz set is countable; the dual Borel conjecture (dBC) says that every sm set is countable.
-In Laver's model (obtained by a countable support iteration of Laver reals
-of length $\aleph_2$)
-the BC holds. Same for the Mathias model.
-In a paper that I (with Kellner+Shelah+Wohofsky) just sent to arxiv.org, we claim that it is not clear if Laver's model satisfies the dBC.
-QUESTION:
- Is that correct? Or is it perhaps known that Laver's model has uncountable sm
- sets?
-Additional remark 1: Bartoszynski and Shelah (MR 2020043) proved in 2003
-that in Laver's model there are no sm sets of size continuum ($\aleph_2$).
-(The MR review states that the paper proves that the sm sets are exactly
-$[\mathbb R] ^{\le \aleph_0}$. This is obviously a typo in the review.)
-Additional remark 2: If many random reals are added to Laver's model (either
-during the iteration, or afterwards), then BC still holds, but there will be sm
-sets of size continuum, so dBC fails in a strong sense.
-
-REPLY [5 votes]: In analogy with the fact that Lusin set are strongly null, Pawlikowski has shown that Sierpinksi sets are strongly meagre, so one might ask if there are
-are Sierpinski sets in the iterated Laver Model. The answer is no; indeed
- any set of positive outer measure contains an uncountable null set after adding a Laver real
-Let $\psi:\omega\to\omega$ be a function growing so quickly that
-$$\lim_{n\to\infty} \prod_{j=n}^\infty \left(1- 2^{-\psi(j)} \right) =1 $$
-Then let $L:\omega\to \omega$ be the generic real added by $\mathbb L$ and define
-$A$ to be the set of those $f\in 2^\omega $ such that
-$f$ is constantly $0$ on the interval $[L(n), L(n) + \psi(n)] $ for all but finitely many $n$.
- $A$ is Lebesgue null and it will be shown that $|A\cap X|=\aleph_1$.
-Suppose not and let $T\in \mathbb L$ and $C\in [X]^{\aleph_0}$ be such that
-$T$ forces that $X\cap A\subseteq C$. For $s\in T$ extending the root of $T$
-define $S_s $ to be the set of all $f\in 2^\omega$ for which there exists infinitely many $ n\in \omega$ such that $ s^\frown n\in T$ and $ f(n+j) = 0 $ for all $j\leq \psi(|s|) $. Then each $S_s$ is of measure one. Since $X$ has positive outer measure there is some $x\in \bigcap_{s\supseteq \text{root}(T)}S_s\cap(X\setminus C)$. It follows that there is then
-$\tilde{T}\subseteq T$ such that $\tilde{T}\in \mathbb L$ and
-$x$ is constantly $0$ on the interval $[m,m+\psi(|s|)]$
-for all $s^\frown m\supseteq \text{root}(\tilde{T})$.
-In other words, $\tilde{T}$ forces that $x$ is in $A$ contradicting that $x\in X\setminus C$.<|endoftext|>
-TITLE: Gluing along closed subschemes
-QUESTION [9 upvotes]: Let $Z \to X$ be a closed immersion of schemes. Is it true that for every morphism $Z \to Y$, the pushout $X \cup_Z Y$ in the category of schemes exists? If yes, a) does it turn out to be simply sthe pushout in the category of locally ringed spaces, b) is the natural morphism $Y \to X \cup_Z Y$ a closed immersion?
-In his paper "Gluing Schemes and a Scheme Without Closed Points", Karl Schwede studies such questions. In particular, he gives an affirmative answer if everything is affine (Theorem 3.4), but also for arbitrary schemes if we also assume that $Z \to Y$ is a closed immersion (Corollary 3.9). My intuition says that it should be also true if we drop this condition, but on the other hand the paper shows with some examples that our intuition may be wrong in the context of pushouts. I'm aware that colimits of schemes are not well-behaved in general, but in my research it would be useful to construct pushouts also when just one inclusion is a closed immersion.
-As with this MO question, I think it is not sufficient just to say that some pushout does not exist just because you don't see one. I'm interested in rigorous proofs.
-
-REPLY [2 votes]: To complement Laurent Moret-Bailly and Karl Schwede's answers (10 years ago!), the pushout of a closed immersion $Z\to X$ along an affine morphism $Z\to Y$ always exists in the category of algebraic spaces. This is a special case of Theorem 1.8 of https://arxiv.org/abs/2205.08623.<|endoftext|>
-TITLE: Complex analytic space with no (positive dim.) subscheme ?
-QUESTION [7 upvotes]: Is there an example of a complex analytic space $X$ that doesn't have any (not necessarily open or closed) positive dimensional subspace $Y$ which is analytically isomorphic to (the complex analytic space associated to) a scheme?
-
-Edit: after D.Arapura's comment, we require $X$ to have dimension $>1$.
-If I remember correctly, there are non "Abelian" complex tori $X=\mathbb{C}^n/\mathrm{Lattice}\;\;$ that do not have any positive dimensional analytic subvariety. Can a counterexample be derived from this?
-Also, any complex algebraic space has an open subspace which is a scheme. So the counterexample (if it exists) must be searched outside algebraic spaces.
-
-What if the question is modified by requiring that $X$ has no $Y$ that is locally closed in the analytic Zariski topology (where opens are complements of analytic subspaces)?
-
-REPLY [2 votes]: Take a complex 2-torus $X$ without curves, and hence, without non-constant meromorphic functions (see e.g. Shafarevich, Basic algebraic geometry, chapter VIII, \S 1, example 2). The only locally closed subsets of $X$ will be $X$ itself, $\varnothing$, finite subsets and the complements of finite subsets. $X$ minus a finite subset can't be algebraic: if it were, it would have a non-constant meromorphic function, and then so would $X$.
-[upd: locally closed here means locally closed in the analytic Zariski topology, so this answers the second question and not the first.]<|endoftext|>
-TITLE: Short five lemma in Banach spaces
-QUESTION [5 upvotes]: Denote by $\mathbf{Ban}$ the category of Banach spaces and bounded linear maps and by $\mathbf{Banc}$ the subcategory of Banach spaces and linear contractions. The isomorphisms of $\mathbf{Ban}$ are the bounded linear bijections (open mapping theorem) and in $\mathbf{Banc}$ they are the isometric (linear) isomorphisms (easy computation). $\mathbf{Ban}$ is additive and has all finite limits and finite colimits, so we can speak of (short) exact sequences.
-proposition: the short five lemma holds in $\mathbf{Ban}$.
-proof: courtesy of the open mapping theorem, the diagram-chase proof for abelian groups transfers verbatim.
-Now for my question:
-Q: does the five short lemma also hold in $\mathbf{Banc}$?
-Since $\mathbf{Banc}$ is not additive, a few words about the definitions. The crucial thing to notice is that kernels and cokernels in $\mathbf{Ban}$ are also kernels and cokernels in $\mathbf{Banc}$. For the sake of illustration, take the case of cokernels. In $\mathbf{Ban}$ they are given by the quotient map $\pi:B\to B/M$ with $M$ a closed subspace of $B$. If $T$ is a bounded linear map whose kernel contains $M$, then it factors uniquely through $\pi$ and the norm of the factorization equals the norm of $T$. But this is precisely what is needed for $\pi$ to be a cokernel in $\mathbf{Banc}$.
-So now, we have two short exact sequences connected by maps $g$, $f$ and $h$ with $g$ and $h$ isometric isomorphisms and $f$ a linear contraction. The short five lemma holds if $f$ is an isometric isomorphism -- is there a way to draw diagrams? In any case, just look at short five lemma.
-Start by noticing that by the above proposition $f$ must be a bijection and thus it has an inverse. What we need to prove is that this inverse is contractive. Alternatively, since we already know that $f$ is injective, the following easy result offers another route.
-proposition: a map $f$ is a quotient iff it is surjective on open unit balls iff it is dense on unit balls.
-Applying the above and chasing down an element of the open unit ball of the codomain of $f:B\to B'$, we get is that there is a $b$ in the open unit ball of $B$ and an $m$ in the kernel of the quotient in the top short exact sequence such that $f(b + m) = b'$. But of course, this does not get us very far.
-At this point, I am beginning to suspect that the lemma does not hold, but I have been unable to rig a counterexample. The hypothesis is very strong and rules out the "usual" gang of suspects.
-regards,
-G. Rodrigues
-
-REPLY [10 votes]: How about considering $\mathbb R^2$ with $l^1$ and $l^2$ norms. The restriction to the $x$-axis is an isometry. The quotient onto the $y$-axis, also an isometry. But not an isometry on the whole space.<|endoftext|>
-TITLE: Numerical Differentiation. What is the best method?
-QUESTION [10 upvotes]: What is the best method for 1D numeric differentiation? Something as glorious as Gaussian quadrature for numeric integration.
-Maybe differential quadrature is such a method? What is its accuracy?
-I'm well aware that it is really easy to have symbolic differentiation in the program (automatic differentiation or truly symbolic algorithm). However to use such methods it is necessary to rewrite all functions to be differentiated. Thus one can't differentiate functions imported from libraries.
-I need differentiation almost with the machine precision.
-
-REPLY [3 votes]: Complex step differentiation (CSD) is well known as an efficient numerical differentiation method:
-$$f^\prime(x)=\mathrm{Im}\frac{f(x+\mathrm{i}h)}{h}+O(h^2),\quad\mathrm{i}:=\sqrt{-1}.$$
-This method requires the function to be analytic (differentiable as a complex function).
-We cannot apply this method to higher order derivatives, but since this method does not use differences it is useful to avoid subtractive cancellation (loss of significance caused when we try to subtract similar numbers). For the derivation and implementations, check the article on SIAM News and the post on Mathworks Blog.
-A noteworthy alternative to CSD is the approach by dual numbers. Dual numbers are defined by:
-$$x=a+b\epsilon,\quad a,b\in\mathbb{R},\quad\epsilon^2=0.$$
-By using Taylor expansion and dual numbers, we obtain
-$$f(a+b\epsilon)=f(a)+bf^\prime(a)\epsilon\quad(\because\epsilon^2=0).$$
-Hence we can obtain the derivative by dual numbers.<|endoftext|>
-TITLE: Looking for an appealing counterexample in probability
-QUESTION [17 upvotes]: There is a commonly-encountered-but-wrong rule of thumb that says something like
-
-If a probability distribution is positively skewed, its mean is greater than its median.
-
-(You sometimes also see it phrased in terms of mean and mode). I'm looking for a nice counterexample to this rule. What do I mean by "nice"? Hard to say exactly, of course, but something like:
-
-It must be continuously differentiable.
-It must have only one maximum and at most two points of inflection.
-Its graph must be "obviously" positively skewed and the mean must be "obviously" less than the median.
-
-I'm aware of the Weibull distribution, which can satisfy (1) and (2) - but for me the graph appears too close to being unskewed and the mean appears too close to the median for it to be an intuitively obvious counterexample. So I'm looking for something where the properties are more easily visible.
-Alternatively: if such a distribution did exist it seems to me that it would be quite well-known. So it seems plausible that the rule of thumb might be almost true, in the sense that there might exist a true statement of the form
-
-If a probability distribution is positively skewed, its mean can be at most __ less than its median.
-
-I'm also very interested to see answers of this form.
-
-REPLY [12 votes]: You are looking for random variables $X$ of median $m(X)$ such that
-$$
-E((X-E(X))^3)>0\quad\mbox{and}\quad E(X)< m(X).
-$$
-Assume there exists two nonnegative random variables $Y$ and $Z$ and an independent Bernoulli sign $B=\pm1$ with $E(B)=0$ such that $X=Y$ on $[B=1]$ and $X=-Z$ on $[B=-1]$.
-Then $m(X)=0$ and $E(X)=\frac12(E(Y)-E(Z))$. Furthermore, assuming $E(Y)< E(Z)$, one can prove that
-$$
-E(Y^3)>E(Z^3)\quad\implies\quad E((X-E(X))^3)>0.
-$$
-To prove this implication, expand $(X-E(X))^3$ using $Y$ and $Z$ to see that $E((X-E(X))^3)>0$ if and only if
-$$
-2(E(Y^3)-E(Z^3))+2(E(Z)^3-E(Y)^3)+3(E(Z)-E(Y))(\mbox{var}(Y)+\mbox{var}(Z))>0.
-$$
-If $E(Z)> E(Y)$, the second and the third terms are positive, hence if the first term is positive as well, the whole LHS is positive. Thus the implication holds.
-Finally, every couple $(Y,Z)$ such that
-$$
-E(Y)< E(Z),\qquad E(Y^3)>E(Z^3)
-$$
-is a solution. Note that every $X$ such that $m(X)=0$ can be written as above.
-First example If $Y$ is distributed like $y$ times a reduced exponential and $Z$ is distributed like $z\sqrt{\pi/2}$ times the absolute value of a reduced normal, then $E(Y)=y$, $E(Y^3)=6y^3$, $E(Z)=z$ and $E(Z^3)=2z^3$, hence every couple of parameters $(y,z)$ such that $y< z< \sqrt[3]{3}y$ yields a solution.
-Second example If $Y$ is distributed like $y$ times a reduced exponential and $Z$ is distributed like $z$ times the sum of two independent reduced exponentials, then $E(Y)=y$, $E(Y^3)=6y^3$, $E(Z)=2z$ and $E(Z^3)=24z^3$, hence every couple of parameters $(y,z)$ such that $\sqrt[3]{4}z< y< 2z$ yields a solution.<|endoftext|>
-TITLE: metaplectic group does not split
-QUESTION [13 upvotes]: I'm trying to understand the Weil representation and hope there are some experts around who can set me straight. Let $F$ be a non-Archimedean local field (I don't mind assuming that the characteristic of $F$ is not $2$ if it simplifies things; also the situation over $\mathbb{R}$ is similar but obviously I need a proof which does not rely on any covering space theory.) and fix a nontrivial continuous additive character $\psi : F \to \mathbb{C}^{\times}$. Then by the Stone-von Neumann theorem there is a unique up to isomorphism irreducible smooth representation of the Heisenberg group with central character $\psi$, and one sees the existence of a projective representation of the symplectic group in this space by the uniqueness part of that theorem.
-So my question is: why does this not lift to an ordinary representation? All the articles I have read refer this claim to Weil's original paper, which is quite long, and my French is not so good. I think one can see this from the fact that the metaplectic group (I'm talking about the two-sheeted cover) is not a trivial extension of the symplectic group by using the fact that the latter group is perfect. So if one constructs the metaplectic group via Maslov cocycles, one needs to show a certain cocycle is not a coboundary. But how?
-Thanks for the help. I hope my question is clear enough.
-
-REPLY [2 votes]: I would like the credit to go to Peter Woit for suggesting Section I.6 of Stephen Kudla's "Notes on the Local Theta Correspondence," which contains a nice proof, but he posted this only as a comment and the bounty ends today.
-The text of Woit's comment is
-
-There's a relatively straight-forward argument in Section I.6 of Stephen Kudla's "Notes on the Local Theta Correspondence", available at www.math.toronto.edu/~skudla/castle.pdf This may just be a restatement of the Rao argument. As mentioned elsewhere, it comes down to invoking non-triviality of the Hilbert symbol<|endoftext|>
-TITLE: On the Existence of Certain Fourier Series
-QUESTION [5 upvotes]: Is there an $f\in L^{1}(T)$ whose partial sums of Fourier series $S_{n}(f)$ satisfies $\|S_{n}(f)\|_{L^{1}(T)} \rightarrow \|f\|_{L^{1}(T)}$ but $S_{n}(f)$ fails to converge to $f$ in $L^1$-norm ?
-
-REPLY [7 votes]: I was hesitating for a while whether to answer or to vote to close and to refer the OP to AoPS, but, since the question has been upvoted, here goes.
-Suppose that $f_k\in L^1$ converges to $f\in L^1$ in the sense of distributions and in measure. Suppose also that $\|f_k\|_1\to \|f\|_1$. Then $f_k\to f$ in $L^1$.
-Indeed, let $g$ be a bounded by $1$ infinitely smooth function such that $\int fg>\|f\|_1-\delta$. Then $\int f_kg > \|f\|_1-2\delta$ for large $k$. Now, $\int_{\{|f_k-f|<\delta\}} f_kg\ge \int_{\{|f_k-f|<\delta\}} fg-\delta\ge \int fg-2\delta\ge \|f\|_1-3\delta$ for large $k$ because the integral of a fixed $L^1$ function $fg$ over a set of small measure is small.
-So, $\int|f_k|\ge \int_{\{|f_k-f|\ge\delta\}} |f_k|+\|f\|_1-3\delta$ whence the first integral is at most $4\delta$ for large $k$. Thus
-$$
-\int |f_k-k|\le \int_{\{|f_k-f|\ge \delta\}} (|f_k|+|f|)+\delta\le 6\delta
-$$
-for large $k$.
-To apply this to $f_k=S_kf$, one only needs to check the convergence in measure. But it immediately follows from the weak type 1-1 bound for $S_k$ (applied to the difference of $f$ and a trigonometric polynomial approximating $f$ in $L^1$, of course).<|endoftext|>
-TITLE: PNT for number fields.
-QUESTION [5 upvotes]: Hi there,
-Here is a part of the book of Murty "an introduction to Sieve methods and applications"
-page 36
-"At this point we invoke some algebraic number theory let $K=Q(\theta)$ where $\theta$ is the solution of the polynomial $f(x)$. The ring of integers $O_{k}$ of K is a Dedekind Domain. it's a classical theorem of Dedekind that for all but finitely many primes $\delta_{f}(p)$ is the number of prime ideals $p$ of $O_{k}$ such that the norm $N_{K/Q}(p)=p$"
-Note: $\delta_{f}(p)$:= number of solutions of $f(x)$ modulo $p$
-Question: What can be said about the size of the set of primes breaking this rule?
-In my opinion, might they be those dividing $disc(f)$?
-Yildo
-
-REPLY [3 votes]: This statement is true for all $p$ not dividing $disc(f)$, as you say. Moreover you can write down exactly what the primes dividing $p$ are: they're the ideals $(p, g(\theta))$ for each irreducible factor $g$ of $f$. The norm of such a prime is the degree of $g$, so the number of degree 1 primes is the number of roots of $f$ mod $p$.<|endoftext|>
-TITLE: Natural transformations as categorical homotopies
-QUESTION [59 upvotes]: Every text book I've ever read about Category Theory gives the definition of natural transformation as a collection of morphisms which make the well known diagrams commute.
-There is another possible definition of natural transformation, which appears to be a categorification of homotopy:
-
-given two functors $\mathcal F,\mathcal G \colon \mathcal C \to \mathcal D$ a natural transformation is a functor $\varphi \colon \mathcal C \times 2 \to \mathcal D$, where $2$ is the arrow category $0 \to 1$, such that $\varphi(-,0)=\mathcal F$ and $\varphi(-,1)=\mathcal G$.
-
-My question is:
-
-why doesn't anybody use this definition of natural transformation which seems to be more "natural" (at least for me)?
-
-(Edit:) It seems that many people use this definition of natural transformation. This arises the following question:
-
-Is there any introductory textbook (or lecture) on category theory that introduces natural transformation in this "homotopical" way rather then the classical one?
-
-(Edit2:) Some days ago I've read a post in nlab about $k$-transfor. In particular I have been interested by the discussion in the said post, because it seems to prove that the homotopical definition of natural transformation should be the right one (or at least a slight modification of it). On the other end this definition have always seemed to be the most natural one, because historically category theory develop in the context of algebraic topology, so now I've a new question:
-
-Does anyone know the logical process that took Mac Lane and Eilenberg to give their (classical) definition of natural transformation?
-Here I'm interested in the topological/algebraic motivation that move those great mathematicians to such definition rather the other one.
-
-REPLY [4 votes]: Following the previous indication of Professor Brown I want to add another possible way to see natural transformation which is a generalization of the previous definition.
-
-Given categories $\mathcal C$ and $\mathcal D$ and two functors between them $\mathcal F,\mathcal G \colon \mathcal C \to \mathcal D$ then a natural transformation $\tau$ can be defined as a functor $\tau \colon \mathcal C \to (\mathcal F \downarrow \mathcal G)$ which arrow components are the diagonal functions, sending each arrow $f \in \mathcal C(c,c')$, with $c,c' \in \mathcal C$ to $(f,f) \in (\mathcal F \downarrow \mathcal G)(\tau(c),\tau(c'))$.
-
-Edit: I think the definition of natural transformation proposed by professor Brown probably can be even a more natural than the one proposed in the question.
-I think that more details are worthed.
-The key ingredient for that definition is the concept of arrow category of a given category $\mathbf D$: such category have morphism of $\mathbf D$ as objects and commutative square as morphisms.
-This category come equipped with two functors $\mathbf {source}, \mathbf{target} \colon \text{Arr}(\mathbf D) \to \mathbf D$ such that for each object (i.e. a morphisms of $\mathbf D$) $f \colon d \to d'$ we have
-$$\mathbf{source}(f)=d$$
-$$\mathbf{target}(f)=d'$$
-while for each $f \in \mathbf D(x,x')$, $g \in \mathbf D(y,y')$ and a morphism $\alpha \in \text{Arr}(\mathbf D)(f,g)$ (i.e. a quadruple $\langle f,g, \alpha_0,\alpha_1\rangle$ where $\alpha_0 \in \mathbf D(x,y)$ and $\alpha_1 \in \mathbf D(x',y')$ such that $\alpha_1 \circ f = g \circ \alpha_0$) we have
-$$\mathbf{source}(\alpha)=\alpha_0$$
-$$\mathbf{target}(\alpha)=\alpha_1$$
-it's easy to prove that these data give two functors (which gives to $\text{Arr}(\mathbf D)$ the structure of a graph internal to $\mathbf{Cat}$).
-Now let's take a look to this new definition of natural transformation:
-
-A natural transformation $\tau$ between two functors $F,G \colon \mathbf C \to \mathbf D$ is a functor $\tau \colon \mathbf C \to \text{Arr}(\mathbf D)$ such that $\mathbf{source} \circ \tau = F$ and $\mathbf{target}\circ \tau = G$.
-
-A functor of this kind associate to every object $c \in \mathbf C$ a morphism $\tau_c \colon F(c) \to G(c)$ in $\mathbf D$, while to every $f \in \mathbf C(c,c')$ it gives the commutative triangle expressing the equality
-$$\tau_{c'} \circ F(f)=\tau_{c'} \circ \mathbf {source}(\tau_f)=\mathbf {target}(\tau_f) \circ \tau_c = G(f) \circ \tau_c$$
-certifying the naturality (in the ordinary sense) of the $\tau_c$.
-This definition reminds the notion of homotopy between maps $f,g \colon X \to Y$ as map of kind $X \to Y^I$ (i.e. an homotopy as a (continuous) family of path of $Y$).
-That's not all, indeed we can reiterate the construction of the arrow category obtaining what I think is called a cubical set
-$$\mathbf D \leftarrow \text{Arr}(\mathbf D) \leftarrow \text{Arr}^2(\mathbf D)\leftarrow \dots $$
-where each arrow should be thought as the pair of functors $\mathbf{source}_{n+1},\mathbf{target}_{n+1} \colon \text{Arr}^{n+1}(\mathbf D) \to \text{Arr}^n (\mathbf D)$.
-In this way we can associate to each category a cubical set. There's also a natural way to associate to every functor a (degree 0) mapping of cubical sets.
-If we consider natural transformation as maps from a category to an arrow category then this correspondence associate to each natural transformation a degree 1 map between such cubical sets (by degree one I mean that the induced map send every object of $\text{Arr}^n(\mathbf C)$ in an object of $\text{Arr}^{n+1}(\mathbf D)$).
-I've found really beautiful this construction because it shows an analogy between categories-functors-natural transformation and complexes-map of complexes-complexes homotopies.<|endoftext|>
-TITLE: Simplest examples of rings that are not isomorphic to their opposites
-QUESTION [36 upvotes]: What are the simplest examples of
-rings that are not isomorphic to their
-opposite rings? Is there a science to constructing them?
-
-The only simple example known to me:
-In Jacobson's Basic Algebra (vol. 1), Section 2.8, there is an exercise that goes as follows:
-Let $u=\begin{pmatrix} 0 & 1 & 0 \\ 0 & 0 & 1\\ 0 & 0 & 0 \end{pmatrix}\in M_3(\mathbf Q)$ and let $x=\begin{pmatrix} u & 0 \\ 0 & u^2 \end{pmatrix}$,
-$y=\begin{pmatrix}0&1\\0&0\end{pmatrix}$, where $u$ is as indicated and $0$ and $1$ are zero and unit matrices in $M_3(\mathbf Q)$. Hence $x,y\in M_6(\mathbf Q)$. Jacobson gives hints to prove that the subring of $M_6(\mathbf Q)$ generated by $x$ and $y$ is not isomorphic to its opposite.
-Examples seem to be well-known to the operator algebras crowd:
-See for example the paper: "A Simple Separable C*-Algebra Not Isomorphic to Its Opposite Algebra" by N. Christopher Phillips, Proceedings of the American Mathematical Society
-Vol. 132, No. 10 (Oct., 2004), pp. 2997-3005.
-
-REPLY [2 votes]: Many examples are already given; here is another one, just for its own interest:
-Let $V$ be a vector space of infinite countable dimension over a countable field $K$.
-Let $E$ be the $K$-algebra of endomorphisms of $V$. I claim that $E$ is not isomorphic to its opposite (even as a ring, i.e., as $\mathbf{Z}$-algebra). Precisely:
-
-(1) for every $g\in E-\{0\}$, $gE=\{gf:f\in E\}$ is uncountable [of continuum cardinal]
-(2) there exists $f\in E-\{0\}$ such that $Ef=\{gf:g\in E\}$ is countable: [namely this holds iff $f$ has finite rank (otherwise it has continuum cardinal)]
-
-Let me justify the non-bracketed assertions. In (2) this holds because if $B$ is a finite subset of $E$ such that $f(B)$ spans $f(E)$, then every element of $Ef$ is determined by its restriction to $B$.
-In (1), just fix a line $L$ not in the kernel of $g$, and let $f$ range over the space $Y$ linear maps $V\to L$. Since the dual of $V$ has uncountable dimension, $Y$ has uncountable [continuum] dimension. And $f\mapsto gf$ is injective in restriction to $Y$.
-Maybe in this case $E$ and $E^{\mathrm{op}}$ are not elementary equivalent, but this would require another argument.<|endoftext|>
-TITLE: Integrating the multinomial over a hypercube
-QUESTION [6 upvotes]: I have come across an integral of the form
-$$\int_{b}^{a}\cdots\int_{b}^{a} \left( \sum_{i=1}^{n}x_i\right)^mdx_1d x_2\dots dx_n.$$
-I have a solution that makes use of the partition function, but I feel there should be a much nicer solution and I'm sure this has been looked at before. Does anybody know a reference?
-Motivation:
-This integral has appeared whilst trying to compute moments of the Voronoi cell of the lattice $A_n$ (see page 462 of Sphere Packings, Lattices and Groups)
-
-REPLY [11 votes]: We want the coefficient of $t^m/m!$ in
- $$ \int_b^a\cdots \int_b^a e^{(x_1+\cdots+x_n)t}dx_1\dots dx_n =
- \left(\frac{e^{at}-e^{bt}}{t}\right)^n. $$
-Expanding by the binomial theorem and then taking the coefficient of $t^m/m!$ from each term will give a formula.<|endoftext|>
-TITLE: Characterization of infinite paths in graphs
-QUESTION [6 upvotes]: First an introduction.
-A directed graph we all know what is, and a graph is serial whenever
-every vertex has a successor. I do not consider the empty graph. A
-pair $(\mathcal{G},s)$ is called a rooted graph when $s \in
-\mathcal{G}$ and $\mathcal{G}$ is a directed serial graph.
-Given a rooted graph $(\mathcal{G},s)$, a $(\mathcal{G},s)$-path is a
-function $\lambda: \mathbb{N} \rightarrow V(\mathcal{G})$ such that
-$\lambda(0) = s$ and for all $i \in \mathbb{N}$,
-$(\lambda(i),\lambda(i+1)) \in E(\mathcal{G})$. Given any rooted graph
-$(\mathcal{G},s)$, we define the set $N(\mathcal{G},s)$ as follows:
-$$N(\mathcal{G},s) = \{ X \subseteq V(\mathcal{G}) \mid \text{ exists
-a $(\mathcal{G},s)$-path $\lambda$ s.t. for all $i \in \mathbb{N}$, }
-\lambda(i) \in X \}$$
-My question is, for what sets $N$ does there exist a rooted directed
-graph $(\mathcal{G},s)$ such that $N = N(\mathcal{G},s)$?
-I am looking for a way of describing (possibly infinite) directed
-serial graphs by giving a set of sets of vertices, each of which
-corresponds to an infinite path starting in a specific start vertex
-$s$. I call these sets neighbourhood sets (a slightly unfortunately
-name in graphs, I agree, but it comes from modal logic and its
-neighbourhood semantics, which I am applying this to).
-I would like to make some restrictions on a family of sets so that I
-can say when such a family of sets do has a corresponding graph,
-i.e. given a set of sets $N$, which properties must $N$ have in order
-to have a rooted directed graph $(\mathcal{G},s)$ such that $N =
-N(\mathcal{G},s)$.
-Define the non-monotonic core of $N$ as follows:
-$$N^{nc} = \{ X \in N \mid \not \exists Y \in N \text{ with } Y \subset X \}$$
-I have come up with a few trivial properties that are all necessary,
-but they are not sufficient, not even when restricted to finite
-graphs. The properties are as follows:
-
-Safety: The universe itself (i.e. the vertex set $V$) has to be contained in $N$
-Reflexivity: There is an element $s$ such that when $X \in N$, we have that $s \in X$
-Upwards closed: If $X \in N$ and $X \subseteq Y \subseteq V$, $Y \in N$
-Countable case: If $X \in N^{nc}$, then $ |X| \leq \omega $
-If $X \in N^{nc}$ and $Y \in N^{nc}$ and $|X \cap Y| = |X \setminus Y| = |Y \setminus X| = \omega$, then there has to be at least another (or infinitely many) $Y \neq Z \in N^{nc}$ with $|X \cap Z| = |X \setminus Z| = |Z \setminus X| = \omega$
-
-Now, it is trivial to check that the $N(\mathcal{G},s)$ satisfies the
-above properties for any directed graph $\mathcal{G}$, but they are
-not sufficient. Consider the following set:
-$$N = \{ \{s,1,2,3\}, \{s,1,2,4\}, \{s,1,3,4\} ,\{s,2,3,4\},
-\{s,1,2,3,4\} \}$$
-I have proven (in a very brute force manner) that the above cannot
-have a corresponding graph, so I will not give the proof here.
-I am looking for the missing properties, and work that has been done
-in this area. I apologize in advance if I have not given enough
-explanation, I have been stuck in this problem too long now to see
-this clearly. If I should explain more, or give examples, please let
-me know, and I will.
-Edit: Added some cases I didn't want to explain, but realized later I should put in anyway, but they both deals with infinite graphs, and I'm first and foremost after managing the subproblem that is finite graphs.
-
-REPLY [2 votes]: (Not an answer, but too long for a comment)
-One way to look at what you're asking for is a theory in some appropriate language which axiomatizes that we have a serial directed rooted graph, and a collection of subsets of the graph which behaves like $N(\mathcal{G},s)$, where the part of the axiomatization that dictates the behaviour of $N(\mathcal{G},s)$ doesn't say anything about the graph relation on $\mathcal{G}$. I'll give an axiomatization where the part that talks about $N(\mathcal{G},s)$ does mention the graph relation, and I'll set it up in such a way that it'll seem unlikely that it can be redone without mentioning the graph relation. I'm not sure about this, it's just an idea, but it's too long for a comment.
-The Language
-Consider the language $(\bar{V}, \bar{N}, \bar{E}, \bar{s}, \bar{\epsilon})$ - two unary relation symbols, one binary relation symbol, one constant symbol, and another binary relation symbol, respectively. I'm going to set up a theory whose finite models will be precisely (sort of) those in which $(\bar{V}, \bar{E})$ is interpreted as a directed serial graph $\mathcal{G}$, $\bar{s}$ gets interpreted as a member $s$ of $\bar{V}$, $\bar{N}$ gets interpreted as $N(\mathcal{G},s)$, and $\bar{\epsilon}$ gets interpreted as the membership relation, a subset of $\bar{V} \times \bar{N}$. Setting up the right theory is easy, the only part that will require a little explanation is how we ensure $\bar{N}$ gets interpreted correctly.
-Axiomatizing $N(\mathcal{G},s)$ when we're allowed to mention the edge relation
-Consider a formula like:
-$$x_1 = \bar{s} \wedge x_1 \neq x_2 \wedge x_1 \neq x_3 \wedge x_2 \neq x_3$$
-$$\wedge \bar{E}(x_1,x_2) \wedge \bar{E}(x_2,x_1) \wedge \neg \bar{E}(x_1,x_3) \wedge \neg \bar{E}(x_3,x_1) \wedge \neg \bar{E}(x_2,x_3) \wedge \neg \bar{E}(x_3,x_2)$$
-It "says" we have three distinct $x_i$, the first one is $s$, and it tells you exactly which pairs stand in edge relation to one another and which don't. You can also tell by looking at it that it defines a set $\{ x_1, x_2, x_3\}$ which contains an infinite path starting at $s$, namely $x_1, x_2, x_1, x_2, \dots$.
-Let's define $\mathcal{R}$ to be the set of formulas in the language $(\bar{s}, \bar{E})$ of the form $\phi(x_1, \dots , x_n)$ which say that the $x_i$ are distinct, and which tell you precisely which pairs of the $x_i$ stand in $\bar{E}$ relation to one another, and which don't. Define $\mathcal{R}^+$ to be those formulas $\phi \in \mathcal{R}$ such that for any rooted directed graph $((V,E),s)$, we have that:
-$(V,s,E)\vDash \phi(v_1, \dots , v_n) \rightarrow \{v_1, \dots , v_n\} \in N((V,E),s)$.
-$\mathcal{R}^-$ will be those $\phi$ such that:
-$(V,s,E)\vDash \phi(v_1, \dots , v_n) \rightarrow \{v_1, \dots , v_n\} \not\in N((V,E),s)$.
-Alright, now here's our theory:
-
-(Rooted serial directed graph) $\bar{V}(\bar{s}) \wedge \forall x \bar{V}(x) \rightarrow [\neg \bar{E}(x,x) \wedge \exists y (\bar{V}(y) \wedge \bar{E}(x,y))]$
-(Schema for what goes in $N$) As $\phi(x_1, \dots ,x_n)$ varies over $\mathcal{R}^+$: $\forall \vec{x} \exists Y \forall x \left (\phi(\vec{x}) \rightarrow \left[ \bar{N}(Y) \wedge \bigwedge _i (x_i \bar{\epsilon} Y) \wedge \left (x \bar{\epsilon} Y \rightarrow \bigvee _i (x = x_i)\right )\right ]\right )$
-(Schema for what stays out of $N$) As $\phi(x_1, \dots ,x_n)$ varies over $\mathcal{R}^-$: $\forall \vec{x} \forall Y \exists x \left (\phi(\vec{x}) \rightarrow \neg \left[ \bar{N}(Y) \wedge \bigwedge _i (x_i \bar{\epsilon} Y) \wedge \left (x \bar{\epsilon} Y \rightarrow \bigvee _i (x = x_i)\right )\right ]\right )$
-
-Checking this axiomatization works
-It's clear that if $\mathcal{G} = (V,E)$ is a finite directed serial graph and $s \in V$, then $(V,N(\mathcal{G},s),E,s,\in)$ is a model of this theory. Conversely, if we have a finite model $(V,N_0,E,s,\epsilon)$ of this theory, then $(V,E)$ forms a directed serial graph with $s \in V$. Now let $N_1$ consist of those $Y \in N_0$ such that $\forall x (x \epsilon Y \rightarrow V(x))$. I claim that:
-$N((V,E),s) = \{ \{x : x \epsilon Y\} : Y \in N_1\}$
-But I won't prove this.
-A remark about some "unnaturalness"
-It should seem like I could have done things differently so that things looked more natural and the above claim could be proved more easily. The way I've written 2 and 3 are not the most natural, but I've done it for a reason. 2 says that if $\vec{x}$ is a tuple which is sure to contain an infinite path starting at $s$, then there's a member of $N$ consisting precisely of the members of $\vec{x}$. 3 says that if $\vec{x}$ is sure to not contain an infinite path starting at $x$, then there's nothing in $N$ consisting precisely of the members of $\vec{x}$. The formulas in 2 and 3 are in prenex normal form, where the matrix is a conditional where the left side only involves the symbols $\bar{E}$ and $\bar{s}$, and the right side only $\bar{N}$ and $\bar{\epsilon}$. Moreover, the antecedents have variables $\vec{x}$ and the consequents have variables $\vec{x}, Y, x$.
-The point
-You want a theory equivalent to axiom 1 and schemas 2 and 3 above, but you want to replace 2 and 3 with (probably finitely many) axioms which don't mention the symbol $\bar{E}$. I feel like there ought to be some interpolation-type theorem (along the lines of Craig Interpolation or Lyndon Interpolation) which says this can't happen, i.e. something which says that a theory in which each axiom mention either only $\bar{V}, \bar{E}, \bar{s}$ or only $\bar{V}, \bar{N}, \bar{s}, \bar{\epsilon}$ can't prove a theory which has axioms like those in schemas 2 or 3.<|endoftext|>
-TITLE: Dynamics of a random "quadratic" directed graph
-QUESTION [7 upvotes]: Let G be a directed graph on N vertices chosen at random, conditional on the requirement that the out-degree of each vertex is 1 and the in-degree of each vertex is either 0 or 2. The "periodic" points of G are those contained in a cycle. What do we know about the statistics of G? For instance, what is the mean number of periodic points, and how do the cycle lengths look?
-By comparison, a random directed graph with all out-degrees 1 (which is to say, the graph of a random function from vertices to vertices) has on order of sqrt(N) periodic points on average.
-(Motivation: the graph of a quadratic rational function f acting on P^1(F_q) looks like this, and I'm wondering what the "expected" dynamics are.)
-
-REPLY [3 votes]: Let $G$ be a directed graph on $N$ vertices such that the out-degree of each vertex is 1 and the in-degree is either 0 or $n$. Letting $N=nt$, there are $t$ vertices with in-degree $n$ and $(n-1)t$ vertices with in-degree 0. Assuming the vertices are labeled, the number of such graphs is
-$$
-\binom{nt}{t}\frac{(nt)!}{(n!)^t}.
-$$
-For $1\leq m\leq t$, the number of such graphs with a fixed $m$-cycle is
-$$
-\binom{nt-m}{t-m}\frac{(nt-m)!}{((n-1)!)^m(n!)^{t-m}},
-$$
-so the number of $m$-periodic points amongst all such graphs is
-$$
-m\cdot\binom{nt-m}{t-m}\frac{(nt-m)!}{((n-1)!)^m(n!)^{t-m}}\cdot\frac{(nt)!}{m(nt-m)!}=\binom{nt-m}{t-m}\frac{(nt)!}{((n-1)!)^m(n!)^{t-m}}.
-$$
-Summing over $1\leq m\leq t$, the average we want is
-$$
-\binom{nt}{t}^{-1}\sum_{m=1}^{t}{n^m\binom{nt-m}{t-m}}=-1+\binom{nt}{t}^{-1}\sum_{m=0}^{t}{n^m\binom{nt-m}{t-m}}.
-$$
-When $n=1$, the average is $t$ (since all points would be periodic). When $n=2$, as in the question, the sum is
-$$
--1+\frac{4^t}{\binom{nt}{t}}\sim-1+\sqrt{\frac{\pi}{2}}\sqrt{N},
-$$
-so the number of periodic points does indeed look random. When $n>2$, I haven't found the identity I need yet.<|endoftext|>
-TITLE: Blocking visibility with cylinders
-QUESTION [16 upvotes]: Suppose you have a supply of infinite-length, opaque, unit-radius cylinders,
-and you would like to block all visibility from a point
-$p \in \mathbb{R}^3$ to infinity with as few cylinders as possible.
-(The cylinders are infinite length in both directions.)
-The cylinders may touch but not interpenetrate, and they
-should be disjoint from $p$,
-leaving a small ball around $p$ empty.
-(Another variation would insist that cylinders be pairwise disjoint,
-i.e., not touching one another.)
-A collection of parallel cylinders arranged to form a "fence" around
-$p$ do not suffice, leaving two line-of-sight $\pm$ rays to infinity.
-Perhaps a grid of cylinders in the pattern illustrated left below
-suffice, but at least if there are not many cylinders, there is
-a view from an interior point to infinity (right below).
-
-I feel like I am missing a simple construction that would
-obviously block all rays from $p$.
-Perhaps crossing the cylinders like the poles of a tipi (teepee)
-could help, but it seems this would at best lead to inefficient
-blockage.
-Suggestions welcome—Thanks!
-Addendum1.
-Perhaps if the weaving above is rendered irregular by displacing the cylinders slightly by different
-amounts, so that cracks do not align, then a sufficient portion of the weaving will block all visibility.
-Here (left below) is the start of Gerhard's first suggested construction (a portion of the weaving above), which I don't see how to complete. But perhaps
-seeing this depiction will aid intuition.
-
-
-
-
-Addendum2.
-To the right above I added (three-quarters of) a forest along the lines (but not exactly as)
-Yaakov suggested.
-
-REPLY [5 votes]: Here is one construction. On the horizontal xy plane place a forest of vertical cylinders of radius r<1/2 (or =1/2 if we allow contacts) centered at each point in $(\mathbb{Z} \backslash{\lbrace0\rbrace})\times(\mathbb{Z}\backslash{\lbrace0\rbrace})$; moreover place 2 similar cylinders parallel to x centered at (y=+/-1, z=0), 2 more parallel to y centered at (x=0, z=+/-1) and the last 2 parallel to z and centered at (x=+/-1, y=0). Then (0,0,0) is blocked by the forest in all directions except those in the xz and yz planes, which are blocked by the other 6 cylinders. The forest clearly does not need to be infinite and it should be easy to find an upper limit on its size.
-${\bf UPDATE}$ As pointed out by Mark in a comment, the forest should be based on $(\mathbb{Z} \backslash{\lbrace0\rbrace})\times(\mathbb{Z}\backslash{\lbrace-1,0,1\rbrace})$.<|endoftext|>
-TITLE: Does completion commute with localization?
-QUESTION [8 upvotes]: Suppose $A$ is a Noetherian (not necessarily local) ring and $\mathfrak{m}\subset A$ a maximal ideal. Then is it true that $$\hat{A}_{\hat{\mathfrak{m}}}=\widehat{A _{\mathfrak{m}}},$$ where hats denote completion and subscripts denote localization? If one uses superscripts to denote completion it would be
-$$(A^{\mathfrak{m}})_{\mathfrak{m^{\mathfrak{m}}}}=(A _{\mathfrak{m}})^{\mathfrak{m} _{\mathfrak{m}}}.$$
-
-REPLY [19 votes]: It is true. $(\widehat{A}, \widehat{m})$ is a Noetherian local ring so your left hand side could be simplified replacing it by $\widehat{A}$. Now let's use the definitions: $\widehat{A} = \varprojlim A/\mathfrak{m}^n$, whereas $\widehat{A_{\mathfrak{m}}} = \varprojlim A_{m}/(\mathfrak{m}A_{\mathfrak{m}})^n$. The desired equality is the result of localization being exact so that $A_{\mathfrak{m}}/(\mathfrak{m}A_{\mathfrak{m}})^n = (A/ \mathfrak{m}^n)_{\mathfrak{m}}$ and the fact that in $A/\mathfrak{m^n}$ everything outside the maximal ideal is already invertible, so that $(A/\mathfrak{m^n})_{\mathfrak{m}} = A/\mathfrak{m}^n$.<|endoftext|>
-TITLE: When is this map completely positive?
-QUESTION [6 upvotes]: Consider the complex $n$-by-$n$ matrices $M_n$.
-Suppose that $A_i$, for $i=1,\ldots,n^2$, satisfy $\mathrm{Tr}(A_i^*
-A_j)=\delta_{ij}$, so that together they form an orthonormal basis for
-$M_n$. Define a linear map $T \colon M_n \to M_n \otimes M_n$ by $T(A_i) = A_i \otimes A_i$.
-
-Question: when is $T$ completely positive?
-
-For example, if $A_i$ are the matrices with a single entry one and the rest zeroes in some fixed basis of $\mathbb{C}^n$, then $T$ is completely positive. In fact, I think these might be the only examples. If $T$ is completely positive, then the following are equivalent to $A_i$ being matrix units as in the above example:
-
-each $A_i$ has rank one;
-each positive semidefinite $A_i$ has trace one;
-the set $\{0,A_1,\ldots,A_{n^2}\}$ is closed under multiplication;
-$T(1)$ is idempotent;
-$T^*(1) \leq 1$;
-$T$ preserves trace.
-
-These are sufficient conditions, but proving they are sufficient doesn't use $\mathrm{Tr}(A_i^* A_j)=\delta_{ij}$ at all. Are they necessary?
-
-REPLY [2 votes]: I think I finally have the answer to this question (some 7 years later, and 3 years after first hearing of the problem myself, but never mind). TL;DR: the matrices with a single entry one and the rest zeroes in some fixed basis of $\mathbb{C}^n$ are indeed the only examples. A detailed proof (with diagrams!) will appear shortly on the arXiv, and I will link to it once it's ready, but I will summarise the gist here for posterity.
-A linear map $\delta:M_n \rightarrow M_n \otimes M_n$ defined by $\delta(A_{ij}) := A_{ij} \otimes A_{ij}$ for some orthonormal basis $(A_{ij})_{i,j=1,...,n}$ of $M_n$ is necessarily a special commutative $\dagger$-Frobenius algebra in the category $\operatorname{fHilb}$ of finite-dimensional Hilbert spaces and complex linear maps, with co-unit $\epsilon: M_n \rightarrow \mathbb{C}$ defined by setting $\epsilon(A_{ij}) := 1$ (together with their respective adjoints $\delta^\dagger$ and $\epsilon^\dagger$). If $\delta$ is a CP map, then so is $\epsilon$, and $(\delta,\epsilon,\delta^\dagger,\epsilon^\dagger)$ is also a special commutative $\dagger$-Frobenius algebra in the category $\operatorname{CPM}(\operatorname{fHilb})$ of finite-dimensional Hilbert spaces and completely positive maps.
-The point is that every isometric comonoid in $\operatorname{CPM}(\operatorname{fHilb})$, such as $(\delta,\epsilon)$ would be when $\delta$ is CP, must necessarily involve pure maps $\delta,\epsilon$. This is because of the purity principle: whenever for pure CP maps $\Psi_i$ and $F$ we have $\sum_i \Psi_i = F$, necessarily $\Psi_i = q_i F$ for all $i$ and some coefficients $q_i \in \mathbb{R}^+$ (not necessarily all non-zero). The broad strokes of the proof are as follows.
-
-The isometry condition $\delta^\dagger \circ \delta = id$ implies that $\delta$ must be $\mathbb{R}^+$-linear combination (not necessarily convex) $\delta = \sum_i q_i V_i$ of isometries $V_i$ with pairwise orthogonal ranges (i.e. $V_j^\dagger \circ V_i$ is the identity $id$ for $i=j$ and the zero map for $i\neq j$).
-The unit laws $(id \otimes \epsilon)\circ \delta = id = (\epsilon \otimes id)\circ \delta $ imply that $\epsilon$ must be pure.
-By post-composing the associativity law with $\epsilon$ in all three possible ways, an associativity law for the isometries can be derived.
-By post-composing the associativity law for isometries with $\epsilon$, all isometries $V_i$ are identified, and orthogonality of ranges proves that $\delta$ must in fact be pure.
-Because $\delta$ and $\epsilon$ are pure, i.e. they arise from the comultiplication and counit of a special commutative $\dagger$-Frobenius algebra of $\operatorname{fHilb}$. Those are exactly the examples given by taking matrices $A_{ij} := |i\rangle\langle j|$ with a single entry one and the rest zero in some fixed orthonormal basis $\big(|i\rangle\big)_{i=1,...,n}$ of $\mathbb{C}^n$.<|endoftext|>
-TITLE: Explicit formula for the trace of an unramified principal series representation of $GL(n,K)$, $K$ $p$-adic.
-QUESTION [18 upvotes]: Let $K$ be a non-arch local field (I'm only interested in the char 0 case), let $\mathbb{G}$ be a connected reductive group over $K$ and let $G=\mathbb{G}(K)$. If $V$ is a smooth irreducible complex representation of $G$ and if $H$ is the Hecke algebra of locally constant complex-valued functions on $G$ with compact support (fix a Haar measure on $G$ to make $H$ an algebra under convolution), then $V$ is naturally an $H$-module, and every $h\in H$ acts on $V$ via a finite rank operator and hence has a trace.
-It is my understanding that in this connected reductive situation, a theorem of Harish-Chandra says that this trace function $t:H\to\mathbb{C}$ can actually be expressed as
-$$t(h)=\int_G tr(g)h(g) dg$$
-for $tr:G\to\mathbb{C}$ an $L^1$ function, called the trace of $V$.
-If $V$ is finite-dimensional then $tr$ is the usual trace. However I realised earlier this week that I do not know one single explicit example of this function if $V$ is infinite-dimensional. I just spent 20 minutes trying to fathom out what I guessed was probably the simplest non-trivial example: if $G=GL(2,K)$ and $V$ is, say, an unramified principal series representation. I failed :-( I could compute the trace of $h$ for various explicit $h$ (typically supported in $GL(2,R)$, $R$ the integers of $K$) but this didn't seem to get me any closer to an actual formula: in particular, although I could figure out $t$ on various functions I couldn't figure out $tr$ on any elements of $G$. On the other hand I imagine that this sort of stuff is completely standard, if you know where to look.
-If $\mu_1$ and $\mu_2$ are unramified characters of $K^\times$ and $V$ is the associated principal series representation of $GL(2,K)$, then what is $tr(g)$ for $g$, say, a diagonal matrix? Or $g$ a unipotent matrix?
-[EDIT: Alexander Braverman points out that I have over-stated Harish-Chandra's result: $t$ is only locally $L^1$. Furthermore one has to be a little careful---more careful than I was at least---because $t$ is only defined via some integrals so one could change it on a set of measure zero---hence in some sense asking to evaluate $t$ at an explicit point makes no sense. However he, in his answer, shows how to make sense of my question anyway, as well as answering it.]
-[EDIT: Loren Spice points out that my paranthetical char 0 comment is actually an assumption in Harish-Chandra's result, and that apparently local integrability is still open in char $p$. I didn't make a very good job of stating H-C's theorem at all!]
-
-REPLY [2 votes]: I am not sure, if you are still interested in this, but here is the general computation:
-Let $\phi \in C_c^\infty (GL_n(F))$, and let $\pi$ be a super-cuspidal representation of a Levi subgroup $M$ of a parabolic $P$ with unipotent radical $N$, and let $\pi_0 = Ind_{P}^{GL_n(F)} \pi$ be the normalized induced representation (assume unitary, irreducible for safety). Let $K$ be compact open subgroup with $GL_n(F) = P K$.
-
-Define $ \phi^K (x) = \int\limits_K \phi(k^{-1}xk) d k,$
-Define $ A\phi^K(m) = \Delta_P(m)^{1/2} \int\limits_{N} \phi^K(mn)d n$ for $m \in M$
-Then we have that $A\phi^K \in C_c^\infty(M)$ and the formula
-$$ tr \pi_0(\phi) = tr \pi( A \phi^K).$$
-
-The same formula is also useful, if the representation is not irreducible (unitarizability is not really an issue, and admissibility follows from the Iwasawa decomposition), but one has to normalize and decompose according to the $K$-isotypes.
-So in your situation, you get a Fourier transform of $A \phi^K$. This in addition with Moshe Adrian answer computes all the irreducible, unitary principal series representation, at least in prinicple.<|endoftext|>
-TITLE: Area of intersection of a family of circles in the plane
-QUESTION [5 upvotes]: Suppose you are given a family F of circles in the plane such that each circle has radius 1. Let G be the family of circles with same centers as in family F but now each circle has radius $r$. Let A be area of union of circles in family F and let B be area of union of circles in family G. Then can we upper bound the number $\frac{B}{A}$ by a function $f(r)$ ?
-One trivial thing is that if the family F contains all disjoint circles then $\frac{B}{A}$ is at most $r^2$. But in general case the geometry is getting weird and complicated .
-
-REPLY [10 votes]: The problem is much simpler than the general Kneser-Poulsen case. Here is an elementary proof of the fact that $B/A\le r^2$ for all $r\ge 1$. To simplify technicalities, I assume that the family of circles is finite (the general case follows as a limit).
-Let $p_1,\dots,p_N$ be the centers of our circles. Let $V$ denote the union of the unit circles and $U$ the union of the circles of radius $r$. Divide $U$ into Voronoi regions $U_i$ of the points $p_i$. Namely, $U_i$ is the set of points $x\in U$ such that $p_i$ is nearest to $x$ among $p_1,\dots,p_N$. (For convenience, remove the points having more than one nearest center; this is a zero measure set.) Each $U_i$ is star-shaped w.r.t. the respective point $p_i$. Indeed, if $x\in U_i$, then $|p_ix|\le r$, hence the segment $[p_ix]$ is contained in $U$. And obviously $p_i$ is the nearest center for every point of this segment.
-Now apply the $(1/r)$-homothety centered at $p_i$ to each set $U_i$. The resulting sets $U_i'$ are disjoint and contained in $V$, hence
-$$
- S(V) \ge\sum S(U_i') = \frac1{r^2} S(U_i) = \frac1{r^2} S(U)
-$$
-where $S$ denotes the area. Thus $S(U) \le r^2 S(V)$, q.e.d.
-The same argument works in $\mathbb R^n$ (with $r^n$ in place of $r^2$), in Alexandrov spaces of nonnegative curvature, and in Riemannian manifolds of nonnegative Ricci curvature (just combine it with the proof of Bishop-Gromov inequality).<|endoftext|>
-TITLE: Which norms have rich isometry groups?
-QUESTION [19 upvotes]: Let $n \ge 2$ be some positive integer. Given a norm $p : \mathbb{R}^n \to \mathbb{R}$, one can inquire about the structure and properties of its isometry group, i.e. the group of all bijections $F:\mathbb{R}^{n}\to\mathbb{R}^{n}$ such that $p\left(v\right)=p\left(F\left(v\right)\right)$ for all $v \in \mathbb{R}^n$. By the Mazur-Ulam theorem, the isometries of $p$ are affine transformations, so the subgroup $Iso_{0}\left(p\right)$ of all isometries which fix the origin, consists of linear transformations, and is therefore a closed subgroup of $GL_n (\mathbb{R})$. Essentially, $Iso_{0}\left(p\right)$ is what we get after modding out the obvious distance-preserving functions (the translations), which have no relation to $p$ whatsoever.
-For some norms, $Iso_{0}\left(p\right)$ is a very small group. For instance, if $p$ is either the $\ell^\infty$ norm or the $\ell^1$ norm, then the group of $Iso_{0}\left(p\right)$ is finite, and this seems to be the case for all $\ell^q$ norms other than $q=2$. On the other hand, for the euclidean norm ($p = \ell^2$), $Iso_{0}\left(p\right)$ is quite a rich group: it is a Lie group of positive dimension (which increases significantly with $n$) - the orthogonal group $O(n)$.
-Obviously, a significant difference between this norm and other possible norms is that it arises from an inner product on $\mathbb{R}^n$. Since there is only one inner product structure on $\mathbb{R}^n$ (up to conjugacy) this essentially gives us just one example of a norm with a rich group of linear isometries. So my questions are:
-
-Are there any other norms (not induced by an inner product) on $\mathbb{R}^n$ which have a rich group of linear isometries? Here let me define "rich" as having positive dimension as a Lie subgroup of $GL_n (\mathbb{R})$ (every closed linear group is canonically a submanifold of $GL_n (\mathbb{R})$ compatible with the group structure).
-If not, why is the property of being induced from an inner product the "right" condition for having many symmetries? That is, from a geometric point of view, why does this property make the norm especially "symmetric" or "smooth"?
-
-REPLY [5 votes]: Sorry to join so late to the party, but I couldn't help noticing there is a missing class of rich matrix norms (which are not operator norms).
-These are the p-Schatten norms on $R^{n^2}$, which see only the singular values of a matrix.
-Its immediate to see that these norms are invariant under left and right multiplication by orthogonal matrices.
-Hence in particular their isometry groups contain copies of $O(n)$.
-So $\dim(Iso(\| \cdot \|_{p-Schatten})) \ge \frac{n(n-1)}{2}$
-Note that for $p \neq 2$ these norms are not induced by inner products.<|endoftext|>
-TITLE: extensions, abelian varieties, $\mathbb{G}_m$
-QUESTION [7 upvotes]: Question: why are there no non-trivial extensions of $\mathbb{G}_m$ by abelian varieties?
-Specifically, let $A$ be an abelian variety over a field $k$. Then it seems to be well known that any extension $$ 0 \to A \to G \to \mathbb{G}_m \to 0$$ in the category of group varieties over $k$ is split. For instance, this is mentioned in Deligne's Theorie de Hodge I page 426 just before Section 3. Chevalley's structure theorem of algebraic groups certainly implies this.
-Question: Is there a simple/direct proof?
-Note that there are non-trivial extensions of abelian varieties by tori (semiabelian varieties).
-Thanks.
-
-REPLY [20 votes]: Using the Kummer exact sequence
-$0\rightarrow\mu_n\rightarrow\mathbb{G}_m\rightarrow\mathbb{G}_m\rightarrow0$ we
-get a long exact sequence
-$$
-0\rightarrow\mathrm{Hom}(\mathbb{G}_m,A)\rightarrow\mathrm{Hom}(\mathbb{G}_m,A)
-\rightarrow\mathrm{Hom}(\mu_n,A)\rightarrow\mathrm{Ext}^1(\mathbb{G}_m,A).
-$$
-As $\mathrm{Hom}(\mathbb{G}_m,A)=0$ this gives an embedding
-$\mathrm{Hom}(\mu_n,A)\hookrightarrow\mathrm{Ext}^1(\mathbb{G}_m,A)$ and at
-least over an algebraically closed field in which $n$ is invertible we always
-have that $\mathrm{Hom}(\mu_n,A)$ is non-trivial and consequently so is
-$\mathrm{Ext}^1(\mathbb{G}_m,A)$.
-This does not contradict Deligne's claim as he seems to be (somewhat implicitly
-it must be admitted) speaking of extensions up to isogeny (in Principle 2.1 this
-is made more clear). In fact Chevalley's theorem implies that this is true:
-Consider a connected affine subgroup $G'\hookrightarrow G$ for which the
-quotient is an abelian variety. Then the kernel of the composite
-$G'\rightarrow\mathbb{G}_m$ is affine and embeds in $A$ and thus is finite. On
-the other hand $G'\rightarrow\mathbb{G}_m$ must be surjective as otherwise $G'$
-would be finite and hence trivial so $G$ would be an abelian variety which is
-not possible. The kernel of $G'=\mathbb{G}_m\rightarrow\mathbb{G}_m$ is then
-some $\mu_n$ which shows that the extension is in the image of
-$\mathrm{Hom}(\mu_n,A)\hookrightarrow\mathrm{Ext}^1(\mathbb{G}_m,A)$ and in
-particular is trivial up to isogeny.
-The final conclusion is that $\mathrm{Ext}^1(\mathbb{G}_m,A)=\mathrm{Hom}(\hat{\mathbb Z}(1),A)$.
-Addendum: I just realised that I didn't answer the actual question about
-whether there is a simpler proof (to the isogeny statement would be my
-interpretation). I doubt it (depending of course somewhat on your definition of
-simpler). A proof (arguably less simple) avoiding the use of Chevalley's theorem
-(but assuming to be on the safe side that the base field is algebraically closed
-of characteristic $0$) would be to prove that for some $n$ the pull back of the
-extension along the $n$'th power on $\mathbb{G}_m$ is trivial as an $A$-torsor
-because then the extension would be described by a $2$-cocycly
-$\mathbb{G}_m\times\mathbb{G}_m\rightarrow A$ and any map of that form is
-constant. That in turn will follow from the fact that for some $m$ the class of
-this extension as an $A$-torsor would be in the image of
-$H^1(\mathbb{G}_m,A[m])\rightarrow H^1(\mathbb{G}_m,A)$. This follows from the
-fact that $H^1(\mathbb{G}_m,A)$ is torsion and then the conclusion follows as
-any class of $H^1(\mathbb{G}_m,C)$ for any finite locally constant sheaf $C$ is
-killed by pulling back along some $n$'th power map.<|endoftext|>
-TITLE: Is there a theory about these kinds of recurrence equations? Is this a known formula?
-QUESTION [5 upvotes]: (New information at bottom)
-Hi.
-For a while, I've been toying around with solving recurrence equations of the form
-$$a_1 = r_{1,1}$$
-$$a_n = \sum_{m=1}^{n-1} r_{n,m} a_m$$
-What are these kind of recurrence equations called? Does one have any references for any general theory behind them?
-The goal is to try to find a non-recursive formula for the coefficients of what is called the "Schroder function" of $e^{uz} - 1$, which satisfies the functional equation
-$$\chi(e^{uz} - 1) = u \chi(z)$$.
-This function can be expressed as
-$$\chi(z) = \sum_{n=1}^{\infty} \chi_n z^n$$
-with
-$$\chi_1 = 1$$
-$$\chi_n = \sum_{m=1}^{n-1} \frac{u^{n-1}}{1 - u^{n-1}} \frac{m!}{n!} S(n, m) \chi_m$$
-a recurrence of the given form, where $S(n, m)$ is a Stirling number of the 2nd kind.
-I managed to find the following formula:
-$$a_n = r_{1,1} \sum_{\substack{1 = m_1 < m_2 < \cdots < m_k = n\\ 2 \le k \le n}}\ \prod_{j=2}^{k} r_{m_j, m_{j-1}},\ n > 1$$
-which sums over $2^{n-2}$ terms, namely, all subsets of the integer interval from 1 to $n$ that contain 1 and $n$. However, is there a way to simplify this for the case I gave, where $r_{n,m} = \frac{u^{n-1}}{1 - u^{n-1}} \frac{m!}{n!} S(n, m)$? I note that in cases like the Bernoulli numbers, these kind of recurrences have solutions expressible as nested sums or as products over many fewer terms than above (with a simple linear index). Is such a thing also possible here with the $r_{n,m}$ I just gave? If so, how? Also, is the above formula already known?
-ADDENDUM (19 Sep 2011): I later found out on a different group (the Usenet newsgroup sci.math) that such a recurrence may be called a "full-history recurrence", though that term seems a little more general than just referring to the specific kind of sum recurrence mentioned above, and googling it did not turn up much (much less the solution formula mentioned above! (and too bad you can't google math formulas!)), and much of what it did turn up seemed to have to do more with the recurrence in the context of algorithmic theory and computer science than with just pure maths. Is there a better name or something more useful I might try looking up?
-EDIT (25 Sep 2011): There is another form of this formula, for the indexing
-$$a_0 = \alpha$$,
-$$a_{n+1} = \sum_{m=0}^{n} r_{n,m} a_m$$.
-This version goes as
-$$a_n = \alpha \sum_{\substack{-1 = m_0 < m_1 < m_2 < \cdots < m_k = n-1\\ 1 \le k \le n}}\ \prod_{j=1}^{k} r_{m_j,m_{j-1}+1},\ n > 0$$.
-Does that ring a bell better? Notice how we can get, e.g. the Bernoulli numbers by setting $\alpha = 1$, $r_{n,m} = -{n \choose m} \frac{1}{n - m + 1}$.
-EDIT/ADD #2 (28 Sep 2011): Unfortunately, Helms' answer did not help very much. However, my access to academic resources is somewhat limited. It would be nice to have a direct reference to something containing the mentioned solution formula or a suitable equivalent and the specific kind of linear recurrence I'm asking about (the general form that is, not necessarily the "Schroder" one above), and also discussion of them of course.
-EDIT/ADD #3 (29 Sep 2011): Yeah. "Eigensequence" did not seem to yield much on Google (incl. Google Scholar and Google Books). It looks to be much too broad. Certainly didn't find anything with the solution formulas I mentioned. Could this usage be peculiar to the OEIS site?
-
-REPLY [3 votes]: I don't know what else this may be called. But if the coefficients $r_{n,m}$ are written as lower triangular matrix $R$ and the coefficients $a_n$ as column-vector $A$ then this is an eigenvector-problem at eigenvalue $\lambda =1$ :
-$ \qquad R*A=A*\lambda = A*1$
-(I should note, that the formulation of the problem indicates, that the diagonal of $R$ at least from the second row on seems to be zero here because the sum-index goes only to rowindex $n$ minus 1)
-Note 2: in the OEIS as well as in its associated electronic journal I've seen the term "eigensequence" and "invariant sequence" with the same meaning.
-Example:
-$ \qquad \small
-\begin{array} {rrrrrr}
- & & & & & | & 1 \\\
- & & & & & | & 2 \\\
- & R*&A =&A & & | & 7 \\\
- & & & & & | & 33 \\\
- & & & & & | & 201 \\\
- - & - & - & - & - & - & - \\\
- 1 & . & . & . & . & | & 1 \\\
- 2 & . & . & . & . & | & 2 \\\
- 1 & 3 & . & . & . & | & 7 \\\
- 1 & 2 & 4 & . & . & | & 33 \\\
- 2 & 3 & 4 & 5 & . & | & 201
- \end{array} $
-Here we have, for a rowindex $n$ : $ \qquad \sum_{m=1}^{n-1} R_{n,m}*A_m = A_n$<|endoftext|>
-TITLE: In the dictionary between Poisson and Quantum, what corresponds to Coisotropic?
-QUESTION [10 upvotes]: I work entirely over a field of characteristic $0$, in case it matters.
-Recall that a Poisson algebra is a commutative algebra $A$ with a bracket $\lbrace,\rbrace : A^{\wedge 2} \to A$ which is (1) a Lie bracket, i.e. it satisfies a Jacobi identity, and (2) a derivation in each variable. Or, maybe even better is that $(A,\lbrace,\rbrace: A^{\wedge 2} \to A)$ is a Poisson algebra if $A$ is a commutative algebra and $\forall a\in A$ $\lbrace a,-\rbrace : A \to A$ is a derivation of $(A,\lbrace,\rbrace)$.
-A coisotrope in $A$ is a vector subspace $I \subseteq A$ which is (1) an ideal for the multiplication on $A$ and (2) Lie subalgebra for $\lbrace,\rbrace$. In particular, I do not demand that $I$ be an ideal for the bracket. The most important examples of Poisson algebras are $A = \mathcal C^\infty(M)$, where $M$ is a symplectic manifold; then an embedded submanifold $N \hookrightarrow M$ is coisotropic ($\mathrm T^\perp N \subseteq \mathrm T N$) iff the vanishing ideal of $N$ is a coisotrope. For more details and equivalent characterizations, see:
-
-Alan Weinstein, Coisotropic calculus and Poisson groupoids, J. Math. Soc. Japan Volume 40, Number 4 (1988), 705-727. http://projecteuclid.org/euclid.jmsj/1230129807
-
-One reason to invent Poisson algebras is that the arise naturally as "deformation" or "quantization" problems: a Poisson algebra is the linear or infinitesimal data for a noncommutative algebra. In the most studied symplectic case, the Poisson algebra is in an important sense "maximally Poisson-noncommutative": the Poisson-center of $A$ consists only of (locally) constant functions. After quantization, the corresponding algebras are maximally noncommutative, and so should be algebras of (bounded) operators on some Hilbert space. In this sense, the quantization of a symplectic manifold $M$ is some Hilbert space $H$, or maybe its projectivization $\mathbb P H = H / \mathbb C^\times$.
-Then the general dictionary says that lagrangian, i.e. minimal coisotropic, submanifolds of a symplectic manifold $M$ should correspond to elements of $H$ (or maybe elements of $\mathbb P H$, i.e. lines in $H$). My question is to understand a generalization of this that relaxes two things:
-
-I am interested in algebras that are Poisson but not symplectic, and so might have center; then I do not expect to have as good a "Hilbert-space" description of the quanization. Rather, my quantizations of Poisson algebras $(A,\lbrace,\rbrace)$ are nothing more nor less than (flat) deformations of $A$ in the $\lbrace,\rbrace$ direction.
-In the non-symplectic setting, one loses a good theory of "lagrangian" submanifold, and the best stand-ins are the coisotropes.
-
-My question is then something along the following:
-
-Suppose I have a Poisson algebra $(A,\lbrace,\rbrace)$ with a coisotrope $I \subseteq A$, and I have a reasonably good quantization of $A$ in the $\lbrace,\rbrace$-direction. What should I expect/hope to see at the quantum level that corresponds to $I$?
-
-Asking the same question in the opposite direction:
-
-Suppose I have an associative algebra $B$ with a formal parameter $\hbar$ (and satisfying some strong flatness/topological freeness conditions), such that $B/(\hbar B)$ is commutative. Then the associated graded algebra $A = \bigoplus (\hbar B)^n / (\hbar B)^{n+1}$ is Poisson. What structures (e.g. ideals, left-modules, etc.) on $B$ become coisotropes in $A$ upon taking associated-graded?
-
-REPLY [6 votes]: This question was already positively answered.
-I'd like to add a remark on a situation where things go in a very clear way supporting Waldmann's statementes. Say $G$ is a Poisson Lie group.
-A (closed) subgroup $H$ is coisotropic iff the annhilator of its Lie algebra $\mathfrak h$ is a Lie subalgebra in $\mathfrak g^*$.
-Quantization of such subgroups are exactly two-sided coideal (this is the subgroup part) which are also one sided ideal (this is the coisotropy assumption).
-A two-sided coideal and one sided ideal inside a Hopf algebra is exactly what is needed to define a subalgebra of coinvariants which is a one sided coideal, i.e. a quantum homogeneous spaces seen as a subalgebra of the quantum algebra. This fact reflects the property that modding out a Poisson-Lie group by a coisotropic subgroup you get a Poisson homogeneous space (of quotient type, i.e. such that the quotient map is Poisson).
-The idea that one sided ideal is the non commutative counterpart of coisotropy was stated in a paper in the beginning of the 90's by Jiang-Hua Lu: the statement was named coisotropic creed.<|endoftext|>
-TITLE: Is there an elegant algebraic proof of this formula for quadratic field discriminants?
-QUESTION [14 upvotes]: Consider the Dirichlet series counting discriminants of real quadratic fields. Quadratic field discriminants are "basically" squarefree integers, so the associated Dirichlet series $\sum D^{-s}$ is "basically" $\zeta(s)/\zeta(2s)$. However, there is the funny business at 2, and one derives the formula
-$\sum D^{-s} = \frac{1}{2} \big( 4^{-s} - 1 \big) \frac{\zeta(s)}{\zeta(2s)} + \frac{1}{2} \big(1 - 4^{-s} \big) \frac{L(s, \chi_4)}{L(2s, \chi_4)},$
-which is a wee bit messy. (This formula, and all the subsequent ones, include 1 as a "quadratic field discriminant" for convenience.)
-However, I was reading a fantastic paper by David Wright, where he considers positive and negative discriminants together, in which case you have the much nicer formula
-$\sum |D|^{-s} = \big(1 - 2^{-s} + 2 \cdot 4^{-s} \big) \frac{\zeta(s)}{\zeta(2s)}.$
-This formula is easy to derive from scratch, but he derives it as a consequence of the beautiful formula
-$\sum |D|^{-s} = \prod_p \Big( \frac{1}{2} \sum_{[K_v : \mathbb{Q}_p] \leq 2} |\text{Disc}(K_v)|^s_p \Big).$
-He uses the parameterization of quadratic fields by $\mathbb{Q}^{\times} / (\mathbb{Q}^{\times})^2$, which may be thought of as $\text{GL}_1$-orbits on a one-dimensional prehomogeneous vector space, where $\text{GL}_1$ acts by $t(x) = t^2 x$ rather than the usual $t(x) = tx$. He then analyzes these orbits by means of an adelic zeta function; note that with the usual action you recover Tate's thesis.
-These formulas generalize quite a bit, with some complications, to $n$th-root extensions
-of any global field (with some restrictions on the characteristic). They also allow for twisting by characters, allowing (for example) a nice formula for $\sum \text{sgn}(D) |D|^{-s}.$ Note that the first formula considered looks nicer when viewed as a linear combination of $\sum |D|^{-s}$ and $\sum \text{sgn}(D) |D|^{-s}$.
-The MathSciNet review says that "similar results, however, can be obtained by class field theory", and this is also hinted at in Wright's paper, but the details aren't worked out. My first question is the following.
-Is there an elegant algebraic proof of the above identity for $\sum |D|^{-s}$ and its generalizations?
-And my second question, which is essentially a vaguer version of the first one, is:
-What is the best way to think of these formulas?
-I quite like the prehomogeneous vector space approach, but I imagine there might be a nice algebraic proof as well, and in particular some kind of ``local-to-global'' principle for quadratic discriminants. I am no expert in class field theory, and I am curious if any of these identities look simple and natural when viewed in the correct light.
-Thank you!
-
-REPLY [12 votes]: The discriminant $D$ of a quadratic number field can be written uniquely as a product of prime discriminants, namely $-4$, $\pm 8$, and $p^* = (-1)^{(p-1)/2}p$ for odd primes $p$.
-The Dirichlet series for odd discriminants therefore simply is
-$$ \sum_{D \text{ odd}} |D|^{-s} = \prod_{p \text{ odd}} (1 + p^{-s}), $$
-and the contribution of the even prime discriminants is taken care of by the factor
-$$ 1 + 4^{-s} + 2 \cdot 8^{-s}. $$
-Both the beautiful as well as the much nicer formula now follow immediately.
-Factorization of quadratic discriminants into prime discriminants holds over totally real algebraic number fields with class number $1$ in the strict sense (see e.g. L. Goldstein, On prime discriminants, Nagoya Math. J. 45, 119-127 (1972); J. Sunley, Remarks concerning generalized prime discriminants, Boulder 1972; J. Sunley, Prime discriminants in real quadratic fields of narrow class number one, Carbondale 1979); a weaker version good enough for the purpose of counting discriminants works if the class number in the strict sense is odd.<|endoftext|>
-TITLE: When is a restricted enveloping algebra a domain? A finitely generated domain?
-QUESTION [8 upvotes]: Suppose $\mathfrak{g}$ is a restricted Lie algebra over a field of characteristic $p>0$. Are there conditions on $\mathfrak{g}$ and its restriction which ensure that its restricted enveloping algebra is a domain? That it is a finitely generated domain? I really care about the graded case, so what if we assume further that $\mathfrak{g}$ is graded, concentrated in positive even degrees, and finite-dimensional in each degree?
-
-Edit: for example, if the restriction is injective, does that mean that $u(\mathfrak{g})$ is a domain? What if you only know that if $x^{[p]}=0$, then $x=0$ – is that good enough? In the graded case, if the restriction is also surjective in all sufficiently large degrees, is $u(\mathfrak{g})$ a finitely generated domain?
-Are there conditions on $\mathfrak{g}$ and its restriction which make $u(\mathfrak{g})$ isomorphic to an ordinary enveloping algebra $U(L)$ of some Lie algebra $L$?
-
-REPLY [3 votes]: Let $g$ be a restricted Lie algebra over a field of positive characteristic $p$. It is not difficult to see that a necessary condition such that the restricted enveloping algebra $u(g)$ of $g$ is a domain is that $g$ has no nonzero $p$-algebraic elements. (An element $x \in g$ is said to be $p$-algebraic if the restricted subalgebra generated by $x$ is finite-dimensional.) The converse of this property is a well-known open question posed by V. Petrogradsky (see "DNIESTER NOTEBOOK: Unsolved Problems in the Theory of Rings and Modules", Problem 3.59). Apparently, this is the Lie theoretical analog of the Kaplansky Problem about zero-divisors of group algebras of torsion-free groups.<|endoftext|>
-TITLE: Universal sets in metric spaces
-QUESTION [11 upvotes]: (I am cross-posting this from math.SE as it seems to be slightly over the top for that site.)
-I saw in the class the theorem:
-Suppose $X$ is a separable metric space, and $Y$ is a polish space (metric, separable and complete) then there exists a $G\subseteq X\times Y$ which is open and has the property:
-For all $U\subseteq X$ open, there exists $y\in Y$ such that $U = \{x\mid\langle x,y\rangle\in G\}$.
-$G$ with this property is called universal.
-The proof is relatively simple, however the $y$ we have from it is far from unique, in fact it seems that it is almost immediate that there are countably many $y$'s with this property.
-My question is whether or not this $G$ can be modified such that for every $U\subseteq X$ open there is a unique $y\in Y$ such that $U = \{x\mid\langle x,y\rangle\in G\}$?
-Perhaps we need to require more, or possibly even less, from $X$ and $Y$?
-Some thoughts:
-Firstly $X$ cannot be finite, otherwise there are less than continuum many open subsets, and since $G$ is open we have that the projection on $Y$ is open, since $Y$ is Polish we have that this projection is of cardinality continuum, which in turn implies there are continuum many $y$'s with the same cut.
-Secondly, as the usual proof goes through a Lusin scheme over $Y$, and using it to define $G$, I thought at first that using the axiom of choice we can select a set of points on which the mapping to open sets of $X$ is 1-1, and somehow remove some of the sets from the scheme. This proved to be a bad idea, as we remove sets that can be used for other open sets.
-Thirdly, I thought about enumerating the open sets according to a rational enumeration so $A_i\subseteq A_j$ if and only if $q_i\le q_j$, and then instead of just placing the open sets of $X$ arbitrarily by the Lusin scheme, we use the rationals somehow.
-
-REPLY [4 votes]: While idly browsing around I stumbled over the follwing paper and remembered this question:
-A.W. Miller, Uniquely Universal Sets, Topology and its Applications 159 (2012), pp. 3033–3041. It's available in various formats here.
-Let me quote the abstract (to avoid confusion: Miller's terminology reverses the rôles of $X$ and $Y$ in your question):
-
-We say that $X \times Y$ satisfies the Uniquely Universal property (UU)
- iff there exists an open set $U \subseteq X \times Y$ such that for
- every open set $W \subseteq Y$ there is a unique cross section of
- $U$ with $U_x=W$. Michael Hrušák raised the question of when does
- $X \times Y$ satisfy UU and noted that if $Y$ is compact, then $X$
- must have an isolated point. We consider the problem when
- the parameter space $X$ is either the Cantor space $2^\omega$ or the Baire
- space $\omega^\omega$.
- We prove the following:
-
-If $Y$ is a locally compact zero dimensional
- Polish space which is not compact, then $2^\omega\times Y$ has UU.
-If $Y$ is Polish, then
- $\omega^\omega \times Y$ has UU iff $Y$ is not compact.
-If $Y$ is a $\sigma$-compact subset of a Polish space which is
- not compact, then $\omega^\omega \times Y$ has UU.
-
-
-His results are mostly positive: “a certain space or family of spaces has UU” and various permanence properties. One nice “negative” result:
-
-Proposition 30: There exists a partition $X\cup Y=2^\omega$ into Bernstein sets
- $X$ and $Y$ such that for every Polish space $Z$ neither
- $Z\times X$ nor $Z\times Y$ has UU.
-
-He also raises a few questions, e.g.:
-
-Question 4: Does $(2^\omega\oplus 1) \times [0,1]$ have UU?
-Question 6: Does either $\mathbb{R} \times \omega$
-or $[0,1]\times \omega$ have UU?
-Or more generally, is there any example of UU for a
-connected parameter space?
-Question 11: Is the converse of Corollary 10 false?
-That is: Does there exist $Y$ such that $\omega^\omega \times Y$ has UU but
-$2^\omega\times Y$ does not have UU?<|endoftext|>
-TITLE: Formally smooth morphisms, the cotangent complex, and an extension of the conormal sequence
-QUESTION [9 upvotes]: I'm reading Daniel Quillen's paper "Homology of commutative rings," in which he proves:
-A finitely presented morphism of rings $A \to B$ is
-
-Formally etale iff $L_{B/A}$ (this denotes the cotangent complex) is homotopy-equivalent to zero
-Formally smooth iff $\Omega_{B/A}$ is projective and $L_{B/A}$ is homotopy-equivalent to it (i.e. acyclic outside degree zero).
-(It's also true, and elementary (not in the paper), that formally unramified iff $\Omega_{B/A} = 0$.)
-
-I've heard that these results are true even without finitely presented hypotheses. In fact, I understand that the fpqc localness of projectivity is what one uses to show that formal smoothness is, in fact, a local property (cf. 2 above). I also know how to show that if $A \to B$ is formally smooth, then the differentials are a projective $B$-module (take a quotient by some polynomial ring, $B = C/I$, and show that the sequence $I/I^2 \to \Omega_{C/A} \otimes_C B \to \Omega_{B/A} \to 0$ is actually split exact).
-In fact, one can show that if $C$ is a formally smooth $A$-algebra and $B = C/I$, then $B$ is smooth iff the conormal sequence above is split exact.
-So I am guessing that there is an extension of the conormal sequence to the cotangent complex, and if this works will prove 2 without finitely presented hypotheses (and thus 1 as well). How does this "long exact sequence" work?
-
-REPLY [6 votes]: We deduce this from the Jacobi-Zariski exact sequence as follows:
-Some notation first: Let $H_i(A,B,W)$ where B is an A-algebra and W is a B-module denote $\pi_i(L_{B/A}\otimes_B W)$.
-Then given an $A$-algebra $B$, a $B$-algebra $C$, and a $C$-module $W$, we have a long-exact sequence, called the Jacobi-Zariski sequence:
-$$\dots \to H_n(A,B,W)\to H_n(A,C,W) \to H_n(B,C,W)\to H_{n-1}(A,B,W)\to \dots \to H_0(B,C,W)\to 0$$.
-Now, let $C$ be an $A$-algebra, let $B$ be a polynomial ring over $A$ with an ideal $I$ such that $B/I\cong C$.
-Then we have the following long-exact sequence:
-$$\dots \to H_n(A,B,C)\to H_n(A,C,C) \to H_n(B,C,C)\to H_{n-1}(A,B,C)\to \dots \to H_0(B,C,C)\to 0$$.
-Since $C$ is a quotient of $B$, it is formally unramified over $B$, so $H_0(B,C,C)\cong \Omega_{C/B}=0$. We also see that $I/I^2$ is precisely $H_1(B,C,C)$ by Proposition 1 of Chapter VI.a of the book Homologie des algèbres commuatatives by Michel André. This gives us a long-exact sequence
-$$\dots \to H_n(A,B,C)\to H_n(A,C,C) \to H_n(B,C,C)\to H_{n-1}(A,B,C)\to \dots $$
-$$\to H_1(A,B,C) \to H_1(A,C,C)\to I/I^2 \to H_0(A,B,C)\to H_0(A,C,C)\to 0$$.
-We see that the conormal exact sequence is the truncation at $I/I^2$, and that this sequence splits if $H_0(A,C,C)$ is projective and $H_1(A,C,C)$ is $0$. Conversely, if the sequence is split-exact, then $H_0(A,C,C)$ is a direct summand of $H_0(A,B,C)$, which is projective (it might be free, but I don't remember), and the kernel of $I/I^2\to H_0(A,B,C)$ is $0$, which combined with the fact that $H_1(A,B,C)=0$ implies that $H_1(A,C,C)=0$.<|endoftext|>
-TITLE: Reference request for projective representations of finite groups over a non-problematic field
-QUESTION [11 upvotes]: I would like to get a reference where I can learn about the theory of projective representations of finite groups over the complex numbers (or over any field K such that the order of the given group under study is invertible in K). And how one can relate this to character theory for linear representations (with which I am more familiar). Thanks.
-
-REPLY [2 votes]: In addition: Karpilovsky, Gregory.
-The Schur Multiplier. London Math. Soc. Monographs, 1987.
-In this book only $\mathbb{C}$ is considered.<|endoftext|>
-TITLE: Explicit extension of Lipschitz function (Kirszbraun theorem)
-QUESTION [17 upvotes]: Kirszbraun theorem states that if $U$ is a subset of some Hilbert space $H_1$, and $H_2$ is another Hilbert space, and $f : U \to H_2$ is a Lipschitz-continuous map, then $f$ can be extended to a Lipschitz function on the whole space $H_1$ with the same Lipschitz constant.
-Now let's take $H_2$ to be the Euclidean space $\mathbb{R}^n$. My question is: Is there way to explicitly construct this extension? Note that the standard proof (e.g. see Federer's geometric measure theory book or Schwartz's nonlinear functional analysis book) is an existence proof, which uses Hausdorff's maximal principle.
-Some remarks:
-1) For $n = 1$, the extension can be constructed explicitly, which works even if $H_1$ is only a metric space (with metric $d$): $\tilde{f}(x) = \inf_{y \in U} \{ f(y) + {\rm Lip}(f) d(x,y) \}$. See for example Mattila's book p. 100.
-2) For $n > 1$, performing the above extension for each component of $f$ results in blowing up the Lipschitz constant by a factor of $\sqrt{n}$.
-
-REPLY [4 votes]: If I remember well the Kirszbraun's extension of a $L$-Lipschitz map $f:U\subset H_1\to H_2$ has the following canonical construction, analogous to the one-dimensional case you mentioned (so in a sense it is explicit).
-Let $\mathcal{Co} (H_2)$ denote the metric space of all non-empty bounded closed convex sets of $H_2$ endowed with the Hausdorff distance. Let $f_*:H_1\to \mathcal{Co}(H_2)$ be defined by $$f_*(x):=\cap_{u\in U}\overline{B}(f(u),L)$$
-(In other words, $f_*$ takes $x\in H_1$ to the set of the admissible values at $x$ for any $L$-Lipschitz extension of $f$ to $U\cup\{x\}$). This map $f_*$ has the same Lipschitz constant of $f$, w.r.to the Hausdorff distance on $\mathcal{Co}(H_2)$.
-Any non-empty bounded closed convex $C$ of a Hilbert space $H$ has a well-defined point $\kappa( C), $ the center of the closed ball of minimum radius containing $C$; this point is unique, and the corresponding map $\kappa: \mathcal{Co}(H)\to H $ is $1$-Lipschitz.
-One can therefore define a canonical $L$-Lipschitz extension of $f$ as $\tilde f:=\kappa \circ f_*$. In case $n=1$, the set $f_*(x)$ is just an interval, its end-points are the inf-convolution you mentioned, and the sup-convolution, and this $\tilde f$ is their arithmetic mean.<|endoftext|>
-TITLE: Products of Ideal Sheaves and Union of irreducible Subvarieties
-QUESTION [7 upvotes]: Assume I have a nonsingular, irreducible, algebraic variety $X$ and irreducible, nonsingular subvarieties $Z_1,\ldots,Z_k\subseteq X$. Let $\mathcal{I}_i$ be the ideal sheaf of $Z_i$ and $\mathcal{I}:=\mathcal{I}_1\cdots\mathcal{I}_k$ the product. My question is whether $\mathcal{I}$ is the ideal sheaf of the union $Z_1\cup\ldots\cup Z_k$. You may assume that $Z_1\cap\ldots\cap Z_k=\emptyset$ or, equivalently, $\mathcal{I}_1+\ldots+\mathcal{I}_k=\mathcal{O}_X$. You may also assume that the $Z_i$ intersect transversally, if that helps.
-
-REPLY [2 votes]: As Sandor and Martin pointed out above, it is ok if the subschemes intersect in the emptyset pairwise. I'm going to provide 3 examples, I think the third one gives a counter-example to even the transversality statements.
-Example 1: Here's an example where it's false without that hypotheses, notice that the varieties are smooth and they intersect pairwise with normal crossings. EDIT: as t3suji pointed out in the comments, these varieties don't intersect transversally in the ambient space, just in the ambient $z = 0$ plane EndOfEdit Consider $X = \mathbb{A}^3$ and set $Z_1 = V(y,z)$, $Z_2 = V(x,z)$, $Z_3 = V(x, z-1)$. Notice that $Z_3$ doesn't intersect any of the other subschemes.
-Then, $I_1 \cap I_2 = (z, xy)$.
-However, $I_1 \cdot I_2 = (xy, yz, xz, z^2)$.
-These ideals are not equal clearly. Now, we can immediately see that multiplying/intersecting by $I_3$ won't change the behavior at the origin at all since the ideal doesn't vanish there, so they are not equal. However, just to be sure, I also did the following computation (with Macaulay2):
-$$I_1 \cdot I_2 \cdot I_3 = (xz, yz, z^3 - z^2, yz^2 - yz, xz^2 - xz).$$
-$$I_1 \cap I_2 \cap I_3 = (z^2 -z, xz, xy).$$
-Macaulay2 also confirmed that the ideals were not equal.
-Example 2: Here's a different example. a 4th variety that doesn't intersect the others at all.
-$X = \mathbb{A}^3$. $I_1 = (x,y)$, $I_2 = (x,z)$, $I_3 = (y,z)$ and $I_4 = (x-1,y-1,z-1)$. Certainly again the $I_4$ doesn't matter, it's just included so that the sum of the ideals is equal to $R = k[x,y,z]$. I wonder if it might be reasonable to say that $Z_1$, $Z_2$ and $Z_3$, as a triple, have transverse intersection at the origin. Anyways:
-$I_1 \cap I_2 \cap I_3 = (xy, xz, yz)$ but,
-$I_1 \cdot I_2 \cdot I_3 = (yz^2, xz^2, y^2z, xyz, x^2z, xy^2, x^2y)$.
-Example 3: Ok, now I'm just going to give three subvarieties which intersect at the origin, pairwise transversally, and such that the product of the ideals is not equal to the intersection. You may add the ideal sheaf of some other variety that doesn't intersect them at all to make the sum of ideals equal the whole structure sheaf.
-$X = \mathbb{A}^4 = \text{Spec}k[x,y,u,v]$. $I_1 = (x,y)$, $I_2 = (u,v)$, $I_3 = (x+u, y+v)$. I believe these have pairwise transverse intersection at the origin. Now then, it is true that $I_1 \cdot I_2 = I_1 \cap I_2$, and likewise with any pair. However, Macaulay2 can be used to verify that $I_1 \cdot I_2 \cdot I_3 \neq I_1 \cap I_2 \cap I_3$. Roughly speaking the problem is that $Z_1 \cup Z_2$ has funny intersection with $Z_3$.
-Ok, let me now give a proof of a correct statement showing that sometimes they are equal.
-Lemma: Suppose that subschemes $Z_1, \dots, Z_k$ have pairwise trivial intersection in some ambient Noetherian scheme $X$. Then $I_{Z_1} \dots I_{Z_K} = I_{Z_1} \cap I_{Z_k}$.
-Proof: The statement is local so we may assume that $X$ is the spectrum of a local ring $(R, \mathfrak{m})$. Now, since $I_{Z_1} + I_{Z_2} = R$, at least one of those ideals must equal $R$ (if not, both would be in the maximal ideal $\mathfrak{m}$, and so would their sum). Likewise with all pairs. Therefore, at most one of the ideals $I_{Z_i}$ is not equal to $R$. But now the statement is obvious. $R \cdot R \dots I_{Z_i} \dots R = I_{Z_i} = R \cap R \cap \dots I_{Z_i} \cap \dots R$.<|endoftext|>
-TITLE: Global Algebraic Proof of the Kahler Identities?
-QUESTION [11 upvotes]: I'm looking at Kahler geometry at the moment and admiring how it manages to do so much with clean global algebraic arguments. One of the big exceptions to all this, however, is the proof of the Kahler identities
-$$
-[\Lambda,\overline{\partial}]=-i \partial^\ast, ~~~~~~
-[\Lambda,\partial]=-i \overline{\partial}^\ast.
-$$
-In the two standard references, Voisin, and Griff + Harris, the identities are proved using arguments that are local and somewhat analaytic. Does there exists anywhere a nice global algebraic proof?
-
-REPLY [7 votes]: Actually, there is an argument that doesn't use neither Weil identities nor flat up to a second order coordinates, instead, it uses the symplectic Hodge star, invented by Brylinski. When you have a 2n-dimensional manifold $M$ with the non-degenerate 2-form $\omega$, you can define an operator $*_s:\Lambda^k(M) \mapsto \Lambda^{2n-k}(M)$ by the standard identity $\alpha \wedge *_s\beta = \omega(\alpha,\beta)\omega^n$. Then, for a closed $\omega$ a really simple computation (you can do it only for the case of two variables, and then use induction on dimension) in Darboux coordinates shows that, up to some sign ($(-1)^{k+1}$, i guess), $*_sd*_s=[\Lambda, d].$ It is written in the Brylinski's article "A differential complex for Poisson manifolds". And then you need to observe that for a Kaehler $M$ symplectic and Riemannian Hodge stars differ by an action of $I$. The Kaehler identity $d^*=[\Lambda, d^c]$ follows from that.<|endoftext|>
-TITLE: Bridge game with only one suit: strategy
-QUESTION [6 upvotes]: This game looks like bridge, but 1- there are only two players Alice and Bob, 2- there is only one suit, whose cards are numbered $1, 2,\ldots,2n$. One deals each player $n$ cards. Therefore Alice knows Bob's cards and conversely; once the cards are dealt, there is no randomness.
-Alice play a card, then Bob. The highest card wins the trick. The winner of the trick leads a card and so on. At the end Alice has got $p$ tricks and Bob $n-p$ tricks. The goal for each player is to get as much tricks as possible with the cards (s)he was dealt.
-My question is about the strategy of play and the number of tricks you expect to win in a given layout. The answer should not be obvious. Let me give an example with $n=3$. If Alice is dealt $6,4,1$, she gets two tricks by leading first the $1$. If she has instead $6,3,2$, she leads the $3$ (equivalently the $2$). I see easily the way to get as many tricks as possible if $n$ is small, say $n\le6$, but I don't see a generalization.
-Of course, the number of expected tricks with a given hand differs whether you begin or you opponent does.
-
-REPLY [19 votes]: I believe the game you describe is two-person single suit whist and was solved by Johan Wastlund in this paper.<|endoftext|>
-TITLE: Sums of uncountably many real numbers
-QUESTION [5 upvotes]: Suppose $S$ is an uncountable set, and $f$ is a function from $S$ to the positive real numbers. Define the sum of $f$ over $S$ to be the supremum of $\sum_{x \in N} f(x)$ as $N$ ranges over all countable subsets of $S$. Is it possible to choose $S$ and $f$ so that the sum is finite? If so, please exhibit such $S$ and $f$.
-
-REPLY [2 votes]: This is a standard result in undergraduate analysis, although it is admittedly somewhat hard to find in the standard references. The following is a very non-standard reference: see the last exercise in II.9.4 of these notes on sequences and series (see p. 69...for now; page numbers are subject to change). They occur in the context of a larger discussion on unordered summation, which is what you are looking into above. The general definition of unordered summability is a bit more complicated (it is a nice special case of convergence with respect to a net, although one needn't use the term), but in the case where the values of the "$S$-indexed sequence" are non-negative, it coincides with what you have given: see Proposition 82.
-Note that this fact comes up sometimes in practice. In this math.SE question I set as a challenge to give a proof of the following fact -- there is no function $f: \mathbb{R} \rightarrow \mathbb{R}$ with a removable discontinuity at every point -- which does not use the kind of uncountable pigeonhole principle argument that you need to answer the current question. And I got a very nice answer!<|endoftext|>
-TITLE: Evaluating the integral $\int_0^\infty \frac{\psi(x)-x}{x^2}dx.$
-QUESTION [8 upvotes]: Let $\psi(x)=\sum_{n\leq x} \Lambda(n)$ be the weighted prime counting function. I am trying to evaluate the integral $$\kappa:=\int_{1}^{\infty}\frac{\psi(x)-x}{x^{2}}dx$$ in several different ways. Originally, this integral came up as a particular part in a particular case for a a formula for a summatory function I was looking at. From now on, let $\gamma$ refer to the Euler-Mascheroni constant.
-(Now Corrected:) I found a fun, elementary approach to this integral which gave $\kappa=-1-\gamma$ if we assume the quantitative prime number theorem. (Precisely, we just need to assume that this integral is absolutely convergent. ) Since I am not too confident about this, I naturally wanted to check by complex analytic methods to see if my answer was correct. My question then is:
-
-What other ways can be used to prove this identity?
-
-I feel like knowing many approaches to this problem will give a greater understanding of certain properties of these functions. A friend suggested that it must be related to the logarithmic derivative of $\zeta(s)$, and certain special values, but I cannot see how to use this.
-Thanks a lot!
-Additional Remark:
-I attempted to use the explicit formula for $\psi(x)$, and deduced $\kappa=-\gamma-1$. Originally I felt this was wrong, but after reading Julian Rosen's answer I think it is correct. Here is the alternate solution:
-Substituting in the explicit formula, and then integrating termwise we have$$\kappa=\int_{1}^{\infty}\left(-\sum_{\rho}\frac{x^{\rho-2}}{\rho}-\frac{\log2\pi}{x^{2}}-\frac{\log\left(1-x^{-2}\right)}{2x^{2}}\right)dx=\sum_{\rho}\frac{1}{\rho(\rho-1)}-\log2\pi+1-\log2$$since
-$$\frac{1}{2}\int_{1}^{\infty}\frac{\log\left(1-x^{-2}\right)^{-1}}{x^{2}}dx=\frac{1}{2}\int_{1}^{\infty}\sum_{i=1}^{\infty}\frac{1}{ix^{2i+2}}dx=\sum_{i=1}^{\infty}\frac{1}{2i(2i+1)}=1-\log2.$$As $$\sum_{\rho}\frac{1}{\rho(\rho-1)}=\sum_{\rho}\frac{1}{\rho-1}-\frac{1}{\rho}=-\sum_{\rho}\frac{1}{1-\rho}+\frac{1}{\rho}=2B=-\gamma-2+\log4\pi$$it follows that $\kappa=-\gamma-1$.
-
-REPLY [3 votes]: I want to prove it with an elementary approach:
-a theorem of Landau say that the PNT is equivalent to $\sum_{n\leq x} \frac{\Lambda(n)}{n}=log(x)-\gamma+o(1)$.
-Now using partial summation we have:
-$\displaystyle \sum_{n\leq x} \frac{\Lambda(n)}{n}=\displaystyle \int_{1}^{x}\frac {d\psi(t)}{t}=\int_{1}^{x}\frac {d(\psi(t)-t)}{t}+\int_{1}^{x}\frac {dt}{t}=log(x)+\int_{1}^{+\infty}\frac {\psi(t)-t}{t^{2}}-\int_{x}^{+\infty}\frac {\psi(t)-t}{t^{2}}+\frac{\psi(x)-x}{x}-\frac{\psi(1)-1}{1}=log(x)+\int_{1}^{+\infty}\frac {\psi(t)-t}{t^{2}}+o(1)+1$
-so that $\displaystyle \int_{1}^{+\infty}\frac {\psi(t)-t}{t^{2}}=-1-\gamma$.<|endoftext|>
-TITLE: What is the standard notation for group action
-QUESTION [20 upvotes]: Please let me know what is the standard notation for group action.
-
-I saw the following three notations for group action.
-(All the images obtained as G\acts X for different deinitions of \acts.)
-(1)
-I saw this one most, but only in handwriting and I like it. But I did not find a better way to write it in LaTeX.
-\usepackage{mathabx,epsfig}
-\def\acts{\mathrel{\reflectbox{$\righttoleftarrow$}}}
-
-(2)
-It is almost as good as 1, but in handwriting this arrow can be taken as $G$.
-\usepackage{mathabx}
-\def\acts{\lefttorightarrow}
-
-(3)
-I saw this one in print, I guess it is used since there is no better symbol in "amssymb".
-\usepackage{amssymb}
-\def\acts{\curvearrowright}
-
-REPLY [6 votes]: As this was the first hit on google for "latex action arrow", but didn't contain what I wanted, let me post what I figured out. But to address other people's issues with the original question: while I agree that in a sentence one should simply say "Let $G$ act on $X$...", I was interested in drawing a (what some people just generically call "commutative") diagram in order to visually display the relationships between several objects (as you do).
-Anyway, there is a symbol $\circlearrowright$, which is almost what I wanted. If you do this:
-
-\usepackage{graphicx}
- G\ \rotatebox[origin=c]{-90}{\$\circlearrowright\$}\ X
-
-you will get an arrow whose beginning and end are right next to the X.<|endoftext|>
-TITLE: Evaluating the integral $\int_{1}^{\infty}\frac{\{u\}}{u^{2}}\left(\log u\right)^{k}du.$
-QUESTION [15 upvotes]: I am trying to find a formula for the following integral for non-negative integer $k$:
-$$\int_1^{\infty}\frac{\{u\}}{u^{2}}\left(\log u\right)^{k}du.$$
-My first thought was to use the formula $$\zeta(s)-\frac{1}{s-1}=1-s\int_1^\infty u^{-s-1}\{u\}du$$ where $\{u\}$ refers to the fractional part. We can then take derivatives with respect to $s$ and use the Laurent expansion for $\zeta(s)$. It follows that each integral must be a finite linear combination of the Stieltjes Constants. All of the coefficients must be integers, and $\gamma_n$ can only appear if $n\leq k$. (This checks out numerically for $k=0,1,2$)
-Unfortunately, I am not sure what the pattern is, but I feel these particular integrals must be very common, and must have been dealt with before. I am hoping someone can give me a reference, or give a solution.
-Thanks a lot,
-
-REPLY [21 votes]: Let $a_k$ be the integral. Then
-$$\begin{eqnarray*}
- \sum_{k \ge 0} \frac{a_k}{k!} t^k &=& \int_1^{\infty} \frac{ \{ u \} }{u^2} e^{t \log u} \, du \\\
- &=& \int_1^{\infty} \{ u \} u^{t-2} \, du \\\
- &=& \frac{1 - \zeta(1 - t) - \frac{1}{t}}{1 - t} \\\
- &=& \frac{1}{1 - t} \left( 1 - \sum_{n \ge 0} \frac{\gamma_n}{n!} t^n \right).
-\end{eqnarray*}$$
-(Generating functions are good for more than combinatorics!) This is equivalent to Julian Rosen's answer, but (I think) packaged slightly more conveniently.<|endoftext|>
-TITLE: What does the space induced by this unusual metric(?) on R/Z look like?
-QUESTION [8 upvotes]: The motivation for this question comes from music theory. Dmitri
-Tymoczko models "good" voice leading as minimizing distance between
-pitches in successive chords. While this theory works well for upper
-voices, it does not work so well for the bass, which tends to move by
-4ths and 5ths quite frequently. (Tymoczko explicitly excludes the
-bass from his model.)
-Taking log (all logs are base 2 in this question) of frequency and
-taking pitches which differ by an octave as equivalent, we get
-$\mathbb{R}/\mathbb{Z}$. For the upper voices, we want the standard
-metric on this.
-For the bass, we want moving by a fifth or a fourth - meaning by $\pm
-\log (3/2)$ to be small. So we want $d(x,x\pm\log(3/2))=k_1$, where
-$k_1$ is probably somewhere around $0.05$. To make this a metric
-space, let's declare that $d(x,y)$ should be the minimum of $|x-y|$,
-$k_1+||x-y|-\log(3/2)|$, and $k_1+||x-y|-\log(4/3)|$.
-We probably want moving by a major third - meaning by $\pm\log(5/4)$
-to also be small, but not as small. I suspect $2k_1$ would make the
-most mathematical sense, but any constant of roughly that magnitude is
-fine. Ditto for minor thirds - this would be movement by
-$\pm\log(6/5)$, with a slightly larger constant.
-If we do this, we might as well make all movements by $\pm\log(p/q)$
-small if $q$ is small.
-CLARIFICATION: I also want the standard metric to be one of the options for getting from $x$ to $y$. So the distance between $0$ and $\sqrt{1/500}$ should be $\sqrt{1/500}$, while the distance between $0$ and $7/12$ should be $k_1+|\log(3/2)-7/12|$. (Musically, $|\log(3/2)-7/12|$ is how far off an even-tempered 5th is from Pythagorean tuning.)
-Question 1: Can one actually define something along these lines that
-satisfies the triangle inequality? (I don't think I actually have; I
-probably need to take the minimum (or infimum) of some infinite
-sequence, but am not entirely sure that works.)
-Question 2: Assuming the answer to (1) is yes, what does this metric
-space look like? Can someone help me with a picture that seems less
-exotic, perhaps comparing it to the Hawaiian Earring or something of
-that sort? In particular, what might the fundamental group look like?
-My background: I'm a combinatorialist and algebraic geometer who happens
-to be the one least unqualified here to be supervising an
-undergraduate independent study on mathematics in music theory. I did
-the standard first year graduate courses in point set topology and
-algebraic topology, but that was almost a dozen years ago.
-
-REPLY [3 votes]: I doubt that you can do this in a way which addresses the intended application. For instance, you say you want bass movement by a fifth to be "small" because it is frequent in traditional music. You also want stepwise movement by a full or half step to be "small." But you can combine a fifth and a half step to make a tritone, and the triangle inequality would then force that to be "small." But this is not something you want. It seems to me that the triangle inequality is not an adequate assumption to impose on what you are trying to model.<|endoftext|>
-TITLE: When is a complete fan a normal fan?
-QUESTION [7 upvotes]: Is there a characterization for when a complete fan in $\mathbf{R}^n$ is the normal fan of a polytope? Thanks!
-
-REPLY [4 votes]: The characterisation is as follows: There should exist a piecewise linear convex function of the fan, linear at each face of top dimension and having different gradients at all these top-dimensional faces.
-Indeed if you have a convex polytope $P$ with vertices $v_i$ in $\mathbb R^n$ this defines you a collection of linear functions $v^*_i$ on $\mathbb R^{n*}$ and the function $\max_i (v^*_i)$ will be a convex function of the dual fan satisfying the above properties.<|endoftext|>
-TITLE: Does a closed immersion of an affine scheme in a smooth scheme factor over an open affine subscheme?
-QUESTION [5 upvotes]: Assume we have an affine scheme $A$ that comes with a closed immersion into a smooth scheme $i \colon A \hookrightarrow M$, which is not necessarily affine.
-Does there exist an affine open subscheme $j \colon V \hookrightarrow M$ such that $A$ already embeds in $V$?
-Expressed in Diagrams, does there exist an affine open subscheme $V$ of $M$ such that we have a factorization
-
- i
-A (----> M
- \ /
- \ /
- V ?
-
-REPLY [5 votes]: There are also examples of smooth proper (nonprojective) varieties $V$ with a finite subset not contained in any affine open subset.<|endoftext|>
-TITLE: Homotopy type of the complement to a subvariety of $\mathbb C^n$
-QUESTION [7 upvotes]: Let $V^k\subset \mathbb C^n$ be a sub variety, such that all its irreducible components have dimension $\ge k$. Is it true that $\mathbb C^n\setminus V^k$ has homotopy type of a CW complex of dimension $\le 2n-k-1$?
-Comments. 1)This is true for $k=n-1$, since in this case $\mathbb C^n\setminus V^{n-1}$ is affine. Case $k=0$ is trivial.
-2) This question would help to answer:
-An analogue of Lefschetz hyperplane theorem for complements to subvarieties in $\mathbb C^n$ ?
-
-REPLY [8 votes]: Take $n=4$ and let $V = \{ z_1=z_2=0 \} \cup \{ z_3=z_4=0 \}$. I claim that $\mathbb{C}^4 \setminus V$ is homotopic to $S^3 \times S^3$, which has nontrivial homology in degree $6$, contrary to your supposed bound, which is in degree $5$.
-Note that $\mathbb{C}^4 \setminus V = \left( \mathbb{C}^2 \setminus \{ (0,0) \} \right)^2$. Taking the quotient by $\mathbb{R}_{+}$, we see that $\mathbb{C}^2 \setminus \{ (0,0) \}$ is
-homotopic to $S^3$, so $\mathbb{C}^4 \setminus V$ is homotopic to $S^3 \times S^3$.
-
-I can prove the required cohomology vanishing if you require that $V$ be Cohen-Macaulay.
-Write $U$ for $\mathbb{C}^4 \setminus V$. We have the Hodge-de Rham spectral sequence: $H^q(U, \Omega^p) \implies H^{p+q}(U, \mathbb{C})$. Singe $U$ is an open subset of $\mathbb{C}^n$, we have $\Omega^p \cong \mathcal{O}^{\oplus \binom{n}{p}}$ so $H^q(U, \Omega^p) \cong H^q(U, \mathcal{O})^{\bigoplus \binom{n}{p}}$.
-We can identify $H^q(U, \mathcal{O})$ with a local cohomology module of $V$, which the Cohen-Macaulay condition should force to be $0$ for $q > n-k-1$. So $H^q(\Omega^p)$'s will be zero for $q>n-k-1$. Then the spectral sequence immediately forces cohomology to vanish for $p+q > n+(n-k-1)$, as you desired.
-I have no idea of how to get a statement in homotopy out of the Cohen-Macaulay condition.<|endoftext|>
-TITLE: Techniques for computing cup products in singular cohomology
-QUESTION [17 upvotes]: Suppose that we are given a CW complex X in terms of the cells and the gluing maps. My understanding is that computing the cup product of the singular cohomology ring from this information is a non-trivial task. I know of two basic strategies that one might take:
-1) If the X is homotopy equivalent to a closed oriented manifold, then we can translate from cup product into intersection product and the problem becomes easier to visualize.
-2) If X is not too complicated, then we can try to find a simple presentation of X as a finite simplicial complex and compute the cup product explicitly for all the cochains.
-My question is: what are other techniques/tricks that can be used to find the cup product?
-Surely there must be some general approaches beyond the naive ones I mentioned. Feel free to strengthen the hypotheses or consider specific situations, as I don't expect there to be one trick which works for everything.
-
-REPLY [29 votes]: This is going to be a perhaps tendentious diatribe. But it is what is.
-Naturality, dimensional arguments, and Poincare duality give a
-reservoir of elementary examples such as spheres and projective spaces.
-In practice, to go from there to more serious examples, one uses spectral
-sequences to bootstrap up, and then one uses still more spectral sequences to
-bootstrap up to still more serious examples. The dirty secret is that modern
-algebraic topologists rarely if ever try to compute cup products by use of
-cochains, which means that they rarely if ever use cochains for serious
-calculations. The huge range of known calculations show how well this works.
-The actual diagonal map $X\longrightarrow X\times X$ can only be helpful
-in the very simplest examples, for the obvious reason that explicit
-calculations must use cellular cochains (not singular, which are far too
-large for explicit computation), and the diagonal map is never cellular:
-it takes the n-skeleton to the 2n-skeleton. To compute cup products with
-cellular cochains, one must find a cellular map homotopic to the diagonal
-map. While such a cellular approximation always exists, it is rarely an
-easy task to write one down explicitly.
-Peter May<|endoftext|>
-TITLE: Codes, lattices, vertex operator algebras
-QUESTION [22 upvotes]: At the end of "Notes on Chapter 1" in the Preface to the Third Edition of Sphere packings, lattices and groups, Conway and Sloane write the following:
-
-Finally, we cannot resist calling attention to the remark of Frenkel, Lepowsky and Meurman, that vertex operator algebras (or conformal field theories) are to lattices as lattices are to codes.
-
-I would like to understand better what the precise analogy is that is being made here.
-Through my attempts to read Frenkel, Lepowsky and Meurman's book, I am aware of the story about how the "exceptional" objects,
-Golay code ---> Leech lattice ---> Moonshine module,
-form a hierarchy with increasingly large symmetry groups,
-Mathieu group M24 ---> Conway group Co1 ---> Monster group,
-and how this hierarchy led to the conjecture that uniqueness results for the Golay code and Leech lattice carry over to a uniqueness property of the Moonshine module. Frenkel, Lepowsky and Meurman speak of many analogies between the theories of codes, lattices, and vertex operator algebras. I have some understanding of the connections between codes and lattices, but so far very little understanding of vertex operator algebras and of their connection with lattices (despite having a bit of relevant physics background in conformal field theory).
-My questions are
-
-Are the parallels alluded to above a peculiar feature of these exceptional structures, or something that holds more generally?
-Is there a "baby example" one can look at of these correspondences - something based on smaller and more elementary objects?
-
-REPLY [10 votes]: I think the analogy you describe cannot be made precise with our current technology. For example, the word "functor" doesn't seem to have made an appearance yet in this context.
-If you have a code, there are methods to construct lattices using it, but some of the constructions (like the Leech lattice) require special properties of the code. If you have a lattice, there are methods to construct vertex operator algebras using it (e.g., the lattice VOA), but some of the constructions, like orbifolds, depend on properties of the lattice, like the existence of automorphisms of certain orders. In the case of the moonshine module, we need the $-1$ automorphism of Leech, which isn't particularly special for lattices. Conjecturally (see work of Dong, Mason, and Montague 1994-95), we could use any fixed-point free automorphism of Leech to get an isomorphic VOA, and that is somewhat more special.
-One class of "baby examples" that arise is when you take the root lattice of a simple (or more generally, reductive) algebraic group. This lattice has an action of the Weyl group. The lattice vertex operator algebra of this lattice has an action of the corresponding Kac-Moody Lie algebra (more generally, I think the centrally extended loop group acts). This is one of the more natural ways to construct $E_8$ from its lattice.
-I'm afraid I'm not qualified to describe good baby examples of the transition from codes to lattices.<|endoftext|>
-TITLE: A bound on the top homology of a complement to a variety in $\mathbb C^n$
-QUESTION [6 upvotes]: Let $V$ be a subvariety of $\mathbb C^n$ with irreducible components of dimension >$0$. Is $H_{2n-1}(\mathbb C^n\setminus V)=0$?
-
-REPLY [3 votes]: The sharp bound is this: For any closed algebraic set $V$ of codimension $d$ in ${\Bbb C}^n$, with $U={\Bbb C}^n \setminus V$, one has $\pi_i(U) = 0$ for $0 < i\leq 2d-2$ and $\pi_{2d-1}(U) \neq 0$. Using the Hurewicz isomorphism, you get the same vanishing and non-vanishing for homology.
-A simple proof is in the appendix to my notes (with Fulton) on equivariant cohomology, http://www.math.washington.edu/~dandersn/eilenberg/ . A slick reason for vanishing was pointed out by David Speyer: given a (nice) map of an $i$-sphere into $U$, the (real) lines between the points in image of the sphere and points in $V$ sweep out a space of dimension at most $(2n-2d)+i+1$. When $i<2d-1$, you can pick a point in $U$ not lying on any such line, and contract your sphere down to that point. The non-vanishing happens because all algebraic sets have nontrivial fundamental classes in Borel-Moore homology. (Vanishing can also be proved using B-M homology.)<|endoftext|>
-TITLE: Probability Problem Involving e
-QUESTION [16 upvotes]: I thought of the following probability problem, which seems to have an answer of 1/e, and wonder if someone has an idea as to how to prove this.
-Suppose a man has a bottle of vitamin pills and wishes to take a half pill per day. He selects a pill from the bottle at random. If it is a whole pill he cuts it in half, takes a half pill, and puts the other half back in the bottle. If it is a half pill, he takes that. He continues this process until the bottle is empty. What is the expected maximum number of half pills in the bottle? If the bottle starts with n pills, and M is the expected maximum number of half pills, then M/n appears to tend to 1/e as n tends to infinity.
-
-REPLY [12 votes]: The problem is equivalent to the following: Suppose there are $n$ bins, and repeatedly, we throw balls which fall in one of the bins (uniformly and independently of the history). What is the maximum number of bins with exactly one ball? In the model with the pills, it is explicitly forbidden to draw a pill which has been drawn twice already, but for the current question, this clearly doesn't matter.
-We can replace discrete time with continuous time, and throw the balls at the events of a Poisson process. This way, the balls falling in a particular bin arrive according to a Poisson process, and different bins are independent. If the problem was to determine the time $t$ to maximize the expected number of bins with exactly one ball at time $t$, then the answer would clearly be to choose $t$ so that the expected number of balls in a bin is 1, and the probability of having exactly one ball would be $1/e$ (we are maximizing $xe^{-x}$).
-By the law of large numbers, the number of bins with exactly one ball will very likely be about $n/e$ at that time. To conclude that $M/n$ converges to $1/e$ in probability, it only remains to show that "exceptional times" with unusually many bins of exactly one ball are not likely to occur. I guess this can be established by quantifying the idea that if at time $t$ there are substantially more than $n/e$ bins with exactly one ball, then most likely there will continue to be so in a time interval after $t$, which is unlikely.<|endoftext|>
-TITLE: Is anything known about this braid group quotient?
-QUESTION [19 upvotes]: Let $B_n$ be the braid group on $n$ strands. As is well known, if $\sigma_i$ is the operation of crossing the string in position $i$ over the string in position $i+1$, then the elements $\sigma_1,\dots,\sigma_{n-1}$ generate $B_n$ and the relations $\sigma_i\sigma_j=\sigma_j\sigma_i$ ($|i-j|\geq 2$) and $\sigma_i\sigma_{i+1}\sigma_i=\sigma_{i+1}\sigma_i\sigma_{i+1}$ give a presentation of the group.
-I am interested in a group that can be obtained from $B_n$ by adding another set of relations. For each $k$, define $T_k$ to be the "twist" of the first $k$ strands. Geometrically, you take hold of the bottom of the first $k$ strands and rotate your hand through 360 degrees in such a way that strands to the left go over strands to the right. In terms of the generators, $T_k=(\sigma_1\dots\sigma_{k-1})^k$, since $\sigma_1\dots\sigma_{k-1}$ takes the $k$th most strand and lays it across the next $k-1$ strands. Let us also define $S_k$ to be the right-over-left twist of the strands from $k+1$ to $n$. That is, $S_k=(\sigma_{k+1}\dots\sigma_{n-1})^{-(n-k)}$. (Also, let us take $T_1$ and $S_{n-1}$ to be the identity.)
-I am interested in the group you get if you start with $B_n$ and add in the relation $T_kS_k=1$ for every $k$. Let me make some utterly trivial observations.
-If $n=2$ then we get the cyclic group $C_2$. That's because $\sigma_1$ is the only generator and $T_2S_2=\sigma_1^2$. If $n=3$ then we get $S_3$. That's because $T_1S_1=\sigma_2^{-2}$ and $T_2S_2=\sigma_1^{-2}$, so the relations we are adding are $\sigma_1^2$ and $\sigma_2^2$, and it is well known that those, together with the braid relations, give a presentation of the symmetric group.
-Beyond that I don't know what to say, though I've convinced myself (without a proof) that when $n=4$ the group is infinite: in general, it seems that the extra relations can be used to do only a limited amount of untwisting. (I do have a proof that there are pure braids that cannot be reduced to the identity once we have four strands. It's a fairly easy exercise and I won't give it here.)
-What exactly is my question? Well, I'd be interested to know whether the word problem in this group is soluble in reasonable time. In the service of that, I'd like to know whether this group is one that people have already looked at, or whether it at least belongs to a class of groups that people have already looked at. (E.g., perhaps the solubility of the word problem follows from some general theory.) And is there some nice way of characterizing the subgroup of $B_n$ that we are quotienting by? That is, which braids belong to the normal closure of the set of braids $T_kS_k$? (One way of answering this would be to characterize their normal forms.)
-The motivation for the question comes from part of an answer that Thurston gave to a question I asked about unknots. It seems to me that this question ought to be relevant to the untying of unknots, but easier.
-One final remark: the word problem in $B_n$ can be solved in polynomial time. (If my understanding is correct, this is a result of Thurston that built on work of Garside.) Since adding more relations makes more braids equal to the identity but also gives more ways of converting a word into another, it is not clear whether the problem I am asking should be easier or harder than the word problem for braid groups. However, my hunch is that it is harder (for large $n$, that is).
-
-REPLY [4 votes]: THIS ANSWER IS WRONG. See the comment by Gowers. The mapping class group of the n-punctured sphere, which the question was about, is not the same as the spherical braid group. It is obtained from the spherical braid group by quotienting out by a full twist of all the strands. The original answer is below for posterity.
-As others have pointed out, this is the braid group on S2. These groups were studied a bit during the 1960's, and then everyone seems to have forgotten all about them. The only book which seems to give braid groups over S2 more than a passing mention is Murasugi and Kurpita's monograph. Only recently does it seem that people have become interested again, because of "braid groups over surfaces" (discussed here for example) and because of the amazing Berrick-Cohen-Wong-Wu paper which connects Brunnian braids over a sphere to homotopy groups of S2! Their result is one of those mysterious alluring connections which makes mathematics worth doing. John Baez wrote a nice blog post about that story.
-It is proven by Fadell and van Buskirk that the braid group over S2 is presented by the braid generators $\sigma_1,\ldots,\sigma_n$ modulo the relations:
-
-$\sigma_i\sigma_j\sigma_i=\sigma_j\sigma_i\sigma_j$ if $\left\vert i-j\right\vert=1$.
-$\sigma_i\sigma_j=\sigma_j\sigma_i$ if $\left\vert i-j\right\vert>1$.
-$(\sigma_1\sigma_2\cdots\sigma_{n-1})(\sigma_{n-1}\cdots\sigma_2\sigma_1)=1$.
-
-Its word problem is solved in a very nice paper by Gillette and van Buskirk by modifying the combing algorithm for braids to work for braids over S2. This has exponential run-time, the same as the combing algorithm for the braid group. But first Thurston ("Word Processing in Groups"), and then Dehornoy, modified the combing algorithm to work in polynomial time, I don't know whether their algorithms can be made to work for the braid group over S2; my intuition is that the answer is surely yes, but surely nobody has bothered doing it yet, because braid groups over S2 are well forgotten.
-EDIT: This is part of Open Question 9.3.10 in Word Processing in Groups (see also Open Question 9.3.9). I don't know whether it has been solved in the two decades since that book's publication.<|endoftext|>
-TITLE: Height of algebraic numbers
-QUESTION [9 upvotes]: I would like to find effective upper bound for the height of $a+b$ and $a/b$ and $ab$ knowing the heights of $a$ and $b$. Thanks.
-
-REPLY [2 votes]: I too was looking for the answer to the same question. It seems necessary to define "height" since there are of course several variants in use. The height $h(a)$ I am interested in is the maximum of the absolute values of the coefficients of the minimal polynomial of the algebraic number $a$. Note that the first answer refers to a different notion of height, since, for example,
-$9 = h(9) = h(3 \cdot 3) \not \leq h(3) + h(3) = 3 + 3 = 6.$
-I imagine there must be upper bounds of the form
-$h(ab) \leq f(d) h(a)^{g(d)} h(b)^{g(d)}$
-for some simple functions $f$ and $g$, where $d$ is the degree of a field extension of $\mathbb{Q}$ containing both $a$ and $b$, for example.
-Are there any such results in the literature, and similarly for $h(a+b)$ and $h(a/b)$?<|endoftext|>
-TITLE: Lower bounds on the easier Waring problem
-QUESTION [18 upvotes]: The easier Waring problem asks for the least number $v=v(k)$ such that every every integer is a sum of $v$ $k$'th powers with signs, i.e. every $n\in \mathbb{N}$ is of the form $$n=x_1^k\pm x_2^k\pm\dotsb\pm x_v^k.$$
-The problem is ``easier'' because unlike the usual Waring problem (without the signs) the existence of $v(k)$ is easy --- the bound $v(k)\leq 2^{k-1}+\tfrac{1}{2}k!$ follows from the repeated differencing. Of course, the upper bounds on the usual Waring problem apply, and so fact $v(k)=O(k\log k)$.
-All the lower bounds on $v(k)$ I have seen come from the congruence considerations. For example, $v(3)\geq 4$ because we need at least four terms modulo $9$. However, if we discard the congruential obstacles is there a non-trivial lower bound? To put bluntly my question is
-
-Is there $k$ large enough so that the set $\{x_1^k\pm x_2^k\pm x_3^k\pm x_4^k\pm x_5^k\}$has zero density?
-
-REPLY [19 votes]: As far as I am aware, nothing unconditional is known. The difficulty is that one has no obvious constraint on the size of the variables, so that the $x_i$ could be arbitrarily large in terms of $n$ in a solution. The difficulty of ruling out solutions (when congruence conditions do not rule out solubility) is related to proving insolubility of generalised Fermat equations.
-However, there is a conditional approach assuming the truth of the generalised ABC Conjecture (or, as Pomerance describes it, the Alphabet Conjecture). Let's stick with the general situation with $v$ summands. Generalised ABC asserts that in any solution of
-$a_1+\ldots +a_s=0$ in which there are no vanishing subsums, one has
-$\underset{1\le i\le s}{\max}|a_i|\ll_{s,\epsilon} \left( \prod_{p|a_1\ldots a_s}p\right)^{(s-1)(s-2)/2+\epsilon}.$ There is disagreement about the specific exponent, but this does not matter so much in the conclusion (see a 1986 paper of Brownawell and Masser for a function field version, and I discuss this in a 1994 paper on Quasi-diagonal behaviour). The vanishing subsums in your equation $\pm x_1^k+\ldots +\pm x_v^k=n$ make life easier (treat them separately, and more powerful estimates are possible), so for simplicity suppose that the representations all have no vanishing subsums. Then one obtains $|x_i|^k\ll |nx_1\ldots x_v|^{v(v-1)/2+\epsilon}$. Provided that $k>v^2(v-1)/2$, it follows that $\max |x_i|\ll |n|^{\alpha+\epsilon}$, where $\alpha=v(v-1)/(2k-v^2(v-1))$. OK ... so far so good. What we have shown thus far is that the variables in a representation are bounded by $|n|^{\alpha +\epsilon}$. The total number of variables available to represent the integers $n$ between $N/2$ and $N$ is consequently no larger than a quantity which is $\ll (N^{\alpha+\epsilon})^v$ (there were $v$ variables). Whenever $v(\alpha+\epsilon)<1$, therefore, the set of integers represented must have zero density. If I have not made any computational errors along the way, this leads us to the conclusion that whenever $k>v^2(v-1)$, then the density of integers represented in the easier Waring problem will be zero. (But remember that this is all conditional on the Generalised ABC Conjecture.) For the specific problem with $v=5$, it looks as if $k>100$ conjecturally does the trick (though smaller $k$ should surely also work).<|endoftext|>
-TITLE: When are these rings regular?
-QUESTION [5 upvotes]: Let $R$ be a noetherian regular domain. Suppose that $a, b \in R$, with $b \neq 0$, and consider the ring $S:=R[\frac{a}{b}]=R[X]/(bX-a)$. Is $S$ regular? If this is not the case are there some conditions on $a$ and $b$ (or on $R$) that imply regularity? For example $a=1$ is enough.
-
-REPLY [3 votes]: If $R = k[x,y]$ and we throw in $\frac {x^2}y$, we get $k[x,y,z]/(zy-x^2)$, not regular. This shows that it's not enough to assume $a,b$ form a regular sequence. The only sufficient condition I can come up with is that $a$ be outside the square of (every/the) maximal ideal<|endoftext|>
-TITLE: Embedding $S_3$ into $Aut(F_2)$
-QUESTION [7 upvotes]: Consider the two (inequivalent) $\mathbb{Z}$-representations $\phi,\psi$ of the symmetric group $S=S_3$ given by
-$(1,2)^\phi=\left(\begin{array}{rr}0 &-1\\\ -1 & 0\end{array}\right), \qquad
-(1,2,3)^\phi=\left(\begin{array}{rr}0 &1\\\ -1 & -1\end{array}\right);$
-$(1,2)^\psi=\left(\begin{array}{rr}0 &1\\\ 1 & 0\end{array}\right), \qquad
-(1,2,3)^\psi=\left(\begin{array}{rr}0 &1\\\ -1 & -1\end{array}\right).$
-Now, let $F=\langle x,y\rangle$ be a free 2-generated group. The representation $\phi$ can be "lifted" to an embedding $\tau:S\to\rm{Aut}(F)$ as follows:
-$(1,2)^\tau=[x\mapsto y^{-1};\quad y\mapsto x^{-1}], \qquad
-(1,2,3)^\tau=[x\mapsto y;\quad y\mapsto x^{-1}y^{-1}].$
-Question. Can one similarly lift $\psi$?
-Remark 1. By "lifting" a representation $\phi:S\to\rm{GL}_2(\mathbb{Z})$ I mean finding an embedding $\tau:S\to\rm{Aut}(F)$ such that $\phi=\tau\alpha$, where $\alpha:\rm{Aut}(F)\to\rm{GL}_2(\mathbb{Z})$ is the natural epimorphism.
-Remark 2. A naïve attempt to send
-$(1,2)\ \mapsto\ [x\mapsto y;\quad y\mapsto x], \qquad
-(1,2,3)\ \mapsto\ [x\mapsto y;\quad y\mapsto x^{-1}y^{-1}]$
-does not give a lifting of $\psi$.
-
-REPLY [8 votes]: As Tom Goodwillie noted in his comment, $GL(2,\Bbb Z)$ can be identified with $Out(F_2)$, so the question can be rephrased in terms of lifting subgroups of $Out(F_2)$ to $Aut(F_2)$. There is a Realization Theorem for finite subgroups of $Aut(F_n)$ and $Out(F_n)$ which says that such a subgroup can always be realized as a group of symmetries of some finite connected graph with fundamental group $F_n$, where the symmetries fix a basepoint in the graph in the case of $Aut(F_n)$. When $n=2$ there are only two graphs to consider, and the relevant one for $S_3$ is the join of two points with three points. This has two symmetry groups isomorphic to $S_3$, but only one of these two groups fixes a basepoint, so this should answer the question.
-The Realization Theorem is discussed in Karen Vogtmann's survey paper "Automorphism groups of free groups and outer space", section II.6. The references given there are to papers by M. Culler, B. Zimmermann, and D. G. Khramtsov from 1981 to 1984.<|endoftext|>
-TITLE: Finite graphs that realize all types over $n$-element sets
-QUESTION [6 upvotes]: Call a graph $G$ $n$-saturated if for every set $A$ of size $n$ of vertices and all $B\subseteq A$ there is a vertex $v\not\in A$ that forms an edge with all $w\in B$ and
-does not form an edge with any $w\in A\setminus B$.
-A countably infinite graph is isomorphic to the random graph iff it is $n$-saturated for all $n$.
-Here are the questions:
-
-Is it true that for all $n$ there is a finite graph (of size at least $n$) that is $n$-saturated?
-If yes, are there reasonable upper and lower bounds on the size of such a graph?
-(An $n$-saturated graph of size $\geq n$ has at least $n+2^n$ vertices, but this is probably far from optimal.)
-Is every finite graph an induced subgraph of a finite $n$-saturated graph?
-(Edit: As Ori Gurel-Gurevich pointed out, the second question is silly. Clearly, if $G$ is $n$-saturated (and of size at least $n$), then it has every graph with $n$-vertices as an induced subgraph.)
-
-Example: There is a $2$-saturated graph: Let the vertices of $G$ be the $2$-element subsets of a set with $6$ elements. Two of those vertices form an edge if they (as sets) have a non-empty intersection. It is easily checked that this graph is $2$-saturated.
-But it also has 30 vertices. That seems a lot.
-
-REPLY [7 votes]: Ben Rossman found a nice construction of $n$-saturated graphs; I adjusted it a bit and wrote it up. See http://research.nii.ac.jp/~rossman/k-ec.pdf . Set theorists like Stefan might enjoy the fact that it's reminiscent of Hausdorff's construction of independent families of (infinite) sets.<|endoftext|>
-TITLE: Non-vanishing of group cohomology in sufficiently high degree
-QUESTION [21 upvotes]: Atiyah in his famous paper , Characters and cohomology of finite groups, after proving completion of representation ring in augmentation ideal is the same as $ K(BG)$, gives bunch of corollaries of this main theorem. One of them that catches my interest is: For any finite non-trivial group $G$ there exists arbitrary large integer $n$ such that $H^n(G,\mathbb{Z})\neq 0 $. I just wonder if anyone can prove this without this powerful theorem.
-
-REPLY [11 votes]: The first purely algebraic proof of this fact seems to be from Leonard Evens:
-
-A Generalization of the Transfer Map in the Cohomology of Groups, Trans. Amer. Math. Soc. 108(1963), 54-65 [Theorem 3]
-
-where he proves the result with help of his norm map. After having established the basic properties of the norm map, the proof is rather elementary: Let $C$ be a cyclic subgroup of prime order of $G$ and let $x$ be a generator of $H^2(C,\mathbb{Z})$. Then the powers of $y = N^G_C(x)$ yield non-trivial cohomology classes of $H^*(G,\mathbb{Z})$ in degrees divisible by $(G:C)$.
-From a historical point of view the norm map already occured in disguise in Evens´ paper
-
-The Cohomology Ring of a Finite Group, Trans. Amer. Math. Soc. 101(1961), 224-239
-
-where he proves finte generation of the cohomology ring.<|endoftext|>
-TITLE: Realizing the diameter of a finite regular graph
-QUESTION [5 upvotes]: Let $X=(V,E)$ be a finite, connected, regular graph with diameter $D$. Is it true that, for every $x\in V$, there exists $y\in V$ such that $d(x,y)=D$? (the answer is clearly yes if $X$ is vertex-transitive).
-
-REPLY [6 votes]: Counter-example http://www.freeimagehosting.net/uploads/9a4165cab4.png
-The diameter is 8, but 1 is centered with at most 5 as distance to every other.<|endoftext|>
-TITLE: Smooth proof of Reidemeister theorem
-QUESTION [8 upvotes]: In another post Ryan Budney has mentioned a "smooth proof" of the theorem of Reidemeister...
-(@Ryan Budney:) Do you know if there is a book (or paper) containing such a proof? I am interested in and can't find one...
-Thank you very much,
-sincerely,
-Johannes Renkl
-
-REPLY [12 votes]: I don't believe it's written up anywhere.
-edit: in the comments Charlie Frohman corrects me:
-
-MR2128054 (2005m:57041)
- Roseman, Dennis(1-IA)
- Elementary moves for higher dimensional knots. (English summary)
- Fund. Math. 184 (2004), 291–310.
- 57R40 (57R45 57R52) DOI: 10.4064/fm184-0-16, eudml
-
-has a proof similar in flavour to the sketch below. Thanks Charlie!
-
-The idea is pretty simple, but maybe a bit of a pain to write up completely. Say $f : [0,1] \times S^1 \to \mathbb R^3$ is a smooth isotopy. Let $\pi : \mathbb R^3 \to \mathbb R^2$ be orthogonal projection onto a 2-dimensional subspace. $\pi \circ f : [0,1] \times S^1 \to \mathbb R^2$ is a 1-parameter family of (possibly singular) knot diagrams, but we can assume that at the initial and terminal parts of the family are regular knot diagrams.
-The next step is to check the co-dimensions of the various types of singularities such maps can have.
-For example, a knot has its derivative. That derivative is "vertical" is a co-dimension 2 condition on a vector. Since a knot is 1-dimensional, it's a co-dimension 1 condition on the knot. Such a vertical derivative for $f$ results in its projection to $\mathbb R^2$ to have a cusp. So a generic knot diagram has no cusps, but a 1-parameter family will have a finite number of cusps, and these appear as Reidemeister moves of "type 1". Technically, this is a formulation of Whitney's theorem that maps between 2-manifolds generically only have fold and cusp-type singularities.
-Further, you can ask about double, triple and multiple-points for the projections, non-transverse double points, and so on. Double points are generic but triple-points are co-dimension 2, and $n$-tuple points are co-dimension $2(n-2)$ in general, so you generically can avoid anything worse than triple points. This gives the Reidemeister "type 3" moves. Tangential double points are also co-dimension 2, and generic such ones produce Reidemeister "type 2" moves.
-As you can see this isn't quite a full proof using only the technology of Guillemin and Pollack, meaning it's not quite just Sard's theorem that we're using. We're really using the "Multijet transversality" type theorem, like Theorem 4.13 from Golubitsky and Guillemin. But I suspect like Whitney's work "The general type of singularity of a set of $2n-1$ functions of $n$ variables" you should be able to massage the above into a solid argument without dredging up too much formalism.
-edit: I do think there is likely a "smart" proof of this that avoids jet transversality. For example, check out Milnor's proof that Morse functions are dense in the space of smooth functions $M \to \mathbb R$ (in Milnor's Morse Theory text). You'd think in principle you'd have to use jet transversality for this but by picking a diverse-enough family of functions to start with, he pulls it all back to the level of Sard's Theorem. I think there should be a similarly slick proof of Reidemeister's theorem, using only Sard's Theorem.<|endoftext|>
-TITLE: Smooth structures compatible with a given C^1 structures
-QUESTION [10 upvotes]: On a manifold equipped with C^k atlas (with k>0) there is essentially one smooth structure compatible with the atlas. According to Wikipedia, this is a result due to Whitney. This is in stark contrast with a C^0 atlas, where there might exist many smooth structures or none at all.
-I was wondering, what is the underlying reason? What makes once-differentiable functions so much better behaved in terms of finding a smooth atlas?
-There are many cases where C^1 makes a world of difference - for example, convergence of Fourier series, but maybe there is some geometric explanation?
-Also according to Wikipedia, on the long line (not technically a manifold) there are infinitely many smooth structures all compatible with a given C^k structure, so perhaps there is some topology involved...
-
-REPLY [15 votes]: See my answer to this question.
-Consider the space (in some appropriate sense ) of all invertible germs of $C^{\infty}$ maps from $\mathbb R^n$ to itself fixing the origin. This is homotopy equivalent to $GL_n(\mathbb R)$: we can deform the smooth map $f$ to its linear approximation by going through $f_t(x)=\frac{f(tx)}{t}$ as $t$ goes to $0$. The same applies to invertible $C^k$ germs for finite $k>0$, but not to invertible $C^0$ germs. (All of this is equally true for global diffeomorphisms/homeomorphisms $\mathbb R^n\to\mathbb R^n$, too. )<|endoftext|>
-TITLE: How should one think about pushforward in cohomology?
-QUESTION [47 upvotes]: Suppose f:X→Y. If I decorate that first sentence with appropriate adjectives, then I get a pushforward map in cohomology H*(X)→H*(Y).
-For example, suppose that X and Y are oriented manifolds, and f is a submersion. Then such a pushforward map exists. In the de Rham picture, we can see this as integrating a form over fibres. In the sheaf cohomology picture, we can see this via the explication of the exceptional inverse image functor.
-The question is how else can we think of this pushforward map. I'd be particularly interested in an answer from the algebraic topology point of view, because I'm hoping that such an answer would eludicate the appropriate level of generality in which a pushforward in cohomology exists (perhaps not only answering the question of for which maps f, but also answering the question of in which cohomology theories can we carry out such a construction).
-
-REPLY [9 votes]: Parallel to the OP's two examples, if a cohomology class is defined through intersection with a submanifold (or subvariety with fundamental class in locally finite homology) then the pushforward is defined through intersection with the image of this subvariety. Note how immediate it is to see the change in codimension ( while codimension is constant when taking preimage, which corresponds to the natural map).
-As others have said, collapse maps can be used to define pushforwards in general. But having models for pushforwards counts towards understanding a particular cohomology theory geometrically. So for example if you can find a good model for the pushforward of tmf then "you'll win a prize"<|endoftext|>
-TITLE: Existence of a nice subset of edges in $k-$regular simple graphs?
-QUESTION [6 upvotes]: Let $G=(V,E)$ be a finite simple $k-$regular graph ($k\geq 1$). Does $G$ necessarily
-contain a subset $E'\subset E$ of edges such that only isolated edges and cycles occur as connected components in $(V,E')$?
-(The answer is easily yes for $k=1,2$.)
-A counterexample would easily give a counterexample to question "Antipodal" maps on regular graphs? in the case $D=2$ by considering
-the complementary graph of $G$ (respectively of two disjoint copies of $G$ if $G$ is
-"too small").
-
-REPLY [6 votes]: It seems like such a subset should always exist.
-Consider the bipartite graph on $2|V(G)|$ vertices corresponding to the adjacency matrix of $G$. Since this graph is regular, by Hall's Theorem it has a perfect matching. In terms of the original $G$, this corresponds to a permutation $\sigma$ on $V(G)$ such that $v$ and $\sigma(v)$ are always adjacent. The cycles of $\sigma$ would then give you the desired decomposition.<|endoftext|>
-TITLE: Flatness of normalization
-QUESTION [18 upvotes]: Let $X$ be a noetherian integral scheme and let $f \colon X' \to X$ be the normalization morphism. It is known that, if non trivial, $f$ is never flat (see Liu, example 4.3.5).
-What happens if we suppose $X$ normal, and we take the normalization in a finite (separable) extension of the function field of $X$? Note that in the easiest case, namely $X=\rm{Spec}(R)$, with $R$ a Dedekind domain, we have that $f$ is flat.
-
-REPLY [4 votes]: If $f: X' \to X$ is flat, then $X$ tends to inherit nice properties of $X'$ (for example, being regular). So if you arrange for $X'$ to be "nicer" than $X$, as in David and one of Karl's examples, then $f$ can't be flat.
-Here is a quick and dirty proof when "nice" = "regular". The claim is that if $R\to S $ is a finite flat local homomorphism of Noetherian local rings and $S$ is regular, then $R$ is regular as well.
-Let $m$ be the maximal ideals of $R$. Then as $S$ is regular, $S/mS$ has finite flat dimension (in fact, projective dim) over $S$. But $S$ is flat over $R$, so $S/mS$ has finite flat dimension over $R$. But as $R$-modules, $S/mS$ is direct sum of copies of $k=R/m$, so $k$ has finite flat dimension over $R$, which characterizes regularity.<|endoftext|>
-TITLE: Rolling-ball game
-QUESTION [23 upvotes]: The analyses
-in two recent MO questions
-("recent" with respect to the original posting in 2011),
-"Rolling a random walk on a sphere"
-and
-"Maneuvering with limited moves on $S^2$,"
-suggest a Rolling-Ball Game, as follows.
-A unit-radius ball sits on a grid point of
-a $\delta \times \delta$ regular grid in the plane,
-with $\delta \neq \pi/2$.
-Player 1 (Blue) rolls the ball to an adjacent
-grid point, and the track of the ball-plane contact point
-is drawn on the ball's surface.
-Player 2 (Red) rolls to an adjacent grid point.
-The two players alternate until each possible
-next move would cause the trace-path to touch itself,
-at which stage the player who last moved wins.
-In the following example, Red wins, as Blue cannot
-move without the path self-intersecting.
-
-
-
-
-Q1.
-What is the shortest possible game, assuming the players cooperate
-to end it as quickly as possible?
-For $\delta=\pi/4$, the above example suggests 6, but
-this min depends on $\delta$. It seems smaller $\delta$ need 8 moves to create
-a cul-de-sac?
-Q2.
-What is the longest possible game, assuming the players cooperate
-to extend it as much as possible?
-Q3.
-Is there any reasonable strategy if the players are truly competing
-(as opposed to cooperating)?
-Addendum.
-We must have $\delta < 2 \pi$ to have
-even one legal move,
-and the first player wins immediately with one move for $\pi \le \delta < 2\pi$ (left below).
-
-The right image just shows a non-intersecting path of no particular significance for $\delta=\pi/8$.
-
-REPLY [7 votes]: For testing potential answers to Q3, let me suggest http://www.math.chalmers.se/~wastlund/Quirks/Game.html.
-The appearance as well as the specific set of bugs might depend on your browser, but as I hope will be clear, the length of the longest path as a function of the angle $\delta$ is quite funny (this is part of the reason I spent too much time on this problem, although I'm also interested in similar path-forming games for more "serious" reasons).
-After drawing a couple of quick sketches I realized I wasn't even able to figure out the behavior when $\delta$ approaches $\pi$ from below. As it turns out, the number of edges in the longest path is 5 throughout the interval $3\pi/4 \leq \delta < \pi$. For $\pi/2 < \delta < 3\pi/4$ it's 7 (although the game-tree changes also at $2\pi/3$). At $\delta = \pi/2$ it has an isolated local minimum of 5, and for angles just smaller than $\pi/2$, it seems to jump to 23.
-Here's why I couldn't let go of this problem: In all the cases where I can visualize the entire game-tree, which is when $\delta\geq \pi/2$,
-(1) Alice, who draws the first arc, wins the game-version.
-(2) Bob, while losing, can force Alice into a maximal-length path. In other words, Alice cannot force a win in fewer moves than the length of the longest path. This property holds for some similar path-forming games where there are explicit winning strategies.
-(3) As a consequence of (1) and (2), the length of the longest path is always odd.
-A couple of other observations: If we allow players to cross their own edges but not the opponent's, then an edge is never a liability, and consequently Alice has a non-losing strategy. If on the other hand we allow players to cross the opponent's edges but not their own, then an extra edge cannot be an advantage, and now Bob has a non-losing strategy (two more questions arise here: are these games too necessarily finite, and can the (winning?) strategies be made explicit rather than just exhibited by strategy-stealing?).
-Therefore I thought for a while that I was on to something, and that there might be a beautiful reason that Alice must win.
-So I let Maple analyze the game for some suitably chosen angles, working symbolically to get reliable results. This is feasible when $\delta$ is such that the ring $\mathbb{Z}[\cos\delta, \sin\delta]$ where the coordinates of the points lie, is either a sub-ring of $\mathbb{Q}$ or of some nice algebraic field (degree 2 or 4).
-To summarize, I found counter-examples to each of (1), (2) and (3): For $\delta = \arctan(24/7)$, Bob wins at move 16 but the longest path has length 23. For $\delta = \arctan(12/5)$, the longest path has length 24 (but Alice can force a win in 13). Moreover, for $\delta=\pi/3$ there is a path of length 29 but Alice wins in 17. For $\delta=\arctan(4/3)$, Bob wins in 16 (I don't know the length of the longest path).
-Addendum. (by J.O'Rourke). I took the liberty of adding an image of
-Johan's $\pi/3$ longest path from his applet, as detailed in his comment below.
-Note the near miss where the 28th segment just misses the 1st segment.<|endoftext|>
-TITLE: Number of graphs with a given number of nodes, edges and triangles
-QUESTION [8 upvotes]: Hi. Does anyone know if it is possible to enumerate the set of labeled/unlabeled graphs (loopless, undirected, only one edge between pairs of nodes) having a given number of nodes, edges and triangles? what about including more information, like number of tetrahedra, etc.? If not, why?
-The solution to the unlabeled case with given number of nodes and edges, and to many other enumeration problems can be given in terms of Polya's theorem (by Harary, de Bruijn, Robinson, Read, Polya, etc.). Is it possible to give the resulting generating functions an interpretation in terms of order theory and use this to study their algebraic properties?
-Are these generating function methods actually useful for computing these numbers when the graphs are 'large'? Thanks!
-Update: Here is the list:
-http://www.win.tue.nl/~aeb/graphs/cospectral/triangles.html
-I haven't seen it in the OEIS, though.
-Does it get any easy if we wanted to enumerate, instead, the set of graphs on given number of vertices and trangles -irrespective- of the number of edges?
-
-REPLY [6 votes]: Polya theory provides us with an algorithm that allows us to compute the number of isomorphism classes of graphs with $n$ vertices and $m$ edges, for given $n$ and $m$. It does not provide a formula in terms of $n$ and $m$ and it does not even provide a generating function. (Well, we can express it in terms of so-called cycle index, but that is just giving a name to the unknown.)
-If now we ask for the number of graphs in terms of $n$, $m$ and the number of triangles then we are lost. I have never seen an enumeration of triangle-free graphs, let alone triangle-free graphs with a given number of edges. And if we cannot count the graphs in a given class, counting isomorphism classes of graphs in the class is usually beyond us as well.
-So counting $K_4$'s as well seems even more hopeless.
-In the cases where Polya's method works, it can provide asymptotic information for large $n$, but not formulas. So for large $n$ we know that the number of isomorphism classes
-of graphs on $n$ vertices is asymptotic to $2^n/n!$, but there is no exact formula.
-And why have these enumeration problems not been solved? Perhaps they are too hard, or we're
-too stupid. (Me, at least.)<|endoftext|>
-TITLE: Is there a 'best' name for a group together with a set it acts on?
-QUESTION [5 upvotes]: This is a question of terminology. I want to talk about the category whose...
-
-...objects are pairs $(G,M)$, where $G$ is a group and $M$ is a $G$-set.
-...morphisms $(G,M)\rightarrow (G',M')$ are pairs $(f_G,f_M)$, where $f_G:G\rightarrow G'$ is a group homomorphism, and $f_M:M\rightarrow M'$ is a set map such that
-$$ f_M(g\cdot m) = f_G(g)\cdot f_M(m)$$
-for all $g\in G$ and $m\in M$.
-
-I've been calling these decorated groups (and their morphisms), since they've been arising in connection with decorated local systems. However, I'd prefer a more standard name, hopefully one which evokes the correct idea before explanation.
-
-REPLY [2 votes]: Angelo's answer above is clearly the correct one. But, somewhat tongue-in-cheek, I would also like to recommend "The category of trivialized groupoids". As I'm sure you're very aware, there's a pretty good analogy
-
-vector bundles : trivialized vector bundles :: groupoids : group actions.<|endoftext|>
-TITLE: Identifying the generating function $ G(a,z) = \sum_{n=0}^{\infty} a^n z^{(n+1)(n+2)/2}. $
-QUESTION [10 upvotes]: I have computed a generating function for a problem involving a particular series, and would like to know if anyone has any references or a categorisation for it? It's
-$$
-G(a,z) = \sum_{n=0}^{\infty} a^n z^{(n+1)(n+2)/2}.
-$$
-It appears to be related to (mock) theta functions, but seems to be simpler.
-In particular, I would like to know whether $G(a,z)$ satisfies any identities?
-Many thanks.
-
-REPLY [5 votes]: Your generating function is related to a simple continued fraction expansion due to Touchard:
-$\sum\limits_{k \ge 0} ( - 1)^k q^{k+1\choose2} v^k $ =$ \frac{1}{{1 + v - \frac{{(1 - q)v}}{{1 + v - \frac{{(1 - q^2 )v}}{ \cdots }}}}}.$
-A simple proof can be found in a paper by H. Prodinger
- http://de.arxiv.org/abs/1102.5186<|endoftext|>
-TITLE: space of homotopy equivalences of $S^1$
-QUESTION [5 upvotes]: Does the space of homotopy equivalences of $S^1$ deformation retract onto the space of homeomorphisms of $S^1$? If so, does anyone have a reference?
-I found that Kneser proved that $Homeo(S^1)$ deformation retracts onto $O(2)$ and $Homeo^+(S^1)$ deformation retracts onto $SO(2)$ (orientation preserving homeos deformation retracts onto rotations). I'd like the space $HE^+(S^1)$ of degree 1 homotopy equivlances of $S^1$ to deformation retract onto these. The space I'm calling $HE^+(S^1)$ may go by $HomEq(S^1)$ or $SG_n$ and seems to be of interest to homotopy theorists for higher $n$.
-
-REPLY [3 votes]: I think Ryan Budney's comment can be made to work. In order to take a straight line homotopy, you need to figure out which rotation you are going to homotope to, and make this choice in a continuous fashion. Here's one possible choice. For a homotopy equivalence $f:S^1\to S^1$, take a lift $F:\mathbb{R}\to\mathbb{R}$. Then $F$ has the property that $F(x+n)=F(x)+n, n\in \mathbb{Z}$ ($\mathbb{Z}$-equivariant). The function $F(x)-x$ is therefore periodic, and attains a minimal value $m(f)$. Take the straightline homotopy from $F(x)$ to $x+m(f)$. This homotopes the continuous $\mathbb{Z}$-equivariant functions to the functions of the form $x+r, r\in\mathbb{R}$, and therefore descends to a homotopy of $HE^+(S^1)$ to $SO(2)$. I think it's clear that this homotopy is continuous with respect to the topology on $HE^+(S^1)$.
-
-REPLY [3 votes]: Put $$HE^+_1(S^1)=\{f\in HE^+(S^1):f(1)=1\}. $$
-There is an evident homeomorphism $m:S^1\times HE_1^+(S^1)\to HE^+(S^1)$ given by $m(z,f)(x)=z f(x)$, and this restricts to give a homeomorphism $S^1\times Homeo_1^+(S^1)\to Homeo^+(S^1)$.
-It will thus suffice to discuss $HE_1^+(S^1)$ and $Homeo_1^+(S^1)$.
-Next, define $e:\mathbb{R}\to S^1$ by $e(t)=\exp(2\pi i t)$. Let $X$ denote the space of maps $$ u:[0,1]\to\mathbb{R} $$
-with $u(0)=0$ and $u(1)=1$, and let $Y$ be the subspace of strictly increasing maps. For any $u\in X$ there is a unique map $p(u):S^1\to S^1$ with $p(u)(e(t))=e(u(t))$ for all $t\in [0,1]$. This construction gives a homeomorphism $p:X\to HE^+_1(S^1)$, and restricts to a homeomorphism $p:Y\to Homeo_1^+(S^1)$. This is a fairly straightforward exercise with covering space theory and topologies on mapping spaces. Now $X$ and $Y$ are both convex, so they have obvious contractions to the identity map given by
-$$ h(t,u)(x) = tx+(1-t)u(x). $$
-It is not hard to see that $Y$ is not closed in $X$, so it cannot be a retract, so $Homeo_1^+(S^1)$ is not a retract of $HE_1^+(S^1)$.<|endoftext|>
-TITLE: Asymptotic Formula for a Mertens Style Sum
-QUESTION [7 upvotes]: Hello,
-I am wondering if there is a simple asymptotic formula for
-$$\sum_{p\leq x}\frac{\left(\log p\right)^{k}}{p},$$
-where $k\geq0$ is some integer. If $k$ is $0,$ by using the Prime Number Theorem we have
-$$\sum_{p\leq x}\frac{1}{p}=\log \log x+b+O\left(e^{-c\sqrt{\log x}}\right).$$
-Similarly, the prime number theorem and integration by parts solves the case $k=1$ and gives
-$$\sum_{p\leq x}\frac{\left(\log p\right)}{p}=\log x+C+O\left(e^{-c\sqrt{\log x}}\right).$$
-My question is do these integrals have a nice asymptotic formula for every $k$? Specifically, I mean with an error term of the form $O\left(e^{-c\sqrt{\log x}}\right).$
-Thanks!
-Remark: This question is related, and in particular if it is solved with a nice enough asymptotic, then so is this. (but not vice versa)
-
-REPLY [6 votes]: Here is an answer that is similar in spirit to Frank and Peter's answers, but possibly simpler.
-Summing by parts, we see that
-$$ \sum_{p\le x} \frac{\log^k p}{p} = (\log x)^{k-1} \sum_{p\leq x} \frac{\log p}{p} -(k-1)\int_{2^-}^x (\log u)^{k-2}\sum_{p\le u} \frac{\log p}{p} \frac{du}{u}.$$
-Now use the formula
-$$ \sum_{p\leq x} \frac{\log p}{p} = \log x + c_1 + O(\exp(-c_2\sqrt{\log x}))$$
-and it is not hard to derive that
-$$ \sum_{p\le x} \frac{\log^k p}{p} = \frac{\log^k x}{k} + c_3 + O(\exp(-c_4\sqrt{\log x})).$$
-This seems easier than dealing with $Li(x)$.<|endoftext|>
-TITLE: semisimplicity of p-adic Galois representations
-QUESTION [9 upvotes]: Is it true that all continuous finite dimensional $p$-adic representations of $Gal(\bar{K}/K)$ are semisimple, where $K$ is a number field, i.e. if $\rho:Gal(\bar{K}/K) \mapsto GL_n(\bar{\mathbb{Q}}_p)$ is a continuous representation, where $K$ is a number field (even $\mathbb{Q}$ if you want), is it semisimple?
-Replacing $\mathbb{Q}_p$ with $\mathbb{C}$, the result is indeed true, using the existence of an invariant Haar measure over the complex numbers. So the natural question (related) is if it exists a $\mathbb{Q}_p$-valued Haar measure in a compact group? Of course some conditions must be removed from the classical formulation (for example it does not make sence to ask that if $f$ is a positive continuous function, then its integral is positive), but the existence of a non-trivial invariant measure such that the volume of the group is 1 should be enough. Any reference is welcome as well
-
-REPLY [8 votes]: Well, actually, the "motivic Haar measure" LSpice refers to is an analogue of Haar measure
-that lives on ${\operatorname{GL}}_n({\mathbb C}((t)))$, not on ${\operatorname{GL}}_n({\mathbb Q}_p)$, and takes values in the Grothendieck ring of varieties, so I think it's not quite relevant here. What is closer to this discussion though is the fact that the usual Haar mesaure on ${\operatorname{GL}}_n({\mathbb Q}_p)$ if it's reasonably normalized (e.g. so that the volume of ${\operatorname{GL}}_n({\mathbb Z}_p)$
-is $1$), actually takes values in $\mathbb Q$ on all reasonable sets that you ever want to consider. More precisely, one can define a sigma-algebra of the so-called "definable sets", and the volumes of definable sets just are in $\mathbb Q$. In this sense they are in
-$\mathbb Q_p$ already, so the trick is that for these sets you do not need any completion of $\mathbb Q$ in order to define their volumes, and so you do not need to worry about using the $p$-adic metric... Most sets one works with turn out to be automatically definable, so this fact may be handy in some other situation.
-Added some hours later: It was my first post on mathoverflow, and I am still not allowed to add comments to others' posts :) -- so this should be a comment to the comment by LSpice that appears in Emerton's post.
-Talking about measure on $\operatorname{GL}_n(\overline{{\mathbb Q}_p})$, there is a paper by E.Hrushovski and D. Kazhdan http://arxiv.org/abs/math/0510133 that talks about integration in algebraically closed valued fields (using logic). As a first approximation, as far as I understand, the values of this measure are something like equivalence classes of definable sets over the residue field (I am certainly being imprecise here). There are several papers by Yimu Yin (the ones to start with are http://arxiv.org/abs/0809.0473, and http://arxiv.org/abs/1006.2467) aimed at clarifying this fundamental work of Hrushovski and Kazhdan in a slightly simplified setting. Unfortunately, I do not know of any non-technical introductory paper about this. There is a short note by Moshe Kamenski http://www.nd.edu/~mkamensk/lectures/motivic.pdf -- maybe this is the best place to start. Also, I hope someone corrects me here if I made any errors in this description.<|endoftext|>
-TITLE: Cohomology of the infinite loop space of the affine grassmanian (as in the generalized Mumford conjecture)
-QUESTION [6 upvotes]: I've been reading Hatcher's survey "A short exposition of the Madsen-Weiss theorem". In it, he outlines a nice proof of the "generalized Mumford conjecture", which asserts that the stable cohomology of the mapping class group is the same as the cohomology of $\Omega^{\infty} AG_{\infty,2}^{+}$. Here $AG_{n,m}$ is the space of affine $m$-planes in $\mathbb{R}^n$ and the "plus" sign indicates the 1-point compactification.
-Now, the Mumford conjecture I know and love asserts that the (rational) stable cohomology ring of the mapping class group is $\mathbb{Q}[e_1,e_2,\ldots]$, where the $e_i$ are the MMM classes.
-Can anyone explain to me why the rational cohomology ring of $\Omega^{\infty} AG_{\infty,2}^{+}$ is a polynomial ring with 1 generator in each even dimension?
-This appears to be explained in Madsen-Tillmann's paper introducing the generalized Mumford conjecture, but that paper is rather formidable and I have been unable to extract an answer to the above question from it (indeed, it doesn't really appear to be talking about the affine Grassmannian at all!).
-
-REPLY [7 votes]: Most of this is not special to the case of $AG_{\infty,2}^+$. For any spectrum $X$, we have a Hurewicz map $h:\pi_{\ast}(X)\to H_{\ast}(X)$, which induces a map $h':\mathbb{Q}\otimes\pi_{\ast}(X)\to\mathbb{Q}\otimes H_{\ast}(X)$. Standard calculations show that $h'$ this is an isomorphism when $X=S^n$ for some $n$, and it follows by induction up the skeleta that $h'$ is an isomorphism for all $X$. Next, the homotopy groups $\pi_{\ast}(X)$ are (essentially by definition) the same as the homotopy groups of the space $\Omega^\infty(X)$, so we have an unstable Hurewicz map $h'':\pi_{\ast}(X)=\pi_{\ast}(\Omega^\infty(X))\to H_{\ast}(\Omega^\infty(X))$. By combinining these we get a map $\mathbb{Q}\otimes H_\ast(X)\to \mathbb{Q}\otimes H_\ast(\Omega^\infty(X))$. Next, every infinite loop space is a homotopy-commutative H-space, and this makes $H_\ast(\Omega^\infty(X))$ into a graded-commutative (and graded-cocommutative) Hopf algebra. Now let $A_*$ be the free graded-commutative ring generated by $\mathbb{Q}\otimes H_\ast(X)$, with the Hopf algebra structure for which the generators are primitive. More explicitly, $A_\ast$ is a tensor product of polynomial algebras (one for each even-dimensional generator in $\mathbb{Q}\otimes H_\ast(X)$) and exterior algebras (one for each odd-dimensional generator). It is now quite formal to construct a canonical map $A_\ast\to\mathbb{Q}\otimes H_\ast(\Omega^\infty(X))$ of bicommutative Hopf algebras. One can then show that this map is always an isomorphism. Indeed, it is possible to reduce to the case $X=S^n$ again, and the groups $\mathbb{Q}\otimes H_\ast(\Omega^k S^{n+k})$ can be calculated by repeated use of Serre spectral sequences, and one can then let $k$ tend to infinity. As $\mathbb{Q}\otimes H_\ast(\Omega^\infty(X))$ is free on primitive generators, it is a standard fact that the dual Hopf algebra $\mathbb{Q}\otimes H^\ast(\Omega^\infty(X))$ is also free on primitive generators in the same degrees.
-For the Madsen-Weiss case, you now just need to know the groups
-$\mathbb{Q}\otimes H_\ast(AG_{\infty,2}^+)$. This is not hard, because $AG^+_{\infty,2}$ is the Thom space of a virtual vector bundle over $BSO(2)=\mathbb{C}P^\infty$, so we can use the Thom isomorphism theorem.<|endoftext|>
-TITLE: Is there an intrinsic definition of the topological index map in $K$-theory?
-QUESTION [9 upvotes]: In the language of $K$-theory, the Atiyah-Singer index theorem says that for a compact manifold $X$ the topological index map $\text{t-index}: K(TX) \to K(T\mathbb R^n) \simeq \mathbb Z$ induced by embedding $X$ in $\mathbb R^n$ is equal to the analytical index map $K(TX) \to \mathbb Z$ obtained by looking at the index of the elliptic operator whose symbol corresponds to the given element in $K(TX)$.
-My question is if there is a definition of the topological index map that does not require an embedding into a euclidean space. Clearly by the index theorem we can take the analytic index as a definition, but is there a more topological/geometric intrinsic definition for t-index that is (relatively) easily seen to be equivalent to the above definition?
-
-REPLY [9 votes]: I think the best answer you will find is the axiomatic characterization of the topological index in Index of Elliptic Operators I.
-One defines an index function to be a map $ind_X: K(TX) \to \mathbb{Z}$ with the properties that $ind_{point}$ is the identity, and for any embedding $i: X \to Y$ the wrong way map $i_!: K(TX) \to K(TY)$ commutes with the index map. It is not hard to show that the index map is actually uniquely characterized by the axioms and that the topological index is an example of an index map. Thus the real content of the index theorem lies in proving that there is an index map which sends the symbol class of an operator to its index.
-From this point of view you should think of embedding $X$ in Euclidean space as a computational crutch rather than an essential part of the theorem. If K-theory were invented before De Rham cohomology then we would all be completely satisfied with the topological characterization of the analytic index map explained above because it tells us everything we need to know to compute the analytic index in terms of K-theoretic invariants. But since we tend to prefer cohomology it makes sense to embed our manifold in a space where it is easy to relate K-theory and cohomology.
-This all is what I tell myself to calm down before I go to bed at night, but in fact I do not find it totally satisfying. I have no qualms embedding a manifold in Euclidean space when I want to prove the Gauss-Bonnet theorem or the Hirzebruch signature theorem, but I find it rather disturbing that one can prove the Riemann-Roch theorem by starting with an algebraic curve, forgetting all of its beautiful structure, and stuffing it carelessly into Euclidean space. I've always wondered if there is another computational crutch analogous to the topological index which would remember a little more structure.<|endoftext|>
-TITLE: How does Tate verify his own conjecture for the Fermat hypersurface?
-QUESTION [28 upvotes]: This question is about Tate's 1963 paper "Algebraic Cycles and Poles of Zeta Functions". Here he announces a conjecture (now known as "the Tate conjecture") which states that certain classes in the cohomology of a projective variety are always explained by the existence of algebraic cycles. In the case of a variety $X/\mathbf{F}_q$ of dimension $n$, the conjecture predicts that the subspace of classes in $H^{2d}(X\otimes\overline{\mathbf{F}}_q,\mathbf{Q}_\ell)(d)$ which are Frobenius-invariant is spanned by the image of the space of algebraic cycles in $X$ of codimension $d$.
-As an example, Tate gives the projective hypersurface $X$ defined by
-$$ X_0^{q+1}+\dots X_r^{q+1} =0,$$ where $r=2i+1$ is odd. Here $X$ admits a large group of automorphisms $U$, namely those projective transformations in the $X_i$ which are unitary with respect to the semilinear form $\sum_i X_iY_i^q$. Using the Lefschetz theorem, it isn't at all hard to compute $H^{2i}(X)$ as a $U$-module: it decomposes as a trivial $U$-module and an irreducible $U$-module of dimension $q(q^r+1)/(q+1)$. And then when you attempt to compute the $q^2$-power Frobenius eigenvalues on the middle cohomology, you find (once again by Lefschetz) that each one is a Gauss sum which in this case is nothing but $\pm q$. (If there is enough demand, I can supply all these calculations here.) Thus, miraculously, all classes in $H^{2i}(X\otimes\overline{\mathbf{F}}_q,\mathbf{Q}_\ell)(i)$ are fixed by some power of Frobenius.
-My question is: How did Tate confirm the existence of the necessary cycles? Surely the hyperplane section of codimension $i$ lands in the part of $H^{2i}$ which has trivial $U$-action (for $U$ translates hyperplanes to other hyperplanes, and these all cohomologically equivalent). In order to verify Tate's conjecture, all you need to do is produce a cycle in $X$ whose projection into the big $U$-irreducible part of $H^{2i}$ is nonzero. How did Tate produce this cycle? Did he lift to characteristic zero and appeal to the Hodge conjecture, or what?
-
-REPLY [24 votes]: I don't know how Tate did it but here is one way. Let $\zeta$ be such that
-$\zeta^{q+1}=-1$ and put $a_j=(0\colon\cdots\colon1\colon\zeta:\cdots\colon0)$,
-$j=0,\ldots,i$ with the $1$ in coordinate $2j$. Then the linear span $L$ of
-these points is contained in the Fermat hypersurfaces and gives a subvariety of
-middle dimension. I claim that its class is not a multiple of the $i+1$'st power
-of the hyperplane section. Indeed, transform $L$ by the automorphism permuting
-the two first coordinates to get $L'$. Then (assuming that $q$ is odd say) $L$
-and $L'$ are disjoint so that $[L]\cdot [L']=0$ but if the class of $L$ would be
-a multiple of the linear subspace section this cannot be. Hence, $[L]$ projects
-non-trivially to the non-trivial irreducible representation.
-Note that Shioda et al have studied cycles on general Fermat hypersurfaces and
-verified the Tate conjecture in many (all?) cases.
-Addendum: It is tricky to get dimensions and stuff correct so let me expand upon this in the simple case when $i=1$ and $q=2$ (this is a classical example appearing for instance on pp. 176-177 of Mumford: Algebraic Geometry I Complex projective varieties -- everything works in characteristic different from $3$). Then we get the line $(a\colon a\zeta\colon b\colon b\zeta)$ and by letting the monomal automorphisms of the surface act we get all $27$ lines on the Fermat cubic. It is of course true that the line is the intersection of the cubic and a linear subspace but the intersection is not proper so the class of $L$ is not a power of the hyper plane section.
-Note incidentally that in comments (p. 180) Mumford points the special nature of characteristic $2$ for this example essentially saying that there an index $2$ subgroup of the automorphism group of the Néron-Severi lattice preserving the canonical class can be realised by automorphisms of the surface.<|endoftext|>
-TITLE: Equivalent definitions of topological K-theory over locally compact spaces
-QUESTION [14 upvotes]: Hello everyone
-My question is the following:
-Given a locally compact space $X$, assume it is also connected for simplicity, its K-theory ring is defined as $\tilde{K}(X^+)$, where $X^+$ is the one point compactification and $\tilde{K}$ denotes reduced K-theory (with respect to the point at infinity). Alternatively, we can take differences $[E]-[F]$, where $E$ and $F$ are isomorphism classes of vector bundles over $X$, each one trivial outside a compact set and both with the same rank. Notice that, since both bundles are trivial outside a compact set, they can be extended to $X^+$.
-So far so good. There is another definition consisting of equivalence classes of triples $(E,F,\alpha)$, where $E$ and $F$ are now bundles over $X$ and $\alpha:E\to F$ is an isomorphism outside a compact set in $X$. Two triples $(E,F,\alpha)$ and $(E',F',\beta)$ are equivalent if there are ''degenerate'' triples $(G,G,id)$, $(H,H,id)$ and isomorphisms $u:E\oplus G\to E'\oplus H$ and $ v:F\oplus G\to F'\oplus H$ commuting with the maps $\alpha\oplus id$ and $\beta\oplus id$. Notice that in this definition, the bundles are not trivial outside a compact set so there is, a priori, no way to extend them to $X^+$.
-Well, both definitions are equivalent. This is easy to prove for compact finite dimensional manifolds (for example Atiyah's book contains a nice argument). For non compact finite dimensional manifolds, it turns out that every vector bundle is a direct summand of a trivial vector bundle (it is even more surprising: for a given bundle, there is a finite covering of the manifold such that the bundle restricted to each set of the covering is trivial). A proof of this is very difficult to find and relies on Dimension theory (Wells, Differential Analysis on Complex Manifolds prop 4.1). Using this property, the proof in this case is immediate.
-Looking for the general proof, I realized that every single place where this equivalence is mentioned simply quotes Segal's paper on equivariant K-theory. I have to admit that I don't understand the argument explained there. It is clear that every locally compact space is open and dense in its one point compactification. Take $U$ open in a compact space $Y$, using the first definition of $K(U)$, we should be able to prove that $K(U)\cong K(Y,B)$, where $B=Y-U$, but that would require ''chopping off'' the bundles outside a compact set and gluing something to make the triple be represented by bundles trivial at infinity and, therefore, with a natural extension to $X^+$. This seems to me as a highly invasive process and I don't really see why it is correct.
-Could someone please help me with this???
-
-REPLY [11 votes]: Let $(E,F,\alpha)$ represent an element of the second group. Wlog $\alpha:E\to F$ is defined globally even though it's only an iso outside $K$. Choose global compactly supported sections $s_1,\dots ,s_n$ of the dual of $E$ such that on $K$ they span the bundle; for large enough $n$ this is possible. Let $E'$ be the trivial rank $n$ bundle and interpret these sections as an injective map $s:E\to E'$. We now have a bundle map $(\alpha,s):E\to F\times E'$ which is injective everywhere. Split off the resulting subbundle of $F\times E'$ and call the other part $F'$. This leads to an isomorphism $E\times F'\cong F\times E'$. Replace $(E,F,\alpha)$ by $(E\times E',F\times E',\alpha\times 1)$. Near infinity, where $\alpha\times 1$ is an iso, the composed iso $E\times E'\to F\times E'\to E\times F'$ looks likes $(1, 0)$ on the $E$ part and therefore looks like $(?,\alpha')$ on the $E'$ part where $\alpha'$ is an iso. Make a little adjustment so that it looks like $(0,\alpha')$ on the $E'$ part. This iso $\alpha':E'\to F'$ near infinity yields $(E',F',\alpha')$ equivalent to $(E,F,\alpha)$, but with the bundles trivial near infinity. (In fact $E'$ is trivial globally.)<|endoftext|>
-TITLE: Is there a geometrical interpretation to Fermat's polygonal number theorem?
-QUESTION [5 upvotes]: The Polygonal number theorem states that every positive integer is the sum of at most $n$ $n$-gonal numbers.
-I do think that expecting a "Proof without words" of this theorem is asking for something unreasonable, but is there any way to "visualize" or to get an intuitive feel for the theorem?
-One motivation for this question is the problem asked by Bjorn Poonen here. If there was such an interpretation, then maybe it would help in some way to answer that question?
-
-REPLY [6 votes]: Let me make some comments on the polygonal number theorem along the theme that things are less thrilling for $k \gt 4$.
-
-Triangular and square numbers are pleasing to represent as dot patterns, after that $k$-gonal numbers are less attractive.
-Around 1796 Gauss proved that every integer is a sum of 3 triangular numbers. This is sometimes called the Eureka Theorem because Gauss was quite pleased with the result.
-Evidently, Cauchy showed (around 1813) that from that one can go to the general polygonal theorem that every integer $n$ can be represented as a sum of $k$ $k$-gonal numbers.
-Tables due to Peppin and Dickson show how to represent $n \lt 120k-240$ as a sum of at most $k$ $k$-gonal` numbers.
-A nice paper by Melyvn Nathanson uses results 2 & 4 to show in a few pages that
-a. for $k \ge 5$ and $n \ge 120k-240$, $n$ can be written as a sum of $k-1$ $k$-gonal numbers of which at most four are different than $0$ or $1$.
-b. For fixed $k$ every large enough odd $n$ is a sum of four $k$-gonal numbers and every large enough even $n$ is a sum of five $k$-gonal numbers one of which is either $0$ or $1$.
-
-So it seems possible to me that taking 2 & 4 as given, it might be possible to express at least some of the steps of a transition to the general polygonal theorem in a somewhat geometric way, however if this hypothetical thing is possible it would probably not be a simplification giving insight as much as a somewhat more complicated path which is interesting mainly for the fact that it can be done at all.
-I would think of an insight giving proof as one which also showed how to represent $n$. The proofs I know of show that there must be a representation but don't tell you how to find it. Changing $n$ to $n+1$ and/or $k$ to $k+1$ can radically change the representation.<|endoftext|>
-TITLE: Irreducible decomposition of tensor product of irreducible $S_n$ representations
-QUESTION [7 upvotes]: Are there well known results on the irreducibles in the decomposition of tensor products of irreducible $S_n$ representations? I would also like to know of some references where I can find formulas (if they exist in the literature) for finding multiplicities.
-
-REPLY [11 votes]: The numbers you want are called Kronecker coefficients. Bürgisser and Ikenmeyer "The complexity of computing Kronecker coefficients" showed that they are hard to compute in general, so in particular there are no "easy" formulas for them. (There are some explicit formulas for simple special cases.)<|endoftext|>
-TITLE: Elliptic curves with Mordell-Weil group Z/2Z over Q
-QUESTION [9 upvotes]: This question is not very precise; I hope it is suitable for the site.
-I have come to a situation where I have to study rational points on an elliptic curve defined over $\mathbb{Q}$. I don't know much about the curve, let alone its equation. I already have one rational point, which sits on a bounded real connected component. What I want to avoid is that this is the only rational point (other than the marked point).
-I am not sure what to use about my curve that will help me get there, so I turn the questio the other way round:
-
-What is known about elliptic curves $E$ over $\mathbb{Q}$ such that $E(\mathbb{Q}) \cong \mathbb{Z}/2 \mathbb{Z}$?
-
-REPLY [14 votes]: Mazur's theorem ensures that there are exactly 15 possible cases for the torsion part of the Mordell-Weil group of an elliptic curve: the cyclic groups $\mathbb{Z}_n$ (with $1\leq n\leq 10$ or $n=12$) and the groups $\mathbb{Z}_2\times\mathbb{Z}_n$ for $n=2,4,6,8$.
-In his paper Universal Bounds on The Torsion of Elliptic Curves, Proc. London. Math. Soc.(1976) 33, 193-237 , Daniel Sion Kubert (who was a student of Mazur) presents in table 3 (page 217) a list of parametrizations for the different possible cases.
-In particular, curves with a $\mathbb{Z}_2$ torsion are parametrized by the following family:
-$$
-\mathbb{Z}_2\ \text{torsion}:\quad y^2=x(x^2+a x+ b), \quad b^2(a^2-4b)\neq 0.
-$$
-The example given in Francesco's answer is a special case with $a=0$.
-As another example, the case with torsion $\mathbb{Z}_2\times \mathbb{Z}_2$ is parametrized by the Legendre family:
-$$
-\mathbb{Z}_2\times\mathbb{Z}_2 \ \text{torsion}:\quad y^2=x(x+r)(x+s), \quad r\neq 0 \neq s \neq r.
-$$
-A slight generalization of the Hesse family parametrizes the curves with torsion $\mathbb{Z}_3$:
-$$
-\mathbb{Z}_3 \ \text{torsion}:\quad y^2+a_1 x y +a_3 y =x^3, \quad a_3^3( a_1^3-27 a_3)\neq 0.
-$$
-For the other groups you might have to use Tate's normal form
-$$
-E(b,c): \quad y^2+(1-c)x y - b y =x^3- b x^2
-$$
-and the condition for a given torsion is expressed as an algebraic condition on $b$ and $c$.
-For example for $\mathbb{Z}_4$, we have $c=0$ and $b^4(1+16b)\neq 0$, which gives:
-$$
-\mathbb{Z}_4 \ \text{torsion}:\quad E(b,c=0): \quad y^2+x y - b y =x^3- b x^2, \quad b^4(1+16b)\neq 0.
-$$
-For a review, you can read chapter 4 of the book of Husemoller . A friendly short review is also available in section 2 of this string theory paper by Aspinwall and Morrison ( they don't present all the 15 cases but for those they analyze, they express everything in Weierstrass form).<|endoftext|>
-TITLE: Torsion freeness and birational maps
-QUESTION [6 upvotes]: Let $f:X\rightarrow Y$ be a birational morphism between smooth varieties and $F$ a torsion free sheaf. Is $f_{\ast} F$ torsion free? If not, are there conditions on either $F$, $f$, $X$ or $Y$ that could ensure the torsion freeness of $F$? For example if $f$ is surj. and $F$ is the canonical bundle on $X$ a theorem of Kollar ensure the torsion freeness of $f_*(\omega_X)$.
-
-REPLY [6 votes]: As already mentioned by Donu and Francesco, you don't need such big guns as Kollár's theorem. Also, the Proposition Francesco cites might seem more serious than it is by virtue of being in EGA...
-The point is this: For an open set $V\subseteq Y$, the module $f_*\mathscr F(V)$ is the same as the module $\mathscr F(f^{-1}V)$. Being torsion on an integral scheme is equivalent to having a non-empty support that is strictly smaller than the ambient scheme. If $f$ is surjective this already gives you what you want, and if it is only dominant you need to think a little more to see that the statement is true.<|endoftext|>
-TITLE: Centralisers in the symmetric group
-QUESTION [6 upvotes]: Let $G$ be a $k$-transitive subgroup of the symmetric group $Sym(n)$, $k\geq 2$, $n$ large. (Make $k$ larger if you think it necessary to make the question below non-trivial/interesting.)
-Write $C(g)$ for the centraliser of an element and $|C(g)|$ for its number of elements.
-What can you say about $\min_{g\in G} |C(g)|$, relative to the size of $G$? Must it be small?
-(Note that $k\geq 2$, $n\geq 3$ immediately imply that $G$ is non-abelian (thus making the question possibly non-stupid).)
-(I should clarify that I expect (hope?) the answer to be quite a bit smaller than $\ll_{\epsilon} |G|^{\epsilon}$. (Assume $k\geq 3$ or $k\geq 4$ if needed.) In this, a $k$-transitive permutation group feels like a different sort of animal from a linear algebraic group $G$, where $|C(g)|$ is most often no smaller than $|G|^{\dim(T)/\dim(G)}$ ($T$ = a maximal torus of $G$).)
-
-REPLY [2 votes]: (Written before clarification at end of question was added). Here is a result which seems to be of a somewhat negative nature in the context of your problem, and your suggested line of attack, I think. I will denote the number of conjugacy classes of $G$ by $k(G)$. The group $ G = {\rm SL}(2,2^{n})$ (n>1) is a triply transitive permutation group on $1+2^{n}$ points and has order $(2^{n}+1)2^{n}(2^{n}-1)$. It has $1+2^{n}$ conjugacy classes, so that $k(G) > |G|^{\frac{1}{3}}$. Furthermore, the only orders of centralizers of non-identity elements of $G$ are $2^n$,$2^{n}-1$ and $2^{n}+1$. Hence the minimum centralizer order for an element of $G$ is only marginally smaller than $|G|^{\frac{1}{3}}.$ Admittedly this is a rather special group and a rather rare situation, but it is an infinite family of "bad" examples.
-Added later: come to think of it, there are even bad solvable examples. If we take any prime power
-$p^{n}$ there is a doubly transitive solvable permutation group $G$ of order $p^{n}(p^{n}-1)$
-and degree $p^{n}$(this is a Frobenius group which is the semidirect product of a vector
-space of size $p^n$ acted on by a Singer cycle of order $p^{n}-1$. A point stablizer is
-cyclic of order $p^n -1)$.` In this group, the only centralizer orders for non-identity
-elements are $p^n$ and $p^{n}-1$. Hence the minimum centralizer order for $G$ is not
-much less than $\sqrt{|G|}.$ So it seems that you can't expect to get much less than
-$\sqrt{|G|}$ for the smallest centralizer order for $G$. I suspect that even this would
-be very difficult to prove without the classification of finite simple groups.
-Later remark: It is perhaps worth remarking explicitly here (though I am sure everybody knows it), that $S_n$ contains an element whose centralizer has order $n-1$ (and this is minimal),
-and that for $n > 4$, $A_n$ contains an element whose centralizer has order $n-2$,
-which is also minimal. Hence the "generic" highly transitive permutation group of degree $n$ behaves as hoped for large $n$ (and for sufficiently large $k$ there are no others, using the Schreir conjecture for $k >7$ or the disallowed Classification of finite simple groups for $k > 5$).<|endoftext|>
-TITLE: Values of the Riemann zeta function and the Ramanujan summation - How strong is the connection?
-QUESTION [9 upvotes]: (This Question was taken from MSE. As Eric Naslund pointed out there, this question is relevant. The summation method mentioned in this question is actually a good answer to it.)
-The Ramanujan Summation of some infinite sums is consistent with a couple of sets of values of the Riemann zeta function. We have, for instance, $$\zeta(-2n)=\sum_{n=1}^{\infty} n^{2k} = 0 (\mathfrak{R}) $$ (for non-negative integer $k$) and $$\zeta(-(2n+1))=-\frac{B_{2k}}{2k} (\mathfrak{R})$$ (again, $k \in \mathbb{N} $). Here, $B_k$ is the $k$'th Bernoulli number. However, it does not hold when, for example, $$\sum_{n=1}^{\infty} \frac{1}{n}=\gamma (\mathfrak{R})$$ (here $\gamma$ denotes the Euler-Mascheroni Constant) as it is not equal to $$\zeta(1)=\infty$$.
-Question: Are the first two examples I stated the only instances in which the ramanujan summation of some infinite series coincides with the values of the Riemann zeta function?
-
-REPLY [7 votes]: The answer can be obtained with the following interpretation of the Ramanujan summation:
-
-More recently, the use of $C(1)$ has been proposed as Ramanujan's summation, since then it can be assured that one series admits one and only one Ramanujan's summation, defined as the value in 1 of the only solution of the difference equation $R(x) − R(x + 1) = f(x)$ that verifies the condition $\int_1^2 R(x)dx=0$.
-
-The function $R(x)=\zeta(z,x)+C\;$ satisfy $R(x) − R(x + 1) = x^{-z}$, $x>0$, $z\in \mathbb C$, $z\ne-1$, where
-$$
-\zeta(z,x)=\sum_{k=0}^\infty\frac1{(k+x)^z}
-$$
-is the Hurwitz zeta function (or its analytic continuation for $\Re z\le 1$.)
-The value of $C\;$ can be found with the help of the shift formula for the derivative $\frac{\partial}{\partial x} \zeta(z,x)=-z \zeta(z+1,x)\;$:
-$$
-\int_1^2 R(x)dx=
-\int_1^2(\zeta(z,x)+C)dx=
-\int_1^2
-\left( -\frac1{z-1}\frac{\partial}{\partial x} \zeta(z-1,x)+C\right)dx=
-$$
-$$
-C-\frac1{z-1}(\zeta(z-1,2)-\zeta(z-1,1))=C+\frac1{z-1}=0.$$
-Hence $C=-\frac1{z-1}$. Also $\zeta(z,1)=\zeta(z)$ and we have
-$$
-\sum_{n=1}^\infty n^{-z}=\zeta(z)-\frac1{z-1}(\mathfrak{R}),\quad z\ne1.
-$$
-So Ramanujan summation transforms the Riemann zeta function into the regularized zeta function. It explains why the value $\gamma$ should be expected for the summation of the harmonic series.<|endoftext|>
-TITLE: Which book would you like to see "texified"?
-QUESTION [19 upvotes]: Let's see if we could use MO to put some pressure on certain publishers...
-Although it is wonderful that it has been put
-online, I think it would make an even greater read if "Hodge Cycles, Motives and Shimura Varieties" by Deligne, Milne, Ogus and Shih would be (re)written in the latex typesetting (well, if I could understand its content..).
-But enough about my opinion, what do you think? Which book(s) would you like to see "texified"? As customary in a CW question, one book per answer please.
-
-REPLY [4 votes]: Adams - Lectures on Lie Groups
-
-REPLY [2 votes]: Stong - Notes on cobordism theory<|endoftext|>
-TITLE: Occurence of trivial representation in a tensor square.
-QUESTION [7 upvotes]: Suppose $G$ is a group and $V$ an irreducible representation of $G$. One has that $V\otimes V\cong \Lambda^2(V)\oplus Sym^2(V)$. It is well-known that if the trivial representation appears as a subrepresentation of $\Lambda^2(V)$ then $V$ is of quaternionic type; while if the trivial representation appears as a subrepresentation of $Sym^2(V)$ then $V$ is a of real type. From this approach, it is clear that the trivial representation cannot appear in both $\Lambda^2(V)$ and $Sym^2(V)$.
-What I am curious about is as follows:
-
-
-Question: Is there is some (relatively easy) way to see why the trivial representation cannot appear in both $\Lambda^2(V)$ and $Sym^2(V)$ without introducing the machinery of real/quaternionic types?
-
-
-As a bit of motivation, if one looks at other subrepresentations, then for example if $G = G_2$ and $V_n$ is an $n$-dimensional irreducible representation of $G_2$, then $V_{64}$ appears as a subrepresentation of both $\Lambda^2(V_{27})$ and $Sym^2(V_{27})$. In particular it is possible for the intertwining number of $\Lambda^2(V)$ and $Sym^2(V)$ to be nonzero, but by the real vs. quaternionic characterization, the trivial representation is somehow special in that it cannot contribute to the intertwining number.
-
-REPLY [14 votes]: This is essentially what Darij wrote, but without mentioning the bilinear forms. (I had written it out before reading far enough into Darij's post to see that he was really doing the same thing, after the part about bilinear forms.) Think of $V\otimes V$ as $\text{Hom}(V^*,V)$. An occurrence of the trivial representation in $V\otimes V$ thus amounts to a $G$-equivariant linear map from $V^*$ to $V$. Since $V$ and therefore also $V^*$ are irreducible, Schur's lemma says that the space of such maps has dimension either 1 (iff $V$ and $V^*$ are isomorphic) or 0. So there's at most one occurrence of the trivial representation in $V\otimes V$.<|endoftext|>
-TITLE: Best constant in comparison between Rademacher and gaussian averages?
-QUESTION [7 upvotes]: Let $(g_k)$ be a sequence of independent standard gaussians variables on a fixed probability space $\Omega$. Let $(\epsilon_k)$ be a sequence of independent rademacher variables.
-What is the best constant in the following inequality:
-$$
-\vert\vert\sum_{k}\epsilon_k \otimes x_k \vert\vert_{L^2(\Omega,E)}
-\leq
-K \vert\vert\sum_{k}g_k\otimes x_k \vert\vert_{L^2(\Omega,E)}\ \ ?
-$$
-for any Banach space $E$, any $x_1,\ldots, x_n \in E$.
-I know that the best constant is $\leq \sqrt{\frac{\pi}{2}}$ (see Diestel,Jarchow,Tonge "Absolutely summing operators" page 239).
-
-REPLY [10 votes]: $\sqrt{\pi\over 2}$, the reciprocal of the $L_1$ norm of a standard gaussian, is the best constant. Let $x_k$ be the kth unit basis vector in $\ell_1$ and let the sum go from $1$ to $N$. The square of the left hand side is $N^2$ and the square of the right hand side is $N+N\sqrt{2\over \pi}(N-1)\sqrt{2\over \pi}$ (multiplied by $K^2$).<|endoftext|>
-TITLE: Why the BGG category O?
-QUESTION [27 upvotes]: Given a finite-dimensional semisimple complex Lie algebra $\mathfrak{g}$, the Bernstein-Gelfand-Gelfand category $\mathcal O$ is the full subcategory of $\mathfrak g$-modules satisfying some finiteness conditions. It contains all finite-dimensional modules as well as all highest-weight modules, it's Noetherian and Artinian, and it's Abelian. It's clear to me why you would want to work in some full subcategory of $\mathfrak g$-modules which has the above properties, but why $\mathcal O$? Is it minimal in some sense with respect to these properties and/or some other important properties?
-
-REPLY [17 votes]: It might be worth pointing out a different motivation for Category O, namely the theory of Harish Chandra (g,K) modules. These are algebraic models for continuous representations of real reductive groups (the real forms of g). Harish Chandra's amazing theory reduces many questions in the representation theory of real groups to this algebraic theory, which can then be studied geometrically, for example by Beilinson-Bernstein localization.
-In any case Category O is essentially the category of Harish Chandra modules associated to the complex reductive group G, when considered as a real Lie group. There are slight subtleties in what kind of semisimplicity/local finiteness etc we require for the center of the enveloping algebra or the maximal torus, but in broad strokes the two coincide, and category O is thus a nice combinatorially accessible model of a very basic object in representation theory of Lie $groups$.<|endoftext|>
-TITLE: Relation between Isogeny, Conics and Fermat's method of infinite descent
-QUESTION [5 upvotes]: Fermat's proof of FLT(4) is an example of infinite descent as is Euler's (or whoever you attribute it to's) proof of FLT(3). There are similar proofs to Fermat's for Diophantine equations like $x^4 + y^4 = 2z^2$.
-I have unsuccessfully tried to view these proofs in terms of group homomorphisms on conics and elliptic curves but it is not at all clear whether this is possible.
-Can we reinterpret these infinite descent proofs geometrically, in terms of curves?
-
-REPLY [8 votes]: (That picture in your avatar is Weil, right? You should start by reading Weil's Number Theory: an approach through history).
-FLT(3) is the assertion that the curve $x^3+y^3=1$ has three rational points (including the point at infinity). The standard process of putting an elliptic curve in Weierstrass form shows that this curve is $y^2=x^3-432$ if I remember correctly. Now use descent on this elliptic curve (maybe an isogeny of degree 3) to show that the Mordell-Weil group is finite of order three (see, e.g. Silverman for the general theory). This may be Euler's proof, maybe it's discussed in Weil's book.
-FLT(4) is weaker than the statement that the equation $x^4+y^4=z^2$ has only the obvious solutions. Again, this becomes the problem of finding the rational points on $y^2=x^4+1$ which is again an elliptic curve and Fermat's proof is a 2-descent showing that the Mordell-Weil group is finite of order four. I'm pretty sure this is in Weil's book.
-I have no idea what conics have to do with any of this.<|endoftext|>
-TITLE: What do you do if you find a typo in an equation of a paper?
-QUESTION [15 upvotes]: I was reading an interesting paper, and early in the introduction there was an equation with a typo in it. I am absolutely sure of this, and all it was missing was a factor of $n!$. Overall, it was an inconsequential part of the paper, and doesn't affect anything, as it was in the section which tells the reader about the background of the area. But still, the equation as written is not correct, and could be confusing to someone reading who is trying to learn about the area.
-In fact, the line right before this particular equation said "it is easily observed that..." which is a little interesting.
-
-My question is: Should I do anything when I see this? Should I just ignore it, or is it polite to email the author? Or is it impolite to do so? If you published a paper, with an inconsequential, but possibly confusing typo in the introduction, would you want someone to tell you about it or just leave it alone?
-
-Thanks for the advice!
-Also please note, I am not referring to a spelling typo, that is definitely not important! Its just possible that this typo could confuse someone reading the paper. (I was confused at first!)
-
-REPLY [5 votes]: I usually email the author in case the paper is online in a place controlled by the author (i. e., his website or arXiv). The only downside of this approach (apart from authors not replying) is that some would just take down their papers (rather than correcting them) when they hear about a mistake in a part they consider substantial. This happened to me once (although fortunately the paper is still avaliable at other places in the net).
-In case the paper is online but not in such a place, this is a more difficult question, but if the error is substantially confusing, I'd still mail the author. Unfortunately, this sometimes means notifying a clueless author that somebody else has put his paper online (most usually, a university workmate, for his students; I am not talking about pirate sites), and rather than correcting it tries to get it offline. I would be careful here.
-In case the paper is offline or behind a paywall, I ignore it unless the mistake really destroys some results from the paper, in which case I rejoice about another little blow to copyrighted literature and the so-called peer review process.
-But then again, I am mostly reading openly avaliable texts, so the first case is by far the most frequent.
-I, personally, would prefer anyone telling me of any mistake, but I do not have (and do not plan to have) papers outside of open access, so there is no contradiction here.<|endoftext|>
-TITLE: Is there a least-fixed-point formulation of inaccessible cardinals?
-QUESTION [11 upvotes]: The infinity axiom can be formulated by defining a function $S$ as
-$$S(N) = \{0\} \cup \{n+1\\ |\\ n \in N\}$$
-(FWIW, I'm assuming the von Neumann ordinals.) The axiom is then
-$$\exists I . I = S(I)$$
-which gives us our first infinite set. Then $\omega$ is the intersection of all the subsets of $I$ that are also fixed points, or the least fixed point of $S$.
-I've been curious about this for a while now: can the first (strongly, uncountable) inaccessible cardinal be defined similarly?
-
-REPLY [5 votes]: One can attain something like a positive solution with the observation that the inaccessible cardinals are precisely the regular fixed points of the beth function $\alpha\mapsto\beth_\alpha$, which is monotone and continuous. In particular, the first inaccessible cardinal is the smallest regular beth fixed point.<|endoftext|>
-TITLE: 2-colimits in the category of cocomplete categories
-QUESTION [6 upvotes]: Let us denote by $\text{Cat}_c$ the $2$-category, whose objects are cocomplete categories, morphisms are cocontinuous functors and morphisms are natural transformations. Is it then true that $\text{Cat}_c$ is $2$-cocomplete, i.e. has every $2$-functor $C : I \to \text{Cat}_c$, where $I$ is a small $1$-category, a $2$-colimit?
-I have the following idea: Let $U :\text{Cat}_c \to \text{Cat}$ be the forgetful $2$-functor. Now $\text{Cat}$ is $2$-cocomplete; a comprehensive reference for this is "The stack of microlocal perverse sheaves" by Ingo Waschkies. Besides, $U$ has a $2$-left adjoint $\widehat{(- )} : \text{Cat} \to \text{Cat}_c$ (see here). So we may consider the cocomplete category $D:=\widehat{\text{colim} UC}$, but of course the functors $C_i \to D$ are not cocontinuous. Thus perhaps we have to define some reflective (and hence cocomplete) subcategory $D'$ of $D$, such that each $C_i \to D$ factors through a cocontinuous functor $C_i \to D'$. But I don't know how to define $D'$.
-I'm also interested in colimits in similar $2$-categories, for example $\text{Cat}_{c\otimes}$, which consists of the cocomplete tensor categories. And actually I want something more, namely that these $2$-categories are $2$-locally presentable. Any hints and references are appreciated!
-
-REPLY [12 votes]: In order not to have to worry about size issues, I'm going to answer the following question instead:
-
-For a (small) cardinal number $\kappa$, is the category of small categories with $\kappa$-small 2-colimits 2-cocomplete?
-
-If you take $\kappa$ to be inaccessible, then this will correspond to your question, under a particular choice of foundations. I presume moreover that you mean "2-colimits" in the weak "up-to-equivalence" sense which the nLab uses (which 2-category theorists traditionally call "bicolimits").
-The fact that the 2-category Cat of small categories is 2-cocomplete, in this sense, has been well-known to category theorists for decades. It is obvious that Cat is cocomplete as a 1-category (since it is locally finitely presentable), and since it is closed symmetric (cartesian) monoidal, it follows by general enriched category theory that it is cocomplete as a category enriched over itself. In the nLab terminology, it has all strict 2-colimits. We then observe that strict pseudo 2-limits, which are 2-limits that represent cones commuting up to isomorphism but satisfy their universal property up to isomorphism (rather than equivalence), are particular strict 2-limits. Since any strict pseudo 2-limit is also a (weak) 2-limit, Cat is 2-cocomplete.
-Now as Zoran pointed out in the comments, there is a 2-monad on Cat whose algebras are categories with $\kappa$-small colimits; let us call this 2-monad $T$. The strict $T$-morphisms are functors which preserve colimits on-the-nose, while the pseudo $T$-morphisms are those which are $\kappa$-cocontinuous in the usual sense (preserve colimits up to isomorphism). Therefore, the question is whether the 2-category $T$-Alg of $T$-algebras and pseudo $T$-morphisms is 2-cocomplete.
-The answer is yes: it was proven by Blackwell, Kelly, and Power in the paper "Two-dimensional monad theory" that for any 2-monad with a rank (preserving $\alpha$-filtered colimits for some $\alpha$) on a strictly 2-cocomplete (strict) 2-category, the 2-category $T$-Alg is (weakly) 2-cocomplete. The 2-monad $T$ has a rank (namely, $\kappa$, more or less), so their theorem applies. I believe this all works just as well in the enriched setting.<|endoftext|>
-TITLE: Publishing journals articles without transferring copyright.
-QUESTION [45 upvotes]: I'm a grad student getting close to submitting my first journal article (which will be single-authored). My understanding is that it's standard practice for authors to transfer the copyright of their paper to the journal in which it is published. I want my article to be published in a journal, but I don't want to transfer the copyright -- I want the article to be in the public domain. How will editors to behave towards such a request? Also, when should I bring up the topic: when I submit the article, or after it's accepted and I'm asked to sign a copyright transfer?
-Some grants apparently have a stipulation that articles written as part of the grant research must be released into the public domain (e.g. grants funded by the US government). In this case, authors presumably sign a consent to publish, instead of a copyright transfer. Hence, there's at least some precedent for what I want to do, though I want my article in the public domain purely because of my personal views on the ethics of copyright.
-I couldn't find too much information about this topic by googling. Oleg Pikhurko has a page discussing his attempt to have his articles revert to the public domain after a period of years, as opposed to instantly. It didn't work out particularly well in his case.
-I'm not sure how much I'm willing to have my ethical ideals damage my career (e.g. by having publications delayed and/or being banned from submitting to journals).
-
-REPLY [16 votes]: I would not advise you to give up your ethical ideals. Frankly, I find it offensive that any publishers expect me to assign to them the copyright in an article that I have written.
-I think there are also some practical reasons to retain copyright, although they are not as pressing now that we can put our papers on the arXiv (which you should, in my opinion, always do). Kevin's addendum form mentions some of the important rights that some copyright transfer forms might take away from you: the right to create derivative works, use portions of the article in other work, and distribute the final version on the web.
-The best solution is, obviously, to publish in journals that always allow the author to retain copyright. Such journals often also have other desirable features, e.g. they make their published articles freely available online, rather than charging huge sums of money to university libraries for subscriptions. Furthermore, publishing good work in these "friendly" journals helps to build up their acceptance in the community, so that future authors will feel less pressure from career considerations to publish in "unfriendly" journals.
-Of course, right now it is still the case that some of the "best" journals (in terms of building up your publication record) are "unfriendly". How far you are willing to compromise between ethical ideals and career considerations is a personal decision. My personal choice so far has been to publish in any journal, but insist on retaining copyright. I've had more luck with this than Pikhurko did so far; out of two publications in "unfriendly" journals, in both cases the publisher was willing to allow me to retain copyright (although they have a bit of trouble actually putting the correct copyright notice on the article).<|endoftext|>
-TITLE: A Bijection Between the Reals and Infinite Binary Strings
-QUESTION [6 upvotes]: Whenever possible, I like to present Cantor's diagonal proof of the uncountability of the reals to my undergraduates. For simplicity, I usually restrict to showing that the subset
-$$
-A = \{x \in [0,1) \mid \text{ the decimal representation of $x$ uses only 0's and 1's} \}
-$$
-is already uncountable. I was thinking recently that it would be nice to add a quick proof that $A$ is actually of precisely the same cardinality as $\mathbb{R}$. That is, I would like to:
-Demonstrate a bijection between $A$ and $\mathbb{R}$.
-My first instinct was to use find an injection from $A$ into $\mathbb{R}$ and vice versa, then appeal to Cantor-Bernstein to say that a bijection exists (even if we don't know how to construct it). The identity map suffices from $A$ into $\mathbb{R}$. For the other direction, I thought of something like "for $x \in \mathbb{R}$, map $x$ to its binary representation, disregarding the decimal point". I'm afraid this function fails to be injective, however. For example, 1 (base 10) can be represented as $.\overline{1}$ (base 2), and so 2 (base 10) can be represented as $1.\overline{1}$ (base 2). Thus, 1 and 2 (base 10) will have the same image under my map.
-Any methods (not necessarily the one I've attempted to start here) are most welcome. I will accept as "correct" the method which demonstrates the bijection with the greatest level of clarity.
-
-REPLY [3 votes]: A more concrete way to fix the OP's idea (which is similar to Stefan's but avoids Cantor-Bernstein) is to simply delete $\mathbb{Q}$ from $A$ to produce a new set $B$. Split $B$ into a countable family of sets $B_k$ where $B_k$ consists of all the elements of $B$ with $k$ leading zeros. There is now an obvious bijection between any $B_k$ and any set of the form $[n,n+1)-\mathbb{Q}$ by simply viewing elements of $B_k$ as sequences in binary instead of decimal and ignore the leading zeros and first 1. There is no need to worry about the OP's original concern since $B$ only consists of irrationals. Since there are countably many $B_k$'s and countably many $[n,n+1)-\mathbb{Q}$, pick your favorite way to match them up. The remainder, $A\cap\mathbb{Q}$, is obviously countably infinite, so biject it with $\mathbb{Q}$.<|endoftext|>
-TITLE: (almost) statistical independence of nodes degrees in a graph
-QUESTION [11 upvotes]: Wireless networks are typically modeled as random geometric graphs. The number of nodes $N$ in the network is drawn from a Poisson distribution with intensity $\lambda$
-$$P(N = n) = \frac{\lambda^n e^{-n}}{n!}.$$
-Once this number has been chosen, the $N$ nodes are placed uniformly at random over a circular (or squared) area of radius $R$. Two nodes are then connected by an edge if their euclidean distance is less than some predefined range, say $r_0$, where $r_0 << R$.
-As a result of Penrose's Theorem to ensure that the graph is $k$-connected, it is sufficient to show that the minimum degree is at least $k$, i.e., $d_{\rm min} \geq k$. This holds true asymptotically for a geometric random graph.
-I came across two papers dealing with connectivity in (wireless) networks (this is one, and here is another one, but there are others by different Authors in the literature). When posing the condition that the minimum degree is at least $k$, I often find this approximation:
-$$ P(G{\rm ~is~k~conn}) \cong P (d_{\rm min} \geq k) \cong P(d \geq k)^n$$
-where $P(d \geq k)$ is the probability that the degree of a node (any node) is at least $k$. The first approximation is essentially true when the number of nodes is at least of a few hundreds. The second approximation is not true since the nodes degrees are correlated. What people say when using it, is that they assume almost statistical independence of the nodes degrees.
-In practice there is an excellent match between simulation results and the approximation above. Since I need to use the very same approximation, I was wondering if someone had a better argument to justify its use, rather than assuming its validity from the very beginning.
-
-REPLY [4 votes]: This is a partial answer concerning the validity of the second approximation. I'll consider the case $k=1$ for simplicity: all the difficulties and tricks will be clear from it already.
-First of all, when people claim that some probability estimate agrees well with simulations, they always mean the difference metric. There is no way to distinguish between the probability $10^{-20}$ and $10^{-100}$ in practice though, for most theoretical purposes, the difference here is much larger than between $0.9$ and $0.1$. Thus, we are interested just in showing that the estimate in question is good when it is neither too small, nor too close to $1$. Assuming that we are on the torus (to avoid rather boring discussion of the boundary effects), and that the number of points $N$ is fixed (for the difference metric conditioning on $N$ is not a big problem) we can say that the estimate in question is just $(1-(1-\pi r^2)^{N-1})^N$, which is neither $0$, nor $1$ if $\pi r^2=\frac{\log N+O(1) }{N}$. We shall carry out the computations under this assumption only because the monotonicity of both the true probability and the estimate in $r$ is obvious. Denote $Q=(1-\pi r^2)^{N-1}\asymp \frac 1N$
-Enumerate the points $x_i$ to throw on the torus and let $f_i$ be the random variable that is $1$ if $x_i$ is isolated and $0$ if not. We are interested in $E\prod_i(1-f_i)$. The most natural idea is to use the truncated PIE. The first term after $1$ is $-\sum_i Ef_i=-NQ$, which is just what we need.
-To use the truncated PIE, we need to estimate $Ef_1\dots f_m$. The estimate from below is straightforward: $x_{k+1}$ has the area at least $1-\min(k,m)\pi r^2$ to choose from, so we get almost $Q^m$ on the multiplicative scale if $m$ is at most a small power of $N$. To make a (rather crude) estimate from above, consider the connected components of the set $x_1,\dots,x_m$ with respect to the connection at distance $2r$ or less. If each component is single point, we get the lower bound above, i.e., essentially $Q^m$. Suppose that there are $p
-TITLE: Why "Classification" of 4 manifolds is NOT possible?
-QUESTION [15 upvotes]: I know classification of 2 manifolds and geometrization for 3 manifolds.
-Why for dimension great or equal to 4, this task become impossible?
-edit: Or should I ask "why geometrization won't be possible for 4 or higher dimension?"
-
-REPLY [34 votes]: As pointed out in a comment by Autumn Kent to Allen Knutson's answer,
-the problem is a bit more subtle than it may appear. In order to
-prove that the homeomorphism problem for compact 4-manifolds, say in
-the topological category, is recursively unsolvable, it is not enough
-to know that (1) every finitely presented group can be realized as
-the fundamental group of some compact 4-manifold, and (2) the isomorphism problem for finitely presented groups is recursively unsolvable.
-Instead, what you do is give a construction which to any finite presentation $< S | P >$ of a group associates a 4-manifold $M(S,P)$ in such a way that $\pi_1(M(S,P))$ is isomorphic to the group defined by
-the presentation $< S | P >$, and moreover two such manifolds are homeomorphic if and only if they have isomorphic fundamental groups.
-Then you have constructed a class of 4-manifolds for which the homeomorphism problem is equivalent to the isomorphism problem for
-finitely presented groups, and therefore unsolvable.
-About "geometrization for manifolds of dimension 4 or higher", well
-as far as I know there is no theorem which says it is impossible. It
-depends on what you mean by `geometric structure', and what you
-want those structures to do for you.<|endoftext|>
-TITLE: A question about MathSciNet etiquette
-QUESTION [60 upvotes]: Hello,
-Recently, a colleague of mine pointed me to a MathSciNet review of one of my papers that is completely off the mark - it is not negative or anything like that, but it grossly misrepresents the contents of the paper (when describing the origins of the techniques and questions in the paper, for instance, as well as the "position" of the results within the current litterature and the meaning of the results themselves).
-I'm not sure what I should do - actually, I probably won't do anything because the paper seems pretty inconsequential and the quality of the review most likely does not matter much. Still, the same thing could have happened with a paper I truly care about, and this led me to wondering what the proper behaviour is: should I contact the reviewer and ask him/her to retract his/her review (explaining why, of course)? Should I contact MathSciNet and let them know that I believe the review is incorrect? The first option raises some "diplomatic" problems, while the second one seems to me both to be abrupt and to waste several people's time... I think, if pressed to act, I would choose the first course of action, but I'd be grateful for any suggestions (e.g on how to say "you absolutely mangled that review!" without being rude..)
-Final note: I'm asking this question anonymously because I don't have that many papers and it would be easy to identify the reviewer by looking at my MathSciNet profile, and I'm not out to embarass anyone.
-Thanks for your help - if the question is inappropriate for this site then please close, of course!
-
-REPLY [11 votes]: Thanks to everyone for the advice. It seems that the question has stopped attracting new reactions, so I'll try to summarize what I took from the discussion.
-
-There is no way (nor should there be) or hiding the fact that I'm the one complaining about the review's quality - anyway, the reviewer signed his/her review so it would be highly discourteous to try to stay anonymous.
-In all probability the reviewer tried to do a fair job and simply failed; it does not seem unlikely that he/she would prefer to be told that the review is off the mark and have a chance to make it right. This helps prevent his/her name to be attached to an erroneous review.
-In view of all this, it seems at least courteous to begin by contacting the reviewer and explaining why the review seems wrong to me, and then give him/her a chance to correct it on his/her own (or to explain me why I misunderstood my paper, of course).
-In the course of such communication, one should as much as possible avoid being too directly critical of the original review (and remember that the reviewer wrote it on what is essentially his/her own free time!).
-If the above option does not produce the desired results, then the way to go (if sufficiently motivated!) is to contact mathrev@ams.org, explain the issues with the review, and leave it to them (at least one contributor thinks that this should be the first step, without any direct contact with the reviewer, but I do not see the downside of contacting the reviewer first and MathSciNet second).
-
-Also, note that a somewhat similar question may be found at How do I fix someone's published error? , and that some of the advice there is relevant to the problem at hand.
-Again, thanks to everyone for the advice!
-Note possibly to be removed at a later point: I plan to accept this answer, so that it appears first to anyone interested in a similar problem at a latter date. I'll wait a few days before doing so, that way you can let me know if this clashes with this forum's usual rules.<|endoftext|>
-TITLE: How to find Casimir operators?
-QUESTION [8 upvotes]: Given a general Lie algebra, is there a general procedure to find all its Casimir operator?
-
-REPLY [3 votes]: I'm assuming you're thinking of some specific matrix representation $X_i \in \mathfrak{g}$ (let's assume it's the defining representation). Compute the Killing form, $\kappa_{ij} \doteq Tr (X_i\cdot X_j)$ (actually usually this is defined in the adjoint representation, but any faithful rep will do). The quadratic Casimir is then simply $ X_i \kappa_{ij} X_j$ (Einstein convention).
-Other Casimirs can be obtained from the characteristic (secular) equation: define $X(\omega) = \omega^i X_i$. The characteristic equation is $\det\left( X(\omega) - \lambda I \right) = \sum\limits_{j} (-\lambda)^{N-j} \phi_j(\omega) \equiv 0 $ ($N$ is the matrix dimension, and/or the dimension of the Lie algebra if you're using the adjoint representation). If you now perform the substitution $\omega^i \to X_i$ in the coefficients $\phi_j (\omega)$, you get Casimir invariants $\phi_j (X)$!
-It might seem like the higher the representation, the more invariants you get, but in fact all the invariants can be expressed in terms of the fundamental invariants of the defining representation. I can't think of many references right at the moment, but e.g. Gilmore: Lie groups, physics and geometry pp. 140 has a nice explanation. Also, google the boldface texts above.<|endoftext|>
-TITLE: Seeking proof for linear algebra constraint problem.
-QUESTION [13 upvotes]: Given a symmetric real matrix with a zero diagonal $M$, I am trying to find a diagonal matrix $D$, such that the matrix $M + D$ is positive definite, and $(M+D)^{-1}$ has a diagonal consisting of all 1's. This problem looks vaguely like a semidefinite programming problem, except that both the matrix $(M+D)$ and it's inverse have linear constraints. Overall, the system has as many constraints as variables.
-Based on small scale numerical testing, it strongly appears that there is always a unique solution. I've implemented an algorithm (which is $O(n^6)$, $n$ being the size of the matrix), that works by constructing a second matrix $X$, and minimizing $||(M+D) X - I||$ with respect to $D$ and then with respect to $X$ in an alternating fashion. Note that I am using the Frobenius norm here. Given the proper initialization, such the $(M+D)$ is positive definite, this usually appears to converge, and each step is simply a quadratic minimization.
-That said, I have no proof that there is a unique solution for $D$, or that the algorithm above works in the general case, and moreover I have a strong intuition that there is an algorithm with is closer to $O(n^3)$.
-What I seeks is proof or a theoretical justification that the solution to $D$
-is unique (or of course a counterexample). Even better would be a provably polynomial time algorithm to find $D$.
-My approach for a proof up till this point has been along the lines of finding some error function $f(X)$, $X = M + D$, such as $f(X) = \sum_i ((X^{-1})_{ii} - 1)^2$. This function (and lots of other variants) have a minimum at the desired solution. My hope was to then show that the function is convex over all positive definite matrices $X$. However I have not been able to accomplish this so far.
-Edit: For the $f(X)$ given above I have found a number of counterexamples to it's convexity, although perhaps the overall method is still salvageable with a different error function.
-Edit: Some additional facts I've been able to show (in part with help from the comments)
-The set of all positive definite matrices $X = M + D$ is clearly a convex set (since it is the intersection of two convex sets, the positive definite matrices and the set of all matrices with non-diagonal elements $M$).
-Moreover, the set of all $X$ above such that all the diagonals elements of $X^{-1}_{ii} \le 1$ is also a convex set. This follows from the convexity of $e^T X^{-1} e$, and the statement above. The solution in question is clearly on the boundary of this set.
-
-REPLY [5 votes]: (Edit: my original answer was perhaps not clear enough, let me try to improve it).
-First some notation: for a matrix $x$, let me denote by $E(x)$ the diagonal matrix with the same diagonal as $x$: if $x=(x_{i,j})_{i,j\leq n}$, $E(x) = (x_{i,j}\delta_{i,j})_{i,j \leq n}$. Equivalently, $E$ is the orthogonal projection on the diagonal matrices when you consider the usual euclidean structure on $M_n$. If will also write $x>0$ to mean $x$ is symmetric positive definite.
-You are asking whether the map $f:x \mapsto x^{-1} - E(x^{-1})$ is a bijection from its domain $D=\{x \in M_n(\mathbb R), x>0\textrm{ and }E(x)=1\}$ to its image $I=\{x \in M_n, x=x^*\textrm{ and }E(x)=0\}$. And the answer is yes. I prove first that $f$ is injective, and then that it is surjective.
-
-f is injective
-In fact let me prove the following fact, which is equivalent to the injectivity of $f$.
-
-Let $x$ be a positive matrix. If $d$ is a diagonal matrix such that $x+d$ is positive and such that $x^{-1}$ and $(x+d)^{-1}$ have the same diagonals, then $d=0$.
-
-Proof: Since $d$ is diagonal, the trace of $d\left(x^{-1} - (x+d)^{-1}\right)$ is zero. But one can write this expression as \[dx^{-1/2}\left(1- (1+x^{-1/2}dx^{-1/2})^{-1}\right)x^{-1/2},\]
-so that taking the trace and denoting by $a=x^{-1/2}dx^{-1/2}$, we get $0= Tr(a(1-(1+a)^{-1}))$.
-If $\lambda_1,\dots,\lambda_n$ are the eigenvalues of $a$, the condition $x+d$ positive becomes $1+\lambda_i >0$, and the last equality becomes $0 = \sum \lambda_i(1-1/(1-\lambda_i)) = \sum \lambda_i^2/(1+\lambda_i)$, which is possible only if the $\lambda_i$ are all zero, i.e. $a=0$, i.e. $d=0$.
-
-** f is surjective **
-The surjectivity is true just for topological reasons. More precisely, to prove that $f$ is surjective, it is enough to prove that it is continuous, open and proper (because this would imply that the image is an open and closed subset of $I$, and hence everything since $I$ is connected). The continuity is obvious. $f$ is even differentiable, and the differential is explicitely computable and easily seen to be invertible at every point, so that $f$ is indeed open. It remains to check that it is proper.
-The proof I have is not completely obvious, maybe I am missing something. Let me only sketch it. Take a sequence $x_k \in D$ that escapes every compact subset of $D$. Since $\|x\|\leq n$ for all $x \in D$, we have that $u_k=\|x_k^{-1}\|\to \infty$ (I consider the operator norm, and the inequality $\|x\|\leq n= Tr(x)$ is because the norm of $x>0$ is its largest eigenvalue, whereas its trace is the sum of its eigenvalues). We want to prove that $\|f(x_k)\|\to \infty$. Assume for contradiction that this is not the case, and that $\|f(x_k)\|\leq C$ for all $k$. We will get a contradiction through a careful study of the spectral decomposition of $x_k$.
-Let $\xi_k$ be a sequence of unit eigenvectors of $x_k$ relative to the smallest eigenvalue of $x_k$, i.e. $x_k \xi_k = 1/u_k \xi_k$. Now the key observation: the assumption that $\|f(x_k)\|\leq C$ implies that, for all diagonal matrix $d$ with $1$ or $-1$ on the diagonal, the distance from $d \xi_k$ to the space $F_k$ spanned by the eigenvectors of $x_k^{-1}$ relative to the eigenvalues in an interval $[u_k/2,u_k]$ goes to zero. For a proof, consider the random diagonal matrix $d$ in which the diagonal entries are iid random variables uniform in $\{-1,1\}$, so that $E(x) = \mathbb E (d x d)$ (hoping there will be no confusion between $E$ and $\mathbb E$). Then $\langle f(x_k) \xi_k,\xi_k\rangle = \mathbb E ( u_k - \langle x_k d \xi_k, d \xi_k\rangle)$. The lhs of this equality is by assumption smaller than $C$. On the rhs, $u_k - \langle x_k d \xi_k, d \xi_k\rangle \geq 0$ because $d \xi_k$ is a unit vector. This implies that $u_k - \langle x_k d \xi_k, d \xi_k\rangle \leq 2^n C$ for any diagonal matrix with $\pm 1$ on the diagonal. But now use the fact that, for $x>0$ in $M_n$, if a unit vector $\xi$ in $\mathbb R^n$ satisfies $\langle x \xi,\xi\rangle \geq \|x\|-\delta$, then $\xi$ is at distance less than $\sqrt{2\delta/\|x\|}$ from the space spanned by the eigenvectors of $x$ relative to eigenvalues in the interval $[\|x\|/2,\|x\|]$ (hint for a proof: consider the decompostion of $\xi$ in an orthonormal basis of eigenvectors of $x$). Here if $\epsilon_k = \sqrt{2^{n+1} C/ u_k}$, we have indeed proved that $d \xi_k$ is at distance less than $\epsilon_k$ from $E_k$ for any diagonal matrix with $\pm 1$ on the diagonal.
-I now claim that there is a vector $\eta_k$ in the canonical basis of $\mathbb R^n$ at distance less than $\sqrt n \epsilon_k$ from $E_k$. This will conclude the proof since it will in particular imply that $\langle x_k \eta_k,\eta_k\rangle \to 0$, whereas the assumption $E(x_k)=1$ implies that $\langle x_k \eta_k,\eta_k\rangle = 1$, a contradiction. To prove the claim, let $i$ be such that the $i$-th coordinate of $\xi_k$ is larger than $1/\sqrt n$ in absolute value. Observe that $\xi_k(i) e_i$ is the expected value of $d \xi$, where $d$ is the same random matrix as above, but conditionned to $d_i = 1$. This implies that $\xi_k(i) e_i$ is at distance at most $\epsilon_k$ from $E_k$, which proves the claim.
-A remark I do not like this proof, since it really relies on finite-dimensional techniques. In particular, it does not extend to general von Neumann algebras (whereas the injectivity part does). I would prefer a more direct proof.<|endoftext|>
-TITLE: A necessary condition for S4-completeness?
-QUESTION [6 upvotes]: It is well-known that the modal logic S4 is complete with respect to the class of all finite quasi-trees (where we interpret the $\Box$ modality as topological interior, and topologize a quasi-tree with the up-set topology). It is also well-known that p-morphisms (open, continuous surjections) preserve modal validity. Thus, for any space $X$, the existence of p-morphisms from $X$ onto every finite quasi-tree is a sufficient condition for $X$ to be S4-complete. This technique can be used to establish, for example, McKinsey and Tarski's famous result that S4 is the logic of any dense-in-itself, metrizable space.
-My question is:
-
-Is this condition also necessary? Said differently: is there a space $X$ and a finite quasi-tree $Q$ such that $X$ is S4-complete but there exists no p-morphism $\rho: X \to Q$?
-
-This seems like a natural question to ask, but I haven't had much luck in finding any discussion about it. Even just a pointer to the right body of literature would be very much appreciated.
-
-Addendum
-Here I'll define my terms a little more carefully, and spell out the translation of my question in terms of the more standard Kripke semantics.
-Recall that quasi-orders are sets equipped with reflexive, transitive binary relations, which is precisely the class of Kripke frames corresponding to S4. A quasi-order $Q = (Q,\leq)$ is called a quasi-tree if $Q/\sim$ is a tree, where $\sim$ is the equivalence relation on $Q$ defined by
-$$x \sim y \iff x \leq y \textrm{ and } y \leq x.$$
-As mentioned in the comments, there is a correspondence between quasi-orders and Alexandrov spaces, one direction of which is given by topologizing quasi-orders with the up-set topology. There is also a notion of a p-morphism between quasi-orders, nicely outlined by Wikipedia. A p-morphism between quasi-order corresponds to an open, continuous map between the corresponding Alexandrov spaces.
-I use the phrase "$X$ is S4-complete" (perhaps somewhat idiosyncratically?) to mean that every formula validated by $X$ is provable in S4; equivalently, $X$ refutes all non-theorems of S4. It is known that if $Q$ is any quasi-order and for each finite quasi-tree $Q_{t}$ there exists a surjective p-morphism $\rho_{t}: Q \to Q_{t}$, then $Q$ is S4-complete. One can then ask:
-
-Is the converse true? Does every S4-complete quasi-order Q admit maps $\rho_{t}$ as above?
-
-If not, then a counter-example can be "lifted" into the topological setting, thus answering my original question. However, a positive answer to this question does not immediately resolve the topological version since the quantification in the topological version is over all spaces, rather than just the Alexandrov spaces. Nonetheless, I would be interested in an answer (or even a hint at an answer) to either question.
-
-REPLY [2 votes]: S4 is complete with respect to a Kripke frame or general frame or topological frame $F$ if and only if $F$ is an S4-frame and for every finite rooted S4-frame $G$, there exists a p-morphism of a generated subframe of $F$ onto $G$.
-The left-to-right implication follows from the existence of Fine’s frame formulas: there is a formula $\alpha_G$ such that for any K4-frame $H$, $\alpha_G$ is refutable in $H$ if and only if there exists a p-morphism of a generated subframe of $H$ onto $G$. One way of constructing $\alpha_G$ is as follows. Assume that $r$ is a root of $G$, and let $R$ be the accessibility relation of $G$. We put
-$$\alpha_G=\Box^+\biggl(\bigwedge_{\substack{i,j\in G\\\\i\ne j}}(p_i\to\neg p_j)\land\bigwedge_{\substack{i,j\in G\\\\i\mathrel Rj}}(p_i\to\Diamond p_j)\land\bigwedge_{i\in G}\Bigl(p_i\to\Box\bigvee_{i\mathrel Rj}p_j\Bigr)\biggr)\to\neg p_r,$$
-where $\Box^+\phi=\phi\land\Box\phi$ (which is equivalent to $\Box\phi$ in S4; the formula above works for K4 as well). Let $\models$ be a valuation in $H$ such that $x\not\models\alpha_G$ for some $x\in H$. Let $H_x$ be the generated subframe of $H$ rooted at $x$. For every $y\in H_x$, there exists a unique $i\in G$ such that $y\models p_i$; put $f(y)=i$. Then $f\colon H_x\to G$ is a p-morphism such that $f(x)=r$. Conversely, given such a p-morphism, one can construct a valuation refuting $\alpha$ by reversing the process.
-Now, if $G$ is an S4-frame, it is a p-morphic image of itself, hence $\alpha_G$ is not valid in $G$, and a fortiori it is not an S4-tautology. Thus, any $H$ wrt which S4 is complete must also refute $\alpha_G$, hence there exists a p-morphism from a generated subframe of $H$ onto $G$.
-Since every finite rooted S4-model is a p-morphic image of a finite quasi-tree, one can restrict attention to such $G$’s.
-I’m not quite familiar with the topological semantics of S4, but I suppose the criterion translates to something to the effect of: S4 is complete wrt a space $X$ iff for every finite quasi-tree $G$, there exists an open subset $U\subseteq X$ and a p-morphism (whatever that means when the space is not ordered) of $U$ onto $G$.<|endoftext|>
-TITLE: Torsion points of abelian varieties in the perfect closure of a function field
-QUESTION [18 upvotes]: The following is a problem, which was recently brought to my attention by H. Esnault and A. Langer.
-Let $K$ be the function field of a smooth curve over the algebraic closure $k$ of the finite field ${\bf F}_p$ (where $p>0$ is a prime number).
-Let $A$ be an abelian variety over $K$ and suppose that the $K|k$-image of $A$ is trivial (ie there are no non-vanishing $K$-homomorphisms from $A$ to an abelian variety over $K$, which has a model over $k$).
-Question : is it true that $\#{\rm Tor}(A(K^{\rm perf}))<\infty$ ? (*)
-Here $K^{\rm perf}$ is the maximal purely inseparable extension of $K$ and
-${\rm Tor}(A(K^{\rm perf}))$ is the subgroup of $A(K^{\rm perf})$ consisting of
-elements of finite order.
-To put things in context, recall that by the Lang-Néron theorem, we have $\#{\rm Tor}(A(K))<\infty$.
-Furthermore, one can show using a specialization argument that
-$\#{\rm Tor}(A(K^{\rm perf}))<\infty$ if $k$ is replaced by a finite extension of ${\bf F}_p$; in this case, the assumption on the $K|k$-image can actually be dropped.
-Notice also that the inequality in question (*) is actually equivalent to the inequality
-$\#{\rm Tor}_p(A(K^{\rm perf}))<\infty$, where ${\rm Tor}_p(A(K^{\rm perf}))$ is the subgroup of $A(K^{\rm perf})$ consisting of the elements, whose order is a power of $p$.
-This follows from the fact the multiplication by $n$ morphism is étale if $p\not|n$.
-Question (*) has a positive answer if $A$ is an elliptic curve by the work of M. Levin, who
-proves a much stronger result (see "On the group of rational points...", Amer. J. Math. 90 (1968)).
-The question (*) is in part complementary to the following other question in MO :
-Etale endomorphisms of abelian varieties in positive characteristic
-
-REPLY [11 votes]: The answer to question (*) is yes. It is Theorem 1.2.2 in the following preprint.<|endoftext|>
-TITLE: The weak equivalences in the covariant model structure
-QUESTION [14 upvotes]: Let $S$ be a simplicial set. Recall that there is a model structure, called the covariant model structure (see HTT ch. 2 and this question), on $\mathbf{SSet}/S$ such that:
-
-The cofibrations are the monomorphisms.
-A map $X \to Y$ of simplicial sets over $S$ is a weak equivalence if $X^\vartriangleleft \sqcup_X S \to Y^\vartriangleleft \sqcup_Y S$ is a categorical equivalence (i.e., a weak equivalence in the Joyal model structure -- that is, one such that when applying the simplicial category functor $\mathfrak{C}$ gives an equivalence of simplicial categories). Here the triangle denotes the left cone.
-The fibrations are determined; the fibrant objects are the left fibrations $Y \to S$.
-
-I think I understand the motivation for most of this: as Lurie explains, left fibrations are the $\infty$-categorical version of categories cofibered in groupoids (so the fibrant objects model a reasonable concept), and the cofibrations are as nice as can be. But I fail to understand the motivation for the weak equivalences -- not least because I don't have a particularly good picture of what these "left cones" are supposed to model. Why should the weak equivalences be what they are?
-
-REPLY [20 votes]: Maybe it would be helpful to think about the analogous situation in ordinary category theory. Suppose you are given a category $\mathcal{E}$ and a functor $F$ from
-$\mathcal{E}$ to the category of sets. There are several ways to encode this functor:
-$(a)$: Via the Grothendieck construction, $F$ determines a category $\mathcal{C}$ cofibered in sets over $\mathcal{E}$, so that for each object $E \in \mathcal{E}$ you can identify $F(E)$ with the fiber $\mathcal{C}_E$ of the map $\mathcal{C} \rightarrow \mathcal{E}$ over $E$.
-$(b)$: Using the functor $F$, you can construct an enlargement $\mathcal{E}_F$ of the category $\mathcal{E}$, adding a single object $v$ with
-$$Hom(E,v) = \emptyset \quad \quad Hom(v,E) = F(E) \quad \quad Hom(v,v) = \{ id \} $$
-Now suppose we are given another functor $G$ from $\mathcal{E}$ to the category of sets,
-and a natural transformation $F \rightarrow G$. Then $G$ determines a category
-$\mathcal{D}$ cofibered in sets over $\mathcal{E}$, and an enlargement $\mathcal{E}_G$ of $\mathcal{E}$. The natural transformation $F \rightarrow G$ determines functors
-$$ \alpha: \mathcal{C} \rightarrow \mathcal{D} \quad \quad \beta: \mathcal{E}_F \rightarrow \mathcal{E}_G$$
-In this situation, the following conditions are equivalent:
-$(i)$: The natural transformation $F \rightarrow G$ is an isomorphism (that is, for each object $E \in \mathcal{E}$, the induced map $F(E) \rightarrow G(E)$ is bijective.
-$(ii)$: The functor $\alpha$ is an equivalence of categories.
-$(iii)$: The functor $\beta$ is an equivalence of categories.
-Now observe that the category $\mathcal{E}_F$ can be described as the pushout (and also homotopy pushout) of the diagram $$\mathcal{E} \leftarrow \mathcal{C} \rightarrow \mathcal{C}^{\triangleleft},$$
-where $\mathcal{C}^{\triangleleft}$ is the category obtained from $\mathcal{E}$ by adjoining a new initial object.
-Let's now forget the original functors $F$ and $G$, and think only about the categories
-$\mathcal{C}$ and $\mathcal{D}$ cofibered in sets over $\mathcal{E}$. The equivalence of conditions $(ii)$ and $(iii)$ shows that functor $\alpha: \mathcal{C} \rightarrow \mathcal{D}$ of categories cofibered over $\mathcal{E}$ is an equivalence of categories if and only if the induced map
-$$ \mathcal{E} \amalg_{ \mathcal{C} } \mathcal{C}^{\triangleleft}
-\rightarrow \mathcal{E} \amalg_{ \mathcal{D} } \mathcal{D}^{\triangleleft}$$
-is an equivalence of categories.
-Now go to the setting of quasi-categories. Assume for simplicity that $S$ is a quasi-category, and let $f: X \rightarrow Y$ be a map of simplicial sets over $S$. If
-$X$ and $Y$ are left-fibered over $S$, then we would like to say that $f$ is a covariant equivalence if and only if it an equivalence of quasi-categories. However, we would like to formulate this condition in a way that will behave well also when $X$ and $Y$ are not fibrant.
-Motivated by the discussion above, we declare that $f$ is a covariant equivalence if and only if it induces a categorical equivalence
-$$ S \amalg_{X} X^{\triangleleft} \rightarrow S \amalg_{Y} Y^{\triangleleft}.$$
-You can then prove that this is a good definition (it gives you a model structure with the cofibrations and fibrant objects that you described, and when $X$ and $Y$ are fibrant a map
-$f: X \rightarrow Y$ is a covariant equivalence if and only if it induces a homotopy equivalence of fibers $X_s \rightarrow Y_s$ for each vertex $s \in S$).<|endoftext|>
-TITLE: Canonical liftings of endomorphisms of ordinary abelian varieties
-QUESTION [7 upvotes]: I am looking for a reference to the following ``well known" fact.
-Let $k$ be a perfect field of prime characteristic $p$ and $W(k)$ its ring of Witt vectors. Let $A_0$ be an ordinary abelian variety over $k$ and let $A$ be an abelian scheme over $W(k)$ that is the canonical (Serre--Tate) lifting of $A_0$. Then every endomorphism $u_0$ of $A_0$ lifts to an endomorphism of $A$. In other words, the natural map $End(A) \to End(A_0)$ is bijective.
-My problem is that I need it for infinite $k$. (I know a couple of references that deal with finite $k$.)
-
-REPLY [6 votes]: The canonical reference is Messing, LNM 264 1972, Chapter V, 3.4, p174.<|endoftext|>
-TITLE: Is there a name for this map induced by bilinear forms?
-QUESTION [6 upvotes]: Let $V$ be a real vector space. A bilinear form $\langle \rangle:V\times V\to {\mathbb{R}}$ induces a linear functional $\theta$ on the tensor product $V\otimes V$ given by sending the finite sum $\sum_i v_i\otimes w_i $ to $\sum_i \langle v_i,w_i\rangle$.
-Is there a name for this induced linear functional?
-In addition, if the bilinear form is symmetric, then this linear functional $\theta$ respects the natural involution on $V\otimes V$. That is $\theta(v\otimes w)=\theta(w\otimes v)$.
-
-REPLY [2 votes]: A bilinear form on $V$ (if non-degenerate) lets you identify $V$ with $V^\star$:
-$v \mapsto \langle v, \cdot \rangle$
-In this case, your map is just a contraction of the identity map on $V^\star \otimes V$ (which, considering our identification, is the same as the identity on $V \otimes V$).
-http://en.wikipedia.org/wiki/Tensor_contraction<|endoftext|>
-TITLE: $Sq^1$ cohomology of spaces
-QUESTION [12 upvotes]: For any space $X$, the first Steenrod square cohomology operation
-$$Sq^1\colon H^\ast(X;\mathbb{Z}_2)\to H^{\ast +1}(X;\mathbb{Z}_2)$$
-is a derivation, meaning that $Sq^1\circ Sq^1 = 0$ and $Sq^1(a\cup b) = Sq^1(a)\cup b + a\cup Sq^1(b)$ (there are no signs since we are working in characteristic two).
-Hence we may form the $Sq^1$-cohomology of the space,
-$$H\left(H^\ast(X;\mathbb{Z}_2),Sq^1\right)$$
-which will be a graded algebra over $\mathbb{Z}_2$.
-I am looking for references on this object. From McCleary's "User's guide to spectral sequences", I know that this is related to the Bockstein spectral sequence. More specifically, I would like to know:
-
-
-What is the precise relationship between the $Sq^1$-cohomology of a space $X$ and $2$-torsion of higher order in $H^\ast(X;\mathbb{Z})$?
-Is there a reference with specific calculations of the $Sq^1$-cohomology of the Eilenberg-Mac Lane spaces $K(\mathbb{Z}_2,n)$?
-Are there any canonical references I should know about (besides McCleary and Mosher-Tangora)?
-
-REPLY [9 votes]: Several people have addressed question 1 (Torsten Ekedahl and Neil Strickland). Question 2 is interesting, but I don't have a good answer for it. For question 3, as Sean Tilson points out, this is a special case of "Margolis homology", a.k.a. $P^s_t$-homology. Try
-
-Adams and Margolis, "Modules over the Steenrod algebra", Topology 10 (1971)
-Anderson and Davis, "A vanishing theorem in homological algebra", Comment. Math. Helv. 48 (1973)
-Margolis, Spectra and the Steenrod algebra (1983)
-
-I also wonder if there is anything helpful in
-
-Adams and Priddy, "Uniqueness of BSO".
-
-You might also search for the phrase "Bockstein acyclic", since $\textrm{Sq}^1$ is the mod 2 Bockstein.<|endoftext|>
-TITLE: Optimal 8-vertex isoperimetric polyhedron?
-QUESTION [16 upvotes]: I know from Marcel Berger's
-
-Geometry Revealed:
-A Jacob's Ladder to Modern Higher Geometry
-(p.531)
-that it is not yet established which polyhedron in $\mathbb{R}^3$ on 8 vertices achieves the optimal isoperimetric ratio $A^3/V^2$, where $A$ is the surface area and $V$ the volume.
-Berger says "We also know that the cube ... [is] not the best for $v=8$" (where $v$ is the number of vertices).
-Many other aspects of isoperimetry for polyhedra are unresolved, but this one especially interests me. It is not even clear to me that it is known that there is an optimal polyhedron for each $v$.
-I've been trying to imagine what would be a strong candidate for an optimal 8-vertex polyhedron. I've been unsuccessful in finding information on this, although it seems likely to have been explored computationally. Does anyone have a candidate, or know of one proposed/calculated? A pointer or reference would be greatly appreciated. Thanks!
-Addendum.
-From the reference Igor provided (Nobuaki Mutoh, "The Polyhedra of Maximal Volume Inscribed in the Unit Sphere and of Minimal Volume Circumscribed about the Unit Sphere," 2009), here is a piece of Mutoh's Fig.1, which computationally verifies the earlier derivation of the max volume inscribed 8-vertex polyhedron by Berman and Haynes ("Volumes of polyhedra inscribed in the unit sphere in $\mathbb{R}^3$," Math. Ann., 188(1): 78-84, 1970, doi: 10.1007/BF01435416, eudml), as mentioned in the comments:
-
-
-
-
-This is surely a candidate for achieving the min of $A^3/V^2$!
-I thank Jean-Marc, Igor, and Anton for the rapid convergence to what I sought.
-...And then a bit later to Henry for showing that this candidate does not in fact achieve the best ratio!
-Here is Henry's polyhedron, if I have interpreted him correctly:
-
-REPLY [17 votes]: An $8$-vertex polyhedron can achieve an isoperimetric ratio of $A^3/V^2 = 159.3243297053\dots$, and based on some quick experiments I'm pretty confident this is optimal (although I wouldn't be shocked if it could be beaten).
-To construct it, let $V_\alpha$ denote the squashed tetrahedron with vertices $(\pm \sqrt{1-\alpha^2},0,\alpha)$ and $(0,\pm \sqrt{1-\alpha^2},-\alpha)$. Then the optimal $8$-vertex polyhedron seems to be the union of $V_\alpha$ and $-\beta V_\gamma$, with $\alpha = 0.2272117725\dots$, $\beta = 0.87345300464\dots$, and $\gamma = 0.83792301859\dots$. The optimal values of $\alpha$, $\beta$, and $\gamma$ are algebraic, but they're pretty complicated and I haven't computed their minimal polynomials.
-For comparison, the maximum volume polyhedron inscribed in a sphere has a worse isoperimetric ratio, namely $162.248792\dots$. For the cube, it's $216$.
-In general there's no reason to expect the optimal polyhedron to be inscribed in a sphere. The $5$-vertex case is a particularly nice example: it consists of an equilateral triangle on the equator of the unit sphere together with $1/\sqrt{2}$ times the north and south poles. This achieves an isoperimetric ratio of $243$, and I'd be very surprised if that's not optimal. Five vertices is few enough that a rigorous proof may be possible, but I can't think of a non-painful way to do it.<|endoftext|>
-TITLE: Axiom of choice and non-measurable set
-QUESTION [8 upvotes]: We know that existence of a Lebesgue non-measurable set follows from the Axiom Of Choice. Is the converse true? That is, does the existence of a Lebesgue non-measurable set imply the Axiom Of Choice?
-
-REPLY [20 votes]: No, the existence of a non-Lebesgue measurable set does not imply the axiom of choice. If ZF is consistent, then set-theorists can construct models of ZF having a non-Lebesgue measurable set, but still not satisfying AC.
-This is quite reasonable, because the existence of a non-Lebesgue measurable set is a very local assertion, having to do only with sets of reals, and thus can be satisfied with a small example, by set-theoretic standards. The axiom of choice, in contrast, is a global assertion insisting that every set, even a very large set, has a well-order. So we don't expect to turn a mere non-measurable set into well-orderings of enormous sets, such as the power set $P(\mathbb{R})$.
-And indeed, one can use forcing to produce a model $L(P(\mathbb{R})^{V[G]})$ which satisfies $ZF+\neg AC$, for similar reasons as in the usual $\neg AC$ models, but since it has the true $P(\mathbb{R})$, it will have all the same non-Lebesgue measurable sets as in the ambient ZFC universe $V[G]$.
-(Finally, let me make a minor objection to the question: consistency is a symmetric relation, and so if $A$ is consistent with $B$, then $B$ would be consistent with $A$, and so one wouldn't ordinarily speak of a "converse". You seem instead to be refering to the implication that AC implies there is a non-measurable set, and this is how I took your question.)<|endoftext|>
-TITLE: On a remark in Foundations of mechanics, 2nd Edition, by Abraham and Marsden
-QUESTION [5 upvotes]: I don't know if this question is appropriate to this site, but I posted here without an answer, so I tried this alternative.
-Given a $2$-form $\omega$ on a manifold $M$, let us denote by $N$ the kernel of $\omega$, i.e. $N:=\{u\in TM : \omega(u,\cdot)=0\}$. Their Proposition 5.1.2 shows that if $\omega$ has constant rank (and is closed) then $N$ is a tangent distribution on $M$ (and completely integrable).
-In the following remark they say that ``the reader can easily prove the converse of the previous conclusion''. While I understand that $N$ is a tangent distribution if and only if $\omega$ has constant rank. Instead I think that, for $\omega$ of constant rank, $N$ can be completely integrable even if $\omega$ is not closed, (e.g. $\omega=e^z dx\wedge dy$).
-Starting from this consideration I have asked myself a question:
-Given a $\Omega\in\mathcal{A}^p(M)$, with $p>1$, whose rank is constant, let us define its kernel $N$ as above. Evidently $N$ is a tangent distribution on $M$, and I find it is completely integrable at least when there exists a $1$-form $\phi$ such that $d\Omega=\phi\wedge\Omega$. Clearly, if $\Omega$ is decomposable then the last condition is even necessary.
-My question (edited after the comment of Willie Wong):
-
-Is this last condition (the ``divisibility'' of $d\Omega$ by $\Omega$) necessary for the complete integrability of $N$ even when $\Omega$ is not decomposable? (Using Frobenius' Theorem I understand the case $p=1$, but what about the case $p>1$?.)
-
-Any suggestion and\or counterexample are welcome.
-
-REPLY [5 votes]: The answer is 'no'. To see why, just take any nondegenerate $2$-form $\omega$ on, say, $\mathbb{R}^6$, that has the property that $d\omega$ is not a multiple of $\omega$. (This will be true for a generic such $2$-form.) The kernel of this $\omega$ is trivial, but now, you can just regard it as being defined on $\mathbb{R}^8$, say, by pullback. Then the kernel has (constant) positive dimension and is obviously integrable, but $d\omega$ is still not a multiple of $\omega$.
-By the way, the theorem about integrability is true in much greater generality than for a single $p$-form: If $\mathcal{I}\subset\Omega^*(M)$ is a graded ideal that is closed under exterior derivative and its `kernel' $N$, which is defined as the set of vectors $v\in T_xM$ such that $\iota(v)\phi$ is in $\mathcal{I}_x$ for all $\phi\in \mathcal{I}_x$, has constant rank, then $N$ is integrable. This is a classic theorem of Cartan on 'Cauchy characteristics', and can, for example, be seen in Exterior Differential Systems by Bryant, et al.<|endoftext|>
-TITLE: On a weak choice principle
-QUESTION [9 upvotes]: [PLEASE SEE EDITS AT BOTTOM OF QUESTION]
-Consider the following set-theoretic axiom:
-
-For each set $X$ there exists a set-indexed collection $\{C_i \to X\}_{i\in I_X}$ of surjections such that for every surjection $Z\to X$ there is a map $C_i\to Z$ for some $i$ such that the obvious triangle commutes.
-
-This is known as WISC (Weakly Initial Set of Covers), and can be interpreted as saying Choice fails to hold in at most a 'small' way. It is clearly implied by AC, and I'm willing to bet that it is independent of other usual set-theoretic axioms (ZF, say). WISC is implied by COSHEP (take $I_X$ to be a singleton for all $X$), SVC and AMC.
-My questions are these:
-
-Does anyone know of a weaker choice principle? (Edit: a global choice principle, or at least one for a sizable collection of sets, like all elements $\bigcup_n \mathcal{P}(\mathbb{R}^n)$)
-
-and
-
-In which popular/common models of set theory does WISC hold? That is, aside from the ones listed at the linked page above (which are particularly category theory-oriented).
-
-(As a bonus question: Come up with a model of ZF that violates WISC or prove we can use forcing to construct one)
-
-Edit: There is also the axiom WISC${}_\kappa$, where we require the set $I_X$ to be bounded by some cardinal $\kappa$ (either less than or at most). This is perhaps more interesting than the unbounded case, especially in topological applications.
-
-Edit2: Benno van den Berg has now shown that Gitik's model of ZF (Israel J. Math 1980) violates WISC. This model, which relies on the consistency of a large cardinal assumption (that is, the existence of an unbounded collection of strongly compact cardinals), has the property that only $\aleph_0$ is a regular cardinal. What Benno showed was that ZF+WISC implies the existence of an unbounded collection of regular cardinals. Now one can clearly ask (thanks to godelian in the comments) whether weaker large cardinal assumptions suffice. One would only need to find a model of ZF in which there is only a bounded collection of regular cardinals. This to me sounds reasonable.
-
-REPLY [3 votes]: After Benno van den Berg's proof, there are now two more proofs:
-First, building on his answer, Asaf removed the requirements for large cardinals in
-
-Asaf Karagila, Embedding Orders Into Cardinals With $DC_\kappa$, Fund. Math. 226 (2014), 143-156, doi:10.4064/fm226-2-4, arXiv:1212.4396.
-
-Second, as suggested by Mike Shulman, a suitable topos of $G$-sets where $G$ is a large topological group violates WISC in its internal language (how to make this construction precise takes a little bit of ingenuity):
-
-David Michael Roberts, The weak choice principle WISC may fail in the category of sets, Studia Logica Volume 103 (2015) Issue 5, pp 1005-1017, doi:10.1007/s11225-015-9603-6, arXiv:1311.3074.<|endoftext|>
-TITLE: Homotopy Fixed Points of SO(2) on Fully Dualizable Algebras
-QUESTION [14 upvotes]: Note: by fixed points, I always mean homotopy fixed points.
-As explained in Jacob Lurie's paper on the cobordism hypothesis, we have an action of O(2) on the $\infty $-groupoid $X$ given by considering fully dualizable objects and invertable morphisms in some symmetric monoidal $(\infty ,2)$ category $\mathcal C$. I am interested in the case when $\mathcal C$ is the category where objects are algebras, 1-cells are bimodules, 2-cells maps of bimodules etc... By an algebra, I want to include algebra objects in some $\infty$-category (e.g. chain complexes), so that $\mathcal C$ really has non-trivial 3-cells, 4-cells and so on.
-Now, I know (from remark 4.2.7) that the fixed points for the induced action of SO(2) on this space correspond to cyclic Frobenius algebras, i.e. smooth, proper algebras $A$, with a non-degenerate and $SO(2)$-equivariant trace
-$A \otimes _{A^e} A \to k$.
-Here, non-degenerate means that the pairing $A\otimes A \to A\otimes _{A^e} A \to k$ is nondegenerate. Recall also that the Hochschild homology $A \otimes _{A^e} A$ carries and $SO(2)$ action, so it makes sense to talk about $SO(2)$-equivariant map above.
-The data of SO(2) equivariance should be equivalent to descending the trace to cyclic homology.
-However, the reason why the fixed points are as above is that both can be identified with 2-d oriented TFTs, after theorem 3.1.8 which describes how to extend from a (n-1)-dimensional TFT to an n-dimensional one.
-Question: Can we calculate this action and identify the fixed point space directly?
-Part of this is not too hard to see (I think): naively, the SO(2) action on $X$ gives a canonical (Morita) automorphism of each (f.d.) algebra $A$. This automorphism is given by the bimodule dual $A^!$ (or the other one $A^\vee$...), so being a fixed point means to give an isomorphism $A^! \to A$, which is the same as a non-degenerate map $A\otimes _{A^e} A \to k$. If the base category where our algebra lives has no higher structure, then I think this is enough, but this does not explain the $SO(2)$ equivariance.
-In particular, where does the SO(2)-equivariance data come from?
-To see this, I tried to go a little deeper (with help from Takuo Matsuoka): the action is given by a map $B\mathbb Z =SO(2) \to Aut(X)$, which we transformed into a map $\mathbb Z \to \Omega Aut(X)$. The data described above corresponds to picking an algebra $A$ and composing to get a map $\mathbb Z \to \Omega X$ = Invertable $A-A$-bimodules. However, we haven't used that the original map from SO(2) is a group homomorphism - this should correspond to the map $\mathbb Z \to \Omega Aut(X)$ being an $E_2$-map. So we get braiding data attached to each of the $A^!$, which should be trivialized (because the E_2 -structure on $\mathbb Z$ is trivial (?)). Then to give a fixed point we must take into account all this data...
-This has only given me a headache so far, but maybe there is an easier way to think about this? I tried (failed?) to be brief, so let me know if anything here is unclear!
-
-REPLY [10 votes]: I might be confused about your question. Are you asking...
-
-How is trivializing the $O(n)$-action the same as giving an $O(n)$-equivariant non-degenerate trace? (as per Lurie's theorem 3.1.8).
-How can we identify the $SO(2)$-action with the usual $SO(2)$-action on Hochschild homology?
-
-For the first one, I think this is pretty well described in Jacob's paper. The key is understanding what the condition "non-degenerate" means in that setting. This is the condition that allows you to turn the equivariant trace into the trivialization of an action.
-If it is the second thing you are asking, here is one way to see this, though there may be an easier way. Let's first discuss the more direct approach which is giving you a headache. Instead of thinking about $E_2$-structures or (homtopical) group homomorphisms, I prefer to think of the map of delooings:
-$$ \alpha: BSO(2) = K(\mathbb{Z}, 2) = \mathbb{CP}^\infty \to BAut(X)$$
-This is equivalent to understanding the $E_2$-structure you mentioned, but for me it is easier to comprehend. A (homotopical) $ SO(2)$-action on $X$ is the same as the map $\alpha$.
-I find this easier probably because of my background. Now $BSO(2) = \mathbb{CP}^\infty$ has a well known cell structure which goes:
-$$ S^2 \cup e_4 \cup e_6 \cup \dots $$
-So part of the $SO(2)$-action is an element in $\pi_2 BAut(X)$ which gives the invertible $A-A$-bimodule you identified up above. This is also called the Serre bimodule in this context. The element in $\pi_2$ actually contains more information: it is a natural transformation (from the identity functor to itself).
-From this point of view, to understand the rest of the $SO(2)$-action, you need to extend this map all the way through the entire CW-structure. You can start doing this explicitly. For example the fact that the Serre is a natural transformation allows you to construct an element in $\pi_3 BAut(X)$ which is essentially the "self braiding" of the Serre with itself. Trivializing this element allows you to extend the map to the 4-skeleton $\mathbb{CP}^2 = S^2 \cup e_4$. This then gives rise to a higher "self braiding" which you then have to trivialize, etc.
-This quickly becomes a huge headache, as each of thee trivializations is supposed to be describable in terms of the basic fully-dualizable structure. While possible, it is not entirely obvious how to procede at each step. Also, even if we succeed this doesn't yet do what you want; we still haven't identified this SO(2)-action with the usual one on Hochschild homology.
-A different approach is to realize that
-$$BSO(2) \simeq |N \Lambda^{op}|$$
- where $\Lambda$ is the cyclic category. This allows us to get a different "filtration" of $K(\mathbb{Z}, 2)$. If we view the map $\alpha$ as classifying an $X$-bundle over $BSO(2)$, then we can use this description, plus the material of Dwyer-Hopkins-Kan "The homotopy theory of cyclic sets" to realize that this is equivalent to the data of a certain cyclic set.
-I think that in the universal case we get a cyclic set which essentially comes from the various ways to decompose a circle cyclicly, and in the specific case at hand we recover the Hochschild homology complex. This would identify the two circle actions. I hope this helps.<|endoftext|>
-TITLE: Multilinear generalization of Cauchy-Schwarz inequality
-QUESTION [9 upvotes]: Let $V$ be a real vector space, and let $(\cdot,\cdot;\cdot,\cdot) : V^4 \to \mathbb{R}$ be a multilinear form with the following properties:
-
-$(x,y;z,w) = (y,x;z,w) = (x,y;w,z)$ (symmetry in the first and second pairs)
-$(x,x;z,z) \ge 0$ (positive semidefiniteness in the first and second pairs).
-
-
-Must such a form satisfy the inequality $$|(x,y;z,w)| \le \sqrt{(x,x;z,z)(y,y;w,w)}?$$
-
-The prototype I have in mind is something like $V = C_c^\infty(\mathbb{R}^n)$, with
-$$(f,g;h,k) = \int f g \nabla h \cdot \nabla k$$
-in which case the inequality follows by using Cauchy-Schwarz twice (first in $\mathbb{R}^n$, and then in $L^2(\mathbb{R}^n)$).
-I'd settle for the inequality
-$$|(x,y;z,w)| \le C({\epsilon}(x,x;z,z) + \epsilon^{-1}(y,y;w,w))$$
-which follows from the above by AM-GM (with $C = 1/2$). I'd also settle for the special case $x=w, y=z$ where it reads $|(x,z;x,z)| \le \sqrt{(x,x;z,z)(z,z;x,x)}$.
-Simply using Cauchy-Schwarz in each pair gives the inequality
-$$|(x,y;z,w)| \le [(x,x;z,z)(x,x;w,w)(y,y;z,z)(y,y;w,w)]^{1/4}$$ which has cross terms that I don't want. Edit: Of course, as Willie Wong points out and zeb's counterexample confirms, this doesn't work.
-Thanks!
-
-REPLY [9 votes]: Even the inequality $(x,z;x,z)^2 \le (x,x;z,z)(z,z;x,x)$ is false:
-Let $V = \mathbb{R}^2$, with basis $x,z$. Take $(x,x;x,x) = 100$, $(x,z;x,x)=0$, $(z,z;x,x)=1$, $(x,x;x,z)=0$, $(x,z;x,z)=50$, $(z,z;x,z)=0$, $(x,x;z,z) = 1$, $(x,z;z,z)=0$, $(z,z;z,z)=100$, and extend to all of $V^4$ by symmetry and multilinearity.
-To check that positive semi-definiteness holds, note that we just need to check that
-$(x+az,x+az;x+bz,x+bz) = 100 + a^2 + 200ab + b^2 + 100a^2b^2 \ge 0$,
-which easily follows from AM-GM.
-Now note that we have $2500 = (x,z;x,z)^2 > (x,x;z,z)(z,z;x,x) = 1$.
-In fact, we even have $6250000 = (x,z;x,z)^4 > (x,x;x,x)(x,x;z,z)(z,z;x,x)(z,z;z,z) = 10000$.
-Edit: On the other hand, we can prove the following inequality:
-$4(x,y;z,w)^2 \le ((x,x;z,z)+(x,x;w,w))((y,y;z,z)+(y,y;w,w))$.
-To see this, note that by positive semi-definiteness we have
-$0 \le (x+ay,x+ay;z+w,z+w) + (x-ay,x-ay;z-w,z-w)$
-$ = 2((x,x;z,z)+(x,x;w,w)) + 8a(x,y;z,w) + 2a^2((y,y;z,z)+(y,y;w,w))$
-for any $a$, and plugging in $a = -\frac{2(x,y;z,w)}{(y,y;z,z)+(y,y;w,w)}$ we get the desired inequality.<|endoftext|>
-TITLE: Book on mixed Hodge structures?
-QUESTION [7 upvotes]: Is there any English textbook about Deligne's mixed Hodge structures? Can you tell me about a reference where they are introduced at least for smooth quasi-projective varieties?
-
-REPLY [7 votes]: Mixed Hodge Structures, Peters and Steenbrink
-
-REPLY [7 votes]: Maybe you already know of the notes by Benoît Audoubert and Orsola Tommasi entitled "Mixed Hodge Structures,"
-notes on Audoubert's seminars on Mixed Hodge structures at the University of Nijmegen in 2002?
-These notes certainly discuss quasi-projective algebraic varieties (in Section 3).
-It is about 50 pages long, with the following (high-level) Table of Contents:
-
-Hodge structures
-Varieties with normal crossings
-Smooth quasi-projective varieties
-The theory of Deligne<|endoftext|>
-TITLE: Are wild problems related to undecidable ones?
-QUESTION [14 upvotes]: In representation theory, there is a well-known notion of a wild classification problem (such problems have been discussed often on this forum, for example, here). In logic, there is a notion of an undecidable problem.
-
-Is there a theorem which says that there is something undecidable about a wild classification problem?
-
-A reference where such issues are discussed would be very helpful.
-
-REPLY [16 votes]: Yes, there is a connection, but I think it is conjectural in its full generality. The mosst general reference could be, where it is proven, that for a subclass of wild algebras, the representation theory is undecidable:
-Mike Prest: Wild representation type and undecidability, Comm. Alg. 19 (3), 1991.
-It is also well-known (it is stated with references for example in Benson), that the representation theory of the algebra used to define wildness (i.e. $k\langle X,Y\rangle$) is undecidable.<|endoftext|>
-TITLE: Wanted: example of a non-algebraic singularity
-QUESTION [31 upvotes]: Given a finitely generated $\def\CC{\mathbb C}\CC$-algebra $R$ and a $\CC$-point (maximal ideal) $p\in Spec(R)$, I define the singularity type of $p\in Spec(R)$ to be the isomorphism class of the completed local ring $\hat R_p$, as a $\CC$-algebra.
-Do there exist non-algebraic singularity types? That is, does there exist a complete local ring with residue field $\CC$ which is formally finitely generated (i.e. has a surjection from some $\CC[[x_1,\dots, x_n]]$), but is not the complete local ring of a finitely generated $\CC$-algebra at a maximal ideal?
-Googling for "non-algebraic singularity" suggests that the answer is yes, but I can't find a specific example. I would expect that it should be possible to write down a power series in two variables $f(x,y)$ so that $\CC[[x,y]]/f(x,y)$ is non-algebraic.
-
-What is a specific formally finitely generated non-algebraic singularity?
-
-REPLY [15 votes]: The main question of the PI has been beautifully answered by Moret-Bailly, but not the
-secondary question arisen from his expectation: "I would expect that it should be possible to write down a power series in two variables f(x,y) so that ℂ[[x,y]]/f(x,y) is non-algebraic."
-though we got quite close in comments.
-So for the record: this is not possible. Indeed, such a singularity would be analytic by a result of Michael Artin ( "On the solutions of analytic equations", Invent. Math. 5 1968, 277–291, cf. Angelo's answer to Analytic vs. formal vs. étale singularities) and then algebraic by Ulrich's comment (that is by Corollary 7.7.3 of the book by Casas-Alvero "Singularities of plane curves", London Mathematical Society Lecture Note Series, 278).
-This is of course consistent with the fact that the example quoted by Moret-Bailly is in
-three variables.<|endoftext|>
-TITLE: Intersection of a smooth projective variety and a plane
-QUESTION [5 upvotes]: Let $X \subset P^n$ be an irreducible smooth complex projective
-variety embedded in the $n$-dimensional projective space.
-Let $k$ be the dimension of $X$ and $d$ its degree.
-Let $L \subset P^n$ be a linear subspace of dimension $n-k$
-and $Z=L \cap X$. Assume that
-(a) $X$ is not contained in any hyperplane of $P^n$ and
-(b) $Z$ is finite of cardinality $d$.
-Question: Is it true that $Z$ spans $L$?
-Comment: I was told that this is true if $X$ is ACM
-(arithmetically Cohen-Macaulay). A reference for this
-would be appreciated.
-
-REPLY [8 votes]: It is true that $Z$ spans $L$ — even if $X$ isn't ACM. You can also allow $X$ to be singular (but you do need $X$ irreducible and non-degenerate, of course). To illustrate one of the main ideas it is useful to first look at the case when $X$
-is a curve.
-If $X$ is a curve. Let $M$ be the span of $Z$ and suppose that $M\neq L$. (In the curve case, $L$ will be a hyperplane). Let $p$ be any point of $X$ outside of $Z$ and let $H$ be any hyperplane containing $M$ and $p$. Then $H\cap X$ contains at least $d+1$ points, so by Bezout's theorem the intersection cannot be zero dimensional. Since $X$ is irreducible and one dimensional, this means that the intersection must be all of $X$, so $X$ is contained in $H$, contrary to hypothesis.
-The general case.
-The idea when $k\geqslant 2$ is to show that if $H$ is a general hyperplane containing $L$ then $H \cap X$ is irreducible and non-degenerate (i.e, the intersection $H\cap X$ is not contained in a smaller linear space of $H$). But now all dimensions have been reduced by $1$, and so iterating this procedure reduces us to the curve case, which we've already solved.
-To set this up, note that hyperplanes in $\mathbb{P}^n$ containing $L$ are parameterized by a $\mathbb{P}^{k-1}$ (If $V$ is the underlying vector space of $\mathbb{P}^{n}$, $W$ the underlying vector space of $L$, then the hyperplanes are parameterized by the projectivization of $(V/W)^{*}$). We'll use $H$ to refer both to a point of $\mathbb{P}^{k-1}$ and the corresponding hyperplane in $\mathbb{P}^n$ containing $L$. Define $\Gamma\subset \mathbb{P}^{k-1}\times (X\setminus Z)$ to be the set
-$$\Gamma = \left\{(H,p) \mid p\in H\right\}$$
-i.e, the pairs $(H,p)$ so that $H$ is a hyperplane containing $L$, and $p$ a point of $H\cap X$ not on $Z$.
-If we fix $p$, then the set of possible $H$'s satisfying this condition are simply the hyperplanes $H$ containing the span of $L$ and $p$, and this is parameterized by a $\mathbb{P}^{k-2}$. In other words, $\Gamma$ is a $\mathbb{P}^{k-2}$ bundle over $X\setminus Z$. (This fibration is where we use $k\geqslant 2$.) Since $X\setminus Z$ is irreducible this implies that $\Gamma$ is irreducible.
-Let $\overline{\Gamma}$ be the Zariski-closure of $\Gamma$ in $\mathbb{P}^{k-1}\times X$. Then $\overline{\Gamma}$ is irreducible since $\Gamma$ is. For a fixed $H\in \mathbb{P}^{k-1}$ the fibre of the projection $\overline{\Gamma}\longrightarrow \mathbb{P}^{k-1}$ over $H$ is simply the intersection $X\cap H$, of dimension $k-1$.
-Now let $q$ be any point of $Z$. Then $q\in X\cap H$ for every $H\in \mathbb{P}^{k-1}$ so $q$ gives a section of $\overline{\Gamma}\longrightarrow\mathbb{P}^{k-1}$. Since $Z$ consists of $d$ distinct points where $d$ is the degree of $X$ we conclude that $q$ is a smooth point of $X$. Finally, since $Z$ is the intersection of all $X\cap H$ for $H\in \mathbb{P}^{k-1}$ this implies that the general intersection $X\cap H$ is smooth at $q$. Summarizing, we have a section of the map which generically lies in the smooth locus of the fibres. Since $\overline{\Gamma}$ is irreducible, this implies that the generic fibre is irreducible, i.e, if $H$ is a generic hyperplane containing $L$, then $H\cap X$ is irreducible.
-(The intuitive reason for this implication is that, generically over $\mathbb{P}^{k-1}$ the section lets us pick out precisely one irreducible component of the fibre. The union of these components gives us a subset of $\overline{\Gamma}$ which has the same dimension as $\overline{\Gamma}$, and hence whose closure must be all of $\overline{\Gamma}$ by irreducibility. But if there is more than one component in a general fibre, this is a contradiction, thus the general fibre must be irreducible. To make this intuitive construction rigorous requires passing to the normalization of $\overline{\Gamma}$ and then looking at the Stein factorization of the map from the normalization to $\mathbb{P}^{k-1}$. The section gives a generic section of the finite part of the Stein factorization, and that allows one to construct the ``union of the components containing the section''.)
-Finally, the same trick as in the curve case also shows us that for any hyperplane $H$, $H\cap X$ must be non-degenerate. Let $Y=H\cap X$, so that $Y$ is a variety of degree $d$ and dimension $k-1$. Let $M$ be the span of $Y$. If $M\neq H$ then pick any point $p\in X\setminus Y$ and let $H'$ be any hyperplane containing $M$ and $p$. Then $H'\cap X$ can't be all of $X$ (since this would contradict the non-degeneracy of $X$), so $Y'=H'\cap X$ must be a subvariety of dimension $k-1$ (more precisely, all components of $Y'$ have dimension $k-1$) and degree $d$. But $Y$ is therefore a component of $Y'$, and the equality of degrees tells us that $Y'$ can't have any other components so we must have $Y'=Y$. This contradicts the fact that $p\in Y'$ and $p\not\in Y$.
-Together this shows the required inductive step: If $H$ is a general hyperplane containing $L$ then $H\cap X$ is irreducible and non-degenerate.
-Other remarks. I'm guessing from the setup of the question that you want to apply the result for a particular $L$ that you have chosen. If, in the application, you're allowed to pick a general $L$ then you can say something stronger. The classical uniform position principle (where ''classical'' in this case means ''established by Joe Harris in the 80's'') states that for a general subspace $L$ of dimension $n-k$ the finite set of $d$-points in $Z=L\cap X$ have the property that any subset of $r+1$ of the points (with $r\leqslant n-k$) span a $\mathbb{P}^{r}$. Picking $r=n-k$, this means that any subset of $n-k+1$ points of $Z$ spans all of $L$, and so in particular $Z$ spans $L$. (Note that $d\geqslant n-k+1$; for instance, as a consequence of the argument above: if $d < n-k+1$ then the $d$ points of $Z$ would never span $L$.)<|endoftext|>
-TITLE: A combinatorial proof for the property of KM numbers?
-QUESTION [8 upvotes]: Kendell-Mann numbers $M(n)$ ( see the sequence A000140 http://oeis.org/A000140 ) have the simple property: $M(n+1) \approx (n-1/2)M(n)$.
-The property can be proved by different methods.
-For eg. The property of Kendall-Mann numbers
-What I am looking for is to find out if a combinatorial proof exists?
-For eg. Let us start: Suppose we look at all the permutations of $n-1$ in the maximal grouping, then at all the permutation of $n$ in that maximal grouping; is there any simple way in which each permutation in the first set gives rise to $n$ permutations in the second? Better yet, a simple way in which about half the $n-1$-permutations give rise to $n$ $n$-permutations each, and the other half give rise to $n+1$ $n$-permutations each?
-Any hints are higly welcomed.
-I hope that the combinatorial proof will makes the reason for the simple property more transparent.
-
-REPLY [8 votes]: Here is a quick and dirty probabilistic analysis which gets the right answer. For a permutation $w \in S_n$, define
-$$I(w) = \sum_{1 \leq i < j \leq n} \begin{cases} -1 & w(i) < w(j) \\ 1 & w(i) >w(j) \\ \end{cases}.$$
-So $I(w) = 2 \# (\mbox{number of inversions of $w$}) - \binom{n}{2}$. If $w$ is chosen uniformly at random, then the expected value of $I(w)$ is $0$.
-Now, let's think about the expected value of $I(w)^2$. Squaring the sum, we get terms indexed by $(i_1, j_1, i_2, j_2)$ with $i_1